Você está na página 1de 212

Superintendência do Sistema Estudual de

SEAS Atendimento Socioeducativo do Estado do Ceará

SOCIOEDUCADOR
Teoria esquemazada, dicas e questões de provas UECE.

Na dúvida, escolha
DINCE e surpreenda-se.

LÍNGUA PORTUGUESA
Prof. Wezeck Nogueira

DIREITOS HUMANOS
Prof. Valdeci Cunha

LEGISLAÇÃO ESPECIAL
Vanques de Melo
ATENÇÃO:
Possíveis alterações deste
material serão colocadas até a
véspera da prova no site
www.editoradince.com

Contém:
Teoria esquematizada
Dicas e macete
Questões de provas anteriores organizadas por assunto (inclusive de 2021) e
outras bancas

2021
Copyright 2021 – DIN.CE Edções Ténicas.
Todos os direitos reservados e protegidos pela Lei nº 9.610/98.
Proibida a reprodução total ou parcial, por qualquer meio, sem autorização expressa e por escrito dos
autores e da editora.

SEAS – Socioeducador – 210p


Capa: Editoração Gráfica:
Gonga / Carlos Kelton
Diagramação: Acabamento:
Emanoel Antônio / Jailson
Ilustração: Revisão:
Emanoel Autores
Impressão: Supervisão editorial:
Gráfica DIN.CE Vanques de Melo
_______________________
NOTA DA EDITORA: LEIA COM ATENÇÃO!
As informações e opiniões apresentadas nesta apostila são de inteira
responsabilidade dos autores e/ou organizadores das respectivas matérias.
A Editora DIN.CE se responsabiliza apenas pelos vícios do produto no que se refere
à sua confecção, considerando a impressão e apresentação. Vícios de atualização,
revisão ou opiniões são de responsabilidade do(s) autor(res) ou organizador(res),
respondendo este(s) pelas sanções previstas na lei.
Esclarece, ainda, que o conteúdo apresentado nesta apostila tem como objetivo
auxiliar ao(a) candidato(a) na preparação ao cargo almejado. Todo o seu conteúdo é
abordado de forma objetiva e resumido, o que nem sempre é suficiente para lograr o
êxito almejado, qual seja, a aprovação. Sendo assim, sugere-se ao(a) candidato(a) que,
na medida do possível, busque outras fontes de consulta para que possa se preparar
melhor.
No mais, em decorrência do tempo para a realização da prova, possíveis erros de
digitação ou de atualizações podem ocorrer. Isto se dá em decorrência da impossibilidade
de uma revisão mais criteriosa, visto que tal serviço demandaria uma média de até 60
dias para ser concluído e muitas vezes, da publicação do edital até a realização das
provas, o prazo é menos de 60 dias.
Assim, no caso de possiveis correções e/ou atualizações, a editora DINCE
disponibilizará em seu site www.editoradince.com, na aba “ATUALIZAÇÕES”, até à
véspera da prova as devidas erratas. Portanto, É MUITO IMPORTANTE que você se
adicione ao nosso WhatsApp (85) 9.8632.4802 para ficar informado a respeito das
novidades relativas ao concurso e ao material respectivo. Pelo que sugerimos que na
véspera da prova der uma olhada no site pra ver se tem alguma informação.
No que diz respeito algumas disciplinas constarem exclusivamente no SITE, isso
se justifica tendo em vista trata-se de conteúdos bastante amplo e coloca-las na apostila
no formato impresso tornaria bastante volumoso o material, além de encarecê-lo bastante.
Ressalta-se, ainda que, em decorrência da grande incidência de
gráficas/copiadoras estarem reproduzindo nossos materiais de forma ilegal,
esclarecemos que não nos responsabilizamos por cópias adquiradas nessas empresas e
que, possíveis ressarcimentos só poderam ser efetivados mediante comprovação de que
o material foi adquido de forma legal.
Por fim, para exclarecimentos de eventuais dúvidas, disponibilizamos, na página
seguinte, nossos meios de comunicação.

Av. 2, 644 – Itaperi - Fortaleza - Ceará


PREZADO(A) CONCURSANDO(A),

Você está adquirindo um produto elaborado por professores que atuam em cursinhos
preparatórios nas respectivas áreas, portanto, confiável e de procedência. Todavia, por se
tratar de apostila, é um material resumindo, porém, de significativa importância. No entanto,
sugerimos, como forma de melhor preparo, a leitura de outras fontes tais como livros específicos,
resumos e exercícios. Nosso objetivo é prepara-lo(a) para a carreira tão
almejada.
Possíveis falhas de impressão ou mesmo de digitação podem ocorrer, assim, caso seja
constatado ou mesmo tenha dúvida em algum conteúdo ou gabarito, entre em contato pelo o e-
mail dos professores ou diretamente pelo da editora din.ce@hotmail.com que lhe
responderemos imediatamente.
Também dispomos do site www.editoradince.com onde serão disponibilizadas possíveis
correções ou atualizações deste material.
No mais, desejamos boa sorte e que Deus lhe abençoe.
SAC – Serviço de Atendimento ao Concurseiro:
Fale conosco: din.ce@hotmail.com
Ou ainda: (85) 3231.6298 / 9.8632.4802 (WhatsApp)

ALGUNS DE NOSSOS PROFESSORES:


Prof. Augusto Sá Prof. Anderson Viana
Prof. Augusto César Prof. Janilson Santos
Prof. Ávila Prof. Joanilson Jr.
Prof. Brando Prof. Nestor Dia.
Prof. Deivis Cavalcante Prof. Valdeci Cunha
Profa. Flaviana Eufrazio. Prof. Walber Siqueira

Tens tu fé? Tem-na em ti mesmo diante de Deus. Bem-aventurado aquele que não
se condena a si mesmo naquilo que aprova. (Romanos 14)
Bem aventura o homem que não anda segundo o caminho dos ímpios, nem se
detém no caminho dos pecadores, nem se assenta na roda dos escarnecedores.
Antes tem o seu prazer na lei do SENHOR, e na sua lei medita de dia e de noite.
(Salmo, 1.2 e 2)
Por isso vos digo que todas as coisas que pedires, orando, crede receber e tê-las-ei
(Marcos, 11.24)

O temor do Senhor é o princípio do conhecimento; mas os insensatos


desprezam a sabedoria e a instrução. (Provérbios 1:7)

Deleita-te também no SENHOR, e te concederá os desejos


do teu coração.
(Salmos 37:4)
OUTRAS PUBLICAÇÕES DA EDITORA DINCE

ADQUIRA AGORA: (85) 3231.6298 / 9.8632.4802 (WhatsApp)


LÍNGUA PORTUGUESA - TEORIA E QUESTÕES 1
Ô ô ô ô Boi Fubá.
LÍNGUA PORTUGUESA
Teoria e questões de provas 01) (Concurso Milagres/2018) Sobre a composição
acima podemos dizer:
organizadas assunto. I – O texto fala de uma realidade existente em algumas
Prof. Wezeck Nogueira regiões do país em que, por necessidade de
sobrevivência e para escapar de situações sociais,
políticas e geográficas (...) adversas, surge à
necessidade de êxodo rural;
II – Pelas informações contidas no texto, podemos
afirmar que essa narrativa acontece na zona rural;
III - Vaca Estrela e Boi Fubá são animas que compõem
o criatório das fazendas, alimentam seus donos com
CONTEÚDO PROGRAMÁTICO: leite e derivados, podem também servir de alimento
Compreensão e interpretação de texto ............................ 3 depois de abatidos;
Ortografia ........................................................................ 11 IV – O refrão do texto é o aboio saudoso do sertanejo
Acentuação gráfica ......................................................... 11 que, desgarrado de suas raízes e de toda a carga
Sílaba (separação e classificação) .................................... 9 afetiva que estas representam, não consegue
Encontros vocálicos e consonantais ................................ 9 desvencilhar- se daquilo que ficou para trás.
Classes Gramaticais (substantivo, adjetivo, artigo, A) I, II e III estão corretas;
numeral, pronome, verbo, advérbio, preposição, crase, B) I, II e IV estão corretas;
conjunção, interjeição) .......................................... 16/38
C) II, III e IV estão corretas;
Formação das palavras ................................................... 23
Sintaxe (frase, oração, tipos de sujeito, predicado, objeto D) I e IV estão corretas;
direto e indireto)......................................................... 24 E) I, III e IV estão corretas.
Concordância nominal e verbal ....................................... 32
Regência nominal e verbal. ............................................. 34 02) (Concurso Milagres/2018) Para caracterizar a
Questões de provas UECE ........................................... 46 personagem enunciadora do texto, as palavras
aparecem com muitos desvios da língua que se
aprende na escola e que é a oficialmente correta.
Marque a opção em que as palavras estão escritas
QUESTÕES DE PROVAS corretamente:
AGENTE ADMINISTRATIVO - PREF. A) Mendigo, mortadela, cuscus, iogurte;
MILAGRES/CE/2018 B) Mendigo, mortandela, cuscuz, iorgute;
TEXTO: C) Mendingo, mortadela, cuscus, iogurte;
Vaca Estrela E Boi Fubá D) Mendigo, mortadela, cuscuz, iogurte;
Patativa do Assaré E) Mendingo, mortandela, cuscuz, iorgurte.
Seu doutó me dê licença pra minha história
contar. 03) (Concurso Milagres/2018) Dado o verso: Aquela
Hoje eu tô na terra estranha e é bem triste o meu penar seca medonha fez tudo se trapaiar,. O termo em
destaque é um pronome:
Mas já fui muito feliz vivendo no meu lugar. Eu tinha
cavalo bão, gostava de campear. E todo dia aboiava A) Relativo;
na porteira do currá. B) Indefinido;
Ê ê ê ê la a a a a ê ê ê ê Vaca Estrela, Ô ô ô ô Boi Fubá. C) Possessivo;
Eu sou fio do Nordeste, não nego meu naturá Mas uma D) Interrogativo;
seca medonha me tangeu de lá pra cá E) Demonstrativo.
Lá eu tinha o meu gadinho, num é bom nem imaginar,
Minha linda Vaca Estrela e o meu belo Boi Fubá 04) (Concurso Milagres/2018) Como já nos referimos
Quando era de tardezinha eu começava a aboiar anteriormente, o desvio da norma culta é um recurso
Ê ê ê ê la a a a a ê ê ê ê Vaca Estrela, Ô ô ô ô Boi Fubá. estilístico utilizado para compor a carga semântica
do texto e atribuir fidelidade. No verso: As água
Aquela seca medonha fez tudo se trapaiar, Não nasceu corre dos óio, começo logo a chorá, o artigo no
capim no campo para o gado sustentar plural e o (Substantivo) termo por ele determinado no
O sertão esturricou, fez os açude secar Morreu minha singular causa um estranhamento nas regras de
Vaca Estrela, se acabou meu Boi Fubá concordância nominal. Sobre a concordância
Perdi tudo quanto tinha, nunca mais pude aboiar marque a opção INCORRETA:
Ê ê ê ê la a a a a ê ê ê ê Vaca Estrela, Ô ô ô ô Boi Fubá. A) A loja oferece atendimento e localização perfeitas;
Hoje nas terra do sul, longe do torrão natá Quando eu B) A loja oferece atendimento e localização perfeitos;
vejo em minha frente uma boiada passar, C) A loja oferece localização e atendimento perfeitos;
As água corre dos óio, começo logo a chorá Lembro a D) A loja oferece localização e atendimento perfeito;
minha Vaca Estrela e o meu lindo Boi Fubá E) A loja oferece atendimento e localização perfeita;
Com saudade do Nordeste, dá vontade de aboiar
Ê ê ê ê la a a a a ê ê ê ê Vaca Estrela, 05) (Concurso Milagres/2018) As água corre dos óio,

www.editoradince.com Acesse e veja se há novidades a respeito deste material


2 LÍNGUA PORTUGUESA - TEORIA E QUESTÕES
começo logo a chorá. A grafia da palavra se parte do tempo. Mas elas são anjos depois do sexo-amor.
apresenta como um problema aos que necessitam É nessa hora que elas se sentem o próprio amor
se expressar na escrita. A utilização do ch ou x é um encarnado e voltam a ser anjos. E levitam. Algumas até
desses “nós”. Marque a opção em que todas as voam. Mas os homens não sabem disso. E nem
palavras estão grafadas corretamente: poderiam... Porque são tomados por um encantamento
A) Faxina, mecha, enxame, mexerica, cachimbo; que os faz dormir nessa hora.
B) Fachina, mecha, enxame, mecherica, cachimbo; Luis Fernando Sabino
C) Faxina, mexa, enchame, mexerica, caximbo; 01. Sobre o texto é correto afirmar:
D) Fachina, mexa, enxame, mecherica, cachimbo; a) Pode ser considerado como uma crônica
E) Faxina, mecha, enchame, mexerica, caximbo; comentário, uma vez que predomina a linguagem
conotativa;
b) É uma crônica narrativa, pois apresenta
Gabarito: 01/X; 02/D; 03/E; 04/A; 05/A
personagens ficcionais;
c) É uma crônica filosófica em que se trabalha o texto
FISCAL DE TRIBUTOS - PREF. JUAZEIRO DO como pretexto para uma análise da vida humana;
NORTE/CE / CEV/URCA
d) Pode ser considerado como uma crônica
O sexto sentido das mulheres argumentativa, pois fundamenta a opinião do autor
Certo dia, parei para observar as mulheres e só dentro de uma tensão dramática.
pude concluir uma coisa: elas não são humanas. São
espiãs. Espiãs de Deus, disfarçadas entre nós. Pare para 02. São características da crônica, exceto:
refletir sobre o sexto sentido. Alguém duvida que ele
exista? a) Registra o circunstancial, o episódio, o efêmero.
E, como explicar que ela saiba exatamente qual b) Deve ser escrita numa linguagem leve, comunicativa,
mulher, entre as presentes, em uma reunião, seja aquela informativa.
que dá em cima de você... Ou é a que você dá em cima? c) Desenvolve-se através de períodos curtos e
E, quando ela antecipa que alguém tem algo contra você, linguagem enxuta.
que alguém está ficando doente ou que você quer
d) Apresenta tensão dramática em torno de personagens
terminar o relacionamento? E, quando ela diz que vai
ficcionais.
fazer frio e manda você levar um casaco? Rio de Janeiro,
40ºC, você vai pegar um avião para São Paulo. Só meia-
hora de vôo. Ela fala para você levar um casaco, porque 03. Observe o fragmento:
"vai fazer frio". Você não leva. O que acontece? O avião “Certo dia, parei para observar as mulheres e só pude
fica preso no tráfego, em terra, por quase duas horas, concluir uma coisa: elas não são humanas. São
depois que você já entrou, antes de decolar. O ar espiãs.”
condicionado chega a pingar gelo de tanto frio que faz lá Sobre os termos, em destaque, podemos afirmar:
dentro!
I- Representam elementos coesivos na estrutura textual.
O sexto sentido não faz sentido! É a
comunicação direta com Deus! Assim é muito fácil...As II- São, respectivamente: uma catáfora – elemento
mulheres são mães! E preparam, literalmente, gente predissente; uma anáfora – elemento remessivo e
dentro de si. Será que Deus confiaria tamanha uma elipse.
responsabilidade a um reles mortal? E não satisfeitas em III- Representam: uma hiperônimo – termo substitutivo;
ensinar a vida, elas insistem em ensinar a vivê-la, de um hipônimo – termo predissente; uma anáfora –
forma íntegra, oferecendo amor incondicional e termo predissente.
disponibilidade integral. Fala-se em "praga de mãe", Estão corretas as assertivas:
"amor de mãe", "coração de mãe"... Tudo isso é meio
mágico... Talvez ele tenha instalado o dispositivo "coração a) I, II e III
de mãe" nos "anjos da guarda" de seus filhos (que, aliás, b) I e II, somente
foram criados à sua imagem e semelhança). c) II e III, somente
As mulheres choram, vazam ou extravasam?
d) I e III, somente
Homens também choram, mas é um choro diferente. As
lágrimas das mulheres têm um não sei quê que não quer
chorar, um não sei quê de fragilidade, um não sei quê de 04. Veja o fragmento:
amor, um não sei quê de tempero divino, que tem um “E preparam, literalmente, gente dentro de si.”
efeito devastador sobre os homens... É choro feminino; é O uso da vírgula justifica-se pela mesma regra em:
choro de mulher... Já viram como as mulheres conversam
com os olhos? Elas conseguem pedir uma à outra para a) Ele não consegue, por exemplo, dirigir sozinho.
mudar de assunto com apenas um olhar. Elas fazem um b) Comprou sapato, e meias, e bolsa.
comentário sarcástico com outro olhar. E apontam uma c) O homem, que é mortal, retorna ao pó.
terceira pessoa com outro olhar. Quantos tipos de olhar
existem? Elas conhecem todos... Parece que freqüentam d) Creio, afirmou Antônio, que este é um caso perdido.
escolas diferentes das que freqüentam os homens! E é
com um desses milhões de olhares que elas enfeitiçam os 05. Em: “E é com um desses milhões de olhares que elas
homens. En-fei-ti-çam! enfeitiçam os homens.” Temos:
O amor leva as mulheres para perto de Deus... já a) Oxímoro
que ele é o próprio amor. Por isso, dizem "estar nas
nuvens", quando apaixonadas. Com todo esse amor de
b) Eufemismo
mãe, esposa e amiga, elas ainda são mulheres a maior c) Hipálage

www.editoradince.com Acesse e veja se há novidades a respeito deste material


LÍNGUA PORTUGUESA - TEORIA E QUESTÕES 3
d) Hipérbole feita de maneira cautelosa por ser o primeiro contato com
o novo texto. Desta leitura, extraem-se informações sobre
o conteúdo abordado e prepara-se o próximo nível de
06. Marque a alternativa em que o uso da colocação
leitura. Durante a interpretação propriamente dita, cabe
pronominal corresponda à norma culta.
destacar palavras-chave, passagens importantes, bem
a) Enquanto se espera o vôo, bebe-se algo. como usar uma palavra para resumir a ideia central de
b) Enquanto se espera o vôo, se bebe algo. cada parágrafo. Este tipo de procedimento aguça a
memória visual, favorecendo o entendimento.
c) Enquanto espera-se o vôo, bebe-se algo.
Não se pode desconsiderar que, embora a
d) Enquanto espera-se o vôo, se bebe algo. interpretação seja subjetiva, há limites. A preocupação
deve ser a captação da essência do texto, a fim de
07. A crônica de Fernando Sabino foi de encômios às responder às interpretações a banca considerou como
mulheres. O termo em destaque, significa: pertinentes.
a) Elogios No caso de textos literários, é preciso conhecer a
ligação daquele texto com outras formas de cultura,
b) Críticas
outros Não se pode desconsiderar que, embora a
c) Comentários interpretação seja subjetiva, há limites. A preocupação
d) Sarcasmos deve ser a captação da essência do texto, a fim de
responder às interpretações a banca considerou como
pertinentes.
08. “E é desses milhões de olhares que elas enfeitiçam os
homens. En-fei-ti-çam!” A alternativa em que a No caso de textos literários, é preciso conhecer a
separação silábica está correta, é: ligação daquele texto com outras formas de cultura,
outros textos e manifestações de arte da época em que o
a) gra- tu- i- to autor viveu. Se não houver esta visão global dos
b) ca- mio- nei- ro momentos literários e dos escritores, a interpretação pode
c) nu- clei- co ficar comprometida. Aqui não se podem dispensar as
dicas que aparecem na referência bibliográfica da fonte e
d) i- tu- iu- ta- ba na identificação do autor.
A última fase da interpretação concentra-se nas
09. “As lágrimas das mulheres têm um não sei quê que perguntas e opções de resposta. Aqui são fundamentais
não quer chorar.” O termo em destaque é: marcações de palavras como não, exceto, errada,
a) Pronome adjetivo indefinido. respectivamente etc que fazem diferença na escolha
adequada. Muitas vezes, em interpretação, trabalha-se
b) Advérbio de intensidade.
com o conceito do "mais adequado", isto é, o que
c) Pronome relativo. responde melhor ao questionamento proposto. Por isso,
d) Pronome substantivo indefinido. uma resposta pode estar certa para responder à pergunta,
mas não ser a adotada como gabarito pela banca
examinadora por haver uma outra alternativa mais
10. “Talvez ele tenha instalado o dispositivo “coração de completa. Ainda cabe ressaltar que algumas questões
mãe” (...) (foram criados à sua imagem e apresentam um fragmento do texto transcrito para ser a
semelhança)” base de análise. Nunca deixe de retornar ao texto, mesmo
O uso do acento grave está correto, exceto em: que aparentemente pareça ser perda de tempo. A
a) Vendo à vista. descontextualização de palavras ou frases, certas vezes,
são também um recurso para instaurar a dúvida do
b) Usava bigode à Hitler. candidato. Leia a frase anterior e a posterior para ter a
c) O homem foi ferido à bala. ideia do sentido global proposto pelo autor, desta maneira
d) Ficaram à contemplar tamanha beleza. a resposta será mais consciente e segura.
Temos, tranquilamente, ser bem-sucedidos numa
interpretação de texto. Para isso, devemos observar o
Gabarito:
seguinte:
1 - Ler todo o texto, procurando ter uma visão geral
do assunto;
2 - Se encontrar palavras desconhecidas, não
interrompa a leitura, vá até o fim,
ininterruptamente;
3 - Ler, ler bem, ler profundamente, ou seja, ler o
INTERPRETAÇÃO DE TEXTO; texto pelo menos umas três vezes ou mais;
ARGUMENTAÇÃO; PRESSUPOSTOS E 4 - Ler com perspicácia, sutileza, malícia nas
entrelinhas;
SUBENTENDIDOS E NÍVEIS DE LINGUAGEM
5 - Voltar ao texto quantas vezes precisar;
Os concursos apresentam questões interpretativas
6 - Não permitir que prevaleçam suas ideias sobre
que têm por finalidade a identificação de um leitor
as do autor;
autônomo. Portanto, o candidato deve compreender os
níveis estruturais da língua por meio da lógica, além de 7 - Partir o texto em pedaços (parágrafos, partes)
necessitar de um bom léxico internalizado. para melhor compreensão;
Para ler e entender bem um texto basicamente 8 - Centralizar cada questão ao pedaço (parágrafo,
deve-se alcançar os dois níveis de leitura: a informativa e partes) do texto correspondente;
de reconhecimento e a interpretativa. A primeira deve ser

www.editoradince.com Acesse e veja se há novidades a respeito deste material


4 LÍNGUA PORTUGUESA - TEORIA E QUESTÕES
9 - Verificar, com atenção e cuidado, o enunciado modo geral, cada argumentação em defesa da tese geral
de cada questão; do texto corresponde a um parágrafo.
10 - Cuidado com os vocábulos: destoa (= Conclusão: a parte final do texto em que
diferente de...), não, correta, incorreta, certa, retomamos a tese central, agora já respaldada pelos
errada, falsa, verdadeira, exceto, e outras; argumentos desenvolvidos ao longo do texto.
palavras que aparecem nas perguntas e que,
às vezes, dificultam a entender o que se
RELAÇÃO ENTRE TESE E ARGUMENTO
perguntou e o que se pediu;
De modo geral, a relação entre tese e argumento
11 - Quando duas alternativas lhe parecem
pode ser compreendida de duas maneiras principais:
corretas, procurar a mais exata ou a mais
completa; Argumento, portanto, Tese (A→ pt→T) ou Tese
porque Argumento (T→ pq→A):
12 - Quando o autor apenas sugerir ideia, procurar
um fundamento de lógica objetiva; (A→ pt→T)
13 - Cuidado com as questões voltadas para “O governo gasta, todos os anos, bilhões de reais
dados superficiais; no tratamento das mais diversas doenças relacionadas ao
tabagismo; os ganhos com os impostos nem de longe
14 - Não se deve procurar a verdade exata dentro
compensam o dinheiro gasto com essas doenças. Além
daquela resposta, mas a opção que melhor se
disso (Ainda, e, também, relação de adição → quando se
enquadre no sentido do texto;
enumeram argumentos a favor de sua tese), as empresas
15 - Às vezes a etimologia ou a semelhança das têm grandes prejuízos por causa de afastamentos de
palavras denuncia a resposta; trabalhadores devido aos males causados pelo fumo.
16 - Procure estabelecer quais foram as opiniões Portanto (logo, por conseguinte, por isso, então →
expostas pelo autor, definido o tema e a observem a relação semântica de conclusão, típica de um
mensagem; silogismo), é mister que sejam proibidas quaisquer
17 - O autor defende ideias e você deve percebê- propagandas de cigarros em todos os meios de
las; comunicação.”
18 - Os adjuntos adverbiais e os predicativos do (T→ pq→A)
sujeito são importantíssimos na interpretação O governo deve imediatamente proibir toda e
do texto. qualquer forma de propaganda de cigarro, porque (uma
Ex.: Ele morreu de fome. vez que, já que, dado que, pois → relação de
causalidade) ele gasta, todos os anos, bilhões de reais no
de fome: adjunto adverbial de causa, determina a
tratamento das mais diversas doenças relacionadas ao
causa na realização do fato ( = morte de "ele" ).
tabagismo; e, muito embora (ainda que, não obstante,
Ex.: ele morreu faminto mesmo que → relação de oposição: usam-se as
faminto: predicativo do sujeito, é o estado em que concessivas para refutar o argumento oposto) os ganhos
"ele" se encontrava quando morreu. com os impostos sejam vultosos, nem de longe eles
compensam o dinheiro gasto com essas doenças.
O QUE É COBRADO NAS PROVAS
Pressupostos e subentendidos
Os concursos apresentam questões interpretativas
que têm por finalidade a identificação de um leitor São informações implícitas num texto, não
autônomo. expressas formalmente, apenas sugeridas por marcas
linguísticas ou pelo contexto. Cabe ao leitor, numa leitura
Esse tipo de texto, que é aplicado nas redações de proficiente, ir além da informação que se encontra
concursos públicos e até mesmo o Enem, inclui diferentes explícita, identificando e compreendendo as informações
gêneros, tais quais, dissertação, artigo de opinião, carta implícitas, ou seja, lendo nas entrelinhas.
argumentativa, editorial, resenha argumentativa, dentre
outros. Os pressupostos são de mais fácil identificação,
estando sugeridos no texto. Os subentendidos são
Todo e qualquer texto argumentativo, como já dito, deduzidos pelo leitor, sendo da sua responsabilidade.
visa ao convencimento de seu ouvinte/leitor. Por isso, ele
sempre se baseia em uma tese, ou seja, o ponto de vista Exemplos:
central que se pretende veicular e a respeito do qual se - Heloísa está cansada de ser professora.
pretende convencer esse interlocutor. Nos gêneros Pressuposto: Heloísa é professora.
argumentativos escritos, sobretudo, convém que essa Subentendido: Talvez porque o salário é baixo ou
tese seja apresentada, de maneira clara, logo de início e há muita indisciplina.
que, depois, através duma argumentação objetiva e de
- Infelizmente, meu marido continua trabalhando
diversidade lexical seja sustentada/defendida, com vistas
fora do país
ao mencionado convencimento.
Pressuposto: O marido está trabalhando fora do
A estrutura geral de um texto argumentativo
país e a mulher não está satisfeita com essa
consiste de introdução, desenvolvimento e conclusão,
situação.
nesta ordem. Cada uma dessas partes, por sua vez tem
função distinta dentro da composição do texto: Subentendido: Talvez por ter melhor salário fora do
país ou por não encontrar trabalho no seu país.
Introdução: é a parte do texto argumentativo em
que apresentamos o assunto de que trataremos e a tese a
ser desenvolvida a respeito desse assunto. Pressupostos
Desenvolvimento: é a argumentação Os pressupostos são informações implícitas
propriamente dita, correspondendo aos desdobramentos adicionais, facilmente compreendidas devido a palavras
da tese apresentada. Esse é o coração do texto, por isso, ou expressões presentes na frase que permitem ao leitor
comumente se desdobra em mais de um parágrafo. De compreender essa informação implícita. O enunciado

www.editoradince.com Acesse e veja se há novidades a respeito deste material


LÍNGUA PORTUGUESA - TEORIA E QUESTÕES 5
depende dessa pressuposição para que faça sentido. gramaticalmente corretos, mas que são de entendimento
Assim, o pressuposto é verdadeiro e irrefutável. para o receptor.
Exemplos de pressupostos: Um grande exemplo é a linguagem regional, que
- Decidi deixar de comer carne. usa de elementos selecionados para determinadas
situações. Veja o exemplo da fala caipira abaixo:
Pressuposto: A pessoa comia carne antes.
“Aos 18 anos, pai Norato deu uma facada num
rapaz, num adjutório, e abriu o pé no mundo. Nunca mais
- Finalmente acabei minha monografia. ninguém botou os olhos em riba dele, afora o afilhado.
Pressuposto: Demorou algum tempo para terminar — Padrinho, eu vim cá chamá o sinhô pra mode i
a monografia. morá mais eu.
— Quá, flo, esse caco de gente num sai daqui
- Alunos que estudam de manhã costumam ter mais não.
melhor rendimento. — Bamo. Buli gente num bole, mais bicho… O
Pressuposto: Há alunos que não estudam de sinhô anda perrengado…”
manhã. (Bernardo Élis, Pai Norato)
Conseguiu compreender bem de que se tratam os
- Desde que ela mudou de casa, nunca mais a vi. níveis de linguagem? A ideia principal não é ter a
Pressuposto: Costumava vê-la antes dela mudar gramática correta, mas ter um contexto compreensível
de casa. entre os falantes. Vamos conhecer os tipos a seguir:
Marcas linguísticas que facilitam a identificação de
pressupostos: Linguagem regional
Verbos que indicam fim, continuidade, mudança e Refere-se aos falares locais, variações na fala que
implicações: começar, continuar, parar, deixar, acabar, ocorrem de acordo com o local geográfico onde os
conseguir,... falantes estão ou são naturais. O Brasil é um país com
Advérbios: felizmente, finalmente, ainda, já, extensa faixa territorial e com muitos falares regionais
depois, antes,... distintos. Algumas das falas características mais
Pronome introdutório de orações subordinadas conhecidas são a nordestina, a fluminense, a mineira, a
adjetivas: que sulista e a baiana.
Locuções que indicam circunstâncias: depois que,
antes que, desde que, visto que,... Linguagem popular
Subentendidos É também chamada de linguagem coloquial. Trata-
se daquela que é usada de forma espontânea e fluente
pelas pessoas. Raramente segue as regras da gramática
Subentendidos normativa e é carregada de vícios de linguagem, tais
São insinuações, informações escondidas, como pleonasmo, cacofonia, barbarismo e solecismo.
dependentes da interpretação do leitor. Não possuem Gírias e expressões vulgares costumam aparecer com
marca linguística, sendo deduzidos através do contexto frequência neste tipo de linguagem. Ela está presente nas
comunicacional e do conhecimento que os destinatários conversas entre amigos, familiares e também em
têm do mundo. Podem ser ou não verdadeiros e podem programas de televisão.
ser facilmente negados, visto serem unicamente da
responsabilidade de quem interpreta a frase.
Gíria
Exemplos de subentendidos:
A gíria é um estilo linguístico que está integrado à
- Quando sair de casa, não se esqueça de levar linguagem popular. Ela está relacionada ao cotidiano de
um casaco. certos grupos sociais, em especial os estudantes, os
Subentendido: Está frio lá fora. esportistas e também os ladrões, por exemplo. Utilizada
como meio de expressão cotidiana, a gíria existe dentro
- Já tenho a garganta seca de tanto falar. de grupos para diferenciá-los, pois só os inseridos
naquele determinado contexto sabem o seu significado.
Subentendidos: Quero beber um copo de água ou
Bem, pelo menos foi assim que ela começou e ainda é
quero parar de falar neste momento.
usada em alguns grupos atualmente. Entretanto, a fala se
encarrega de espalhar a gíria e seus significados e,
- Você vai a pé para casa agora? muitos grupos, ainda que não tenham ligação entre si,
Subentendidos: Eu posso lhe dar uma carona ou é podem falar a mesma gíria.
perigoso andar a pé na rua a estas horas.
Linguagem vulgar
NÍVEIS DE LINGUAGEM Indo de encontro com a linguagem culta, a vulgar é
A fala e a escrita, em uma determinada situação extremamente fora do padrão gramatical e faz parte da
de comunicação, apresentam os ditos “níveis de fala dos analfabetos ou semianalfabetos. As estruturas
linguagem”. Esses dizem respeito à concordância em que gramaticais são “bagunçadas” e barbarismos são
o emissor e o receptor estão para que possam ser frequentes. A exemplo temos os vícios “Nóis vai”, “vamo
compreendidos, e para tanto, existem linguagens ir”, “pra mim comer”.
diferentes para ocasiões distintas.
A Gramática Normativa dita as regras de Linguagem culta
coerência, porém, na fala e escrita, especialmente O total oposto da linguagem tratada anteriormente.
informal, podemos usar elementos que não estão A língua padrão é aquela ensinada nas escolas, usada

www.editoradince.com Acesse e veja se há novidades a respeito deste material


6 LÍNGUA PORTUGUESA - TEORIA E QUESTÕES
em livros didáticos e, muitas vezes, é a usada nos defendendo então um determinado ponto de
telejornais. É mais comum usar esse tipo de linguagem na vista.
escrita. Ela reflete prestígio social e cultural, mas não é só  texto dissertativo expositivo: exposição de
isso que deve definir um indivíduo. uma ideia usando conceitos, comparações,
definições, descrições e informações.
GÊNEROS E TIPOS TEXTUAIS  texto descritivo: relato e exposição de
acontecimento, lugar ou pessoa sendo, assim,
A comunicação é o processo que envolve troca de rico em adjetivos que possam transmitir
informações entre interlocutores usando signos e regras imagens.
semióticas para se completar. A ação básica de transmitir
e receber outra mensagem de volta configura um  texto narrativo: apresentações ações em
processo social primário possibilitado pela linguagem. determinado tempo e espaço. Sua estrutura é
composta por apresentação, desenvolvimento,
Por meio da linguagem, é possível interagir com
clímax e desfecho.
outros indivíduos, bem como alterar o discurso conforme
o contexto. Observe que, ao longo do dia, é possível que
estejamos envolvidos em diferentes tipos de situações, O QUE SÃO OS GÊNEROS LITERÁRIOS?
cada uma exigindo um comportamento linguístico Os gêneros literários compõem a classificação
adequado. que abrange somente os textos literários. AO
O resultado é o surgimento dos tipos e gêneros agrupamento é feito de acordo com a forma,
textuais. São situações nas quais o falante ou escritor características formais comuns nas obras literárias,
baseia a construção de determinado discurso visando critérios estruturais, semânticos e contextuais. Sendo
atender, de forma efetiva, o contexto no qual está assim, podemos encontrar gêneros literários dramático,
inserido. Vejamos, a seguir, o que são tipos e gêneros lírico, épico, narrativo, entre outros.
textuais, suas diferenças e relações, além de como isso
pode cair na sua prova. O que são os gêneros textuais?
Os gêneros textuais são estruturas que se
DIFERENÇA ENTRE GÊNEROS E TIPOS TEXTUAIS posicionam em funções sociais específicas e, por isso,
Os tipos textuais configuram-se como modelos ocorrem vieram da necessidade de interação e
fixos e abrangentes que objetivam a distinção e definição comunicação do ser humano, além das variações
da estrutura, bem como aspectos linguísticos de narração, apresentadas pelos diferentes contextos de discurso. O
dissertação, descrição e explicação. Os tipos textuais uso dos gêneros é ativado sempre que um falante é
apresentam estrutura definida e número limitado de inserido em algum tipo de situação comunicativa.
possibilidades, de cinco a nove tipos. E como isso acontece? Um determinado gênero
Os gêneros textuais, por sua vez, apresentam textual, mesmo que de forma inconsciente, é selecionado
maior diversidade e exercem funções sociais específicas. pelo falante de forma a atender às necessidades daquela
Ademais, são passíveis de modificações ao longo do situação. A intenção é transmitir a mensagem ao
tempo, mesmo que preservando características interlocutor de forma efetiva, o que acontece desde as
preponderantes. Quer um exemplo prático? A carta! Até mensagens de WhatsApp até ofícios organizacionais.
pouco tempo, era um dos principais meios de Tal variabilidade, de certa forma, impede a
comunicação escrita à distância. enumeração fixa dos tipos de gênero textual. No entanto,
Mas, com o advento da tecnologia, acabou certas peculiaridades comuns facilitam seu agrupamento,
perdendo espaço para o e-mail. No entanto, certos como a característica inerente de tipos estáveis de
elementos linguísticos foram preservados, como enunciados, estruturas e conteúdos temáticos que
remetente, destinatário, saudações e cumprimentos finais. facilitam sua definição.
É interessante pontuar que os aspectos gerais dos tipos Podemos citar alguns gêneros textuais e, inclusive,
são concretizados em situações de comunicação dos separá-los dentro de determinadas categorias de tipos
gêneros textuais. textuais:

O QUE SÃO OS TIPOS TEXTUAIS GÊNEROS TEXTUAIS EM TEXTOS NARRATIVOS:


Os tipos textuais, ou tipologias textuais, são a  Conto
forma sob a qual o texto se apresenta determinando a  Biografia
estrutura padrão que rege como cada um será construído.
A tipologia textual é classificada de acordo com objetivo,  Fábula
estrutura e finalidade do texto.  Romance
Podemos classificar os tipos textuais nas seguintes  Lenda
categorias:  Novela
 texto explicativo prescritivo: instrui o receptor  Carta
com relação a determinado procedimento sem
permitir, assim, sua liberdade de ação.
Gêneros textuais em textos expositivos:
 texto explicativo injuntivo: também conhecido  Resumo
como texto instrucional, busca orientar o
receptor com o uso de verbos no imperativo no  Resenha
sentido de persuadi-lo.  Reportagem
 texto dissertativo argumentativo: exposição de  Notícia
tema ou assunto com o uso de argumentos  Monografia
 Verbete de dicionário

www.editoradince.com Acesse e veja se há novidades a respeito deste material


LÍNGUA PORTUGUESA - TEORIA E QUESTÕES 7
 Cardápio (Marcelo Gleiser, Retalhos cósmicos. S.Paulo:
Gêneros textuais em textos descritivos: Companhia das Letras, 1999, pp. 75-77)
 Diário
 Relato de viagem 1. TRE-RN – Analista Judiciário – Análise de Sistemas
– Julho/2005) Conforme demonstram as
Gêneros textuais em textos injuntivos:
afirmações entre parênteses, o autor confere em
 Receita seu texto estas duas acepções distintas ao termo
 Manual de instruções indiferença, relacionado à Natureza:
 Regras de jogo (A) crueldade (indiferente a tanta dor e sofrimento) e
 Bula generosidade (o que chamaríamos de comportamento
 Lista de compras
decente).
Gêneros textuais em textos argumentativos: (B) hipocrisia (por trás dessa ações assassinas se
esconde um motivo simples) e inflexibilidade
 Artigo de opinião (predadores não sentem a menor culpa).
 Crônica (C) impiedade (indiferente a tanta dor e sofrimento) e
 Editorial alheamento (não tem nada a dizer sobre compaixão
 Carta (se endereçada à sociedade, como uma ou ética de comportamento).
carta aberta) (D) isenção (isentamos o resto do mundo animal desse
tipo de comportamento) e pretexto (para propagar seu
DNA).
QUESTÕES DE CONCURSOS
(E) insensibilidade (sua sobrevivência e a da sua espécie
A indiferença da natureza dependem dessa atividade) e determinação
Eu me lembro do choque e da irritação que (indiferente a tanta dor e sofrimento).
sentia, quando criança, ao assistir a documentários
sobre a violência do mundo animal; batalhas mortais
2. (TRE-RN – Analista Judiciário – Análise de
entre escorpiões e aranhas, centenas de formigas
Sistemas /2005) Considere as afirmações abaixo.
devorando um lagarto ainda vivo, baleias assassinas
atacando focas e pingüins, leões atacando antílopes I. Os atributos relacionados às hienas, no primeiro
etc. Para finalizar, apareciam as detestáveis hienas, parágrafo, traduzem nossa visão “humana” do mundo
“rindo” enquanto comiam os restos de algum pobre natural.
animal. II. A pergunta que abre o segundo parágrafo é respondida
Como a Natureza pode ser assim tão cruel e com os exemplos arrolados nesse mesmo parágrafo.
insensível, indiferente a tanta dor e sofrimento? (Vou III. A frase A ideia de compaixão é puramente humana é
me abster de falar da dor e do sofrimento que a utilizada como comprovação da tese de que a
espécie dominante do planeta, supostamente a de natureza é cruel e insensível.
maior sofisticação, cria não só para os animais, mas Em relação ao texto, está correto APENAS o que se
também para si própria.) Certos exemplos são afirma em:
particularmente horríveis: existe uma espécie de (A) I.
vespa cuja fêmea deposita seus ovos dentro de
lagartas. Ela paralisa a lagarta com seu veneno, e, (B) II.
quando os ovos chocam, as larvas podem se (C) III.
alimentar das entranhas da lagarta, que assiste viva (D) I e II.
ao martírio de ser devorada de dentro para fora, sem (E) I e III.
poder fazer nada a respeito. A resposta é que a
Natureza não tem nada a dizer sobre compaixão ou
ética de comportamento. Por trás dessas ações 3. (TRE-RN – Analista Judiciário – Análise de
assassinas se esconde um motivo simples: a Sistemas) Considerando-se o contexto em que se
preservação de uma determinada espécie por meio da emprega, o elemento em destaque na frase
sobrevivência e da transmissão de seu material (A) Vou me abster de falar da dor e do sofrimento traduz
genético para as gerações futuras. Portanto, para a indiferença do autor em relação ao fenômeno que
entendermos as intenções da vespa ou do leão, temos está analisando.
que deixar de lado qualquer tipo de julgamento sobre (B) Por trás dessas ações assassinas se esconde um
a “humanidade” desses atos. Aliás, não é à toa que a motivo simples revela o tom de sarcasmo, perseguido
palavra humano, quando usada como adjetivo, pelo autor.
expressa o que chamaríamos de comportamento
(C) a Natureza não tem nada a dizer sobre compaixão ou
decente. Parece que isentamos o resto do mundo
ética de comportamento expõe os motivos ocultos que
animal desse tipo de comportamento, embora não
regem o mundo animal.
faltem exemplos que mostram o quanto é fácil nos
juntarmos ao resto dos animais em nossas ações (D) Mas aqui poderíamos também estar falando da
“desumanas”. espécie humana refere-se diretamente ao que se
afirmou na frase anterior.
A ideia de compaixão é puramente humana.
Predadores não sentem a menor culpa quando matam (E) Por trás dessas ações assassinas esconde-se um
as suas presas, pois sua sobrevivência e a da sua motivo simples anuncia uma exemplificação que em
espécie dependem dessa atividade. E dentro da seguida se dará.
mesma espécie? Para propagar seu DNA, machos
podem batalhar até a morte por uma fêmea ou pela 4. (TRE-RN – Analista Judiciário – Análise de
liderança do grupo. Mas aqui poderíamos também Sistemas) Considerando-se o choque e a irritação
estar falando da espécie humana, não? que o autor sentia, quando criança, com as cenas

www.editoradince.com Acesse e veja se há novidades a respeito deste material


8 LÍNGUA PORTUGUESA - TEORIA E QUESTÕES
de crueldade do mundo animal, percebe-se que, determinada espécie quando se for estar transmitindo
com o tipo de argumentação que desenvolve em o material genético.
seu texto, ele pretende: (E) Ao paralisar a lagarta com veneno, a vespa terá
(A) justificar sua tolerância, no presente, com a crueldade depositado seus ovos nela, e as larvas logo se
que efetivamente existe no mundo natural. alimentariam das entranhas da lagarta, que nada
(B) se valer da ciência adquirida, para fazer compreender poderá ter feito para impedi-lo.
como natural a violência que efetivamente ocorre na
Natureza. 8. (TRE-RN – Analista Judiciário – Análise de
(C) se valer da ciência adquirida, para justificar a Sistemas) Temos que deixar de lado qualquer tipo
crueldade como um recurso necessário à propagação de julgamento sobre a “humanidade” desses atos.
de todas as espécies. O segmento sublinhado no período acima pode ser
(D) justificar suas intolerâncias de menino, reações corretamente substituído, sem prejuízo para o
naturais diante da efetiva crueldade que se propaga sentido, por:
pelo mundo animal. (A) nos isentarmos a.
(E) se valer da ciência adquirida, para apresentar a (B) nos eximir para.
hipótese de que os valores morais e éticos contam (C) nos abster de.
muito para o funcionamento da Natureza.
(D) subtrair-nos em
(E) furtar-nos com.
5. (TRE-RN – Analista Judiciário – Análise de
Sistemas) Quanto à concordância verbal, está
inteiramente correta a seguinte frase: 9. (TRE-RN – Analista Judiciário – Análise de
Sistemas) Está inteiramente correta a pontuação
(A) De diferentes afirmações do texto podem-se
do seguinte período:
depreender que os atos de grande violência não
caracterizam apenas os animais irracionais. (A) Paralisada pelo veneno da vespa nada pode fazer, a
lagarta, a não ser assistir viva à sua devoração, pelas
(B) O motivo simples de tantos atos supostamente cruéis,
larvas, que saem dos ovos ali chocados.
que tanto impressionaram o autor quando criança, só
anos depois se esclareceram. (B) Nada pode fazer, a lagarta paralisada, pelo veneno da
vespa, senão assistir viva, à sua devoração pelas
(C) Ao longo dos tempos tem ocorrido incontáveis
larvas que saem dos ovos, e passam a se alimentar,
situações que demonstram a violência e a crueldade
das entranhas da vítima.
de que os seres humanos se mostram capazes.
(C) A pobre lagarta, paralisada pelo veneno da vespa
(D) A todos esses atos supostamente cruéis, cometidos
assiste sem nada poder fazer, à sua devoração pelas
no reino animal, aplicam-se, acima do bem e do mal, a
larvas, tão logo saiam estas dos ovos, que, a
razão da propagação das espécies.
compulsória hospedeira, ajudou a chocar.
(E) Depois de paralisadas as lagartas com o veneno das
(D) Compulsória hospedeira, paralisada pelo veneno da
vespas, advirá das próprias entranhas o martírio das
vespa, a pobre lagarta assiste à devoração de suas
larvas que as devoram inapelavelmente.
próprias entranhas pelas larvas, sem poder esboçar
qualquer tipo de reação.
6. (TRE-RN – Analista Judiciário – Análise de (E) Sem qualquer poder de reação, já que paralisada pelo
Sistemas) NÃO admite transposição para a voz veneno da vespa a lagarta, compulsoriamente,
passiva o seguinte segmento do texto: chocará os ovos, e depois se verá sendo devorada,
(A) centenas de formigas devorando um lagarto. pelas larvas que abrigou em suas entranhas.
(B) ao assistir a documentários sobre a violência do
mundo animal. 10. (TRE-RN – Analista Judiciário – Análise de Sistem)
(C) uma espécie de vespa cuja fêmea deposita seus ovos Atente para as frases abaixo.
dentro de lagartas.
I. Quando criança assistia a documentários sobre a vida
(D) Predadores não sentem a menor culpa. selvagem.
(E) quando matam as suas presas. II. Tais documentários me irritavam.
III. Nesses documentários exibiam-se cenas de extrema
7. (TRE-RN – Analista Judiciário – Análise de violência.
Sistemas) Está inteiramente adequada a Essas frases estão articuladas de modo correto e
articulação entre os tempos verbais na seguinte coerente no seguinte período:
frase:
(A) Irritavam-me aqueles documentários sobre a vida
(A) Predadores não sentirão a menor culpa a cada vez selvagem que assisti quando criança, nos quais
que matarem uma presa, pois sabem que sua continham cenas que exibiam extrema violência.
sobrevivência sempre dependerá dessa atividade.
(B) Naqueles documentários sobre a vida selvagem, a
(B) Se predadores hesitassem a cada vez que tiveram de que quando criança assistia, me irritava, conquanto
matar uma presa, terão posto em risco sua própria exibissem cenas de extrema violência.
sobrevivência, que depende da caça.
(C) Uma vez que exibiam cenas de extrema violência,
(C) Nunca faltarão exemplos que deixassem bem claro o irritava-me com aqueles documentários sobre a vida
quanto é fácil que nos viessem a associar aos selvagem, assistidos quando criança.
animais, em nossas ações “desumanas”.
(D) As cenas de extrema violência me irritavam, quando
(D) Por trás dessas ações assassinas sempre houve um criança, por assistir tais documentários sobre a vida
motivo simples, que estará em vir a preservar uma selvagem, em que eram exibidas.

www.editoradince.com Acesse e veja se há novidades a respeito deste material


LÍNGUA PORTUGUESA - TEORIA E QUESTÕES 9
(E) Os documentários sobre a vida selvagem, a que
Ditongo Tritongo Hiato
assistia quando era criança, irritavam-me porque neles
eram exibidas cenas de extrema violência.
Encontro
Encontro de
semivogal,
Encontro de uma duas vogais
GABARITO: 1. C; 2. A; 3. D; 4. B; 5. E; 6. B; 7. A; 8. vogal e
vogal com uma (nunca de
C; 9.D 10.E semivogal
semivogal na semivogais) em
(sempre nessa
mesma sílaba. sílabas
ordem) na
diferentes.
mesma sílaba.

iguais (i-guais)
FONÉTICA: ENCONTROS papai (pa-pai)
saguão (sa- álcool (ál-co-ol)
CONSONANTAIS E VOCÁLICOS, oi (a palavra "oi"
guão) navio (na-vi-o)
não se separa)
SÍLABA (DIVISÃO E CLASSIFICAÇÃO) sabão (sa-bão)
uruguaio (u- saída (sa-í-da)
ru-guai-o).
FONÉTICA
A Fonética e a Fonologia são uma das partes da
Gramática e tratam dos aspectos fônicos, físicos e DITONGO
fisiológicos de uma língua. Ditongo é o encontro vocálico em que uma vogal e
A Gramática registra e descreve todos os uma semivogal ficam juntas, mesmo quando se faz a
aspectos das línguas. Como sabemos, esses aspectos separação de sílabas. Exemplos: céu (céu), tranquilo
são diversos e seu estudo é organizados em partes: (tran-qui-lo), boi (boi).
Fonética e Fonologia, Morfologia e Sintaxe De acordo com a posição da vogal e da semivogal,
(morfossintaxe), Semântica e Estilística. os ditongos podem ser: crescentes ou decrescentes.
Neste texto vamos refletir a respeito da primeira Ditongos crescentes são aqueles em que a
parte dos estudos da Gramática Descritiva, a Fonética semivogal vem antes da vogal (sv + v). Exemplos: igual
e Fonologia, que tratam dos aspectos fônicos, (i-gual), quota (quo-ta), pátria (pá-tria).
físicos e fisiológicos da nossa língua. Ditongos decrescentes são aqueles em que a
vogal vem antes da semivogal (v + sv). Exemplos: meu
(meu), herói (he-rói), cai (cai).
 Fonética
De acordo com a pronúncia, os ditongos podem
A Fonética é o estudo dos aspectos
ser orais ou nasais.
acústicos e fisiológicos dos sons efetivos (reais) dos
atos de fala no que se refere à produção, Ditongos orais são os pronunciados apenas pela
articulação e variedades. Em outras palavras, boca. É o caso de ai, ia, iu, ui, eu, éu, ue, ei, éi, ie, oi, ói,
a Fonética preocupa-se com os sons da fala em sua io, au, ua, ao, oa, ou, uo, oe, eo, ea. Exemplos: mau
realização concreta. Quando um falante pronuncia a (mau), sei (sei), viu (viu).
palavra 'dia', à Fonética interessa de que forma a Ditongos nasais são os pronunciados pela boca e
consoante /d/ é pronunciada: /d/ /i/ /a/ ou /dj/ /i/ /a/. pelo nariz. É o caso de ão, ãe, õe, am, an, em, en, ãi, ui
(ocorre apenas na palavra "muito"). Exemplos: mãe
(mãe), levem (le-vem), muito (mui-to).
 Fonologia Palavras com ditongo
A Fonologia é o estudo dos Fonemas (os sons)  automóvel (au-to-mó-vel)
de uma língua. Para a Fonologia, o fonema é uma
 boi (boi)
unidade acústica que não é dotada de significado.
Isso significa que os fonemas são os diferentes sons  cai (cai)
que produzimos para exprimir nossas ideias,  céu (céu)
sentimentos e emoções a partir da junção de unidades  deus (deus)
distintas. Essas unidades, juntas, formam as sílabas e  eu (eu)
as palavras.  frequente (fre-quen-te)
A palavra 'Fonema' tem origem grega (fono =  gratuito (gra-tui-to)
som + emas = unidades distintas) e representa  herói (he-rói)
as menores unidades sonoras que formam as  igual (i-gual)
palavras. As palavras são a unidade básica da  jeito (jeito)
interação verbal e são criadas pela junção de unidades  levem (le-vem)
menores: as sílabas e os sons, na fala, ou as sílabas
 madeira (ma-dei-ra)
e letras, na escrita.
 mãe (mãe)
Os fonemas são classificados em vogais,
 mau (mau)
semivogais e consoantes. Essa classificação existe
em virtude dos diferentes tipos de sons produzidos pela  meu (meu)
corrente de ar que sai dos nossos pulmões e é liberada,  muito (mui-to)
com ou sem obstáculos, pela boca e/ou pelo nariz.  noite (noi-te)
 oi (oi)
ENCONTROS VOCÁLICOS  outro (ou-tro)
 papai (pa-pai)
Encontros vocálicos são encontros de vogais ou
 pátria (pá-tria
semivogais, sem consoantes intermediárias.
 quota (quo-ta)
Há três tipos de encontros vocálicos: ditongo,
hiato e tritongo.  respeito (res-pei-to)
 sabão (sa-bão)

www.editoradince.com Acesse e veja se há novidades a respeito deste material


10 LÍNGUA PORTUGUESA - TEORIA E QUESTÕES
 sei (sei)  tio (ti-o)
 tranquilo (tran-qui-lo)  unicórnio (u-ni-cór-ni-o)
 viu (viu)  voo (vo-o)
 zoológico (zo-o-ló-gi-co)
TRITONGO ATENÇÃO! Os ditongos e os tritongos não se
Tritongo é o encontro vocálico em que semivogal + separam, somente os hiatos.
vogal + semivogal ficam juntas, mesmo quando se faz a
separação de sílabas. Exemplos: Uruguai (U-ru-guai),
saguão (sa-guão), enxaguam (en-xa-guam). Encontros vocálicos e consonantais
Os encontros vocálicos são o encontro de vogais
Os tritongos podem ser orais ou nasais.
ou semivogais, sem consoantes entre elas. Exemplos:
Tritongos orais são os pronunciados apenas pela
mãe, poeta, piada.
boca. Exemplos: Paraguai (Pa-ra-guai), enxaguei (en-xa-
guei), iguais (i-guais). Os encontros vocálicos são classificados em
ditongo, tritongo e hiato.
Tritongos nasais são os pronunciados pela boca
Os encontros consonantais são a sequência de
e pelo nariz. As consoantes "m" e "n" podem acompanhar
os tritongos. Quando isso acontecer, os tritongos são duas ou mais consoantes, sem a presença de vogais
entre elas. Exemplos: Brasil, cacto, flor, objetivo,
classificados como tritongos nasais. Exemplos: quão
(quão), saguões (sa-guões), enxaguem (en-xa-guem). psicologia.
Palavras com tritongo Os encontros consonantais podem ser separáveis
ou inseparáveis.
 apaziguou (a-pa-zi-guou)
Encontros consonantais separáveis: quando a
 desaguou (de-sa-guou)
palavra é separada, as consoantes ficam mesma sílaba.
 enxaguam (en-xa-guam) Exemplos: advogado (ad-vo-ga-do), sorte (sor-te), torta
 enxaguei (en-xa-guei) (tor-ta).
 enxaguem (en-xa-guem) Encontros consonantais inseparáveis: quando a
 iguais (i-guais) palavra é separada, as consoantes ficam em sílabas
 minguem (min-guem) diferentes. Exemplos: blusa (blu-sa), livro (li-vro), trem
 Paraguai (Pa-ra-guai) (trem).
 paraguaio (pa-ra-guai-o)
 quais (quais) EXERCÍCIOS
 quão (quão)
 saguão (sa-guão) 01. (Fasp) Indique a alternativa cuja sequência de
 saguões (sa-guões)
vocábulos apresenta, na mesma ordem, o seguinte:
ditongo, hiato, hiato, ditongo:
 Uruguai (U-ru-guai)
a) jamais, Deus, luar, daí
 uruguaio (u-ru-guai-o);
b) joias, fluir, jesuíta, fogaréu
 uruguaiano (u-ru-guai-a-no)
c) ódio, saguão, leal, poeira
d) quais, fugiu, caiu, história
HIATO
Resposta:
Hiato é o encontro vocálico em que duas vogais
aparecem juntas numa palavra, mas ficam em sílabas Alternativa b: joi-as (ditongo), flu-ir (hiato), je-su-í-ta
diferentes quando se faz a separação de sílabas. (hiato), fo-ga-réu (ditongo)
Exemplos: raiz (ra-iz), Saara (Sa-a-ra), país (pa-ís). Quanto às alternativas restantes:
Palavras com hiato a) ja-mais (ditongo), Deus (ditongo), lu-ar (hiato), da-
 afiado (a-fi-a-do) í hiato)
 águia (á-gui-a) c) ó-dio (ditongo), sa-guão (tritongo), le-al (hiato), po-ei-ra
 álcool (ál-co-ol)
(hiato, em "o-e", e ditongo, em "ei")
 baú (ba-ú)
d) quais (tritongo), fu-giu (ditongo), ca-iu (ditongo, em
"iu", e hiato, em "a-i"), his-tó-ria (ditongo)
 coelho (co-e-lho)
 dia (di-a)
 elogio (e-lo-gi-o) 02. (UFV-MG) As sílabas das palavras psicossocial e
traído estão corretamente separadas em:
 faísca (fa-ís-ca)
a) psi – cos – so – ci– al / tra – í – do
 gênio (gê-ni-o)
b) p– si– cos – so – cial / tra – í – do
 hiato (hi-a-to)
c) psi – co – sso – ci – al/ tra– í– do
 iate (i-a-te)
d) p – si – co – sso – cial / tra – í – do
 juiz (ju-iz)
e) psi – co – sso – ci – al / traí – do
 karaokê (ka-ra-o-kê)
Resposta:
 leão (le-ão)
 lua (lu-a)
a) Alternativa adequada, visto que em ambas as palavras
as sílabas estão separadas adequadamente.
 moeda (mo-e-da)
 navio (na-vi-o) b) Alternativa inadequada, visto que as sílabas referentes
à primeira palavra não foram separadas de forma
 oceano (o-ce-a-no)
correta.
 país (pa-ís)
c) Alternativa inadequada, uma vez que as sílabas da
 quieto (qui-e-to)
palavra “traído” estão adequadamente separadas,
 raiz (ra-iz)
mas no que diz respeito à palavra “psicossocial” o
 Saara (Sa-a-ra) mesmo não ocorre.
 saída (sa-í-da)

www.editoradince.com Acesse e veja se há novidades a respeito deste material


LÍNGUA PORTUGUESA - TEORIA E QUESTÕES 11
d) Alternativa inadequada, haja vista que as sílabas sistemático das relações entre fonemas e grafemas,
condizentes à palavra “psicossocial” não foram visando a chegar ao domínio da ortografia do português.
corretamente separadas. O ensino e a aprendizagem dessas regras (leia-se,
e) Alternativa inadequada, pois em ambas as palavras há implicitamente, da ortografia) não são fáceis, tendo em
uma separação errônea das sílabas. vista que o nosso sistema linguístico não é um sistema
fechado, ou seja, não é um sistema transparente de
correlações biunívocas (termo a termo) entre letras e
03. (Urcamp – RS) Os grupos de letras encontrados na
sons. Por exemplo, há regras absolutas, das quais não se
palavra craque são classificados como:
apresentam exceções, como a obrigatoriedade de se
a) apenas encontros consonantais.
acentuarem todas as proparoxítonas, ou de empregar “m”
b) encontro consonantal e dígrafos, respectivamente. antes de “p” e “b”, e “n” antes de outras letras consoantes,
c) dígrafo e encontro consonantal, respectivamente. para indicar a nasalidade da vogal que as antecede. Mas
d) apenas dígrafos. a acentuação de paroxítonas tem uma variedade de
e) Nenhuma das alternativas anteriores. regras bem maior do que a acentuação das
Resposta: proparoxítonas e o emprego da letra “m” pode variar
a) Alternativa incorreta, em virtude da presença, também conforme a situação: no início de sílaba, ela tem um som
de um dígrafo, demarcado pelo grupo “qu”. próprio, como em “mato” ou “cometa”, e não mais indica o
som nasal da vogal; isso mostra também que a posição
b) Alternativa correta, haja vista a demarcação de tais
dessa, e também de outras letras, pode mudar o seu valor
ocorrências: encontro consonantal “cr” e dígrafo “qu”.
fonético, o que terá implicações para o processo de
c) Alternativa incorreta, dada a sequência de um encontro leitura.
consonantal e um dígrafo.
Além disso, há sons com um leque maior de
d) Alternativa incorreta, pois, assim como há um dígrafo, registros, dificultando a formulação de uma só regra. Por
há também um encontro consonantal. exemplo, o fonema /s/ tem várias maneiras de ser
e) Alternativa incorreta, razão essa que se explica pelo representado: russo, caroço, conserto, concerto, excerto,
fato de a alternativa “B” se apresentar como calça, falsa, havendo diversas justificativas para a escolha
verdadeira. do grafema. Do mesmo modo, há letras que representam
sons diferentes, como o “x”: táxi, exato, vexame. O
04. Marque a alternativa que contenha apenas encontros professor deve estar atento às idiossincrasias do sistema,
vocálicos formados por hiato: considerando os erros mais recorrentes entre os alunos e
buscando exemplos que possam ser discutidos, a fim de
a) pouco, Paraguai, depois;
conseguirem concluir e fazer alguma generalização,
b) trouxeram, alguém, saíram; ressaltando-se que certos registros podem ser garantidos
c) atuou, iguais, saíram; por regras, outros têm de ser aprendidos, praticamente,
d) oceano, raiz, coar; um a um. Isso significa dizer que em alguns casos será
e) Nenhuma das alternativas anteriores. possível apelar para o raciocínio amparado nas regras
Resposta: que favorecem as generalizações e, em outros casos,
Letra D será necessário desenvolver estratégias para a
Nos encontros vocálicos das palavras 'oceano', 'raiz' e memorização.
'coar', temos apenas hiatos: o-ce-a-no; ra-iz; co-ar. Ressalta-se também que as regras para leitura
nem sempre coincidem com as de escrita. Por exemplo: a
05. Marque a alternativa cuja palavra apresenta o letra “s”, posicionada entre vogais terá sempre o som de
“z”, na leitura; porém, na escrita não se pode usar só a
encontro vocálico Tritongo:
letra “s” nesse contexto. Vejam-se as palavras “pezinho” e
a) Atuou
“exagero”. Do mesmo modo, pode-se ler “tumati”, mas é
b) Melancia preciso escrever “tomate”. Pode-se dizer, então, que a
c) Milhões língua escrita – na sua dimensão ortográfica – não é
d) Iguais exatamente a língua oral, sendo esse um aspecto a ser
e) Trouxeram lembrado no ensino de regras ortográficas, e no ensino da
Resposta: língua de um modo geral.
Letra D As regras ortográficas podem ser uma ferramenta
A palavra 'IGUAIS' é formada pelo encontro vocálico útil para os aprendizes, desde que não sejam apenas
Tritongo: i-guais. memorizadas, mas resultado de uso, observação, análise,
e novamente uso, do fato em questão, bem orientados
pelo trabalho docente.

ORTOGRAFIA E ACENTUAÇÃO EMPREGO DAS LETRAS K, W E Y


As regras ortográficas referem-se à normatização Siglas e símbolos: kg (quilograma), km
da escrita de palavras a partir de fatos observados na (quilômetro), K (potássio).
língua. A ortografia é, pois, uma convenção que tem de Antropônimos (e respetivas palavras derivadas)
ser obedecida. O estudo das regras ortográficas no originários de línguas estrangeiras: Kelly, Darwin,
ambiente escolar, em geral, acontece quando os darwinismo.
aprendizes já demonstram ter compreendido que as Topônimos (e respetivas palavras derivadas)
unidades menores da fala são representadas por letras, originários de línguas estrangeiras: Kosovo, Kuwait,
ou seja, já perceberam o princípio alfabético do sistema kuwaitiano.
de escrita da língua portuguesa. Nessa fase do processo
Palavras estrangeiras não adaptadas para o
de alfabetização, recomenda-se um trabalho mais
português: feedback, hardware, hobby.

www.editoradince.com Acesse e veja se há novidades a respeito deste material


12 LÍNGUA PORTUGUESA - TEORIA E QUESTÕES
acentuação pois, como vimos acima, independem da
USO DO X/CH fonética.
O x é utilizado nas seguintes situações: Abaixo estão descritas as regras de acentuação
gráfica:
Geralmente, depois dos ditongos: caixa, deixa,
peixe. ● Acentuam-se as palavras monossílabas tônicas
terminadas em a, e, o, seguidas ou não de s.
Depois da sílaba -me: mexer, mexido, mexicano.
Ex.: já, fé, pés, pó, só, ás.
Palavras com origem indígena ou africana: xará,
xavante, xingar. ● Acentuam-se as palavras oxítonas terminadas
em a, e, o, seguidas ou não de s, em, ens.
Depois da sílaba inicial -en: enxofre, enxada,
enxame. Ex.:cajá, café, jacaré, cipó, também, parabéns,
metrô, inglês alguém, armazém, conténs, vinténs.
Exceções:
A palavra "mecha" (porção de cabelo) escreve-se ATENÇÃO: Não se acentuam: as oxítonas
com ch. terminadas em i e u, e em consoantes nem os
infinitivos em i, seguidos dos pronomes
O verbo "encher" escreve-se com ch. O mesmo
oblíquos lo, la, los, las
acontece com as palavras que dele derivem: enchente,
encharcar, enchido. Ex.: ali, caqui, rubi, bambu, rebu, urubu, sutil,
clamor, fi-lo, puni-la, reduzi-los, feri-las.

USO DO H ● Acentuam-se as palavras paroxítonas exceto


aquelas terminadas em a, e, o, seguidas ou não de s, em,
O h é utilizado nas seguintes situações: ens, bem como prefixos paroxítonos terminados em i ou r.
No final de algumas interjeições: Ah!, Oh!, Uh! Ex.: dândi, júri, órfã, César, mártir, revólver,
Por força da etimologia: habilidade, hoje, homem. álbum, bênção, bíceps, espelho, famosa, medo, ontem,
Nos dígrafos ch, lh, nh: flecha, vermelho, manha. socorro, polens, hifens, pires, tela, super-homem.
Nas palavras compostas: mini-hotel, sobre- ATENÇÃO: Acentuam-se as paroxítonas
humano, super-homem. terminados em ditongo oral seguido ou não de
Exceção: A palavra Bahia quando se refere ao s.Ex.: jóquei, superfície, água, área,
estado é uma exceção. O acidente geográfico aniversario, ingênuos.
"baía" é grafado sem h. ● Acentuam-se as palavras proparoxítonas sem
exceção.
USO DO S/Z Ex.: ótimo, incômoda, podíamos, abóbora,
O s é utilizado nas seguintes situações: bússola, cântaro, dúvida, líquido, mérito, nórdico, política,
relâmpago, têmpora.
Nos adjetivos terminados pelos sufixos -oso/-osa
que indicam grande quantidade, estado ou circunstância: ● Acentuam-se os ditongos abertos ei, oi, eu,
bondoso, feiosa, oleoso. seguidos ou não de s em palavras monossílabas e
oxítonas.
Nos sufixo -ês, -esa, -isa que indicam origem, título
ou profissão: marquês, francesa, poetisa. Ex.: carretéis, dói, herói, chapéu, anéis.
Depois de ditongos: coisa, maisena, lousa. ATENÇÃO: não se acentuam ditongos abertos
Na conjugação dos verbos pôr e querer: pôs, quis, ei, oi, eu, seguidos ou não de s em palavras
quiseram. paroxítonas. Ex.: ideia, plateia, assembleia.
O Z, por sua vez, é utilizado nas seguintes Não se acentua, pela nova ortografia, palavras
situações: paroxítonas com hiato oo seguidos ou não de s.
-Nos sufixos -ez/-eza que formam substantivos a Ex.: voos, enjoo, abençoo.
partir de adjetivos: magro - magreza, belo - ● Também não se acentuam as palavras
beleza, grande - grandeza. paroxítonas com hiato ee.
-No sufixo - izar, que forma verbo: atualizar, Ex.: creem, leem, veem, deem.
batizar, hospitalizar. ● Acentuam-se sempre as palavras que
contenham i, u: tônicas; formam hiatos; formam sílabas
ACENTUAÇÃO GRÁFICA sozinhas ou são seguidos de s; não seguidas de nh; não
precedidas de ditongo em paroxítonas; nem repetidas.
O português, assim como outras línguas
neolatinas, apresenta acento gráfico. Sabemos que toda Ex.: aí, balaústre, baú, egoísta, faísca, heroína,
palavra da Língua portuguesa de duas ou mais sílabas saída, saúde, viúvo, juízes, Piauí. Pela regra exposta
possui uma sílaba tônica. acima, não se acentuam: rainha, xiita, ruim, juiz, feiura.
Observe as sílabas tônicas das palavras arte, ● Pela nova ortografia, não se acentua com acento
gentil, táxi e mocotó. Você constatou que a tonicidade agudo u tônico dos grupos que, qui, gue, gui: argui,
recai sobre a sílaba inicial em arte, a final em gentil, a arguis, averigue, averigues, oblique, obliques, apazigues.
inicial em táxi e a final em mocotó. ● Da mesma forma não se usa mais o trema:
Além disso, você notou que a sílaba tônica nem aguento, frequente, tranquilo, linguiça, aguentar, arguição,
sempre recebe acento gráfico. Portanto, todas as palavras unguento, tranquilizante. Emprega-se o til para indicar a
com duas ou mais sílabas terão acento tônico, mas nem nasalização de vogais: afã, coração, devoções, maçã,
sempre terão acento gráfico. A tonicidade está para a relação etc.
oralidade (fala) assim como o acento gráfico está para a ● O acento diferencial foi excluído. Mantém-se
escrita (grafia). É importante aprender as regras de apenas nestas quatro palavras, para distinguir uma da
outra que se grafa de igual maneira:

www.editoradince.com Acesse e veja se há novidades a respeito deste material


LÍNGUA PORTUGUESA - TEORIA E QUESTÕES 13
-pôde (verbo poder no tempo passado) / pode b) para, ruido, tranquilidade
(verbo poder no tempo presente); c) para, ruído, tranquilidade
-pôr ( verbo) / por (preposição); d) pára, ruido, tranqüilidade
-vem ( verbo vir na 3ª pessoa do singular) / e) pára, ruído, tranqüilidade
-vêm ( verbo vir na 3ª pessoa do plural);
-tem ( verbo ter na 3ª pessoa do singular) / têm ( 06. (FCC- TRT-2012) Terminado o .........., o ..........
verbo ter na 3ª pessoa do plural). recebeu .......... aplausos.
a) vôo - herói - veemêntes
QUESTÕES DE CONCURSOS b) voo - heroi - veemêntes
01. (FCC – 2012 – TRF – 2ª R – Analista Judiciário – c) vôo - heroi - veementes
Execução de Mandados) A frase redigida com d) voo, herói, veemêntes
clareza e correção gramatical e ortográfica é: e) vôo, herói, veementes
a) Não é a toa que se diz que futebol e religião não se
discute, pois sempre que surge este debate exalta-se
07. (TRE-MT-2016) A alternativa em que as duas
os ânimos e todos hão de tomar uma atitude
palavras acentuadas não seguem a mesma regra
defensiva.
de acentuação é:
b) Estamos de fato vivendo em uma outra era, onde
a) ninguém - também
haveria mais liberdade, ainda que nos sentimos muito
mais sós do que antes sentiamos. b) dólar - pólo
c) Para os que aceitam e creem em Deus, todos os c) eficiência - próprio
caminhos já estão traçados e aos homens só cabem d) escrúpulos - síntese
percorrê-los de modo a cumprir os desígneos divinos. e) heróis - bóia
d) Muitos cientistas, ao fazerem a apologia da ciência e
criticarem a religião com acidês inaudita, ficando no 08. (TRE-MT) Segue a mesma regra de acentuação de
mesmo patamar dos religiosos mais intransigentes. país a palavra:
e) Os agnósticos parecem ter uma postura equilibrada, a) saúde
tão distante do sectarismo dos muito devotos como do
radicalismo dos ateus mais extremados. b) aliás
c) táxi
02. (FCC - 2012 - TST - Analista Judiciário – Taquigrafia) d) grêmios
Segundo os preceitos da gramática normativa do e) heróis
português do Brasil, a única palavra dentre as
citadas abaixo que NÃO deve ser pronunciada com 09. (TRE-ES) "Aí" é acentuada pelo mesmo motivo de:
o acento tônico recaindo em posição idêntica
àquela em que recai na palavra avaro é: a) aquí
b) dá
a) mister.
c) é
b) filantropo.
d) baú
c) gratuito.
e) porém
d) maquinaria.
e) ibero
10. (TRE-RJ ) A alternativa que apresenta erro quanto
à acentuação em um dos vocábulos é:
03. (FCC- 2015- TRT) À luz de seu magnífico ............-
de-sol, ..............., parece uma cidade ............... . a) lápis - júri
b) bônus - hífen
a) por, Paranavaí, tranquila
c) ânsia - série
b) por, Paranavai, tranquila
d) raízes - amável
c) por, Paranavai, tranqüila
e) Anhangabaú - bambú
d) pôr, Paranavaí, tranqüila
e) pôr, Paranavaí, tranquila
GABARITO: 1-A; 2-E; 3-C; 4-A; 5-C; 6-E; 7-B; 8-A; 9-D;
10-E
04. (FCC- 2012 –TRT) Assinale a alternativa em que
todas as palavras são paroxítonas (foram omitidos Resposta:
os acentos): Q.10: "E". A palavra "bambu", assim como "tatu", "Parati",
"pitu", "caju", não recebe acento gráfico, pois somente as
a) rubrica - avaro - pegada - acrobata
oxítonas terminadas em "a", "e", "o", "em" e "ens" são
b) mister - filantropo - misantropo - condor acentuadas; as que terminam em "i" e "u" não serão;
c) pegaso - prototipo - arquetipo - rubrica já "Anhagabaú" é acentuada por que apresenta hiato
d) necromancia - quiromancia - ibero - nobel formado com sílaba anterior, como os casos de "saúde",
e) nenhuma das anteriores "saída", "faísca", "Maíra"´; nos demais casos, todas as
palavras estão corretamente grafadas.

05. (FCC- TRT- 2014) Por favor, .......... com esse ..........
pois precisamos de .......... .
a) para, ruído, tranqüilidade

www.editoradince.com Acesse e veja se há novidades a respeito deste material


14 LÍNGUA PORTUGUESA - TEORIA E QUESTÕES
(A segunda oração é perceptível mediante o
ARTICULAÇÃO DO TEXTO: COESÃO E contexto. Assim, sabemos que o que está sendo oferecido
COERÊNCIA são ingressos para o concerto.)
A Coesão e a Coerência são mecanismos
fundamentais na construção textual. Conjunção
Para que um texto seja eficaz na transmissão da A conjunção liga orações estabelecendo relação
sua mensagem é essencial que faça sentido para o leitor. entre elas.
Além disso, deve ser harmonioso, de forma a que Exemplo: Nós não sabemos quem é o culpado,
a mensagem flua de forma segura, natural e agradável mas ele sabe. (adversativa)
aos ouvidos.
Coesão Lexical
COESÃO TEXTUAL A coesão lexical consiste na utilização de palavras
A coesão é resultado da disposição e da correta que possuem sentido aproximado ou que pertencem a um
utilização das palavras que propiciam a ligação entre mesmo campo lexical. São elas: sinônimos, hiperônimos,
frases, períodos e parágrafos de um texto. Ela colabora nomes genéricos, entre outros.
com sua organização e ocorre por meio de palavras Exemplo: Aquela escola não oferece as condições
chamadas de conectivos. mínimas de trabalho. A instituição está literalmente caindo
aos pedaços.
MECANISMOS DE COESÃO
A coesão pode ser obtida através de alguns Coerência Textual
mecanismos: anáfora e catáfora. A Coerência é a relação lógica das ideias de um
A anáfora e a catáfora se referem à informação texto que decorre da sua argumentação - resultado
expressa no texto e, por esse motivo, são qualificadas especialmente dos conhecimentos do transmissor da
como endofóricas. mensagem.
Enquanto a anáfora retoma um componente, a Um texto contraditório e redundante ou cujas
catáfora o antecipa, contribuindo com a ligação e a ideias iniciadas não são concluídas, é um texto
harmonia textual. incoerente. A incoerência compromete a clareza do
discurso, a sua fluência e a eficácia da leitura.
ALGUMAS REGRAS Assim a incoerência não é só uma questão de
conhecimento, decorre também do uso de tempos verbais
Confira abaixo algumas regras que garantem a
e da emissão de ideias contrárias.
coesão textual:
Exemplos:
O relatório está pronto, porém o estou finalizando
Referência
até agora. (processo verbal acabado e inacabado)
Pessoal: utilização de pronomes pessoais e
Ele é vegetariano e gosta de um bife muito mal
possessivos. Exemplo: João e Maria casaram. Eles são
passado. (os vegetarianos são assim classificados pelo
pais de Ana e Beto. (Referência pessoal anafórica)
fato de se alimentar apenas de vegetais)
Demonstrativa: utilização de pronomes
Fatores de Coerência
demonstrativos e advérbios. Exemplo: Fiz todas as
tarefas, com exceção desta: arquivar a correspondência. São inúmeros os fatores que contribuem para a
(Referência demonstrativa catafórica) coerência de um texto, tendo em vista a sua abrangência.
Vejamos alguns:
Comparativa: utilização de comparações através
de semelhanças. Exemplo: Mais um dia igual aos
outros… (Referência comparativa endofórica) Conhecimento de Mundo
É o conjunto de conhecimento que adquirimos ao
Substituição longo da vida e que são arquivados na nossa memória.
Substituir um elemento (nominal, verbal, frasal) por São o chamados frames (rótulos), esquemas
outro é uma forma de evitar as repetições. (planos de funcionamento, como a rotina alimentar: café
da amanhã, almoço e jantar), planos (planejar algo com
Exemplo: Vamos à prefeitura amanhã, eles irão na
um objetivo, tal como jogar um jogo), scripts (roteiros, tal
próxima semana.
como normas de etiqueta).
Observe que a diferença entre a referência e a
Exemplo: Peru, Panetone, frutas e nozes. Tudo a
substituição está expressa especialmente no fato de que
postos para o Carnaval!
a substituição acrescenta uma informação nova ao texto.
Uma questão cultural nos leva a concluir que a
No caso de “João e Maria casaram. Eles são pais
oração acima é incoerente. Isso porque “peru, panetone,
de Ana e Beto”, o pronome pessoal referencia as pessoas
frutas e nozes” (frames) são elementos que pertencem à
João e Maria, não acrescentando informação adicional ao
celebração do Natal e não à festa de carnaval.
texto.

Inferências
Elipse
Através das inferências, as informações podem ser
Um componente textual, quer seja um nome, um
simplificadas se partimos do pressuposto que os
verbo ou uma frase, pode ser omitido através da elipse.
interlocutores partilham do mesmo conhecimento.
Exemplo: Temos ingressos a mais para o concerto.
Você os quer?

www.editoradince.com Acesse e veja se há novidades a respeito deste material


LÍNGUA PORTUGUESA - TEORIA E QUESTÕES 15
Exemplo: Quando os chamar para jantar não Não há momento em que a taxa por volta na pista seja
esqueça que eles são indianos. (ou seja, em princípio, diferente de R$l0,00.
esses convidados não comem carne de vaca) e) O quadrado é azul com manchas vermelhas dentro.
O objeto azul é um quadrado com manchas vermelhas
Fatores de contextualização dentro.
Há fatores que inserem o interlocutor na
mensagem providenciando a sua clareza, como os títulos 03. (TRF) O sentido de "...o autor de um bom manual
de uma notícia ou a data de uma mensagem. de aritmética para o ensino médio não é
Exemplo: necessariamente um intelectual, mas, se ele
— Está marcado para às 10h. escrever esse livro adotando critérios
pedagógicos inovadores e eficazes, pode ser" fica
— O que está marcado para às 10h? Não sei
profundamente alterado com a substituição de "se
sobre o que está falando.
ele escrever esse livro" por:
a) caso ele escreva esse livro;
Informatividade
b) conquanto ele escreva esse livro;
Quanto maior informação não previsível um texto
c) desde que ele escreva esse livro;
tiver, mais rico e interessante ele será. Assim, dizer o que
é óbvio ou insistir numa informação e não desenvolvê-la, d) uma vez que ele escreva esse livro;
com certeza desvaloriza o texto. e) escrevendo ele esse livro.
Exemplo: O Brasil foi colonizado por Portugal.
04. (TRF) O sentido de “associará a felicidade ao
Princípios Básicos exercício das virtudes, que, embora árduo,
compele os homens a se defrontar com a
Após termos visto os fatores acima, é essencial ter
realidade” sofre profunda alteração se o
em atenção os seguintes princípios para se obter um texto
enunciado for reescrito do seguinte modo:
coerente:
a) associará a felicidade ao exercício das virtudes, que,
Princípio da Não Contradição - ideias
conquanto árduo, compele os homens a se defrontar
contraditórias
com a realidade;
Princípio da Não Tautologia - ideias redundantes
b) associará a felicidade ao exercício das virtudes, que,
Princípio da Relevância - ideias que se relacionam sendo árduo, compele todavia os homens a se
defrontar com a realidade;
QUESTÕES DE CONCURSOS c) associará a felicidade ao exercício das virtudes, que é
árduo, sim, contudo compele os homens a se
01. (ASEP) "Como a natureza, é capaz de preservar os defrontar com a realidade;
ganhos e erradicar os erros para continuar a
existir."; a forma EQUIVOCADA de reescrever-se d) associará a felicidade ao exercício das virtudes, que,
esse mesmo segmento é: árduo, compele por isso mesmo os homens a se
defrontar com a realidade;
a) É capaz, como a natureza, de preservar os ganhos e
erradicar os erros para continuar a existir; e) associará a felicidade ao exercício das virtudes, o qual,
árduo que seja, compele ainda assim os homens a se
b) Como a natureza, para continuar a existir, é capaz de defrontar com a realidade.
preservar os ganhos e erradicar os erros;
c) Para continuar a existir, é capaz de preservar os
ganhos e erradicar os erros, como a natureza; 05. (TRF) Altera-se profundamente o sentido de "e
todas as ações boas ou más se justificam, desde
d) É capaz de preservar os ganhos, como a natureza, e que sirvam à causa operária.", caso se dê à oração
erradicar os erros para continuar a existir; em destaque a seguinte forma negativa:
e) É capaz de preservar os ganhos e erradicar os erros, a) exceto se não servirem à causa operária;
como a natureza, para continuar a existir.
b) salvo se não servirem à causa operária;
c) a menos que não sirvam à causa operária;
02. (CGJ) “a diminuição dos processos normais de
metabolismo e crescimento economiza energia...” d) a não ser que não sirvam à causa operária;
equivale a “a energia é economizada ao se e) uma vez que não sirvam à causa operária.
reduzirem os processos normais de metabolismo e
crescimento”; o item abaixo em que as duas frases 06. (TRF) A forma que substitui adequadamente a
NÃO se equivalem é: palavra mas, no trecho "compele os homens a se
a) O governo criou a cesta básica para ajudar os pobres. defrontar com a realidade não do ponto de vista da
O governo pretende ajudar os pobres, criando a cesta realização do prazer, mas do dever fazer",
básica. mantendo o sentido original, é:
b) Os alunos estão mais contentes hoje do que ontem. a) senão que também;
Os alunos estavam menos contentes ontem do que estão b) senão;
hoje. c) não obstante;
c) Melões estão na promoção: compre dois a R$l,00 cada d) no entanto;
e leve outro de graça. e) quando não.
Melões estão na promoção, três por R$2,00.
d) O aluguel é de R$10,00 por volta na pista, a qualquer
hora.

www.editoradince.com Acesse e veja se há novidades a respeito deste material


16 LÍNGUA PORTUGUESA - TEORIA E QUESTÕES
07. (TRF) O pronome sublinhado em "Não surpreende
que assim seja, porque desta definição depende CLASSE DE PALAVRAS
em parte o estabelecimento das credenciais dos
CLASSES DE PALAVRAS: FUNÇÕES,
atores que hoje estão envolvidos na luta dos
negros pelo lugar na sociedade brasileira que nem GÊNERO, NÚMERO, EMPREGO.
a abolição, nem os cem anos que a seguiram lhes As palavras se distribuem em classes fora de
propiciaram." está em clara referência ao termo: contexto, na forma como aparecem do dicionário. Já a
a) negros; função só pode ser determinada a partir da análise de sua
ocorrência em frases.
b) atores;
Vejamos a diferença entre classe e função. O
c) credenciais;
dicionário classifica a palavra galinha como substantivo.
d) cem anos; Esta é a classe de que a palavra faz parte. Em frases,
e) envolvidos. podemos identificar a função do substantivo galinha:
A galinha bota ovos.
08. (TRF) A substituição feita abaixo do termo Nesta frase, o substantivo galinha exerce a função
sublinhado no período "Algo semelhante, embora de núcleo do sujeito.
em ponto menor, acontece com a abolição da Vi uma galinha carijó
escravidão" que implica alteração de sentido é:
Nesta outra, o substantivo galinha exerce a função
a) não obstante em ponto menor; de núcleo do objeto direto.
b) ainda que em ponto menor; Gosto de carne de galinha.
c) conquanto em ponto menor; Nesta terceira frase, o substantivo galinha exerce a
d) desde que em ponto menor; função de núcleo do adjunto adnominal.
e) apesar de que em ponto menor. A NGB divide as palavras da língua portuguesa em
dez classes, chamadas de classes gramaticais. São
elas:
09. (TRF) O pronome destacado em "... e a frente do
movimento pelo fim das discriminações raciais. 1) substantivo - palavra que dá nome aos seres:
Ambas são políticas, mas a primeira O é de forma crianças, língua, tribo, Brasil.
mediatizada" tem a função de substituir, no texto: 2) adjetivo - palavra que caracteriza os seres e as
a) o adjetivo político; coisas: longa, humanitário, novo.
b) o substantivo discriminações; 3) verbo - palavra que indica fato ou estado:
aprender, mandar, permanecer, estar.
c) o verbo ser;
4) pronome - palavra que representa ou
d) o substantivo movimento;
acompanha o substantivo, considerado-o
e) o numeral primeira. apenas como pessoa do discurso: eles, sua.
5) numeral - palavra que indica quantidade ou
10. (TRF) Dentre as modificações impostas à frase ordem: 1533, II.
"Embora seguindo regras distintas, as duas 6) artigo - palavra que acompanha o substantivo,
frentes são importantes e se alimentam determinado ou indeterminado-o: a, uma.
mutuamente ou, pelo menos, deveriam fazê-lo", a
7) advérbio - palavra que indica circunstância de
que NÃO implica mudança de sentido é:
lugar, modo, intensidade etc.: aqui, bem,
a) Embora seguindo regras, distintas as duas frentes, são
8) preposição - palavra que serve para ligar dois
ambas importantes e se alimentam mutuamente ou,
termos de uma oração: de.
pelo menos, deveriam fazê-lo.
9) conjunção - palavra que serve para relacionar
b) Embora as duas frentes sigam regras distintas, são
duas orações ou termos semelhantes de uma
ambas importantes e se alimentam mutuamente ou,
mesma oração: e.
ao menos, deveriam fazê-lo.
10) Interjeição - palavra que expressa sentimento
c) Embora seguindo regras distintas, as duas frentes são
ou emoção: Nossa!.
importantes e se nutrem mutuamente, já que
precisariam fazê-lo. Essas dez classes de palavras distribuem-se em
dois grupos:
d) Embora seguindo regras distintas, as duas frentes ou
são importantes ou se alimentam mutuamente: pelo a) variáveis: palavras que podem apresentar
menos, deveriam fazê-lo. mudança na forma, quando contextualizadas.
São elas: substantivo, adjetivo, verbo, artigo,
e) Embora com regras distintas, as duas frentes ou são
pronome e numeral.
importantes ou se alimentam mutuamente, já que
assim deveriam fazê-lo. b) invariáveis: palavras que não apresentam
mudança na forma, mesmo contextualizadas.
São elas: advérbio, preposição, conjunção e
GABARITO: 01.D 02.C 03.B 04.D 05.E 06.B 07.A interjeição.
08.D 09.A 10.B

TIPOS DE CLASSES DE PALAVRAS


→ Palavras variáveis
Os substantivos, adjetivos, artigos, numerais,
pronomes e verbos são palavras variáveis, pois têm a
constituição modificada para marcar alguns elementos
gramaticais, como o

www.editoradince.com Acesse e veja se há novidades a respeito deste material


LÍNGUA PORTUGUESA - TEORIA E QUESTÕES 17
 gênero (masculino/feminino); Exemplos:
 número (singular/plural); - O amor de Antônia era forte (não é qualquer
 pessoa (primeira, segunda e terceira); amor, sabe-se que é um específico).
 tempo (pretéritos, presente, futuros); - A história revelará a verdade (refere-se à
história da humanidade).
 modo (indicativo, subjuntivo, imperativo).
- Os bandidos atacaram novamente (sabe-se que
→ Palavras invariáveis
são as mesmas pessoas que cometeram os crimes).
Os advérbios, as preposições, as conjunções e
- As rosas do jardim estão secas (o sujeito que
as interjeições são palavras invariáveis, já que não
fez tal afirmação fez menção a determinadas flores).
comportam transformações em suas formas.

SUBSTANTIVO  Indefinido: generaliza o substantivo.


Substantivo é uma classe gramatical cuja Exemplos:
função é nomear os seres em geral. Apesar de essa - Falta um cantor para completar o espetáculo
conceituação estar presente em vários locais, há que se (pode ser o João, o José, o Miguel, qualquer um que
destacar a sua incompletude, já que o substantivo pode cante).
também ser responsável por denominar: - Gostaria de saber se há uma lanchonete na
 ações (abraço, chute); região (no caso, a pessoa não especificou qual a
 postulados físicos (inércia); lanchonete, podendo ser a do João, a da Maria, entre
outras)
 aspectos emocionais e psicológicos (covardia,
esquizofrenia, ansiedade, amor, ódio); - Em cima da mesa, estão uns biscoitos (a
informação impossibilita saber a marca, o tipo das
 elementos socioculturais (pobreza,
bolachas).
inteligência), entre outros.
- Seria ótimo conhecer umas atrizes famosas
(não se sabe quais atrizes são).
→ Classificações dos substantivos
 Comum: é responsável por nomear ADJETIVO
a generalidade dos seres da mesma espécie, dos
O adjetivo é uma palavra ou locução (iniciada
elementos abstratos, dos objetos e dos fenômenos da
por preposições: de, em, com, sem) que confere
natureza. Exemplos: casa, ódio, neve.
características, estados, qualidades aos seres.
 Próprio: faz referência a um ser específico. Também pode instituir relações de tempo, de espaço,
Exemplos: Maria, Panela de Barro Restaurante (no de finalidade, de procedência com o substantivo.
caso, panela de barro identifica um restaurante
Exemplos:
determinado).
- Banca de revistas (locução adjetiva)
 Primitivo: termos que não se originaram de
outros existentes na mesma língua. Exemplos: maçã, - Avaliação semanal (tempo)
porta, livro. - Cidade estrangeira (espaço)
 Derivado: palavras que provêm de outras. - Vinho chileno (procedência)
Exemplos: macieira (árvore) – maçã (fruta), portaria – - Emergência ortopédica (finalidade)
porta, livreiro – livro.
 Simples: são constituídos por apenas um
→ Classificações dos adjetivos
radical (parte da palavra que carrega o sentido principal
dela). Exemplos: garrafa, tênis, feijão.  Primitivos: não advêm de outro termo
 Compostos: têm mais de um radical em sua existente na língua e possuem apenas um radical.
estrutura. Exemplos: beija-flor, passatempo (verbo Ressalta-se que existem poucos adjetivos primitivos.
passar + substantivo tempo). Exemplos: azul, roxo, verde, branco, grande,
 Concreto: nomeiam seres de existência escuro, liso, feliz, triste.
própria, isto é, figuras independentes que fazem parte DICA: A maior parte dos adjetivos primitivos
de um universo real ou imaginário. Exemplos: caneta, são cores.
vampiro (entidade), São Paulo (cidade), Ministério da
 Derivados: são originados de outras palavras.
Saúde (instituição).
Assim, são acrescentados afixos (partes das palavras
 Abstrato: designam qualidades, ações, que carregam um sentido complementar ao principal,
sentimentos, estados, sensações. Exemplos: soberba, por exemplo, infeliz, em que o in significa não) ao
riso, solidão, juventude, conforto. radical.
Exemplos:
ARTIGO - desfavorável - favor
O artigo é a palavra que precede os - esverdeado - verde
substantivos a fim de determiná-los, tanto de
- europeia – Europa
maneira particular, por meio do uso de o, a, os, as,
quanto de modo vago, ao utilizar um, uma, uns, umas.
 Simples: tem apenas um radical.
→ Classificações dos artigos Exemplos: azul, desfavorável, escuro.
 Definido: individualiza o substantivo, ou seja,
leva o interlocutor a saber do que se trata  Compostos: possuem mais de um radical.
especificamente. Exemplos: amarelo-canário, sociopolítico.

www.editoradince.com Acesse e veja se há novidades a respeito deste material


18 LÍNGUA PORTUGUESA - TEORIA E QUESTÕES
- Os professores nos auxiliaram a entender a
NUMERAL matéria.
O numeral é responsável por quantificar, de - A informação não foi enviada a eles.
forma exata, os seres (pessoas, objetos, entre outros).  Pronomes possessivos: determinam uma
Além disso, também tem a função de identificar a relação de posse, de algo pertencente às pessoas do
posição ocupada por um ser em um contexto discurso.
específico.
Um possuidor Vários possuidores
Um Vários Vários
→ Classificações dos numerais Um objeto
objeto objetos objetos
 Cardinais: apresentam o número preciso de
algo. Destaca-se o fato de que, mesmo que os Meu/ Meus/ Nosso/ Nossos
1ª pessoa
numerais sejam considerados palavras variáveis, nesta minha minhas nossa /nossas
categoria, apenas o termo um, o dois e os referentes às Teus/ Vosso/ Vossos/
2ª pessoa Teu/ tua
centenas a partir de duzentos são modificados. tuas vossa vossas
Exemplos: Seus/ Seus/
- Encontrei apenas um lápis no estojo. 3ª pessoa Seu/ sua Seu/ sua
suas suas
- Na sala de cirurgia, havia uma enfermeira Exemplos:
e quatro médicos.
- Nossas férias foram especiais.
- Recebi duzentas moedas.
- João, onde está a sua tarefa?

 Ordinais: conforme a própria palavra já


anuncia, diz respeito à ordem, assim sempre se  Pronomes demonstrativos: são utilizados
estabelecerá uma relação entre vários seres. para determinar as distâncias tanto físicas quanto
cronológicas de algo em relação às pessoas do
Exemplos: discurso.
- O segundo a alcançar a linha de chegada é
Variáveis Invariáveis
goiano.
- É a milésima vez que digo isso. este, esta, estes, estas isto
esse, essa, esses, essas isso
PRONOME aquele, aquela, aqueles, aquelas aquilo
O pronome, além de estabelecer quais são os Exemplos:
seres que fazem parte diretamente da
interlocução (1ª e 2ª pessoas), ainda são empregados - Este joelho só serve para doer. (proximidade de
para indicar os demais presentes no discurso (3ª quem fala)
pessoa), ou seja, essa classe gramatical faz referência
a quem fala, com quem se fala e a de quem ou do que
se fala. Diante dessa característica, normalmente
substituem os substantivos. DICA:
Exemplo: Para se referir ao celular, o homem deve
Luíza (substantivo) comprou um carro. utilizar o pronome “este”.
Agora, ela (pronome) chega mais rapidamente aos
- Finalmente chegou esta hora. (atualidade)
lugares.
- O grande acontecimento de hoje foi este: ir ao
supermercado. (introduz uma ideia)
→ Classificações dos pronomes
- Nossa, essa pulseira é linda. (proximidade de
 Pronomes pessoais: representam as pessoas quem ouve)
gramaticais. - Nesse intervalo, eu fiz muitas coisas. (tempo
Caso reto Caso oblíquo que está imediatamente antes do presente)
(função de (função de - Comprei um celular, mas esse não funciona
sujeito) complemento) muito bem. (retoma uma informação)
Tônicos - Você conhece aquela cachoeira? (distância
Átonos tanto de quem fala quanto de quem ouve)
(com
(sem preposição) - Consegui construir um celeiro e um
preposição)
lago. Este ficou bem feito, entretanto aquele não.
Singular eu me mim (elemento referido anteriormente a outro)
Singular tu te ti - Naquela década, as mulheres não podiam
Singular ele/ela se, o, a, lhe si, ele/ela votar. (tempo distante)
Obs: contração da preposição em + pronome
Plural nós nos nós
aquela = naquela.
Plural vós vos vós
Plural eles/elas Se, os, as, lhes si, eles/elas  Pronomes indefinidos: fazem referência, de
Exemplos: maneira vaga, à 3ª pessoa gramatical.
- Eu fui ao shopping hoje. Exemplos:

www.editoradince.com Acesse e veja se há novidades a respeito deste material


LÍNGUA PORTUGUESA - TEORIA E QUESTÕES 19
- Algum membro da plateia gostaria de falar?  Subjuntivo: apresenta hipóteses, dúvidas.
- Todas as flores estão belíssimas durante a Exemplo: Se eu quisesse, estudaria muito mais.
primavera.
 Imperativo: manifesta ordem, pedido,
- Certas pessoas não praticam os sugestão.
exercícios certos.
Exemplo: Faça a sua tarefa, Rodrigo.
Pronome (antes do substantivo)
adjetivo (depois do substantivo)
→ Tempos
- Presente: o instante no qual ocorre a ação
 Pronomes relativos: iniciam novas orações ao verbal coincide com o do discurso.
substituírem um substantivo ou mesmo um pronome Exemplo: Eu amo você.
antecedente.
Exemplos:
- Passado: o momento em que acontece a ação
- Visitamos a cidade onde minha avó mora. verbal é anterior ao do discurso.
- Fui eu quem escolheu a decoração. - Pretérito perfeito: o fato exposto tem final bem
- Caíram as ações cuja liquidez era tida como delimitado e concluído antes de ser exteriorizado, por
certa. meio do uso da língua.
Exemplo: Eu corri durante a manhã de hoje.
 Pronomes interrogativos: são observados em
frases ou orações interrogativas, sejam elas diretas, ou - Pretérito imperfeito: o episódio exteriorizado
seja, cuja conclusão se dá por meio do uso de ponto de pelo verbo não foi finalizado quando um novo
interrogação e o início mediante a colocação do aconteceu.
pronome, sejam elas indiretas, isto é, terminadas por
Exemplo: No momento em que começamos a
ponto-final e entremeadas pelos termos de teor
ler, havia o barulho da reforma.
questionador.
Além disso, também apresenta fatos passados
Exemplos:
que eram habituais.
- Quanto custa esta dúzia de bananas?
Exemplo: Andrea cuidava de seus animais
- Eu gostaria de saber quem teve a brilhante diariamente.
ideia de pintar a parede.
- Qual é o dia da Proclamação da República?
- Pretérito mais-que-perfeito: a ocorrência
contida no verbo é anterior à outra que também é
VERBO situada no passado.
Os verbos são palavras que expressam uma Degustei a sobremesa feita pela Fernanda, mas,
ação, um estado, um fenômeno, os quais se antes disso, eu comera um macarrão.
encontram situados cronologicamente. Essa classe de
palavras é uma das que mais flexiona, pois se adapta à
- Futuro do presente: indica episódios cujas
pessoa, ao número, ao tempo, ao modo, além de conter
ocorrências serão concretizadas depois da fala ou da
as formas nominais.
escrita.
Exemplo: Amanhã viajarei para a praia.
→ Formas nominais
 Infinitivo: expressa o fato verbal em si, - Futuro do pretérito: expressa um fato futuro,
portanto não há pistas do início ou término da ação, mas conectado a um segundo que está situado no
estado ou fenômeno. Assim, adquire valor de passado.
substantivo.
Exemplo: A cabeleireira confirmou
Exemplo: Nadar é um ótimo esporte. que viria agora.
 Gerúndio: determina o processo, ou seja, algo
que está acontecendo no momento do discurso. → Classificações dos verbos
Exemplo: Victor está caminhando.
 Regulares: independentemente da
 Particípio: marca a conclusão de um fato. conjugação, o verbo segue o paradigma, ou seja,
Muitas vezes adquire valor de adjetivo. mantém o seu radical, e as desinências (final da
palavra) seguem um padrão.
→ Conjugações Exemplos:
 1ª conjugação: verbos terminados em - - Eu amo, tu amas, ele ama, nós amamos, vós
ar. Exemplos: cantar, beijar, mascarar. amais, eles amam.
 2ª conjugação: verbos terminados em -er. - Eu abraço, tu abraças, ele abraça, nós
Exemplos: beber, comer, fazer. abraçamos, vós abraçais, eles abraçam.
 3ª conjugação: verbos terminados em -ir. Perceba que o radical am- e abraç- permanecem
Exemplos: partir, dividir, rir. os mesmos, e as desinências coincidem entre os dois
verbos.

→ Modos
 Indicativo: exprime certeza.
Exemplo: Eu paguei a conta da internet.

www.editoradince.com Acesse e veja se há novidades a respeito deste material


20 LÍNGUA PORTUGUESA - TEORIA E QUESTÕES
 Irregulares: não estão de acordo com o Exemplos: acaso, provavelmente,
paradigma, assim podem sofrer modificação tanto o eventualmente, quiçá, talvez, porventura.
radical quanto as desinências. - O evento, provavelmente, será cancelado.
Exemplo:  Tempo: situam a ação, a qualidade ou a
- Eu faço, tu fazes, ele faz, nós fazemos, vós circunstância no tempo.
fazeis, eles fazem. Exemplos: amanhã, sempre, cedo, tarde, hoje,
Observe que o radical faz foi modificado na nunca, antes, depois, outrora, então, aí.
conjugação da 1ª pessoa do singular: faço. - Farei isso depois.

 Anômalos: apresentam substanciais  Lugar: marcam o local no qual ocorreu um


irregularidades em seus radicais. episódio, ou onde se percebeu uma qualidade, ou
Exemplo: circunstância.
Verbo ir – eu vou (presente do indicativo), eu ia Exemplos: lá, ali, aqui, aí, longe, onde, perto,
(pretérito imperfeito do indicativo), eu fui (pretérito abaixo, acima.
perfeito do indicativo), quando eu for (futuro do - Nossa, como você está longe de casa.
subjuntivo).
 Defectivos: são verbos cuja conjugação não CONJUNÇÃO
existe em determinadas pessoas do discurso.
Conjunção é um elemento gráfico, sonoro e
Exemplo: eu chovo (inexistente). semântico que estabelece uma união entre orações
ou entre palavras, desde que estas exerçam idênticas
 Abundantes: apresentam mais de uma forma funções sintáticas e estejam na mesma oração. Essa
para uma flexão específica. classe de palavras pode ser dividida em:
Exemplo: particípio de matar — matado e morto.  subordinativas: conecta duas orações, sendo
uma delas fundamental para a construção do sentido
completo da outra;
ADVÉRBIO
São palavras que se conectam aos verbos a
 Coordenativas: que liga elementos
independentes, ou seja, une orações ou termos de
fim de apresentar uma circunstância relativa à ação,
idêntica função gramatical detentores de sentido
estado, fenômeno verbal. Além disso, podem associar-
completo.
se aos adjetivos, conferindo uma determinação das
qualidades expressas por eles. Por fim, são capazes de
se juntar a outros advérbios, o que desencadeia uma → Tipos de conjunções coordenativas
intensificação dos sentidos ali presentes.  Aditiva: expressa soma.
→ Principais classificações dos advérbios Exemplo: Fui ao cinema e comi pipoca.
 Modo: advérbios que acrescentam ao verbo,
adjetivo ou a outro advérbio a maneira como aconteceu
o que eles expressam.  Adversativa: estabelece uma oposição entre
as orações.
Exemplos: bem, mal, assim, depressa, devagar,
tranquilamente, facilmente. Exemplo: As crianças gostam do Natal, mas as
famílias estão cada vez mais descrentes.
- Rosana pegou rapidamente a vassoura.

 Alternativa: constrói uma alternância de ideias,


 Intensidade: constituem uma maximização ou mas também exclui uma para que a outra vigore.
minimização da ideia manifestada pelo termo a que se
ligou. Exemplo: Independência ou morte! (D. Pedro)
Exemplos: muito, pouco, meio, bastante, ainda,
bem, mal, quase, apenas.  Conclusiva: introduz uma consequência e um
- Jônatas gritou bastante no show. fechamento do raciocínio desenvolvido na oração
anterior.
Exemplo: Os cachorros brincaram na
 Afirmação: confirmam a mensagem lama, portanto eles se sujaram.
transmitida.
Exemplos: certamente, realmente, seguramente,
sim, efetivamente.  Explicativa: apresenta uma justificativa.
- A classe desistiu realmente de fazer a Exemplo: Não assisto mais aos DVDs, pois vejo
atividade. filmes on-line.

 Negação: trazem uma ideia contrária à → Tipos de conjunções subordinativas


existente no verbo, adjetivo ou advérbio.  Causais: insere a causa de um acontecimento
Exemplos: jamais, não, absolutamente. presente na outra oração.
- Não quero comer sanduíche. Exemplo: Reescreva este trecho da
redação porque está confuso.

 Dúvida: transmitem uma incerteza relativa ao


que está previsto na oração.

www.editoradince.com Acesse e veja se há novidades a respeito deste material


LÍNGUA PORTUGUESA - TEORIA E QUESTÕES 21
 Condicionais: estabelece um requisito para a
concretização de algo. INTERJEIÇÃO
Exemplo: Se eu fizer exercícios físicos, São as palavras ou um grupo delas que marcam,
emagrecerei. de maneira forte e abrupta, as reações, os
sentimentos, as emoções.
 Conformativa: expressa uma concordância. Exemplos:
Exemplo: Segundo a notícia do jornal, a - Puxa!
pandemia gerou muitos mortos. - Cuidado!
- Ufa!
 Concessiva: manifesta uma ideia que - Nossa!
contrapõe a existente na oração anterior, mas não tem - Tomara!
a força de anulá-la. - Credo!
Exemplo: Só beberei água quando chegar em
casa, embora esteja com sede.

 Comparativa: faz um paralelo.


Exemplo: Tomás levanta da cama como um urso
sai de sua toca no inverno. DICA:
A surpresa encontra na interjeição a sua forma
 Consecutiva: apresenta um desdobramento. verbal.
Exemplo: A opulência era tamanha que os olhos
dela brilharam. EXERCÍCIOS RESOLVIDOS
 Proporcionais:constrói uma relação de
Questão 1- (UFES - adaptada) O fragmento onde a
igualdade entre os fatos contidos em cada uma das
palavra que é pronome relativo é:
orações.
A) (...) a tuberculose está mais próxima
Exemplo: À medida que as pessoas
do que imaginamos (...) (linha 2)
desmatavam a floresta, crescia o buraco na camada de
ozônio. B) A estimativa de órgãos oficiais é de que hoje em dia
cerca de um terço da população mundial (...) (linha
11)
 Temporais: expressa uma circunstância de C) (...) a pessoa que tem o bacilo, mas não
tempo. desenvolveu a doença (...) (linhas 12-13)
Exemplo: Quando o relógio apontar dez horas, D) (...) a expectativa é de que a chance de
tomarei o meu café. desenvolvimento da doença seja apenas 10% (...)
 Finais: expõe o objetivo de algo presente na (linha 14)
outra oração. E) (...) sendo que, ao final de 10 anos, a grande maioria
Exemplo: Estude bastante para que você tenha (...) (linha 17)
sucesso. Resolução
Alternativa C. Nesse trecho, "que" é pronome relativo
PREPOSIÇÃO porque retoma o termo imediatamente anterior, "pessoa",
A preposição é o termo que conecta duas sendo sujeito do verbo "tem"; além disso, introduz uma
outras palavras, construindo relações de sentido e oração.
dependência entre elas.
Questão 2 (UPE – 2015 - adaptada) Acerca de algumas
→ Classificação das preposições relações semânticas presentes nas frases e orações
a seguir, assinale a alternativa correta.
 Essenciais: exercem apenas a função de
preposição. A) Ao afirmar: “Gosto de trabalhar com o português,
embora inglês seja a que eu mais leio.”, o
Exemplos: a, ante, contra, de, entre, sob, sobre. entrevistado faz uma afirmação e, em seguida,
- A agenda está sobre a mesa. apresenta uma causa para o que foi afirmado.
- A filha de Antenor se retirou do recinto. B) Com o trecho: “Shakespeare fez muita besteira, mas
tem três ou quatro obras perfeitas”, o locutor faz
uma declaração e, em seguida, introduz a
 Acidentais: palavras de classes diversas que,
explicação do conteúdo declarado.
às vezes, desempenham a função prepositiva.
C) No trecho: “Aprendi desde cedo a ter o cuidado de
Exemplos: como, conforme, mediante, segundo,
não rimar ao escrever uma frase.”, o segmento
senão, visto.
destacado tem valor temporal.
- Isabela vai à casa do namorado todos os
D) O trecho: “Se todo mundo erra na crase é a regra da
dias, fora segunda-feira.
crase que está errada, como aliás está.” é
introduzido por um segmento que tem valor
concessivo.
E) No trecho: “insistiram que o certo é ‘veado’ quando o
Brasil inteiro pronuncia ‘viado’”, os segmentos estão

www.editoradince.com Acesse e veja se há novidades a respeito deste material


22 LÍNGUA PORTUGUESA - TEORIA E QUESTÕES
interligados por uma relação de causa e a) Como estivesse cansado, não foi trabalhar.
consequência. b) Assim que fores ao Rio, não te esqueças de
Resolução avisar-me.
Alternativa C. No trecho, o segmento destacado tem c) Retirou-se antes, já que assim o quis.
valor semântico temporal, ou seja, seu sentido indica uma
circunstância de tempo na qual se situa uma ação, no caso, o d) Não se aborreça, que estamos aqui para ouvi-lo.
cuidado que o autor revela ter "ao escrever uma frase”. e) Não compareceu, porque não foi avisado.

QUESTÕES DE CONCURSOS ///


6. (SANTA CASA) O "que" está com função de
1.(IBGE) Assinale o par de frases em que as preposição na alternativa:
palavras sublinhadas são substantivo e pronome, a) Veja que lindo está o cabelo da nossa amiga!
respectivamente: b) Diz-me com quem andas, que eu te direi quem
a) A imigração tornou-se necessária. / É dever és.
cristão praticar o bem. c) João não estudou mais que José, mas entrou na
b) A Inglaterra é responsável por sua economia. / Faculdade.
Havia muito movimento na praça. d) O Fiscal teve que acompanhar o candidato ao
c) Fale sobre tudo o que for preciso. / O consumo banheiro.
de drogas é condenável. e) Não chore que eu já volto.
d) Pessoas inconformadas lutaram pela abolição. /
Pesca-se muito em Angra dos Reis. 7. (BB) "Saberão que nos tempos do passado o
e) Os prejudicados não tinham o direito de doce amor era julgado um crime."
reclamar. / Não entendi o que você disse. a) 1 preposição
b) 3 adjetivos
2. (U-BRASÍLIA) Assinale o item que só contenha c) 4 verbos
preposições:
d) 7 palavras átonas
a) durante, entre, sobre
e) 4 substantivos
b) com, sob, depois
c) para, atrás, por
8. (UC-MG) Em "Orai porque não entreis em
d) em, caso, após tentação", o valor da conjunção do período é de:
e) após, sobre, acima a) causa
b) condição
3.(TTN) Observe as palavras grifadas da seguinte c) conformidade
frase: "Encaminhamos a V. Senhoria
d) explicação
cópia autêntica do Edital nº 19/82." Elas são,
respectivamente: e) finalidade
a) verbo, substantivo, substantivo
b) verbo, substantivo, advérbio 9. (UF-MG) As expressões sublinhadas
correspondem a um adjetivo, exceto em:
c) verbo, substantivo, adjetivo
a) João Fanhoso anda amanhecendo sem
d) pronome, adjetivo, substantivo
entusiasmo.
e) pronome, adjetivo, adjetivo
b) Demorava-se de propósito naquele complicado
banho.
4. (CESGRANRIO) Assinale a opção em que a c) Os bichos da terra fugiam em desabalada
locução grifada tem valor adjetivo: carreira.
a) "Comprei móveis e objetos diversos que entrei a d) Noite fechada sobre aqueles ermos perdidos da
utilizar com receio." caatinga sem fim.
b) "Azevedo Gondim compôs sobre ela dois e) E ainda me vem com essa conversa de
artigos." homem da roça.
c) "Pediu-me com voz baixa cinquenta mil réis."
d) "Expliquei em resumo a prensa, o dínamo, as 10. (ITA) Assinalar a alternativa que corretamente
serras..." preenche a lacuna da sentença: "....... meus
e) "Resolvi abrir o olho para que vizinhos sem conselhos, ele pediu demissão."
escrúpulos não se apoderassem do que era a) Entrementes
delas."
b) Máxime
c) Mormente
5. (EPCAR) Aponte a alternativa em que a palavra
d) Malgrado
em negrito é conjunção explicativa:
e) Destarte

www.editoradince.com Acesse e veja se há novidades a respeito deste material


LÍNGUA PORTUGUESA - TEORIA E QUESTÕES 23
Derivação regressiva
GABARITO: 1. E; 2.A; 3.C; 4.E; 5.D; 6. D; 7. E; 8. E.; 9. Na derivação regressiva, também chamada de
B; 10. D formação deverbal, ocorre redução da palavra primitiva e
não acréscimo.
 boteco (de botequim)
 dispensa (do verbo dispensar)
PROCESSOS DE FORMAÇÃO DE  bandeja (do verbo bandejar)
PALAVRAS  remoinho (do verbo remoinhar)

Maneira como os morfemas se organizam para


formar as palavras. Derivação imprópria
Neologismo Na derivação imprópria, atualmente chamada de
Beijo pouco, falo menos ainda. conversão por algumas gramáticas, não há alteração da
Mas invento palavras palavra primitiva, que permanece igual. Há, contudo,
Que traduzem a ternura mais funda mudança de classe gramatical com consequente
E mais cotidiana. mudança de significado: verbos passam a substantivos,
Inventei, por exemplo, a verbo teadorar. adjetivos passam a advérbios, substantivos passam a
Intransitivo: adjetivos,...
Teadoro, Teodora.  jantar (verbo para substantivo)
(BANDEIRA, Manuel. Estrela da vida inteira. Rio de  andar (verbo para substantivo)
Janeiro: José Olympio, 1970)
 prodígio (substantivo para adjetivo)
A formação de palavras é feita por dois
processos principais: a derivação e a composição.  baixo (adjetivo para advérbio)
Contudo, existem também outros processos de formação
de palavras que, embora com menor regularidade e Composição
sistematicidade, contribuem para a formação de novas
Na composição, a formação de uma nova palavra
palavras, como a abreviação, a reduplicação, o
ocorre a partir da junção de duas ou mais palavras
hibridismo, a combinação e a intensificação. Vejamos.
simples ou radicais. Formam-se assim palavras
compostas com significação própria.
Derivação O processo de composição pode ocorrer por
Na derivação, a formação de uma nova palavra justaposição ou por aglutinação.
ocorre a partir de uma única palavra simples ou radical,
ao qual se juntam afixos, formando uma nova palavra com
Composição por aglutinação
significação própria.
Ocorre composição por aglutinação quando há
Existem cinco tipos de derivação: derivação
alteração das palavras formadoras. Ocorre a fusão de
prefixal, derivação sufixal, derivação parassintética,
duas ou mais palavras simples ou radicais, havendo
derivação regressiva e derivação imprópria.
supressão de fonemas. Os elementos formadores
perdem, assim, a sua identidade ortográfica e fonológica
Derivação prefixal porque a nova palavra composta apresenta apenas um
Na derivação prefixal acrescenta-se um prefixo a acento tônico.
uma palavra já existente.  aguardente (água + ardente);
 desnecessário (des- + necessário)  embora (em + boa + hora);
 contramão (contra- + mão)  planalto (plano + alto);
 antebraço (ante- + braço)  vinagre (vinho + acre).
 infeliz (in- + feliz) Composição por justaposição
Ocorre composição por justaposição quando não
Derivação sufixal há alteração das palavras formadoras. Ocorre apenas a
junção de duas ou mais palavras simples ou radicais, que
Na derivação sufixal acrescenta-se um sufixo a
uma palavra já existente mantêm a mesma ortográfica e acentuação que
apresentavam antes do processo de composição. A maior
 ciumento (ciúme + -ento) parte das palavras compostas por justaposição estão
 velozmente (veloz + -mente) ligadas com um hífen. Contudo, é possível a escrita de
 orgulhoso (orgulho + -oso) palavras compostas por justaposição sem hífen ou
 namorico (namoro + ico) apenas escritas juntas.
 arco-íris;

Derivação parassintética  beija-flor;

Na derivação parassintética acrescenta-se  guarda-chuva;


simultaneamente um sufixo e um prefixo a uma palavra já  segunda-feira;
existente  chapéu de chuva;
 espairecer (es- + pairar + -ecer)  fim de semana;
 esquentar (es- + quente + -ar)  girassol;
 entediar (en- + tédio + -ar)  paraquedas;
 desgelar (des- + gelo + -ar)  passatempo;
.  pontapé.

www.editoradince.com Acesse e veja se há novidades a respeito deste material


24 LÍNGUA PORTUGUESA - TEORIA E QUESTÕES
2. Indique em qual alternativa a palavra não é formada por
Abreviação composição por justaposição.
Na abreviação é apenas utilizada parte da palavra, a) quinta-feira;
em vez de ser utilizada a palavra na sua totalidade. Essa b) paraquedas;
parte de palavra passa a existir como uma palavra c) guarda-sol;
autônoma. Neste tipo de formação de palavras estão
d) tique-taque.
incluídas formas reduzidas das palavras, como vídeo (de
videocassete) e também as siglas, formadas pelas letras
iniciais de um nome composto por duas ou mais palavras. 3. Identifique a palavra formada pelo mesmo processo de
 foto (de fotografia); formação que a palavra contramão.
 moto (de motocicleta); a) supracitado;
 pneu (de pneumático); b) embora;
 ONU (Organização das Nações Unidas); c) espairecer;
 PUC (Pontifícia Universidade Católica); d) rapidamente.
 FAB (Força Aérea Brasileira).
Gabarito: 01/C; 02/D; 03/A
Reduplicação
Na reduplicação ocorre a repetição de vogais ou
consoantes na formação de uma palavra imitativa. Neste
tipo de formação de palavras também estão incluídas as
SINTAXE: TERMOS DA ORAÇÃO
palavras onomatopaicas. A reduplicação é também SINTAXE é o estudo da relação das palavras
chamada de duplicação silábica. dentro de um determinado contexto, isto é, a função que
 pingue-pongue cada palavra desempenha dentro de uma oração e como
cada uma delas atua na construção do sentido da frase.
 tique-taque
Esse estudo é chamado de análise sintética e se resume
 bombom em dividir uma frase em períodos para compreender
 zum-zum como cada palavra se relaciona com as demais. Antes de
 pipilar realizar essa análise, é importante saber o conceito de
cada um dos elementos.
Hibridismo Frase – pode ser formada por apenas uma
palavra, como por um conjunto delas. Considera-se frase
No hibridismo ocorre a junção de palavras simples toda expressão que consiga transmitir um sentido
ou radicais provenientes de línguas diferentes. completo:
 monóculo (grego mono + latim oculus) – Frase verbal: quando é formada por um verbo
 nonacosaedro (latim nona + grego cosa e edro) – Frase nominal: quando não tem nenhum verbo
 sociologia (latim socio + grego logia) Oração – uma oração é um enunciado linguístico
que necessariamente deve ser composto por um verbo,
Combinação ou ainda, uma locução verbal.
Na combinação ocorre a formação de uma nova Período – considera-se um período cada
palavras através da junção de partes de outras palavras. enunciado que seja formado por uma ou mais orações.
 aborrecente (aborrecer + adolescente) Pode ser classificado em:
 portunhol (português + espanhol) -Período simples: quando se tem apenas uma
oração
 showmício (show + comício)
-Período composto: quando se tem mais de uma
oração
Intensificação Uma oração é composta necessariamente por
Na intensificação ocorre a criação de uma nova um sujeito e predicado.
palavra através do alargamento do sufixo de uma palavra
existente. Ocorre maioritariamente na utilização do sufixo
verbal -izar. TIPOS DE SUJEITO
 obstaculizar (em vez de obstar) – Simples: quando se tem apenas um núcleo
 protocolizar (em vez de protocolar) – Composto: quando se tem dois ou mais núcleo
 culpabilizar (em vez de culpar) – Oculto: quando o sujeito não está explícito na
oração, mas pode ser identificado
– Inexistente: quando a ação verbal da frase não
EXERCÍCIOS está atribuída a nenhum ser. Nestes casos usam-se os
1. A palavra boteco é formada por derivação. verbos impessoais
a) prefixal; – Indeterminado: quando não se pode identificar
a quem se atribui a ação verbal
b) sufixal;
c) regressiva;
PREDICAÇÃO VERBAL
d) parassintética;
Predicação completa: quando aparece o verbo
e) imprópria.
sozinho, isto é, sem nenhum tipo de complemento. Nestes

www.editoradince.com Acesse e veja se há novidades a respeito deste material


LÍNGUA PORTUGUESA - TEORIA E QUESTÕES 25
casos, o verbo consegue transmitir todo o sentido do c) indeterminado;
enunciado. d) inexistente;
Predicação incompleta: quando o verbo exige um e) claro, simples e determinado.
complemento, pois sozinho não transmite o sentido, ou
seja, precisa de mais elementos para que se torne
coerente. 4.(FCC-TRE-CE-2015)Marque a oração em que o termo
destacado é sujeito: ///

a) houve muitas brigas no jogo;


ESTUDO DA RELAÇÃO ENTRE OS TERMOS DA b) Ia haver mortes, se a polícia não interviesse;
ORAÇÃO c) faz dois anos que há bons espetáculos;
Segundo uma análise sintática, a oração se d) existem muitas pessoas desonestas;
encontra dividida em:
e) há muitas pessoas desonestas.
termos essenciais;
termos integrantes;
5.(FCC-TRE-AL-2009) Indique a única frase que não
termos acessórios. tem verbo de ligação:
Os termos essenciais da oração são o sujeito e o a) o sol estava muito quente;
predicado.
b) nossa amizade continua firme;
Os termos integrantes da oração são o objeto
c) suas palavras pareciam sinceras;
direto, o objeto indireto, o predicativo do sujeito, o
predicativo do objeto, o complemento nominal e o agente d) ele andava triste;
da passiva. e) ele andava rapidamente.
Os termos acessórios da oração são o adjunto
adnominal, o adjunto adverbial e o aposto. 6.(FCC-TRE-MG-2016) Considere a frase:
Exemplos de análise sintática “Ele andava triste porque não encontrava a
Amanhã, a Madalena pagará suas dívidas ao companheira”, os verbos grifados são respectivamente : ///

banco. a) transitivo direto - de ligação;


Sujeito: a Madalena b) de ligação - intransitivo;
Predicado: pagará suas dívidas ao banco c) de ligação - transitivo - indireto;
Objeto direto: suas dívidas d) transitivo direto - transitivo indireto;
Objeto indireto: ao banco e) de ligação - transitivo direto.
Adjunto adverbial: amanhã
Adjunto adnominal: a, suas 7.(FCC-TRT-AP-2009) Na praça deserta um homem
O diretor está livre de compromissos. caminhava - o sujeito é:
Sujeito: o diretor a) indeterminado;
Predicado: está livre de compromissos b) inexistente;
c) simples;
QUESTÕES DE CONCURSOS d) oculto por elipse;
1.( FCC-TRT-CE-209) Na oração: “Foram chamados às e) composto.
pressas todos os vaqueiros da fazenda vizinha”, o
núcleo do sujeito é: 8.(FCC-TRE-BA-2013) Na oração: “Anunciaram
a) todos; grandes novidades” - o sujeito é:
b) fazenda; a) simples;
c) vizinha; b) composto;
d) vaqueiros; c) indeterminado;
e) pressas. d) elíptico;
e) inexistente.
2.(FCC-TRE-AL-2016) Assinale a alternativa em que o
sujeito está incorretamente classificado: 9.(FCC-TRT-AM-2008) “O toque dos sinos ao cair da
a) chegaram, de manhã, o mensageiro e o guia (sujeito noite era trazido lá da cidade pelo vento”. O termo
composto); grifado é:///

b) fala-se muito neste assunto (sujeito indeterminado); a) sujeito;


c) vai fazer frio à noite (sujeito inexistente); b) objeto direto;
d) haverá oportunidade para todos (sujeito inexistente); c) objeto indireto;
e) não existem flores no vaso (sujeito inexistente). d) complemento nominal;
e) agente da passiva.
3.(FCC-TRE-MG-2012) Em “Éramos três velhos
amigos, na praia quase deserta”, o sujeito desta 10.(FCC- TRE-MA-2009)“Eu andava satisfeito com o
oração é: mundo e comigo mesmo”, o período é:
a) subentendido; a) simples;
b) claro, composto e determinado; b) composto por coordenação;

www.editoradince.com Acesse e veja se há novidades a respeito deste material


26 LÍNGUA PORTUGUESA - TEORIA E QUESTÕES
c) composto por subordinação; d) oculto;
d) composto por coordenação e subordinação; e) oração sem sujeito.
e) composto de duas orações.
18.(FCC- TRE-MA-2005)Na expressão: “Ordem e
11. (FCC- TRT-GO-2009) Na oração “Mestre Reginaldo, progresso, esse é o nosso lema” – o sujeito é:
o impoluto, é uma sumidade no campo das a) simples;
ciências” - o termo grifado é: ///
b) composto;
a) adjunto adnominal; c) indeterminado;
b) vocativo; d) oculto;
c) predicativo; e) inexistente.
d) aposto;
e) sujeito simples. 19. (FCC-TRT-CE-2009) Já na expressão “O prefeito
Odorico nomeou Dirceu Borboleta ajudante de
12. (FCC- TRT-CE) Na expressão: “por todos era ordens” – as palavras grifadas funcionam como:
apedrejado o Luizinho”, o termo grifado é: /// a) objeto direto;
a) objeto direto; b) objeto indireto;
b) objeto indireto; c) predicativo do sujeito;
c) sujeito; d) aposto;
d) complemento nominal; e) predicativo do objeto
e) agente da passiva.
20. (FCC-TRE-BA-20012) O verbo de “confio este carro
13. (FCC-TRE-RS-2007)Dentre as orações abaixo, uma à distinção dos senhores passageiros” é:
contém complemento nominal. Qual? a) transitivo direto;
a) Meu pensamento é subordinado ao seu. b) transitivo indireto;
b) Você não deve faltar ao encontro. c) transitivo direto e indireto;
c) Irei à sua casa amanhã. d) intransitivo;
d) Venho da cidade às três horas. e) de ligação.
e) Voltaremos pela rua escura ...
Gabarito:
14. Assinale a alternativa em que o termo grifado é 1. D 11. D
adjunto adnominal: ///
2. E 12. E
a) Sua falta aos encontros sufocava o nosso amor. 3. A 13. A
b) Ela é uma fera maluca. 4. D 14. C
c) Ela é maluca por lambada nacional. 5. E 15. E
d) Não tenho medo da louca.
6. E 16. C
e) O amor de Deus é o primeiro mandamento.
7. C 17. B
15. Em “a linguagem do amor está nos olhos” – os 8. C 18. B
termos grifados são respectivamente: 9. E 19. E
a) complemento nominal e predicativo do sujeito; 10. A 20. C
b) adjunto adnominal e predicativo do sujeito;
c) adjunto adnominal e objeto direto;
d) complemento nominal e adjunto adverbial;
e) adjunto adnominal e adjunto adverbial. TEMPOS, MODOS E VOZES VERBAIS.
EMPREGO DE TEMPOS VERBAIS:
16 (FCC-TTE-BA-2015). “Diga ao povo que fico” é um Temos três tempos verbais:
período: Passado ou pretérito, presente e futuro.
a) simples; Pretérito ou passado: Aconteceu antes do
b) composto por coordenação; instante que se fala.
c) composto por subordinação; Ex.: Ontem fui à academia mais cedo.
d) composto por coordenação e subordinação; Presente: Acontece no instante da fala
e) composto de três orações. Ex.: Eu treino nesta academia
Futuro: Acontecerá depois do instante da fala.
17. (FCC- TRE-SC-2006) “Saúde e felicidade são as Ex.: Irei à academia daqui umas duas horas.
minhas aspirações na vida” – nessa expressão o
sujeito é:
O pretérito pode ser:
a) simples; Pretérito perfeito: O fato passado foi concluído
b) composto; totalmente
c) indeterminado; Ex.: Ele estudou toda a matéria hoje de manhã.

www.editoradince.com Acesse e veja se há novidades a respeito deste material


LÍNGUA PORTUGUESA - TEORIA E QUESTÕES 27
Pretérito imperfeito: O fato passado não foi QUESTÕES DE CONCURSOS
concluído totalmente.
1. (FCC- MPU - 2007) A frase que NÃO admite
Ex.: Ele conversava muito durante a aula. transposição para a voz passiva é:
Pretérito mais-que-perfeito: O fato passado é (A) Fiquei observando a construção caprichosa da teia da
anterior a outro fato também passado e terminado. aranha.
Ex.: Quando eu cheguei na festa, ele já tinha saído (B) Os vegetarianos não fiquem aliviados.
(C) Tudo isso compõe uma trama de vida e morte.
O futuro pode ser: (D) Eu teria reservado um melhor arremate para esta
Futuro do presente: O fato acontece após o crônica.
momento da fala, mas já terminado antes de outro fato (E) A natureza vai explicitando suas verdades o tempo
futuro. todo.
Ex.: Quando sua mãe chegar, eu contarei tudo
para ela.
2 (FCC-TRF -2007) Transpondo-se para a voz passiva
Futuro do pretérito: Um fato futuro que pode a frase transmiti o respeito de meus pais pelas
ocorrer depois de um fato passado. ficções, a forma verbal resultante será
Ex.: Se eu tivesse os livros, estudaria nas férias. (A) fora transmitido.
(B) transmitiram-se.
(C) foi transmitido.
EMPREGO DE MODOS VERBAIS: (D) terá sido transmitido.
Indicativo – mostra uma certeza. A pessoa que (E) transmitiram-me.
fala é precisa sobre o fato.
Ex.: Eu gosto de feijoada.
3 (FCC- TC – PB-2006) Ele não é emitido por
Subjuntivo – Mostra incerteza. A pessoa fala motores...
mostra dúvida sobre o fato
Transpondo-se a frase acima para a voz ativa, a forma
Ex.: Talvez eu viaje no final de semana. verbal correta passa a ser (A) emitia.
Imperativo – mostra uma atitude de ordem ou (B) emitem.
solicitação
(C) tinham emitido.
Ex.: Não jogue bola agora.
(D) serão emitidos.
(E) é para ser emitido.
FORMAS NOMINAIS DO VERBO:
O verbo pode ter funções de nomes (nominais),
4 (FCC-TRT 4 REGIÃO-2006) Transpondo-se para a
como substantivo, adjetivo e advérbio.
voz passiva a frase tudo o que me importou na
Infinitivo impessoal (não flexiona o verbo): dá vida já escrevi, ela ficará:
significado ao verbo de modo indefinido e vago. Ele deve
ser usado em locuções verbais, sem sujeito definido, com (A) Tudo o que me importou na vida já foi por mim
sentido imperativo etc.. (B) Tudo o que a vida me importou já fora escrito por mim.
Ex.: É preciso amar (C) A vida já me importou em tudo o que escrevi.
Infinitivo pessoal (flexiona o verbo): Ele deve (D) Já está escrito na vida tudo o que ela me importou.
ser usado com sujeito definido, quando desejar (E) Tudo o que me importou na vida já tenho escrito.
determinar o sujeito, quando o sujeito da segunda oração
for diferente e quando uma ação for correspondente.
5 (FCC- TRE – AP-2006)
1ª pessoa do singular: sem desinências
Texto I
2ª pessoa do singular: Radical + ES
O jivaro Um sr. Matter, que fez uma viagem de
3ª pessoa do singular: sem desinências exploração à América do Sul, conta a um jornal sua
1ª pessoa do plural: Radical + MOS conversa com um índio jivaro, desses que sabem
2ª pessoa do plural: Radical + DES reduzir a cabeça de um morto até ela ficar bem
pequenina. Queria assistir a uma dessas operações, e
3ª pessoa do plural: Radical + EM
o índio lhe disse que exatamente ele tinha contas a
Gerúndio: pode servir como adjetivo ou advérbio. acertar com um inimigo.
A ação está acontecendo no momento que se fala.
O Sr. Matter:
Ex.: eu estou falando com você
− Não, não! Um homem, não.
Na escola havia meninos vendendo picolés
Faça isso com a cabeça de um macaco.
(função de adjetivo)
E o índio:
Quando estava saindo de casa, vi um carro
branco. (função de advérbio). − Por que um macaco? Ele não me fez nenhum
Particípio: Resultado de uma ação que terminou, mal!
podendo flexionar em gênero número e grau. È usado na (Rubem Braga, Recado de primavera)
formação dos tempos compostos. Texto II
O João tem dormido cedo nas últimas semanas. Anedota búlgara Era uma vez um czar naturalista
que caçava homens.

www.editoradince.com Acesse e veja se há novidades a respeito deste material


28 LÍNGUA PORTUGUESA - TEORIA E QUESTÕES
Quando lhe disseram que também se caçam 10 (FCC-TRF 4ª Região-2007) ... onde são degradadas
borboletas [e andorinhas ficou muito espantado e por bactérias.
achou uma barbaridade. Transpondo-se a frase para a voz ativa, a forma verbal
(Carlos Drummond de Andrade, Alguma poesia) passa a ser, corretamente,
A transposição para a voz passiva de uma frase de (A) degradou.
um dos textos está correta em: (B) degradam.
(A) Uma dessas operações queria ser assistida pelo Sr. (C) seriam degradadas.
Matter.
(D) tinham degradado.
(B) Tinha contado a um jornal sua conversa com um índio
(E) está sendo degradada.
jivaro.
(C) A cabeça de um morto sabe reduzir-se até ficar bem
pequenina. GABARITO: 1.B; 2.C; 3.B; 4.A; 5.D; 6.B; 7.A; 8.B; 9.C ;
10.B
(D) Foi feita por ele uma viagem de exploração à América
do Sul.
(E) Nenhum mal ele lhe havia feito!
COLOCAÇÃO PRONOMINAL
6 (FCC- TRE PB - 2007) A construção que NÃO admite
transposição para a voz passiva é: A Colocação Pronominal respeita aos três tipos
de posição que os pronomes átonos me, te, o, a, lhe, nos,
(A) Os astrônomos antigos colocaram-na no centro do
vos, os, as, lhes podem ocupar na oração:
universo.
 Próclise - o pronome é colocado antes do verbo.
(B) A mensagem chegou com o título de “A Bela Azul”.
 Mesóclise - o pronome é colocado no meio do
(C) O coração coloca as razões do amor no centro do
verbo.
universo.
(D) Anunciam os cientistas a agonia de nossa Bela Azul.
 Ênclise - o pronome é colocado depois do verbo.
Embora existam regras, a colocação dos
(E) A presença da natureza por vezes nos desvia da pronomes está pendente de fatores como por exemplo, o
leitura de um livro. ritmo, a ênfase e o estilo.

7 –(FCC- PM - BA - 2007) Transpondo-se para a voz USO DA PRÓCLISE


ativa a frase As ações repressivas passam a ser 1. Orações negativas, que contenham palavras tais
legitimadas pelo referendo da população, a forma como não, ninguém, nunca.
verbal resultante será: Exemplos:
(A) passa a legitimar.  Não o quero aqui.
(B) passam a legitimar.  Nunca o vi assim.
(C) legitimam-se. 2. Pronomes relativos, indefinidos ou demonstrativos.
(D) têm passado a se legitimar. Exemplos:
(E) passam a legitimar-se.  Foi ela que o fez.
 Alguns lhes deram maus conselhos.
8 ( FCC-TRE MS-2007) . NÃO admite transposiçãopara  Isso me lembra algo.
a voz passiva a seguinte construção: 3. Verbos antecedidos por advérbios ou expressões
(A) A orientação do nosso ensino deveria contemplar adverbiais, exceto quando haja vírgula depois do
nossa fecundidade indisciplinada. advérbio, uma vez que dessa forma o advérbio deixa de
(B) Uma revolução na orientação do ensino brasileiro atrair o pronome.
depende de uma combinação de múltiplas iniciativas. Exemplos:
(C) A leitura responsável de um texto sempre considerará  Ontem me disseram que havia greve hoje.
a possibilidade de seus múltiplos sentidos.  Agora, descansa-se.
(D) A maioria dos professores considera tão somente uma 4. Orações exclamativas e orações que exprimam
solução única para cada problema. desejo de que algo aconteça.
(E) O método dialético estimula, acima de qualquer Exemplos:
certeza dogmática, a valorização das contradições.  Deus nos dê forças.
 Oxalá me dês a boa notícia.
9 ( FCC-TRF 4ª Região - 2007) A seguinte construção 5. Orações com conjunções subordinativas.
NÃO admite transposição para a voz passiva: Exemplos:
(A) Isso esclarece um pouco a razão das tensões (...)  Embora se sentisse melhor, saiu.
(B) (...) detestamos a hipocrisia e a falsidade (...)  Conforme lhe disse, hoje vou sair mais cedo.
(C) Vivemos, assim, sobre esse fio de navalha entre a 6. Verbo no gerúndio regido da preposição em.
verdade e o disfarce. Exemplos:
(D) As chamadas “regras de convívio” supõem, sempre,  Em se tratando de confusão, ele está presente.
algum “mascaramento”.  Em se decidindo pelo churrasco, eu trato da
(E) (...) que nos olhe nos olhos (...) carne.
7. Orações interrogativas.
Exemplos:

www.editoradince.com Acesse e veja se há novidades a respeito deste material


LÍNGUA PORTUGUESA - TEORIA E QUESTÕES 29
 Quando te deram a notícia?  Queimou-se quando estava preparando o jantar.
 Quem te presenteou?  Animou-se para ir à festa.
O pronome "se" também pode ter a função de
USO DA MESÓCLISE sujeito indeterminado.
A Mesóclise é possível apenas com verbos do Exemplos:
Futuro do Presente ou do Futuro do Pretérito. Se  Procura-se cãozinho.
houver palavra atrativa, todavia, dá-se preferência ao uso  Alugam-se casas.
da Próclise.
O pronome "se" pode, ainda, ter a função de
Exemplos: simplesmente realçar o discurso.
 Orgulhar-me-ei dos meus alunos. Exemplos:
 Orgulhar-me-ia dos meus alunos.  Lá se vai a minha chance de ganhar na loto.
 Foi-se embora sem dizer nada.
USO DA ÊNCLISE
Quando o uso da Próclise e da Mesóclise não for
possível, usa-se a Ênclise. A colocação de pronome EXERCÍCIOS
depois do verbo é atraída pelas seguintes situações: Texto
1. Verbo no imperativo afirmativo.
Qualquer barulho que se fizesse, principalmente os mais
Exemplos:
 Depois de terminar, chamem-nos. desagradáveis, ecoava pelo edifício inteiro, verdadeira
 Para começar, joguem-lhes a bola! caixa acústica. Mas nada de alguém assobiando ou
2. Verbo no infinitivo impessoal. cantando uma canção. Um amigo meu casou-se com a
Exemplos: mais bela do bairro porque a conquistou com O Vôo da
 Gostaria de pentear-te a minha maneira. Abelha, o grande desafio dos assobiadores.
 O seu maior sonho é casar-se. Certamente ouvíamos música no edifício, mas
3. Verbo inicia a oração. emitida por aparelhos de som, alguns muito possantes,
Exemplos:
não, porém, pela voz natural, direto da garganta,
 Fiz-lhe a pessoa mais feliz do mundo.
amadora, desafinada, catarrenta e sem acompanhamento
 Acordei e surpreendi-me com o café da manhã.
4. Verbo no gerúndio (sem a preposição em, pois de orquestra. A voz do CD não é a voz ao vivo, solta,
quando regido pela preposição em deve ser usada a gratuita, à qual nos referimos.
Próclise). – Não se incomode, dona Mariana – replicou ela.
Exemplos: – Acabo de assinar contrato com a Tupi, vou ganhar uma
 Vive a vida encantando-me com as suas nota preta.
surpresas.
01. Assinale a alternativa em que, mudando-se a
 Faço sempre bolos diferentes experimentando- colocação pronominal de próclise para ênclise e
lhes ingredientes novos.
vice-versa, a frase continua correta.
(A) que se fizesse
COM LOCUÇÃO VERBAL (B) um amigo meu casou-se
Nos exemplos acima existe apenas um verbo (C) porque a conquistou
atraindo o pronome. (D) à qual nos referimos
Agora, vejamos como ocorre a colocação do (E) não se incomode
pronome nas locuções verbais. Lembrando que as regras
citadas para os verbos na forma simples devem ser Texto
seguidas. Assim como no progresso alucinante de nossos
1. Usa-se a Ênclise depois do verbo auxiliar ou dias há um negativo inerente, também nas vivências
depois do verbo principal nas locuções verbais em que o anteriores e superadas havia um positivo injustamente
verbo principal está no infinitivo ou no gerúndio.
sepultado junto com o negativo que as caracterizou.
Exemplos:
 Devo explicar-te o que se passou
02. Como em “que as caracterizou”, a colocação
 Devo-lhe explicar o que se passou.
pronominal está correta em
2. Caso não haja palavra que atraia a Próclise,
usa-se a Ênclise depois do verbo auxiliar em que o verbo A) Entregarei-as na próxima semana.
principal está no particípio. B) Os seres humanos haviam conseguido-as.
Exemplos: C) Não se sabe quem as revelou.
 Foi-lhe explicado como deveria agir. D) Espero que examinem-se todos os detalhes.
Perguntaram-me uma vez se eu saberia calcular o
 Tinha-lhe feito as vontades se não tivesse sido
mal criado. Brasil daqui a vinte e cinco anos. Nem daqui a vinte e
cinco minutos, quanto mais vinte e cinco anos.
PRONOME SE E SUAS FUNÇÕES
O pronome "se" pode ter a função de objeto direto 03. Da mesma maneira que em “Perguntaram-me”, linha
ou, por vezes, de objeto indireto em orações em cuja voz 01, a colocação do pronome oblíquo átono está
verbal é reflexiva. correta em
Exemplos:

www.editoradince.com Acesse e veja se há novidades a respeito deste material


30 LÍNGUA PORTUGUESA - TEORIA E QUESTÕES
A) Os alunos leram muito, preparando-se para as Substantivos masculinos terminados em “ão”
avaliações. substitui por “ã”:
B) Se pudesse, o professor educaria-os. Ex.: o Anão por a anã ou o capitão por a capitã.
C) Os estudantes haviam preparado-se para as
avaliações. Substantivos masculinos terminados em “r”
D) Os estudantes que indagaram-me são interessados. acrescenta a letra “a”:
E, na maioria das vezes, quando se descrevem as Ex.: o cantor por a cantora
características físicas, morais e mentais de um
brasileiro, não se nota que na verdade se estão Pode acontecer em que substantivos masculinos
descrevendo os sintomas físicos, morais e mentais da terminados em “or” substitui por “eira”:
fome. Ex.: O arrumador por a arrumadeira.

04. O pronome “se” está posto antes do verbo, por estar Tem substantivo que terminado em “e” mudam
atraído por para “a” no feminino
A) outro pronome. Ex.: Elefante por elefanta
B) um advérbio.
C) uma conjunção coordenativa. Substantivo terminado com “ês”, “L” ou “z”
D) uma conjunção subordinativa. acrescenta o “a” no feminino.
Ex.: Freguês fica freguesa
05. Considere os itens seguintes:
I. Os agentes penitenciários haviam deslocado-se para o Conforme o sentido da frase pode ser feminino ou
presídio. masculino.
II. Há menas confusão na rua. Ex.: A capital (cidade); o capital (dinheiro)
III. Cada um dos agentes prestarão juramento.
IV. Todos os agentes devem assistir ao hasteamento da FLEXÃO NOMINAL DE NÚMEROS
bandeira. Os nomes (substantivos, adjetivos, pronomes,
Marque a alternativa verdadeira. numerais), de modo geral admitem a flexão de número:
Singular e plural.
A) Em I, está correta a colocação do pronome oblíquo
átono.
Plural dos substantivos simples
B) Em II, está correta a concordância nominal. Aos substantivos que terminam em vogal, ditongo
C) Em III, está correta a concordância verbal. oral e consoante ‘n’ devem ser acrescidos a
consoante ‘s’ ao final da palavra. Observe os exemplos:
D) Em IV, está correta a regência verbal.
herói – heróis
Aos substantivos que terminam em
Gabarito consoante ‘m’ devem ser acrescidos as consoantes
01. B 02. C 03. A 04. D 05.D ‘ns’ ao final da palavra. Observe os exemplos:
abordagem – abordagens
Aos substantivos que terminam com as
consoantes ‘r’ e ‘z’ devem ser acrescidos ‘es’ ao final da
FLEXÃO NOMINAL E VERBAL palavra. Observe os exemplos:
hambúrguer – hambúrgueres
FLEXÃO NOMINAL Nos substantivos que terminam
Flexão nominal é o estudo do gênero e número em ‘al’, ‘el’, ‘ol’, ‘ul’, deve ser substituída a
dos substantivos, adjetivos, numerais e pronomes. consoante ‘l’ por ‘is’. Observe os exemplos:
Essencialmente é o estudo do plural e gêneros dos girassol – girassóis
nomes. * Há duas exceções:
Ex.: Mala e malas ou cachorro e cachorra mal – males
cônsul – cônsules
FLEXÃO NOMINAL DE GÊNERO: Os substantivos que terminam em ‘il’ são
pluralizados de duas formas:
Os substantivos masculinos são precedidos pelo
a) Em palavras oxítonas terminadas em ‘il’:
artigo “o”.
juvenil – juvenis
Ex. O cachorro ou o piloto. b) Em palavras paroxítonas terminadas em ‘il’:
Os substantivos femininos são precedidos pelo inútil – inúteis
artigo “a”. Os substantivos terminados em
Ex.: A cachorra ou a pilota. consoante ‘s’ fazem o plural de duas formas:
a) Em substantivos monossilábicos ou oxítonos, há
Formação do feminino: o acréscimo de ‘es’.
Substantivos masculinos terminados em “o” algoz – algozes
substitui por “a”: b) Os substantivos paroxítonos ou proparoxítonos
são invariáveis.
Ex.: piloto por pilota
férias – férias

www.editoradince.com Acesse e veja se há novidades a respeito deste material


LÍNGUA PORTUGUESA - TEORIA E QUESTÕES 31
ônibus – ônibus QUESTÕES DE CONCURSOS
Os substantivos terminados em ‘ão’ podem ser
pluralizados de três formas: 1. (FCC-TRT-RJ-2012) Assinale o par de vocábulos
que formam o plural como órfão e mata-burro,
a) Substituindo o ‘ão’ por ‘es’:
respectivamente:
doação – doações
a) cristão / guarda-roupa
b) Substituindo o ‘ão’ por ‘ães’:
alemão – alemães b) questão / abaixo-assinado
c) Substituindo o ‘ão’ por ‘ãos’: c) alemão / beija-flor
cidadão – cidadãos d) tabelião / sexta-feira
d) Os substantivos terminados em e) cidadão / salário-família
consoante ‘x’ são invariáveis:
córtex – córtex
2. (FCC-TRE-ES-2012) Relativamente à concordância
dos adjetivos compostos indicativos de cor, uma,
FLEXÃO VERBAL dentre as seguintes, está errada. Qual?
Das classes de palavras a que tem mais flexões é a) saia amarelo-ouro
a do Verbo. b) papel amarelo-ouro
Os verbos sofrem flexão em modo, tempo, número c) caixa vermelho-sangue
e pessoa.
d) caixa vermelha-sangue
e) caixas vermelho-sangue
Modo:
Mostra em que contexto acontece o verbo.
3. (FCC-TRT-CE-2008) Indique a frase correta:
Temos:
a) Mariazinha e Rita são duas leva-e-trazes.
Modo indicativo – A pessoa que fala tem certeza
do que tá dizendo. b) Os filhos de Clotilde são dois espalhas-brasas.
Ex.: Eu vou assistir ao jogo hoje. c) O ladrão forçou a porta com dois pés-de-cabra.
Modo subjuntivo – A pessoa que fala tem dúvidas d) Godofredo almoçou duas couves-flor.
sobre o que tá dizendo. e) Alfredo e Radagásio são dois gentilhomens.
Ex.: Espero que você jogue bem hoje a noite
Modo imperativo – A pessoa fala uma ordem ou 4. (FCC-SEMEF-AM--2014) Flexão incorreta:
faz um pedido. a) os cidadãos
Ex.: Pare de dizer estas besteiras! b) os açúcares
c) os cônsules
Tempo: d) os tóraxes
São ações que podem ocorrer no passado e) os fósseis
(pretérito), presente ou futuro.
No passado temos o pretérito: Perfeito, imperfeito, 5. (FCC-SEFAZ-BA-2016) Mesma pronúncia de
mais que perfeito, perfeito composto do indicativo e mais "bolos":
que perfeito composto do indicativo, imperfeito do
a) tijolos
subjuntivo e o mais que perfeito composto do subjuntivo.
b) caroços
No presente temos o presente do indicativo e o
presente do subjuntivo. c) olhos
No futuro temos futuro: do presente do indicativo, d) fornos
do pretérito do indicativo, do presente composto do e) rostos
indicativo, do pretérito composto do indicativo, do
subjuntivo e o composto do subjuntivo. 6. (FCC-TRE-MA-2013) Não varia no plural:
a) tique-taque
Número:
b) guarda-comida
Singular ou plural
c) beija-flor
Eu preciso estudar (singular)
d) pára-lama
Nós precisamos estudar (plural)
e) cola-tudo

Pessoa:
7. (FCC-TRT-ES-2012) Está mal flexionado o adjetivo
1ª pessoa: Eu e nós. Seria a pessoa que está na alternativa:
falando
a) Tecidos verde-olivas
2ª pessoa: Tu e vós. Seria a pessoa com quem se
b) Festas cívico-religiosas
está falando
c) Guardas noturnos luso-brasileiros
3ª pessoa: ele e eles. Seria a pessoa de quem
estão falando. d) Ternos azul-marinho
e) Vários porta-estandartes

8. (FCC-TRT-BA-2012) Na sentença "Há frases que


contêm mais beleza do que verdade", temos grau:

www.editoradince.com Acesse e veja se há novidades a respeito deste material


32 LÍNGUA PORTUGUESA - TEORIA E QUESTÕES
a) comparativo de superioridade
b) superlativo absoluto sintético Concordância com: a maioria, a maior parte, a
c) comparativo de igualdade metade,...
d) superlativo relativo Preferencialmente, o verbo estabelece
concordância com a 3.ª pessoa do singular. Contudo, o
e) superlativo por meio de acréscimo de sufixo
uso da 3.ª pessoa do plural é igualmente aceitável: a
maioria das pessoas quer, a maioria das pessoas querem.
9. (FCC-DETRAN-20120) Assinale a alternativa em que
a flexão do substantivo composto está errada:
Concordância com um dos que
a) os pés-de-chumbo d) os cavalos-vapor
O verbo estabelece sempre concordância com a
b) os corre-corre e) os vaivéns 3.ª pessoa do plural: um dos que ouviram, um dos que
c) as públicas-formas estudarão, um dos que sabem.

10. (FCC-DETRAN-2009) Aponte a alternativa em que Concordância com nem um nem outro
haja erro quanto à flexão do nome composto: O verbo pode estabelecer concordância com a
a) vice-presidentes, amores-perfeitos, os bota-fora 3.ª pessoa do singular ou do plural: nem um nem outro
b) tico-ticos, salários-família, obras-primas veio, nem um nem outro vieram.
c) reco-recos, sextas-feiras, sempre-vivas
d) pseudo-esferas, chefes-de-seção, pães-de-ló Concordância com verbos impessoais
e) pisca-piscas, cartões-postais, mulas-sem-cabeça O verbo estabelece sempre concordância com a
3.ª pessoa do singular, uma vez que não possui um
sujeito: havia pessoas, houve problemas, faz dois dias, já
GABARITO: 1. A; 2. D; 3. C; 4. D; 5. E; 6. E; 7. A; 8. A; 9. amanheceu.
B; 10. E
Concordância com a partícula apassivadora se
O verbo estabelece concordância com o objeto
direto, que assume a função de sujeito paciente, podendo
CONCORDÂNCIA NOMINAL E VERBAL ficar no singular ou no plural: vende-se casa, vendem-se
casas.
CONCORDÂNCIA NOMINAL E VERBAL
Concordância verbal é a concordância em Concordância com a partícula de
número e pessoa entre o sujeito gramatical e o verbo. indeterminação do sujeito se
Concordância nominal é a concordância em O verbo estabelece sempre concordância com a
gênero e número entre os diversos nomes da oração, 3.ª pessoa do singular quando a frase é formada por
ocorrendo principalmente entre o artigo, o substantivo e o verbos intransitivos ou por verbos transitivos indiretos:
adjetivo. precisa-se de funcionário, precisa-se de funcionários.
Concordância em gênero indica a flexão em
masculino e feminino.
Concordância com o infinitivo pessoal
Concordância em número indica a flexão em
O verbo no infinitivo sofre flexão sempre que
singular e plural.
houver um sujeito definido, quando se quiser definir o
Concordância em pessoa indica a flexão em 1.ª, sujeito, quando o sujeito da segunda oração for diferente
2.ª ou 3.ª pessoa. do da primeira: é para eles lerem, acho necessário
Exemplos de concordância verbal comprarmos comida, eu vi eles chegarem tarde.
Eu li;
Ele leu; Concordância com o infinitivo impessoal
Nós lemos; O verbo no infinitivo não sofre flexão quando não
Eles leram. houver um sujeito definido, quando o sujeito da segunda
Exemplos de concordância nominal oração for igual ao da primeira oração, em locuções
verbais, com verbos preposicionados e com verbos
O vizinho novo; imperativos: eles querem comprar, passamos para ver
A vizinha nova; você, eles estão a ouvir.
Os vizinhos novos;
As vizinhas novas. Concordância com o verbo ser
O verbo estabelece concordância com o
CASOS EXCEPCIONAIS DE CONCORDÂNCIA predicativo do sujeito, podendo ficar no singular ou no
VERBAL plural: isto é uma mentira, isto são mentiras; quem é você,
Concordância com pronome relativo que quem são vocês.
O verbo estabelece concordância com o
antecedente do pronome: sou eu que quero, somos nós CASOS PARTICULARES DE CONCORDÂNCIA
que queremos, são eles que querem. NOMINAL
Concordância com pronome relativo quem Concordância com pronomes pessoais
O verbo estabelece concordância com o O adjetivo estabelece concordância em gênero e
antecedente do pronome ou fica na 3.ª pessoa do número com o pronome pessoal: ela é simpática, ele é
singular: sou eu quem quero, sou eu quem quer. simpático, elas são simpáticas, eles são simpáticos.

www.editoradince.com Acesse e veja se há novidades a respeito deste material


LÍNGUA PORTUGUESA - TEORIA E QUESTÕES 33
d) Já não se pode lançar mão de razões racistas para se
Concordância com vários substantivos camuflar um interesse econômico ou político.
O adjetivo estabelece concordância em gênero e e) Não deixarão de haver, infelizmente, novas atrocidades
número com o substantivo que está mais próximo: coletivas, em nome de novos pretextos.
caderno e caneta nova, caneta e caderno novo. Pode
também estabelecer concordância com a forma no 2 (FCC-TRE-GO-2015). A alternativa cuja lacuna pode
masculino plural: caneta e caderno novos, caderno e ser preenchida por qualquer das duas formas
caneta novos. verbais indicadas entre parênteses é:
a) Um dos seus ideais ____________ morrer pela pátria.
Concordância com vários adjetivos (era/eram);
Quando há dois ou mais adjetivos no singular, o b) Somos nós que _____________ morrer ouvindo o
substantivo permanece no singular apenas se houver um canto do sabiá. (sonho/ sonhamos);
artigo entre os adjetivos. Sem a presença de um artigo, o c) Um bando de aves ________________ nas palmeiras.
substantivo deverá ser escrito no plural: o escritor (cantava/cantavam);
brasileiro e o chileno, os escritores brasileiro e chileno.
d) Já não ________________ mais os sítios onde eu vivi.
(existe/existem);
Concordância com: é proibido, é permitido, é e) Em minha terra não ________________ belezas
preciso, é necessário, é bom naturais. (falta/faltam).
Estas expressões estabelecem concordância em
gênero e número com o substantivo quando há um artigo
3.(FCC- SEFAZ-BA-2016) Assinale a alternativa desse
que determina o substantivo, mas permanecem
exercício que preenche, corretamente, as lacunas
invariáveis no masculino singular quando não há artigo: é
do texto.
permitida a entrada, é permitido entrada, é proibida a
venda, é proibido venda. A Polícia Civil apreendeu 415,4 quilos de crack
__________ em uma casa na Avenida Salim Farah
Maluf. No local, também __________ dois quilos de
Concordância com: bastante, muito, pouco, maconha. Um homem de 28 anos e um
meio, longe, caro e barato adolescente de 17 __________ .
Estas palavras estabelecem concordância em a) escondidos … havia … foram detidos
gênero e número com o substantivo quando possuem
b) escondido … havia … foram detido
função de adjetivo: comi meio chocolate, comi meia maçã,
há bastante procura, há bastantes pedidos, vi muitas c) escondidos … haviam … foi detido
crianças, vi muitos adultos. d) escondido … haviam … foram detidos
e) escondidos … havia … foram detido
Concordância com menos
A palavra menos permanece sempre invariável, 4.(FCC-TRT-2026) Assinale a alternativa da questão
quer atue como advérbio ou como adjetivo: menos em que a concordância está corretamente
tristeza, menos medo, menos traições, menos pedidos. estabelecida:
a) Foi três horas de viagem para chegar ao local do
Concordância com: mesmo, próprio, anexo, evento.
obrigado, quite, incluso b) Há de existir prováveis discussões para a finalização
Estas palavras estabelecem concordância em do projeto.
gênero e número com o substantivo: resultados anexos, c) Só foi recebido pelo coordenador quando deu cinco
informações anexas, as próprias pessoas, o próprio horas no relógio.
síndico, ele mesmo, elas mesmas. d) Fazia dias que participavam do processo seletivo em
questão.
Concordância com um e outro e) Choveu aplausos ao término da palestra do
Com a expressão um e outro, o adjetivo deverá ser especialista em Gestão.
sempre escrito no plural, mesmo que o substantivo esteja
no singular: um e outro aluno estudiosos, uma e outra 5 (FCC- TRE-BA-2008). Assinale a alternativa do
pergunta respondidas. exercício em que a concordância verbal está
correta.
QUESTÕES DE CONCURSOS a) Haviam cooperativas de catadores na cidade de São
Paulo.
1 (FCC-TRF-2012)encontram-se plenamente
b) O lixo de casas e condomínios vão para aterros.
respeitadas na frase:
c) O tratamento e a destinação corretos do lixo evitaria
a) Desenvolveu-se ao longo do século XX vários
que 35% deles fosse despejado em aterros.
conceitos científicos, inclusive o de etnia.
d) Fazem dois anos que a prefeitura adia a questão do
b) Deve-se à antropologia moderna alguns avanços
lixo.
históricos, sobretudo no que diz respeito ao conceito
de raça. e) Somos nós quem paga a conta pelo descaso com a
coleta de lixo.
c) Constam entre as mais cruéis manifestações de
barbárie a promoção dos terrores raciais, levada a
cabo pelos nazistas. 6 (FCC-TRT-2018) Encontram-se respeitadas as
normas de concordância em:

www.editoradince.com Acesse e veja se há novidades a respeito deste material


34 LÍNGUA PORTUGUESA - TEORIA E QUESTÕES
A. Cada um dos que se deparam com os problemas B. Durante séculos exercem influência sobre os artistas
produzidos pelas novas tecnologias acabam ocidentais o conceito de beleza que vigorava na antiga
desenvolvendo problemas de convivência. Grécia.
B. Caminhar a esmo é visto por muitos intelectuais como C. Sabem-se que os padrões de beleza para os gregos
um modo de protestar contra a lógica dominante, uma estão atrelados a uma visão específica de equilíbrio.
vez que se distancia de fins utilitaristas. D. Tanto Apolo quanto Dioniso são divindades cuja
C. O excesso de ruídos nas grandes cidades terminam simbologia, especialmente no plano das artes, são
por prejudicar a capacidade de concentração das riquíssimas.
pessoas. E. Criar ou apreciar obras de arte requerem a ativação de
D. Consideram-se o silêncio uma das formas de protesto valores relativos à beleza, os quais são complexos e
atuais, uma vez que se projeta contra os hábitos de variáveis.
consumo da sociedade.
E. Voltar-se para si e para o silêncio que nos permite 10 (FCC-TRT-2017)Quanto à concordância padrão,
contemplar nossas próprias vidas constituem uma está escrita corretamente a frase:
necessidade do mundo contemporâneo.
A. O homem sempre buscou capturar o instante em
imagens, e isso nunca foi tão fácil quanto hoje,
7(FCC- Prefeitura de Macapá-2018) As normas de quando o ato de registrar se tornou mais importante
concordância foram plenamente respeitadas na que o próprio registro.
seguinte frase: B. Atualmente, constata-se muitas maneiras de
A. Couberam às pesquisas realizadas na década de 1970 compartilhar informação, mas nenhum meio de
indagar se haveria um gene responsável por controlar comunicação vem se mostrando tão poderoso quanto
o ritmo circadiano. as redes sociais.
B. Esclareceu-se, em descobertas recentes, os C. Em meados da década passada, fotografar alimentos
mecanismos biológicos que fazem com que o corpo se envolviam uma série de questionamentos que
adapte às diferentes fases do dia, geradas pelas parecem não fazer mais sentido na sociedade dos
rotações da Terra. dias de hoje.
C. Já se aceita, por meio do estudo do ritmo circadiano, D. Em 2016, uma pesquisa com usuários da internet
aprofundado por diversas pesquisas científicas, concluiu que algumas pessoas que postam
noções como a de que o estado de maior alerta para a excessivamente nas redes sociais o faz por
maioria das pessoas está entre 6h e 12h. necessidade de aprovação.
D. Pesquisas já demonstraram que o surgimento de E. Decidir entre devorar ou clicar têm perturbado aqueles
diversas doenças, como a depressão, pode estar que oscilam entre desfrutar o momento da refeição e
atrelado a disfunções no sistema que regula o relógio partilhá-lo, ainda que a distância, com amigos e
biológico. familiares.
E. Na década de 1980, os vencedores do prêmio Nobel
de 2017 identificaram um gene e uma proteína Gabarito: 1.D; 2.C; 3.B; 4.D; 5.E; 6.B; 7.D; 8.C; 9.A; 10.A
produzida por ele, à qual chamou de "PER", cujos
níveis oscilavam nas 24 horas do dia.

8 (FCC-DPE-AM-2018)As regras da concordância REGÊNCIA NOMINAL E VERBAL


padrão estão plenamente respeitadas na frase:
REGÊNCIA VERBAL E NOMINAL
A. Os africanos livres eram responsáveis pela fabricação
de telhas, potes, tijolos, enfim, tudo que eram A regência é o campo da língua portuguesa que
produzidos na olaria. estuda as relações de concordância entre os verbos (ou
nomes) e os termos que completam seu sentido. Ou seja,
B. De origem cabinda, Apolinária tinha 24 anos quando
estuda a relação de subordinação que ocorre entre um
chegou ao Brasil, acompanhado de outros africanos
verbo (ou um nome) e seus complementos.
livres.
A regência é necessário visto que algumas
C. A autora se interessou pela vida de africanos livres no
palavras da língua portuguesa (verbo ou nome) não
Brasil, como Apolinária, que chegou a Manaus em
possuem seu sentido completo.
1855.
Observe o exemplo abaixo:
D. O registro escrito da vida de muitos desses
trabalhadores se perderam, mas a contribuição deles -Muitas crianças têm medo. (medo de quê?)
para a história do Brasil é indelével. -Muitas crianças têm medo de fantasmas.
E. Ainda que reste muitas zonas de silêncio, já se percebe Perceba que o nome pede complemento
esforços no sentido de evidenciar a importância antecedido de preposição (“de” = preposição e
dessas pessoas. “fantasmas” = complemento).
IMPORTANTE: A regência estabelece uma
9 (FCC-SEFAZ-GO-2018)A frase em que a relação entre um termo principal (termo regente) e
concordância está em conformidade com a norma- o termo que lhe serve de complemento (termo
padrão da língua é: regido) e possui dois tipos: REGÊNCIA NOMINAL
A. Apolo, assim como a divindade antípoda, Dioniso, está e REGÊNCIA VERBAL.
nos fundamentos da representação de beleza para os
gregos. 1. REGÊNCIA NOMINAL
Regência nominal é quando um nome
(substantivo, adjetivo) regente determina para o nome

www.editoradince.com Acesse e veja se há novidades a respeito deste material


LÍNGUA PORTUGUESA - TEORIA E QUESTÕES 35
regido a necessidade do uso de uma preposição, ou seja, incessante, inconstante, indeciso, infatigável, lento,
o vínculo entre o nome regente e o seu termo regido se morador, negligente, perito, pertinaz, prático, residente,
estabelece por meio de uma preposição. sábio, sito, versado...
A relação entre um nome regente e seu termo
regido se estabelece sempre por meio de uma Nomes que exigem a preposição "contra":
preposição. Atentado, Blasfêmia, combate, conspiração,
Exemplo: declaração, luta, fúria, impotência, litígio, protesto,
- Os trabalhadores ficaram satisfeito com o acordo, reclamação, representação...
que foi favorável a eles.
Veja: "satisfeito" é o termo regente e "com o Nomes que exigem a preposição "para":
acordo" é o termo regido, "favorável" é o termo regente e Mau, próprio, odioso, útil...
"a eles" é o termo regido.
OBS.: Quando um pronome relativo (que, qual, 2. REGÊNCIA VERBAL
cujo etc.) é regido por um nome, deve-se
Dizemos que regência verbal é a maneira como o
introduzir, antes do relativo, a preposição que o
verbo (termo regente) se relaciona com seus
nome exige.
complementos (termo regido).
Exemplo:
OBS.: Nas relações de regência verbal, o vínculo
- A proposta a que éramos favoráveis não foi entre o verbo e seu termo regido (complemento
discutida na reunião. (quem é favorável, é favorável a verbal) pode ser dar com ou sem a presença de
alguma coisa/alguém) preposição.
Exemplo:
PRINCIPAIS CASOS - Nós assistimos ao último jogo da Copa.
Nomes que exigem o uso da preposição “a”: Veja: "assistimos" é o termo regente, "ao" é a
Acessível, acostumado, adaptado, adequado, preposição e "último jogo" é o termo regido.
afeição, agradável, alheio, alusão, análogo, anterior, apto, Basicamente precisamos saber que:
atento, atenção, avesso, benéfico, benefício, caro,
compreensível, comum, contíguo, contrário, OBS.: Um verbo pode ter sentido completo, sem
desacostumado desagradável, desatento, desfavorável, necessitar de complementos. São os verbos
desrespeito, devoto, equivalente, estranho, favorável, fiel, intransitivos.
grato, habituado, hostil, horror, idêntico, imune, Há verbos que não possuem sentido completo,
inacessível, indiferente, inerente, inferior, insensível, necessitam de complemento. São os verbos transitivos.
Junto, leal, necessário, nocivo, obediente, odioso, ódio, Exemplos:
ojeriza, oneroso, paralelo, peculiar, pernicioso, - Transitivo direto: quando seu sentido se
perpendicular, posterior, preferível, preferência, completa com o uso de um objeto direto (complemento
prejudicial, prestes, propenso, propício, proveitoso, sem preposição).
próximo, rebelde, rente, respeito, semelhante, sensível,
simpático, superior, traidor, último, útil, visível, vizinho… Exemplo: A avó carinhosa agrada a netinha.
"Agrada" é verbo transitivo direto e "a netinha" e o
objeto direto.
Nomes que exigem o uso da preposição “de”:
- Transitivo indireto: quando seu sentido se
Abrigado, amante, amigo ávido, capaz, certo, completa com o uso de um objeto indireto (complemento
cheio, cheiro, comum, contemporâneo, convicto, com preposição).
cúmplice, descendente, desejoso, despojado, destituído,
Exemplo: Ninguém confia em estranhos.
devoto, diferente, difícil, doente, dotado, duro, êmulo,
escasso, fácil, feliz, fértil, forte, fraco, imbuído, impossível, "Confia" é verbo transitivo indireto, "em" é a
incapaz, indigno, inimigo, inocente, inseparável, isento, preposição e "estranhos" é o objeto indireto.
junto, livre, longe, louco, maior, medo, menor, natural, - Transitivo direto e indireto: quando seu sentido
orgulhoso, passível, piedade, possível, prodígio, próprio, e completa com os dois objetos (direto e indireto).
querido, rico, seguro, sujo, suspeito, temeroso, vazio... Exemplo: Devolvi o livro ao vendedor. "Devolvi" é
verbo transitivo direto e indireto, "o livro" é objeto direto e
Nomes que exigem a preposição “sobre”: "vendedor" é objeto indireto.
Opinião, discurso, discussão, dúvida, insistência,
influência, informação, preponderante, proeminência, REGÊNCIA DE ALGUNS VERBOS
triunfo, Para estudarmos a regência dos verbos, devemos
dividi-los em dois grupos:
Nomes que exigem a preposição “com”: 1. O primeiro, dos verbos que apresentam uma
Acostumado, afável, amoroso, analogia, determinada regência na variedade padrão e
aparentado, compatível, cuidadoso, descontente, outra regência na variedade coloquial;
generoso, impaciente, impaciência, incompatível, ingrato, 2. E o segundo dos verbos que, na variedade
intolerante, mal, misericordioso, obsequioso, ocupado, padrão, apresentam mais de uma regência.
parecido, relacionado, satisfeito, severo, solícito, triste...
PRIMEIRO GRUPO - Verbos que apresentam uma
Nomes que exigem a preposição "em": regência na variedade padrão e outra na variedade
Abundante, atento, bacharel, constante, doutor, coloquial:
entendido, erudito, fecundo, firme, hábil, incansável, VERBO ASSISTIR

www.editoradince.com Acesse e veja se há novidades a respeito deste material


36 LÍNGUA PORTUGUESA - TEORIA E QUESTÕES
- SENTIDO: “Auxiliar”, “caber, pertencer” e “ver, (iniciado pela preposição a. Preferir (alguma coisa) a
presenciar, atuar como expectador”. É nesse último (outra)
sentido que ele é usado. - VARIEDADE COLOQUIAL (Exemplos): Os
- VARIEDADE PADRÃO (Exemplos): Ele não brasileiros preferem mais o futebol que o vôlei; Você
assiste a filme de violência; Pela TV, assistimos à preferiu (mais) trabalhar que estudar; Prefiro (muito mais)
premiação dos atletas olímpicos. Assistir com significado silêncio do que a agitação da cidade. É empregado com
de ver, presenciar: É verbo transitivo indireto (VTI), expressões comparativas (“mais... que”, “muito mais
apresenta objeto indireto iniciado pela preposição a. ...que”, “do que” etc.). Preferir (mais) (uma coisa) do que
Quem assiste, assiste a (alguma coisa). (outra).
- VARIEDADE COLOQUIAL (Exemplos): Ela não
assiste filmes de violência. Assistir com significado de ver, VERBO VISAR
presenciar: É verbo transitivo direto (VTD); apresenta
- SENTIDO: O emprego mais usual do verbo
objeto direto. Assistir (alguma coisa)
“visar” é no sentido de “objetivar, ter como meta”.
- VARIEDADE PADRÃO (Exemplos): Todo artista
VERBO IR e CHEGAR visa ao sucesso; Suas pesquisas visavam à criação de
- VARIEDADE PADRÃO (Exemplos): No domingo, novos remédios. É VTI, com preposição a iniciando o
nós iremos a uma festa; O prefeito foi à capital falar com o objeto indireto. Visar a (alguma coisa)
governador; Os funcionários chegam bem cedo ao - VARIEDADE COLOQUIAL (Exemplos): Todo
escritório. Apresentam a preposição a iniciando o adjunto artista visa o sucesso; Suas pesquisas visavam a criação
adverbial de lugar: Ir a (algum lugar), Chegar a (algum de novos remédios. É VTD, apresenta objeto sem
lugar) preposição (objeto direto). Visar (alguma coisa)
- VARIEDADE COLOQUIAL (Exemplos): No
domingo, nós iremos em uma festa; Os funcionários
SEGUNDO GRUPO - VERBOS QUE, NA VARIEDADE
chegam bem cedo no escritório. Apresentam a preposição
PADRÃO, APRESENTAM MAIS DE UMA REGÊNCIA
em iniciando o adjunto adverbial de lugar: Ir em (algum
(DEPENDENDO DO SENTIDO/SIGNIFICADO EM QUE
lugar), Chegar em (algum lugar)
SÃO EMPREGADOS):
VERBO ASPIRAR
VERBO OBEDECER/DESOBEDECER
- TRANSITIVIDADE (Sentido): Verbo transitivo
- VARIEDADE PADRÃO (Exemplos): A maioria direto (sugar/respirar) Verbo transitivo indireto (pretender)
dos sócios do clube obedecem ao regulamento; Quem
- EXEMPLOS: Sentiu fortes dores quando aspirou
desobedece às leis de trânsito deve ser punido. São VTI;
o gás. O Ex-governador aspirava ao cargo de presidente.
exigem objeto indireto iniciado pela preposição a.
Obedecer a (alguém/alguma coisa), Desobedecer a
(alguém/alguma coisa) VERBO ASSISTIR
- VARIEDADE COLOQUIAL (Exemplos): A maioria - TRANSITIVIDADE (Sentido): Verbo transitivo
dos sócios do clube obedecem o regulamento; Quem direto (ajudar); Verbo transitivo indireto (ver); Verbo
desobedece as leis de trânsito deve ser punido. São transitivo indireto (pertencer)
transitivos direto (VTD); apresentam objeto sem - EXEMPLOS: Rapidamente os paramédicos
preposição inicial. Obedecer (alguém/alguma coisa), assistiram os feridos. Você assistiu ao filme? O direito de
Desobedecer (alguém/alguma coisa) votar assisti a todo cidadão.

VERBO PAGAR e PERDOAR VERBO INFORMAR


- SENTIDO: Obs.: Se o objeto for coisa (e não - TRANSITIVIDADE (Sentido): Verbo transitivo
pessoa), ambos são transitivos direto, tanto na variedade direto e indireto (passar informação)
padrão, como na coloquial. Exemplo: Você não pagou o - EXEMPLOS: Algumas rádios informam as
aluguel. O verbo pagar também é empregado com condições das estradas aos motoristas. Algumas rádios
transitivo direto e indireto. (Pagar alguma coisa para informam os motoristas das condições das estradas
alguém) A empresa pagava excelentes salários a seus
funcionários.
- VARIEDADE PADRÃO (Exemplos): A empresa VERBO QUERER
não paga aos funcionários faz dois meses; É ato de - TRANSITIVIDADE (Sentido): Verbo transitivo
nobreza perdoar a um amigo. São VTI quando o objeto é direto (desejar); Verbo transitivo indireto (amar/gostar)
gente; exigem preposição a iniciando o objeto indireto. - EXEMPLOS: Todos queremos um Brasil menos
Pagar a (alguém), Perdoar a (alguém) desigual. Isabela queria muito aos avós.
- VARIEDADE COLOQUIAL (Exemplos): A
empresa não paga os funcionários faz dois meses; É um VERBO VISAR
ato de nobreza perdoar um amigo. São VTD, apresentam
- TRANSITIVIDADE (Sentido): Verbo transitivo
objeto sem preposição (objeto direto): Pagar (alguém),
direto (mirar); Verbo transitivo direto (pôr visto); Verbo
Perdoar (alguém)
transitivo indireto (objetivar)

VERBO PREFERIR
- VARIEDADE PADRÃO (Exemplos): Os
QUESTÕES DE CONCURSOS
brasileiros preferem futebol ao vôlei; Você preferiu 01 (FCC – TRT-2012) A frase que, segundo os
trabalhar a estudar. Prefiro silêncio à agitação da cidade. preceitos da gramática normativa do português do
É VTDI; exige dois objetos: um direto outro indireto Brasil, está correta quanto à regência é:

www.editoradince.com Acesse e veja se há novidades a respeito deste material


LÍNGUA PORTUGUESA - TEORIA E QUESTÕES 37
a) A cada pequena discussão, costumava lhe chamar de b) Como reagir à recepção de textos aos quais jamais
aventureiro e até como irresponsável, e disso já se houve solicitação nossa?
havia coletado muitas provas. c) A autora refere-se ao deus Janus Bifronte, às duas
b) Nada daquela maluca versão interessava a ele, faces suas em cujas representavam-se o passado e o
principal testemunha do caso, e por isso manifestou- futuro.
se quanto à imediata retirada do indesejável d) Quem matou o hábito das cartas foi o telefone, em que
depoimento. o reinado começou junto com o século XX.
c) A afinidade entre os colegas intensificava-se ao mesmo e) Os e-mails acabam chegando a destinatários de cuja
tempo que seus estudos se desenvolviam, e disso privacidade não costumam respeitar.
surgiu uma amizade que todos tinham orgulho.
d) Sua obra é daquelas que se pode dizer tudo, menos
06 (FCC- TRE-MA- 2012) … e favoreça os seus amores
que passará despercebida a futuras gerações, seja
por ela… ///

para negar-lhe méritos, seja para reconhecê-los.


O verbo que exige o mesmo tipo de complemento que
e) Aquele professor é a verdadeira razão de que muitos
o grifado acima está empregado em:
estudantes decidiram dedicar-se à pesquisa, o que lhe
faz ser constantemente mencionado como exemplo a a) A jovem é irmã de Hersé…
ser seguido. b) … este espetáculo a corrói…
c) … Palas Atena vai à morada da Inveja…
02 (FCC-SEMEF-MA- 2017) Cronistas de reinos d) … e ordena-lhe que…
passados, gênios das navegações […] não falam e) Assiste com despeito aos sucessos dos homens…
de discos, pratos ou charutos voadores … ///

O verbo que NÃO foi empregado com o mesmo tipo 07 (FCC-CESPE-2018) Julgo o item (C ou E)
de complemento que o verbo grifado acima está
em: Segundo o Ministério da Justiça, a partir de 2011,
outros estados devem integrar-se gradativamente
b) Enjoaram de nós? ao sistema. A previsão é que, em nove anos, todos
c) Venceu a hipótese de naves … os brasileiros estejam cadastrados em uma base
d) Começou com um piloto norte-americano de caças … de dados unificada na Polícia Federal.
e) … que simplesmente desistimos deles? Pergunta:
O emprego da preposição a na combinação “ao” (em
03 (FCC-TRE-BA-2012) … Amapá, Roraima, Pará e negrito e sublinhado) é exigência sintática do verbo
Amazonas possuem mais da metade de seu “integrar”.
território nessa categoria.
O verbo grifado acima tem o mesmo tipo de 08) (FCC-SEMEF-MA- 2019) … salvo durante os
complemento que o verbo empregado em: intervalos de torpor a que aludi. ///

a) Terras indígenas e unidades de conservação O verbo que exige o mesmo tipo de complemento que
contribuem de modo quase parelho para o número o grifado acima está em:
total de áreas protegidas… a) … começarão por Sherlock Holmes.
b) … cujo nome já diz tudo… b) … mas que fosse marcadamente diferente das
c) … esse número no mesmo período foi de 2,1%. narrativas …
d) Quase metade da Amazônia brasileira pertence hoje à c) … no entanto ele possuía enorme delicadeza de toque
categoria de área protegida por lei contra a …
devastação … d) … qualidades que espelham as de seu criador.
e) É pouco mais do que um quarto de todas as terras do e) … um novo caso produza em Holmes uma onda de
Brasil. entusiasmo …

04 (FCC-SEFAZ-BA- 2017) … o recurso à coerção 9(FCC-DETRAN-SP-2019)) A moça ____ feições me


atenta contra os princípios do direito internacional lembro, encontrava-se à porta ____ me referi.
…///

a) cujas – à qual
O verbo que exige o mesmo tipo de complemento que b) de cujas – a que
o grifado acima está empregado em:
c) de cujas – a qual
a) Se nossos objetivos maiores incluem a decidida defesa
dos direitos humanos … d) cujas – a que
b) … o Brasil incorpora plenamente esses valores a sua e) de cujas – à que
ação externa …
c) A ONU constitui o foro privilegiado para … 10 (FCC- TRE-GO-2016) Assinale a opção que
d) Em meados da década de 90 surgiram vozes que … completa corretamente as lacunas das frases:
e) … a relação […] passou por várias etapas. O lugar ____ moro é muito pequeno.
Esse foi o número ____ gostei mais.
05 (FCC-TRT-CE-2012)) Está correto o emprego do III. O filme ____ enredo é fraco, tem dado grande
elemento sublinhado na frase: prejuízo.
a) Quem não se irrita por ser o destinatário de mensagens a) onde – que – cujo
por cujo assunto não tem o menor interesse? b) em que – de que – cujo o
c) que – que – cujo o

www.editoradince.com Acesse e veja se há novidades a respeito deste material


38 LÍNGUA PORTUGUESA - TEORIA E QUESTÕES
d) em que – de que – cujo ( palavra que não aceita artigo )
e) no qual – do qual – cujo o Observação 1:
Para saber se uma palavra aceita ou não o artigo,
11 (FCC-TRT 2012) A interiorização das universidades basta usar o seguinte artifício:
federais e a criação de novos institutos I. se pudermos empregar a combinação da antes
tecnológicos também mudam a cara do Nordeste… da palavra, é sinal de que ela aceita o artigo
3o parágrafo)
///(
II. se pudermos empregar apenas a preposição de,
O mesmo tipo de complemento grifado acima está na é sinal de que não aceita.
frase: Ex.: Vim da Bahia. (aceita)
a) … que mexeram com a renda … Vim de Brasília (não aceita)
b) … que mais crescem na região. Vim da Itália. (aceita)
c) … que movimentam milhões de reais … Vim de Roma. (não aceita)
d) A outra face do “novo Nordeste” está no campo. Nunca ocorre crase:
e) … onde as condições são bem menos favoráveis … 1) Antes de masculino.
Caminhava a passo lento.
12 (FCC-SEFAZ-BA-2015) As crianças diferem entre si (preposição)
quanto aos tipos de brinquedos ……………:
algumas crianças ………… brincar com bonecas
…………… jogar xadrez, baralho, tênis e pingue- 2) Antes de verbo.
pongue. Estou disposto a falar.
a) de que gostam – preferem – mais do que (preposição)
b) que gostam – preferem – mais que
c) que gostam – preferem – do que 3) Antes de pronomes em geral.
d) que gostam – preferem – mais do que Eu me referi a esta menina.
e) de que gostam – preferem – a (preposição e pronome demonstrativo)
Eu falei a ela.
GABARITO: 1B; 2 A; 3 B; 4E; 5.A; 6.B; 7.C; 8.A; 9.B; (preposição e pronome pessoal)
10.D; 11.C; 12.E
4) Antes de pronomes de tratamento.
Dirijo-me a Vossa Senhoria.
(preposição)
OCORRÊNCIA DA CRASE
Observações 2:
CRASE 1. Há três pronomes de tratamento que aceitam o
É a fusão de duas vogais da mesma natureza. No artigo e, obviamente, a crase: senhora, senhorita e dona.
português assinalamos a crase com o acento grave (`). Dirijo-me à senhora.
Observe: 2. Haverá crase antes dos pronomes que
Obedecemos ao regulamento. aceitarem o artigo, tais como: mesma, própria...
(a+o) Eu me referi à mesma pessoa.
Não há crase, pois o encontro ocorreu entre duas 2. Com as expressões formadas de palavras
vogais diferentes. Mas: repetidas.
Obedecemos à norma. Venceu de ponta a ponta.
(a+a) (preposição)
Há crase, pois temos a união de duas vogais Observação 3:
iguais ( a + a = à ) É fácil demonstrar que entre expressões desse tipo
ocorre apenas a preposição:
Regra Geral: Caminhavam passo a passo.
Haverá crase sempre que: (preposição)
I. o termo antecedente exija a preposição a; No caso, se ocorresse o artigo, deveria ser o artigo
II. o termo consequente aceite o artigo a. o e teríamos o seguinte: Caminhavam passo ao passo – o
que não ocorre.
Fui à cidade.
6) Antes dos nomes de cidade.
( a + a = preposição + artigo )
Cheguei a Curitiba.
( substantivo feminino )
(preposição)
Conheço a cidade.
Observação 4:
( verbo transitivo direto – não exige preposição )
Se o nome da cidade vier determinado por algum
( artigo )
adjunto adnominal, ocorrerá a crase.
( substantivo feminino )
Cheguei à Curitiba dos pinheirais.
Vou a Brasília. (adjunto adnominal)
( verbo que exige preposição a ) 7) Quando um a (sem o s de plural) vem antes de
( preposição ) um nome plural.

www.editoradince.com Acesse e veja se há novidades a respeito deste material


LÍNGUA PORTUGUESA - TEORIA E QUESTÕES 39
Falei a pessoas estranhas. adjunto adnominal não aceita o artigo, portanto não ocorre
(preposição) a crase.
Por outro lado, se vier determinada por um adjunto
adnominal, aceita o artigo e ocorre a crase. Ex.:
Observação 5:
Volte a casa cedo.
Se o mesmo a vier seguido de s haverá crase.
(preposição sem artigo)
Falei às pessoas estranhas.
Volte à casa dos seus pais.
(a + as = preposição + artigo)
(preposição sem artigo)
Sempre ocorre crase:
(adjunto adnominal)
1) Na indicação pontual do número de horas.
Às duas horas chegamos.
2) Crase antes de terra.
(a + as)
A palavra terra, no sentido de chão firme, tomada
Para comprovar que, nesse caso, ocorre
em oposição a mar ou ar, se não vier determinada, não
preposição + artigo, basta confrontar com uma expressão
aceita o artigo e não ocorre a crase. Ex.:
masculina correlata.
Já chegaram a terra.
Ao meio-dia chegamos.
(preposição sem artigo)
(a + o)
Se, entretanto, vier determinada, aceita o artigo e
2) Com a expressão à moda de e à maneira de.
ocorre a crase. Ex.:
A crase ocorrerá obrigatoriamente mesmo que
Já chegaram à terra dos antepassados.
parte da expressão (moda de) venha implícita.
(preposição + artigo)
Escreve à (moda de) Alencar.
(adjunto adnominal)
3) Nas expressões adverbiais femininas.
Expressões adverbiais femininas são aquelas que
se referem a verbos, exprimindo circunstâncias de tempo, 3) Crase antes dos pronomes relativos.
de lugar, de modo... Antes dos pronomes relativos quem e cujo não
Chegaram à noite. ocorre crase. Ex.:
(expressão adverbial feminina de tempo) Achei a pessoa a quem procuravas.
Caminhava às pressas. Compreendo a situação a cuja gravidade você se
referiu.
(expressão adverbial feminina de modo)
Antes dos relativos qual ou quais ocorrerá crase se
Ando à procura de meus livros.
o masculino correspondente for ao qual, aos quais. Ex.:
(expressão adverbial feminina de fim)
Esta é a festa à qual me referi.
Este é o filme ao qual me referi.
Observações 6:
Estas são as festas às quais me referi.
No caso das expressões adverbiais femininas,
Estes são os filmes aos quais me referi.
muitas vezes empregamos o acento indicativo de crase
(`), sem que tenha havido a fusão de dois as. É que a
tradição e o uso do idioma se impuseram de tal sorte que, 4) Crase com os pronomes demonstrativos aquele (s),
ainda quando não haja razão suficiente, empregamos o aquela (s), aquilo.
acento de crase em tais ocasiões. Sempre que o termo antecedente exigir a
4) Uso facultativo da crase preposição a e vier seguido dos pronomes
Antes de nomes próprios de pessoas femininos e demonstrativos: aquele, aqueles, aquela, aquelas, aquilo,
antes de pronomes possessivos femininos, pode ou não haverá crase. Ex.:
ocorrer a crase. Falei àquele amigo.
Ex.: Falei à Maria. Dirijo-me àquela cidade.
(preposição + artigo) Aspiro a isto e àquilo.
Falei à sua classe. Fez referência àquelas situações.
(preposição + artigo)
Falei a Maria. 5) Crase depois da preposição até.
(preposição sem artigo) Se a preposição até vier seguida de um nome
Falei a sua classe. feminino, poderá ou não ocorrer a crase. Isto porque essa
preposição pode ser empregada sozinha (até) ou em
(preposição sem artigo)
locução com a preposição a (até a). Ex.:
Note que os nomes próprios de pessoa femininos
Chegou até à muralha.
e os pronomes possessivos femininos aceitam ou não o
artigo antes de si. Por isso mesmo é que pode ocorrer a (locução prepositiva = até a)
crase ou não. (artigo = a)
Chegou até a muralha.
CASOS ESPECIAIS: (preposição sozinha = até)
1) Crase antes de casa. (artigo = a)
A palavra casa, no sentido de lar, residência
própria da pessoa, se não vier determinada por um 6) Crase antes do que.

www.editoradince.com Acesse e veja se há novidades a respeito deste material


40 LÍNGUA PORTUGUESA - TEORIA E QUESTÕES
Em geral, não ocorre crase antes do que. Ex.: Esta dama meiga e sossegada, fica sentada tricotando
é a cena a que me referi. tranquilamente, impassível ___ propensão de seu
Pode, entretanto, ocorrer antes do que uma crase marido ___ investigar assassinatos.
da preposição a com o pronome demonstrativo a (Adaptado de P.D.James, op.cit.)
(equivalente a aquela). Preenchem corretamente as lacunas da frase acima, na
Para empregar corretamente a crase antes do que ordem dada:
convém pautar-se pelo seguinte artifício: a) à - à - a
I. se, com antecedente masculino, ocorrer ao que / aos b) a - à - a
que, com o feminino ocorrerá crase;
c) à - a - à
Ex.: Houve um palpite anterior ao que você deu.
d) a - à - à
(a+o)
e) à - a – a
Houve uma sugestão anterior à que você deu.
(a+a)
3. (FCC-TRF-3R-2007) É preciso suprimir um ou mais
sinais de crase em:
II. se, com antecedente masculino, ocorrer a que, no a) À falta de coisa melhor para fazer, muita gente assiste
feminino não ocorrerá crase. à televisão sem sequer atentar para o que está vendo.
Ex.: Não gostei do filme a que você se referia. b) Cabe à juventude de hoje dedicar-se à substituição dos
(ocorreu a que, não tem artigo) apelos do mercado por impulsos que, em sua verdade
Não gostei da peça a que você se referia. natural, façam jus à capacidade humana de sonhar.
(ocorreu a que, não tem artigo) c) Os sonhos não se adquirem à vista: custa tempo para
se elaborar dentro de nós a matéria de que são feitos,
Observação:
às vezes à revelia de nós mesmos.
O mesmo fenômeno de crase (preposição a +
d) Compreenda-se quem aspira à estabilidade de um
pronome demonstrativo a) que ocorre antes do que, pode
emprego, mas prestem-se todas as homenagens
ocorrer antes do de. Ex.:
àquele que cultiva seus sonhos.
Meu palpite é igual ao de todos.
e) Quem acha que agracia à juventude de hoje com
(a + o = preposição + pronome demonstrativo) elogios ao seu pragmatismo não está à salvo de ser o
Minha opinião é igual à de todos. responsável pela frustração de toda uma geração.
(a + a = preposição + pronome demonstrativo)
7) há / a 4. (FCC-TRT - 24ª REGIÃO-2011) Considere as frases
Nas expressões indicativas de tempo, é preciso seguintes:
não confundir a grafia do a (preposição) com a grafia do I. As inovações no ramo da estética permitem ___ um
há (verbo haver). grande número de pessoas se sentirem mais
Para evitar enganos, basta lembrar que, nas belas.
referidas expressões: II. Sempre existiu preocupação com a beleza, embora
a (preposição) indica tempo futuro (a ser mudem os critérios ___ que ela obedece.
transcorrido); III. A beleza, ___ parte alguns exageros, deve ser
há (verbo haver) indica tempo passado (já buscada até mesmo com intervenções cirúrgicas.
transcorrido). Ex.: As lacunas das frases acima estarão corretamente
Daqui a pouco terminaremos a aula. preenchidas, respectivamente, por:
Há pouco recebi o seu recado. a) à - a - à
b) a - a - a
QUESTÕES DE CONCURSOS c) a - à - à
d) à - à - a
1. (FCC-TST-2012) Considere:
e) a - a - à
___ angústia de imaginar que o homem pode estar só
no universo soma-se a curiosidade humana, que
se prende ___ tudo o que é desconhecido, para 5. (FCC-TRT - 24ª REGIÃO-2011) Justifica-se
que não desapareça de todo o interesse por pistas plenamente o emprego de ambos os sinais de
que dariam embasamento___ teses de que haveria crase em:
vida em outros planetas. a) Ela pode voltar à qualquer momento, fiquemos atentos
Preenchem corretamente as lacunas da frase acima, à sua chegada.
na ordem dada: b) Dispôs-se à devolver o livro, à condição de o liberarem
a) À - a - às da multa por atraso.
b) A - à - as c) Postei-me à entrada do cinema, mas ela faltou também
c) À - a - as à esse compromisso.
d) A - a - às d) Àquela altura da velhice já não assistia à filmes
trágicos, apenas aos de humor.
e) À - à - as
e) Não confie à priminha os documentos que obtive à
revelia do nosso advogado.
2. (FCC-TRF - 5ª REGIÃO-2012) O detetive Gervase
Fen, que apareceu em 1944, é um homem de face
corada, muito afeito ___ frases inteligentes e 6. (FCC-TRE-RN-2011) Graças ___ resistência de
citações dos clássicos; sua esposa, Dolly, uma portugueses e espanhóis, a Inglaterra furou o

www.editoradince.com Acesse e veja se há novidades a respeito deste material


LÍNGUA PORTUGUESA - TEORIA E QUESTÕES 41
bloqueio imposto por Napoleão e deu início ___ É como disse Platão: “De todos os animais
campanha vitoriosa que causaria ___ queda do selvagens, o homem jovem é o mais difícil de domar.”
imperador francês.
Preenchem as lacunas da frase acima, na ordem dada, 3. Reticências (…)
a) a - à - a Usa-se para:
b) à - a - a a) Indicar dúvidas ou hesitação:
c) à - à - a Sabe… preciso confessar uma coisa: naquela
d) a - a - à viagem gastei todas as minhas economias.
e) à - a – à b) Interromper uma frase incompleta
sintaticamente:
7. (FCC-TRT - 1ª REGIÃO-2011) O avanço rumo ___ Talvez se você pedisse com jeitinho…
um desenvolvimento sustentável depende de c) Concluir uma frase gramaticalmente incompleta
diversos fatores, entre os quais estão o estímulo com a intenção de estender a reflexão:
___ novas tecnologias e o compromisso ético de Pedofilia, estupros, assassinatos, pessoas sem ter
empresas que tenham como prioridade o respeito onde morar, escândalos ligados à corrupção… assim
___ causas ambientais. caminha a humanidade.
Preenchem corretamente as lacunas da frase acima, d) Suprimir palavras em uma transcrição:
na ordem dada:
“O Cristo não pediu muita coisa. (…) Ele só pediu
a) a - à - as que nos amássemos uns aos outros.” (Chico Xavier)
b) a - a - às
c) à - a - as 4. Parênteses ( )
d) a - à - às Os parênteses são usados para:
e) à - à - as a) Isolar palavras, frases intercaladas de caráter
explicativo, datas e, também, podem substituir a vírgula
GABARITO: 1. A; 2. B; 3. E; 4. E; 5. E; 6. C; 7. B ou o travessão:
Rosa Luxemburgo nasceu em Zamosc (1871).
Numa linda tarde primaveril (meu caçula era um
bebê nessa época), ele veio nos visitar pela última vez.
PONTUAÇÃO
Sinais de pontuação são recursos prosódicos que 5. Ponto de exclamação (!)
conferem às orações ritmo, entoação e pausa, bem como Em que situações utilizar:
indicam limites sintáticos e unidades de sentido. Na a) Após vocativo:
escrita, substituem, em parte, o papel desempenhado Juliana, bom dia!
pelos gestos na fala, garantindo coesão, coerência e boa
compreensão da informação transmitida. b) Final de frases imperativas:
Confira abaixo os dez sinais de pontuação Fuja!
utilizados na nossa língua, assim como as situações em c) Após interjeição:
que devem ser empregados, seguidas de exemplos. Ufa! Graças a Deus!
d) Após palavras ou frases de caráter emotivo,
1. Ponto (.) expressivo:
O ponto pode ser utilizado para: Que lástima!
a) Indicar o final de uma frase declarativa:
Acho que Pedro está gostando de você. 6. Ponto de interrogação (?)
b) Separar períodos: Quando utilizar:
Ela vai estudar mais tempo. Ainda é cedo. a) Em perguntas diretas:
c) Abreviar palavras: Quando você chegou?
V. Ex.ª (Vossa excelência) b) Às vezes, pode ser utilizada junto com o ponto
de exclamação para enfatizar o enunciado:
2. Dois-pontos (:) Não acredito, é sério?!
Deve ser utilizado com as seguintes finalidades:
a) Iniciar fala de personagens: 7. Vírgula (,)
Ela gritou: Esse é o sinal de pontuação que exerce o maior
número de funções, por isso aparece em várias situações.
– Vá embora! A vírgula marca pausas no enunciado, indicando que os
b) Anteceder apostos ou orações apositivas, termos por ela separados não formam uma unidade
enumerações ou sequência de palavras que explicam sintática, apesar de estarem na mesma oração.
e/ou resumem ideias anteriores. A seguir confira as situações em que se deve
Esse é o problema dessa geração: tem liberdade, utilizar vírgula.
mas não tem responsabilidade. a) Separar o vocativo:
Anote meu número de telefone: 863820847. Marília, vá à padaria comprar pães para o lanche.
c) Anteceder citação direta:

www.editoradince.com Acesse e veja se há novidades a respeito deste material


42 LÍNGUA PORTUGUESA - TEORIA E QUESTÕES
b) Separar apostos:
Camila, minha filha caçula, presenteou-me com p) Para separar orações subordinadas adverbiais
este relógio. (desenvolvidas ou reduzidas), principalmente se
estiverem antepostas à oração principal:
c) Separar o adjunto adverbial antecipado ou A casa, tão cara que ela desistiu da compra, hoje
intercalado: está entregue às baratas.
Os políticos, muitas vezes, visam somente os
próprios interesses. q) Para separar as orações intercaladas:
Ficou doente, creio eu, por conta da chuva de
d) Separar elementos de uma enumeração: ontem.
Meus bolos prediletos são os de chocolate, coco,
doce de leite e nata com morangos. r) Para separar as orações substantivas
antepostas à principal:
e) Isolar expressões explicativas: Quando me formarei, ainda não sei.
Faça um bolo de chocolate, ou melhor, de
chocolate e morangos. 8. Ponto e vírgula (;)
a) Utiliza-se ponto e vírgula para separar os itens
f) Separar conjunções intercaladas: de uma sequência de outros itens:
Os deputados não explicaram, porém, o porquê de Para preparar o bolo vamos precisar dos seguintes
tantas faltas. ingredientes:
1 xícara de trigo;
g) Separar o complemento pleonástico antecipado: 4 ovos;
Havia no rosto dela ódio, uma ira, uma raiva que 1 xícara de leite;
não possuía justificativa. 1 xícara de açúcar;
1 colher de fermento.
h) Isolar o nome do lugar na indicação de datas: b) Utilizamos ponto e vírgula, também, para
São Paulo, 10 de Dezembro de 2016. separar orações coordenadas muito extensas ou orações
coordenadas nas quais já se tenha utilizado a vírgula:
“O rosto de tez amarelenta e feições inexpressivas,
i) Separar termos coordenados assindéticos:
numa quietude apática, era pronunciadamente vultuoso, o
Vim, vi, venci. (Júlio César) que mais se acentuava no fim da vida, quando a bronquite
crônica de que sofria desde moço se foi transformando
j) Marcar a omissão de um termo: em opressora asma cardíaca; os lábios grossos, o inferior
Maria gosta de praticar esportes, e eu, de comer. um tanto tenso.” (O Visconde de Inhomerim – Visconde
(omissão do verbo gostar) de Taunay)
Antes da conjunção, como nos casos abaixo:
9. Travessão (—)
k) Quando as orações coordenadas possuem O travessão deve ser utilizado para os seguintes
sujeitos diferentes: fins:
Os políticos estão cada vez mais ricos, e seus a) Iniciar a fala de um personagem no discurso
eleitores, cada vez mais pobres. direto:
l) Quando a conjunção “e” repete-se com o objetivo Então ela disse:
de enfatizar alguma ideia (polissíndeto): — Gostaria que fosse possível fazer a viagem
Eu alerto, e brigo, e repito, e faço de tudo para ela antes de Outubro.
perceber que está errada, porém nunca me escuta.
b) Indicar mudança do interlocutor nos diálogos:
m) Utilizamos a vírgula quando a conjunção “e” — Querido, você já lavou a louça?
assume valores distintos que não retratam sentido de — Sim, já comecei a secar, inclusive.
adição (adversidade, consequência, por):
Teve febre a noite toda, e ainda está muito fraca. c) Unir grupos de palavras que indicam itinerários:
Entre orações: O descaso do poder público com relação à rodovia
Belém—Brasília é decepcionante.
n) Para separar as orações subordinadas adjetivas
explicativas: d) Substituir a vírgula em expressões ou frases
Amélia, que não se parece em nada com a Amélia explicativas:
da canção, não suportou seu jeito grosseiro e mandão. Dizem que Elvis — o rei do rock — na verdade,
detestava atuar.
o) Para separar as orações coordenadas
sindéticas e assindéticas, com exceção das orações 10. Aspas (“”)
iniciadas pela conjunção “e”:
As aspas são utilizadas com os seguintes
Pediu muito, mas não conseguiu convencer-lhe. objetivos:

www.editoradince.com Acesse e veja se há novidades a respeito deste material


LÍNGUA PORTUGUESA - TEORIA E QUESTÕES 43
a) Isolar palavras ou expressões que fogem à cópia dos documentos solicitados para análise pelo
norma culta, como gírias, estrangeirismos, palavrões, profissional responsável.
neologismos, arcaísmos e expressões populares: C. O contribuinte que fabrica ou comercializa água
A aula do professor foi “irada”. mineral, natural ou artificial fica obrigado a utilizar o
Ele me pediu um “feedback” da resposta do selo fiscal de controle nos lacres de água envasada
cliente. em vasilhame retornável – independentemente, da
unidade da federação de origem.
D. Ao produtor contratado será exigido disponibilizar à
b) Indicar uma citação direta:
secretaria, acesso a um sistema digital que possibilite,
“Ia viajar! Viajei. Trinta e quatro vezes, às pressas, a realização de pedidos, homologações, consultas ao
bufando, com todo o sangue na face, desfiz e refiz a status dos pedidos em análise – como condição para
mala.” (O prazer de viajar – Eça de Queirós) a continuidade do contrato.
Observação: Quando houver necessidade de E. O sistema de controle de informação conforme
utilizar aspas dentro de uma sentença onde ela já esteja estabelecido em contrato, precisa assegurar: sigilo,
presente, usa-se a marcação simples (‘), não dupla (“). integridade, autenticidade e disponibilidade dos dados,
para viabilizar a execução das ações de fiscalização e
monitoramento dos processos.
QUESTÕES DE CONCURSOS
1. (FCC-SEGEP-MA-2017)A frase que permanece
4.(FCC-TRT-2017) Está plenamente adequada a
correta com o acréscimo de uma vírgula após o
pontuação do seguinte período:
termo sublinhado é:
A. A produção cinematográfica como é sabido, sempre
A. Infelizmente não se tem notícia da responsabilização
bebeu na fonte da literatura, mas o cinema declarou-
penal dos importadores de equipamentos para piratear
se, independente das outras artes há mais de meio
sinal de TV pelo crime de contrabando.
século.
B. Todos os brasileiros devem se preocupar com o
B. Sabe-se que, a produção cinematográfica sempre
problema da fabricação de equipamentos para piratear
considerou a literatura como fonte de inspiração, mas
sinal de TV por assinatura.
o cinema declarou-se independente das outras artes,
C. Ninguém sabe quando os tais 'gatonets' deixaram de há mais de meio século.
existir em nosso país nem mesmo se todos os
C. Há mais de meio século, o cinema declarou-se
responsáveis serão punidos.
independente das outras artes, embora a produção
D. Dizem que alguns dos equipamentos utilizados para cinematográfica tenha sempre considerado a literatura
piratear sinal de TV por assinatura às vezes não ligam como fonte de inspiração.
nem desligam.
D. O cinema declarou-se independente, das outras artes,
E. Autoridades poderão multar os responsáveis pela há mais de meio século; porém, sabe-se, que a
pirataria de sinal de TV e também colocá-los na produção cinematográfica sempre bebeu na fonte da
prisão. literatura.
E. A literatura, sempre serviu de fonte inspiradora do
2 (FCC-SEGEP-MA-2018) É plenamente adequada a cinema, mas este, declarou-se independente das
pontuação da seguinte frase: outras artes há mais de meio século − como é sabido.
A. Não obstante, as opiniões em contrário, há quem
admita que, o ser humano, jamais pode ser explicado, 5 (FCC- TRT-2016) Está pontuada corretamente, a
mas tão somente compreendido. frase:
B. O escritor Machado de Assis notadamente um mestre A. Nascido em Cuiabá, em 1916 Manoel de Barros
da ironia, já comparou o fenômeno da traição estreou, com o livro, Poemas Concebidos sem Pecado
amorosa, com a naturalidade de uma pedra que cai. em 1937.
C. O autor do texto em foco, Hélio Pellegrino, era, além de B. Cronologicamente vinculado, à Geração de 45, mas
escritor muito talentoso, um renomado, inquieto e formalmente, ao Modernismo brasileiro, Manoel de
politizado psicanalista. Barros criou um estilo próprio.
D. Uma tragédia humana a rigor, não se explica, tal como C. Subvertendo a sintaxe e criando construções que não
a entende o autor do texto, no qual aliás, nos lembra a respeitam as normas da língua padrão, Manoel de
diferença entre explicar e compreender. Barros é comparado a Guimarães Rosa.
E. Distinguir entre explicar e compreender, constitui uma D. Em 1986, o poeta Carlos Drummond de Andrade
obrigação especialmente para aqueles, que narram os declarou, que Manoel de Barros era o maior poeta
fatos, e interpretam uma notícia. brasileiro vivo.
E. Antonio Houaiss, um dos mais importantes filólogos e
3( FCC-SEFAZ-GO-2018)Os sinais de pontuação estão críticos brasileiros confessou nutrir, pela obra de
empregados corretamente em: Manoel de Barros grande admiração.
A. Será descredenciada a empresa que após ter sido
notificada, descumprir as exigências estabelecidas na 6( FCC-TRT-2015) A frase que está pontuada como
legislação tributária relativa à certificação requerida, dispõe a gramática normativa é:
para a industrialização ou comercialização de
A. Foi o caráter destrutivo da guerra tanto física quanto
produtos de origem animal.
moral, que finalmente levou os países em confronto a
B. Para efetuar o cadastro de beneficiário, o cidadão conhecerem as situações mais dramáticas e
deverá preencher o formulário disponível no site da desumanas.
secretaria, por meio do qual também encaminhará

www.editoradince.com Acesse e veja se há novidades a respeito deste material


44 LÍNGUA PORTUGUESA - TEORIA E QUESTÕES
B. Podemos indagar o sentido que têm essas b) cientificamo-lo de que na marcha do processo de
manifestações recentes: seriam, por acaso, a restituição de suas contribuições, verificou-se a
expressão de que o cidadão comum chegou a seu ausência da declaração de beneficiários;
limite no que se refere a aceitar, inerte, a usurpação c) o Instituto de Previdência do Estado, vem solicitar de
de seus direitos? V.Sa. o preenchimento da declaração;
C. Nem pelas razões alegadas contudo, altera-se o d) encaminhamos a V.Sa., para o devido preenchimento,
mecanismo de apelo imediato à justiça quando os o formulário em anexo;
cidadãos não se entendem entre si, modo de agir que
e) estamos remetendo em anexo, o formulário.
impede esforços de outros tipos de mediações.
D. Não é de se admirar que: muitos críticos
contemporâneos entendam como mero exibicionismo, GABARITO:
certas atitudes e práticas artísticas das primeiras 1A 6B
décadas do século XX. 2C 7B
E. Estudar o ontem em função do hoje − e também do 3B 8B
amanhã, é o propósito das reflexões, que ocorrem,
4C 9C
semanalmente no centro de cultura instalado numa
das regiões mais carentes da cidade. 5C 10D

7(FCC-TRE-RS-2015) Assinale a opção em que a


supressão das vírgulas alteraria o sentido do
anunciado: SIGNIFICAÇÃO CONTEXTUAL DE
a) os países menos desenvolvidos vêm buscando, PALAVRAS E EXPRESSÕES
ultimamente, soluções para seus problemas no acervo
Uma palavra possui por definição muitos
cultural dos mais avançados;
significados os quais mudam dependendo do contexto
b) alguns pesquisadores, que se encontram onde ele é inserido. Para sabermos essas diferenças
comprometidos com as culturas dos países dentro dos contextos é preciso entender alguns termos e
avançados, acabam se tornando menos criativos; assim saber defini-los. Lembrando que um palavra pode
c) torna-se, portanto, imperativa uma revisão modelo possuir mais de uma definição.
presente do processo de desenvolvimento
tecnológico;
DENOTAÇÃO E CONOTAÇÃO
d) a atividade científica, nos países desenvolvidos, é tão
Denotação
natural quanto qualquer outra atividade econômica;
Uma palavra é usada no sentido denotativo
e) por duas razões diferentes podem surgir, da interação
(próprio ou literal) quando apresenta seu significado
de uma comunidade com outra, mecanismos de
original, independentemente do contexto frásico em que
dependência.
aparece. Quando se refere ao seu significado mais
objetivo e comum, aquele imediatamente reconhecido e
8 (FCC-DPE-SP-2015) Assinale a opção em que está muitas vezes associado ao primeiro significado que
corretamente indicada a ordem dos sinais de aparece nos dicionários, sendo o significado mais literal
pontuação que devem preencher as lacunas da da palavra.
frase abaixo: A denotação tem como finalidade informar o
“Quando se trata de trabalho científico ___ duas coisas receptor da mensagem de forma clara e objetiva,
devem ser consideradas ____ uma é a contribuição assumindo assim um caráter prático e utilitário. É utilizada
teórica que o trabalho oferece ___ a outra é o valor em textos informativos, como jornais, regulamentos,
prático que possa ter. manuais de instrução, bulas de medicamentos, textos
a) dois pontos, ponto e vírgula, ponto e vírgula científicos, entre outros.
b) dois pontos, vírgula, ponto e vírgula; “O elefante é um mamífero”.
c) vírgula, dois pontos, ponto e vírgula;
d) pontos vírgula, dois pontos, ponto e vírgula; Conotação
e) ponto e vírgula, vírgula, vírgula. Uma palavra é usada no sentido conotativo
(figurado) quando apresenta diferentes significados,
sujeitos a diferentes interpretações, dependendo do
9 (FCC-TRE-MA-2015) Assinale o exemplo em que há contexto frásico em que aparece. Quando se refere a
emprego incorreto da vírgula: sentidos, associações e ideias que vão além do sentido
a) como está chovendo, transferi o passeio; original da palavra, ampliando sua significação mediante a
b) não sabia, por que todos lhe viravam o rosto; circunstância em que a mesma é utilizada, assumindo um
c) ele, caso queira, poderá vir hoje; sentido figurado e simbólico.
d) não sabia, por que não estudou; A conotação tem como finalidade provocar
sentimentos no receptor da mensagem, através da
e) o livro, comprei-o por conselho do professor.
expressividade e afetividade que transmite. É utilizada
principalmente numa linguagem poética e na literatura,
10 (FCC-TRE-SE-2015) Assinale o trecho sem erro de mas também ocorre em conversas cotidianas, em letras
pontuação: de música, em anúncios publicitários, entre outros.
a) vimos pela presente solicitar de V.Sas., que nos “Você tem um coração de pedra!”.
informe a situação econômica da firma em questão;

www.editoradince.com Acesse e veja se há novidades a respeito deste material


LÍNGUA PORTUGUESA - TEORIA E QUESTÕES 45
preferível uma linguagem mais objetiva, com vocábulos
ANTÔNIMO E SINÔNIMO ou expressões que sejam mais adequadas às finalidades
requeridas. Quando não é feito de forma proposital, ou
Conhecer o significado das palavras é importante,
seja, causado por algum tipo de erro as ambiguidades são
pois só assim o falante ou escritor será capaz de
consideradas vícios de linguagem.
selecionar a palavra certa para construir a sua
mensagem. Por esta razão, é importante conhecer fatos Os tipos comuns de ambiguidade, como vício de
linguísticos como: sinonímia e antonímia. linguagem são:
Sinonímia (sinônimos): palavras que possuem
significados iguais ou semelhantes. USO INDEVIDO DE PRONOMES POSSESSIVOS
“adversário e antagonista”. A mãe pediu à filha que arrumasse o seu quarto.
“transformação e metamorfose”. Qual quarto? o da mãe ou da filha? Para evitar
Antonímia (antônimos): palavras que possuem ambiguidade:
significados opostos. A mãe pediu à filha que arrumasse o próprio
“bendizer e maldizer”. quarto.
“progredir e regredir”.
COLOCAÇÃO INADEQUADA DAS PALAVRAS
HOMÔNIMOS E PARÔNIMOS A criança feliz foi ao parque.
Homônimos são palavras com escrita ou A criança ficou feliz ao chegar no parque, ou
pronúncia iguais, com significado (sentido) diferente. estava assim antes?
“A manga está uma delícia.” Feliz, a criança foi ao parque.
“A manga da camisa ficou perfeita.” A estudante falou com o garoto que estudava
enfermagem.
Tipos de homônimos: homógrafos, homófonos e
homônimos perfeitos. Quem estuda enfermagem, a estudante ou o
garoto? para corrigir:
Homógrafos – Homófonos – Homônimos
mesma grafia e grafia diferente perfeitos – A estudante de enfermagem falou com o garoto;
som e mesmo mesma grafia e
diferente.“Eu co som.‘A cela do som.“Vou pegar USO INDEVIDO DE FORMAS NOMINAIS
meço a presídio está dinheiro A moça reconheceu a amiga frequentando a
trabalhar em lotada.” no banco.” academia.
breve.” “A sela do “O banco da Quem estava na academia? a moça ou a amiga?
“O começo do cavalo está praça quebrou.” para corrigir:
filme foi ótimo.” velha.”
A moça reconheceu a amiga que estava
frequentando a academia.
Parônimos são palavras com escrita e pronúncia
parecidas, mas com significado (sentido) diferente.
POLISSEMIA
“O homem fez uma bela descrição da mulher”
Polissemia é um conceito da área da linguística
“Use a sua discrição, Paulo” com origem no termo grego polysemos, que significa “algo
Outro exemplo: que tem muitos significados”. Uma palavra polissêmica é
Amoral – nem contrário e nem conforme a moral uma palavra que reúne vários significados.
Imoral – contrário à moral A palavra “vela” é um dos exemplos de polissemia.
Ela pode significar a vela de um barco; a vela feita de cera
que serve para iluminar ou pode ser a conjugação
AMBIGUIDADES
do verbo velar, que significa estar vigilante.
Ambiguidade ou anfibologia é o nome dado, dentro
A polissemia constitui uma propriedade básica das
da linguística na língua portuguesa, à duplicidade de
unidades léxicas e um elemento estrutural da linguagem.
sentidos, onde alguns termos, expressões, sentenças
O oposto da polissemia é a monossemia, onde uma
apresentam mais de uma acepção ou entendimento
palavra assume só um significado.
possível. Em outras palavras, ocorre quando, por falta de
clareza, há duplicidade de sentido da frase. Apesar de ser Exemplo 1
um recurso aceitável dentro da linguagem poética ou A letra da música do Chico Buarque é incrível.
literária, deve ser na maioria das vezes, evitado em A letra daquele aluno é inteligível
construções textuais de caráter técnico, informativo, ou Meu nome começa com a letra D.
pragmático.
Logo, constatamos que a palavra “letra” é um
O uso da ambiguidade pode resultar na má termo polissêmico, visto que abarca significados distintos
interpretação da mensagem, ocasionando múltiplos dependendo de sua utilização.
sentidos. É importante lembrar que toda comunicação
estabelece uma finalidade, uma intenção para com o
interlocutor, e para que isso ocorra, a mensagem tem de HIPERONÍMIA E HIPONÍMIA
estar clara, precisa e coerente. A hiperonímia indica uma relação hierárquica de
Na publicidade observamos o uso e o abuso da significado que uma palavra superior estabelece com uma
linguagem plurissignificante, por meio dos trocadilhos e palavra inferior.
jogos de palavras, procurando chamar a atenção do A hiponímia indica, assim, essa mesma relação
interlocutor para a mensagem. Caso o autor não se julgue hierárquica de significado. Foca-se, no entanto, na
preparado para utilizar corretamente a ambiguidade, é perspectiva da palavra hierarquicamente inferior.

www.editoradince.com Acesse e veja se há novidades a respeito deste material


46 LÍNGUA PORTUGUESA - TEORIA E QUESTÕES
País é hiperônimo de Brasil.
Mamífero é hiperônimo de cavalo.
Jogo é hiperônimo de xadrez.
PROVA UECE GABARITADA
Brasil é hipônimo de país. SECRETARIA DAS CIDADES/CE - CATEGORIA/NÍVEL:
Os hiperônimos: ANALISTA FINANCEIRO PLENO I/2013
Texto
Apresentam um sentido abrangente;
01 Clichês são expressões tão utilizadas e
Transmitem a ideia de um todo;
02 repetidas que se desgastaram e se afastaram de
Representam as características genéricas de uma
03 seu significado original. Essa espécie de "preguiça
classe;
04 linguística", que poupa esforços, inibe a reflexão e
Permitem a formação de subclasses associadas a 05
elas. multiplica a passividade entre interlocutor e
06 receptor, permeia todos os níveis da linguagem, da
Os hipônimos: 07
conversa de elevador aos discursos políticos,
Apresentam um sentido restrito; 08
passando, obviamente, pela mídia.
Transmitem a ideia de um item ou uma parte de 09
um todo; Ao usar clichês como muletas do discurso, o
10
texto certamente flui com facilidade – a linguagem,
Representam as características específicas de 11
uma subclasse; 12 porém, empobrece. O clichê nasce como uma ideia
Permitem a associação a uma classe superior 13 criativa, mas é repetida à exaustão e se transforma
mais abrangente. 14 em um cacoete. Ele está inserido num contexto que
15 a gíria nunca alcança, e o provérbio sempre
Exemplos de hiperônimos e hipônimos 16 ultrapassa - a gíria pressupõe vitalidade, e o
17 provérbio, ao contrário, já nasce cristalizado.
Hiperônimo Hipônimos
18 Entre os chavões mais comuns estão as
Cor verde, azul, amarelo, vermelho, 19 locuções e combinações invariáveis de palavras
branco,…
20 (sempre as mesmas, na mesma ordem), como "frio
Fruta maçã, banana, manga, abacaxi, 21 e calculista", "mentira deslavada" e "chuva
jaca,… 22 torrencial". Esse tipo de clichê está presente na
veículo carro, automóvel, moto, bicicleta, 23 linguagem falada e escrita, seja formal ou informal.
ônibus,… 24 O desconforto em relação ao uso de clichês
esporte natação, futebol, patinação, atletismo, 25
está na denotação de falta de originalidade,
esgrima,… 26
exigindo um mínimo de produção e de
animal cobra, onça, cachorro, urubu, urso,… 27
interpretação. Por outro lado, os clichês presentes
ferramenta martelo, serrote, alicate, enxada, 28
em um texto, um filme ou uma conversa apenas são
chave de fenda,… 29
30 entendidos como tal se os interlocutores tiverem
31 referências em comum.
Uso de hiperônimos e hipônimos A tensão entre a necessidade de ser
32
O uso de hiperônimos e hipônimos é essencial entendido e a vontade de fazê-lo com expedientes
33
para a construção de uma boa coesão lexical num texto. criativos e originais pode levar, num extremo, à
34
Os hiperônimos e hipônimos atuam como um recurso
coesivo lexical que permite a abordagem de um tema 35 adoção de uma linguagem privada e ininteligível.
evitando repetições vocabulares. 36 Segundo o psicanalista e sociólogo alemão Alfred
Além disso, desempenham uma função anafórica 37 Lorenzer, o indivíduo se afasta da interação social
no texto, fazendo referência a uma informação 38 por conta do uso de palavras-chave, que ele
previamente mencionada sem a repetir, através do uso de 39 emprega sem pensar no que significam e que
substantivos genéricos e específicos. 40 recebe e repassa como moeda de mercado.
41 (Adaptado de Tatiana Napoli. Língua Portuguesa. São
REFERÊNCIAS 42 Paulo: Escala educacional, nº 17, p. 48-51)
ALMEIDA, Napoleão Mendes de. Gramática metódica da língua COMPREENSÃO TEXTUAL
portuguesa: curso único e completo. Saraiva, 1963. 1) No texto acima transcrito, a autora defende a ideia de
___ Mendes de et al. Língua Portuguesa. São Paulo, Saraiva, que:
1999.
A) O clichê, por ser criativo, enriquece o texto literário.
BAGNO, Marcos. Preconceito lingüístico: o que é, como se faz.
Edições Loyola, 1999. B) As locuções verbais estão entre os chavões mais
BECHARA, Evanildo. Moderna gramática portuguesa. Nova usados cotidianamente.
Fronteira, 2012. C) A combinação de palavras de um mesmo clichê
CEGALLA, Domingos Paschoal. Novíssima gramática da língua comporta variações.
portuguesa. 2008. D) Em princípio, o clichê revela-se criativo, mas seu
MARCUSCHI, Luiz Antônio. Compreensão de texto: algumas emprego reiterado o transforma em cacoete.
reflexões. O livro didático de português: múltiplos olhares, v.
3, p. E) Gíria, provérbio e clichê têm o mesmo impacto no
discurso.

www.editoradince.com Acesse e veja se há novidades a respeito deste material


LÍNGUA PORTUGUESA - TEORIA E QUESTÕES 47
POLISSEMIA/SEMÂNTICA AGENTE PENITENCIÁRIO –CEV/UECE
2) Atente para o significado das seguintes palavras, no
Texto – Exploração Sexual Infantil
contexto em que elas aparecem no texto aqui
(Diário do Nordeste. 21 de março de 2006)
analisado, e assinale a opção correta:
Uma ação integrada iniciará uma nova fase no
A) “Ininteligível” (l. 37) – clara, objetiva, compreensível.
combate à exploração sexual infanto-juvenil no Estado. A
B) “Cacoete” (l. 14) – hábito repetitivo, mania.
assinatura do convênio, inédito em Fortaleza, aconteceu,
C) “Permeia” (l. 06) – exclui, afasta.
ontem à noite, no Restaurante Estoril, na Praia de
D) “Muletas” (l. 10) – bastões de braço curvo, aos quais
Iracema, e contou com a participação de diversas
se apoiam os coxos.
autoridades em nível local e nacional.
E) “Provérbio” (l. 16) – o mesmo que “gíria”.
O convênio faz parte de uma ação integrada entre
FUNÇÕES MORFO-SINTÁTICA DA PALAVRA QUE Governo Federal, estados e municípios, através do
3) A função morfológica e sintática da palavra “que”, na Programa de Ações Integradas e Referenciais de
linha 15, é: Enfrentamento à Exploração Sexual Infanto-Juvenil no
A) Pronome relativo e objeto direto. Território Brasileiro (PAIR).
B) Pronome relativo e sujeito. O termo do acordo foi assinado pela prefeita de
C) Substantivo e sujeito. Fortaleza, Luizianne Lins, com a participação do
D) Preposição e complemento nominal. secretário adjunto dos Direitos Humanos do Ministério da
E) Preposição e objeto direto. Justiça, médico cearense Mário Mamede. Representando
o Estado, esteve a coordenadora estadual do Núcleo de
CONCORDÂNCIA VERBAL Enfrentamento, Maria de Lourdes.
4) A concordância verbal está correta, exceto em: O programa envolve diversos ministérios, como o
A) Quantos de vós passarão no concurso? do Desenvolvimento Social (MDS), Educação e Saúde,
B) Mais de um aluno fez a prova. além da Secretaria Especial de Direitos Humanos e do
C) A maioria das pessoas saíram correndo. Programa de Atendimento às Crianças e aos
D) Somos nós quem paga aos funcionários. Adolescentes Vítimas de Tráfico para Fins Sexuais no
E) Elaborou-se ótimas questões. Ceará.
O convênio prevê a liberação inicial de uma verba
EMPREGO DE PORQUÊS no valor de R$ 350 mil, que serão utilizados na
5) Observe o emprego de “porque”, “porquê”, “por que” e capacitação de aproximadamente 400 pessoas ligadas
“por quê” e marque a opção incorreta: às diversas secretarias do município e do Estado, nas
A) Você não saiu ontem por que choveu? áreas de educação, saúde e assistência social, além das
B) Você chorou tanto, por quê? varas da Infância e da Juventude. Segundo a presidente
C) Esta é a rua por que o cortejo passou. da Fundação da Criança e da Família Cidadã (FUNCI),
D) Não fiz boa prova porque não estudei. Glória Diógenes, a diferença dessa ação das que vinham
E) Quero saber o porquê de tanta demora. sendo executadas no estado é que, agora, os agentes
que integram a rede de atendimento e proteção às vítimas
EMPREGO DA CRASE de exploração sexual receberão uma assistência
6) Falhou o emprego da crase em: especializada e voltada para a articulação, o diagnóstico e
A) Fez grandes elogios a nossa casa e à sua. o encaminhamento.
B) Roberta chagará lá para às dez horas. Fortaleza é a oitava cidade do País a integrar o
C) A prova será das treze às quinze horas. PAIR. Até o momento, esclareceu Glória, o município não
D) Fale-me da cidade à qual você se referiu. possuía uma política pública eficaz na prevenção e no
E) Àquela hora já não havia mais ninguém na igreja. combate ao problema. “Até assumirmos a prefeitura, o
Estado fazia ações isoladas e o município, inerte, não
CONCORDÂNCIA NOMINAL investia nessa problemática. Agora vamos agir de forma
7) Observe com atenção as frases abaixo e marque eficaz, operacionalizando recursos e pessoal, de maneira
aquela em que há erro de concordância: a garantir a consolidação de resultados duradouros”,
A) Há menos alunos hoje aqui. explicou Glória Diógenes.
B) As filhas são tais qual a mãe. 01. O objetivo central do texto é:
C) Vocês agiram certos naquele caso. A) mostrar que há uma ação que envolve a administração
D) Há moradores que não estão quites com o condomínio. pública para o combate à exploração sexual infanto-
E) Esses assuntos parecem muito interessantes. juvenil.
B) denunciar o descaso da população e dos poderes
Gabarito federal, estadual e municipal para o problema da
01 02 03 04 05 06 07 exploração sexual infanto-juvenil.
D B A E A B C C) dizer os nomes das pessoas que estão participando do
combate à exploração sexual infanto-juvenil no
território brasileiro.

www.editoradince.com Acesse e veja se há novidades a respeito deste material


48 LÍNGUA PORTUGUESA - TEORIA E QUESTÕES
D) revelar o montante de dinheiro que irá ser empregado D) Os agentes receberão...
para o combate à exploração sexual infanto-juvenil no
Estado do Ceará. 08. Está na voz passiva o verbo da frase:
A) Uma ação integrada iniciará uma nova fase no
02. É verdadeiro concluir do texto: combate à exploração sexual infanto-juvenil no
A) Fortaleza é a oitava maior cidade do Brasil, com Estado.
relação à população e à extensão territorial. B) O convênio faz parte de uma ação integrada entre o
B) o convênio foi assinado pelos ministros do Governo Federal, os estados e o município.
Desenvolvimento Social, da Educação e da Saúde. C) O termo do acordo foi assinado pela prefeita de
C) a presidente da FUNCI informou que os agentes Fortaleza, Luizianne Lins.
envolvidos no trabalho de proteção às vítimas terão D) O município não possuía uma política pública eficaz na
uma orientação especializada. prevenção e no combate ao problema.
D) a quantia de R$ 350.000,00 será utilizada na
capacitação das polícias federal, estadual e municipal. 09. A alteração da frase original do texto fica correta em:
A) A assinatura do convênio aconteceu, durante à noite,
03. A correspondência entre a expressão e o sentido no Restaurante Estoril, na Praia de Iracema.
está verdadeira na alternativa: B) Os convênios prevêm a liberação inicial de uma verba
A) “através do” → meio no valor de R$ 350 mil.
B) “como” → comparação C) Segundo a presidente da FUNCI, Glória Diógenes, os
C) “aproximadamente” → lugar agentes serão capacitados.
D) “Segundo” → numeração D) Representando o estado, esteve à coordenadora
estadual do Núcleo de Enfrentamento, Maria de
04. A acentuação gráfica, a ortografia, o emprego do Lourdes.
sinal indicativo de crase e a pontuação da frase
estão corretos em: 10. Considere os itens seguintes:
A) Em relação as cidades do interior cearence, por I. Os agentes penitenciários haviam deslocado-se para o
exemplo, Ipú e Acarau, está havendo um trabalho de presídio.
conscientização. II. Há menas confusão na rua.
B) Em relação às cidades do interior cearense, por III. Cada um dos agentes prestarão juramento.
exemplo, Ipu e Acaraú, está havendo um trabalho de IV. Todos os agentes devem assistir ao hasteamento da
conscientização. bandeira.
C) Em relação as cidades do interior cearense, por Marque a alternativa verdadeira.
exemplo, Ipu e Acarau, estar havendo um trabalho de A) Em I, está correta a colocação do pronome oblíquo
concientização. átono.
D) Em relação às cidades do interior cearence, por B) Em II, está correta a concordância nominal.
exemplo, Ipú e Acaraú, estar havendo um trabalho de C) Em III, está correta a concordância verbal.
concientização.
D) Em IV, está correta a regência verbal.

05. Como “exploração” e “explicou”, estão corretamente Gabarito: 01/A; 02/C; 03/A; 04/B; 05/D; 06/B; 07/D; 08/C;
escritas com X: 09/C; 10/D
A) exborrachar e exbranquiçar.
B) excoltar e excoriação.
ANOTAÇÕES:
C) exquisito e extender.
___________________________________________________
D) expandir e explosão.
___________________________________________________
___________________________________________________
06. O pronome de tratamento apropriado para o Ministro ___________________________________________________
da Justiça é: ___________________________________________________
A) Vossa Eminência. ___________________________________________________
C) Vossa Magnificência. ___________________________________________________
B) Vossa Excelência. ___________________________________________________
D) Vossa Senhoria. ___________________________________________________
___________________________________________________
___________________________________________________
07. Está empregada no tempo futuro a forma verbal da
___________________________________________________
passagem:
___________________________________________________
A) A assinatura do convênio, inédito em Fortaleza, ___________________________________________________
aconteceu... ___________________________________________________
B) O convênio faz parte... ___________________________________________________
C) O convênio prevê a liberação...

www.editoradince.com Acesse e veja se há novidades a respeito deste material


DIREITOS HUMANOS - Teoria esquematizada, dicas e questões de concursos 1
Os direitos humanos objetivam a proteção dos
DIREITOS HUMANOS direitos da pessoa humana independentemente de
Teoria esquematizada, dicas e questões qualquer condição, tanto no plano interno quanto no
de concursos. plano internacional. Por um lado os direitos humanos
constituem assunto afeto ao Direito Constitucional
Prof. Valdeci Cunha
(proteção interna), por outro, constituem assunto de
Direito Internacional Público (proteção internacional).
CONTEÚDO PROGRAMÁTICO:
O conceito de direitos humanos. ...................................... 1
Noções gerais sobre Direitos Humanos. .......................... 2
Questões de concursos .......................................... 11
A evolução histórica dos Direitos Humanos. .................. 12
Questões de concursos .......................................... 17
Os direitos humanos na Organização das Nações
Unidas. ..................................................................... 18
Os direitos humanos na Organização dos Estados
Americanos. .............................................................. 19
Questões de concursos .......................................... 31
A Declaração Universal dos Direitos Humanos. ............. 31
Questões de concursos .......................................... 33
A Corte Interamericana de Direitos Humanos. ............... 34
Questões de concursos .......................................... 35 Terminologia:
A natureza jurídica da incorporação de normas Direitos do HOMEM: direitos
internacionais. .......................................................... 36 inerentes a todos os seres humanos
(Ex.: vida, igauldade, liberdade,
Regras mínimas das Nações Unidas para proteção de
jovens privados de liberdade. ................................... 38 segurança pessoal...
Questões de concursos .......................................... 45 Direitos Direitos FUNDAMENTAIS: direitos
Individuais positivados em uma Constituição
Regras mínimas das Nações Unidas para a (ou Carta Política)
Administração da Justiça da Infância e da Juventude
– Regras de Beijing. ................................................. 45 Direitos HUMANOS: direitos
positivados em tratados de direito
Questões de concursos .......................................... 49
humanos.
Convenções da ONU sobre os direitos da criança. ........ 35
Historicamente, os direitos humanos ratificam o
Questões de concursos .......................................... 49 desenvolvimento da dignidade da pessoa humana
Questões de provas passadas ..................................... 59 consistindo no principal direito fundamental fixado em
quatro valores interdependentes:
a) Liberdade;
O CONCEITO DE DIREITOS HUMANOS b) Igualdade;
Conceito: c) Segurança Pessoal; e
Direitos Humanos são os direitos fundamentais d) Solidariedade ou Fraternidade.
da pessoa humana. No regime democrático, toda Estes quatro valores supracitados prolongam a
pessoa deve ter a sua dignidade respeitada e a sua vida social dos indivíduos dando verdadeira dignidade à
integridade protegida, independentemente da origem, pessoa humana.
raça, etnia, gênero, idade, condição econômica e social,
orientação ou identidade sexual, credo religioso ou A dignidade é algo inerente a todo ser humano e
convicção política. dele não se desprende em nenhuma situação ou
hipótese, sempre dotada de universalidade e
São aqueles direitos inerentes à pessoa indivisibilidade.
humana, que visam resguardar a sua integridade física e
psicológica perante seus semelhantes e perante o Estado A ideia base que já se tinha, até mesmo no
em geral. De forma a limitar os poderes das autoridades, pensamento clássico, é que a dignidade da pessoa
garantindo, assim, o bem estar social através da humana era uma qualidade intrínseca do indivíduo
igualdade, fraternidade e da proibição de qualquer irrenunciável e inalienável, constituindo elemento que
espécie de discriminação. qualifica o ser humano como tal e dele não pode ser
separado, de tal sorte que não se pode cogitar na
Normalmente o conceito de direitos humanos possibilidade de determinada pessoa ser titular de uma
tem a ideia também de liberdade de pensamento e de pretensão a que lhe seja concedida dignidade. Esta,
expressão, e a igualdade perante a lei. portanto, como qualidade integrante e irrenunciável da
A expressão "Direitos Humanos" designa os própria condição humana.
"direitos fundamentais", dos quais os demais direitos são Caso exista universalidade em relação à
decorrência. Assim, na verdade, os Direitos Humanos não terminologia dos direitos humanos, este é o caminho da
são um ramo a mais do Direito, como o Direito Penal, o dignidade da pessoa humana. Pode-se dizer que esta
Direito Comercial etc. Os Direitos Humanos são a raiz de dignidade não se diferencia em nenhum lugar do mundo
todos os direitos. e, portanto, é dotada de universalidade, sendo esta uma
das principais características dos direitos humanos já em

www.editoradince.com - Acesse e veja se há novidades a respeito deste material – CUIDADO: cópia é crime.
2 D DIREITOS HUMANOS - Teoria esquematizada, dicas e questões de concurso
sua concepção contemporânea, ou seja, a que
conhecemos nos dias atuais.
Kant ensina que o ser humano está acima de tudo
e que nele se centra o sentido de todo o sentido da
existência e afirmando a qualidade peculiar e
insubstituível deste ser, diz-nos também que no reino dos
fins tudo tem um preço ou uma dignidade. Quando uma
coisa tem preço, pode pôr-se em vez dela qualquer outra
coisa equivalente; mas quando uma coisa está cima de
todo o preço e, portanto não permite equivalente, então
ela tem dignidade.
O conceito de direitos humanos teve seu
crescimento histórico de maneira sucessiva e gradual. Em
verdade, a noção de direitos humanos expandiu-se
historicamente.
CARACTERÍSTICAS DOS DIREITOS HUMANOS
É possível entender os direitos humanos em
dimensões ou gerações sucessivas que foram se
Algumas das características mais importantes
acumulando e aglutinado umas às outras, sendo a dos direitos humanos são:
primeira tratando sobre direitos referentes às liberdade  Os direitos humanos são fundados sobre o
públicas (direitos civis e políticos), sucedendo-se de outra respeito pela dignidade e o valor de cada
relacionada à igualdade (direitos sociais, econômicos e pessoa;
culturais) e uma terceira, vinculada à solidariedade ou  Os direitos humanos são universais, o que
fraternidade (direito ao desenvolvimento, à paz e à quer dizer que são aplicados de forma igual
autodeterminação dos povos, por exemplo).
e sem discriminação a todas as pessoas;
SE LIGA: Os direitos humanos não são uma  Os direitos humanos são inalienáveis, e
informação ou um dado, mas uma invenção, uma ninguém pode ser privado de seus direitos
construção humana, em permanente processo de
construção e reconstrução.
humanos; eles podem ser limitados em
situações específicas. Por exemplo, o
Consoante a lição do excelso mestre italiano direito à liberdade pode ser restringido se
Norberto Bobbio, na obra, A Era dos Direitos, ―os direitos
uma pessoa é considerada culpada de um
humanos nascem como direitos naturais universais,
desenvolvem-se como direitos positivos particulares crime diante de um tribunal e com o devido
(quando cada Constituição incorpora Declarações de processo legal;
Direitos), para finalmente encontrarem sua plena  Os direitos humanos são indivisíveis, inter-
realização como direitos positivos universais‖. relacionados e interdependentes, já que é
Este processo de internacionalização dos insuficiente respeitar alguns direitos
direitos humanos constitui um movimento muito recente humanos e outros não. Na prática, a
na história, emergindo no pós 2ª Guerra Mundial, como violação de um direito vai afetar o respeito
resposta dos povos aos crimes, atrocidades e horrores por muitos outros;
praticados pelo nazismo. Nas palavras de Flávia  Todos os direitos humanos devem,
Piovesan, ―Se a 2ª Guerra significou a ruptura com os
direitos humanos, o Pós-Guerra deveria significar a sua
portanto, ser vistos como de igual
reconstrução. É neste cenário que se desenha o esforço importância, sendo igualmente essencial
de reconstrução dos direitos humanos, como paradigma e respeitar a dignidade e o valor de cada
referencial ético a orientar a ordem internacional pessoa.
contemporânea. Fortalece-se a ideia de que a proteção
dos direitos humanos não deve se reduzir ao domínio
reservado do Estado, isto é, não deve se restringir à
competência nacional exclusiva ou à jurisdição doméstica NOÇÕES GERAIS SOBRE DIREITOS
exclusiva, porque revela tema de legítimo interesse HUMANOS.
internacional.‖
Para uma melhor compreensão acerca do assunto,
faz-se necessário inicialmente diferenciarmos direitos do
DISTINÇÃO DE DIREITOS HUMANOS DOS DIREITOS homem, direitos humanos e direitos fundamentais:
FUNDAMENTAIS
Direitos do homem - são aqueles direitos inatos
O que distingue os Direitos Humanos ou Direitos da pessoa humana. São direitos não positivados (não
Fundamentais de outras formas de ordenamento jurídico escritos).
é que, sendo o Direito fundamentado nos Direitos
intrínsecos do Homem, este só pode ter como fonte a Direitos fundamentais - são os direitos do
liberdade, estando o ser humano sujeito apenas à lei e a homem positivados (escritos) numa Constituição.
não à prepotência e à astúcia de um chefe ou de quem Direitos humanos - conotam aqueles direitos
quer que seja. De acordo com o que reza o artigo 5º da escritos/positivados em tratados e convenções
nossa Constituição, os Direitos Fundamentais são o internacionais específicos de direitos humanos, quer no
direito à vida, à liberdade, à igualdade, à segurança e à plano global (ONU), quer nos contextos regionais (dentre
propriedade. os quais o interamericano).

www.editoradince.com - Acesse e veja se há novidades a respeito deste material – CUIDADO: cópia é crime.
DIREITOS HUMANOS - Teoria esquematizada, dicas e questões de concursos 3
HISTÓRIA DOS DIREITOS HUMANOS Com a Idade Moderna, os racionalistas dos
Um dos documentos mais antigos que se séculos XVII e XVIII, reformulam as teorias do direito
vinculam aos direitos humanos é o Cilindro de Ciro, natural, deixando este de estar submetido a uma ordem
que contém uma declaração do rei persa Ciro II depois de divina. Para os racionalistas, todos os homens são, por
sua conquista da Babilônia em 539 a.C. Foi descoberto natureza, livres, e têm certos direitos inatos de que não
em 1879 e a Organização das Nações Unidas o traduziu podem ser despojados quando entram em sociedade. Foi
em 1971 a todos os seus idiomas oficiais. Pode ser esta corrente de pensamento que acabou por inspirar o
resultado de uma tradição mesopotâmica centrada na atual sistema internacional de proteção dos direitos do
figura do "rei justo", cujo primeiro exemplo conhecido é o homem.
rei Urukagina, de Lagash, que reinou durante o século A evolução destas correntes veio a dar frutos pela
XXIV a.C. Cabe destacar, também, nessa tradição, primeira vez na Inglaterra, e depois nos Estados Unidos.
Hamurabi da Babilônia e seu famoso Código de A Magna Carta (1215) deu garantias contra a
Hamurabi, que data do século XVIII a.C. O Cilindro de arbitrariedade da Coroa, e influenciou diversos
Ciro apresentava características inovadoras, documentos, como, por exemplo, o Habeas
especialmente em relação à religião. Nele, era declarada Corpus (1679), que foi a primeira tentativa para impedir
a liberdade de religião e a abolição da escravatura. Tem as detenções ilegais. A Declaração Americana da
sido valorizado positivamente por seu sentido humanista e Independência surgiu a 4 de Julho de 1776: nela,
inclusive foi descrito como a primeira declaração de constavam os direitos naturais do ser humano que o
direitos humanos. poder político deve respeitar. Esta declaração teve, como
Na Roma antiga, havia o conceito jurídico da base, a Declaração de Virgínia proclamada a 12 de Junho
concessão da cidadania romana a todos os romanos. de 1776, onde estava expressa a noção de direitos
O cristianismo, durante a Idade Média, foi a afirmação da individuais.
defesa da igualdade de todos os homens numa mesma Durante a Revolução Inglesa, a burguesia
dignidade. Foi também durante esta época que conseguiu satisfazer suas exigências de ter alguma
os filósofos cristãos recolheram e desenvolveram a teoria classe de seguridade contra os abusos da coroa e limitou
do direito natural, em que o indivíduo está no centro de o poder dos reis sobre seus súditos, proclamando a Lei
uma ordem social e jurídica justa, mas a lei divina tem de Habeas corpus em 1679. Em 1689, o Parlamento
prevalência sobre o direito laico tal como é definido pelo impôs, a Guilherme III de Inglaterra, na Carta de
imperador, o rei ou o príncipe. Logo, foram criadas muitas Direitos (ou Declaração de direitos), uma série de
teorias no decorrer do tempo. princípios sobre os quais os monarcas não podiam
Documentos posteriores, como a Carta Magna legislar ou decidir.
da Inglaterra, de 1215, e a Carta de Mandén, de 1222, No século XVII e XVIII, filósofos europeus,
se têm associado também aos direitos humanos. Os destacando-se John Locke, desenvolveram o conceito do
direitos humanos ou coletivos foram forjados ao longo da direito natural. Os direitos naturais, para Locke, não
história, através de debates realizados por filósofos dependiam da cidadania nem das leis de um Estado, nem
e juristas. estavam necessariamente limitadas a um grupo étnico,
A conquista da América no século XVI pelos cultural ou religioso em particular. A teoria do contrato
espanhóis resultou em um debate sobre direitos humanos social, de acordo com seus três principais formuladores, o
na Espanha. Isto marcou a primeira vez que se discutiu o já citado Locke, Thomas Hobbes e Jean-Jacques
assunto na Europa. Rousseau, se baseia em que os direitos do indivíduo são
naturais e que, no estado de natureza, todos os homens
Muitos filósofos e historiadores do direito são titulares de todos os direitos.
consideram que não se pode falar de direitos humanos
até a modernidade no Ocidente. Até então, as normas da A primeira declaração dos direitos humanos da
comunidade, concebidas na relação com a ordem época moderna é a Declaração de Direitos de Virgínia
cósmica, não deixavam espaço para o ser humano como de 12 de junho de 1776, escrita por George Mason e
sujeito singular, concebendo-se o direito primariamente proclamada pela Convenção da Virgínia. Esta medida
como a ordem objetiva da sociedade. A sociedade influenciou Thomas Jefferson na declaração dos direitos
estamental tem seu centro em grupos como a família, humanos que existe na Declaração da Independência dos
a linhagem ou as corporações profissionais ou laborais, o Estados Unidos da América de 4 de julho de 1776, assim
que implica que não se concebem faculdades próprias do como também influenciou a Assembleia Nacional francesa
ser humano enquanto tal. Pelo contrário, se entende que em sua declaração, a Declaração dos Direitos do Homem
toda faculdade atribuível ao indivíduo deriva de um e do Cidadão de 1789. Esta definia o direito individual.
duplo status: o do sujeito no seio da família e o desta na A Declaração dos Direitos do Homem e do Cidadão,
sociedade. "Fora do Estado, não há direitos". proclamada na França em 1789, e as reivindicações ao
longo dos séculos XIV e XV em prol das liberdades,
A existência dos direitos subjetivos, tal e como se alargaram o campo dos direitos humanos e definiram os
pensam na atualidade, será objeto de debate durante os direitos econômicos e sociais.
séculos XVI, XVII e XVIII, o que é relevante porque
habitualmente se diz que os direitos humanos são produto A noção de direitos humanos não experimentou
da afirmação progressiva da individualidade e que a ideia grandes mudanças até o século seguinte. Com o início
de direitos do homem apareceu pela primeira vez durante das lutas operárias, surgiram novos direitos que
a luta burguesa contra o sistema do Antigo Regime. pretendiam dar solução a determinados problemas sociais
Sendo esta a consideração mais estendida, outros através da intervenção do Estado. Neste processo, são
autores consideram que os direitos humanos são uma importantes a Revolução Russa e a Revolução Mexicana.
constante na História e têm suas raízes no mundo Desde o nascimento da Organização das
clássico; também sua origem se encontra na afirmação do Nações Unidas em 1945, o conceito de direitos humanos
cristianismo da dignidade moral do homem enquanto se tem universalizado, alcançando uma grande
pessoa. importância na cultura jurídica internacional. Em 10 de
dezembro de 1948, a Declaração Universal dos Direitos
Humanos foi adotada e proclamada pela Assembleia

www.editoradince.com - Acesse e veja se há novidades a respeito deste material – CUIDADO: cópia é crime.
4 D DIREITOS HUMANOS - Teoria esquematizada, dicas e questões de concurso
Geral das Nações Unidas em sua Resolução 217 A (III),
como resposta aos horrores da Segunda Guerra RESUMO DA EVOLUÇÃO HISTÓRICA DO DH:
Mundial e como intento de assentar as bases da nova
ordem internacional que surgia depois do armistício. A formação do rol de normas de direitos humanos
confunde-se com a história da humanidade e é produto de
Mas o momento mais importante, na história dos diversas origens, que podem ser localizadas em
Direitos do Homem, é durante 1945-1948. Em 1945, diferentes civilizações e que se apoiam nos mais variados
os Estados tomam consciência das tragédias e fundamentos. Confira-se:
atrocidades vividas durante a 2ª Guerra Mundial, os
levando a criar a Organização das Nações Unidas (ONU)
em prol de estabelecer e manter a paz no mundo. Foi Antiguidade:
através da Carta das Nações Unidas, assinada a 20 de  O Código de Hamurábi (1690 a. C)
Junho de 1945, que os povos exprimiram a sua consagrava a todos os indivíduos direitos como a vida a
determinação "em preservar as gerações futuras do propriedade e a honra.
flagelo da guerra; proclamar a fé nos direitos
fundamentais do Homem, na dignidade e valor da pessoa
 O Povo Judeu, nos Dez Mandamentos, definia
humana, na igualdade de direitos entre homens e normas relativas à proteção à vida (―não matarás‖), ao
mulheres, assim como das nações, grande e pequenas; direito de propriedade (―não roubarás‖), à proteção da
em promover o progresso social e instaurar melhores família (―não cometerás adultério‖) e da honra (―não darás
condições de vida numa maior liberdade". A criação das falso testemunho‖).
Nações Unidas simboliza a necessidade de um mundo de  Na Grécia Antiga, fazia-se alusão a um Direito
tolerância, de paz, de solidariedade entre as nações, que natural anterior ao indivíduo e superior a suas leis e
faça avançar o progresso social e económico de todos os valores como a liberdade, a igualdade e a
povos. participação política.
Os principais objetivos das Nações Unidas,  Em Roma, a Lei das Doze Tábuas também
passam por manter a paz, a segurança internacional, conferir direitos como a igualdade e a propriedade aos
desenvolver relações amigáveis entre as nações, realizar cidadãos romanos.
a cooperação internacional resolvendo problemas
Mas atente: nesse momento histórico, era traço
internacionais do cariz econômico, social, intelectual e
comum a praticamente todos os povos o fato de
humanitário, desenvolver e encorajar o respeito pelos
que os estrangeiros não faziam jus aos mesmos
direitos humanos e pelas liberdades fundamentais sem
direitos. A mudança veio com a DOUTRINA
qualquer tipo de distinção.
CRISTÃ, que não só veio a reiterar e acrescentar
Assim, a 10 de Dezembro de 1948, a Assembleia novos valores, como também avançar
Geral das Nações Unidas proclamou a Declaração enfaticamente na consagração da universalidade
Universal dos Direitos Humanos. que é inerente aos direitos humanos.
A Declaração Universal dos Direitos Humanos é
fundamental na nossa Sociedade pois quase todos os
documentos relativos aos direitos humanos têm, como Idade média:
referência, esta Declaração, a qual alguns Estados fazem  A Magna Carta, outorgada pelo Rei João Sem
referência direta nas suas constituições nacionais. Terra, da Inglaterra, em 1215, é um marco importante, ao
A Declaração Universal dos Direitos Humanos limitar os poderes do monarca inglês frente aos membros
ganhou uma importância extraordinária, contudo não da nobreza que, em contrapartida, adquiriam certos
obriga juridicamente que todos os Estados a respeitem e, direitos, como a liberdade de locomoção, o livre acesso à
devido a isso, a partir do momento em que foi justiça e certa proteção na área tributária.
promulgada, foi necessário a preparação de inúmeros  O Bill of Rights, de 1689, avançaria na
documentos que especificassem os direitos presentes na garantia de direitos e na limitação do poder estatal, fator
declaração e, assim, forçassem os Estados a cumpri-la. estreitamente relacionado com a proteção dos direitos
Foi nesse contexto que, no período entre 1945-1966, humanos.
nasceram vários documentos, entre os quais se destacam
os Pactos Internacionais de Direitos Humanos de 1966.
Idade moderna e idade contemporânea:
Assim, a junção da Declaração Universal dos
Direitos Humanos, os dois pactos efetuados em 1966,  O ideário iluminista marcou a Independência
nomeadamente o Pacto Internacional dos Direitos Civis e Americana, em 1776, e alguns dos principais documentos
Políticos e o Pacto Internacional dos Direitos Econômicos, relacionados com esse fato, a exemplo da Declaração de
Sociais e Culturais, bem como os dois protocolos Direitos do Bom Povo da Virgínia, de 1775, e
facultativos do Pacto dos Direitos Civis e Políticos (que, a Constituição dos EUA, de 1787.
em 1989, aboliu a pena de morte), constituem a Carta  A Revolução Francesa também foi guiada pelo
Internacional dos Direitos do Homem. ideário iluminista e vio a consagrar diversos direitos da
Em 2016, no entanto, o secretário-geral da Anistia pessoa em documentos como a Declaração dos Direitos
Internacional, Salil Shetty, no relatório anual da do Homem e do Cidadão, de 1789, e as Constituições
organização, questionou a eficiência do atual sistema de de 1791 e 1793, que reconheceram expressamente a
proteção dos direitos humanos no mundo, segundo ele liberdade e a igualdade inerentes ao ser humano.
incapaz de resolver problemas como a crise migratória na
 A partir da segunda metade do século XIX, a
Europa, a perseguição aos defensores dos direitos
preocupação com os direitos humanos passa a abranger
humanos, as violências sexual e de gênero,
as questões sociais, emergindo ideários como
a impunidade, o aliciamento de crianças por organizações o Marxismo.
criminosas, a tortura, a violência policial, a manutenção
da pena de morte em alguns países e as remoções  Também no século XIX, a difusão de valores
forçadas de populações. humanistas leva ao fortalecimento da preocupação com a

www.editoradince.com - Acesse e veja se há novidades a respeito deste material – CUIDADO: cópia é crime.
DIREITOS HUMANOS - Teoria esquematizada, dicas e questões de concursos 5
regulamentação da guerra, com vistas a diminuir seu  Indivisibilidade, interdependência e
impacto negativo sobre a vida humana. É quando surge complementaridade: Os direitos humanos são
o direito humanitário. indivisíveis, interdependentes e complementares.
 O início do século XX foi marcado por uma maior  Primazia da norma mais favorável: Os
preocupação social. Após a I Guerra, surgem as primeiras direitos humanos não podem ser empregados para
organizações internacionais que atribuíram relevância à eliminar direitos ou para justificar a inobservância de um
proteção dos direitos humanos: a Liga das Nações e direito. Diante d e um conflito entre duas normas de
a Organização Internacional do Trabalho (OIT). direitos humanos, deve ser aplicada aquela que melhor
 Após a II Guerra Mundial, os direitos humanos proteja a dignidade humana.
adquirem o caráter de prioridade da sociedade ATENÇÃO! Esse princípio é consagrado no art.
internacional, mormente a partir da criação da ONU 5º do Pacto dos Direitos Civis e Políticos, que
(1945) e da proclamação da Declaração Universal determina que ―2. Não se admitirá qualquer
dos Direitos Humanos (1948). Ela consiste em uma restrição ou suspensão dos direitos humanos
mera resolução da ONU e que, nesse sentido, não é fundamentais reconhecidos ou vigentes em
tecnicamente um tratado e não teria, a princípio, forca qualquer Estado-parte no presente Pacto em
vinculante. Esse período pós-II Guerra é caracterizado virtude de leis, convenções, regulamentos ou
pela abrangente positivação. costumes, sob pretexto de que o presente Pacto
 Atualmente, em decorrência da complexidade da não os reconheça ou os reconheça em menor
vida social, o escopo dos direitos humanos aumentou grau‖.
sensivelmente, abrangendo inclusive outras áreas, como
 Essa norma também é reiterada pelo art. 29 da
o meio ambiente e o comércio.
Convenção Americana de Direitos Humanos (Pacto de
São José):
FUNDAMENTO DOS DH: Art. 29. Nenhuma disposição desta Convenção
São três as principais teorias: pode ser interpretada no sentido de:
 Teoria JUSNATURALISTA (PRINCIPAL): Os a) permitir a qualquer dos Estados-Partes,
direitos humanos se fundamentam em uma ordem grupo ou pessoa, suprimir o gozo e exercício dos
superior, universal, imutável e inderrogável; direitos e liberdades reconhecidos na Convenção
ou limitá-los em maior medida do que a nela
 Teoria POSITIVISTA: Alicerça tais direitos na prevista;
ordem jurídica posta, pelo que somente seriam
reconhecidos como direitos humanos aqueles positivados. b) limitar o gozo e exercício de qualquer direito
ou liberdade que possam ser reconhecidos de
 Teoria MORALISTA (DE PERELMAN): acordo com as leis de qualquer dos Estados-
Fundamenta os direitos humanos na ―experiência e Partes ou de acordo com outra convenção em que
consciência moral de um determinado povo‖, ou seja, na seja parte um dos referidos Estados;
convicção social acerca da necessidade da proteção de c) excluir outros direitos e garantias que são
determinado valor. inerentes ao ser humano ou que decorrem da
forma democrática representativa de governo; e
CARACTERÍSTICAS DO DH: d) excluir ou limitar o efeito que possam
 Universalidade: Os direitos humanos se produzir a Declaração Americana dos Direitos e
referem a todos os membros da espécie humana, sem Deveres do Homem e outros atos internacionais
distinção. da mesma natureza.
 Inerência: Os direitos humanos pertencem a  Caráter não exaustivo da lista de fatores
todos os indivíduos pela simples circunstância de serem de discriminação: A lista de fatores de discriminação
pessoas humanas. Em suma, basta a condição de ser apresentadas nas normas de direitos humanos não têm
pessoa humana. caráter exaustivo. Ex.: é proibida a discriminação de cor,
sexo, língua, religião etc. (rol exemplificativo).
 Transnacionalidade: Os direitos humanos
pertencem à pessoa independentemente de sua
nacionalidade ou do fato de ser apátrida. FONTES DO DH:
 Historicidade e proibição de retrocesso: As fontes se dividem em duas classes:
Não configuram uma pauta fixa e estática, definida em um  Fontes materiais: São os fatos sociais e deias
único momento da história. Ao revés, há um catálogo políticas.
aberto a novos direitos. Mas veja: a noção de
 Fontes formais: São iguais às fontes do DIP
historicidade dos direitos humanos não comporta a
(tratados, costumes, jurisprudência, doutrina,
possibilidade de que as normas que consagram certos
resoluções, soft law etc.).
direitos desapareçam do ordenamento jurídico ou tenham
seu escopo de proteção reduzido. Vigora a proibição do
retrocesso. CLASSIFICAÇÃO DOS DIREITOS HUMANOS -
 Indisponibilidade, inalienabilidade e GERAÇÕES
irrenunciabilidade: Os direitos humanos são Em 1979, em uma conferência do Instituto
indisponíveis, inalienáveis e irrenunciáveis. Internacional de Direitos Humanos, Karel Vasak propôs
uma classificação dos direitos humanos em gerações,
 Imprescritibilidade: Os direitos humanos são inspirado no lema da Revolução Francesa (liberdade,
imprescritíveis. Mas atente: o Brasil não assinou a
igualdade, fraternidade).
Convenção sobre a Imprescritibilidade dos Crimes de
Guerra e dos Crimes contra a Humanidade.

www.editoradince.com - Acesse e veja se há novidades a respeito deste material – CUIDADO: cópia é crime.
6 D DIREITOS HUMANOS - Teoria esquematizada, dicas e questões de concurso
Classificação tradicional: a doutrina divide os direitos jurídica. As regras têm teor mais concreto e específico,
humanos em gerações: pautando mais diretamente o comportamento humano.
 Primeira geração: Direitos civis e políticos, ou
direitos de liberdade. Afirma-se a partir de ideais PRINCIPAIS MECANISMOS(INSTRUMENTOS) DE
iluministas e liberais em voga nos séculos XVIII e XIX e DIREITOS HUMANOS
dos movimentos político-sociais da descolonização da
Internacionais
América Latina. Tais direitos são oponíveis contra o
Estado.  Declaração Universal dos Direitos Humanos
(Aprovada Pela Assembleia Geral das Nações
 Segunda geração: Refere-se aos direitos
Unidas em 10/12/48)
econômicos, sociais e culturais. São também conhecidos
como ―direitos de igualdade‖. Relaciona-se com as  Pacto Internacional dos Direitos Civis e
consequências negativas da Revolução Industrial e do Políticos (Ratificado pelo Brasil em
liberalismo sobre significativos contingentes humanos. 24/01/1992)
Exigem do Estado prestações positivas.  Pacto Internacional dos Direitos
 Terceira geração: São os ―direitos de Econômicos, Sociais, Culturais e
fraternidade‖, de caráter difuso, que não se distinguem Ambientais (Ratificado pelo Brasil em
especificamente a um indivíduo ou a um grupo social, 24/01/1992)
mas ao próprio gênero humano como um todo. Ex.: direito  Pacto de San José ou Convenção
ao meio ambiente, à comunicação e ao patrimônio comum Americana Sobre Direito Humanos (22/11/69
da humanidade. – Promulgada pelo Brasil em 06/11/92)
 Quarta geração: PAULO BONAVIDES defende
a existência de uma quarta geração dos direitos humanos, Nacional
adequada ao período da globalização e à formação de um  Constituição Federal (Artigo 5º e Incisos)
mundo marcado por fronteiras nacionais mais
permeáveis. GLOBALIZAÇÃO.
 Quinta geração: Paulo Bonavides defende 1) Declaração Universal dos Direitos Humanos
ainda a existência de uma quinta geração de direitos Não há como há como falar em direitos humanos
humanos, preocupada com a PAZ MUNDIAL. sem se remeter ao mais importante e universal dos
documentos que tratam do tema, já feito pela
OBS.: para alguns autores, a caracterização dos
humanidade. Como o nome bem diz uma declaração,
direitos humanos em gerações fere a
assinada, após a Segunda Guerra Mundial, pela maioria
indivisibilidade e a interdependência desses
dos povos do planeta. O primeiro documento, dessa
direitos, gerando uma visão fragmentária e
natureza, com caráter universal. Neste site a mesma está
hierarquizada.
em linguagem de cordel.
A Declaração Universal dos Direitos Humanos foi
Classificação conforme o Direito Internacional dos produto do medo que o homem causou a si mesmo com
Direitos Humanos: as dimensões dos direitos as duas grandes guerras mundiais. Transformando-se
humanos: num documento onde direitos mínimos foram anunciados,
Com a celebração, em 1966, do Pacto direitos que garantem a dignidade humana, preservando
Internacional dos Direitos Civis e Políticos e do Paco valores básicos. FOI O PRIMEIRO ESFORÇO MUNDIAL
Internacional dos Direitos Econômicos, Sociais e PARA CONSTRUÇÃO DE UMA REALIDADE ONDE
Culturais, a doutrina passou a classificar os direitos PRINCÍPIOS BÁSICOS DA CIVILIZAÇÃO FORAM
econômicos em apenas duas dimensões: ANUNCIADOS.
 1ª DIMENSÃO: DIREITOS CIVIS E POLÍTICOS; Todavia como o próprio nome diz era apenas uma
 2ª DIMENSÃO: DIREITOS ECONÔMICOS, declaração, não uma lei. Assim, resumia-se numa carta
SOCIAIS E CULTURAIS. de intenção, sendo importante o seu significado, a
 3ª DIMENSÃO: Carlos Weis trata ainda da iniciativa da qual resultou, sem dúvida um passo
dimensão adicional dos direitos humanos, os fundamental e histórico. Mas sem punição para as nações
―direitos globais‖, que corresponderiam aos que desrespeitassem os direitos ali previstos.
direitos de terceira geração. Aprovada na Assembleia Geral das Nações
O termo ―dimensão‖ é mais adequado para compor Unidas, em 10 de dezembro de 1948.
uma classificação dos direitos humanos, visto que a Há quem pregue a necessidade de se fazer uma
expressão ―geração‖ pode induzir a erro, dando a nova declaração universal, tendo-se conta que muitos
entender que tais direitos se substituem ao longo da povos, não são signatários da atual declaração, além de
história, o que não é o caso. que a mesma, ter sido feita no Ocidente, respeitou
Força normativa: somente experiências, cultura e costumes ocidentais.
Na medida em que são consagrados em normas
jurídicas, internacionais ou internas, os direitos humanos 2) Pacto Internacional dos Direitos Civis e Políticos
ganham força vinculante, tornando-se modelos de
Não se poderia falar de direitos humanos sem falar
conduta obrigatórios para o Estado e para todos os
de importante pacto, feito, sobretudo, pelo Ocidente, na
membros da sociedade e cuja inobservância enseja a
busca de obrigar os países do antigo bloco comunista a
possibilidade de sanções.
respeitar direitos civis do cidadão.
As normas de direitos humanos podem funcionar
como princípios, mas também como regras. O São os principais direitos civis:
princípio é mais abstrato e genérico, orientando toda a  Direito à Igualdade
aplicação e interpretação de outras normas da ordem  Direito do Preso
 Direito de Ir e Vir

www.editoradince.com - Acesse e veja se há novidades a respeito deste material – CUIDADO: cópia é crime.
DIREITOS HUMANOS - Teoria esquematizada, dicas e questões de concursos 7
 Direito à Informação menos respeitados no mundo atual, na era da
 Direito à Opinião terceirização, sendo inúmeros os pais de família
desempregados. Muito limitado o seguro desemprego, já
 Direito de Associar-se
uma conquista, em função de tal direito ser previsto na
 Direito ao Voto Constituição.
 Acesso à Justiça A fome é um mal que aflige milhões de brasileiros,
 Direito à Liberdade de Expressão sendo notável a quantidade de subnutridos, sobretudo no
 Presunção de Inocência Nordeste brasileiro e o alto índice de mortalidade infantil,
 Proibição de Prisão por Dívida que coloca o Brasil entre os países com pior nível de vida
do mundo, piores IDH (índice de desenvolvimento
 Proibição à Tortura humano).
 Princípio do Contraditório
Antes de entrar na questão dos direitos Direitos Sociais
Econômicos, Sociais, Culturais e Ambientais, de boa
sugestão tratar, antes, de outros mecanismos, criando-se Aqui incluído o direito à aposentadoria, ao auxílio
para aqueles a parte especial, capítulo independente mais doença, ao salário maternidade, de forma tão precária
adiante: fornecidos pelo INSS, falido, mal administrado,
constantemente envolvido nas mais estranhas denúncias
de falcatruas. NÃO PAGANDO O QUE DEVE !
Pacto de Santo José - Convenção Americana sobre Incluído também, nessa parte, o respeito à pessoa
Direitos Humanos do idoso, à criança, à estabilidade da família tão
Enquanto os Pactos Internacionais e a Declaração importante para educação e formação do cidadão.
Universal valem pata todo o planeta, o Pacto de San José Inspirado em tais princípios foram criadas duas
é regional, tendo validade somente para o continente importantes leis: uma que trata da proteção ao idoso,
americano. outra que trata da Criança e do Adolescente.
O Pacto de San José foi assinado por países Todavia, nesse campo, o mais importante dos
americanos, na cidade de San José, por isso tem tal direitos, sem dúvida, O DIREITO À SAÚDE, tão negado
nome, mas conhecimento também como Convenção ao povo brasileiro, tão desrespeitado pelos hospitais, não
Americana dos Direitos Humanos. Valendo como lei entre fornecido pelo Poder Público, transformado em
os países, dela signatários. ferramenta de lucro pelos famosos e escandalosos planos
de saúde.
Constituição Brasileira No Brasil, também no Estado do Ceará, em
Por sua feita, a Constituição Brasileira, de 1988, Fortaleza... o direito à saúde inexiste, resume-se ao mais
conhecida como Constituição Cidadã, transformou em imprevisível caos, sinônimo de descaso,
princípios constitucionais vários itens constantes tanto na descompromisso, desrespeito a todas as normas mais
Declaração Universal dos Direitos Humanos, como elementares e incompetência administrativa. Em meio a
presentes no Pacto de San José. O que é muito tudo, os competentes médicos sofrem, fazendo a escolha
importante, tendo-se em conta que a Constituição de um de Sophia, pacientes se amontoam nos corredores,
país é sua lei maior, sua carta magna. enquanto a propagando oficial quer fazer crer o contrário.
Sendo interessante que o cidadão brasileiro,
conheça ao menos seu artigo 5º e seus 77 incisos, pois Direitos Culturais
neles, presentes princípios básicos importantes, O DIREITO À EDUCAÇÃO contido em tal item.
constantemente desrespeitados por autoridades Não o simples direito a ser alfabetizado, a aprender a ler.
arbitrárias, seja através de atos, seja por omissão. Não o simples direito a fazer um curso médio de formação
para ser um assalariado, mas uma educação para
Pacto Internacional dos Direitos Econômicos Sociais cidadania, para que o cidadão entenda a história, o
Culturais e Ambientais - DESCAS’S mundo, entenda-se em tal contexto e mude sua própria
história, sendo agente construtor de uma nova Sociedade.
O mais importante de todos, fazendo-se
necessário tratamento especial para assunto, que será Envolve respeito às minorias: negros, índios, bem
dividido em tópicos. Como o Pacto Internacional dos como acesso às descobertas científicas, tipo INTERNET,
Direitos Civis e Políticos, o Brasil é signatário de tal pacto, transporte, medicina etc.
que vale como lei em território brasileiro. Sem falar que Direito ainda a Ter acesso não apenas a cultura
muitos dos seus princípios, também se encontram produzida, mas a produzir, também cultura, tão importante
presentes na Constituição Brasileira. Tal pacto ficou para a identidade não só individual como coletiva.
popularmente conhecido pela sigla DESCA’S.
A vantagem de um princípio de direito humano Direitos Ambientais
constar em lei nacional ou valer como lei no Brasil é que Por fim os direitos ambientais. Vez que a ambição
uma vez desrespeitado tal princípios há mecanismos pelo lucro tem feito o homem esquecer que ele é parte e
legais de obrigar que o mesmo seja cumprido: AÇÃO depende do meio ambiente. Que destruir o meio ambiente
POPULAR – MANDADO DE SEGURANÇA – MANDADO é destruir a própria vida, sem falar que a preservação do
DE INJUNÇÃO – AÇÃO CIVIL PÚBLICA, entre outros meio ambiente é direito não apenas dos que vivem no
meios jurídicos. presente, como também das gerações futuras.
Não se justificando a caça predatória, a destruição
Direitos Econômicos louca das florestas, a morte dos rios, a poluição do ar,
Trata do direito do cidadão ao trabalho, ao tudo em nome do lucro, da riqueza de alguns. Não
alimento, a um padrão de vida digno, ao direito de podendo haver qualidade de vida digna onde o meio
sindicalizar-se, a não ser escravizado. Um dos direitos

www.editoradince.com - Acesse e veja se há novidades a respeito deste material – CUIDADO: cópia é crime.
8 D DIREITOS HUMANOS - Teoria esquematizada, dicas e questões de concurso
ambiente é desrespeitado, onde árvores e animais são 11 - Historicidade: Estão vinculados
destruídos violentamente sem nenhuma justificativa. ao desenvolvimento histórico e cultural do ser humano.
Assim, caso tais mecanismos não venham a ser 12 - Irrenunciabilidade: Os direitos humanos não
respeitados, certamente toda a humanidade caminha para podem ser objeto de renúncia.
o abismo, a humanidade certamente, que pouca 13 - Vedação do Retrocesso ou do Regresso
importância tem para o Universo, perecerá. A humanidade (efeito "Cliquet"): Uma vez estabelecidos os direitos
com tanta cultura, mas com tão pouca inteligência, na humanos, não se admite o retrocesso visando a sua
verdade, um paradoxo único e ninguém sobrará para limitação ou diminuição.
contar a história, ou melhor, a quem importaria conhecer a
14 - Efetividade: O Estado deve garantir a
história de uma espécie tão tola, irresponsável, irracional
efetivação dos direitos humanos (no mínimo, os direitos
????
civis e políticos).
15 - Limitabilidade: Os direitos humanos podem
SUPER DICAS: ser limitados em situações excepcionais previstas nas
legislações, como, por exemplo, a prisão de um individuo
Dica 3.
que cometeu um delito (limitação do direito de ir e vir). No
O sistema de proteção internacional de direitos Brasil, é possível limitar o direito de reunião nos casos de
humanos é dividida em duas partes: um sistema global estado de defesa e estado de sítio (art. 136 e 139 da CF).
(pactos e convenções internacionais da ONU) e um
sistema regional de proteção (sistema interamericano,
europeu e africano). Dica 7.
O procedimento de incorporação dos tratados
internacionais podem ser resumidos assim:
Dica 4.
Assinatura + Itamaraty + Decreto Legislativo (aprovando
A comissão interamericana de direitos humanos
art. 49,I CRFB) + Depósito + Decreto do Executivo
tem por fim promover, fiscalizar e proteger os direitos
(ratificação Art. 84, VIII CRFB)
humanos na América, podendo fazer recomendações aos
Governos.
Os membros são eleitos pela Assembleia Geral da Dica 8.
Organização. Os direitos humanos, na Constituição brasileira
recebem nome de fundamentais, e fazem parte do título II
da CR/88. Ficando localizados entre os artigos 5 e o 17.
Dica 5.
Contudo devemos ficar atentos à dica de que este rol não
A Corte interamericana de direitos humanos é taxativo admitindo outros direitos não concebidos
composta por 7 juízes nacionais dos Estados membros da dentro do título II, como por exemplo, o princípio da
OEA, eleitos a título pessoal pelos Estados-partes da anterioridade tributária (art. 150, III, b) sendo uma garantia
Convenção. fundamental à segurança. Isso só é possível graças ao
Ela tem competência consultiva (consulta sobre art. 5, §2 que prevê que ―Os direitos e garantias
interpretação de convenção ou tratados) e contenciosa e expressos nesta Constituição não excluem outros
está situada em San José da Costa Rica. decorrentes do regime e dos princípios por ela adotados,
ou dos tratados internacionais em que a República
Dica 6. Federativa do Brasil seja parte.‖
Características dos Direitos Humanos
1 - Universalidade: Todo e qualquer ser humano Dica 9.
é sujeito ativo desses direitos podendo pleiteá-los É bom alertar que estes direitos fazem parte do
em qualquer foro nacional ou internacional (Declaração e núcleo irreformável da Constituição, constituindo
Programa de Ação de Viena - 1993 - §5º). cláusulas pétreas (art. 60, §4, IV). É voz corrente que,
2 - Imprescritibilidade: Não sofrem alterações apesar do art. 60, §4, IV se referir a ―direitos e garantias
com o decurso do tempo, pois têm caráter eterno. individuais, ele se refere a todos os direitos e garantias
fundamentais.
3 - Individualidade: Podem ser exercidos por
apenas um indivíduo.
4 - Complementaridade: Os direitos humanos Dica 10.
devem ser interpretados em conjunto não havendo A cada momento histórico foram agregando
hierarquia entre eles. direitos aos já concebidos formando o conjunto de direitos
5 - Inviolabilidade: Esses direitos não podem ser humanos/fundamentais que temos atualmente.
descumpridos por nenhuma pessoa ou autoridade. Por isso, convencionou-se chamar de gerações
6 - Indisponibilidade: Esses direitos não podem ou dimensões de direitos humanos. Nesse sentido,
ser renunciados. pode-se afirmar que os direitos foram sendo conquistados
e organizados da seguinte ordem:
7 - Inalienáveis: Estão fora do comércio.
a) 1ª Dimensão ou geração (França - 1789) –
8 - Indivisibilidade: Não podem ser divididos,
Direito à prestações negativas - obrigações de
compõem um único conjunto de direitos.
não fazer impostas ao Estado, como os direitos
9 - Interdependência: Os direitos humanos estão e garantias individuais tais quais o direito à
vinculados uns aos outros. liberdade, vida, propriedade, inviolabilidade de
10 - Inter-relacionariedade: Os direitos humanos correspondência, domicílio, telefônica e assim
e os sistemas de proteção se inter- por diante.
relacionam, possibilitando às pessoas escolher entre o b) 2ª Dimensão ou geração (México 1917 e
mecanismo de proteção global ou regional. Alemanha 1919) – Direito à prestação positiva

www.editoradince.com - Acesse e veja se há novidades a respeito deste material – CUIDADO: cópia é crime.
DIREITOS HUMANOS - Teoria esquematizada, dicas e questões de concursos 9
– obrigações de fazer impostas ao Estado, Dica 12.
como os direitos sociais, de onde podemos O art. 5, IX trás a inviolabilidade de imprensa,
extrair o direito à saúde, educação, independente de censura ou licença. Muito se tem visto
previdência, alimentação etc. sobre este assunto. O certo é que as emissoras de rádio e
c) 3ª Dimensão ou geração – busca tutelar a televisão devem se adequar e respeitar os valores éticos
sociedade como coletividade - trata-se dos e sociais da pessoa e da família (art. 221, IV). Outro ponto
direitos metaindividuais (ou transindividuais ou deste inciso (IX) que merece destaque é a inviolabilidade
ainda coletivos lato sensu) de onde extraímos da imagem, que cada vez mais, com as novas
os direitos difusos e coletivos strictu sensu tecnologias, sofre elasticidade. Atualmente, o direito a
como direito a solidariedade, fraternidade, a imagem pode ser relativizado, isto é, é possível utilizar
paz e assim por diante. imagem de pessoas ―comuns‖ (que não são pessoas
d) 4ª Dimensão ou geração – apesar de públicas) quando houver cunho jornalístico, não expuser
divergente, temos como posição predominante ao ridículo e não houver intenção pecuniária (fins
o direito à engenharia genética, como comerciais).
clonagem, congelamento de embrião, células
tronco etc. Dica 13.
e) 5ª Dimensão ou geração – é o direito à O art. 5, XI constitui uma das mais incidentes
realidade virtual, isto é, o direito cibernético questões de prova dos últimos anos. Segundo esse inciso
como direitos sobre o software, direito autoral ―a casa é asilo inviolável do indivíduo, ninguém nela
de obras publicadas por meio digital, pirataria podendo penetrar sem consentimento do morador, salvo
etc. em caso de flagrante delito ou desastre, ou para prestar
f) 6ª Dimensão ou geração – é o direito socorro, ou, durante o dia, por determinação judicial‖.
encontrado na declaração de independência Deve se ter em mente que:
dos Estados Unidos, qual seja, direito de a) Casa (sob a ótica deste inciso) é qualquer
buscar a felicidade, como por exemplo o compartimento privado não aberto ao público,
direito das famílias eudemônicas, união onde a pessoa exerça sua profissão ou utilize
homoafetiva, cirurgia de redesignação sexual como moradia. Neste conceito enquadram-se
dentre outros. no conceito clínicas particulares, escritório de
P.S: É bom frisar que a doutrina mais tradicional advocacia, consultório médico, trailer, quarto de
concebem somente até a terceira dimensão, eis que os hotel ocupado e assim por diante;
demais seriam antigos direitos adaptados às novas b) Em caso de flagrante delito, desastre ou para
exigências da sociedade moderna. prestar socorro (fds), pode a qualquer hora e
Inclusive, é possível encontrar outros em qualquer dia sem determinação judicial;
posicionamentos do que vem a ser 4ª, 5ª e 6ª geração de c) se não for nas três hipóteses citadas (fds)
direitos. somente durante o dia por determinação
judicial.
Dica 11. Sobre o conceito de dia, o STF adota o critério
cronológico segundo o qual dia é o período compreendido
O artigo 5, VI, VII e VIII, trata da liberdade de
entre 6:00 e 18:00h1.
convicção filosófica, política e religiosa (esta última
conhecida como princípio do laicivismo ou
Estado leigo ou Estado laico ou Estado não Dica 14.
confessional). Segundo esta regra, todos podem deixar O art. 5, XII estabelece a inviolabilidade do sigilo
de cumprir obrigações em razão de suas convicções da correspondência e das comunicações telegráficas, de
religiosas, filosóficas e políticas (este ato conhecido como dados e das comunicações telefônicas.
escusa de consciência). Porém, se a obrigação for legal Em se tratando de correspondência, telegráficas e
(prevista em lei) imposta a todos, esta terá que ser dados outras autoridades podem violar o sigilo, desde que
cumprida, independente de suas convicções (art. 5, VIII). fundamentado em razões constitucionais ou em norma
Como exemplo, podemos citar a obrigação legal. Autoridades carcerárias podem violar o sigilo de
eleitoral (votar) e alistamento militar. correspondência de preso. Comissão paramentar de
Se a obrigação prevista em lei não for cumprida, inquérito pode determinar a quebra do sigilo bancário e
será imposta uma sanção, como multa no caso de fiscal.
inadimplência eleitoral e serviço alternativo no caso de No caso de comunicações telefônicas (que deve
obrigação militar. Somente após o não cumprimento das ser entendida como espécies do gênero interceptação
sanções é que haverá restrições (suspensão) dos direitos telefônica e escuta telefônica), somente é possível sua
políticos nos moldes do art. 15, IV. violação por ordem judicial e para fins criminais
A leitura deste inciso não pode nos induzir a (investigação criminal ou instrução processual penal).
interpretá-lo como forma de vedação ao ensino religioso Fiquem ligados! Interceptação telefônica só por
na rede pública de ensino, até porque o art. 210, §1 ordem judicial, mas quebra do sigilo telefônico, como
autoriza o ensino religioso em escola pública, desde que a contêm dados (conta reversa – número dos telefones para
matrícula seja facultativa, por óbvio não será permitido o qual ligou, tempo de ligação etc.), pode ser violado por
ensinar os fundamentos de uma única religião. O art. 33 outras autoridades ou órgãos como a própria CPI citada
da L. 9394/96 (Lei de Diretrizes e Bases da Educação) acima.
que estabelece o respeito à diversidade cultural religiosa
no Brasil e veda qualquer forma de proselitismo.
Dica 15.
O art. 5º prevê o princípio da inafastabilidade do
controle jurisdicional ou princípio do acesso ao

www.editoradince.com - Acesse e veja se há novidades a respeito deste material – CUIDADO: cópia é crime.
10 D DIREITOS HUMANOS - Teoria esquematizada, dicas e questões de concurso
Judiciário no inciso XXXV segundo o qual ―a lei não previsão inicial no Código de Processo Penal em sentido
excluirá da apreciação do Poder Judiciário lesão ou contrário, atualmente, o silêncio não pode ser interpretado
ameaça a direito‖. Este preceito quer dizer que todos tem em prejuízo da defesa. Trata-se de uma faceta do
direito, não só ao acesso material ao órgão judicial, mas princípio da não autoincriminação (―nemo tenetur se
também à prestação jurisdicional que deve ser adequada, detegere‖). Este direito pode ser externado de diversas
efetiva e tempestiva. formas, como a possibilidade do motorista se eximir da
A Constituição excepciona este princípio em realização do exame do bafômetro intituído pela Lei
algumas hipóteses como no caso da competência 11.705/08 (HC 166.377 -STJ).
privativa do Senado Federal para julgar determinadas
autoridades (art. 52, I, II, p.ú.), o impedimento de Dica 19.
impetração de habeas corpus contra punições
Uma das mudanças de interpretação recentes de
disciplinares militares (art. 42, §1 e 142, §2) e o
maior repercussão foi a ocorrida no art. 5 LXVII que
impedimento de ações judiciais referente à disciplina e às
dispõe ―não haverá prisão civil por dívida, salvo a do
competições desportivas antes de esgotarem as
responsável pelo inadimplemento voluntário e inescusável
instâncias da justiça desportiva (art. 217,§1).
de obrigação alimentícia e a do depositário infiel.‖. Isso
porque ao julgar recurso extraordinário (RE 466343/SP) o
Dica 16. STF passou a conceder hierarquia supralegal a
O art. 5, XLVI traz o princípio da individualização convenção interamericana de direitos humanos, mais
da pena que, em diversos julgados está sendo utilizado conhecido como pacto de San José da Costa Rica. Com
para declarar a inconstitucionalidade de artigos que esse novo status o pacto, que em seu art. 7, §7 veda
generalizam penas, sem levar em conta a peculiaridade qualquer prisão civil por dívida, salvo inadimplemento de
de cada apenado, como no caso da inconstitucionalidade obrigação alimentícia, passou a ter hierarquia maior que o
da lei que vedava progressão de regime em crime art. 652 do Código Civil (lei ordinária), que regulamenta o
hediondo (HC 82959 e Súmula vinculante 26) e a inciso constitucional. Por isso, para o STF é ilícita a
inconstitucionalidade da proibição de liberdade provisória prisão civil de depositário infiel, qualquer que seja a
para presos em flagrante por tráfico ilícito de modalidade do depósito (Súmula vinculante 25).
entorpecentes (HC 92687).
Dica 20.
Dica 17. A emenda constitucional 45/04, conhecida como
reforma do judiciário, incluiu no art. 5 mais um inciso
O art. 5, LI e LII versam sobre extradição, que é o
(LXXVIII) e dois parágrafos (§§ 3º e 4º). O §3 concedeu
ato em que uma pessoa é enviada a outro território em
aos tratados e convenções internacionais o mesmo
razão da competência para julgamento e/ou punição.
status que as emendas constitucionais, desde que
Por ser um ato que depende de requerimento, cumpram dois requisitos:
sempre haverá um viés ativo e um passivo. A extradição
a) Requisito material – dispor sobre direitos
ativa é ocupada por aquele que ocupa o pólo ativo do
humanos.
pedido extradicional (extradição ativa) e a extradição
passiva é realizada pelo legitimado passivo do processo b) Requisito formal - serem aprovados, em cada
de extradição (país requerido). Por isso, apesar do art. 5, Casa do Congresso Nacional, em dois turnos,
LI afirmar que ―nenhum brasileiro será extraditado, salvo o por três quintos dos votos dos respectivos
naturalizado...‖ é possível a extradição de brasileiro membros.
nato em seu viés ativo, isto é, um brasileiro pode ser Os tratados e convenções internacionais que não
extraditado de outro país para o Brasil, como aconteceu dispuserem sobre direitos humanos possuem a mesma
com Salvatore Cacciola, preso em Mônaco e extraditado hierarquia que lei ordinária. Já os que versam sobre
para o Brasil em razão dos crimes contra o sistema direitos humanos e aprovados antes da E.C. 45/04
financeiro ao qual foi condenado. possuem força supralegal.
Já o brasileiro naturalizado será extraditado ativa
e passivamente. Esta última hipótese somente ocorrerá Dica 21.
se no momento do crime comum cometido o estrangeiro
Como afirmado acima, além do §3, a reforma do
ainda não havia adquirido a naturalização brasileira
judiciário incluiu o § 4º no art. 5º, neste estabeleceu que
(―crime comum praticado antes da naturalização‖) ou se
―o Brasil se submete à jurisdição de Tribunal Penal
praticar tráfico ilícito de entorpecentes.
Internacional a cuja criação tenha manifestado adesão.‖
Neste caso, não importa saber o exato momento Essa adesão já havia acontecido antes da Emenda
da naturalização, já que a Constituição afirma que esta Constitucional através do Decreto Legislativo 112/02 e
extradição pode ocorrer a qualquer tempo, antes ou Decreto Executivo 4.388/02, este último trazendo toda a
depois da naturalização. redação do Estatuto de Roma de 17/07/98, conhecido
O estrangeiro poderá ser extraditado se tiver como Tribunal Penal Internacional (TPI).
cumprido os demais requisitos estabelecidos na Lei Logo no preâmbulo a lei do TPI estabelece que
6815/80 (Estatuto do estrangeiro). Há vedação somente este Tribunal é complementar (subsidiário) à jurisdição
se o crime praticado for político ou de opinião (art.5, nacional.
LII).
O problema é que há institutos previstos no
Estatuto que ―aparentemente‖ colidem com os preceitos
Dica 18. fundamentais. Como a possibilidade de se enviar uma
O Direito de permanecer calado previsto no art. pessoa que está no Brasil para ser julgada perante o TPI,
5, LXII é conhecido como ―aviso de Miranda‖ ganhou assemelhando-se a extradição. Este envio deve ocorrer
corpo na Inglaterra e servia de proteção contra mesmo nas hipóteses onde não são admitidas extradição
perseguições religiosas pelo Estado. E apesar da (por colidir, por exemplo, com um dispositivo específico do
estatuto do estrangeiro). Ao julgar a Pet 4625/República

www.editoradince.com - Acesse e veja se há novidades a respeito deste material – CUIDADO: cópia é crime.
DIREITOS HUMANOS - Teoria esquematizada, dicas e questões de concursos 11
do Sudão, o STF realizou a distinção entre a extradição e C Os direitos humanos são inalienáveis, e ninguém pode
o instituto previsto no Dec. 4.388/02, que seria ser privado de seus direitos humanos; mas eles
denominado entrega ou surrender ou remisse. podem ser limitados em situações específicas.
Para isso, lembrou que a extradição só pode ser D O direito à liberdade não pode ser restringido em
formulada por Estado soberano estrangeiro, ao passo que nenhuma condição, mesmo se uma pessoa for
a entrega pode ser requerida por organismo internacional. considerada culpada por um crime, com o devido
Ademais, para acabar com qualquer dúvida, fez uso do processo legal.
art. 102 do Estatuto e Roma que prevê ―a) Por „entrega‟,
entende-se a entrega de uma pessoa por um Estado ao
04. (NUCEPE - 2012 - PM-PI - Sargento da Polícia Militar)
Tribunal, nos termos do presente Estatuto. b) Por
. Em relação ao conceito dos Direitos Humanos,
„extradição‟, entende-se a entrega de uma pessoa por um
identifique com V as alternativas VERDADEIRAS e F,
Estado a outro Estado, conforme previsto em um tratado,
as FALSAS e marque, em seguida, a
em uma convenção ou no direito interno.‖
sequência CORRETA.
( ) O núcleo do conceito de Direitos Humanos se encontra
QUESTÕES DE CONCURSOS no reconhecimento da dignidade da pessoa humana.
01. (FCC - 2018 - Câmara Legislativa do Distrito Federal - Essa dignidade expressa num sistema de valores,
Consultor Legislativo - Direitos Humanos, Minorias, exerce uma função orientadora sobre a ordem jurídica
Cidadania e Sociedade) Uma vez estabelecidos, os porquanto estabelece ―o bom e o justo‖ para o
Direitos Humanos não podem ser retirados do homem.
ordenamento, em razão do princípio da ( ) Direitos Humanos é uma expressão moderna, mas o
A inter-relacionaridade. princípio que invoca é tão antigo quanto a própria
humanidade. É que determinados direitos e liberdades
B indisponibilidade.
são fundamentais para a existência humana.
C inerência.
( ) Os Direitos Humanos surgiram a partir do século XX, e
D vedação do retrocesso. devem ser utilizados apenas nos países democráticos.
E inesgotabilidade. ( ) Os Direitos Humanos são considerados fundamentais
porque sem eles a pessoa humana não consegue
02. (PM RO - PM RO - Polícia Militar de Rondônia - existir ou não é capaz de se desenvolver e de
Aspirante da Polícia Militar/2018) Segundo o ―Curso participar plenamente da vida.
de Direitos Humanos, de André Ramos de Carvalho‖, ( ) Os Direitos Humanos devem privilegiar apenas a
o conceito de Direitos Humanos é: parcela da população mais carente, fato que justifica
A Consistem em um conjunto de direitos considerados sua própria existência.
indispensável para uma vida humana pautada na A V, V, F, V, F
liberdade, igualdade e dignidade. B V, V, V, V, V
B Consistem em um aglomerado de Leis e Decretos que C V, V, F, F, V
visam dar qualidade aos seres vivos.
D F, F, V, F, V
C Consistem na busca da real qualidade de vida, exigindo
E V, V, F, F, F
atenção especial por parte do Estado.
D Consistem em um rol de obrigações indispensáveis
para uma vida humana pautada na liberdade, 05. (IADES - 2019 - PC-DF - Perito Criminal - Verificação
fraternidade e sobriedade. de Aprendizagem - 2ª Prova) São características dos
Direitos humanos
E Consistem no conjunto de Leis e Deveres
indispensáveis para uma vida humana com qualidade. A universalidade, indivisibilidade, renunciabilidade,
historicidade, aplicabilidade imediata e caráter
declaratório.
03. (FAUEL - 2018 - IPRERINE - PR - Advogado) Analise
B universalidade, proibição de retrocesso, disponibilidade
o trecho a seguir e assinale alternativa que NÃO
individual, historicidade, caráter meramente
apresenta uma das características do conceito de
declaratório e imprescritibilidade.
direitos humanos.
C universalidade, irrenunciabilidade, imprescritibilidade,
―Os direitos humanos visam proteger indivíduos e
indivisibilidade, proibição de retrocesso, aplicabilidade
grupos contra ações que interferem nas liberdades
imediata e caráter declaratório.
fundamentais e na dignidade humana. Eles estão
expressos em tratados, no direito internacional D universalidade, interdependência, não
consuetudinário, conjuntos de princípios e outras complementariedade, alienabilidade, renunciabilidade,
modalidades do Direito. A legislação de direitos humanos imprescritibilidade e proibição de retrocesso.
obriga os Estados a agir de uma determinada maneira e E universalidade, irrenunciabilidade, prescritibilidade,
proíbe os Estados de se envolverem em atividades indivisibilidade, proibição de retrocesso, aplicabilidade
específicas. No entanto, a) a legislação não estabelece os imediata e caráter declaratório.
direitos humanos. Os direitos humanos são direitos
inerentes a cada pessoa simplesmente por ela ser um
06. (IBADE - 2020 - Prefeitura de Cariacica - ES - Guarda
humano‖.
Municipal I) Sobre a teoria das gerações/dimensões
(Site Oficial da ONU, 2018, com adaptações) dos direitos humanos, lançada pelo jurista Karel
A Os direitos humanos estão fundados sobre o respeito Vasak, no ano de 1979, é correto dizer que o direito à
pela dignidade e o valor de cada pessoa. educação é um direito de:
B Uma de suas características é a universalidade, isto é, A 1ª dimensão.
são aplicados de forma igual e sem discriminação a B 2ª dimensão.
todas as pessoas.

www.editoradince.com - Acesse e veja se há novidades a respeito deste material – CUIDADO: cópia é crime.
12 D DIREITOS HUMANOS - Teoria esquematizada, dicas e questões de concurso
C 3ª dimensão. não é superação de uma fase pela outra, ou seja, significa
D 4ª dimensão. surgiram direitos novos ou perspectivas novas sobre
direitos já reconhecidos, sempre objetivando uma maior
E 5ª dimensão.
proteção à pessoa humana.
Nas antigas cidades gregas, defendiam-se os
07. (FEPESE - 2019 - SAP-SC - Agente Penitenciário) A direitos que não poderiam ser desconsiderados pelos
teoria das gerações ou dimensões dos direitos governantes e nem pelos seus próprios titulares, uma vez
humanos expõe perspectivas desses direitos em que que eram vistos como direitos naturais, ou seja,
se incluem em cada geração ou dimensão inerentes à pessoa humana. Surge a ideia de direitos
determinados direitos e princípios. naturais superior ao direito positivo. No entanto, os
Conforme essa divisão clássica da doutrina, Direitos Humanos eram reconhecidos apenas aos
é correto afirmar: cidadãos, uma vez que os escravos não eram vistos como
A os direitos de segunda geração ou dimensão se referem pessoas. Já na Roma antiga, o amplo exercício de direitos
aos direitos civis e políticos, compreendendo os civis era reconhecido apenas ao homem mais velho de
direitos de liberdade, englobando as liberdades cada família. Reconhecia certas liberdades básicas aos
clássicas, negativas ou formais. cidadãos, evidenciando sementes da primeira geração de
B os direitos de quinta geração ou dimensão consistem na direitos. Na Roma clássica surgi o ius gentium que
possibilidade de participação na formação da vontade atribuía alguns direitos aos estrangeiros embora em
do Estado, retratando os direitos à democracia e à quantidade inferior aos dos romanos.
informação. A Idade Média foi marcada pelo feudalismo, um
C os direitos de quarta geração ou dimensão se sistema que subjugava certa parte da população às
caracterizam por condensar os direitos e liberdades condições de vassalagem perante um suserano, dono das
civis, políticas, econômicas, sociais e culturais. terras cultiváveis e do exército. A Igreja católica
influenciava os valores a serem reconhecidos como
D os direitos de terceira geração ou dimensão fundamentais ao homem e consequentemente
consubstanciam como titulares a coletividade, estabeleceram uma abordagem canônica do direito
consagrando o princípio da solidariedade e incluindo natural. Santo Agostinho considerava que o governo, o
direitos como o da paz, ao desenvolvimento, ao meio direito e a propriedade eram guardiões da Lei Eterna de
ambiente equilibrado. Deus, que poderia intervir nessas instituições quando
E os direitos de primeira geração ou dimensão são julgasse oportuno. Ademais, se as leis terrenas fossem
aqueles relativos aos direitos econômicos, sociais e contrárias à Lei de Deus, não deveriam ser obedecidas.
culturais, em que se acentua o princípio da igualdade. A primeira mudança sensível ocorre com a
Religião, por meio do segundo grande Cisma da Igreja,
gabarito: 01/D; 02/A; 03/D; 04/A; 05/C; 06/B; 07/D fruto da Reforma Protestante. Com o Iluminismo a
realidade social passa a ser objeto de indagação. O poder
estatal, que era explicado pela vontade divina, é
compreendido como força de vontade popular. O Direito
A EVOLUÇÃO HISTÓRICA DOS Natural é revisto: considerado na Idade Média como
vinculado à vontade de Deus, a partir da Escola de Direito
DIREITOS HUMANOS. Natural de Grotius é visto como produto da razão. Hobbes
INTRODUÇÃO deu andamento ao pensamento de Hugo Grotius, a quem
Para compreender os Direitos Humanos, é se atribui a origem do jusnaturalismo, que sustentava a
necessário inicialmente apresentar uma análise de sua imutabilidade do Direito Natural. Locke, outro
evolução histórica, uma vez que os mesmos são produtos jusnaturalista, sustentou a teoria jurídico-política de modo
da construção histórica, frutos das lutas contra o poder. diferente à de Hobbes, uma vez que considerava o
homem a partir dos princípios da razão com a capacidade
Não há uma definição consensual na doutrina de elaborar uma doutrina moral, ou seja, uma lei natural, e
sobre direitos essenciais àpessoa humana, por isso, através desta lei ensinaria os deveres da vida. Com isso,
encontramos definições como: direitos humanos, direitos abre-se o caminho para o positivismo.
morais, direitos naturais, direitos públicos, dentre outros.
DICA: A expressão Direitos Humanos é a mais A ORIGEM DOS DIREITOS HUMANOS
adotada no contexto internacional, sendo opção Por mais que hoje o termo Direitos Humanos seja
da Organização das Nações Unidas (ONU) na bem conhecido e os tenhamos como direitos garantidos,
Declaração Universal dos Direitos Humanos de importante destacar que nem sempre foi assim. Essa
1948. história é longa e tem seus primeiros momentos quando o
Falar sobre direitos humanos, significa o estudo ser humano manifestou a consciência da necessidade de
integrado dos direitos individuais, sociais, econômicos e viver em grupo, se organizando em sociedades. Na
políticos fundamentais, isto é, Direitos Humanos visto medida em que esses grupos foram surgindo, a vida dos
como sinônimo de Direitos Fundamentais indivíduos pertencentes a eles passou a ser baseada em
Ao longo da história houve três gerações de relações sociais, como a interação cultural, religiosa,
Direitos Fundamentais, entretanto, alguns propõem uma econômica e a comunicação, que exerciam um papel
quarta geração, até mesmo quinta, não existindo, porém, muito importante para a harmonia daquelas sociedades.
um reconhecimento constitucional positivo de sua Mas, para que essas relações funcionassem bem,
existência, nem uma concordância quanto ao seu real as regras passaram a ser um fator essencial, conduzindo
conteúdo. as condutas e o comportamento de todos os que estão
A denominação gerações de direitos humanos submetidos a elas. É nesse contexto de regras das
reflete o caráter acumulativo de reconhecimento e sociedades nascentes que surgiram os primeiros
proteção destes direitos ao longo da história. Destarte, elementos dos Direitos Humanos.

www.editoradince.com - Acesse e veja se há novidades a respeito deste material – CUIDADO: cópia é crime.
DIREITOS HUMANOS - Teoria esquematizada, dicas e questões de concursos 13
Foi em 539 a.C que o primeiro desses elementos A conquista da América no século XVI pelos
apareceu na história da humanidade. Conhecido espanhóis resultou em um debate sobre direitos humanos
como Cilindro de Ciro, ele marcava a libertação do povo na Espanha. Isto marcou a primeira vez que se discutiu o
hebreu da Babilônia, além de permitir a liberdade religiosa assunto na Europa.
e estabelecer a igualdade racial na região da Pérsia (atual Muitos filósofos e historiadores do direito
Irã). consideram que não se pode falar de direitos humanos
Mas, ainda assim, a ideia de Direitos Humanos até a modernidade no Ocidente. Até então, as normas
ainda estava longe de ser um consenso global nessa da comunidade, concebidas na relação com a ordem
época. Para efeitos de comparação, em 450 a.C (cerca de cósmica, não deixavam espaço para o ser humano
89 anos depois do Cilindro de Ciro) foi decretada a Lei das como sujeito singular, concebendo-se o direito
Dozes Tábuas na Roma Antiga, que diferente da primariamente como a ordem objetiva da sociedade. A
concepção social e humana do documento da Babilônia, sociedade estamental tem seu centro em grupos como a
permitia a execução de bebês que nasciam com família, a linhagem ou as corporações profissionais ou
deficiências ou deformidades. Dessa forma, apesar de laborais, o que implica que não se concebem faculdades
seu elemento original ter nascido na antiguidade, os próprias do ser humano enquanto tal. Pelo contrário, se
Direitos Humanos tiveram de passar por um longo entende que toda faculdade atribuível ao indivíduo deriva
processo de aprimoramento. de um duplo status: o do sujeito no seio da família e o
desta na sociedade. "Fora do Estado, não há direitos".
A EVOLUÇÃO HISTÓRICA DOS DIREITOS A existência dos direitos subjetivos, tal e como se
HUMANOS. pensam na atualidade, será objeto de debate durante os
séculos XVI, XVII e XVIII, o que é relevante porque
Um dos documentos mais antigos que se vinculam habitualmente se diz que os direitos humanos são
aos direitos humanos é o Cilindro de Ciro, que contém produto da afirmação progressiva da individualidade
uma declaração do rei persa Ciro II depois de sua e que a ideia de direitos do homem apareceu pela
conquista da Babilônia em 539 a.C. Foi descoberto em primeira vez durante a luta burguesa contra o sistema
1879 e a Organização das Nações Unidas o traduziu em do Antigo Regime. Sendo esta a consideração mais
1971 a todos os seus idiomas oficiais. Pode ser resultado estendida, outros autores consideram que os direitos
de uma tradição mesopotâmica centrada na figura do "rei humanos são uma constante na História e têm suas
justo", cujo primeiro exemplo conhecido é o rei raízes no mundo clássico; também sua origem se
Urukagina, de Lagash, que reinou durante o século XXIV encontra na afirmação do cristianismo da dignidade moral
a.C. Cabe destacar, também, nessa tradição, Hamurabi do homem enquanto pessoa.
da Babilônia e seu famoso Código de Hamurabi, que data
do século XVIII a.C. O Cilindro de Ciro apresentava Com a Idade Moderna, os racionalistas dos
características inovadoras, especialmente em relação à séculos XVII e XVIII, reformulam as teorias do direito
religião. Nele, era declarada a liberdade de religião e a natural, deixando este de estar submetido a uma ordem
abolição da escravatura. Tem sido valorizado divina. Para os racionalistas, todos os homens são, por
positivamente por seu sentido humanista e inclusive foi natureza, livres, e têm certos direitos inatos de que
descrito como a primeira declaração de direitos não podem ser despojados quando entram em
humanos. sociedade. Foi esta corrente de pensamento que
acabou por inspirar o atual sistema internacional de
Na Roma antiga, havia o conceito jurídico da proteção dos direitos do homem.
concessão da cidadania romana a todos os romanos. O
cristianismo, durante a Idade Média, foi a afirmação da A evolução destas correntes veio a dar frutos
defesa da igualdade de todos os homens numa mesma pela primeira vez na Inglaterra, e depois nos Estados
dignidade. Foi também durante esta época que os Unidos. A Magna Carta (1215) deu garantias contra a
filósofos cristãos recolheram e desenvolveram a teoria do arbitrariedade da Coroa, e influenciou diversos
direito natural, em que o indivíduo está no centro de uma documentos, como por exemplo o Habeas Corpus (1679),
ordem social e jurídica justa, mas a lei divina tem que foi a primeira tentativa para impedir as detenções
prevalência sobre o direito laico tal como é definido ilegais. A Declaração Americana da Independência
pelo imperador, o rei ou o príncipe. Logo, foram criadas surgiu a 4 de Julho de 1776: nela, constavam os
muitas teorias no decorrer do tempo. direitos naturais do ser humano que o poder político
Segundo Comparato(2008, p. 11), ―O Cristianismo, deve respeitar. Esta declaração teve, como base, a
em particular levou às últimas consequências o Declaração de Virgínia proclamada a 12 de Junho de
ensinamento ecumênico de Isaías, envolvendo-o na 1776, onde estava expressa a noção de direitos
exigência do amor universal‖. individuais.
Foi, então, no período axial que foram lançados os Durante a Revolução Inglesa, a burguesia
fundamentos intelectuais para a compreensão da pessoa conseguiu satisfazer suas exigências de ter alguma
humana e, pela primeira vez na história, o homem foi classe de seguridade contra os abusos da coroa e limitou
considerado um ser dotado de igualdade essencial, o poder dos reis sobre seus súditos, proclamando a Lei
liberdade e razão, embora persistam inúmeras diferenças de Habeas corpus em 1679. Em 1689, o Parlamento
de sexo, raça, religião e costumes sociais, que, se impôs, a Guilherme III de Inglaterra, na Carta de Direitos
respeitados, tem-se a existência dos direitos (ou Declaração de direitos - Bill of Rights), uma série
universais. de princípios sobre os quais os monarcas não podiam
legislar ou decidir.
Documentos posteriores, como a Carta Magna
da Inglaterra, de 1215, e a Carta de Mandén, de 1222,
se têm associado também aos direitos humanos. Os DECLARAÇÃO DOS DIREITOS DO HOMEM E DO
direitos humanos ou coletivos foram forjados ao CIDADÃO DE 1789
longo da história, através de debates realizados por No século XVII e XVIII, filósofos europeus,
filósofos e juristas. destacando-se John Locke, desenvolveram o conceito
do direito natural. Os direitos naturais, para Locke, não

www.editoradince.com - Acesse e veja se há novidades a respeito deste material – CUIDADO: cópia é crime.
14 D DIREITOS HUMANOS - Teoria esquematizada, dicas e questões de concurso
dependiam da cidadania nem das leis de um Estado, nem Assim, a 10 de Dezembro de 1948, a Assembleia
estavam necessariamente limitadas a um grupo étnico, Geral das Nações Unidas proclamou a Declaração
cultural ou religioso em particular. A teoria do contrato Universal dos Direitos Humanos.
social, de acordo com seus três principais A Declaração Universal dos Direitos Humanos é
formuladores, o já citado Locke, Thomas Hobbes e fundamental na nossa Sociedade pois quase todos os
Jean-Jacques Rousseau, se baseia em que os direitos documentos relativos aos direitos humanos têm,
do indivíduo são naturais e que, no estado de natureza, como referência, esta Declaração, a qual alguns
todos os homens são titulares de todos os direitos. Estados fazem referência direta nas suas constituições
A primeira declaração dos direitos humanos da nacionais.
época moderna é a Declaração de Direitos de Virgínia A Declaração Universal dos Direitos Humanos
de 12 de junho de 1776, escrita por George Mason e ganhou uma importância extraordinária, contudo não
proclamada pela Convenção da Virgínia. Esta medida obriga juridicamente que todos os Estados a respeitem e,
influenciou Thomas Jefferson na declaração dos direitos devido a isso, a partir do momento em que foi
humanos que existe na Declaração da Independência promulgada, foi necessário a preparação de inúmeros
dos Estados Unidos da América de 4 de julho de 1776, documentos que especificassem os direitos presentes na
assim como também influenciou a Assembleia Nacional declaração e, assim, forçassem os Estados a cumpri-la.
francesa em sua declaração, a Declaração dos Direitos Foi nesse contexto que, no período entre 1945-1966,
do Homem e do Cidadão de 1789. Esta definia o direito nasceram vários documentos, entre os quais se destacam
individual. A Declaração dos Direitos do Homem e do os Pactos Internacionais de Direitos Humanos de 1966.
Cidadão, proclamada na França em 1789, e as
Assim, a junção da Declaração Universal dos
reivindicações ao longo dos séculos XIV e XV em prol
Direitos Humanos, os dois pactos efetuados em 1966,
das liberdades, alargaram o campo dos direitos
nomeadamente o Pacto Internacional dos Direitos Civis e
humanos e definiram os direitos econômicos e
Políticos e o Pacto Internacional dos Direitos Econômicos,
sociais.
Sociais e Culturais, bem como os dois protocolos
A noção de direitos humanos não experimentou facultativos do Pacto dos Direitos Civis e Políticos (que,
grandes mudanças até o século seguinte. Com o início em 1989, aboliu a pena de morte), constituem a Carta
das lutas operárias, surgiram novos direitos que Internacional dos Direitos do Homem.
pretendiam dar solução a determinados problemas sociais
Em 2016, no entanto, o secretário-geral da Anistia
através da intervenção do Estado. Neste processo, são
Internacional, Salil Shetty, no relatório anual da
importantes a Revolução Russa e a Revolução Mexicana.
organização, questionou a eficiência do atual sistema de
Desde o nascimento da Organização das proteção dos direitos humanos no mundo, segundo ele
Nações Unidas em 1945, o conceito de direitos incapaz de resolver problemas como a crise migratória na
humanos se tem universalizado, alcançando uma Europa, a perseguição aos defensores dos direitos
grande importância na cultura jurídica internacional. Em humanos, as violências sexual e de gênero, a
10 de dezembro de 1948, a Declaração Universal dos impunidade, o aliciamento de crianças por organizações
Direitos Humanos foi adotada e proclamada pela criminosas, a tortura, a violência policial, a manutenção da
Assembleia Geral das Nações Unidas em sua Resolução pena de morte em alguns países e as remoções forçadas
217 A (III), como resposta aos horrores da Segunda de populações.
Guerra Mundial e como intento de assentar as bases da
nova ordem internacional que surgia depois do armistício. GERAÇÃO DE DIREITOS HUMANOS
Mas o momento mais importante, na história dos Primeira geração de Direitos Humanos:
Direitos do Homem, é durante 1945-1948. Em 1945, os
Estados tomam consciência das tragédias e atrocidades No final da Idade Média, ocorre a formação dos
vividas durante a 2ª Guerra Mundial, os levando a criar a Estados absolutistas. Somente com o surgimento do
Organização das Nações Unidas (ONU) em prol de comércio e a substituição do regime feudal pelo sistema
estabelecer e manter a paz no mundo. Foi através da de produção capitalista que os direitos humanos de
Carta das Nações Unidas, assinada a 20 de Junho de primeira geração iniciaram seu efetivo desenvolvimento,
1945, que os povos exprimiram a sua determinação "em como prerrogativas jurídicas oponíveis em face dos
preservar as gerações futuras do flagelo da guerra; próprios governantes. A materialização dos Direitos
proclamar a fé nos direitos fundamentais do Homem, na Fundamentais inicia na Inglaterra, com a Magna Carta de
dignidade e valor da pessoa humana, na igualdade de 1215, produto do conflito entre o Rei João e os barões. A
direitos entre homens e mulheres, assim como das referida Carta impôs limitações ao poder absoluto,
nações, grande e pequenas; em promover o progresso garantindo-se aos indivíduos certos Direitos
Fundamentais. Em 1629, o Petition of Rights, o Habeas
social e instaurar melhores condições de vida numa maior
liberdade". A criação das Nações Unidas simboliza a Corpus, que protegia a liberdade de locomoção, inspirou o
necessidade de um mundo de tolerância, de paz, de ordenamento do mundo todo e, principalmente, o Bill of
Rights de 1689, este documento reconheceu alguns
solidariedade entre as nações, que faça avançar o
progresso social e económico de todos os povos. direitos ao indivíduo, tais como o direito de liberdade, à
segurança e à propriedade privada.
Os principais objetivos das Nações Unidas,
passam por manter a paz, a segurança internacional, A primeira geração dos direitos humanos
consolidou-se na fase de resistência aos poderes dos
desenvolver relações amigáveis entre as nações, realizar
monarcas absolutistas, em decorrência da luta da
a cooperação internacional resolvendo problemas
internacionais do cariz econômico, social, intelectual e burguesia pela salvaguarda de direitos individuais básicos
humanitário, desenvolver e encorajar o respeito pelos tais como a vida, a liberdade e a propriedade. A classe
direitos humanos e pelas liberdades fundamentais sem burguesa adquiria importância política, em razão de sua
qualquer tipo de distinção. ascensão econômica. Surge o Estado Moderno,
centralizando o poder político.

www.editoradince.com - Acesse e veja se há novidades a respeito deste material – CUIDADO: cópia é crime.
DIREITOS HUMANOS - Teoria esquematizada, dicas e questões de concursos 15
O marco documental foi a Declaração dos Direitos internacionais para estabelecer um núcleo fundamental de
do Homem e do Cidadão, aprovada pela Assembleia Direitos Humanos Internacionais. Dessa forma, elabora a
Nacional Francesa, em 26 de agosto de 1789. Declaração Universal de Direitos Humanos de 1948, a
Os direitos de primeira geração têm por escopo a Declaração Americana dos Direitos do Homem, de 1969,
defesa das pessoas em face ao arbítrio dos governantes, em São José da Costa Rica, com objetivo de tornar
mormente quanto a preservação de sua vida, de sua universal aplicação dos Direitos Humanos. Surge
liberdade de locomoção, amplo exercício profissional e da organizações não-estatais, como a Anistia Internacional, a
possibilidade de constituírem patrimônio, sem que este Comissão Internacional dos Juristas, o Instituto
seja confiscado pela exigência de tributos excessivos. Interamericano dos Direitos Humanos, com a finalidade
Destarte, o conteúdo dos Direitos Fundamentais nessa de divulgação de ideias e educação em Direitos
época seriam os Direitos Individuais relativos à liberdade Humanos. Houve a incorporação dos direitos sociais nos
e à igualdade. E a base do Estado Liberal é o direito de diversos ordenamentos jurídicos. No entanto, estes
propriedade privada, que é absoluto e intocável. direitos não se realizam plenamente hodiernamente,
principalmente pela insuficiência de recursos para que o
Foi a partir das revoluções Inglesa, Americana e
Estado lhes confira eficácia, o que inicialmente os relegou
Francesa que os princípios liberais políticos e econômicos
à esfera meramente programática.
foram consagrados. Nesse período, triunfou o liberalismo,
e não a democracia, havendo somente no futuro uma
fusão desses dois. Terceira Geração dos Direitos Humanos:
Nos séculos XVII e XVIII a atitude de omissão do Esta fase não possui uma identificação clara dos
estado diante dos problemas sociais e econômicos agentes operadores, pois emergiu dos apelos de uma
conduziu os homens a um capitalismo desumano. O sociedade massificada, visando a preservação dos
século XIX foi marcado pelas misérias sociais que a interesses coletivos ou difusos relacionados com a
Revolução Industrial agravou e que o Liberalismo proteção do meio ambiente, preservação do patrimônio
colaborou. Combatida, principalmente, pelo marxismo e o histórico e cultural, qualidade de vida no ambiente urbano
fascismo, a liberal-democracia estava encurralada. e rural, tutela sobre a comunicação social, a bioética,
Sendo assim, o Estado não tinha mais como ampliação dos direitos políticos, autodeterminação dos
ignorar os problemas sociais e econômicos. Após a povos, o amplo acesso a informação e preservação da
primeira Guerra Mundial, as novas Constituições privacidade.
preocuparam com a política do Estado, mas, também, É identificável na sociedade a partir da década de
com o direito e o dever do Estado em reconhecer e sessenta, devido às mudanças políticas, expansão do
garantir as novas demandas sociais. mercado, o surgimento das transnacionais, o desequilíbrio
ecológico, disseminação de fatos inverídicos pelos meios
de comunicação, e o progresso tecnológico. Surge
Segunda Geração de Direitos Humanos:
reclamações coletivas com objetivo de buscar a
A segunda geração de direitos humanos nasce efetivação dos aspectos inerentes a condição humana.
das lutas sociais que buscavam uma maior
Mesmo que os direitos das gerações anteriores
salvaguarda das condições necessárias ao
não tenham sido plenamente efetivados, o avanço
desenvolvimento pleno da humanidade, mas seus
civilizatório fez com que surgisse uma vasta gama de
protagonistas foram as classes operárias, que
situações em que a personalidade humana é atingida,
apareceram em consequência da industrialização na
ensejando a enunciação de novos direitos. E
Europa. Essa classe operária tinha formalmente
gradativamente, tais prerrogativas jurídicas estão se
resguardado direitos da primeira geração, mas eram
incorporando aos diversos ordenamentos jurídicos, no
explorados pelos detentores do capital, careciam de
plano infraconstitucional nas leis fundamentais.
saneamento básico em suas residências, educação,
atendimento médico, proteção jurídica adequada em face Alguns novos direitos são apenas os antigos
das péssimas condições de trabalho, de remuneração e adaptados às novas exigências do momento, isto é,
jornada de trabalho. alguns dos direitos fundamentais da primeira geração e
da segunda geração estão sendo revitalizados e até
Karl Marx e de Friederich Engels editaram o
mesmo ganhando importância e atualidade, de modo
Manifesto Comunista que juntamente com outros
especial em face das novas formas de agressão.
documentos, tais como a encíclica papal Rerum Novarum
de Leão XIII de 1891, a Constituição Mexicana de 1917 e Sendo assim, percebe-se que o desafio atual não
a Constituição Alemã de 1919, fortaleceram o é procurar investigar as futuras classes de direitos, mas
desenvolvimento dos ideais de universalidade e sim verificar as modernas reivindicações sociais, para
socialismo (direitos sociais) dos direitos humanos. Com reconhecer e formular novos direitos (ou nova
essas Constituições os direitos sociais passam a ser conformação de direitos clássicos). E analisar os meios
considerados direitos fundamentais dos seres humanos. mais adequados para imprimir efetiva concretude aos
direitos já reconhecidos e, em grande parte, já
Após a Segunda Guerra Mundial já não se admitia
incorporados em tratados internacionais ou mesmo
mais o Estado nos moldes liberais clássicos de não
positivados nos ordenamentos jurídicos particulares.
intervenção. O Estado passa a ser um administrador da
sociedade e neste momento deve aproveitar os laços

www.editoradince.com - Acesse e veja se há novidades a respeito deste material – CUIDADO: cópia é crime.
16 D DIREITOS HUMANOS - Teoria esquematizada, dicas e questões de concurso

www.editoradince.com - Acesse e veja se há novidades a respeito deste material – CUIDADO: cópia é crime.
DIREITOS HUMANOS - Teoria esquematizada, dicas e questões de concursos 17

3 – Término da Segunda Guerra Mundial =


SIMPLIFICANDO O ASSUNTO: consciência da necessidade de não se permitir que seres
humanos sofressem aquelas atrocidades cometidas pelos
CONCEITO E BREVE EVOLUÇÃO: nazistas contra os judeus e outros grupos de pessoas.
Direitos Humanos: Somatória de valores e de atos Houve então a criação da Organização das
que possibilitam a todos uma vida digna. São previstos Nações Unidas e a declaração de inúmeros Tratados
em tratados internacionais. Internacionais de Direitos Humanos, como ―A Declaração
São direitos que podem ser exercidos individual ou Universal de Direitos Humanos‖.
coletivamente.
Se previstos em uma Constituição, são chamados
de Direitos Fundamentais.
Ganharam importância no século XX e estão
incorporados aos pensamentos jurídico do século XXI. Os
doutrinadores sustentam que o fundamento e justificativa
estariam ligados ao positivismo ou ao jusnaturalismo.
Positivismo: estruturação jurídica (previsão
legal)– Norberto Bobbio e Hans Kelsin defendem que uma
vez previstos no ordenamento jurídico, podem ser
efetivamente exigidos dentro de um Estado.
Além do plano interno, sua proteção se dá em
Tratados e Convenções Internacionais.
Jusnaturalismo: Pessoa Humana é o fundamento
absoluto dos Direitos Humanos, independente do lugar
em que esteja, devendo se tratado de modo justo e
solidário. Dalmo de Abreu Dallari e Fábio Konder Ordem histórica de proteção:
Comparato que defendem esse posicionamento. 1. Carta Magna Libertatum (1215) – onde o Rei
ATENÇÃO: O direito só existe em função do João sem terra começou a reconhecer direitos;
homem, e é nele que se fundamento todo e 2. Petition of rights (1628) – quando se começou a
qualquer direito. exigir autorização do parlamento para que o
monarca praticasse diversos atos;
3. Habeas Corpus act (1679);
Três Marcos Históricos Fundamentais:
4. Bill of rights (1689) - supremacia do parlamento
 O Iluminismo
sobre a vontade do Rei;
 A Revolução Francesa
5. Declaração de Direitos do Estado da Virgínia
 Término da 2ª Guerra Mundial (1776);
1 – Iluminismo = ressalta a razão, o espirito crítico 6. Declaração de independência dos EUA (1776) –
e a fé na ciência. impondo limites para o Estado;
Procurou compreender a essência das coisas e 7. Constituição Federal dos EUA (1787).
das pessoas.
2 – Revolução Francesa = fez nascer os ideias
representativos dos Direitos Individuais e Humanos: QUESTÕES DE CONCURSOS
- Igualdade 01. (CESPE / CEBRASPE - 2021 - PC-DF - Escrivão de
- Fraternidade Polícia da Carreira de Polícia Civil do Distrito Federal)
- Liberdade Acerca da concepção e da evolução histórica dos
direitos humanos, julgue o item a seguir.

www.editoradince.com - Acesse e veja se há novidades a respeito deste material – CUIDADO: cópia é crime.
18 D DIREITOS HUMANOS - Teoria esquematizada, dicas e questões de concurso
Ainda antes de Cristo, foram lançados os primeiros 05. (VUNESP - 2018 - PC-SP - Delegado de Polícia) Esse
fundamentos intelectuais da igualdade essencial entre documento histórico de remota conquista dos direitos
todos os homens e, por conseguinte, da afirmação da humanos foi editado com o escopo de assegurar a
existência de direitos universais. Supremacia do Parlamento sobre a vontade do Rei,
controlando e reduzindo os abusos cometidos pela
nobreza em relação aos seus súditos, em especial
02. (CESPE - 2019 - PRF - Policial Rodoviário Federal)
declarando, dentre outras conquistas, o direito de
Acerca de aspectos da teoria geral dos direitos
petição, eleições livres e a proibição de fianças
humanos, da sua afirmação histórica e da sua relação
exorbitantes e de penas severas:
com a responsabilidade do Estado, julgue o próximo
item. A Petition of Rights, de 1628.
Todos os direitos humanos foram afirmados em um único B Habeas Corpus Act, de 1679.
momento histórico. C The Bill of Rights, de 1689.
D Declaração de Direitos do Homem e do Cidadão, de
03. (IF-TO - 2018 - IF-TO - Assistente de Aluno) Na 1789.
história, há dois grandes movimentos que foram E Magna Carta, de 1215.
fundamentais para a base da Declaração dos Direitos
Humanos, elaborada pela Organização das Nações
06. (VUNESP - 2018 - PC-SP - Delegado de Polícia) No
Unidas (ONU), criada em 1948. Quais foram esses
tocante à temática dos direitos humanos,
dois acontecimentos históricos que influenciaram a
considerando seu surgimento e sua evolução
Declaração Universal dos Direitos Humanos?
histórica, assinale a alternativa que contempla correta
Com base no exposto acima, marque a e cronologicamente seus marcos históricos
alternativa correta. fundamentais.
A A Revolução Industrial (1760) e a Revolta dos Malês A O iluminismo, o constitucionalismo e o socialismo.
(1835).
B O cristianismo, o socialismo e o constitucionalismo.
B A Revolução Francesa (1789) e a Abolição da
C A Magna Carta, a Constituição Alemã de Weimar e a
Escravidão no Brasil (1888).
Declaração de Independência dos Estados Unidos da
C A Revolução Francesa (1789) e a Independência dos América.
Estados Unidos (1776).
D
D A Independência dos Estados Unidos (1776) e a Bill of
A Magna Carta, a queda da Bastilha na França e a
Rights (1689).
criação da Organização das Nações Unidas.
E A Petition of Rights (1628) e a Guerra do Paraguai
E O iluminismo, a Revolução Francesa e o fim da
(1864).
Segunda Guerra Mundial.

04. (VUNESP - 2018 - MPE-SP - Analista Jurídico do


Gabarito: 01/C; 02/E; 03/C; 04/B; 05/A; 06/E
Ministério Público) Em relação ao conceito, evolução
histórica e dimensões dos Direitos Humanos, assinale
a alternativa correta.
A As Declarações americana (1776) e francesa (1789) OS DIREITOS HUMANOS NA
são documentos relacionados aos direitos humanos
de segunda geração ou dimensão. ORGANIZAÇÃO DAS NAÇÕES UNIDAS.
B As distinções apresentadas na doutrina entre as A ONU foi fundada oficialmente em 24 de Outubro
expressões direitos humanos e direitos fundamentais de 1945, em São Francisco, Califórnia, ao final da
são focadas na ideia de que os direitos humanos são Segunda Guerra Mundial. Representou importante
absolutos ao passo que os direitos fundamentais mecanismo de cooperação internacional, a fim de
podem ser relativizados no caso concreto. construir a paz no pós-Guerra, e prevenir guerras futuras.
C A expressão direitos humanos ou direitos do homem é A ONU, que substituiu a Liga das Nações, voltou-se para
reservada aos direitos relacionados com posições os seguintes objetivos:
básicas das pessoas, inscritos em diplomas • Manter a paz e da segurança internacionais
normativos de cada Estado. São direitos que vigem (vertente repressiva – forma de inibição da
numa ordem jurídica concreta, sendo, por isso, violação de direitos baseada na punição com
garantidos e limitados no espaço e no tempo, pois são base legal).
assegurados na medida em que cada Estado os • Promover os direitos humanos no âmbito
consagra. internacional (vertente promocional –
D Na visão majoritária da doutrina, a Declaração caracteriza-se pela adoção de medidas
Universal dos Direitos Humanos não é um tratado capazes de criar o sentimento de
internacional, no sentido formal, e, apesar de orientar pertencimento e um senso de identidade social
as relações sociais no âmbito da proteção da para romper com o isolamento dos guetos e
dignidade da pessoa humana, não possui, em si, força com a repulsa e a hostilidade da mútua
vinculante. exclusão entre as comunidades excluídas e a
E Os direitos humanos de quarta geração ou dimensão sociedade que as exclui, favorecendo o
são os direitos difusos relacionados à sociedade atual, respeito à diversidade).
a exemplo do direito ambiental, frequentemente • Cooperar internacionalmente nas esferas social e
violados sob os mais diversos aspectos. econômica.
Esses objetivos, porém, não têm sido buscados de
forma equilibrada. Tem-se concedido peso especialmente

www.editoradince.com - Acesse e veja se há novidades a respeito deste material – CUIDADO: cópia é crime.
DIREITOS HUMANOS - Teoria esquematizada, dicas e questões de concursos 19
maior à manutenção da paz do que à promoção de inscrever e ao qual todas as pessoas aspiram. As Nações
direitos humanos e à cooperação internacional. Unidas definiram uma ampla gama de direitos
Segundo a ONU: ―Os direitos humanos são direitos internacionalmente aceites, incluindo direitos civis,
inerentes a todos os seres humanos, Independentemente culturais, económicos, políticos e sociais. Também
de raça, sexo, nacionalidade, etnia, idioma, religião ou estabeleceu mecanismos para promover e proteger esses
qualquer outra condição‖. direitos, e auxiliar os Estados a cumprirem as suas
Os direitos humanos incluem o direito à vida e à responsabilidades.
liberdade, à liberdade de opinião e de expressão, o direito As bases desse corpo de leis são a Carta das
ao trabalho e à educação, entre e muitos outros. Todos Nações Unidas e a Declaração Universal dos Direitos
merecem estes direitos, sem discriminação. Humanos, adotadas pela Assembleia Geral em 1945 e
O Direito Internacional dos Direitos Humanos em 1948, respetivamente. Desde então, as Nações
estabelece as obrigações dos governos de agirem de Unidas expandiram gradualmente a lei de direitos
determinadas maneiras ou de se absterem de certos atos, humanos para abranger padrões específicos para
a fim de promover e proteger os direitos humanos e as mulheres, crianças, pessoas com deficiência, minorias e
liberdades de grupos ou indivíduos. outros grupos vulneráveis, que agora possuem direitos
A expressão formal dos direitos humanos inerentes que os protegem da discriminação.
se dá através das normas internacionais de direitos
humanos. Uma série de tratados internacionais dos
direitos humanos e outros instrumentos surgiram a partir
de 1945, conferindo uma forma legal aos direitos OS DIREITOS HUMANOS NA
humanos inerentes. ORGANIZAÇÃO DOS ESTADOS
A criação das Nações Unidas viabilizou um
fórum ideal para o desenvolvimento e a adoção dos AMERICANOS.
instrumentos internacionais de direitos humanos. Outros 1 EVOLUÇÃO HISTÓRICA DO TEMA DE
instrumentos foram adotados a nível regional, refletindo DIREITOS HUMANOS NO ÂMBITO DA OEA
as preocupações sobre os direitos humanos particulares a Apesar de oficialmente criada em 1948, a OEA, ou
cada região. a ideia de uma comunidade continental que congregasse
A maioria dos países também adotou constituições os Estados em busca de objetivos comuns, não surgiu
e outras leis que protegem formalmente os direitos naquele momento. O nascimento da OEA foi
humanos básicos. Muitas vezes, a linguagem utilizada consequência de um longo processo de estreitamento de
pelos Estados vem dos instrumentos internacionais de laços entre os países das Américas, que se iniciou com a
direitos humanos. primeira de nove Conferências Internacionais Americanas,
As normas internacionais de direitos humanos em 1889, na cidade de Washington.
consistem, principalmente, de tratados e costumes, bem Quase sessenta anos depois, em 1948, na cidade
como declarações, diretrizes e princípios, entre outros. de Bogotá, a Nona Conferência Internacional Americana
Desde o estabelecimento das Nações Unidas, tinha como objetivo estruturar de forma definitiva o
em 1945 – em meio ao forte lembrete sobre os horrores sistema interamericano. O final da Segunda Grande
da Segunda Guerra Mundial –, um de seus objetivos Guerra e a criação, logo em seguida, da Organização das
fundamentais tem sido promover e encorajar o respeito Nações Unidas, certamente influenciaram os países do
aos direitos humanos para todos. continente a tentar, finalmente, adotar alguns
Os direitos humanos são comumente instrumentos que seriam essenciais à concretização do
compreendidos como aqueles direitos inerentes ao ser sistema, como a Carta da Organização dos Estados
humano. O conceito de Direitos Humanos reconhece que Americanos, que formalmente dá vida à OEA.
cada ser humano pode desfrutar de seus direitos
Na mesma ocasião, foi adotada também a
humanos sem distinção de raça, cor, sexo, língua,
Declaração Americana dos Direitos e Deveres do Homem.
religião, opinião política ou de outro tipo, origem social ou
Destarte, desde o nascimento, a OEA já contava com seu
nacional ou condição de nascimento ou riqueza.
primeiro instrumento de proteção dos direitos humanos
Os direitos humanos são garantidos legalmente como obrigação multilateral.
pela lei de direitos humanos, protegendo indivíduos e
grupos contra ações que interferem nas liberdades O dever de fazer valer os direitos intrínsecos à
fundamentais e na dignidade humana. pessoa humana e de responsabilizar aqueles Estados que
não o façam foi alçado à condição de compromisso
Estão expressos em tratados, no direito
internacional dentro da OEA logo em seu nascedouro. Ao
internacional consuetudinário, conjuntos de princípios e
violar os direitos de um de seus cidadãos, o Estado estará
outras modalidades do Direito. A legislação de direitos
cometendo uma violação não apenas contra aquele
humanos obriga os Estados a agir de uma determinada
indivíduo, mas contra todos os Estados membros do
maneira e proíbe os Estados de se envolverem em
sistema. Dessa forma, os demais integrantes da
atividades específicas. No entanto, a legislação não
organização passam a ter o direito de cobrar de um de
estabelece os direitos humanos. Os direitos humanos são
seus semelhantes o cumprimento dessas diretrizes
direitos inerentes a cada pessoa simplesmente por ela ser
traçadas pela Declaração.
um humano.
Tratados e outras modalidades do Direito A citada Declaração tem caráter de resolução, e
costumam servir para proteger formalmente os direitos de não possui natureza jurídica de tratado stricto sensu,
indivíduos ou grupos contra ações ou abandono dos como os instrumentos que a seguiram. Apesar disso, é
governos, que interferem no desfrute de seus direitos um importante marco no tratamento do tema no âmbito
humanos. interamericano. E mais, ela obriga todos os membros da
organização1, pois a Carta da OEA exige a aplicação de
Uma das grandes conquistas das Nações
Unidas é a criação de um corpo abrangente de leis de
direitos humanos – um código universal e protegido 1
A Corte Interamericana dispôs sobre a matéria em 1989, após
internacionalmente, no qual todas as nações se podem consulta efetuada pelo Governo da Colômbia, nos termos da

www.editoradince.com - Acesse e veja se há novidades a respeito deste material – CUIDADO: cópia é crime.
20 D DIREITOS HUMANOS - Teoria esquematizada, dicas e questões de concurso
seus próprios dispositivos sobre direitos humanos em Humanos) e os demais instrumentos normativos do
combinação com a Declaração. sistema podem ser levados à apreciação da Comissão.
Advém de tal entendimento uma importante A Corte Interamericana, por sua vez, como órgão
conclusão. Alguns países membros do Sistema de caráter jurisdicional e que possui previsão expressa
Interamericano resistem em firmar compromissos apenas na Convenção Americana, julga somente aqueles
internacionais em matéria de direitos humanos. países que expressamente aceitaram sua competência
Notadamente, os Estados Unidos da América, Cuba e obrigatória e se submeteram à jurisdição daquele órgão
Canadá não ratificaram nenhum dos tratados colegiado.
interamericanos sobre o tema2. Todavia, ainda assim é
possível sua inserção no sistema de proteção dos direitos
2 A COMISSÃO INTERAMERICANA DE DIREITOS
humanos, uma vez que todos assinaram ou aderiram à
HUMANOS
Carta da OEA e se comprometeram internacionalmente a
respeitar a Declaração Americana. O Sistema Interamericano de Direitos Humanos,
como se observou no tópico supra, é bifásico, e conta
Onze anos após a adoção da Carta da OEA, em
com dois órgãos distintos, a Comissão Interamericana de
1959, na Quinta Reunião de Consulta dos Ministros das
Direitos Humanos e a Corte Interamericana de Direitos
Relações Exteriores, em Santiago do Chile, foi criada a
Humanos. No presente trabalho, analisaremos mais
Comissão Interamericana de Direitos Humanos.
detidamente a Comissão Interamericana.
Concebida inicialmente como órgão de consulta, a
Comissão teve suas atribuições ampliadas nos anos A Comissão Interamericana de Direitos Humanos
seguintes. possui dupla vinculação. Está prevista e se submete aos
regulamentos tanto da Carta da OEA quanto da
Em 1969, finalmente a Comissão Interamericana
Convenção Americana sobre Direitos Humanos. Ela é
foi introduzida ao texto da Carta da OEA, pelo Protocolo
composta por sete membros, eleitos pela Assembleia
de Buenos Aires3. Nesse mesmo ano, já uma década
Geral para mandatos de quatro anos, permitida uma
após a criação formal da Comissão, foi apresentado pelo
reeleição. Os eleitos são representantes não de seus
Conselho da OEA durante a Conferência Especializada
próprios países, mas de todos os Estados membros da
Americana sobre Direitos Humanos, em São José da
OEA, e se reúnem na sede da Comissão, em Washington,
Costa Rica, um projeto final de tratado internacional sobre
em pelo menos duas sessões ao ano.
o tema de direitos humanos, a chamada Convenção
Americana sobre Direitos Humanos. Além disso, os Comissionados podem realizar
visitas in loco aos Estados, a fim de averiguar aspectos
Nesse instrumento, que disciplina em detalhes os
referentes a casos específicos em trâmite ou para
deveres dos Estados membros da organização e estrutura
elaborar relatórios sobre a situação geral dos direitos
de forma definitiva o Sistema Interamericano de Proteção
humanos nos países visitados.
dos Direitos Humanos, previa-se a criação de uma Corte
para julgar as violações ocorridas na região. A convenção No âmbito da Comissão, existem diversas
entrou em vigor em 1978, após alcançar o mínimo de relatorias, que se dedicam a temas relevantes de direitos
onze ratificações, e, no ano seguinte, na mesma cidade humanos, como as questões das pessoas privadas de
de São José da Costa Rica, foi fundada a Corte liberdade ou das comunidades indígenas.
Interamericana de Direitos Humanos. Além da Carta da OEA, com seus Protocolos, e da
Ressalte-se que, como órgão integrante da Convenção Americana sobre Direitos Humanos, a
estrutura da OEA previsto no próprio texto da Carta, após Comissão Interamericana possui também estatuto e
a reforma trazida pelo Protocolo de Buenos Aires, à regulamento próprios. São estes instrumentos básicos
Comissão Interamericana estão submetidos todos os que disciplinam o funcionamento da instituição e o
Estados membros da organização. Até mesmo aqueles procedimento adotado quando ocorre uma denúncia de
que não ratificaram a Convenção Americana (que explicita violação.
as violações de forma mais clara e sistemática e estrutura Uma das características mais importantes da
o Sistema Interamericano de Proteção dos Direitos Comissão Interamericana é a possibilidade de postulação
atribuída a qualquer pessoa, grupo de pessoas ou
entidade não governamental. Alguém que sofra, presencie
Opinião Consultiva nº 10 do ano de 1989. À época, a Corte, ou tome conhecimento de uma violação de direitos
após convocar audiência pública a fim de ouvir os Estados humanos pode efetuar denúncia diretamente ao órgão da
membros da OEA interessados, sedimentou o entendimento OEA. Essa previsão, inserida no rol de atribuições da
que a Declaração Americana dos Direitos e Deveres do Comissão em 1965, na II Conferência Interamericana
Homem, embora seja uma resolução e não tenha a natureza Extraordinária, no Rio de Janeiro, aproxima o Sistema
jurídica de tratado stricto sensu, vincula todos os Estados Interamericano das vítimas. E de pessoas físicas e
membros da Organização dos Estados Americanos. organizações não governamentais, obviamente os
2
Convenção Americana sobre Direitos Humanos (1969), Estados também podem apresentar denúncias contra
Convenção Interamericana para Prevenir e Punir a Tortura outros países membros.
(1985), Protocolo Adicional à Convenção Americana sobre Ao receber uma denúncia de violação de direitos
Direitos Humanos em Matéria de Direitos Econômicos, Sociais humanos, a Comissão Interamericana deverá observar se
e Culturais (1988), Protocolo à Convenção Americana sobre estão presentes alguns requisitos essenciais. Entre tais
Direitos Humanos Referente à Abolição da Pena de Morte exigências, está aquele que é o princípio basilar dos
(1990), Convenção Interamericana sobre o Desaparecimento órgãos jurisdicionais internacionais: o prévio esgotamento
Forçado de Pessoas (1994), Convenção Interamericana para dos recursos internos. De acordo com esse preceito, um
Prevenir, Punir e Erradicar a Violência contra a Mulher (1994), Estado não pode ser acionado perante a jurisdição
Convenção Interamericana para a Eliminação de Todas as internacional sem que lhe seja permitido resolver a
Formas de Discriminação contra as Pessoas com Deficiência questão internamente.
(1999).
3 O órgão judicial internacional não pode substituir o
Cf. artigo 106 da Carta da OEA (após as reformas do Protocolo
Judiciário estatal, até mesmo em respeito à soberania dos
de Buenos Aires).

www.editoradince.com - Acesse e veja se há novidades a respeito deste material – CUIDADO: cópia é crime.
DIREITOS HUMANOS - Teoria esquematizada, dicas e questões de concursos 21
Estados. Apenas se esgotados todos os remédios porquê do não atendimento às medidas consignadas, o
disponíveis no âmbito interno, ou caso ocorra uma das Estado receberá um Segundo Informe da Comissão,
exceções ao esgotamento, como demora injustificada ou reiterando as recomendações.
ineficácia do recurso, é que a questão pode ultrapassar os Na hipótese do país não atender às
limites do Estado e ser levada ao foro internacional4. recomendações da Comissão, o caso pode ser levado à
Outro requisito relevante é a ausência de Corte Interamericana de Direitos Humanos, com a
litispendência internacional. Ou seja, um mesmo caso não anuência dos peticionários. Inicia-se, então, naquele
pode ser levado simultaneamente ao Sistema colegiado uma ação de responsabilidade internacional do
Interamericano de Proteção dos Direitos Humanos e ao Estado por violação de direitos humanos. Ao contrário do
Sistema Universal da Organização das Nações Unidas, que ocorre no primeiro órgão, a Corte Interamericana
deve-se optar por um dos mecanismos. apenas recebe casos por intermédio da Comissão ou dos
Presentes todos os requisitos, a petição será Estados-partes. Não há, no âmbito desse órgão
encaminhada ao Estado supostamente violador, para que jurisdicional, possibilidade de início do procedimento pelos
este se manifeste sobre os requisitos de admissibilidade peticionários, sejam estes as vítimas, seus familiares ou
da denúncia. Após, a Comissão chamará mais uma vez organizações não- governamentais. No entanto, após
as partes para que estas apresentem observações iniciado o procedimento, os peticionários participam de
adicionais, e então decidirá se admite ou não a petição. todas as fases do rito, possuindo cadeira cativa e voz
Caso positivo, há a abertura formal de um caso, e é atuante nas sessões de julgamento dos casos.
franqueada nova oportunidade para que os litigantes Muito se discute sobre o caráter das
firmem seus posicionamentos, desta vez sobre o mérito recomendações da Comissão Interamericana de Direitos
da questão. Humanos. A própria Comissão entende que suas
Nesse momento, surge, e é incentivada pela decisões têm efeitos vinculantes, haja vista emanarem de
Comissão, a possibilidade de negociação para se atingir um órgão integrante da estrutura da OEA. Decorreria da
uma solução amistosa. Politicamente pode ser própria Carta da organização internacional a
desgastante para o Estado ser reconhecido pelo Sistema obrigatoriedade de cumprimento das recomendações.
Interamericano como violador de direitos humanos. Essa Alguns Estados adotam esse entendimento e procuram
repercussão internacional que uma condenação pode encarar tais recomendações como obrigações assumidas
trazer a um Estado é o que se convencionou chamar de a nível internacional.
―caixa de ressonância‖5 ou ―power to embarass‖6, pois o Contudo, outros países acreditam que as
constrangimento trazido por um relatório de mérito da recomendações não passam exatamente disso,
Comissão Interamericana reconhecendo uma violação recomendações de um órgão político, e que seria a Corte
perante os membros da comunidade internacional, em o único órgão cujas decisões vinculariam juridicamente os
especial seus similares do continente americano, acaba Estados.
por, muitas vezes, surtir mais efeitos do que uma decisão Mesmo que as recomendações da Comissão nem
de efeitos concretos. sempre tenham a receptividade ou a concretização
É claro que os Estados estão sujeitos, em vários almejadas, não se pode negar, como já se mencionou
casos, às amarras de seu direito interno, que anteriormente, os efeitos da ―caixa de ressonância‖. Ainda
frequentemente impedem ou dificultam a realização de que o Estado não cumpra efetivamente as
acordos nesses litígios. Contudo, este é um instrumento recomendações, o simples fato de ser reconhecido como
de bastante valia para o sistema, e certamente é um de violador de direitos humanos por um foro multilateral é
seus mais eficazes mecanismos. Via de regra, há a prejudicial à sua imagem e à própria política externa
intermediação de um árbitro indicado pela Comissão, mas daquele país.
as negociações para a solução amistosa podem ocorrer
até mesmo no âmbito interno dos Estados.
3 A ATUAÇÃO DO BRASIL PERANTE A COMISSÃO
Superada a fase da solução amistosa sem que INTERAMERICANA DE DIREITOS HUMANOS
esta tenha um desfecho positivo, a Comissão
Dentre os foros multilaterais dos quais o Brasil faz
Interamericana tem duas possibilidades: ou decide que
parte, certamente o Sistema Interamericano de Direitos
não houve violação, ou manifesta-se pela ocorrência de
Humanos, em especial a Comissão Interamericana, é dos
violação a um ou mais dispositivos protegidos por
que mais destaque tem tido nos anos mais recentes. A
instrumento internacional. Neste último caso, a Comissão
defesa estatal perante a Comissão, que no início tinha um
apresenta relatório preliminar de recomendações, que é
cunho mais político e diplomático, sofreu uma evolução
transmitido ao Estado.
natural, em decorrência do adensamento jurisdicional
Esse Estado, que no momento já é considerado ocorrido em todo o sistema.
um violador de direitos humanos para todos os efeitos,
Nos anos 40 e 50, no período ainda anterior à
terá um prazo para se manifestar sobre o cumprimento
implementação da Comissão Interamericana, o Brasil
das recomendações. Caso silencie ou não justifique o
mostrou-se bastante ativo no âmbito da OEA ao lidar com
a matéria. Durante a negociação da Declaração
4
André de Carvalho Ramos esclarece que a regra do Americana de Direitos e Deveres do Homem, por
esgotamento dos recursos internos “obteve grande aceitação no exemplo, o chefe da delegação brasileira, João Neves da
Direito Internacional graças ao seu papel de redutor de tensões Fontoura, já sugeria a criação de uma Corte regional com
entre os Estados. Com efeito, é respeitada a soberania estatal ao o intuito de tutelar os direitos previstos naquele
se enfatizar o caráter subsidiário da jurisdição internacional, instrumento internacional, ambição que só viria a se
que só é acionada após o esgotamento dos recursos internos. Ao concretizar décadas mais tarde.
mesmo tempo, fornece-se uma alternativa de solução pacífica da Contudo, a postura brasileira a partir de meados
controvérsia ao Estado de origem do estrangeiro, que pugna por dos anos 60, com o início do regime militar, passou a ser
uma reparação de danos”. (Ramos, 2001, p. 75) defensiva e isolacionista, reflexo do autoritarismo que
5
Cf. Couto, 2006 e Piovesan, 2002. vigia à época. Os efeitos da Guerra Fria, que à época
6
Cf. Breda dos Santos, 1998. ainda polarizava o mundo, também contribuíam para esse

www.editoradince.com - Acesse e veja se há novidades a respeito deste material – CUIDADO: cópia é crime.
22 D DIREITOS HUMANOS - Teoria esquematizada, dicas e questões de concurso
afastamento. O país, talvez receoso das consequências A evolução do tema dos Direitos Humanos no
que poderiam advir do Sistema Interamericano de Direitos Brasil e a valorização do Sistema Interamericano de
Humanos, e possivelmente preocupado com os tais Proteção podem ser observadas a olhos vistos. Mas ainda
efeitos da ―caixa de ressonância‖, recusou-se a assinar a há muito que evoluir no tratamento da questão.
tão batalhada Convenção Americana sobre Direitos
Humanos.
CONVENÇÃO AMERICANA SOBRE DIREITOS
Apenas na década de 80, com o fim do governo HUMANOS
militar e o arrefecimento da Guerra Fria, iniciaram-se as
(Assinada na Conferência Especializada Interamericana
discussões para adesão e ratificação à Convenção. O
sobre Direitos Humanos,
ilustre professor Cançado Trindade, Consultor Jurídico do
Itamaraty após o fim do regime autoritário (e futuro Juiz da San José, Costa Rica, em 22 de novembro de 1969)
Corte Interamericana de Direitos Humanos), foi um dos PREÂMBULO
pioneiros na discussão da inserção do Brasil no Sistema Os Estados americanos signatários da presente
Interamericano de Proteção dos Direitos Humanos. Convenção,
O pontapé inicial das discussões foi dado em Reafirmando seu propósito de consolidar neste
1985, mas o efeito concreto – a ratificação da Convenção Continente, dentro do quadro das instituições democráticas,
Americana – só ocorreu em 1992, após a consolidação do um regime de liberdade pessoal e de justiça social, fundado
processo de redemocratização e a promulgação da no respeito dos direitos essenciais do homem;
chamada Constituição cidadã, que elevava os direitos Reconhecendo que os direitos essenciais do
humanos à condição de prevalência na política externa homem não derivam do fato de ser ele nacional de
pátria7. E alguns anos mais tarde, em 1998, é que o país determinado Estado, mas sim do fato de ter como
reconheceu a competência da Corte Interamericana. fundamento os atributos da pessoa humana, razão por que
Inicialmente, o Estado brasileiro, encarava a justificam uma proteção internacional, de natureza
Comissão Interamericana de Direitos Humanos mais convencional, coadjuvante ou complementar da que oferece
como um espaço para a diplomacia ventilar problemas de o direito interno dos Estados americanos;
cada Estado do que exatamente um órgão de contencioso Considerando que esses princípios foram
internacional. Com o passar dos anos, a estrutura estatal consagrados na Carta da Organização dos Estados
de defesa perante o sistema teve que se profissionalizar. Americanos, na Declaração Americana dos Direitos e
O aumento significativo do número de casos brasileiros Deveres do Homem e na Declaração Universal dos Direitos
perante a Comissão, aliado à estruturação das do Homem e que foram reafirmados e desenvolvidos em
organizações não-governamentais pátrias e ao início da outros instrumentos internacionais, tanto de âmbito mundial
admissão de casos brasileiros à fase de mérito, assim como regional;
como a elevação dos primeiros casos do Brasil à esfera Reiterando que, de acordo com a Declaração
da Corte Interamericana foram decisivos na mudança da Universal dos Direitos do Homem, só pode ser realizado o
atuação do Estado. ideal do ser humano livre, isento do temor e da miséria, se
Inexistem regras que disciplinem, no plano interno, forem criadas condições que permitam a cada pessoa gozar
de que forma deverá ser conduzida a defesa do Estado dos seus direitos econômicos, sociais e culturais, bem como
brasileiro perante a Comissão Interamericana. O certo é dos seus direitos civis e políticos; e
que as matérias levadas a esse foro serão sempre Considerando que a Terceira Conferência
complexas e delicadas, envolvendo aspectos políticos, Interamericana Extraordinária (Buenos Aires, 1967) aprovou
jurídicos e diplomáticos, do interesse de diversos atores. a incorporação à própria Carta da Organização de normas
mais amplas sobre direitos econômicos, sociais e
Dessarte, faz-se necessário que os órgãos da
educacionais e resolveu que uma convenção
Administração Pública, notadamente a Advocacia-Geral
interamericana sobre direitos humanos determinasse a
da União, o Ministério das Relações Exteriores e a
estrutura, competência e processo dos órgãos
Secretaria Especial dos Direitos Humanos, que seriam os
encarregados dessa matéria,
mais diretamente envolvidos nas questões levadas ao
Convieram no seguinte:
Sistema Interamericano, atuem conjuntamente, cada qual
dentro de suas esferas de competência, na condução da PARTE I
defesa do Estado brasileiro que será apresentada perante DEVERES DOS ESTADOS E DIREITOS PROTEGIDOS
a Comissão, órgão colegiado não jurisdicional da OEA. CAPÍTULO I
Nenhum desses participantes deve desempenhar ENUMERAÇÃO DE DEVERES
seu papel de forma isolada. A unificação do discurso e a Artigo 1. Obrigação de respeitar os direitos
atuação em coordenação deve ser buscada no plano 1. Os Estados Partes nesta Convenção comprometem-se a
interno, a fim de que a defesa estatal seja uníssona e respeitar os direitos e liberdades nela reconhecidos e a
coerente na esfera internacional. garantir seu livre e pleno exercício a toda pessoa que esteja
O ordenamento brasileiro também tenta sujeita à sua jurisdição, sem discriminação alguma por
acompanhar a evolução dos tempos. A recente reforma motivo de raça, cor, sexo, idioma, religião, opiniões políticas
do Judiciário que prevê status constitucional a tratados de ou de qualquer outra natureza, origem nacional ou social,
direitos humanos é um exemplo. Contudo, ainda estamos posição econômica, nascimento ou qualquer outra condição
longe de países como a Argentina, onde o Sistema social.
Interamericano de Proteção dos Direitos Humanos já é 2. Para os efeitos desta Convenção, pessoa é todo ser
tão enraizado que a Suprema Corte utiliza julgados da humano.
Corte Interamericana, mesmo que tratem de outros Artigo 2. Dever de adotar disposições de direito interno
Estados, como precedentes para suas decisões. Se o exercício dos direitos e liberdades mencionados no
artigo 1 ainda não estiver garantido por disposições
legislativas ou de outra natureza, os Estados Partes
7 comprometem-se a adotar, de acordo com as suas normas
Cf. artigo 4º da Constituição da República Federativa do Brasil
constitucionais e com as disposições desta Convenção, as
de 1988.

www.editoradince.com - Acesse e veja se há novidades a respeito deste material – CUIDADO: cópia é crime.
DIREITOS HUMANOS - Teoria esquematizada, dicas e questões de concursos 23
medidas legislativas ou de outra natureza que forem a. os trabalhos ou serviços normalmente exigidos
necessárias para tornar efetivos tais direitos e liberdades. de pessoa reclusa em cumprimento de
CAPÍTULO II sentença ou resolução formal expedida pela
DIREITOS CIVIS E POLÍTICOS autoridade judiciária competente. Tais
Artigo 3. Direito ao reconhecimento da personalidade trabalhos ou serviços devem ser executados
jurídica sob a vigilância e controle das autoridades
Toda pessoa tem direito ao reconhecimento de sua públicas, e os indivíduos que os executarem
personalidade jurídica. não devem ser postos à disposição de
particulares, companhias ou pessoas jurídicas
Artigo 4. Direito à vida
de caráter privado;
1. Toda pessoa tem o direito de que se respeite sua
b. o serviço militar e, nos países onde se admite a
vida. Esse direito deve ser protegido pela lei e, em geral,
isenção por motivos de consciência, o serviço
desde o momento da concepção. Ninguém pode ser
nacional que a lei estabelecer em lugar daquele;
privado da vida arbitrariamente.
c. o serviço imposto em casos de perigo ou
2. Nos países que não houverem abolido a pena de morte,
calamidade que ameace a existência ou o bem-
esta só poderá ser imposta pelos delitos mais graves, em
estar da comunidade; e
cumprimento de sentença final de tribunal competente e em
conformidade com lei que estabeleça tal pena, promulgada d. o trabalho ou serviço que faça parte das
antes de haver o delito sido cometido. Tampouco se obrigações cívicas normais.
estenderá sua aplicação a delitos aos quais não se aplique Artigo 7. Direito à liberdade pessoal
atualmente. 1. Toda pessoa tem direito à liberdade e à segurança
3. Não se pode restabelecer a pena de morte nos Estados pessoais.
que a hajam abolido. 2. Ninguém pode ser privado de sua liberdade física, salvo
4. Em nenhum caso pode a pena de morte ser aplicada por pelas causas e nas condições previamente fixadas pelas
delitos políticos, nem por delitos comuns conexos com constituições políticas dos Estados Partes ou pelas leis de
delitos políticos. acordo com elas promulgadas.
5. Não se deve impor a pena de morte a pessoa que, no 3. Ninguém pode ser submetido a detenção ou
momento da perpetração do delito, for menor de dezoito encarceramento arbitrários.
anos, ou maior de setenta, nem aplicá-la a mulher em 4. Toda pessoa detida ou retida deve ser informada das
estado de gravidez. razões da sua detenção e notificada, sem demora, da
6. Toda pessoa condenada à morte tem direito a solicitar acusação ou acusações formuladas contra ela.
anistia, indulto ou comutação da pena, os quais podem ser 5. Toda pessoa detida ou retida deve ser conduzida, sem
concedidos em todos os casos. Não se pode executar a demora, à presença de um juiz ou outra autoridade
pena de morte enquanto o pedido estiver pendente de autorizada pela lei a exercer funções judiciais e tem direito a
decisão ante a autoridade competente. ser julgada dentro de um prazo razoável ou a ser posta em
Artigo 5. Direito à integridade pessoal liberdade, sem prejuízo de que prossiga o processo. Sua
1. Toda pessoa tem o direito de que se respeite sua liberdade pode ser condicionada a garantias que assegurem
integridade física, psíquica e moral. o seu comparecimento em juízo.
2. Ninguém deve ser submetido a torturas, nem a penas ou 6. Toda pessoa privada da liberdade tem direito a recorrer a
tratos cruéis, desumanos ou degradantes. Toda pessoa um juiz ou tribunal competente, a fim de que este decida,
privada da liberdade deve ser tratada com o respeito devido sem demora, sobre a legalidade de sua prisão ou detenção
à dignidade inerente ao ser humano. e ordene sua soltura se a prisão ou a detenção forem
ilegais. Nos Estados Partes cujas leis prevêem que toda
3. A pena não pode passar da pessoa do delinqüente.
pessoa que se vir ameaçada de ser privada de sua
4. Os processados devem ficar separados dos condenados, liberdade tem direito a recorrer a um juiz ou tribunal
salvo em circunstâncias excepcionais, e ser submetidos a competente a fim de que este decida sobre a legalidade de
tratamento adequado à sua condição de pessoas não tal ameaça, tal recurso não pode ser restringido nem
condenadas. abolido. O recurso pode ser interposto pela própria pessoa
5. Os menores, quando puderem ser processados, devem ou por outra pessoa.
ser separados dos adultos e conduzidos a tribunal 7. Ninguém deve ser detido por dívidas. Este princípio não
especializado, com a maior rapidez possível, para seu limita os mandados de autoridade judiciária competente
tratamento. expedidos em virtude de inadimplemento de obrigação
6. As penas privativas da liberdade devem ter por finalidade alimentar.
essencial a reforma e a readaptação social dos Artigo 8. Garantias judiciais
condenados.
1. Toda pessoa tem direito a ser ouvida, com as devidas
Artigo 6. Proibição da escravidão e da servidão
garantias e dentro de um prazo razoável, por um juiz ou
1. Ninguém pode ser submetido a escravidão ou a tribunal competente, independente e imparcial, estabelecido
servidão, e tanto estas como o tráfico de escravos e o tráfico anteriormente por lei, na apuração de qualquer acusação
de mulheres são proibidos em todas as suas formas. penal formulada contra ela, ou para que se determinem
2. Ninguém deve ser constrangido a executar trabalho seus direitos ou obrigações de natureza civil, trabalhista,
forçado ou obrigatório. Nos países em que se prescreve, fiscal ou de qualquer outra natureza.
para certos delitos, pena privativa da liberdade 2. Toda pessoa acusada de delito tem direito a que se
acompanhada de trabalhos forçados, esta disposição não presuma sua inocência enquanto não se comprove
pode ser interpretada no sentido de que proíbe o legalmente sua culpa. Durante o processo, toda pessoa
cumprimento da dita pena, imposta por juiz ou tribunal tem direito, em plena igualdade, às seguintes garantias
competente. O trabalho forçado não deve afetar a mínimas:
dignidade nem a capacidade física e intelectual do recluso. a. direito do acusado de ser assistido
3. Não constituem trabalhos forçados ou obrigatórios para gratuitamente por tradutor ou intérprete, se não
os efeitos deste artigo:

www.editoradince.com - Acesse e veja se há novidades a respeito deste material – CUIDADO: cópia é crime.
24 D DIREITOS HUMANOS - Teoria esquematizada, dicas e questões de concurso
compreender ou não falar o idioma do juízo ou a ordem, a saúde ou a moral públicas ou os direitos ou
tribunal; liberdades das demais pessoas.
b. comunicação prévia e pormenorizada ao acusado 4. Os pais, e quando for o caso os tutores, têm direito a
da acusação formulada; que seus filhos ou pupilos recebam a educação religiosa e
c. concessão ao acusado do tempo e dos meios moral que esteja acorde com suas próprias convicções.
adequados para a preparação de sua defesa; Artigo 13. Liberdade de pensamento e de expressão
d. direito do acusado de defender-se pessoalmente 1. Toda pessoa tem direito à liberdade de pensamento e
ou de ser assistido por um defensor de sua de expressão. Esse direito compreende a liberdade de
escolha e de comunicar-se, livremente e em buscar, receber e difundir informações e idéias de toda
particular, com seu defensor; natureza, sem consideração de fronteiras, verbalmente ou
e. direito irrenunciável de ser assistido por um por escrito, ou em forma impressa ou artística, ou por
defensor proporcionado pelo Estado, qualquer outro processo de sua escolha.
remunerado ou não, segundo a legislação 2. O exercício do direito previsto no inciso precedente não
interna, se o acusado não se defender ele pode estar sujeito a censura prévia, mas a
próprio nem nomear defensor dentro do prazo responsabilidades ulteriores, que devem ser expressamente
estabelecido pela lei; fixadas pela lei e ser necessárias para assegurar:
f. direito da defesa de inquirir as testemunhas a. o respeito aos direitos ou à reputação das demais
presentes no tribunal e de obter o pessoas; ou
comparecimento, como testemunhas ou peritos, b. a proteção da segurança nacional, da ordem
de outras pessoas que possam lançar luz sobre pública, ou da saúde ou da moral públicas.
os fatos; 3. Não se pode restringir o direito de expressão por vias ou
g. direito de não ser obrigado a depor contra si meios indiretos, tais como o abuso de controles oficiais ou
mesma, nem a declarar-se culpada; particulares de papel de imprensa, de freqüências
e radioelétricas ou de equipamentos e aparelhos usados na
h. direito de recorrer da sentença para juiz ou difusão de informação, nem por quaisquer outros meios
tribunal superior. destinados a obstar a comunicação e a circulação de idéias
3. A confissão do acusado só é válida se feita sem coação e opiniões.
de nenhuma natureza. 4. A lei pode submeter os espetáculos públicos a censura
4. O acusado absolvido por sentença passada em julgado prévia, com o objetivo exclusivo de regular o acesso a eles,
não poderá ser submetido a novo processo pelos mesmos para proteção moral da infância e da adolescência, sem
fatos. prejuízo do disposto no inciso 2.
5. O processo penal deve ser público, salvo no que for 5. A lei deve proibir toda propaganda a favor da guerra, bem
necessário para preservar os interesses da justiça. como toda apologia ao ódio nacional, racial ou religioso que
Artigo 9. Princípio da legalidade e da retroatividade constitua incitação à discriminação, à hostilidade, ao crime
Ninguém pode ser condenado por ações ou omissões que, ou à violência.
no momento em que forem cometidas, não sejam Artigo 14. Direito de retificação ou resposta
delituosas, de acordo com o direito aplicável. Tampouco se 1. Toda pessoa atingida por informações inexatas ou
pode impor pena mais grave que a aplicável no momento da ofensivas emitidas em seu prejuízo por meios de difusão
perpetração do delito. Se depois da perpetração do delito a legalmente regulamentados e que se dirijam ao público em
lei dispuser a imposição de pena mais leve, o delinqüente geral, tem direito a fazer, pelo mesmo órgão de difusão, sua
será por isso beneficiado. retificação ou resposta, nas condições que estabeleça a lei.
Artigo 10. Direito a indenização 2. Em nenhum caso a retificação ou a resposta eximirão das
Toda pessoa tem direito de ser indenizada conforme a lei, outras responsabilidades legais em que se houver incorrido.
no caso de haver sido condenada em sentença passada em 3. Para a efetiva proteção da honra e da reputação, toda
julgado, por erro judiciário. publicação ou empresa jornalística, cinematográfica, de
Artigo 11. Proteção da honra e da dignidade rádio ou televisão, deve ter uma pessoa responsável que
1. Toda pessoa tem direito ao respeito de sua honra e ao não seja protegida por imunidades nem goze de foro
reconhecimento de sua dignidade. especial.
2. Ninguém pode ser objeto de ingerências arbitrárias ou Artigo 15. Direito de reunião
abusivas em sua vida privada, na de sua família, em seu É reconhecido o direito de reunião pacífica e sem armas. O
domicílio ou em sua correspondência, nem de ofensas exercício de tal direito só pode estar sujeito às restrições
ilegais à sua honra ou reputação. previstas pela lei e que sejam necessárias, numa sociedade
3. Toda pessoa tem direito à proteção da lei contra tais democrática, no interesse da segurança nacional, da
ingerências ou tais ofensas. segurança ou da ordem públicas, ou para proteger a saúde
Artigo 12. Liberdade de consciência e de religião ou a moral públicas ou os direitos e liberdades das demais
pessoas.
1. Toda pessoa tem direito à liberdade de consciência e de
religião. Esse direito implica a liberdade de conservar sua Artigo 16. Liberdade de associação
religião ou suas crenças, ou de mudar de religião ou de 1. Todas as pessoas têm o direito de associar-se livremente
crenças, bem como a liberdade de professar e divulgar sua com fins ideológicos, religiosos, políticos, econômicos,
religião ou suas crenças, individual ou coletivamente, tanto trabalhistas, sociais, culturais, desportivos ou de qualquer
em público como em privado. outra natureza.
2. Ninguém pode ser objeto de medidas restritivas que 2. O exercício de tal direito só pode estar sujeito às
possam limitar sua liberdade de conservar sua religião ou restrições previstas pela lei que sejam necessárias, numa
suas crenças, ou de mudar de religião ou de crenças. sociedade democrática, no interesse da segurança nacional,
3. A liberdade de manifestar a própria religião e as próprias da segurança ou da ordem públicas, ou para proteger a
crenças está sujeita unicamente às limitações prescritas saúde ou a moral públicas ou os direitos e liberdades das
pela lei e que sejam necessárias para proteger a segurança, demais pessoas.

www.editoradince.com - Acesse e veja se há novidades a respeito deste material – CUIDADO: cópia é crime.
DIREITOS HUMANOS - Teoria esquematizada, dicas e questões de concursos 25
3. O disposto neste artigo não impede a imposição de 6. O estrangeiro que se ache legalmente no território de um
restrições legais, e mesmo a privação do exercício do direito Estado Parte nesta Convenção só poderá dele ser expulso
de associação, aos membros das forças armadas e da em cumprimento de decisão adotada de acordo com a lei.
polícia. 7. Toda pessoa tem o direito de buscar e receber asilo em
Artigo 17. Proteção da família território estrangeiro, em caso de perseguição por delitos
1. A família é o elemento natural e fundamental da políticos ou comuns conexos com delitos políticos e de
sociedade e deve ser protegida pela sociedade e pelo acordo com a legislação de cada Estado e com os
Estado. convênios internacionais.
2. É reconhecido o direito do homem e da mulher de 8. Em nenhum caso o estrangeiro pode ser expulso ou
contraírem casamento e de fundarem uma família, se entregue a outro país, seja ou não de origem, onde seu
tiverem a idade e as condições para isso exigidas pelas leis direito à vida ou à liberdade pessoal esteja em risco de
internas, na medida em que não afetem estas o princípio da violação por causa da sua raça, nacionalidade, religião,
não-discriminação estabelecido nesta Convenção. condição social ou de suas opiniões políticas.
3. O casamento não pode ser celebrado sem o livre e pleno 9. É proibida a expulsão coletiva de estrangeiros.
consentimento dos contraentes. Artigo 23. Direitos políticos
4. Os Estados Partes devem tomar medidas apropriadas no 1. Todos os cidadãos devem gozar dos seguintes direitos e
sentido de assegurar a igualdade de direitos e a adequada oportunidades:
equivalência de responsabilidades dos cônjuges quanto ao a. de participar na direção dos assuntos públicos,
casamento, durante o casamento e em caso de dissolução diretamente ou por meio de representantes
do mesmo. Em caso de dissolução, serão adotadas livremente eleitos;
disposições que assegurem a proteção necessária aos b. de votar e ser eleitos em eleições periódicas
filhos, com base unicamente no interesse e conveniência autênticas, realizadas por sufrágio universal e
dos mesmos. igual e por voto secreto que garanta a livre
5. A lei deve reconhecer iguais direitos tanto aos filhos expressão da vontade dos eleitores; e
nascidos fora do casamento como aos nascidos dentro do c. de ter acesso, em condições gerais de
casamento. igualdade, às funções públicas de seu país.
Artigo 18. Direito ao nome 2. A lei pode regular o exercício dos direitos e
Toda pessoa tem direito a um prenome e aos nomes de oportunidades a que se refere o inciso anterior,
seus pais ou ao de um destes. A lei deve regular a forma de exclusivamente por motivos de idade, nacionalidade,
assegurar a todos esse direito, mediante nomes fictícios, se residência, idioma, instrução, capacidade civil ou mental, ou
for necessário. condenação, por juiz competente, em processo penal.
Artigo 19. Direitos da criança Artigo 24. Igualdade perante a lei
Toda criança tem direito às medidas de proteção que a sua Todas as pessoas são iguais perante a lei. Por
condição de menor requer por parte da sua família, da conseguinte, têm direito, sem discriminação, a igual
sociedade e do Estado. proteção da lei.
Artigo 20. Direito à nacionalidade Artigo 25. Proteção judicial
1. Toda pessoa tem direito a uma nacionalidade. 1. Toda pessoa tem direito a um recurso simples e rápido
2. Toda pessoa tem direito à nacionalidade do Estado em ou a qualquer outro recurso efetivo, perante os juízes ou
cujo território houver nascido, se não tiver direito a outra. tribunais competentes, que a proteja contra atos que violem
3. A ninguém se deve privar arbitrariamente de sua seus direitos fundamentais reconhecidos pela constituição,
nacionalidade nem do direito de mudá-la. pela lei ou pela presente Convenção, mesmo quando tal
Artigo 21. Direito à propriedade privada violação seja cometida por pessoas que estejam atuando no
1. Toda pessoa tem direito ao uso e gozo dos seus bens. A exercício de suas funções oficiais.
lei pode subordinar esse uso e gozo ao interesse social. 2. Os Estados Partes comprometem-se:
2. Nenhuma pessoa pode ser privada de seus bens, salvo a. a assegurar que a autoridade competente
mediante o pagamento de indenização justa, por motivo de prevista pelo sistema legal do Estado decida
utilidade pública ou de interesse social e nos casos e na sobre os direitos de toda pessoa que interpuser
forma estabelecidos pela lei. tal recurso;
3. Tanto a usura como qualquer outra forma de exploração b. a desenvolver as possibilidades de recurso
do homem pelo homem devem ser reprimidas pela lei. judicial; e
Artigo 22. Direito de circulação e de residência c. a assegurar o cumprimento, pelas autoridades
1. Toda pessoa que se ache legalmente no território de um competentes, de toda decisão em que se tenha
Estado tem direito de circular nele e de nele residir em considerado procedente o recurso.
conformidade com as disposições legais. CAPÍTULO III
2. Toda pessoa tem o direito de sair livremente de qualquer DIREITOS ECONÔMICOS, SOCIAIS E CULTURAIS
país, inclusive do próprio. Artigo 26. Desenvolvimento progressivo
3. O exercício dos direitos acima mencionados não pode Os Estados Partes comprometem-se a adotar providências,
ser restringido senão em virtude de lei, na medida tanto no âmbito interno como mediante cooperação
indispensável, numa sociedade democrática, para prevenir internacional, especialmente econômica e técnica, a fim de
infrações penais ou para proteger a segurança nacional, a conseguir progressivamente a plena efetividade dos direitos
segurança ou a ordem públicas, a moral ou a saúde que decorrem das normas econômicas, sociais e sobre
públicas, ou os direitos e liberdades das demais pessoas. educação, ciência e cultura, constantes da Carta da
4. O exercício dos direitos reconhecidos no inciso 1 pode Organização dos Estados Americanos, reformada pelo
também ser restringido pela lei, em zonas determinadas, por Protocolo de Buenos Aires, na medida dos recursos
motivo de interesse público. disponíveis, por via legislativa ou por outros meios
5. Ninguém pode ser expulso do território do Estado do qual apropriados.
for nacional, nem ser privado do direito de nele entrar. CAPÍTULO IV

www.editoradince.com - Acesse e veja se há novidades a respeito deste material – CUIDADO: cópia é crime.
26 D DIREITOS HUMANOS - Teoria esquematizada, dicas e questões de concurso
SUSPENSÃO DE GARANTIAS, INTERPRETAÇÃO E Homem e outros atos internacionais da mesma
APLICAÇÃO natureza.
Artigo 27. Suspensão de garantias Artigo 30. Alcance das restrições
1. Em caso de guerra, de perigo público, ou de outra As restrições permitidas, de acordo com esta Convenção,
emergência que ameace a independência ou segurança do ao gozo e exercício dos direitos e liberdades nela
Estado Parte, este poderá adotar disposições que, na reconhecidos, não podem ser aplicadas senão de acordo
medida e pelo tempo estritamente limitados às exigências com leis que forem promulgadas por motivo de interesse
da situação, suspendam as obrigações contraídas em geral e com o propósito para o qual houverem sido
virtude desta Convenção, desde que tais disposições não estabelecidas.
sejam incompatíveis com as demais obrigações que lhe Artigo 31. Reconhecimento de outros direitos
impõe o Direito Internacional e não encerrem discriminação Poderão ser incluídos no regime de proteção desta
alguma fundada em motivos de raça, cor, sexo, idioma, Convenção outros direitos e liberdades que forem
religião ou origem social. reconhecidos de acordo com os processos estabelecidos
2. A disposição precedente não autoriza a suspensão dos nos artigos 76 e 77.
direitos determinados seguintes artigos: 3 (Direito ao CAPÍTULO V
reconhecimento da personalidade jurídica); 4 (Direito à DEVERES DAS PESSOAS
vida); 5 (Direito à integridade pessoal); 6 (Proibição da Artigo 32. orrelação entre deveres e direitos
escravidão e servidão); 9 (Princípio da legalidade e da
1. Toda pessoa tem deveres para com a família, a
retroatividade); 12 (Liberdade de consciência e de religião);
comunidade e a humanidade.
17 (Proteção da família); 18 (Direito ao nome); 19 (Direitos
da criança); 20 (Direito à nacionalidade) e 23 (Direitos 2. Os direitos de cada pessoa são limitados pelos direitos
políticos), nem das garantias indispensáveis para a proteção dos demais, pela segurança de todos e pelas justas
de tais direitos. exigências do bem comum, numa sociedade democrática.
3. Todo Estado Parte que fizer uso do direito de suspensão
deverá informar imediatamente os outros Estados Partes na PARTE II
presente Convenção, por intermédio do Secretário-Geral da MEIOS DA PROTEÇÃO
Organização dos Estados Americanos, das disposições cuja CAPÍTULO VI
aplicação haja suspendido, dos motivos determinantes da ÓRGÃOS COMPETENTES
suspensão e da data em que haja dado por terminada tal Artigo 33
suspensão. São competentes para conhecer dos assuntos relacionados
Artigo 28. Cláusula federal com o cumprimento dos compromissos assumidos pelos
1. Quando se tratar de um Estado Parte constituído como Estados Partes nesta Convenção:
Estado federal, o governo nacional do aludido Estado Parte a. a Comissão Interamericana de Direitos
cumprirá todas as disposições da presente Convenção, Humanos, doravante denominada a Comissão;
relacionadas com as matérias sobre as quais exerce e
competência legislativa e judicial. b. a Corte Interamericana de Direitos Humanos,
2. No tocante às disposições relativas às matérias que doravante denominada a Corte.
correspondem à competência das entidades componentes CAPÍTULO VII
da federação, o governo nacional deve tomar COMISSÃO INTERAMERICANA DE DIREITOS
imediatamente as medidas pertinente, em conformidade HUMANOS
com sua constituição e suas leis, a fim de que as Seção 1 — Organização
autoridades competentes das referidas entidades possam Artigo 34
adotar as disposições cabíveis para o cumprimento desta
A Comissão Interamericana de Direitos Humanos compor-
Convenção.
se-á de sete membros, que deverão ser pessoas de alta
3. Quando dois ou mais Estados Partes decidirem constituir
autoridade moral e de reconhecido saber em matéria de
entre eles uma federação ou outro tipo de associação,
direitos humanos.
diligenciarão no sentido de que o pacto comunitário
Artigo 35
respectivo contenha as disposições necessárias para que
continuem sendo efetivas no novo Estado assim organizado A Comissão representa todos os membros da Organização
as normas da presente Convenção. dos Estados Americanos.
Artigo 29. Normas de interpretação Artigo 36
Nenhuma disposição desta Convenção pode ser 1. Os membros da Comissão serão eleitos a título pessoal,
interpretada no sentido de: pela Assembléia Geral da Organização, de uma lista de
candidatos propostos pelos governos dos Estados
a. permitir a qualquer dos Estados Partes, grupo ou
membros.
pessoa, suprimir o gozo e exercício dos direitos e
liberdades reconhecidos na Convenção ou limitá- 2. Cada um dos referidos governos pode propor até três
los em maior medida do que a nela prevista; candidatos, nacionais do Estado que os propuser ou de
qualquer outro Estado membro da Organização dos
b. limitar o gozo e exercício de qualquer direito ou
Estados Americanos. Quando for proposta uma lista de três
liberdade que possam ser reconhecidos de
candidatos, pelo menos um deles deverá ser nacional de
acordo com as leis de qualquer dos Estados
Estado diferente do proponente.
Partes ou de acordo com outra convenção em
que seja parte um dos referidos Estados; Artigo 37
c. excluir outros direitos e garantias que são 1. Os membros da Comissão serão eleitos por quatro anos
inerentes ao ser humano ou que decorrem da e só poderão ser reeleitos uma vez, porém o mandato de
forma democrática representativa de governo; e três dos membros designados na primeira eleição expirará
ao cabo de dois anos. Logo depois da referida eleição,
d. excluir ou limitar o efeito que possam produzir a
serão determinados por sorteio, na Assembléia Geral, os
Declaração Americana dos Direitos e Deveres do
nomes desses três membros.

www.editoradince.com - Acesse e veja se há novidades a respeito deste material – CUIDADO: cópia é crime.
DIREITOS HUMANOS - Teoria esquematizada, dicas e questões de concursos 27
2. Não pode fazer parte da Comissão mais de um nacional Qualquer pessoa ou grupo de pessoas, ou entidade não-
de um mesmo Estado. governamental legalmente reconhecida em um ou mais
Artigo 38 Estados membros da Organização, pode apresentar à
As vagas que ocorrerem na Comissão, que não se devam Comissão petições que contenham denúncias ou queixas
à expiração normal do mandato, serão preenchidas pelo de violação desta Convenção por um Estado Parte.
Conselho Permanente da Organização, de acordo com o Artigo 45
que dispuser o Estatuto da Comissão. 1. Todo Estado Parte pode, no momento do depósito do
Artigo 39 seu instrumento de ratificação desta Convenção ou de
A Comissão elaborará seu estatuto e submetê-lo-á à adesão a ela, ou em qualquer momento posterior, declarar
aprovação da Assembléia Geral e expedirá seu próprio que reconhece a competência da Comissão para receber e
regulamento. examinar as comunicações em que um Estado Parte alegue
Artigo 40 haver outro Estado Parte incorrido em violações dos direitos
Os serviços de secretaria da Comissão devem ser humanos estabelecidos nesta Convenção.
desempenhados pela unidade funcional especializada que 2. As comunicações feitas em virtude deste artigo só
faz parte da Secretaria-Geral da Organização e devem podem ser admitidas e examinadas se forem apresentadas
dispor dos recursos necessários para cumprir as tarefas que por um Estado Parte que haja feito uma declaração pela
lhe forem confiadas pela Comissão. qual reconheça a referida competência da Comissão. A
Seção 2 — Funções Comissão não admitirá nenhuma comunicação contra um
Estado Parte que não haja feito tal declaração.
Artigo 41
3. As declarações sobre reconhecimento de competência
A Comissão tem a função principal de promover a
podem ser feitas para que esta vigore por tempo indefinido,
observância e a defesa dos direitos humanos e, no exercício
por período determinado ou para casos específicos.
do seu mandato, tem as seguintes funções e atribuições:a.
estimular a consciência dos direitos humanos nos povos da 4. As declarações serão depositadas na Secretaria-Geral
América; da Organização dos Estados Americanos, a qual
encaminhará cópia das mesmas aos Estados membros da
b. formular recomendações aos governos dos
referida Organização.
Estados membros, quando o considerar
conveniente, no sentido de que adotem medidas Artigo 46
progressivas em prol dos direitos humanos no 1. Para que uma petição ou comunicação apresentada de
âmbito de suas leis internas e seus preceitos acordo com os artigos 44 ou 45 seja admitida pela
constitucionais, bem como disposições Comissão, será necessário:
apropriadas para promover o devido respeito a a. que hajam sido interpostos e esgotados os
esses direitos; recursos da jurisdição interna, de acordo com
c. preparar os estudos ou relatórios que considerar os princípios de direito internacional
convenientes para o desempenho de suas geralmente reconhecidos;
funções; b. que seja apresentada dentro do prazo de seis
d. solicitar aos governos dos Estados membros que meses, a partir da data em que o presumido
lhe proporcionem informações sobre as medidas prejudicado em seus direitos tenha sido
que adotarem em matéria de direitos humanos; notificado da decisão definitiva;
e. atender às consultas que, por meio da Secretaria- c. que a matéria da petição ou comunicação não
Geral da Organização dos Estados Americanos, esteja pendente de outro processo de
lhe formularem os Estados membros sobre solução internacional; e
questões relacionadas com os direitos humanos d. que, no caso do artigo 44, a petição contenha o
e, dentro de suas possibilidades, prestar-lhes o nome, a nacionalidade, a profissão, o
assessoramento que eles lhe solicitarem; domicílio e a assinatura da pessoa ou
f. atuar com respeito às petições e outras pessoas ou do representante legal da
comunicações, no exercício de sua autoridade, entidade que submeter a petição.
de conformidade com o disposto nos artigos 44 a 2. As disposições das alíneas a e b do inciso 1 deste
51 desta Convenção; e artigo não se aplicarão quando:
g. apresentar um relatório anual à Assembléia Geral a. não existir, na legislação interna do Estado de
da Organização dos Estados Americanos. que se tratar, o devido processo legal para a
Artigo 42 proteção do direito ou direitos que se alegue
Os Estados Partes devem remeter à Comissão cópia dos tenham sido violados;
relatórios e estudos que, em seus respectivos campos, b. não se houver permitido ao presumido
submetem anualmente às Comissões Executivas do prejudicado em seus direitos o acesso aos
Conselho Interamericano Econômico e Social e do recursos da jurisdição interna, ou houver sido
Conselho Interamericano de Educação, Ciência e Cultura, a ele impedido de esgotá-los; e
fim de que aquela vele por que se promovam os direitos c. houver demora injustificada na decisão sobre os
decorrentes das normas econômicas, sociais e sobre mencionados recursos.
educação, ciência e cultura, constantes da Carta da Artigo 47
Organização dos Estados Americanos, reformada pelo A Comissão declarará inadmissível toda petição ou
Protocolo de Buenos Aires. comunicação apresentada de acordo com os artigos 44 ou
Artigo 43 45 quando:
Os Estados Partes obrigam-se a proporcionar à Comissão a. não preencher algum dos requisitos estabelecidos no
as informações que esta lhes solicitar sobre a maneira pela artigo 46;
qual o seu direito interno assegura a aplicação efetiva de b. não expuser fatos que caracterizem violação dos
quaisquer disposições desta Convenção. direitos garantidos por esta Convenção;
Seção 3 — Competência c. pela exposição do próprio peticionário ou do
Artigo 44 Estado, for manifestamente infundada a petição

www.editoradince.com - Acesse e veja se há novidades a respeito deste material – CUIDADO: cópia é crime.
28 D DIREITOS HUMANOS - Teoria esquematizada, dicas e questões de concurso
ou comunicação ou for evidente sua total Artigo 50
improcedência; ou 1. Se não se chegar a uma solução, e dentro do prazo que
d. for substancialmente reprodução de petição ou for fixado pelo Estatuto da Comissão, esta redigirá um
comunicação anterior, já examinada pela relatório no qual exporá os fatos e suas conclusões. Se o
Comissão ou por outro organismo internacional. relatório não representar, no todo ou em parte, o acordo
Seção 4 — Processo unânime dos membros da Comissão, qualquer deles poderá
Artigo 48 agregar ao referido relatório seu voto em
1. A Comissão, ao receber uma petição ou comunicação na separado. Também se agregarão ao relatório as
qual se alegue violação de qualquer dos direitos exposições verbais ou escritas que houverem sido feitas
consagrados nesta Convenção, procederá da seguinte pelos interessados em virtude do inciso 1, e, do artigo 48.
maneira: 2. O relatório será encaminhado aos Estados interessados,
a. se reconhecer a admissibilidade da petição ou aos quais não será facultado publicá-lo.
comunicação, solicitará informações ao 3. Ao encaminhar o relatório, a Comissão pode formular as
Governo do Estado ao qual pertença a proposições e recomendações que julgar adequadas.
autoridade apontada como responsável pela Artigo 51
violação alegada e transcreverá as partes 1. Se no prazo de três meses, a partir da remessa aos
pertinentes da petição ou comunicação. As Estados interessados do relatório da Comissão, o assunto
referidas informações devem ser enviadas não houver sido solucionado ou submetido à decisão da
dentro de um prazo razoável, fixado pela Corte pela Comissão ou pelo Estado interessado, aceitando
Comissão ao considerar as circunstâncias de sua competência, a Comissão poderá emitir, pelo voto da
cada caso; maioria absoluta dos seus membros, sua opinião e
b. recebidas as informações, ou transcorrido o conclusões sobre a questão submetida à sua consideração.
prazo fixado sem que sejam elas recebidas, 2. A Comissão fará as recomendações pertinentes e fixará
verificará se existem ou subsistem os motivos um prazo dentro do qual o Estado deve tomar as medidas
da petição ou comunicação. No caso de não que lhe competirem para remediar a situação examinada.
existirem ou não subsistirem, mandará 3. Transcorrido o prazo fixado, a Comissão decidirá, pelo
arquivar o expediente; voto da maioria absoluta dos seus membros, se o Estado
c. poderá também declarar a inadmissibilidade ou a tomou ou não medidas adequadas e se publica ou não seu
improcedência da petição ou comunicação, relatório.
com base em informação ou prova CAPÍTULO VIII
supervenientes; CORTE INTERAMERICANA DE DIREITOS HUMANOS
d. se o expediente não houver sido arquivado, e Seção 1 — Organização
com o fim de comprovar os fatos, a Comissão Artigo 52
procederá, com conhecimento das partes, a 1. A Corte compor-se-á de sete juízes, nacionais dos
um exame do assunto exposto na petição ou Estados membros da Organização, eleitos a título pessoal
comunicação. Se for necessário e dentre juristas da mais alta autoridade moral, de
conveniente, a Comissão procederá a uma reconhecida competência em matéria de direitos humanos,
investigação para cuja eficaz realização que reúnam as condições requeridas para o exercício das
solicitará, e os Estados interessados lhes mais elevadas funções judiciais, de acordo com a lei do
proporcionarão todas as facilidades Estado do qual sejam nacionais, ou do Estado que os
necessárias; propuser como candidatos.
e. poderá pedir aos Estados interessados qualquer 2. Não deve haver dois juízes da mesma nacionalidade.
informação pertinente e receberá, se isso lhe
Artigo 53
for solicitado, as exposições verbais ou
escritas que apresentarem os interessados; 1. Os juízes da Corte serão eleitos, em votação secreta e
e pelo voto da maioria absoluta dos Estados Partes na
Convenção, na Assembléia Geral da Organização, de uma
f. pôr-se-á à disposição das partes interessadas, a
lista de candidatos propostos pelos mesmos Estados.
fim de chegar a uma solução amistosa do
assunto, fundada no respeito aos direitos 2. Cada um dos Estados Partes pode propor até três
humanos reconhecidos nesta Convenção. candidatos, nacionais do Estado que os propuser ou de
qualquer outro Estado membro da Organização dos
2. Entretanto, em casos graves e urgentes, pode ser
Estados Americanos. Quando se propuser uma lista de três
realizada uma investigação, mediante prévio consentimento
candidatos, pelo menos um deles deverá ser nacional de
do Estado em cujo território se alegue haver sido cometida a
Estado diferente do proponente.
violação, tão somente com a apresentação de uma petição
Artigo 54
ou comunicação que reúna todos os requisitos formais de
admissibilidade. 1. Os juízes da Corte serão eleitos por um período de seis
Artigo 49 anos e só poderão ser reeleitos uma vez. O mandato de
três dos juízes designados na primeira eleição expirará ao
Se se houver chegado a uma solução amistosa de acordo
cabo de três anos. Imediatamente depois da referida
com as disposições do inciso 1, f, do artigo 48, a Comissão
eleição, determinar-se-ão por sorteio, na Assembléia Geral,
redigirá um relatório que será encaminhado ao peticionário e
os nomes desses três juízes.
aos Estados Partes nesta Convenção e, posteriormente,
transmitido, para sua publicação, ao Secretário-Geral da 2. O juiz eleito para substituir outro cujo mandato não haja
Organização dos Estados Americanos. O referido relatório expirado, completará o período deste.
conterá uma breve exposição dos fatos e da solução 3. Os juízes permanecerão em funções até o término dos
alcançada. Se qualquer das partes no caso o solicitar, ser- seus mandatos. Entretanto, continuarão funcionando nos
lhe-á proporcionada a mais ampla informação possível. casos de que já houverem tomado conhecimento e que se
encontrem em fase de sentença e, para tais efeitos, não
serão substituídos pelos novos juízes eleitos.

www.editoradince.com - Acesse e veja se há novidades a respeito deste material – CUIDADO: cópia é crime.
DIREITOS HUMANOS - Teoria esquematizada, dicas e questões de concursos 29
Artigo 55 Geral da Organização, que encaminhará cópias da mesma
1. O juiz que for nacional de algum dos Estados Partes no aos outros Estados membros da Organização e ao
caso submetido à Corte, conservará o seu direito de Secretário da Corte.
conhecer do mesmo. 3. A Corte tem competência para conhecer de qualquer
2. Se um dos juízes chamados a conhecer do caso for de caso relativo à interpretação e aplicação das disposições
nacionalidade de um dos Estados Partes, outro Estado desta Convenção que lhe seja submetido, desde que os
Parte no caso poderá designar uma pessoa de sua escolha Estados Partes no caso tenham reconhecido ou
para fazer parte da Corte na qualidade de juiz ad hoc. reconheçam a referida competência, seja por declaração
3. Se, dentre os juízes chamados a conhecer do caso, especial, como prevêem os incisos anteriores, seja por
nenhum for da nacionalidade dos Estados Partes, cada um convenção especial.
destes poderá designar um juiz ad hoc. Artigo 63
4. O juiz ad hoc deve reunir os requisitos indicados no 1. Quando decidir que houve violação de um direito ou
artigo 52. liberdade protegidos nesta Convenção, a Corte determinará
5. Se vários Estados Partes na Convenção tiverem o que se assegure ao prejudicado o gozo do seu direito ou
mesmo interesse no caso, serão considerados como uma liberdade violados. Determinará também, se isso for
só Parte, para os fins das disposições anteriores. Em caso procedente, que sejam reparadas as conseqüências da
de dúvida, a Corte decidirá. medida ou situação que haja configurado a violação desses
Artigo 56 direitos, bem como o pagamento de indenização justa à
O quorum para as deliberações da Corte é constituído por parte lesada.
cinco juízes. 2. Em casos de extrema gravidade e urgência, e quando se
Artigo 57 fizer necessário evitar danos irreparáveis às pessoas, a
Corte, nos assuntos de que estiver conhecendo, poderá
A Comissão comparecerá em todos os casos perante a
tomar as medidas provisórias que considerar
Corte.
pertinentes. Se se tratar de assuntos que ainda não
Artigo 58
estiverem submetidos ao seu conhecimento, poderá atuar a
1. A Corte terá sua sede no lugar que for determinado, na pedido da Comissão.
Assembléia Geral da Organização, pelos Estados Partes na Artigo 64
Convenção, mas poderá realizar reuniões no território de
1. Os Estados membros da Organização poderão consultar
qualquer Estado membro da Organização dos Estados
a Corte sobre a interpretação desta Convenção ou de outros
Americanos em que o considerar conveniente pela maioria
tratados concernentes à proteção dos direitos humanos nos
dos seus membros e mediante prévia aquiescência do
Estados americanos. Também poderão consultá-la, no que
Estado respectivo. Os Estados Partes na Convenção
lhes compete, os órgãos enumerados no capítulo X da
podem, na Assembléia Geral, por dois terços dos seus
Carta da Organização dos Estados Americanos, reformada
votos, mudar a sede da Corte.
pelo Protocolo de Buenos Aires.
2. A Corte designará seu Secretário.
2. A Corte, a pedido de um Estado membro da
3. O Secretário residirá na sede da Corte e deverá assistir Organização, poderá emitir pareceres sobre a
às reuniões que ela realizar fora da mesma. compatibilidade entre qualquer de suas leis internas e os
Artigo 59 mencionados instrumentos internacionais.
A Secretaria da Corte será por esta estabelecida e Artigo 65
funcionará sob a direção do Secretário da Corte, de acordo A Corte submeterá à consideração da Assembléia Geral
com as normas administrativas da Secretaria-Geral da da Organização, em cada período ordinário de sessões, um
Organização em tudo o que não for incompatível com a relatório sobre suas atividades no ano anterior. De maneira
independência da Corte. Seus funcionários serão especial, e com as recomendações pertinentes, indicará os
nomeados pelo Secretário-Geral da Organização, em casos em que um Estado não tenha dado cumprimento a
consulta com o Secretário da Corte. suas sentenças.
Seção 3 — Procedimento
Artigo 60 Artigo 66
A Corte elaborará seu estatuto e submetê-lo-á à aprovação 1. A sentença da Corte deve ser fundamentada.
da Assembléia Geral e expedirá seu regimento.
2. Se a sentença não expressar no todo ou em parte a
Seção 2 — Competência e funções
opinião unânime dos juízes, qualquer deles terá direito a
Artigo 61 que se agregue à sentença o seu voto dissidente ou
1. Somente os Estados Partes e a Comissão têm direito de individual.
submeter caso à decisão da Corte. Artigo 67
2. Para que a Corte possa conhecer de qualquer caso, é A sentença da Corte será definitiva e inapelável. Em caso
necessário que sejam esgotados os processos previstos de divergência sobre o sentido ou alcance da sentença, a
nos artigos 48 a 50. Corte interpretá-la-á, a pedido de qualquer das partes,
desde que o pedido seja apresentado dentro de noventa
Artigo 62 dias a partir da data da notificação da sentença.
1. Todo Estado Parte pode, no momento do depósito do Artigo 68
seu instrumento de ratificação desta Convenção ou de 1. Os Estados Partes na Convenção comprometem-se a
adesão a ela, ou em qualquer momento posterior, declarar cumprir a decisão da Corte em todo caso em que forem
que reconhece como obrigatória, de pleno direito e sem partes.
convenção especial, a competência da Corte em todos os 2. A parte da sentença que determinar indenização
casos relativos à interpretação ou aplicação desta compensatória poderá ser executada no país respectivo
Convenção. pelo processo interno vigente para a execução de
2. A declaração pode ser feita incondicionalmente, ou sob sentenças contra o Estado.
condição de reciprocidade, por prazo determinado ou para
casos específicos. Deverá ser apresentada ao Secretário-

www.editoradince.com - Acesse e veja se há novidades a respeito deste material – CUIDADO: cópia é crime.
30 D DIREITOS HUMANOS - Teoria esquematizada, dicas e questões de concurso
Artigo 69 Artigo 75
A sentença da Corte deve ser notificada às partes no caso Esta Convenção só pode ser objeto de reservas em
e transmitida aos Estados Partes na Convenção. conformidade com as disposições da Convenção de Viena
CAPÍTULO IV sobre o Direito dos Tratados, assinada em 23 de maio de
DISPOSIÇÕES COMUNS 1969.
Artigo 70 rtigo 76
1. Os juízes da Corte e os membros da Comissão gozam, 1. Qualquer Estado Parte, diretamente, e a Comissão ou a
desde o momento de sua eleição e enquanto durar o seu Corte, por intermédio do Secretário-Geral, podem submeter
mandato, das imunidades reconhecidas aos agentes à Assembléia Geral, para o que julgarem conveniente,
diplomáticos pelo Direito Internacional. Durante o exercício proposta de emenda a esta Convenção.
dos seus cargos gozam, além disso, dos privilégios 2. As emendas entrarão em vigor para os Estados que
diplomáticos necessários para o desempenho de suas ratificarem as mesmas na data em que houver sido
funções. depositado o respectivo instrumento de ratificação que
2. Não se poderá exigir responsabilidade em tempo algum corresponda ao número de dois terços dos Estados Partes
dos juízes da Corte, nem dos membros da Comissão, por nesta Convenção. Quanto aos outros Estados Partes,
votos e opiniões emitidos no exercício de suas funções. entrarão em vigor na data em que depositarem eles os seus
Artigo 71 respectivos instrumentos de ratificação.
Os cargos de juiz da Corte ou de membro da Comissão são Artigo 77
incompatíveis com outras atividades que possam afetar sua 1. De acordo com a faculdade estabelecida no artigo 31,
independência ou imparcialidade conforme o que for qualquer Estado Parte e a Comissão podem submeter à
determinado nos respectivos estatutos. consideração dos Estados Partes reunidos por ocasião da
Artigo 72 Assembléia Geral, projetos de protocolos adicionais a esta
Os juízes da Corte e os membros da Comissão perceberão Convenção, com a finalidade de incluir progressivamente no
honorários e despesas de viagem na forma e nas condições regime de proteção da mesma outros direitos e liberdades.
que determinarem os seus estatutos, levando em conta a 2. Cada protocolo deve estabelecer as modalidades de sua
importância e independência de suas funções. Tais entrada em vigor e será aplicado somente entre os Estados
honorários e despesas de viagem serão fixados no Partes no mesmo.
orçamento-programa da Organização dos Estados Artigo 78
Americanos, no qual devem ser incluídas, além disso, as 1. Os Estados Partes poderão denunciar esta Convenção
despesas da Corte e da sua Secretaria. Para tais efeitos, a depois de expirado um prazo de cinco anos, a partir da data
Corte elaborará o seu próprio projeto de orçamento e da entrada em vigor da mesma e mediante aviso prévio de
submetê-lo-á à aprovação da Assembléia Geral, por um ano, notificando o Secretário-Geral da Organização, o
intermédio da Secretaria-Geral. Esta última não poderá nele qual deve informar as outras Partes.
introduzir modificações. 2. Tal denúncia não terá o efeito de desligar o Estado Parte
Artigo 73 interessado das obrigações contidas nesta Convenção, no
Somente por solicitação da Comissão ou da Corte, que diz respeito a qualquer ato que, podendo constituir
conforme o caso, cabe à Assembléia Geral da Organização violação dessas obrigações, houver sido cometido por ele
resolver sobre as sanções aplicáveis aos membros da anteriormente à data na qual a denúncia produzir efeito.
Comissão ou aos juízes da Corte que incorrerem nos casos CAPÍTULO XI
previstos nos respectivos estatutos. Para expedir uma DISPOSIÇÕES TRANSITÓRIAS
resolução, será necessária maioria de dois terços dos votos Seção 1 — Comissão Interamericana de Direitos
dos Estados Membros da Organização, no caso dos Humanos
membros da Comissão; e, além disso, de dois terços dos Artigo 79
votos dos Estados Partes na Convenção, se se tratar dos Ao entrar em vigor esta Convenção, o Secretário-Geral
juízes da Corte. pedirá por escrito a cada Estado membro da Organização
PARTE III que apresente, dentro de um prazo de noventa dias, seus
DISPOSIÇÕES GERAIS E TRANSITÓRIAS candidatos a membro da Comissão Interamericana de
CAPÍTULO X Direitos Humanos. O Secretário-Geral preparará uma lista
ASSINATURA, RATIFICAÇÃO, RESERVA, EMENDA, por ordem alfabética dos candidatos apresentados e a
PROTOCOLO E DENÚNCIA encaminhará aos Estados membros da Organização pelo
Artigo 74 menos trinta dias antes da Assembléia Geral seguinte.
1. Esta Convenção fica aberta à assinatura e à ratificação Artigo 80
ou adesão de todos os Estados membros da Organização A eleição dos membros da Comissão far-se-á dentre os
dos Estados Americanos. candidatos que figurem na lista a que se refere o artigo 79,
2. A ratificação desta Convenção ou a adesão a ela efetuar- por votação secreta da Assembléia Geral, e serão
se-á mediante depósito de um instrumento de ratificação ou declarados eleitos os candidatos que obtiverem maior
de adesão na Secretaria-Geral da Organização dos Estados número de votos e a maioria absoluta dos votos dos
Americanos. Esta Convenção entrará em vigor logo que representantes dos Estados membros. Se, para eleger
onze Estados houverem depositado os seus respectivos todos os membros da Comissão, for necessário realizar
instrumentos de ratificação ou de adesão. Com referência a várias votações, serão eliminados sucessivamente, na
qualquer outro Estado que a ratificar ou que a ela aderir forma que for determinada pela Assembléia Geral, os
ulteriormente, a Convenção entrará em vigor na data do candidatos que receberem menor número de votos.
depósito do seu instrumento de ratificação ou de adesão. Seção 2 — Corte Interamericana de Direitos Humanos
3. O Secretário-Geral informará todos os Estados Artigo 81
membros da Organização sobre a entrada em vigor da Ao entrar em vigor esta Convenção, o Secretário-Geral
Convenção. solicitará por escrito a cada Estado Parte que apresente,
dentro de um prazo de noventa dias, seus candidatos a juiz
da Corte Interamericana de Direitos Humanos. O

www.editoradince.com - Acesse e veja se há novidades a respeito deste material – CUIDADO: cópia é crime.
DIREITOS HUMANOS - Teoria esquematizada, dicas e questões de concursos 31
Secretário-Geral preparará uma lista por ordem alfabética Assinale a alternativa que indica todas as
dos candidatos apresentados e a encaminhará aos Estados afirmativas corretas.
Partes pelo menos trinta dias antes da Assembléia Geral A São corretas apenas as afirmativas 1 e 2.
seguinte.
B São corretas apenas as afirmativas 2 e 4.
Artigo 82
C São corretas apenas as afirmativas 3 e 4.
A eleição dos juízes da Corte far-se-á dentre os candidatos
que figurem na lista a que se refere o artigo 81, por votação D São corretas apenas as afirmativas 1, 2 e 3.
secreta dos Estados Partes, na Assembléia Geral, e serão
declarados eleitos os candidatos que obtiverem maior Gabarito: 01/a; 02/b; 03/A
número de votos e a maioria absoluta dos votos dos
representantes do Estados Partes. Se, para eleger todos os
juízes da Corte, for necessário realizar várias votações,
serão eliminados sucessivamente, na forma que for A DECLARAÇÃO UNIVERSAL DOS
determinada pelos Estados Partes, os candidatos que
receberem menor número de votos. DIREITOS HUMANOS.
Preâmbulo
QUESTÕES DE CONCURSOS Considerando que o reconhecimento da dignidade
inerente a todos os membros da família humana e dos
01. (IADES - 2018 - APEX Brasil - Analista - Prospecção seus direitos iguais e inalienáveis constitui o fundamento
de Mercados) A respeito da Organização dos Estados da liberdade, da justiça e da paz no mundo;
Americanos (OEA), é correto afirmar que
Considerando que o desconhecimento e o
A tem como principais pilares a democracia, os direitos desprezo dos direitos do Homem conduziram a atos de
humanos, a segurança e o desenvolvimento. barbárie que revoltam a consciência da Humanidade e
B tem como principal objetivo o desenvolvimento do que o advento de um mundo em que os seres humanos
comércio internacional no continente americano. sejam livres de falar e de crer, libertos do terror e da
C não permite a existência de Estado observador miséria, foi proclamado como a mais alta inspiração do
permanente, isto é, países que não são do continente Homem;
americano não podem acompanhar as atividades da Considerando que é essencial a proteção dos
Organização. direitos do Homem através de um regime de direito, para
D todos os respectivos Estados-membros possuem que o Homem não seja compelido, em supremo recurso,
representação da Organização no país. à revolta contra a tirania e a opressão;
E a respectiva Assembleia Geral realiza quatro sessões Considerando que é essencial encorajar o
por ano, com o objetivo de acompanhar os trabalhos desenvolvimento de relações amistosas entre as nações;
realizados pelos Grupos de Trabalho. Considerando que, na Carta, os povos das Nações
Unidas proclamam, de novo, a sua fé nos direitos
02. (FEPESE - 2017 - PC-SC - Escrivão de Polícia Civil) A fundamentais do Homem, na dignidade e no valor da
Organização dos Estados Americanos estabeleceu pessoa humana, na igualdade de direitos dos homens e
como propósitos essenciais: das mulheres e se declaram resolvidos a favorecer o
progresso social e a instaurar melhores condições de vida
1. Promover e consolidar o desenvolvimento agrário dentro de uma liberdade mais ampla;
regional, como fonte de subsistência.
Considerando que os Estados membros se
2. Promover e consolidar a democracia parlamentarista, comprometeram a promover, em cooperação com a
respeitado o princípio da supremacia legislativa. Organização das Nações Unidas, o respeito universal e
3. Promover, por meio da ação cooperativa, seu efetivo dos direitos do Homem e das liberdades
desenvolvimento econômico, social e cultural. fundamentais;
4. Promover e consolidar a democracia representativa, Considerando que uma concepção comum destes
respeitado o princípio da não intervenção. direitos e liberdades é da mais alta importância para dar
Assinale a alternativa que indica todas as plena satisfação a tal compromisso:
afirmativas corretas. A Assembleia Geral proclama a presente
A São corretas apenas as afirmativas 1 e 2. Declaração Universal dos Direitos Humanos como ideal
B São corretas apenas as afirmativas 3 e 4. comum a atingir por todos os povos e todas as nações, a
fim de que todos os indivíduos e todos os órgãos da
C São corretas apenas as afirmativas 1, 2 e 4.
sociedade, tendo-a constantemente no espírito, se
D São corretas apenas as afirmativas 1, 3 e 4. esforcem, pelo ensino e pela educação, por desenvolver o
E São corretas apenas as afirmativas 2, 3 e 4. respeito desses direitos e liberdades e por promover, por
medidas progressivas de ordem nacional e internacional,
o seu reconhecimento e a sua aplicação universais e
01. (FEPESE - 2017 - PC-SC - Agente de Polícia Civil) A
efetivos tanto entre as populações dos próprios Estados
Organização dos Estados Americanos tem como
membros como entre as dos territórios colocados sob a
princípios:
sua jurisdição.
1. A boa-fé deve reger as relações dos Estados entre si.
Artigo 1°
2. A educação dos povos deve orientar-se para a justiça,
Todos os seres humanos nascem livres e iguais
a liberdade e a paz.
em dignidade e em direitos. Dotados de razão e de
3. Toda agressão armada deverá ser repelida na mesma consciência, devem agir uns para com os outros em
medida e esforço. espírito de fraternidade.
4. A justiça e a segurança demandam a eliminação de
focos de dissidência.

www.editoradince.com - Acesse e veja se há novidades a respeito deste material – CUIDADO: cópia é crime.
32 D DIREITOS HUMANOS - Teoria esquematizada, dicas e questões de concurso
Artigo 2° Artigo 13°
Todos os seres humanos podem invocar os 1.Toda a pessoa tem o direito de livremente
direitos e as liberdades proclamados na presente circular e escolher a sua residência no interior de um
Declaração, sem distinção alguma, nomeadamente de Estado.
raça, de cor, de sexo, de língua, de religião, de opinião 2.Toda a pessoa tem o direito de abandonar o país
política ou outra, de origem nacional ou social, de fortuna, em que se encontra, incluindo o seu, e o direito de
de nascimento ou de qualquer outra situação. Além disso, regressar ao seu país.
não será feita nenhuma distinção fundada no estatuto
Artigo 14°
político, jurídico ou internacional do país ou do território da
naturalidade da pessoa, seja esse país ou território 1.Toda a pessoa sujeita a perseguição tem o
independente, sob tutela, autônomo ou sujeito a alguma direito de procurar e de beneficiar de asilo em outros
limitação de soberania. países.
Artigo 3° 2.Este direito não pode, porém, ser invocado no
caso de processo realmente existente por crime de direito
Todo indivíduo tem direito à vida, à liberdade e à
comum ou por atividades contrárias aos fins e aos
segurança pessoal.
princípios das Nações Unidas.
Artigo 4°
Artigo 15°
Ninguém será mantido em escravatura ou em
1.Todo o indivíduo tem direito a ter uma
servidão; a escravatura e o trato dos escravos, sob todas
nacionalidade.
as formas, são proibidos.
2.Ninguém pode ser arbitrariamente privado da
Artigo 5°
sua nacionalidade nem do direito de mudar de
Ninguém será submetido a tortura nem a penas ou nacionalidade.
tratamentos cruéis, desumanos ou degradantes.
Artigo 16°
Artigo 6°
1.A partir da idade núbil, o homem e a mulher têm
Todos os indivíduos têm direito ao o direito de casar e de constituir família, sem restrição
reconhecimento, em todos os lugares, da sua alguma de raça, nacionalidade ou religião. Durante o
personalidade jurídica. casamento e na altura da sua dissolução, ambos têm
Artigo 7° direitos iguais.
Todos são iguais perante a lei e, sem distinção, 2.O casamento não pode ser celebrado sem o livre
têm direito a igual proteção da lei. Todos têm direito a e pleno consentimento dos futuros esposos.
proteção igual contra qualquer discriminação que viole a 3.A família é o elemento natural e fundamental da
presente Declaração e contra qualquer incitamento a tal sociedade e tem direito à proteção desta e do Estado.
discriminação.
Artigo 17°
Artigo 8°
1.Toda a pessoa, individual ou coletiva, tem direito
Toda a pessoa tem direito a recurso efetivo para à propriedade.
as jurisdições nacionais competentes contra os actos que
2.Ninguém pode ser arbitrariamente privado da
violem os direitos fundamentais reconhecidos pela
sua propriedade.
Constituição ou pela lei.
Artigo 18°
Artigo 9°
Toda a pessoa tem direito à liberdade de
Ninguém pode ser arbitrariamente preso, detido ou
pensamento, de consciência e de religião; este direito
exilado.
implica a liberdade de mudar de religião ou de convicção,
Artigo 10° assim como a liberdade de manifestar a religião ou
Toda a pessoa tem direito, em plena igualdade, a convicção, sozinho ou em comum, tanto em público como
que a sua causa seja equitativa e publicamente julgada em privado, pelo ensino, pela prática, pelo culto e pelos
por um tribunal independente e imparcial que decida dos ritos.
seus direitos e obrigações ou das razões de qualquer Artigo 19°
acusação em matéria penal que contra ela seja deduzida.
Todo o indivíduo tem direito à liberdade de opinião
Artigo 11° e de expressão, o que implica o direito de não ser
1.Toda a pessoa acusada de um ato delituoso inquietado pelas suas opiniões e o de procurar, receber e
presume-se inocente até que a sua culpabilidade fique difundir, sem consideração de fronteiras, informações e
legalmente provada no decurso de um processo público ideias por qualquer meio de expressão.
em que todas as garantias necessárias de defesa lhe Artigo 20°
sejam asseguradas.
1.Toda a pessoa tem direito à liberdade de reunião
2.Ninguém será condenado por ações ou e de associação pacíficas.
omissões que, no momento da sua prática, não
2.Ninguém pode ser obrigado a fazer parte de uma
constituíam ato delituoso à face do direito interno ou
associação.
internacional. Do mesmo modo, não será infligida pena
mais grave do que a que era aplicável no momento em Artigo 21°
que o ato delituoso foi cometido. 1.Toda a pessoa tem o direito de tomar parte na
Artigo 12° direção dos negócios, públicos do seu país, quer
diretamente, quer por intermédio de representantes
Ninguém sofrerá intromissões arbitrárias na sua
livremente escolhidos.
vida privada, na sua família, no seu domicílio ou na sua
correspondência, nem ataques à sua honra e reputação. 2.Toda a pessoa tem direito de acesso, em
Contra tais intromissões ou ataques toda a pessoa tem condições de igualdade, às funções públicas do seu país.
direito a proteção da lei. 3.A vontade do povo é o fundamento da autoridade
dos poderes públicos: e deve exprimir-se através de

www.editoradince.com - Acesse e veja se há novidades a respeito deste material – CUIDADO: cópia é crime.
DIREITOS HUMANOS - Teoria esquematizada, dicas e questões de concursos 33
eleições honestas a realizar periodicamente por sufrágio 2.Todos têm direito à proteção dos interesses
universal e igual, com voto secreto ou segundo processo morais e materiais ligados a qualquer produção científica,
equivalente que salvaguarde a liberdade de voto. literária ou artística da sua autoria.
Artigo 22° Artigo 28°
Toda a pessoa, como membro da sociedade, tem Toda a pessoa tem direito a que reine, no plano
direito à segurança social; e pode legitimamente exigir a social e no plano internacional, uma ordem capaz de
satisfação dos direitos econômicos, sociais e culturais tornar plenamente efetivos os direitos e as liberdades
indispensáveis, graças ao esforço nacional e à enunciadas na presente Declaração.
cooperação internacional, de harmonia com a Artigo 29°
organização e os recursos de cada país.
1.O indivíduo tem deveres para com a
Artigo 23° comunidade, fora da qual não é possível o livre e pleno
1.Toda a pessoa tem direito ao trabalho, à livre desenvolvimento da sua personalidade.
escolha do trabalho, a condições equitativas e 2.No exercício deste direito e no gozo destas
satisfatórias de trabalho e à proteção contra o liberdades ninguém está sujeito senão às limitações
desemprego. estabelecidas pela lei com vista exclusivamente a
2.Todos têm direito, sem discriminação alguma, a promover o reconhecimento e o respeito dos direitos e
salário igual por trabalho igual. liberdades dos outros e a fim de satisfazer as justas
3.Quem trabalha tem direito a uma remuneração exigências da moral, da ordem pública e do bem-estar
equitativa e satisfatória, que lhe permita e à sua família numa sociedade democrática.
uma existência conforme com a dignidade humana, e 3.Em caso algum estes direitos e liberdades
completada, se possível, por todos os outros meios de poderão ser exercidos contrariamente e aos fins e aos
proteção social. princípios das Nações Unidas.
4.Toda a pessoa tem o direito de fundar com Artigo 30°
outras pessoas sindicatos e de se filiar em sindicatos para Nenhuma disposição da presente Declaração pode
defesa dos seus interesses. ser interpretada de maneira a envolver para qualquer
Artigo 24° Estado, agrupamento ou indivíduo o direito de se entregar
Toda a pessoa tem direito ao repouso e aos a alguma atividade ou de praticar algum ato destinado a
lazeres, especialmente, a uma limitação razoável da destruir os direitos e liberdades aqui enunciados.
duração do trabalho e as férias periódicas pagas.
Artigo 25° QUESTÕES DE CONCURSOS
1.Toda a pessoa tem direito a um nível de vida
01. (IBADE - 2021 - IAPEN - AC - Auxiliar Administrativo)
suficiente para lhe assegurar e à sua família a saúde e o
Conforme a Declaração Universal dos Direitos
bem-estar, principalmente quanto à alimentação, ao
Humanos, proclamada pela Resolução nº 217ª (III) da
vestuário, ao alojamento, à assistência médica e ainda
Assembleia Geral das Nações Unidas, de 10 de
quanto aos serviços sociais necessários, e tem direito à
dezembro de 1948, assinale a alternativa CORRETA.
segurança no desemprego, na doença, na invalidez, na
viuvez, na velhice ou noutros casos de perda de meios de A Ninguém pode ser arbitrariamente preso, detido ou
subsistência por circunstâncias independentes da sua exilado (Art. 9º)
vontade. B Toda a pessoa acusada de um ato delituoso presume-
2.A maternidade e a infância têm direito a ajuda e se culpado até que se prove o contrário (Art. 11º §1)
a assistência especiais. Todas as crianças, nascidas C Ninguém pode ser arbitrariamente privado da sua
dentro ou fora do matrimônio, gozam da mesma proteção propriedade, exceto por conflitos civis (Art. 17º §2)
social. D Todos deverão fazer parte de uma associação (Art. 20º
Artigo 26° §2)
1.Toda a pessoa tem direito à educação. A E Todos têm direito a salário diferente por trabalho igual,
educação deve ser gratuita, pelo menos a correspondente devido a condições peculiares do indivíduo (Art. 23º
ao ensino elementar fundamental. O ensino elementar é §2)
obrigatório. O ensino técnico e profissional dever ser
generalizado; o acesso aos estudos superiores deve estar 02. (IBADE - 2021 - IAPEN - AC - Advogado) Conforme a
aberto a todos em plena igualdade, em função do seu Declaração Universal dos Direitos Humanos,
mérito. proclamada pela Resolução nº 217ª (III) da
2.A educação deve visar à plena expansão da Assembleia Geral das Nações Unidas, de 10 de
personalidade humana e ao reforço dos direitos do dezembro de 1948, assinale a alternativa CORRETA.
Homem e das liberdades fundamentais e deve favorecer a A Todo ser humano acusado de um ato delituoso será
compreensão, a tolerância e a amizade entre todas as presumidamente culpado até que a sua inocência
nações e todos os grupos raciais ou religiosos, bem como tenha sido provada de acordo com a lei, em
o desenvolvimento das atividades das Nações Unidas julgamento público no qual lhe tenham sido
para a manutenção da paz. asseguradas todas as garantias necessárias à sua
3.Aos pais pertence a prioridade do direito de defesa (Artigo 11)
escolher o género de educação a dar aos filhos. B Todo ser humano poderá ser obrigado a fazer parte de
Artigo 27° uma associação (Artigo 20)
1.Toda a pessoa tem o direito de tomar parte C Ninguém será arbitrariamente preso, detido ou exilado
livremente na vida cultural da comunidade, de fruir as (Artigo 9)
artes e de participar no progresso científico e nos
benefícios que deste resultam.

www.editoradince.com - Acesse e veja se há novidades a respeito deste material – CUIDADO: cópia é crime.
34 D DIREITOS HUMANOS - Teoria esquematizada, dicas e questões de concurso
D Somente os cidadãos capazes têm o direito de ser, em respeito a esses direitos e liberdades, e, pela adoção
todos os lugares, reconhecido como pessoa perante a de medidas progressivas de caráter nacional e
lei (Artigo 6) internacional, por assegurar o seu reconhecimento e a
E Todo ser humano estará sujeito à interferências do sua observância universal e efetiva, tanto entre os
Poder Público em sua vida privada, em sua família, povos dos próprios Estados-Membros, quanto entre os
em seu lar ou em sua correspondência (Artigo 12) povos dos territórios sob sua jurisdição. Assim,
conforme proclamou a Declaração Universal dos
Direitos Humanos, todo ser humano:
04. (AOCP - 2021 - PC-PA - Delegado de Polícia Civil) A
a) Tem capacidade para gozar os direitos e as liberdades
Declaração Universal dos Direitos Humanos de 1948
estabelecidos nesta Declaração, sem distinção de
surge no pós-guerra como reação aos horrores
qualquer espécie, seja de raça, cor, sexo, idioma,
vivenciados pelo mundo com as experiências nazi-
religião, opinião política ou de outra natureza, origem
fascistas. Dentre as seguintes alternativas, assinale a
nacional ou social, riqueza, nascimento, com algumas
que NÃO representa uma garantia prevista nesse
restrições.
importante instrumento.
b) Poderá fazer distinção fundada na condição política,
A Direito de deixar qualquer país, inclusive o próprio, e a
jurídica ou internacional do país ou território a que
este regressar.
pertença uma pessoa, quer se trate de um território
B Direito à prestação jurisdicional efetiva para defesa independente, sob tutela, sem governo próprio, quer
contra atos que violem direitos fundamentais. sujeito a qualquer outra limitação de soberania.
C Direito de acesso à Corte Interamericana de Direitos c) Tem direito à vida, à liberdade, podendo esta ser
Humanos em duplo grau de jurisdição no caso de restringida, e à segurança pessoal a critério da
ações propostas diretamente no Supremo Tribunal administração pública através da polícia militar, civil e
Federal. federal.
D Direito de procurar e de gozar asilo em outros países. d) Tem o direito de ser, em todos os lugares, reconhecido
E Direito de contrair matrimônio e fundar uma família. como pessoa perante a lei, salvo nos casos previstos
em lei específica.
05. (Ag.Pen.SEJUS-RJ/FUNRIO) Considerando que os e) Tem direito a receber dos tribunais nacionais
Estados-Membros se comprometeram a promover, em competentes remédio efetivo para os atos que violem
cooperação com as Nações Unidas, o respeito os direitos fundamentais que lhe sejam reconhecidos
universal aos direitos e liberdades humanas pela constituição ou pela lei.
fundamentais e a observância desses direitos e
liberdades, e que uma compreensão comum desses Gabarito: 01/A; 02/C; 03/E; 04/C; 05/A; 06/E
direitos e liberdades é da mais alta importância para o
pleno cumprimento desse compromisso, pode-se
afirmar que:
a) A Declaração Universal dos Direitos Humanos é um A CORTE INTERAMERICANA DE
dos documentos básicos das Nações Unidas e foi
assinada em 1948. Nela, são enumerados os direitos DIREITOS HUMANOS.
que todos os seres humanos possuem. A Corte Interamericana de Direitos humanos é
b) Declaração Universal dos Direitos Humanos é uma instituição judiciária autônoma cujo objetivo é a
considerada um acordo, pois este termo é usado, aplicação e a interpretação da Convenção Americana
geralmente, para caracterizar negociações bilaterais sobre Direitos Humanos. A Corte exerce suas funções em
de natureza política, econômica, comercial, cultural, conformidade com as disposições da citada Convenção e
científica e técnica. Acordos podem ser firmados entre deste Estatuto.
países ou entre um país e uma organização A Corte Interamericana de Direitos Humanos é
internacional. um órgão judicial autônomo que tem sede em San
c) Declaração Universal dos Direitos Humanos é José, Costa Rica, cujo propósito é aplicar e interpretar
considerada um tratado já que tratados são atos a Convenção Americana de Direitos Humanos e outros
bilaterais ou multilaterais aos quais se deseja atribuir tratados de Direitos Humanos. Faz parte do chamado
especial relevância política. Sistema Interamericano de Proteção aos Direitos
d) Declaração Universal dos Direitos Humanos é uma Humanos.
convenção, pois essa palavra costuma ser empregada
para designar atos multilaterais, oriundos de Funções
conferências internacionais e que abordem assunto de A Corte exerce competência contenciosa e
interesse geral. consultiva.
e) Declaração Universal dos Direitos Humanos é um Os idiomas oficiais da Corte são os mesmos
protocolo e se designa a acordos menos formais que adotados pela OEA, quais sejam o espanhol,
os tratados. O termo é utilizado, ainda, para designar português, inglês e o francês. Os idiomas de trabalho são
a ata final de uma conferência internacional. aqueles que decida a Corte a cada ano. Não obstante,
para um caso específico, pode-se adotar também como
06. (Ag.Pen.SEJUS-RJ/FUNRIO) A Assembleia Geral idioma de trabalho aquele de uma das partes, sempre que
proclamou a Declaração Universal dos Direitos este seja a língua oficial desta.
Humanos como o ideal comum a ser atingido por
todos os povos e todas as nações, com o objetivo de Competência contenciosa
que cada indivíduo e cada órgão da sociedade, tendo
sempre em mente esta Declaração, se esforcem, A Corte tem competência litigiosa, para conhecer
através do ensino e da educação, por promover o de qualquer caso relativo à interpretação e aplicação das

www.editoradince.com - Acesse e veja se há novidades a respeito deste material – CUIDADO: cópia é crime.
DIREITOS HUMANOS - Teoria esquematizada, dicas e questões de concursos 35
disposições da Convenção Americana de Direitos  Humberto Antonio Sierra Porto (Colômbia)
Humanos a que lhe seja submetida apreciação, sempre  L. Patricio Pazmiño Freire (Equador)
os Estados signatários reconheçam esta competência, por
declaração ou convenções especiais.
Brasileiros na Corte
Basicamente conhece dos casos em que se
alegue que um dos Estados-membros tenha violado um O juiz Roberto de Figueiredo Caldas, que tomou
direito ou liberdade protegido pela Convenção, sendo posse em fevereiro de 2013 e atualmente é Presidente da
necessário que se tenham esgotados os procedimentos Corte IDH, é o segundo brasileiro a ocupar assento na
previstos nesta. Corte. Antes, o jurista Antônio Augusto Cançado
Trindade, atualmente juiz da Corte Internacional de
As pessoas, grupos ou entidades que não sejam o
Justiça, foi o representante do Brasil de 1995 a 2006,
Estado não têm capacidade de impetrar casos junto à
tendo ocupado a presidência por duas vezes (1999-2001
Corte, mas podem recorrer à Comissão Interamericana de
e 2002-2003).
Direitos Humanos. A Comissão pode, então, levar os
assuntos diante desta, sempre que o Estado questionado
haja reconhecido sua competência. Em todos os casos, a Casos contra o Estado brasileiro na Corte
Comissão deve comparecer em todos os casos O Brasil conta com 9 (nove) casos que tiveram
apreciados pela Corte. tramitação ou estão em processamento perante a Corte:
O procedimento junto à Corte é de caráter  Vladimir Herzog e outros, Brasil. Caso 12.879.
contraditório. Termina com uma sentença judicial Data: 22 de abril de 2016;
motivada, obrigatória, definitiva e inapelável. Se a decisão  Pueblo Indígena Xucuru e seus membros, Brasil.
não expressa, no todo ou parcialmente, a opinião Caso 12.728 Data: 16 de março 2016;
unânime dos juízes, qualquer destes tem direito a que se
junte sua opinião dissidente ou individual.  Cosme Rosa Genoveva, Evandro de Oliveira e
outros (Favela Nova Brasília), Brasil. Caso 11.566.
Em caso de desacordo sobre o sentido ou alcance Data: 19 de maio 2015;
da decisão, a Corte o interpretará por solicitação de
qualquer das partes, sempre que esta solicitação seja  Trabalhadores da Fazenda Brasil Verde, Brasil.
apresentada dentro de noventa dias a partir da notificação Caso 12.066; Data: 6 de março de 2015;
da sentença.  Julia Gomes Lund y Otros (Guerrilha do
Araguaia), Brasil. Caso 11.552. 26 de março de 2009;
Competência consultiva  Sétimo Garibaldi, Brasil. Caso 12.478. 24 de
dezembro de 2007;
Os Estados-membros da OEA podem consultar a
Corte acerca da interpretação da Convenção Americana  Arley José Escher y Otros, Brasil. Caso 12.353.
de Direitos Humanos ou de outros tratados concernentes 20 de dezembro de 2007;
à proteção dos Direitos Humanos no âmbito dos Estados  Gilson Nogueira de Carvalho, Brasil. Caso
[3]
americanos. Além disso, podem consultá-la, dentro da 12.058. 13 de janeiro de 2005;
sua competência, também os órgãos da Organização dos  Damião Ximenes Lopes, Brasil. Caso 12.237. 1
Estados Americanos. de outubro de 2004.
Pode a Corte, ainda, a pedido de um Estado- DICA DE CONCURSO: A Corte Interamericana de
membro da OEA, emitir parecer sobre a compatibilidade Direitos Humanos, a pedido de um Estadomembro
entre qualquer de suas leis internas e os da Organização, poderá emitir pareceres sobre a
mencionados tratados internacionais. compatibilidade entre qualquer de suas leis internas
e os tratados concernentes à proteção dos direitos
Composição humanos nos Estados americanos. (MPE/ES
A Corte é composta por sete juízes, naturais dos Promotor de Justiça Substituto/2013)
Estados-membros da OEA, eleitos a título pessoal
entre juristas da mais elevada autoridade moral, de
reconhecida competência em matéria de Direitos
QUESTÕES DE CONCURSOS
Humanos, que reúnam as condições requeridas para o 01. (DPE-RJ - 2021 - DPE-RJ - Residência Jurídica) ―A
exercício das mais elevadas funções judiciais conforme Corte Interamericana de Direitos Humanos (...) é
da lei do país do qual seja nacional ou do Estado que lhe órgão jurisdicional do sistema interamericano que
proponha a candidatura. Não pode haver mais de um juiz resolve sobre os casos de violação de direitos
da mesma nacionalidade. humanos perpetrados pelos Estados-partes da OEA e
Os juízes da Corte são eleitos para um mandato que tenham ratificado a Convenção Americana. Sua
de seis anos e somente podem ser reeleitos uma vez. O sede é na cidade de San José, Costa Rica. Trata-se
juiz eleito para substituir a outro, cujo mandato não tenha de um tribunal supranacional interamericano, capaz de
ainda expirado, completa tal mandato. condenar os Estados-partes na Convenção Americana
Atualmente (2016) fazem parte da Corte os juízes por violação de direitos humanos, desde que estes
(em ordem de precedência, estabelecida pelo art. 13 do tenham aceitado a competência contenciosa do
tribunal‖. (MAZZUOLI, Valerio de Oliveira. Curso de
Estatuto da Corte):
direitos humanos. 4. ed. São Paulo: Método, 2017, p.
 Roberto F. Caldas (Brasil) - Presidente 149). Considerando o texto acima e as regras contidas
 Eduardo Ferrer Mac-Gregor Poisot (México), na Convenção Americana de Direitos Humanos, quem
Vice-presidente ou quais órgãos podem submeter um caso à Corte
 Eduardo Vio Grossi (Chile) Interamericana de Direitos Humanos?
 Elizabeth Odio Benito (Costa Rica) A Somente os Estados e a Comissão Interamericana de
Direitos Humanos podem submeter um caso à Corte
 Eugenio Raúl Zaffaroni (Argentina)
Interamericana de Direitos Humanos.

www.editoradince.com - Acesse e veja se há novidades a respeito deste material – CUIDADO: cópia é crime.
36 D DIREITOS HUMANOS - Teoria esquematizada, dicas e questões de concurso
B Somente a Comissão Interamericana de Direitos de 1945, conferindo uma forma legal aos direitos
Humanos pode submeter um caso à Corte humanos inerentes.
Interamericana de Direitos Humanos. A criação das Nações Unidas viabilizou um
C A Defensoria Pública, o Ministério Público, os Estados e fórum ideal para o desenvolvimento e a adoção dos
a Comissão Interamericana de Direitos Humanos instrumentos internacionais de direitos humanos. Outros
podem submeter um caso à Corte Interamericana de instrumentos foram adotados a nível regional, refletindo
Direitos Humanos. as preocupações sobre os direitos humanos particulares a
D Qualquer pessoa natural pode submeter um caso à cada região.
Corte Interamericana de Direitos Humanos. A maioria dos países também adotou constituições
E A Defensoria Pública, o Ministério Público e as Cortes e outras leis que protegem formalmente os direitos
Constitucionais de cada Estado, além da Comissão humanos básicos. Muitas vezes, a linguagem utilizada
Interamericana de Direitos Humanos podem submeter pelos Estados vem dos instrumentos internacionais de
um caso à Corte Interamericana de Direitos Humanos. direitos humanos.
As normas internacionais de direitos humanos
02. (FCC - 2019 - DPE-AM - Analista Jurídico de consistem, principalmente, de tratados e costumes, bem
Defensoria - Ciências Jurídicas) Em 2014, a Corte como declarações, diretrizes e princípios, entre outros.
Interamericana de Direitos Humanos proferiu sentença
na qual, entre outras obrigações, determinou que o Tratados
Brasil adotasse, em prazo razoável, medidas Um tratado é um acordo entre os Estados, que se
necessárias para tipificar o delito de desaparecimento comprometem com regras específicas. Tratados
forçado de pessoas conforme os parâmetros internacionais têm diferentes designações, como pactos,
interamericanos. Tal sentença se refere ao caso cartas, protocolos, convenções e acordos. Um tratado é
conhecido como legalmente vinculativo para os Estados que tenham
A trabalhadores da Fazenda Brasil Verde. consentido em se comprometer com as disposições do
B Amarildo Dias de Souza. tratado – em outras palavras, que são parte do tratado.
C Setimo Garibaldi. Um Estado pode fazer parte de um tratado através
D Gomes Lund e outros (―Guerrilha do Araguaia‖). de uma ratificação, adesão ou sucessão.
E Favela Nova Brasília. A ratificação é a expressão formal do
consentimento de um Estado em se comprometer com um
tratado. Somente um Estado que tenha assinado o tratado
anteriormente – durante o período no qual o tratado
03. (FUMARC - 2018 - PC-MG - Escrivão de Polícia Civil)
esteve aberto a assinaturas – pode ratificá-lo.
Sobre a Corte Interamericana de Direitos Humanos,
é CORRETO afirmar: A ratificação consiste de dois atos processuais: a
nível interno, requer a aprovação pelo órgão
A A Comissão Interamericana e os Estados Partes podem
constitucional apropriado – como o Parlamento, por
submeter um caso à Corte Interamericana, admitida a
exemplo. A nível internacional, de acordo com as
legitimação do indivíduo, nos termos da Convenção
disposições do tratado em questão, o instrumento de
Americana.
ratificação deve ser formalmente transmitido ao
B A Corte Interamericana não apresenta competência depositário, que pode ser um Estado ou uma organização
consultiva. internacional como a ONU.
C É órgão jurisdicional do sistema regional, composto por A adesão implica o consentimento de um Estado
15 juízes nacionais de Estados Membros da OEA, que não tenha assinado anteriormente o instrumento.
eleitos a título pessoal pelos Estados Partes da Estados ratificam tratados antes e depois de este ter
Convenção. entrado em vigor. O mesmo se aplica à adesão.
D O Estado brasileiro reconheceu a competência Um Estado também pode fazer parte de um
jurisdicional da Corte Interamericana em dezembro de tratado por sucessão, que acontece em virtude de uma
1998. disposição específica do tratado ou de uma declaração. A
maior parte dos tratados não são auto-executáveis. Em
Gabarito: 01/A; 02/D; 03/D alguns Estados tratados são superiores à legislação
interna, enquanto em outros Estados tratados recebem
status constitucional e em outros, apenas certas
disposições de um tratado são incorporadas à legislação
A NATUREZA JURÍDICA DA interna.
INCORPORAÇÃO DE NORMAS Um Estado pode, ao ratificar um tratado, formular
INTERNACIONAIS. reservas a ele, indicando que, embora consinta em se
comprometer com a maior parte das disposições, não
O Direito Internacional dos Direitos concorda com se comprometer com certas disposições.
Humanos estabelece as obrigações dos governos de No entanto, uma reserva não pode derrotar o objeto e o
agirem de determinadas maneiras ou de se absterem de propósito do tratado.
certos atos, a fim de promover e proteger os direitos Além disso, mesmo que um Estado não faça parte
humanos e as liberdades de grupos ou indivíduos. de um tratado ou não tenha formulado reservas, o Estado
pode ainda estar comprometido com as disposições do
NORMAS INTERNACIONAIS DE DIREITOS HUMANOS tratado que se tornaram direito internacional
A expressão formal dos direitos humanos inerentes consuetudinário ou constituem normas imperativas do
se dá através das normas internacionais de direitos direito internacional, como a proibição da tortura. Todos
humanos. Uma série de tratados internacionais dos os tratados das Nações Unidas estão reunidos
direitos humanos e outros instrumentos surgiram a partir em treaties.un.org.

www.editoradince.com - Acesse e veja se há novidades a respeito deste material – CUIDADO: cópia é crime.
DIREITOS HUMANOS - Teoria esquematizada, dicas e questões de concursos 37
interna, mediante uma cláusula geral de recepção
Declarações, resoluções etc. adotadas pelos órgãos automática plena. Com o ato da ratificação, a regra
das Nações Unidas internacional passa a vigorar de imediato tanto na
Normas gerais do direito internacional – princípios ordem jurídica internacional, como na ordem
e práticas com os quais a maior parte dos Estados jurídica interna, sem a necessidade de uma norma
concordaria – constam, muitas vezes, em declarações, de direito nacional que a integre ao sistema
proclamações, regras, diretrizes, recomendações e jurídico. Esta sistemática da incorporação
princípios. automática reflete a concepção monista, pela qual
Apesar de não ter nenhum feito legal sobre os o Direito Internacional e o Direito interno compõem
Estados, elas representam um consenso amplo por parte uma mesma unidade, uma única ordem jurídica,
da comunidade internacional e, portanto, têm uma força inexistindo qualquer limite entre a ordem jurídica
moral forte e inegável em termos na prática dos Estados, internacional e a ordem interna.
em relação a sua conduta das relações internacionais. O Min. Gilmar Mendes conduziu voto que hoje
O valor de tais instrumentos está no representa o posicionamento majoritário do STF,
reconhecimento e na aceitação por um grande número de atribuindo a qualidade de norma supralegal aos tratados
Estados e, mesmo sem o efeito vinculativo legal, podem de direitos humanos (repisando o antigo posicionamento
ser vistos como uma declaração de princípios do Min. Sepúlveda Pertence quando do julgamento do
amplamente aceitos pela comunidade internacional. RHC 79.785/RJ, j. 29/03/2000) e não admitindo o caráter
A Declaração das Nações Unidas sobre os Direitos constitucional aos tratados aprovados anteriormente à EC
dos Povos Indígenas, por exemplo, recebeu o apoio dos n. 45/04, a qual acrescentou o §3º ao art. 5º da
Estados Unidos em 2010, o último dos quatro Estados- Constituição Federal.
membros da ONU que se opuseram a ela. Para Mendes, os tratados de direitos humanos
Ao adotar a Declaração, os Estados se exercem um ―efeito paralisante‖ sobre toda a legislação
comprometeram a reconhecer os direitos dos povos infraconstitucional que com eles seja conflitante e, uma
indígenas sob a lei internacional, com o direito de serem vez aprovados na forma prevista pelo §3º do art. 5º da
respeitados como povos distintos e o direito de determinar Constituição Federal, passam a ter o status de emenda
seu próprio desenvolvimento de acordo com sua cultura, constitucional.
prioridades e leis consuetudinárias (costumes).
Natureza jurídica da norma de direito internacional
NATUREZA JURÍDICA público
Com efeito, além de integrarem o rol de normas Trazemos a lume a questão surgida com a EC n.
constitucionais, os tratados sobre direitos humanos têm 45, de 08/12/2004, a qual acrescentou o parágrafo
reconhecida a sua aplicabilidade imediata, conforme reza terceiro ao art. 5º da Constituição Federal, com a seguinte
o mesmo art. 5º da Constituição Federal, agora no seu redação:
parágrafo primeiro e, uma vez incorporados, passam a § 3º. Os tratados e convenções internacionais
incorporar a natureza de cláusulas pétreas. Destacamos: sobre direitos humanos que forem aprovados,
Atribuindo-lhes a Constituição a natureza de em cada Casa do Congresso Nacional, em dois
“normas constitucionais”, passam os tratados de turnos, por três quintos dos votos dos respectivos
proteção dos direitos humanos, pelo mandamento membros, serão equivalentes às emendas
do citado § 1º do seu art. 5º, a ter aplicabilidade constitucionais. (grifei)
imediata, dispensando- se, dessa forma, a edição Dessa forma, considera-se sobre a natureza
de decreto de execução para que irradiem seus jurídica das normas internacionais no ordenamento
efeitos tanto no plano interno como no plano jurídico que:
internacional. (...). Como se já não bastasse, é 1 – Na vigência da redação originária do art. 5º da
ainda de se ressaltar que todos os direitos Constituição, o STF fixou o entendimento de
inseridos nos referidos tratados, incorporando-se que os tratados e convenções internacionais
imediatamente no ordenamento interno brasileiro ingressam em nosso ordenamento jurídico
(CF, art. 5º, § 1º), por serem normas também como norma de natureza infraconstitucional,
definidoras dos direitos e garantias fundamentais, ainda que versem sobre direitos fundamentais;
passam a ser cláusulas pétreas, não podendo ser 2 – A emenda 45/2004 introduziu o § 3º no art. 5º
suprimidos nem mesmo por emenda à da CF, equiparando os tratados sobre direitos
Constituição (CF, art. 60, § 1º, IV). É o que se humanos a norma constitucional, desde que
extrai do resultado da interpretação dos §§ 1º e 2º, aprovados pelo procedimento idêntico ao das
do art. 5º da Lei Fundamental, em cotejo com o art. emendas constitucionais;
60, § 4º, IV, da mesma Carta. Isso porque o §1º do 3 – Mesmo após a emenda 45, é possível a
art. 5º da Constituição da República, como se viu, existência de tratados e convenções
dispõe expressamente que „as normas definidoras internacionais sobre direitos fundamentais sem
dos direitos e garantias fundamentais têm a prerrogativa da natureza constitucional de
aplicação imediata‟. (MAZZUOLI, 2005, p. 233- suas disposições, caso não sejam aprovados
234). pelo procedimento especial das emendas;
4 – Tratados que não veiculem normas sobre
No mesmo sentido, encontramos o posicionamento
direitos humanos continuam a ser integrados
de Flávia Piovesan (1997, p. 106-107), para quem:
ao ordenamento como norma
(...) em face da incorporação automática os infraconstitucional;
tratados internacionais incorporam-se de imediato 5 – Assim como as demais normas, a Emenda
ao direito nacional em virtude do ato da ratificação. Constitucional 45/2004 tem aplicabilidade
(…) Em suma, em face da sistemática da imediata e vigência para o futuro, podendo
incorporação automática, o Estado reconhece a eventualmente alcançar os efeitos futuros de
plena vigência do Direito Internacional na ordem relações jurídicas constituídas no passado,

www.editoradince.com - Acesse e veja se há novidades a respeito deste material – CUIDADO: cópia é crime.
38 D DIREITOS HUMANOS - Teoria esquematizada, dicas e questões de concurso
respeitados o direito adquirido, o ato jurídico Recordando, além disso, a seção 11 da Re 1986/
perfeito e a coisa julgada; 10 do Conselho Econômico e Social, maio de 1986, na
6 - Na ausência de dispositivo transitório expresso qual, entre outras coisas, foi pedido ao Secretário Geral
a respeito dos tratados e convenções sobre que apresentasse Comitê de Prevenção do Delito e Luta
direitos humanos já em vigor, deve prevalecer contra a Delinquência, no seu décimo período de
a interpretação de que foram recepcionados sessões, um relatório sobre os progressos realizados a
como norma constitucional em sentido material, das Regras, e também foi pedido ao Oitavo Congresso
já que não se admite a existência de das Nações Unidas sobre Prevenção do Delito e
inconstitucionalidade formal superveniente; Tratamento do Delinquente que as Regras propostas,
7 - A dicotomia entre tratados materiais e com vistas a sua aprovação,
formalmente constitucionais e tratados Alarmada pelas condições e circunstâncias pelas
materialmente constitucionais só tem lugar quais os jovens estão privados de sua liberdade em todo
após a vigência da emenda 45, por não haver o mundo,
em relação aos tratados e convenções
Conscientes de que os jovens, quando se
precedentes a possibilidade
encontram privados de liberdade, são extremamente
inconstitucionalidade formal superveniente, não
vulneráveis aos maus-tratos, à vitimização e à violência
admitida sequer em relação ao poder
de seus direitos,
constituinte originário, muito menos quando o
padrão de confronto estaria corporificado em Preocupada pelo fato de que muitos sistemas não
norma proveniente do constituinte derivado. estabelecem diferença entre adultos e jovens nas distintas
fases da administração da justiça e consequência disso,
DICA: O STF já se pronunciou acerca do status de
muitos jovens estão detidos em prisões e centros penais
um Tratado Internacional quando internalizado ao
junto com os adultos,
ordenamento jurídico brasileiro. O mesmo tem, em
regra, natureza jurídica de lei ordinária e está 1. Afirma que a reclusão de um jovem em um
posicionado hierarquicamente abaixo da estabelecimento deve ser feita apenas em último caso e
Constituição Federal, no mesmo patamar das Leis pelo menor espaço de tempo necessário;
Complementares, Leis Delegadas e Medidas 2. Reconhece que, devido a sua grande
Provisórias. vulnerabilidade, os jovens privados de liberdade requerem
e proteção especial e que deverão ser garantidos seus
direitos e bem-estar durante o período em que estejam
privados de sua liberdade e também após este;
REGRAS MÍNIMAS DAS NAÇÕES 3. Observa, com satisfação, o valioso trabalho da
UNIDAS PARA PROTEÇÃO DE JOVENS Secretaria e a colaboração estabelecida na preparação
PRIVADOS DE LIBERDADE - UNICEF das Regras entre a Secretaria e os especialistas, os
profissionais, as organizações intergovernamentais, os
O OITAVO CONGRESSO DAS NAÇÕES UNIDAS meios não oficiais, sobretudo a Anistia Internacional, a
SOBRE A PREVENÇÃO DO DELITO E DO Defesa das Crianças Internacional- Movimento
TRATAMENTO DO DELINQUENTE Internacional e Rãdda Barnen (Save the Children da
Tendo presentes a Declaração universal dos Suécia), e as instituições científicas que se ocupam dos
Direitos Humanos (Resolução 217 A (lll) da Assembleia direitos das crianças e da Justiça da Infância e da
Geral, de 10 de dezembro de 1948); o Pacto Internacional Juventude;
de Direitos Civis e Políticos (Resolução 2200 A (XXI) da 4. Aprova o projeto de Regras mínimas das ações
Assembleia Geral, anexo, de 16 de dezembro de 1966); a Unidas para os jovens privados de liberdade, que figura
Convenção contra a Tortura e Outros Tratamentos ou como anexo à presente resolução;
Penas Cruéis, Desumanas ou Degradantes (Resolução 5. Exorta o Comitê de Prevenção do Delito e a
39/46 da Assembleia Geral, de 10 de dezembro de 1984); Delinquência a formular medidas para aplicação eficaz
a Convenção sobre os Direitos da Criança (Resolução das Regras, com a assistência dos institutos das Nações
44/25 da Assembleia Geral, de 20 de novembro de 1989); Unidas para a prevenção e o tratamento do delinquente;
como também outros instrumentos internacionais relativos 6. Convida os Estados Membros a adaptarem, que
à proteção dos direitos e ao bem-estar dos jovens, necessário, sua legislação, suas políticas e suas práticas
Tendo, também, presentes as Regras mínimas nacionais, particularmente a capacitação de todas as
para o tratamento dos reclusos aprovadas pelo Primeiro categorias do pessoal da justiça da infância e da
Congresso das Nações Unidas sobre Prevenção do Delito juventude, ao espírito das Regras e a chamar para elas a
e Tratamento do Delinquente, atenção das autoridades competentes e do público em
Tendo presente, também, o Conjunto de princípios geral;
para a proteção de todas as pessoas submetidas a 7. Convida, também, os Estados Membros a
qualquer forma de detenção ou prisão, aprovado pela informarem ao Secretário Geral os seus esforços para
Assembleia Geral na sua Resolução 43/173, de 9 de aplicar as Regras na legislação, na política e na prática, e
dezembro de 1988, a apresentarem relatórios periódicos ao Comitê de
Recordando a Resolução 40/33 da Assembleia Prevenção de Delito e Luta contra a Delinquência das
Geral, de 29 de novembro de 1985 e as Regras Mínimas Nações Unidas, sobre os resultados alcançados na sua
das Nações Unidas para a Administração da Justiça da aplicação;
Infância e da Juventude, 8. Pede ao Secretário geral que procure dar a
Recordando, também, a Resolução 21 do Sétimo maior difusão possível ao texto das Regras em todos os
Congresso das Nações Unidas sobre a Prevenção do idiomas oficiais das Nações Unidas e convida os Estados
Delito e Tratamento do Delinquente, na qual se pediu a Membros a realizarem o mesmo esforço;
preparação de regras mínimas das Unidas para a 9. Pede ao Secretário Geral e solicita aos Estados
proteção dos jovens privados de liberdade, Membros a consignação dos recursos necessários para

www.editoradince.com - Acesse e veja se há novidades a respeito deste material – CUIDADO: cópia é crime.
DIREITOS HUMANOS - Teoria esquematizada, dicas e questões de concursos 39
garantir o bom êxito na aplicação e na execução das 6. As Regras deverão estar à disposição do
Regras, em particular no que se refere à contratação, à pessoal de justiça da infância e da juventude nos seus
capacitação e ao intercâmbio de pessoal da justiça da idiomas nacionais. Os jovens que não conheçam
infância e da juventude de todas as categorias; suficientemente bem o idioma falado pelo pessoal do
10. Insta todos os órgãos competentes do sistema estabelecimento de detenção deverão ter direito aos
das Nações Unidas, em particular o Fundo das Nações serviços de um intérprete, sempre que seja necessário,
Unidas para a Infância, as comissões regionais e os particularmente durante os reconhecimentos médicos e as
organismos especializadas, os institutos das Nações autuações disciplinares.
Unidas, para a prevenção do delito e o tratamento do 7. Quando necessário, os Estados deverão
delinquente, e todas as organizações intergovernamentais incorporar as presentes Regras a sua legislação ou
e não-governamentais interessadas, a colaborarem com a modificá-las em consequência, e estabelecer eficazes no
Secretaria e adotarem as medidas necessárias para caso de falta de observância, incluída a indenização nos
garantir um esforço concentrado, dentro de suas casos em que haja prejuízo aos jovens. Além disso, os
respectivas esferas de competência técnica no fomento Estados deverão vigiar a aplicação das Regras.
da aplicação das Regras; 8. As autoridades competentes procurarão, a todo
11. Convida a Subcomissão de Prevenção de momento, que o público compreenda, cada vez mais, que
Discriminações e Proteção às Minorias, da Comissão de o cuidado dos jovens detidos e sua preparação para a
Direitos Humanos, a examinar o novo instrumento reintegração à sociedade constituem um serviço social de
internacional, com vistas a fomentar a aplicação de suas grande importância e, deverão ser adotadas medidas
disposições. eficazes para fomentar os contatos abertos entre os
ANEXO jovens e a comunidade local.
Regras Mínimas das Nações Unidas para a proteção 9. Nenhuma das disposições contidas nas
dos Jovens Privados de Liberdade presentes regras deverá ser interpretada no sentido de se
excluir a aplicação dos instrumentos e normas pertinentes
*(Direitos Humanos, Recompilação de
das Nações Unidas, nem dos referentes aos direitos
Instrumentos Internacionais - publicações das Nações
humanos, reconhecidos pela comunidade internacional e
Unidas, n° de venda S . 83 . XIV l - sec . G)
relativos à atenção e à proteção de crianças e
I . PERSPECTIVAS FUNDAMENTAIS adolescentes.
1. O sistema de justiça da infância e da juventude 10. No caso da aplicação prática das regras
deverá respeitar os direitos e a segurança dos jovens e específicas contidas nos capítulos II a V, inclusive, das
fomentar seu bem-estar físico e mental. Não deveria ser presentes regras, ser incompatível com as regras que na
economizado esforço para abolir, na medida do possível, primeira parte, as últimas prevalecerão sobre as
a prisão de jovens. primeiras.
2. Só se poderá privar de liberdade os jovens de II. EFEITOS E APLICAÇÃO DAS REGRAS
acordo com os princípios e procedimentos estabelecidos
11. Devem ser aplicadas, aos efeitos das
nas presentes Regras, assim como nas Regras Mínimas
presentes Regras, as seguintes definições:
das Nações Unidas para a Administração da Justiça da
Infância e da Juventude (Regras de Beijing). A privação a) Entende-se por jovem uma pessoa de idade
de liberdade de um jovem deverá ser decidida apenas em inferior a 18 anos. A lei deve estabelecer a idade-limite
último caso e pelo menor espaço de tempo possível. antes da qual a criança não poderá ser privada de sua
Deverá ser limitada a casos excepcionais, por exemplo, liberdade;
como efeito de cumprimento de uma sentença depois da b) Por privação de liberdade, entende-se toda
condenação, para os tipos mais graves de delitos, e tendo forma de detenção ou prisão, assim como a internação
presente, devidamente, todas as circunstâncias e em outro estabelecimento público ou privado, de onde
condições do caso. A duração máxima da punição deve não se permita a saída livre do jovem, ordenado por
ser determinada pela autoridade judicial antes que o qualquer autoridade judicial, administrativa ou outra
jovem seja privado de sua liberdade. Não se deve deter autoridade pública.
ou prender os jovens sem que nenhuma acusação tenha 12. A privação da liberdade deverá ser efetuada
sido formulada contra eles. em condições e circunstâncias que garantam o respeito
3. O objetivo das seguintes regras é estabelecer aos direitos humanos dos jovens. Deverá ser garantido,
normas mínimas aceitas pelas Nações Unidas para a aos jovens reclusos em centros, o direito a desfrutar de
proteção dos jovens privados de liberdade em todas as atividades e programas úteis que sirvam para fomentar e
suas formas, de maneira compatível com os direitos garantir seu são desenvolvimento e sua dignidade,
humanos e liberdades fundamentais, e com vistas a se promover seu sentido de responsabilidade e fomentar,
opor aos efeitos prejudiciais de todo tipo de detenção e a neles, atitudes e conhecimentos que ajudem a
fomentar a integração na sociedade. desenvolver suas possibilidades como membros da
4. Estas Regras deverão ser aplicadas, sociedade.
imparcialmente, a todos os jovens, sem discriminação de 13. Por razão de sua situação, não se deverá
nenhum tipo por razão de raça, cor, sexo, idioma, religião, negar aos jovens privados de liberdade seus direitos civis,
nacionalidade, opinião política ou de outro tipo, práticas econômicos, políticos, sociais ou culturais
ou crenças culturais, fortuna, nascimento, situação de correspondentes, de acordo com a legislação nacional ou
família, origem étnica ou social ou incapacidade. Deverão internacional e que sejam compatíveis com a privação da
ser respeitadas as crenças religiosas e culturais, assim liberdade, como, por exemplo, os direitos e prestações da
como as práticas e preceitos morais dos jovens. previdência social, a liberdade de associação e, ao
5. As Regras estão concebidas para ter padrões alcançar a idade mínima exigida associação pela lei, o
práticos de referência e dar orientação aos profissionais direito de contrair matrimônio.
que participam da administração do sistema de justiça da 14. A proteção dos direitos individuais dos jovens
e da juventude. no que diz respeito, especialmente, à legalidade da

www.editoradince.com - Acesse e veja se há novidades a respeito deste material – CUIDADO: cópia é crime.
40 D DIREITOS HUMANOS - Teoria esquematizada, dicas e questões de concurso
execução das medidas de detenção, será garantida pela a um terceiro apropriado e independente, ter acesso ao
autoridade judicial competente, enquanto que os objetivos expediente e consultá-lo, se assim o solicitar. À raiz de
de integração social deverão ser garantidos por um órgão sua liberação, todo jovem terá o direito de ter seu
devidamente constituído que esteja autorizado a visitar os expediente extinto.
jovens e que não pertença à administração do centro de 20. Nenhum jovem poderá ser admitido num centro
detenção, através de inspeções regulares e outras formas de detenção sem uma ordem de internamento válida de
de controle. uma autoridade judicial, administrativa de caráter público.
15. As Regras presentes são aplicadas a todos os Os detalhes desta ordem deverão ser consignados,
centros e estabelecimentos onde haja jovens privados de imediatamente, no registro. Nenhum jovem será detido
liberdade. As Partes I, II, IV e V das Regras se aplicam a em nenhum centro onde não exista esse registro.
todos os centros de estabelecimentos onde haja jovens B. INGRESSO, REGISTRO, DESLOCAMENTO A
detidos, enquanto que a Parte III se aplica a jovens sob MUDANÇA
detenção provisória ou em espera de julgamento.
21. Em todos os lugares onde haja jovens detidos,
16. As Regras serão aplicadas no contexto das deverá ser mantido um registro completo e confiável da
condições econômicas, sociais e culturais predominantes seguinte informação relativa a cada um dos jovens
em cada Estado Membro. admitidos:
III. JOVENS DETIDOS OU EM PRISÃO PREVENTIVA a) dados relativos à identidade do jovem;
17. Supõem-se inocentes os jovens detidos sob b) a causa da reclusão, assim como seus motivos
detenção provisória ou em espera de julgamento ("prisão e autoridade que ordenou;
preventiva") e deverão ser tratados como tais. Na medida
c) o dia e a hora do ingresso, da mudança e da
do possível, deverá ser evitada, e limitada a
liberação;
circunstâncias excepcionais, a detenção antes da
celebração do julgamento. Como consequência, deverá d) detalhes da notificação de cada ingresso,
ser feito todo o possível para aplicar medidas mudança ou liberação do jovem aos pais e tutores que
substitutivas. Quando, apesar disso, recorrer-se à estivessem responsáveis no momento de ser internado;
detenção preventiva, os tribunais de jovens e os órgãos e) detalhes sobre os problemas de saúde física e
de investigação deverão dar máxima prioridade ao mais mental conhecidos, incluído o uso indevido de drogas e
rápido andamento possível do trâmite desses casos, para álcool.
que a detenção seja a menor possível. De todas as 22. A informação, acima mencionada, relativa ao
maneiras, os jovens detidos ou em espera de julgamento ingresso, lugar de internação, mudança e liberação,
deverão estar separados dos declarados culpados. deverá ser notificada, sem demora, aos pais e tutores ou
18. As condições de detenção de um jovem que ao parente mais próximo do jovem.
não tenha sido julgado deverão ser ajustadas às 23. Após o ingresso, e o mais rápido possível,
seguintes Regras e a outras disposições concretas que serão preparados e apresentados à direção relatórios
sejam necessárias e apropriadas, dadas as exigências da completos e demais informações pertinentes sobre a
presunção de inocência, da duração da detenção e da situação pessoal e circunstâncias de cada jovem.
condição e circunstâncias jurídicas dos jovens. Entre 24. No momento do ingresso, todos os jovens
essas disposições, figurarão as seguintes, sem que esta deverão receber uma cópia do regulamento que rege o
enumeração tenha caráter limitativo: centro de detenção e uma descrição completa de seus
a) Os jovens terão direito à assessorai jurídica e direitos e obrigações num idioma que possam
poderão solicitar assistência jurídica gratuita, quando compreender, junto à direção das autoridades
existente, e se comunicar com seus assessores jurídicos. competentes perante as quais podem formular queixas,
Nessa comunicação, deverá ser respeitada a intimidade e assim como dos organismos e organizações públicos ou
seu caráter confidencial. privados que prestem assistência jurídica. Para os jovens
b) Deverá ter dada aos jovens a oportunidade de analfabetos ou que não possam compreender o idioma de
efetuar um trabalho remunerado e de continuar estudos forma escrita, a informação deve ser comunicada de
ou capacitação, mas não serão obrigados a isso. Em maneira que possa ser completamente compreendida.
nenhum caso será mantida a detenção por razões de 25. Todos os jovens deverão ser ajudados a
trabalho, estudos ou capacitação. compreender os regulamentos que regem a organização
c) Os jovens estarão autorizados a receber e interna do centro, os objetivos e metodologia do
conservar materiais de entretenimento e recreio que tratamento utilizado, as exigências e procedimentos
sejam compatíveis com os interesses da administração da disciplinares, outros métodos utilizados para se obter
justiça. informação e formular queixas, e qualquer outra questão
IV. ADMINISTRAÇÃO DOS CENTROS DE DETENÇÃO que facilite a compreensão total de seus direitos e
DE JOVENS A. ANTECEDENTES obrigações durante o internamente.
19. todos os relatórios, incluídos os registros 26. O transporte de jovens deverá ser efetuado às
jurídicos e médicos, as atas das autuações disciplinares, custas da administração, em veículos ventilados e
assim como os demais documentos relacionados forma, o iluminados, e em condições que não tragam nenhum
conteúdo e os dados do tratamento, deverão formar um sofrimento físico ou moral . Os jovens não serão enviados
expediente pessoal e que deverá ser atualizado, acessível de um centro a outro, arbitrariamente.
somente a pessoas autorizadas e classificado de maneira C. CLASSIFICAÇÃO OU DESTINAÇÃO
que se torne facilmente compreensível. Sempre que 27. Depois do ingresso, o jovem será entrevistado
possível, todo jovem terá direito a expor objeções a o mais rápido possível e será preparado um relatório
qualquer fato ou opinião que figure no seu de modo que psicológico e social, onde existam os dados pertinentes
se possa retificar as afirmações inexatas, infundadas ou ao tipo e nível concretos de tratamento e programa que o
injustas. Para o exercício deste direito, seria necessário jovem requer. Este relatório, junto com outro preparado
estabelecer procedimentos que permitissem ao jovem, ou pelo funcionário médico que recebeu o jovem no

www.editoradince.com - Acesse e veja se há novidades a respeito deste material – CUIDADO: cópia é crime.
DIREITOS HUMANOS - Teoria esquematizada, dicas e questões de concursos 41
momento do ingresso, deverá ser apresentado ao diretor entregue limpa, mantida em bom estado e trocada
para se decidir o lugar mais adequado para a instalação regulamentar por motivo de asseio.
do jovem no centro e determinar o tipo e o nível 34. As instalações sanitárias deverão ser de um
necessários de tratamento e de programa que deverão nível adequado e estar localizadas de maneira que o
ser aplicados. jovem possa satisfazer suas necessidades físicas na
28. A detenção de jovens só será feita em intimidade e de forma asseada e decente.
condições que levem em conta, plenamente, suas 35. A posse de objetos pessoais é um elemento
necessidades e situações concretas, assim como os fundamental do direito à intimidade e é indispensável para
requisitos especiais que exijam sua idade, personalidade, o bem-estar psicológico do jovem. O direito de todo jovem
sexo e tipo de delito, e sua saúde física e mental, e que possuir objetos pessoais e dispor lugares seguros para
garantam sua proteção contra influências nocivas e guardá-los deverá ser reconhecido e respeitado
situações de risco. O critério principal para separar os plenamente. Os objetos pessoais que o jovem decida não
diversos grupos de jovens privados de liberdade deverá conservar ou que sejam confiscados deverão ser
ser o tipo de assistência que melhor se adapte às depositados em lugar seguro, e se fará um inventário dos
necessidades concretas dos interessados e a proteção de mesmos, assinado pelo jovem. Serão tomadas medidas
seu bem-estar e integridade física, mental e moral. necessárias para que tais objetos sejam conservados em
29. Em todos os centros, os jovens deverão estar bom estado. Todos os artigos, assim como também o
separados dos adultos, a não ser que sejam da mesma dinheiro, deverão ser restituídos ao jovem em liberdade,
família. Em condições de supervisão, será possível reunir salvo o dinheiro autorizado ou os objetos que tenha
os jovens com adultos cuidadosamente selecionados, no enviado ao exterior. Se o jovem recebe remédios ou se é
marco de um programa especial, cuja utilidade para os descoberto que ele os tem, o médico deverá decidir sobre
jovens interessados tenha sido demonstrada de forma seu uso.
incontestável. 36. Na medida do possível, os jovens terão direito
30. Devem ser organizados centros de detenção a usar suas próprias roupas. Os centros de detenção
abertos para jovens. Entende-se por centros de detenção cuidarão para que todos os jovens tenham roupas
abertos aqueles onde as medidas de segurança são pessoais apropriadas ao clima e suficientes para mantê-
escassas ou nulas. A população desses centros de los em boa saúde. Tais roupas não deverão ser, de modo
detenção deverá ser a mais pequena possível . O número algum, degradantes ou humilhantes. Os jovens que saiam
de jovens internados em centros fechados deverá ser do centro, ou aqueles abandoná-lo por qualquer motivo,
também suficientemente pequeno para que o tratamento poderão usar suas próprias roupas.
possa ter caráter individual. Os centros de detenção para 37. Todos os centros de detenção devem garantir
jovens deverão estar descentralizados e ter um tamanho que todo o jovem terá uma alimentação adequadamente
que facilite o acesso das famílias dos jovens e seu preparada e servida nas horas habituais, em qualidade e
contato com elas. Será conveniente estabelecer quantidade que satisfaçam as normas da dietética, da
pequenos centros de detenção e integrá-los ao contexto higiene e da saúde e, na medida do possível, as
social, econômico e cultural da comunidade. exigências religiosas e culturais. Todo jovem deverá ter, a
D. AMBIENTE FÍSICO OU ALOJAMENTO todo momento, água limpa e potável.
31. Os jovens privados de liberdade terão direito a E. EDUCAÇÃO, FORMAÇÃO PROFISSIONAL OU
contar com locais e serviços que satisfaçam a todas as TRABALHO
exigências da higiene e da dignidade humana. 38. Todo jovem em idade de escolaridade
32. O desenho dos centros de detenção para obrigatória terá o direito de receber um ensino adaptado
jovens e o ambiente físico deverão corresponder a sua as suas idades e capacidades e destinado a prepará-lo
finalidade, ou seja, a reabilitação dos jovens internados, para sua reintegração na sociedade. Sempre que
em tratamento, levando devidamente em conta a sua possível, este ensino deverá ser feito fora do
necessidade de intimidade, de estímulos sensoriais, de estabelecimento, em escolas da comunidade e, em
possibilidades de associação com seus companheiros e qualquer caso, a cargo de professores competentes,
de participação em atividades esportivas, exercícios através de programas integrados ao sistema de ensino
físicos e atividades de entretenimento. O desenho e a público para que, quando sejam postos em liberdade, os
estrutura dos centros de detenção para jovens deverão jovens possam continuar seus estudos sem dificuldade. A
ser tais que reduzam ao mínimo o perigo de incêndio e administração dos estabelecimentos deverá prestar
garantam uma evacuação segura dos locais. Deverá ser atenção especial ao ensino dos jovens de origem
feito um sistema eficaz de alarme para caso de incêndio, estrangeira ou com necessidades culturais ou étnicas
assim como procedimentos estabelecidos e devidamente particulares. Os jovens analfabetos ou que apresentem
ensaiados que garantam a segurança dos jovens. Os problemas cognitivos ou de aprendizagem terão direito a
centros de detenção não estarão localizados em zonas de receber um ensino especial.
conhecidos riscos para a saúde ou onde existam outros 39. Os jovens que já tenham ultrapassado a idade
perigos. de escolaridade obrigatória que desejem continuar seus
33. Os dormitórios deverão ser, normalmente, para estudos deverão ser autorizados e incentivados nesse
pequenos grupos ou individuais, tendo presentes os sentido, e deverá ser feito todo o possível para que
costumes locais. O isolamento em celas individuais tenham acesso a programas de ensino adequados.
durante a noite, só poderá ser imposto em casos 40. Os diplomas ou certificados de estudos
excepcionais e unicamente pelo menor espaço de tempo outorgados aos jovens durante sua detenção não deverão
possível. Durante a noite, todas as zonas destinadas a indicar, de modo algum, que os jovens tenham estado
dormitórios, inclusive as habitações individuais e os detidos.
dormitórios coletivos, deverão ter uma vigilância regular e
41. Todo centro de detenção deverá facilitar o
discreta para assegurar a proteção de cada jovem. Cada
acesso dos jovens a uma biblioteca bem provida de livros
jovem terá, segundo os costumes locais ou nacionais,
e jornais instrutivos e recreativos que sejam adequados, e
roupa de cama individual suficiente, que deverá ser

www.editoradince.com - Acesse e veja se há novidades a respeito deste material – CUIDADO: cópia é crime.
42 D DIREITOS HUMANOS - Teoria esquematizada, dicas e questões de concurso
deverá ser estimulada e permitida a utilização, ao qualquer religião legalmente reconhecida como de sua
máximo, dos serviços da biblioteca. escolha, de não participar de cultos religiosos e de
42. Todo jovem terá direito a receber formação recusar livremente o ensino, a assessoria e a doutrinação
para exercer uma profissão que o prepare para um futuro religiosa.
emprego. H. DETENÇÃO MÉDICA
43. Os jovens poderão optar pela classe de 49. Todo jovem deverá receber atenção médica
trabalho que desejem realizar, levando devidamente em adequada, tanto preventiva como corretiva, incluída a
conta uma seleção profissional racional e as exigências atenção odontológica, oftalmológica e de saúde mental,
da administração do estabelecimento. assim como os produtos farmacêuticos e dietas especiais
44. Todas as normas racionais e internacionais de que tenham sido receitados pelo médico. Normalmente,
proteção aplicadas ao trabalho da criança e aos toda esta atenção médica deverá ser prestada aos jovens
trabalhadores jovens deverão ser aplicadas aos jovens reclusos através dos serviços e instalações sanitários
privados de liberdade. apropriados da comunidade onde esteja localizado o
centro de detenção, com o objetivo de evitar que se
45. Sempre que possível, deverá ser dada aos
estigmatize o jovem e de promover sua dignidade pessoal
jovens a oportunidade de realizar um trabalho
e sua integração à comunidade.
remunerado e, se for factível, no âmbito da comunidade
local, que complemente a formação profissional realizada, 50. Todo jovem terá o direito a ser examinado por
com o objetivo de aumentar a possibilidade de que um médico, imediatamente depois de seu ingresso em um
encontrem um trabalho conveniente quando se centro de jovens, com o objetivo de se constatar qualquer
reintegrarem às suas comunidades. O tipo de trabalho prova de maus-tratos anteriores e verificar qualquer
deverá ser tal que proporcione uma formação adequada, estado físico ou mental que requeira atenção médica.
produtiva para os jovens depois de sua liberação. A 51. Os serviços médicos à disposição dos jovens
organização e os métodos de trabalho regentes nos deverão tratar de detectar e cuidar de toda doença física
centros de detenção deverão ser semelhantes, o mais ou mental, todo uso indevido de substância e qualquer
possível, aos que são aplicados em um trabalho similar na outro estado que possa constituir um obstáculo para a
comunidade, para que os jovens fiquem preparados para integração do jovem na sociedade. Todo centro de
as condições de trabalho normais. detenção de jovens deverá ter acesso imediato a
46. Todo jovem que efetue um trabalho terá direito instalações e equipamento médicos adequados que
a uma remuneração justa. O interesse dos jovens e de tenham relação com o número e as necessidades de seus
sua formação profissional não deve ser subordinado ao residentes, assim como a pessoal capacitado em saúde
propósito de realizar benefícios para o centro de detenção preventiva em tratamento de urgências médicas. Todo
ou para um terceiro. Uma parte da remuneração do jovem jovem que esteja doente, apresente sintomas de
deverá ser reservada para constituir um fundo, que lhe dificuldades físicas ou mentais ou se queixe de doença,
será entregue quando posto em liberdade. O jovem deverá ser examinado rapidamente por um funcionário
deverá ter o direito de utilizar o restante dessa médico.
remuneração para adquirir objetos de uso pessoal, 52. Todo funcionário médico que tenha razões
indenizar a vítima prejudicada pelo seu delito, ou enviar à para estimar que a saúde física ou mental de tenha sido
família ou a outras pessoas fora do centro. afetada, ou possa vir a ser, pela prolongada reclusão,
F. ATIVIDADES RECREATIVAS greve de fome ou qualquer circunstância da reclusão,
deverá comunicar este imediatamente ao diretor do
47. Todo jovem deverá dispor, diariamente, de
estabelecimento e a autoridade independente responsável
tempo disponível para praticar exercícios físicos ao ar
pelo bem-estar do jovem.
livre, se o tempo permitir, durante o qual se proporcionará
normalmente uma educação recreativa e física adequada. 53. Todo jovem que sofra de uma doença deverá
Para tais atividades, serão colocados à sua disposição receber tratamento numa instituição especializada, sob
terreno suficiente, instalações e equipamentos supervisão médica independente. Serão adotadas
necessários. Todo jovem deverá dispor, diariamente, de medidas, de acordo com organismos competentes, para
tempo adicional para atividades de entretenimento, parte que, caso seja necessário, possa continuar o tratamento
das quais deverão ser dedicadas, se o jovem assim o sanitário mental depois da liberação.
desejar, a desenvolver aptidões nas artes. O centro de 54. Os centros de detenção deverão organizar
detenção deverá verificar se todo jovem é fisicamente programas de prevenção do uso indevido de drogas e de
apto para participar dos programas de educação física reabilitação, administrados por pessoal qualificado. Estes
disponíveis. Deverá ser oferecida educação física programas deverão ser adaptados à idade, sexo e a
corretiva e terapêutica, sob supervisão médica, aos outras circunstâncias dos jovens interessados, e deverão
jovens necessitados. ser oferecidos serviços de desintoxicação, dotados de
G. RELIGIÃO pessoal qualificado, aos jovens toxicômanos ou
alcoólatras.
48. Todo jovem terá o direito de cumprir os
preceitos de sua religião, participar dos cultos ou reuniões 55. Somente serão receitados remédios para um
organizados no estabelecimento ou celebrar seus próprios necessário ou por razões médicas e, possível, depois do
cultos e ter em seu poder livros ou objetos de culto e de consentimento do jovem. Em particular, nunca serão
instrução religiosa de seu credo. Se no centro de receitados para se obter informação ou confissão, nem
detenção houver um número suficiente de jovens que como castigo reprimir o jovem. Os jovens nunca serão
professam uma determinada religião, deverá ser nomeado objeto para experimentar o emprego de tratamentos. O
ou admitir-se-á um ou mais representantes autorizados uso de qualquer remédio sempre ser autorizado e
desse culto que poderão organizar, periodicamente, cultos efetuado pelo médico qualificado.
religiosos e efetuar visitas pastorais particulares aos I. VERIFICAÇÃO DA DOENÇA, DE ACIDENTE E MORTE
jovens de sua religião. Todo jovem terá o direito de 56. A família ou o tutor de um jovem, ou qualquer
receber visitas de um representante qualificado de outra pessoa designada pelo mesmo, têm o direito de

www.editoradince.com - Acesse e veja se há novidades a respeito deste material – CUIDADO: cópia é crime.
DIREITOS HUMANOS - Teoria esquematizada, dicas e questões de concursos 43
serem informados, caso solicitem, sobre o estado do por uma lei ou regulamento. Esses instrumentos não
jovem e qualquer mudança que aconteça nesse sentido. deverão causar lesão, dor, humilhação, nem degradação,
Em caso de falecimento, requeira o envio do jovem a um e deverão ser usados de forma restrita e pelo menor
centro médico fora do centro ou um estado que exija período de tempo possível. Por ordem do diretor da
tratamento por mais de 48 horas no serviço clínico do administração, estes instrumentos poderão ser utilizados
centro de detenção, o diretor do centro deverá avisar, para impedir que o menor prejudique a outros ou a si
imediatamente, à família, ao tutor ou a qualquer outra mesmo ou cause sérios danos materiais. Nesse caso, o
pessoa designada pelo jovem. diretor deverá consultar, imediatamente, o pessoal médico
57. Em caso de falecimento de um jovem durante e outro pessoal competente e informar à autoridade
o período de privação de liberdade, o parente mais administrativa superior.
próximo terá o direito de examinar a certidão de óbito, de 65. Em todo centro onde haja jovens detidos,
ver o cadáver e de decidir seu destino. Em caso de deverá ser proibido o porte e o uso de armas por parte
falecimento de um jovem durante sua detenção, deverá dos funcionários.
ser feita uma pesquisa independente sobre as causas da L. PROCEDIMENTOS DISCIPLINARES
morte, cujas conclusões deverão ficar à disposição do
66. Todas as medidas e procedimentos
parente mais próximo. Tal pesquisa deverá ser feita
disciplinares Deverão contribuir para a segurança e para
quando a morte do jovem ocorrer dentro dos seis meses
uma vida comunitária ordenada e ser compatíveis com o
seguintes à data de sua liberação, e quando houver
respeito à dignidade inerente do jovem e com o objetivo
suspeita de que a morte tem relação com o período de
fundamental do tratamento institucional, ou seja, infundir
reclusão.
um sentimento de justiça e de respeito por si mesmo e
58. O jovem deverá ser informado, imediatamente, pelos direitos fundamentais de toda pessoa.
da morte ou da doença ou de um acidente grave com um
67. Todas as medidas disciplinares que sejam
familiar e poderá ir ao enterro ou, em caso de doença
cruéis, desumanas ou degradantes, estarão estritamente
grave de um parente, ir visitar o enfermo.
proibidas, incluídos os castigos corporais, o recolhimento
J. CONTATOS COM A COMUNIDADE EM GERAL em cela escura e as penalidades de isolamento ou de
59. Deverão ser utilizados todos os meios para solitária, assim como qualquer outro castigo que possa
garantir uma comunicação adequada dos jovens com o pôr em perigo a saúde física ou mental do menor. A
mundo exterior, comunicação esta que é parte integrante redução de alimentos e a restrição ou proibição de
do direito a um tratamento justo e humanitário e é contato com familiares estarão proibidas, seja qual for a
indispensável para a reintegração dos jovens à finalidade. O trabalho será considerado, sempre, um
sociedade. Deverá ser permitida aos jovens a instrumento de educação e um meio de promover o
comunicação com seus familiares, seus amigos e outras respeito próprio do jovem, como preparação para sua
pessoas ou representantes de organizações prestigiosas reintegração à comunidade, e nunca deverá ser imposto
do exterior; sair dos centros de detenção para visitar seu como castigo disciplinar. Nenhum jovem poderá ser
lar e sua família e obter permissão especial para sair do castigado mais de uma vez pela mesma infração. Os
estabelecimento por motivos educativos, profissionais ou castigos coletivos devem ser proibidos.
outras razões importantes. Em caso de o jovem estar 68. As leis ou regulamentos aprovados pela
cumprindo uma pena, o tempo passado fora do autoridade administrativa competente deverão
estabelecimento deverá ser contado como parte do estabelecer normas relativas aos seguintes pontos,
período de cumprimento da sentença. levando-se em conta as características, necessidades e
60. Todo jovem deverá ter o direito de receber direitos fundamentais do jovem:
visitas regulares e frequentes, a princípio uma vez por a) a conduta que seja uma infração disciplinar;
semana e, pelo menos, uma vez por mês, em condições
b) o caráter e a depuração dos castigos
que respeitem a necessidade de intimidade do jovem, o
disciplinares que podem ser aplicados;
contato e a comunicação, sem restrições, com a família e
com o advogado de defesa. c) a autoridade competente para impor estes
castigos;
61. Todo jovem terá o direito de se comunicar por
escrito ou por telefone, pelo menos duas vezes por d) a autoridade competente no grau de apelação.
semana, com a pessoa de sua escolha, salvo se, 69. Um relatório de má conduta deverá ser
legalmente, não puder fazer uso desse direito, e deverá apresentado, imediatamente, à autoridade com que
receber a assistência necessária para que possa exercer deverá decidir a respeito, sem delongas injustificadas. A
eficazmente esse direito. Todo jovem terá o direito a autoridade competente deverá examinar o caso com
receber toda a correspondência a ele dirigida. cuidado.
62. Os jovens deverão ter a oportunidade de se 70. Um castigo disciplinar só será imposto a um
informar, periodicamente, os acontecimentos através de jovem se estiver estritamente de acordo com o disposto
jornais, revistas ou outras publicações, programas de nas leis ou regulamentos em vigor. Nenhum jovem será
rádio, televisão e cinema, como também através de visitas castigado sem que tenha sido devidamente informado da
dos representantes de qualquer clube ou organização de infração que o acusam, de maneira que possa entender, e
caráter legal que o jovem esteja interessado. sem que tenha a oportunidade de se defender, incluído o
K. IMITAÇÃO DA COERÇÃO FÍSICA OU USO DA direito apelar a uma autoridade competente imparcial.
FORÇA Deverá ser feita uma ata completa com todas as
autuações disciplinares.
63. O uso de instrumentos de coerção e a força,
com qualquer fim, deverá ser proibido, salvo nos casos 71. Nenhum jovem deverá ter, a seu encargo,
estabelecidos no Artigo 63. funções disciplinares, salvo no que se refere à supervisão
de certas atividades sociais, educativas ou esportivas de
64. Somente em casos excepcionais se poderá
autogestão.
usar a força ou instrumentos de coerção, quando todos os
demais meios de controle tenham esgotado e fracassado, M. INSPEÇÃO A RECLAMAÇÕES
e apenas pela forma expressamente autorizada e descrita

www.editoradince.com - Acesse e veja se há novidades a respeito deste material – CUIDADO: cópia é crime.
44 D DIREITOS HUMANOS - Teoria esquematizada, dicas e questões de concurso
72. Os inspetores qualificados ou uma entidade possível, deverão proporcionar alojamento, trabalho e
devidamente constituída, de nível equivalente, que não roupas convenientes ao jovem, assim como os meios
pertençam à administração do centro deverão ter a necessários para sua subsistência depois de sua
faculdade de efetuar visitas periódicas, sem prévio aviso, liberação. Os representantes de organismos que prestam
por iniciativa própria e gozar de plenas garantias de estes serviços deverão ser consultados, e terão acesso
independência no exercício desta função. Os inspetores aos jovens durante sua reclusão, com vistas à assistência
deverão ter acesso, sem restrição, a todas as pessoas que possam prestar para sua reintegração na
empregadas ou que trabalhem nos estabelecimentos ou comunidade.
instalações onde haja, ou possa haver, jovens privados de O. FUNCIONÁRIOS
liberdade, e a todos os jovens e a toda a documentação
81. O pessoal deverá ser competente e contar com
dos estabelecimentos.
um número suficiente de especialistas, como educadores,
73. Nas inspeções, deverão participar funcionários instrutores profissionais, assessores, assistentes sociais,
médicos especializados, adscritos à entidade inspetora ou psiquiatras e psicólogos. Normalmente, estes funcionários
a serviço da saúde pública, os quais deverão avaliar o e outros especialistas deverão formar parte do pessoal
cumprimento das regras relativas ao ambiente físico, à permanente, mas isso não excluirá os auxiliares de tempo
higiene, ao alojamento, à comida, ao exercício e aos parcial ou voluntários, quando for apropriado, e trouxer
serviços médicos, assim como a quaisquer outros benefícios ao estabelecimento. Os centros de detenção
aspectos ou condições da vida do centro que afetem a deverão aproveitar todas as possibilidades e modalidades
saúde física e mental dos jovens. Todos os jovens terão de assistência corretiva, educativa, moral, espiritual e de
direito a falar confidencialmente com os inspetores. outra índole que estejam disponíveis na comunidade e
74. Determinada a inspeção, o inspetor deverá que sejam idôneas, em função das necessidades e dos
apresentar um relatório com suas conclusões. Este problemas particulares dos jovens reclusos.
relatório incluirá uma avaliação da forma como o centro 82. A administração deverá selecionar e contratar,
de detenção observa as presentes Regras e disposições cuidadosamente, pessoal de todas as classes e
pertinentes da legislação acional, assim como categorias, já que o bom andamento dos centros de
recomendações sobre as medidas consideradas detenção depende da integridade, atitude humanitária,
necessárias para garantir seu cumprimento. Todo ato capacidade e competência dos funcionários para tratar os
descoberto por um inspetor, que indique uma violação das jovens, assim como os seus dotes pessoais para o
disposições legais relativas aos direitos dos jovens ou ao trabalho.
funcionamento do centro de detenção, deverá ser
83. Para alcançar tais objetivos, deverão ser
comunicado às autoridades competentes para
designados funcionários profissionais, com remuneração
investigação e para que se exija as responsabilidades
suficiente para atrair e reter homens e mulheres capazes.
correspondentes.
Deverá ser dado, a todo momento, estímulo aos
75. Todo jovem deverá ter a oportunidade de funcionários dos centros de detenção de jovens para que
apresentar, a todo momento, petições ou queixas ao desempenhem suas funções e obrigações profissionais
diretor do estabelecimento ou a seu representante de forma humanitária, dedicada, profissional, justa e
autorizado. eficaz, comportem-se, a todo momento, de tal maneira
76. Todo jovem terá direito de enviar, pela via que mereçam e obtenham o respeito dos jovens, e sejam,
prescrita e sem censura quanto ao conteúdo, uma petição para estes, um modelo e uma perspectiva positivos.
ou queixa à administração central dos estabelecimentos 84. A administração deverá adotar formas de
para jovens, à autoridade judicial ou a qualquer outra organização e de gestão que facilitem a comunicação
autoridade competente, e a ser informado, sem demora, entre as diferentes categorias de funcionários de cada
da resposta. centro de detenção, para que seja intensificada a
77. Deverá se tentar criar um escritório cooperação entre os diversos serviços dedicados à
independente (ombudsman) encarregado de receber e atenção de jovens, também entre o pessoal e a
pesquisar as queixas formuladas pelos jovens privados de administração, com vistas a conseguir que o pessoal em
sua liberdade e de ajudar na obtenção de soluções contato direto com os jovens possa atuar em condições
equitativas. que favoreçam o desempenho eficaz de suas tarefas.
78. Para a formulação de uma queixa, todo jovem 85. O pessoal deverá receber uma formação que
terá o direito de solicitar assistência aos membros de sua permita o desempenho eficaz de suas funções,
família, a assessores jurídicos, a grupos humanitários ou particularmente a capacitação em psicologia infantil,
outros, quando possível. Será prestada assistência aos proteção da infância e critérios e normas internacionais de
jovens analfabetos, quando estes necessitem recorrer aos direitos humanos e direitos da criança, incluídas as
serviços de organismos ou organizações públicas ou presentes Regras. O pessoal deverá manter e aperfeiçoar
privadas, que oferecem assessoria jurídica ou que sejam seus conhecimentos e capacidade profissional,
competentes para receber reclamações. comparecendo a cursos de formação no serviço, que
N. REINTEGRAÇÃO NA SOCIEDADE serão organizados, periodicamente.
79. Todos os jovens deverão ser beneficiados com 86. O diretor do centro deverá estar devidamente
medidas concebidas para ajudar sua reintegração na Qualificado para sua função, por sua capacidade
sociedade, na vida familiar, na educação ou no trabalho administrativa, por uma formação adequada e por sua
depois de postos em liberdade. Para tal fim, deverão ser experiência na matéria, e deverá dispor de todo o seu
estabelecidos certos procedimentos, inclusive a liberdade tempo para a sua função oficial.
antecipada, e cursos especiais. 87. No desempenho de suas funções, o pessoal
80. As autoridades competentes deverão criar ou dos centros de detenção deverá respeitar e proteger a
recorrer a serviços que ajudem a reintegração dos jovens dignidade e os direitos humanos fundamentais de todos
na sociedade, e contribuam para diminuir os preconceitos os jovens, especialmente:
existentes contra eles. Estes serviços, na medida do

www.editoradince.com - Acesse e veja se há novidades a respeito deste material – CUIDADO: cópia é crime.
DIREITOS HUMANOS - Teoria esquematizada, dicas e questões de concursos 45
a) nenhum membro do pessoal do centro de Quais estão corretas?
detenção ou da instituição deverá infligir, instigar ou A) Apenas I.
tolerar nenhum ato de tortura, nem forma alguma de
B) Apenas II.
tratamento, castigo ou medida corretiva ou disciplinar
severa, cruel, desumana ou degradante, sob nenhum C) Apenas III.
pretexto ou circunstância de qualquer tipo; D) Apenas I e II.
b) todo o pessoal deverá impedir e combater, E) I, II e III.
severamente, todo ato de corrupção, comunicando-o, sem
demora, às autoridades competentes; 03. (Médico Judiciário/PJ/RJ) Segundo as Regras
c) todo o pessoal deverá respeitar estas Regras. Mínimas das Nações Unidas para a Proteção dos
Quando tiverem motivos para suspeitar que estas Regras Jovens Privados de Liberdade (Volpi), no que se
foram gravemente violadas, ou possam vir a ser, deverão refere à educação, formação profissional e trabalho,
comunicar as suas autoridades superiores ou órgãos julgue os itens que seguem:
competentes com responsabilidade para supervisionar ou I. Todo jovem que efetue um trabalho terá direito a uma
remediar a situação; remuneração justa.
d) todo o pessoal deverá velar pela total proteção II. Todas as normas nacionais e internacionais de
da saúde física e mental dos jovens, incluída a proteção proteção aplicadas ao trabalho da criança e aos
contra a exploração e maus tratos físicos, sexuais e trabalhadores jovens deverão ser aplicados aos
efetivos e deverá adotar, com urgência, medidas para que jovens privados de liberdade.
recebam atenção médica, sempre que necessário;
III. Todo jovem terá direito a receber formação para
e) todo o pessoal deverá respeitar o direito dos exercer uma profissão que o prepare para um futuro
jovens à intimidade e deverá respeitar, em particular, emprego, desde que lhe seja oferecida a condição
todas as questões confidenciais relativas aos jovens ou às para pagamento.
suas famílias que cheguem a conhecer no exercício de
sua atividade profissional; Quais estão corretas?
f) todo o pessoal deverá reduzir, ao mínimo, as A) Apenas I.
diferenças entre a vida dentro e fora do centro de B) Apenas I e II.
detenção que tendam a diminuir o devido respeito à C) Apenas III.
dignidade dos jovens como seres humanos. D) Apenas I e III.
Tradução ao português de Betsáida Dias Capilé E) I, II e III.
Revisão de Emílio Garcia Mendez e Lidia Galeano.
(04. FAURGS - 2016 - TJ-RS - Pedagogo Judiciário)
QUESTÕES DE COCNURSOS Conforme previsto nas ―Regras Mínimas das Nações
Unidas para a Proteção dos Jovens Privados de
01. (CESPE/PB - FUNDAC/PB - Advogado) No que se Liberdade‖, pode-se afirmar que
refere às regras mínimas propostas pelas Nações
Unidas para a proteção de jovens privados de A o ensino dos jovens privados de liberdade deverá ser
liberdade, assinale a opção correta. feito fora do estabelecimento, em escolas da
comunidade, sempre que possível.
A. Os jovens privados de liberdade que, durante a
detenção, realizarem estudos, receberão os B os diplomas ou certificados de estudos outorgados aos
respectivos diplomas e certificados com o registro das jovens durante sua detenção deverão indicar que os
qualificações obtidas nos cursos bem como do mesmos estavam detidos, tendo em vista sua
período em que ficaram detidos. aceitação em níveis de ensino compatíveis.
B. As regras mínimas prevêem a recomendação para que C todo jovem terá direito a receber formação, porém, essa
o jovem privado de liberdade trabalhe, mas não de deve ser genérica e não voltada especificamente para
forma remunerada. um futuro emprego.
C. Os servidores dos centros onde haja jovens privados D em hipótese alguma será previsto trabalho aos jovens
de liberdade são proibidos de portar e de usar armas. privados de liberdade, considerando-se as normas
nacionais e internacionais de proteção ao trabalho de
D. Cartas encaminhadas pelos jovens privados de crianças e adolescentes.
liberdade às autoridades judiciais deverão ser
encaminhadas ao diretor do estabelecimento para E toda a remuneração do jovem deverá ser reservada
análise do conteúdo. para constituir um fundo, que lhe será entregue
quando posto em liberdade, não devendo ser utilizada
para indenização da vítima prejudicada pelo seu
02. (Médico Judiciário/PJ/RJ) Segundo as Regras delito.
Mínimas das Nações Unidas para a Proteção dos
Jovens Privados de Liberdade (Volpi), em todos os
lugares onde haja jovens detidos, deverá ser mantido Gabarito: 01/C; 02/E; 03/D; 04/A;
em registro completo e confiável da(s) seguinte(s)
informação(ões) relativa a cada um dos jovens
admitidos: REGRAS MÍNIMAS DAS NAÇÕES
I. A causa da reclusão, assim como seus motivos e a UNIDAS PARA A ADMINISTRAÇÃO DA
autoridade que ordenou. JUSTIÇA DA INFÂNCIA E DA
II. O dia e a hora do ingresso, da mudança e da liberação. JUVENTUDE – REGRAS DE BEIJING.
III. Detalhes sobre os problemas de saúde física e mental PRIMEIRA PARTE - PRINCÍPIOS GERAIS
conhecidos, incluído o uso indevido de drogas e
álcool. 1. Orientações fundamentais

www.editoradince.com - Acesse e veja se há novidades a respeito deste material – CUIDADO: cópia é crime.
46 D DIREITOS HUMANOS - Teoria esquematizada, dicas e questões de concurso
1.1 Os Estados Membros procurarão, em c) aplicar cabalmente e com justiça as regras que
consonância com seus respectivos interesses gerais, se enunciam a seguir.
promover o bem-estar da criança e do adolescente e de 3. Ampliação do âmbito de aplicação das regras
sua família.
3.1 As disposições pertinentes das regras não só
1.2 Os Estados Membros se esforçarão para criar se aplicarão aos jovens infratores, mas também àqueles
condições que garantam à criança e ao adolescente uma que possam ser processados por realizar qualquer ato
vida significativa na comunidade, fomentando, durante o concreto que não seria punível se fosse praticado por
período de idade em que ele é mais vulnerável a um adultos.
comportamento desviado, um processo de
3.2 Procurar-se-á estender o alcance dos
desenvolvimento pessoal e de educação o mais isento
princípios contidos nas regras a todos os jovens
possível do crime e da delinquência.
compreendidos nos procedimentos relativos à atenção à
1.3 Conceder-se-á a devida atenção à adoção de criança e ao adolescente e a seu bem-estar.
medidas concretas que permitam a mobilização de todos
3.3 Procurar-se-á também estender o alcance dos
os recursos disponíveis, com a inclusão da família, de
princípios contidos nas regras aos infratores adultos
voluntários e outros grupos da comunidade, bem como da
jovens.
escola e de demais instituições comunitárias, com o fim
de promover o bem-estar da criança e do adolescente, 4. Responsabilidade penal
reduzir a necessidade da intervenção legal e tratar de 4.1 Nos sistemas jurídicos que reconheçam o
modo efetivo, equitativo e humano a situação de conflito conceito de responsabilidade penal para jovens, seu
com a lei. começo não deverá fixar-se numa idade demasiado
1.4 A Justiça da Infância e da Juventude será precoce, levando-se em conta as circunstâncias que
concebida como parte integrante do processo de acompanham a maturidade emocional, mental e
desenvolvimento nacional de cada país e deverá ser intelectual.
administrada no marco geral de justiça social para todos 5. Objetivos da Justiça da Infância e da Juventude
os jovens, de maneira que contribua ao mesmo tempo 5.1 O sistema de Justiça da Infância e da
para a sua proteção e para a manutenção da paz e da Juventude enfatizará o bem-estar do jovem e garantirá
ordem na sociedade. que qualquer decisão em relação aos jovens infratores
1.5 As presentes regras se aplicarão segundo o será sempre proporcional às circunstâncias do infrator e
contexto das condições econômicas, sociais e culturais da infração.
que predominem em cada um dos Estados Membros. 6. Alcance das faculdades discricionárias
1.6 Os serviços da Justiça da Infância e da 6.1 Tendo-se em conta as diversas necessidades
Juventude se aperfeiçoarão e se coordenarão especiais dos jovens, assim como a diversidade de
sistematicamente com vistas a elevar e manter a medidas disponíveis, facultar-se-á uma margem suficiente
competência de seus funcionários, os métodos, enfoques para o exercício de faculdades discricionárias nas
e atitudes adotadas. diferentes etapas dos processos e nos distintos níveis da
2. Alcance das regras e definições utilizadas administração da Justiça da Infância e da Juventude,
2.1 As regras mínimas uniformes que se enunciam incluídos os de investigação, processamento, sentença e
a seguir se aplicarão aos jovens infratores com das medidas complementares das decisões.
imparcialidade, sem distinção alguma, por exemplo, de 6.2 Procurar-se-á, não obstante, garantir a devida
raça, cor, sexo, idioma, religião, opinião política ou de competência em todas as fases e níveis no exercício de
qualquer outra natureza, origem nacional ou social, quaisquer dessas faculdades discricionárias.
posição econômica, nascimento ou qualquer outra 6.3 Quem exercer tais faculdades deverá estar
condição. especialmente preparado ou capacitado para fazê-lo
2.2 Para os fins das presentes regras, os Estados judiciosamente e em consonância com suas respectivas
Membros aplicarão as definições seguintes, de forma funções e mandatos.
compatível com seus respectivos sistemas e conceitos 7. Direitos dos jovens
jurídicos: 7.1 Respeitar-se-ão as garantias processuais
a) jovem é toda a criança ou adolescente que, de básicas em todas as etapas do processo, como a
acordo com o sistema jurídico respectivo, pode responder presunção de inocência, o direito de ser informado das
por uma infração de forma diferente do adulto; acusações, o direito de não responder, o direito à
b) infração é todo comportamento (ação ou assistência judiciária, o direito à presença dos pais ou
omissão) penalizado com a lei, de acordo com o tutores, o direito à confrontação com testemunhas e a
respectivo sistema jurídico; interrogá-las e o direito de apelação ante uma autoridade
c) jovem infrator é aquele a quem se tenha superior.
imputado o cometimento de uma infração ou que seja 8. Proteção da intimidade
considerado culpado do cometimento de uma infração. 8.1 Para evitar que a publicidade indevida ou o
2.3 Em cada jurisdição nacional procurar-se-á processo de difamação prejudiquem os jovens, respeitar-
promulgar um conjunto de leis, normas e disposições se-á, em todas as etapas, seu direito à intimidade.
aplicáveis especificamente aos jovens infratores, assim 8.2 Em princípio, não se publicará nenhuma
como aos órgãos e instituições encarregados das funções informação que possa dar lugar à identificação de um
de administração da Justiça da Infância e da Juventude, jovem infrator.
com a finalidade de: 9. Cláusula de salvaguarda
a) satisfazer as diversas necessidades dos jovens 9.1 Nenhuma disposição das presentes regras
infratores, e ao mesmo tempo proteger seus direitos poderá ser interpretada no sentido de excluir os jovens do
básicos; âmbito da aplicação das Regras Mínimas Uniformes para
b) satisfazer as necessidades da sociedade; o Tratamento dos Prisioneiros, aprovadas pelas Nações

www.editoradince.com - Acesse e veja se há novidades a respeito deste material – CUIDADO: cópia é crime.
DIREITOS HUMANOS - Teoria esquematizada, dicas e questões de concursos 47
Unidas, e de outros instrumentos e normas relativos ao social, educacional, profissional, psicológica, médica e
cuidado e à proteção dos jovens reconhecidos pela física que requeiram, tendo em conta sua idade, sexo e
comunidade internacional. características individuais.
SEGUNDA PARTE - lNVESTIGAÇÃO E TERCEIRA PARTE - DECISÃO JUDICIAL E MEDIDAS
PROCESSAMENTO 14. Autoridade competente para decidir
10. Primeiro contato 14.1 Todo jovem infrator, cujo caso não tenha sido
10.1 Sempre que um jovem for apreendido, a objeto de remissão (de acordo com a regra será
apreensão será notificada imediatamente a seus pais ou apresentado à autoridade competente Juizado, tribunal,
tutor e, quando não for possível tal notificação imediata, junta, conselho etc.), que decidirá de acordo com os
será notificada aos pais ou tutor no mais breve prazo princípios de um processo imparcial e justo.
possível. 14.2 Os procedimentos favorecerão os interesses
10.2 O juiz, funcionário ou organismo competentes do jovem e serão conduzidos numa atmosfera de
examinarão sem demora a possibilidade de pôr o jovem compreensão, que lhe permita participar e se expressar
em liberdade. livremente.
10.3 Os contatos entre os órgãos encarregados de 15. Assistência judiciária e direitos dos pois o tutores
fazer cumprir a lei e o jovem infrator serão estabelecidos 15.1 O jovem terá direito a se fazer representar por
de modo a que seja respeitada a sua condição jurídica, um advogado durante todo o processo ou a solicitar
promova-se o seu bem-estar e evite-se que sofra dano, assistência judiciária gratuita, quando prevista nas leis do
resguardando-se devidamente as circunstâncias do caso. país.
11. Remissão dos casos 15.2 Os pais ou tutores terão direito de participar
11.1 Examinar-se-á a possibilidade, quando dos procedimentos e a autoridade competente poderá
apropriada, de atender os jovens infratores sem recorrer requerer a sua presença no interesse do jovem. Não
às autoridades competentes, mencionadas na regra 14.1 obstante, a autoridade competente poderá negar a
adiante, para que os julguem oficialmente. participação se existirem motivos para presumir que a
11. 2 A polícia, o ministério público e outros exclusão é necessária aos interesses do jovem.
organismos Que se ocupem de jovens infratores terão a 16. Relatórios de investigação social
faculdade de arrolar tais casos sob sua jurisdição, sem 16.1 Para facilitar a adoção de uma decisão justa
necessidade de procedimentos formais, de acordo com por parte da autoridade competente, a menos que se
critérios estabelecidos com esse propósito nos tratem de infrações leves, antes da decisão definitiva será
respectivos sistemas jurídicos e também em harmonia efetuada uma investigação completa sobre o meio social
com os princípios contidos nas presentes regras. e as circunstâncias de vida do jovem e as condições em
11.3 Toda remissão que signifique encaminhar o que se deu a prática da infração.
jovem a instituições da comunidade ou de outro tipo 17. Princípios norteadores da decisão judicial o das
dependerá do consentimento dele, de seus pais ou medidas
tutores; entretanto, a decisão relativa à remissão do caso
17.1 A decisão da autoridade competente pautar-
será submetida ao exame de uma autoridade competente,
se-á pelos seguintes princípios:
se assim for solicitado.
a) a resposta à infração será sempre proporcional
11.4 Para facilitar a tramitação jurisdicional dos
não só às circunstâncias e à gravidade da infração, mas
casos de jovens, procurar-se-á proporcionar à
também às circunstâncias e às necessidades do jovem,
comunidade programas tais como orientação e supervisão
assim como às necessidades da sociedade;
temporária, restituição e compensação das vítimas.
b) as restrições à liberdade pessoal do jovem
12. Especialização policial
serão impostas somente após estudo cuidadoso e se
12.1 Para melhor desempenho de suas funções, reduzirão ao mínimo possível;
os policiais que tratem frequentemente ou de maneira
c) não será imposta a privação de liberdade
exclusiva com jovens ou que se dediquem
pessoal a não ser que o jovem tenha praticado ato grave,
fundamentalmente à prevenção da delinquência de jovens
envolvendo violência contra outra pessoa ou por
receberão instrução e capacitação especial. Nas grandes
reincidência no cometimento de outras infrações sérias, e
cidades, haverá contingentes especiais de polícia com
a menos que não haja outra medida apropriada;
essa finalidade.
d) o bem-estar do jovem será o fator
13. Prisão preventiva
preponderante no exame dos casos.
13.1 Só se aplicará a prisão preventiva como
17.2 A pena capital não será imposta por qualquer
último recurso e pelo menor prazo possível.
crime cometido por jovens.
13.2 Sempre que possível, a prisão preventiva
17.3 Os jovens não serão submetidos a penas
será substituída por medidas alternativas, como a estrita
corporais.
supervisão, custódia intensiva ou colocação junto a uma
família ou em lar ou instituição educacional. 17.4 A autoridade competente poderá suspender o
processo em qualquer tempo.
13.3 Os jovens que se encontrem em prisão
preventiva gozarão de todos os direitos e garantias 18. Pluralidade das medidas aplicáveis
previstos nas Regras Mínimas para o Tratamento de 18.1 Uma ampla variedade de medidas deve estar
Prisioneiros, aprovadas pelas Nações Unidas. à disposição da autoridade competente, permitindo a
13.4 Os jovens que se encontrem em prisão flexibilidade e evitando ao máximo a institucionalização.
preventiva estarão separados dos adultos e recolhidos a Tais medidas, que podem algumas vezes ser
estabelecimentos distintos ou em recintos separados nos aplicadas simultaneamente, incluem:
estabelecimentos onde haja detentos adultos. a) determinações de assistência, orientação e
13.5 Enquanto se encontrem sob custódia, os supervisão;
jovens receberão cuidados, proteção e toda assistência - b) liberdade assistida;

www.editoradince.com - Acesse e veja se há novidades a respeito deste material – CUIDADO: cópia é crime.
48 D DIREITOS HUMANOS - Teoria esquematizada, dicas e questões de concurso
c) prestação de serviços à comunidade; reabilitação do jovem num ambiente comunitário e, tanto
d) multas, indenizações e restituições; quanto possível, na unidade familiar.
e) determinação de tratamento institucional ou QUINTA PARTE - TRATAMENTO INSTITUCIONAL
outras formas de tratamento; 26. Objetivos do tratamento institucional
f)determinação de participar em sessões de grupo 26.1 A capacitação e o tratamento dos jovens
e atividades similares; colocados em instituições têm por objetivo assegurar seu
g) determinação de colocação em lar substituto, cuidado, proteção, educação e formação profissional para
centro de convivência ou outros estabelecimentos permitir-lhes que desempenhem um papel construtivo e
educativos; produtivo na sociedade.
h) outras determinações pertinentes. 26.2 Os jovens institucionalizados receberão os
cuidados, a proteção e toda a assistência necessária
18.2 Nenhum jovem será excluído, total ou
social, educacional, profissional, psicológica, médica e
parcialmente, da supervisão paterna, a não ser que as
física que requeiram devido à sua idade, sexo e
circunstâncias do caso o tornem necessário.
personalidade e no interesse do desenvolvimento sadio.
19. Caráter excepcional da institucionalização
26.3 Os jovens institucionalizados serão mantidos
19.1 A internação de um jovem em uma instituição separados dos adultos e serão detidos em
será sempre uma medida de último recurso e pelo mais estabelecimentos separados ou em partes separadas de
breve período possível. um estabelecimento em que estejam detidos adultos.
20. Prevenção de demoras desnecessárias 26.4 A jovem infratora institucionalizada merece
20.1 Todos os casos tramitarão, desde o começo, especial atenção no que diz respeito às suas
de maneira expedita e sem demoras desnecessárias. necessidades e problemas pessoais. Em nenhum caso
21. Registros receberá menos cuidado, proteção, assistência,
21.1 Os registros de jovens infratores serão de tratamento e capacitação que o jovem do sexo masculino.
caráter estritamente confidencial e não poderão ser Será garantido seu tratamento equitativo.
consultados por terceiros. Só terão acesso aos arquivos 26.5 No interesse e para o bem-estar do jovem
as pessoas que participam diretamente da tramitação do institucionalizado, os pais e tutores terão direito de acesso
caso ou outras pessoas devidamente autorizadas. às instituições.
21.2 Os registros dos jovens infratores não serão 26.6 Será estimulada a cooperação interministerial
utilizados em processos de adultos em casos e interdepartamental para proporcionar adequada
subsequentes que envolvam o mesmo infrator. formação educacional ou, se for o caso, profissional ao
22. Necessidade de profissionalismo e capacitação jovem institucionalizado, para garantir que, ao sair, não
esteja em desvantagem no plano da educação.
22.1 Serão utilizados a educação profissional, o
treinamento em serviço, a reciclagem e outros meios 27. Aplicação das Regras Mínimas para o Tratamento
apropriados de instrução para estabelecer e manter a dos Prisioneiros, aprovadas pelas Nações Unidas
necessária competência profissional de todo o pessoal 27.1 Em princípio, as Regras Mínimas para o
que se ocupa dos casos de jovens. Tratamento dos Prisioneiros e as recomendações
22.2 O quadro de servidores da Justiça da Infância conexas serão aplicáveis, sempre que for pertinente, ao
e da Juventude deverá refletir as diversas características tratamento dos jovens infratores institucionalizados,
dos jovens que entram em contato com o sistema. inclusive os que estiverem em prisão preventiva.
Procurar-se-á garantir uma representação equitativa de 27.2 Deverão ser feitos esforços para implementar
mulheres e minorias nos órgãos da Justiça da Infância e os princípios relevantes das mencionadas Regras
da Juventude. Mínimas na maior medida possível, para satisfazer as
QUARTA PARTE - TRATAMENTO EM MEIO ABERTO necessidades específicas do jovem quanto à sua idade,
sexo e personalidade.
23. Execução efetivadas medidas
28. Uso frequente e imediato da liberdade condicional
23.1 Serão adotadas disposições adequadas para
o cumprimento das determinações ditadas pela 28.1 A liberdade condicional da instituição será
autoridade competente, mencionadas na regra 14.1, por utilizada pela autoridade pertinente na maior medida
essa mesma autoridade ou por outra diferente, se as possível e será concedida o mais cedo possível.
circunstâncias assim o exigirem. 28.2 O jovem liberado condicionalmente de uma
23.2 Tais dispositivos incluirão a faculdade da instituição será assistido e supervisionado por funcionário
autoridade competente para modificar periodicamente as designado e receberá total apoio da comunidade.
determinações segundo considere adequado, desde que 29. Sistemas semi-institucionais
a modificação se paute pelos princípios enunciados 29.1 Procurar-se-á estabelecer sistemas semi-
nestas regras. institucionais, como casas de semiliberdade, lares
24. Prestação da assistência necessária educativos, centros de capacitação diurnos e outros
24.1 Procurar-se-á proporcionar aos jovens, em sistemas apropriados que possam facilitar a adequada
todas as etapas dos procedimentos, assistência em reintegração dos jovens na sociedade.
termos de alojamento, ensino e capacitação profissional, SEXTA PARTE - PESQUISA, PLANEJAMENTO E
emprego ou qualquer outra forma de assistência útil e FORMULAÇÃO DE POLÍTICAS E AVALIAÇÃO
prática para facilitar o processo de reabilitação. 30. A Pesquisa da base do planejamento e da
25. Mobilização de voluntários e outros serviços formulação e a avaliação de políticas
comunitários 30.1 Procurar-se-á organizar e fomentar as
25.1 Os voluntários, as organizações voluntárias, pesquisas necessárias como base do efetivo
as instituições locais e outros recursos da comunidade planejamento e formulação de políticas.
serão chamados a contribuir eficazmente para a

www.editoradince.com - Acesse e veja se há novidades a respeito deste material – CUIDADO: cópia é crime.
DIREITOS HUMANOS - Teoria esquematizada, dicas e questões de concursos 49
30.2 Procurar-se-á revisar e avaliar periodicamente B provar para a sociedade que a internação de
as tendências, os problemas e as causas da delinquência adolescentes não tem outra utilidade que não a de
e da criminalidade de jovens, assim como as diversas estigmatizar, desmoralizar, denegrir e embrutecer o
necessidades particulares do jovem sob custódia. indivíduo.
30.3 Procurar-se-á estabelecer regularmente um C garantir que o crime por eles pratricado seja aniquilado,
mecanismo de avaliação e pesquisa no sistema de negado, expiado pelo sofrimento da pena que, desse
administração da Justiça da Infância e da Juventude, e modo, restabeleceria o direito lesado.
coletar e analisar os dados e a informação pertinentes D aplicar-lhes um castigo, retribuindo com o mal. o mal
com vistas à devida avaliação e ao aperfeiçoamento do praticado, em uma ótica retribucionista.
sistema.
E assegurar-lhes assistência ,proteção, educação e
30.4 A prestação de serviços na administração da formação profissional, a fim de os ajudar a
Justiça da Infância e da Juventude será sistematicamente desempenhar um papel construtivo e produtivo na
planejada e executada como parte integrante dos sociedade.
esforços de desenvolvimento nacional. Tradução em
português de Maria Josefina Becker. Estas Regras foram
publicadas pela primeira vez, em português, pela 03. (FUNIVERSA - 2015 - Secretaria da Criança - DF -
FUNABEM em 1988. Técnico Socioeducativo - Administrativo)
Considerando as Regras de Beijing (regras mínimas
das Nações Unidas para a administração da justiça da
QUESTÕES DE CONCURSOS infância e da juventude), assinale a alternativa correta.
01. (FUNIVERSA - 2015 - Secretaria da Criança - DF - A Nos sistemas jurídicos que reconheçam o conceito
Técnico Socioeducativo - Administrativo) de responsabilidade penal para jovens, o começo des
Considerando as Regras de Beijing (regras mínimas sa responsabilidade penal não deverá se fixar em
das Nações Unidas para a administração da justiça da uma idade demasiadamente precoce, levando-se em
infância e da juventude), assinale a alternativa correta. conta, necessariamente, as circunstâncias que
acompanham a maturidade física ou biológica,
A Nos sistemas jurídicos que reconheçam o conceito
emocional, mental e intelectual.
de responsabilidade penal para jovens, o começo
dessa responsabilidade penal não deverá se fixar em B As disposições pertinentes das regras não só
uma idade demasiadamente precoce, levando-se em se aplicarão aos jovens infratores, mas também
conta, necessariamente, as circunstâncias que àqueles que possam ser processados por realizar
acompanham a maturidade física ou biológica, qualquer ato concreto que não seria punível se fosse
emocional, mental e intelectual. praticado por adultos.
B As disposições pertinentes das regras não só C As garantias processuais básicas, como a
se aplicarão aos jovens infratores, mas também presunção de inocência, o direito de ser informado
àqueles que possam ser processados por realizar das acusações, a assistência judiciária, a presença
qualquer ato concreto que não seria punível se fosse dos pais ou tutores, a confrontação com testemunhas
praticado por adultos. e o direito a interrogá-las, a publicidade do julgamento
e o direito de apelação ante uma autoridade
C As garantias processuais básicas, como a
superior, serão respeitadas em todas as etapas do
presunção de inocência, o direito de ser informado
processo.
das acusações, a assistência judiciária, a presença
dos pais ou tutores, a confrontação com testemunhas D Toda remissão que signifique encaminhar o jovem
e o direito a interrogá-las, a publicidade do julgamento a instituições da comunidade não dependerá
e o direito de apelação ante uma autoridade do consentimento dele ou de seu representante
superior, serão respeitadas em todas as etapas do legal, sendo a decisão relativa à remissão do caso
processo. submetida ao exame de uma autoridade competente.
D Toda remissão que signifique encaminhar o jovem E Para facilitar a adoção de uma decisão justa por
a instituições da comunidade não dependerá parte da autoridade competente, antes da decisão
do consentimento dele ou de seu representante definitiva, será efetuada uma investigação completa a
legal, sendo a decisão relativa à remissão do caso respeito do meio social, das circunstâncias de vida do
submetida ao exame de uma autoridade competente. jovem e das condições em que se deu a prática da
infração, independentemente da gravidade ou da
E Para facilitar a adoção de uma decisão justa por
natureza dessa infração.
parte da autoridade competente, antes da decisão
definitiva, será efetuada uma investigação completa a
respeito do meio social, das circunstâncias de vida do Gabarito: 01/B; 02/E; 03/B
jovem e das condições em que se deu a prática da
infração, independentemente da gravidade ou da
natureza dessa infração.
CONVENÇÕES DA ONU SOBRE OS
02. (IBADE - 2018 - SEJUDH - MT - Assistente do DIREITOS DA CRIANÇA.
Sistema Socioeducativo – Assistente Administrativo) o
De acordo com as Regras Mínimas das Nações DECRETO N 99.710, DE 21 DE NOVEMBRO DE 1990.
Unidas para a Administração da Justiça da Infância e Promulga a Convenção sobre os Direitos da
da Juventude - Regras de Beijing, a formação e o Criança.
tratamento dos menores colocados em instituição têm O PRESIDENTE DA REPÚBLICA, usando da
por objetivo: atribuição que lhe confere o art. 84, inciso IV, da
A ajudá-los a lidar com a colocação em família substituta. Constituição, e

www.editoradince.com - Acesse e veja se há novidades a respeito deste material – CUIDADO: cópia é crime.
50 D DIREITOS HUMANOS - Teoria esquematizada, dicas e questões de concurso
Considerando que o Congresso Nacional aprovou, Tendo em conta que a necessidade de
pelo Decreto Legislativo n° 28, de 14 de setembro de proporcionar à criança uma proteção especial foi
1990, a Convenção sobre os Direitos da Criança, a qual enunciada na Declaração de Genebra de 1924 sobre os
entrou em vigor internacional em 02 de setembro de 1990, Direitos da Criança e na Declaração dos Direitos da
na forma de seu artigo 49, inciso 1; Criança adotada pela Assembleia Geral em 20 de
Considerando que o Governo brasileiro ratificou a novembro de 1959, e reconhecida na Declaração
referida Convenção em 24 de setembro de 1990, tendo a Universal dos Direitos Humanos, no Pacto Internacional
mesmo entrado em vigor para o Brasil em 23 de outubro de Direitos Civis e Políticos (em particular nos Artigos 23
de 1990, na forma do seu artigo 49, incisos 2; e 24), no Pacto Internacional de Direitos Econômicos,
Sociais e Culturais (em particular no Artigo 10) e nos
DECRETA:
estatutos e instrumentos pertinentes das Agências
Art. 1° A Convenção sobre os Direitos da Criança, Especializadas e das organizações internacionais que se
apensa por cópia ao presente Decreto, será executada e interessam pelo bem-estar da criança;
cumprida tão inteiramente como nela se contém.
Tendo em conta que, conforme assinalado na
Art. 2° Este Decreto entra em vigor na data de sua Declaração dos Direitos da Criança, "a criança, em virtude
publicação. de sua falta de maturidade física e mental, necessita
Art. 3° Revogam-se as disposições em contrário. proteção e cuidados especiais, inclusive a devida
Brasília, 21 de novembro de 1990; 169° da Independência proteção legal, tanto antes quanto após seu nascimento";
e 102° da República. Lembrado o estabelecido na Declaração sobre os
FERNANDO COLLOR Princípios Sociais e Jurídicos Relativos à Proteção e ao
Francisco Rezek Bem-Estar das Crianças, especialmente com Referência à
Adoção e à Colocação em Lares de Adoção, nos Planos
CONVENÇÃO SOBRE OS DIREITOS DA Nacional e Internacional; as Regras Mínimas das Nações
Unidas para a Administração da Justiça Juvenil (Regras
CRIANÇA de Pequim); e a Declaração sobre a Proteção da Mulher e
Preâmbulo da Criança em Situações de Emergência ou de Conflito
Os Estados Partes da presente Convenção, Armado;
Considerando que, de acordo com os princípios Reconhecendo que em todos os países do mundo
proclamados na Carta das Nações Unidas, a liberdade, a existem crianças vivendo sob condições
justiça e a paz no mundo se fundamentam no excepcionalmente difíceis e que essas crianças
reconhecimento da dignidade inerente e dos direitos necessitam consideração especial;
iguais e inalienáveis de todos os membros da família Tomando em devida conta a importância das
humana; tradições e dos valores culturais de cada povo para a
Tendo em conta que os povos das Nações Unidas proteção e o desenvolvimento harmonioso da criança;
reafirmaram na carta sua fé nos direitos fundamentais do Reconhecendo a importância da cooperação
homem e na dignidade e no valor da pessoa humana e internacional para a melhoria das condições de vida das
que decidiram promover o progresso social e a elevação crianças em todos os países, especialmente nos países
do nível de vida com mais liberdade; em desenvolvimento;
Reconhecendo que as Nações Unidas Acordam o seguinte:
proclamaram e acordaram na Declaração Universal dos PARTE I
Direitos Humanos e nos Pactos Internacionais de Direitos
Humanos que toda pessoa possui todos os direitos e Artigo 1
liberdades neles enunciados, sem distinção de qualquer Para efeitos da presente Convenção considera-se
natureza, seja de raça, cor, sexo, idioma, crença, opinião como criança todo ser humano com menos de dezoito
política ou de outra índole, origem nacional ou social, anos de idade, a não ser que, em conformidade com a lei
posição econômica, nascimento ou qualquer outra aplicável à criança, a maioridade seja alcançada antes.
condição; Artigo 2
Recordando que na Declaração Universal dos 1. Os Estados Partes respeitarão os direitos
Direitos Humanos as Nações Unidas proclamaram que a enunciados na presente Convenção e assegurarão sua
infância tem direito a cuidados e assistência especiais; aplicação a cada criança sujeita à sua jurisdição, sem
Convencidos de que a família, como grupo distinção alguma, independentemente de raça, cor, sexo,
fundamental da sociedade e ambiente natural para o idioma, crença, opinião política ou de outra índole, origem
crescimento e bem-estar de todos os seus membros, e nacional, étnica ou social, posição econômica,
em particular das crianças, deve receber a proteção e deficiências físicas, nascimento ou qualquer outra
assistência necessárias a fim de poder assumir condição da criança, de seus pais ou de seus
plenamente suas responsabilidades dentro da representantes legais.
comunidade; 2. Os Estados Partes tomarão todas as medidas
Reconhecendo que a criança, para o pleno e apropriadas para assegurar a proteção da criança contra
harmonioso desenvolvimento de sua personalidade, deve toda forma de discriminação ou castigo por causa da
crescer no seio da família, em um ambiente de felicidade, condição, das atividades, das opiniões manifestadas ou
amor e compreensão; das crenças de seus pais, representantes legais ou
Considerando que a criança deve estar familiares.
plenamente preparada para uma vida independente na Artigo 3
sociedade e deve ser educada de acordo com os ideais 1. Todas as ações relativas às crianças, levadas a
proclamados nas Cartas das Nações Unidas, efeito por instituições públicas ou privadas de bem estar
especialmente com espírito de paz, dignidade, tolerância, social, tribunais, autoridades administrativas ou órgãos
liberdade, igualdade e solidariedade;

www.editoradince.com - Acesse e veja se há novidades a respeito deste material – CUIDADO: cópia é crime.
DIREITOS HUMANOS - Teoria esquematizada, dicas e questões de concursos 51
legislativos, devem considerar, primordialmente, o com a lei e os procedimentos legais cabíveis, que tal
interesse maior da criança. separação é necessária ao interesse maior da criança. Tal
2. Os Estados Partes se comprometem a determinação pode ser necessária em casos específicos,
assegurar à criança a proteção e o cuidado que sejam por exemplo, nos casos em que a criança sofre maus
necessários para seu bem-estar, levando em tratos ou descuido por parte de seus pais ou quando
consideração os direitos e deveres de seus pais, tutores estes vivem separados e uma decisão deve ser tomada a
ou outras pessoas responsáveis por ela perante a lei e, respeito do local da residência da criança.
com essa finalidade, tomarão todas as medidas 2. Caso seja adotado qualquer procedimento em
legislativas e administrativas adequadas. conformidade com o estipulado no parágrafo 1 do
3. Os Estados Partes se certificarão de que as presente artigo, todas as partes interessadas terão a
instituições, os serviços e os estabelecimentos oportunidade de participar e de manifestar suas opiniões.
encarregados do cuidado ou da proteção das crianças 3. Os Estados Partes respeitarão o direito da
cumpram com os padrões estabelecidos pelas criança que esteja separada de um ou de ambos os pais
autoridades competentes, especialmente no que diz de manter regularmente relações pessoais e contato
respeito à segurança e à saúde das crianças, ao número direto com ambos, a menos que isso seja contrário ao
e à competência de seu pessoal e à existência de interesse maior da criança.
supervisão adequada. 4. Quando essa separação ocorrer em virtude de
Artigo 4 uma medida adotada por um Estado Parte, tal como
Os Estados Partes adotarão todas as medidas detenção, prisão, exílio, deportação ou morte (inclusive
administrativas, legislativas e de outra índole com vistas à falecimento decorrente de qualquer causa enquanto a
implementação dos direitos reconhecidos na presente pessoa estiver sob a custódia do Estado) de um dos pais
Convenção. Com relação aos direitos econômicos, sociais da criança, ou de ambos, ou da própria criança, o Estado
e culturais, os Estados Partes adotarão essas medidas Parte, quando solicitado, proporcionará aos pais, à
utilizando ao máximo os recursos disponíveis e, quando criança ou, se for o caso, a outro familiar, informações
necessário, dentro de um quadro de cooperação básicas a respeito do paradeiro do familiar ou familiares
internacional. ausentes, a não ser que tal procedimento seja prejudicial
ao bem-estar da criança. Os Estados Partes se
Artigo 5
certificarão, além disso, de que a apresentação de tal
Os Estados Partes respeitarão as petição não acarrete, por si só, consequências adversas
responsabilidades, os direitos e os deveres dos pais ou, para a pessoa ou pessoas interessadas.
onde for o caso, dos membros da família ampliada ou da
Artigo 10
comunidade, conforme determinem os costumes locais,
dos tutores ou de outras pessoas legalmente 1. De acordo com a obrigação dos Estados Partes
responsáveis, de proporcionar à criança instrução e estipulada no parágrafo 1 do Artigo 9, toda solicitação
orientação adequadas e acordes com a evolução de sua apresentada por uma criança, ou por seus pais, para
capacidade no exercício dos direitos reconhecidos na ingressar ou sair de um Estado Parte com vistas à reunião
presente convenção. da família, deverá ser atendida pelos Estados Partes de
forma positiva, humanitária e rápida. Os Estados Partes
Artigo 6
assegurarão, ainda, que a apresentação de tal solicitação
1. Os Estados Partes reconhecem que toda não acarretará consequências adversas para os
criança tem o direito inerente à vida. solicitantes ou para seus familiares.
2. Os Estados Partes assegurarão ao máximo a 2. A criança cujos pais residam em Estados
sobrevivência e o desenvolvimento da criança. diferentes terá o direito de manter, periodicamente,
Artigo 7 relações pessoais e contato direto com ambos, exceto em
1. A criança será registrada imediatamente após circunstâncias especiais. Para tanto, e de acordo com a
seu nascimento e terá direito, desde o momento em que obrigação assumida pelos Estados Partes em virtude do
nasce, a um nome, a uma nacionalidade e, na medida do parágrafo 2 do Artigo 9, os Estados Partes respeitarão o
possível, a conhecer seus pais e a ser cuidada por eles. direito da criança e de seus pais de sair de qualquer país,
2. Os Estados Partes zelarão pela aplicação inclusive do próprio, e de ingressar no seu próprio país. O
desses direitos de acordo com sua legislação nacional e direito de sair de qualquer país estará sujeito, apenas, às
com as obrigações que tenham assumido em virtude dos restrições determinadas pela lei que sejam necessárias
instrumentos internacionais pertinentes, sobretudo se, de para proteger a segurança nacional, a ordem pública, a
outro modo, a criança se tornaria apátrida. saúde ou a moral públicas ou os direitos e as liberdades
de outras pessoas e que estejam acordes com os demais
Artigo 8 direitos reconhecidos pela presente convenção.
1. Os Estados Partes se comprometem a respeitar Artigo 11
o direito da criança de preservar sua identidade, inclusive
a nacionalidade, o nome e as relações familiares, de 1. Os Estados Partes adotarão medidas a fim de
acordo com a lei, sem interferências ilícitas. lutar contra a transferência ilegal de crianças para o
exterior e a retenção ilícita das mesmas fora do país.
2. Quando uma criança se vir privada ilegalmente
de algum ou de todos os elementos que configuram sua 2. Para tanto, aos Estados Partes promoverão a
identidade, os Estados Partes deverão prestar assistência conclusão de acordos bilaterais ou multilaterais ou a
e proteção adequadas com vistas a restabelecer adesão a acordos já existentes.
rapidamente sua identidade. Artigo 12
Artigo 9 1. Os Estados Partes assegurarão à criança que
1. Os Estados Partes deverão zelar para que a estiver capacitada a formular seus próprios juízos o direito
criança não seja separada dos pais contra a vontade dos de expressar suas opiniões livremente sobre todos os
mesmos, exceto quando, sujeita à revisão judicial, as assuntos relacionados com a criança, levando-se
autoridades competentes determinarem, em conformidade

www.editoradince.com - Acesse e veja se há novidades a respeito deste material – CUIDADO: cópia é crime.
52 D DIREITOS HUMANOS - Teoria esquematizada, dicas e questões de concurso
devidamente em consideração essas opiniões, em função a) incentivarão os meios de comunicação a difundir
da idade e maturidade da criança. informações e materiais de interesse social e cultural para
2. Com tal propósito, se proporcionará à criança, a criança, de acordo com o espírito do artigo 29;
em particular, a oportunidade de ser ouvida em todo b) promoverão a cooperação internacional na
processo judicial ou administrativo que afete a mesma, produção, no intercâmbio e na divulgação dessas
quer diretamente quer por intermédio de um informações e desses materiais procedentes de diversas
representante ou órgão apropriado, em conformidade com fontes culturais, nacionais e internacionais;
as regras processuais da legislação nacional. c) incentivarão a produção e difusão de livros para
Artigo 13 crianças;
1. A criança terá direito à liberdade de expressão. d) incentivarão os meios de comunicação no
Esse direito incluirá a liberdade de procurar, receber e sentido de, particularmente, considerar as necessidades
divulgar informações e ideias de todo tipo, linguísticas da criança que pertença a um grupo
independentemente de fronteiras, de forma oral, escrita minoritário ou que seja indígena;
ou impressa, por meio das artes ou por qualquer outro e) promoverão a elaboração de diretrizes
meio escolhido pela criança. apropriadas a fim de proteger a criança contra toda
2. O exercício de tal direito poderá estar sujeito a informação e material prejudiciais ao seu bem-estar,
determinadas restrições, que serão unicamente as tendo em conta as disposições dos artigos 13 e 18.
previstas pela lei e consideradas necessárias: Artigo 18
a) para o respeito dos direitos ou da reputação dos 1. Os Estados Partes envidarão os seus melhores
demais, ou esforços a fim de assegurar o reconhecimento do
b) para a proteção da segurança nacional ou da princípio de que ambos os pais têm obrigações comuns
ordem pública, ou para proteger a saúde e a moral com relação à educação e ao desenvolvimento da
públicas. criança. Caberá aos pais ou, quando for o caso, aos
Artigo 14 representantes legais, a responsabilidade primordial pela
educação e pelo desenvolvimento da criança. Sua
1. Os Estados Partes respeitarão o direito da
preocupação fundamental visará ao interesse maior da
criança à liberdade de pensamento, de consciência e de
criança.
crença.
2. A fim de garantir e promover os direitos
2. Os Estados Partes respeitarão os direitos e
enunciados na presente convenção, os Estados Partes
deveres dos pais e, se for o caso, dos representantes
prestarão assistência adequada aos pais e aos
legais, de orientar a criança com relação ao exercício de
representantes legais para o desempenho de suas
seus direitos de maneira acorde com a evolução de sua
funções no que tange à educação da criança e
capacidade.
assegurarão a criação de instituições, instalações e
3. A liberdade de professar a própria religião ou as serviços para o cuidado das crianças.
próprias crenças estará sujeita, unicamente, às limitações
3. Os Estados Partes adotarão todas as medidas
prescritas pela lei e necessárias para proteger a
apropriadas a fim de que as crianças cujos pais trabalhem
segurança, a ordem, a moral, a saúde pública ou os
tenham direito a beneficiar-se dos serviços de assistência
direitos e liberdades fundamentais dos demais.
social e creches a que fazem jus.
Artigo 15
Artigo 19
1 Os Estados Partes reconhecem os direitos da
1. Os Estados Partes adotarão todas as medidas
criança à liberdade de associação e à liberdade de
legislativas, administrativas, sociais e educacionais
realizar reuniões pacíficas.
apropriadas para proteger a criança contra todas as
2. Não serão impostas restrições ao exercício formas de violência física ou mental, abuso ou tratamento
desses direitos, a não ser as estabelecidas em negligente, maus tratos ou exploração, inclusive abuso
conformidade com a lei e que sejam necessárias numa sexual, enquanto a criança estiver sob a custódia dos
sociedade democrática, no interesse da segurança pais, do representante legal ou de qualquer outra pessoa
nacional ou pública, da ordem pública, da proteção à responsável por ela.
saúde e à moral públicas ou da proteção aos direitos e
2. Essas medidas de proteção deveriam incluir,
liberdades dos demais.
conforme apropriado, procedimentos eficazes para a
Artigo 16 elaboração de programas sociais capazes de
1. Nenhuma criança será objeto de interferências proporcionar uma assistência adequada à criança e às
arbitrárias ou ilegais em sua vida particular, sua família, pessoas encarregadas de seu cuidado, bem como para
seu domicílio ou sua correspondência, nem de atentados outras formas de prevenção, para a identificação,
ilegais a sua honra e a sua reputação. notificação, transferência a uma instituição, investigação,
2. A criança tem direito à proteção da lei contra tratamento e acompanhamento posterior dos casos acima
essas interferências ou atentados. mencionados de maus tratos à criança e, conforme o
Artigo 17 caso, para a intervenção judiciária.
Os Estados Partes reconhecem a função Artigo 20
importante desempenhada pelos meios de comunicação e 1. As crianças privadas temporária ou
zelarão para que a criança tenha acesso a informações e permanentemente do seu meio familiar, ou cujo interesse
materiais procedentes de diversas fontes nacionais e maior exija que não permaneçam nesse meio, terão
internacionais, especialmente informações e materiais direito à proteção e assistência especiais do Estado.
que visem a promover seu bem-estar social, espiritual e 2. Os Estados Partes garantirão, de acordo com
moral e sua saúde física e mental. Para tanto, os Estados suas leis nacionais, cuidados alternativos para essas
Partes: crianças.

www.editoradince.com - Acesse e veja se há novidades a respeito deste material – CUIDADO: cópia é crime.
DIREITOS HUMANOS - Teoria esquematizada, dicas e questões de concursos 53
3. Esses cuidados poderiam incluir, inter alia, a Artigo 23
colocação em lares de adoção, a kafalah do direito 1. Os Estados Partes reconhecem que a criança
islâmico, a adoção ou, caso necessário, a colocação em portadora de deficiências físicas ou mentais deverá
instituições adequadas de proteção para as crianças. Ao desfrutar de uma vida plena e decente em condições que
serem consideradas as soluções, deve-se dar especial garantam sua dignidade, favoreçam sua autonomia e
atenção à origem étnica, religiosa, cultural e linguística da facilitem sua participação ativa na comunidade.
criança, bem como à conveniência da continuidade de
2. Os Estados Partes reconhecem o direito da
sua educação.
criança deficiente de receber cuidados especiais e, de
Artigo 21 acordo com os recursos disponíveis e sempre que a
Os Estados Partes que reconhecem ou permitem o criança ou seus responsáveis reúnam as condições
sistema de adoção atentarão para o fato de que a requeridas, estimularão e assegurarão a prestação da
consideração primordial seja o interesse maior da criança. assistência solicitada, que seja adequada ao estado da
Dessa forma, atentarão para que: criança e às circunstâncias de seus pais ou das pessoas
a) a adoção da criança seja autorizada apenas encarregadas de seus cuidados.
pelas autoridades competentes, as quais determinarão, 3. Atendendo às necessidades especiais da
consoante as leis e os procedimentos cabíveis e com criança deficiente, a assistência prestada, conforme
base em todas as informações pertinentes e fidedignas, disposto no parágrafo 2 do presente artigo, será gratuita
que a adoção é admissível em vista da situação jurídica sempre que possível, levando-se em consideração a
da criança com relação a seus pais, parentes e situação econômica dos pais ou das pessoas que cuidem
representantes legais e que, caso solicitado, as pessoas da criança, e visará a assegurar à criança deficiente o
interessadas tenham dado, com conhecimento de causa, acesso efetivo à educação, à capacitação, aos serviços
seu consentimento à adoção, com base no de saúde, aos serviços de reabilitação, à preparação para
assessoramento que possa ser necessário; o emprego e às oportunidades de lazer, de maneira que a
b) a adoção efetuada em outro país possa ser criança atinja a mais completa integração social possível
considerada como outro meio de cuidar da criança, no e o maior desenvolvimento individual factível, inclusive
caso em que a mesma não possa ser colocada em um lar seu desenvolvimento cultural e espiritual.
de adoção ou entregue a uma família adotiva ou não logre 4. Os Estados Partes promoverão, com espírito de
atendimento adequado em seu país de origem; cooperação internacional, um intercâmbio adequado de
c) a criança adotada em outro país goze de informações nos campos da assistência médica
salvaguardas e normas equivalentes às existentes em seu preventiva e do tratamento médico, psicológico e
país de origem com relação à adoção; funcional das crianças deficientes, inclusive a divulgação
de informações a respeito dos métodos de reabilitação e
d) todas as medidas apropriadas sejam adotadas,
dos serviços de ensino e formação profissional, bem
a fim de garantir que, em caso de adoção em outro país, a
como o acesso a essa informação, a fim de que os
colocação não permita benefícios financeiros indevidos
Estados Partes possam aprimorar sua capacidade e seus
aos que dela participarem;
conhecimentos e ampliar sua experiência nesses campos.
e) quando necessário, promover os objetivos do Nesse sentido, serão levadas especialmente em conta as
presente artigo mediante ajustes ou acordos bilaterais ou necessidades dos países em desenvolvimento.
multilaterais, e envidarão esforços, nesse contexto, com
Artigo 24
vistas a assegurar que a colocação da criança em outro
país seja levada a cabo por intermédio das autoridades ou 1. Os Estados Partes reconhecem o direito da
organismos competentes. criança de gozar do melhor padrão possível de saúde e
dos serviços destinados ao tratamento das doenças e à
Artigo 22
recuperação da saúde. Os Estados Partes envidarão
1. Os Estados Partes adotarão medidas esforços no sentido de assegurar que nenhuma criança
pertinentes para assegurar que a criança que tente obter se veja privada de seu direito de usufruir desses serviços
a condição de refugiada, ou que seja considerada como sanitários.
refugiada de acordo com o direito e os procedimentos
2. Os Estados Partes garantirão a plena aplicação
internacionais ou internos aplicáveis, receba, tanto no
desse direito e, em especial, adotarão as medidas
caso de estar sozinha como acompanhada por seus pais
apropriadas com vistas a:
ou por qualquer outra pessoa, a proteção e a assistência
humanitária adequadas a fim de que possa usufruir dos a) reduzir a mortalidade infantil;
direitos enunciados na presente convenção e em outros b) assegurar a prestação de assistência médica e
instrumentos internacionais de direitos humanos ou de cuidados sanitários necessários a todas as crianças,
caráter humanitário dos quais os citados Estados sejam dando ênfase aos cuidados básicos de saúde;
parte. c) combater as doenças e a desnutrição dentro do
2. Para tanto, os Estados Partes cooperarão, da contexto dos cuidados básicos de saúde mediante, inter
maneira como julgarem apropriada, com todos os alia, a aplicação de tecnologia disponível e o fornecimento
esforços das Nações Unidas e demais organizações de alimentos nutritivos e de água potável, tendo em vista
intergovernamentais competentes, ou organizações não- os perigos e riscos da poluição ambiental;
governamentais que cooperem com as Nações Unidas, d) assegurar às mães adequada assistência pré-
no sentido de proteger e ajudar a criança refugiada, e de natal e pós-natal;
localizar seus pais ou outros membros de sua família a
e) assegurar que todos os setores da sociedade, e
fim de obter informações necessárias que permitam sua
em especial os pais e as crianças, conheçam os
reunião com a família. Quando não for possível localizar
princípios básicos de saúde e nutrição das crianças, as
nenhum dos pais ou membros da família, será concedida
vantagens da amamentação, da higiene e do saneamento
à criança a mesma proteção outorgada a qualquer outra
ambiental e das medidas de prevenção de acidentes, e
criança privada permanente ou temporariamente de seu
tenham acesso à educação pertinente e recebam apoio
ambiente familiar, seja qual for o motivo, conforme o
para a aplicação desses conhecimentos;
estabelecido na presente convenção.

www.editoradince.com - Acesse e veja se há novidades a respeito deste material – CUIDADO: cópia é crime.
54 D DIREITOS HUMANOS - Teoria esquematizada, dicas e questões de concurso
f) desenvolver a assistência médica preventiva, a b) estimular o desenvolvimento do ensino
orientação aos pais e a educação e serviços de secundário em suas diferentes formas, inclusive o ensino
planejamento familiar. geral e profissionalizante, tornando-o disponível e
3. Os Estados Partes adotarão todas as medidas acessível a todas as crianças, e adotar medidas
eficazes e adequadas para abolir práticas tradicionais que apropriadas tais como a implantação do ensino gratuito e
sejam prejudicais à saúde da criança. a concessão de assistência financeira em caso de
necessidade;
4. Os Estados Partes se comprometem a promover
e incentivar a cooperação internacional com vistas a c) tornar o ensino superior acessível a todos com
lograr, progressivamente, a plena efetivação do direito base na capacidade e por todos os meios adequados;
reconhecido no presente artigo. Nesse sentido, será dada d) tornar a informação e a orientação educacionais
atenção especial às necessidades dos países em e profissionais disponíveis e accessíveis a todas as
desenvolvimento. crianças;
Artigo 25 e) adotar medidas para estimular a frequência
Os Estados Partes reconhecem o direito de uma regular às escolas e a redução do índice de evasão
criança que tenha sido internada em um estabelecimento escolar.
pelas autoridades competentes para fins de atendimento, 2. Os Estados Partes adotarão todas as medidas
proteção ou tratamento de saúde física ou mental a um necessárias para assegurar que a disciplina escolar seja
exame periódico de avaliação do tratamento ao qual está ministrada de maneira compatível com a dignidade
sendo submetida e de todos os demais aspectos relativos humana da criança e em conformidade com a presente
à sua internação. convenção.
Artigo 26 3. Os Estados Partes promoverão e estimularão a
1. Os Estados Partes reconhecerão a todas as cooperação internacional em questões relativas à
crianças o direito de usufruir da previdência social, educação, especialmente visando a contribuir para a
inclusive do seguro social, e adotarão as medidas eliminação da ignorância e do analfabetismo no mundo e
necessárias para lograr a plena consecução desse direito, facilitar o acesso aos conhecimentos científicos e técnicos
em conformidade com sua legislação nacional. e aos métodos modernos de ensino. A esse respeito, será
dada atenção especial às necessidades dos países em
2. Os benefícios deverão ser concedidos, quando
desenvolvimento.
pertinentes, levando-se em consideração os recursos e a
situação da criança e das pessoas responsáveis pelo seu Artigo 29
sustento, bem como qualquer outra consideração cabível 1. Os Estados Partes reconhecem que a educação
no caso de uma solicitação de benefícios feita pela da criança deverá estar orientada no sentido de:
criança ou em seu nome. a) desenvolver a personalidade, as aptidões e a
Artigo 27 capacidade mental e física da criança em todo o seu
1. Os Estados Partes reconhecem o direito de toda potencial;
criança a um nível de vida adequado ao seu b) imbuir na criança o respeito aos direitos
desenvolvimento físico, mental, espiritual, moral e social. humanos e às liberdades fundamentais, bem como aos
2. Cabe aos pais, ou a outras pessoas princípios consagrados na Carta das Nações Unidas;
encarregadas, a responsabilidade primordial de propiciar, c) imbuir na criança o respeito aos seus pais, à sua
de acordo com suas possibilidades e meios financeiros, própria identidade cultural, ao seu idioma e seus valores,
as condições de vida necessárias ao desenvolvimento da aos valores nacionais do país em que reside, aos do
criança. eventual país de origem, e aos das civilizações diferentes
3. Os Estados Partes, de acordo com as condições da sua;
nacionais e dentro de suas possibilidades, adotarão d) preparar a criança para assumir uma vida
medidas apropriadas a fim de ajudar os pais e outras responsável numa sociedade livre, com espírito de
pessoas responsáveis pela criança a tornar efetivo esse compreensão, paz, tolerância, igualdade de sexos e
direito e, caso necessário, proporcionarão assistência amizade entre todos os povos, grupos étnicos, nacionais
material e programas de apoio, especialmente no que diz e religiosos e pessoas de origem indígena;
respeito à nutrição, ao vestuário e à habitação. e) imbuir na criança o respeito ao meio ambiente.
4. Os Estados Partes tomarão todas as medidas 2. Nada do disposto no presente artigo ou no
adequadas para assegurar o pagamento da pensão Artigo 28 será interpretado de modo a restringir a
alimentícia por parte dos pais ou de outras pessoas liberdade dos indivíduos ou das entidades de criar e dirigir
financeiramente responsáveis pela criança, quer residam instituições de ensino, desde que sejam respeitados os
no Estado Parte quer no exterior. Nesse sentido, quando princípios enunciados no parágrafo 1 do presente artigo e
a pessoa que detém a responsabilidade financeira pela que a educação ministrada em tais instituições esteja
criança residir em Estado diferente daquele onde mora a acorde com os padrões mínimos estabelecidos pelo
criança, os Estados Partes promoverão a adesão a Estado.
acordos internacionais ou a conclusão de tais acordos,
Artigo 30
bem como a adoção de outras medidas apropriadas.
Nos Estados Partes onde existam minorias
Artigo 28
étnicas, religiosas ou linguísticas, ou pessoas de origem
1. Os Estados Partes reconhecem o direito da indígena, não será negado a uma criança que pertença a
criança à educação e, a fim de que ela possa exercer tais minorias ou que seja indígena o direito de, em
progressivamente e em igualdade de condições esse comunidade com os demais membros de seu grupo, ter
direito, deverão especialmente: sua própria cultura, professar e praticar sua própria
a) tornar o ensino primário obrigatório e disponível religião ou utilizar seu próprio idioma.
gratuitamente para todos; Artigo 31

www.editoradince.com - Acesse e veja se há novidades a respeito deste material – CUIDADO: cópia é crime.
DIREITOS HUMANOS - Teoria esquematizada, dicas e questões de concursos 55
1. Os Estados Partes reconhecem o direito da prisão perpétua sem possibilidade de livramento por
criança ao descanso e ao lazer, ao divertimento e às delitos cometidos por menores de dezoito anos de idade;
atividades recreativas próprias da idade, bem como à livre b) nenhuma criança seja privada de sua liberdade
participação na vida cultural e artística. de forma ilegal ou arbitrária. A detenção, a reclusão ou a
2. Os Estados Partes respeitarão e promoverão o prisão de uma criança será efetuada em conformidade
direito da criança de participar plenamente da vida cultural com a lei e apenas como último recurso, e durante o mais
e artística e encorajarão a criação de oportunidades breve período de tempo que for apropriado;
adequadas, em condições de igualdade, para que c) toda criança privada da liberdade seja tratada
participem da vida cultural, artística, recreativa e de lazer. com a humanidade e o respeito que merece a dignidade
Artigo 32 inerente à pessoa humana, e levando-se em
1. Os Estados Partes reconhecem o direito da consideração as necessidades de uma pessoa de sua
criança de estar protegida contra a exploração econômica idade. Em especial, toda criança privada de sua liberdade
e contra o desempenho de qualquer trabalho que possa ficará separada dos adultos, a não ser que tal fato seja
ser perigoso ou interferir em sua educação, ou que seja considerado contrário aos melhores interesses da criança,
nocivo para sua saúde ou para seu desenvolvimento e terá direito a manter contato com sua família por meio
físico, mental, espiritual, moral ou social. de correspondência ou de visitas, salvo em circunstâncias
excepcionais;
2. Os Estados Partes adotarão medidas
legislativas, administrativas, sociais e educacionais com d) toda criança privada de sua liberdade tenha
vistas a assegurar a aplicação do presente artigo. Com tal direito a rápido acesso a assistência jurídica e a qualquer
propósito, e levando em consideração as disposições outra assistência adequada, bem como direito a impugnar
pertinentes de outros instrumentos internacionais, os a legalidade da privação de sua liberdade perante um
Estados Partes, deverão, em particular: tribunal ou outra autoridade competente, independente e
imparcial e a uma rápida decisão a respeito de tal ação.
a) estabelecer uma idade ou idades mínimas para
a admissão em empregos; Artigo 38
b) estabelecer regulamentação apropriada relativa 1. Os Estados Partes se comprometem a respeitar
a horários e condições de emprego; e a fazer com que sejam respeitadas as normas do direito
humanitário internacional aplicáveis em casos de conflito
c) estabelecer penalidades ou outras sanções
armado no que digam respeito às crianças.
apropriadas a fim de assegurar o cumprimento efetivo do
presente artigo. 2. Os Estados Partes adotarão todas as medidas
possíveis a fim de assegurar que todas as pessoas que
Artigo 33
ainda não tenham completado quinze anos de idade não
Os Estados Partes adotarão todas as medidas participem diretamente de hostilidades.
apropriadas, inclusive medidas legislativas,
3. Os Estados Partes abster-se-ão de recrutar
administrativas, sociais e educacionais, para proteger a
pessoas que não tenham completado quinze anos de
criança contra o uso ilícito de drogas e substâncias
idade para servir em suas forças armadas. Caso recrutem
psicotrópicas descritas nos tratados internacionais
pessoas que tenham completado quinze anos mas que
pertinentes e para impedir que crianças sejam utilizadas
tenham menos de dezoito anos, deverão procurar dar
na produção e no tráfico ilícito dessas substâncias.
prioridade aos de mais idade.
Artigo 34
4. Em conformidade com suas obrigações de
Os Estados Partes se comprometem a proteger a acordo com o direito humanitário internacional para
criança contra todas as formas de exploração e abuso proteção da população civil durante os conflitos armados,
sexual. Nesse sentido, os Estados Partes tomarão, em os Estados Partes adotarão todas as medidas
especial, todas as medidas de caráter nacional, bilateral e necessárias a fim de assegurar a proteção e o cuidado
multilateral que sejam necessárias para impedir: das crianças afetadas por um conflito armado.
a) o incentivo ou a coação para que uma criança Artigo 39
se dedique a qualquer atividade sexual ilegal;
Os Estados Partes adotarão todas as medidas
b) a exploração da criança na prostituição ou apropriadas para estimular a recuperação física e
outras práticas sexuais ilegais; psicológica e a reintegração social de toda criança vítima
c) a exploração da criança em espetáculos ou de qualquer forma de abandono, exploração ou abuso;
materiais pornográficos. tortura ou outros tratamentos ou penas cruéis, desumanos
Artigo 35 ou degradantes; ou conflitos armados. Essa recuperação
Os Estados Partes tomarão todas as medidas de e reintegração serão efetuadas em ambiente que estimule
caráter nacional, bilateral e multilateral que sejam a saúde, o respeito próprio e a dignidade da criança.
necessárias para impedir o sequestro, a venda ou o Artigo 40
tráfico de crianças para qualquer fim ou sob qualquer 1. Os Estados Partes reconhecem o direito de toda
forma. criança a quem se alegue ter infringido as leis penais ou a
Artigo 36 quem se acuse ou declare culpada de ter infringido as leis
Os Estados Partes protegerão a criança contra penais de ser tratada de modo a promover e estimular seu
todas as demais formas de exploração que sejam sentido de dignidade e de valor e a fortalecer o respeito
prejudiciais para qualquer aspecto de seu bem-estar. da criança pelos direitos humanos e pelas liberdades
fundamentais de terceiros, levando em consideração a
Artigo 37 idade da criança e a importância de se estimular sua
Os Estados Partes zelarão para que: reintegração e seu desempenho construtivo na sociedade.
a) nenhuma criança seja submetida a tortura nem 2. Nesse sentido, e de acordo com as disposições
a outros tratamentos ou penas cruéis, desumanos ou pertinentes dos instrumentos internacionais, os Estados
degradantes. Não será imposta a pena de morte nem a Partes assegurarão, em particular:

www.editoradince.com - Acesse e veja se há novidades a respeito deste material – CUIDADO: cópia é crime.
56 D DIREITOS HUMANOS - Teoria esquematizada, dicas e questões de concurso
a) que não se alegue que nenhuma criança tenha a) das leis de um Estado Parte;
infringido as leis penais, nem se acuse ou declare culpada b) das normas de direito internacional vigentes
nenhuma criança de ter infringido essas leis, por atos ou para esse Estado.
omissões que não eram proibidos pela legislação nacional
PARTE II
ou pelo direito internacional no momento em que foram
cometidos; Artigo 42
b) que toda criança de quem se alegue ter Os Estados Partes se comprometem a dar aos
infringido as leis penais ou a quem se acuse de ter adultos e às crianças amplo conhecimento dos princípios
infringido essas leis goze, pelo menos, das seguintes e disposições da convenção, mediante a utilização de
garantias: meios apropriados e eficazes.
I) ser considerada inocente enquanto não for Artigo 43
comprovada sua culpabilidade conforme a lei; 1. A fim de examinar os progressos realizados no
II) ser informada sem demora e diretamente ou, cumprimento das obrigações contraídas pelos Estados
quando for o caso, por intermédio de seus pais ou de Partes na presente convenção, deverá ser estabelecido
seus representantes legais, das acusações que pesam um Comitê para os Direitos da Criança que
contra ela, e dispor de assistência jurídica ou outro tipo de desempenhará as funções a seguir determinadas.
assistência apropriada para a preparação e apresentação 2. O comitê estará integrado por dez especialistas
de sua defesa; de reconhecida integridade moral e competência nas
III) ter a causa decidida sem demora por áreas cobertas pela presente convenção. Os membros do
autoridade ou órgão judicial competente, independente e comitê serão eleitos pelos Estados Partes dentre seus
imparcial, em audiência justa conforme a lei, com nacionais e exercerão suas funções a título pessoal,
assistência jurídica ou outra assistência e, a não ser que tomando-se em devida conta a distribuição geográfica
seja considerado contrário aos melhores interesses da equitativa bem como os principais sistemas jurídicos.
criança, levando em consideração especialmente sua 3. Os membros do comitê serão escolhidos, em
idade ou situação e a de seus pais ou representantes votação secreta, de uma lista de pessoas indicadas pelos
legais; Estados Partes. Cada Estado Parte poderá indicar uma
IV) não ser obrigada a testemunhar ou a se pessoa dentre os cidadãos de seu país.
declarar culpada, e poder interrogar ou fazer com que 4. A eleição inicial para o comitê será realizada, no
sejam interrogadas as testemunhas de acusação bem mais tardar, seis meses após a entrada em vigor da
como poder obter a participação e o interrogatório de presente convenção e, posteriormente, a cada dois anos.
testemunhas em sua defesa, em igualdade de condições; No mínimo quatro meses antes da data marcada para
V) se for decidido que infringiu as leis penais, ter cada eleição, o Secretário-Geral das Nações Unidas
essa decisão e qualquer medida imposta em decorrência enviará uma carta aos Estados Partes convidando-os a
da mesma submetidas a revisão por autoridade ou órgão apresentar suas candidaturas num prazo de dois meses.
judicial superior competente, independente e imparcial, de O Secretário-Geral elaborará posteriormente uma lista da
acordo com a lei; qual farão parte, em ordem alfabética, todos os
candidatos indicados e os Estados Partes que os
VI) contar com a assistência gratuita de um
designaram, e submeterá a mesma aos Estados Partes
intérprete caso a criança não compreenda ou fale o
presentes à Convenção.
idioma utilizado;
5. As eleições serão realizadas em reuniões dos
VII) ter plenamente respeitada sua vida privada
Estados Partes convocadas pelo Secretário-Geral na
durante todas as fases do processo.
Sede das Nações Unidas. Nessas reuniões, para as quais
3. Os Estados Partes buscarão promover o o quorum será de dois terços dos Estados Partes, os
estabelecimento de leis, procedimentos, autoridades e candidatos eleitos para o comitê serão aqueles que
instituições específicas para as crianças de quem se obtiverem o maior número de votos e a maioria absoluta
alegue ter infringido as leis penais ou que sejam acusadas de votos dos representantes dos Estados Partes
ou declaradas culpadas de tê-las infringido, e em presentes e votantes.
particular:
6. Os membros do comitê serão eleitos para um
a) o estabelecimento de uma idade mínima antes mandato de quatro anos. Poderão ser reeleitos caso
da qual se presumirá que a criança não tem capacidade sejam apresentadas novamente suas candidaturas. O
para infringir as leis penais; mandato de cinco dos membros eleitos na primeira
b) a adoção sempre que conveniente e desejável, eleição expirará ao término de dois anos; imediatamente
de medidas para tratar dessas crianças sem recorrer a após ter sido realizada a primeira eleição, o presidente da
procedimentos judiciais, contando que sejam respeitados reunião na qual a mesma se efetuou escolherá por sorteio
plenamente os direitos humanos e as garantias legais. os nomes desses cinco membros.
4. Diversas medidas, tais como ordens de guarda, 7. Caso um membro do comitê venha a falecer ou
orientação e supervisão, aconselhamento, liberdade renuncie ou declare que por qualquer outro motivo não
vigiada, colocação em lares de adoção, programas de poderá continuar desempenhando suas funções, o Estado
educação e formação profissional, bem como outras Parte que indicou esse membro designará outro
alternativas à internação em instituições, deverão estar especialista, dentre seus cidadãos, para que exerça o
disponíveis para garantir que as crianças sejam tratadas mandato até seu término, sujeito à aprovação do comitê.
de modo apropriado ao seu bem-estar e de forma 8. O comitê estabelecerá suas próprias regras de
proporcional às circunstâncias e ao tipo do delito. procedimento.
Artigo 41 9. O comitê elegerá a mesa para um período de
Nada do estipulado na presente Convenção dois anos.
afetará disposições que sejam mais convenientes para a 10. As reuniões do comitê serão celebradas
realização dos direitos da criança e que podem constar: normalmente na sede das Nações Unidas ou em qualquer

www.editoradince.com - Acesse e veja se há novidades a respeito deste material – CUIDADO: cópia é crime.
DIREITOS HUMANOS - Teoria esquematizada, dicas e questões de concursos 57
outro lugar que o comitê julgar conveniente. O comitê se b) conforme julgar conveniente, o comitê
reunirá normalmente todos os anos. A duração das transmitirá às agências especializadas, ao Fundo das
reuniões do comitê será determinada e revista, se for o Nações Unidas para a Infância e a outros órgãos
caso, em uma reunião dos Estados Partes da presente competentes quaisquer relatórios dos Estados Partes que
convenção, sujeita à aprovação da Assembleia Geral. contenham um pedido de assessoramento ou de
11. O Secretário-Geral das Nações Unidas assistência técnica, ou nos quais se indique essa
fornecerá o pessoal e os serviços necessários para o necessidade, juntamente com as observações e
desempenho eficaz das funções do comitê de acordo com sugestões do comitê, se as houver, sobre esses pedidos
a presente convenção. ou indicações;
12. Com prévia aprovação da Assembleia Geral, c) comitê poderá recomendar à Assembleia Geral
os membros do Comitê estabelecido de acordo com a que solicite ao Secretário-Geral que efetue, em seu nome,
presente convenção receberão emolumentos estudos sobre questões concretas relativas aos direitos
provenientes dos recursos das Nações Unidas, segundo da criança;
os termos e condições determinados pela assembleia. d) o comitê poderá formular sugestões e
Artigo 44 recomendações gerais com base nas informações
recebidas nos termos dos Artigos 44 e 45 da presente
1. Os Estados Partes se comprometem a
convenção. Essas sugestões e recomendações gerais
apresentar ao comitê, por intermédio do Secretário-Geral
deverão ser transmitidas aos Estados Partes e
das Nações Unidas, relatórios sobre as medidas que
encaminhadas à Assembleia geral, juntamente com os
tenham adotado com vistas a tornar efetivos os direitos
comentários eventualmente apresentados pelos Estados
reconhecidos na convenção e sobre os progressos
Partes.
alcançados no desempenho desses direitos:
PARTE III
a) num prazo de dois anos a partir da data em que
entrou em vigor para cada Estado Parte a presente Artigo 46
convenção; A presente convenção está aberta à assinatura de
b) a partir de então, a cada cinco anos. todos os Estados.
2. Os relatórios preparados em função do presente Artigo 47
artigo deverão indicar as circunstâncias e as dificuldades, A presente convenção está sujeita à ratificação. Os
caso existam, que afetam o grau de cumprimento das instrumentos de ratificação serão depositados junto ao
obrigações derivadas da presente convenção. Deverão, Secretário-Geral das Nações Unidas.
também, conter informações suficientes para que o comitê Artigo 48
compreenda, com exatidão, a implementação da
A presente convenção permanecerá aberta à
convenção no país em questão.
adesão de qualquer Estado. Os instrumentos de adesão
3. Um Estado Parte que tenha apresentado um serão depositados junto ao Secretário-Geral das Nações
relatório inicial ao comitê não precisará repetir, nos Unidas.
relatórios posteriores a serem apresentados conforme o
Artigo 49
estipulado no sub-item b) do parágrafo 1 do presente
artigo, a informação básica fornecida anteriormente. 1. A presente convenção entrará em vigor no
trigésimo dia após a data em que tenha sido depositado o
4. O comitê poderá solicitar aos Estados Partes
vigésimo instrumento de ratificação ou de adesão junto ao
maiores informações sobre a implementação da
Secretário-Geral das Nações Unidas.
convenção.
2. Para cada Estado que venha a ratificar a
5. A cada dois anos, o comitê submeterá relatórios
convenção ou a aderir a ela após ter sido depositado o
sobre suas atividades à Assembleia Geral das Nações
vigésimo instrumento de ratificação ou de adesão, a
Unidas, por intermédio do Conselho Econômico e Social.
convenção entrará em vigor no trigésimo dia após o
6. Os Estados Partes tornarão seus relatórios depósito, por parte do Estado, de seu instrumento de
amplamente disponíveis ao público em seus respectivos ratificação ou de adesão.
países.
Artigo 50
Artigo 45
1. Qualquer Estado Parte poderá propor uma
A fim de incentivar a efetiva implementação da emenda e registrá-la com o Secretário-Geral das Nações
Convenção e estimular a cooperação internacional nas Unidas. O Secretário-Geral comunicará a emenda
esferas regulamentadas pela convenção: proposta aos Estados Partes, com a solicitação de que
a) os organismos especializados, o Fundo das estes o notifiquem caso apoiem a convocação de uma
Nações Unidas para a Infância e outros órgãos das Conferência de Estados Partes com o propósito de
Nações Unidas terão o direito de estar representados analisar as propostas e submetê-las à votação. Se, num
quando for analisada a implementação das disposições prazo de quatro meses a partir da data dessa notificação,
da presente convenção que estejam compreendidas no pelo menos um terço dos Estados Partes se declarar
âmbito de seus mandatos. O comitê poderá convidar as favorável a tal Conferência, o Secretário-Geral convocará
agências especializadas, o Fundo das Nações Unidas conferência, sob os auspícios das Nações Unidas.
para a Infância e outros órgãos competentes que Qualquer emenda adotada pela maioria de Estados
considere apropriados a fornecer assessoramento Partes presentes e votantes na conferência será
especializado sobre a implementação da Convenção em submetida pelo Secretário-Geral à Assembleia Geral para
matérias correspondentes a seus respectivos mandatos. sua aprovação.
O comitê poderá convidar as agências especializadas, o 2. Uma emenda adotada em conformidade com o
Fundo das Nações Unidas para Infância e outros órgãos parágrafo 1 do presente artigo entrará em vigor quando
das Nações Unidas a apresentarem relatórios sobre a aprovada pela Assembleia Geral das Nações Unidas e
implementação das disposições da presente convenção aceita por uma maioria de dois terços de Estados Partes.
compreendidas no âmbito de suas atividades;

www.editoradince.com - Acesse e veja se há novidades a respeito deste material – CUIDADO: cópia é crime.
58 D DIREITOS HUMANOS - Teoria esquematizada, dicas e questões de concurso
3. Quando uma emenda entrar em vigor, ela será Princípio 4
obrigatória para os Estados Partes que as tenham aceito, A criança gozará os benefícios da previdência
enquanto os demais Estados Partes permanecerão social. Terá direito a crescer e criar-se com saúde; para
obrigados pelas disposições da presente convenção e isto, tanto à criança como à mãe, serão proporcionados
pelas emendas anteriormente aceitas por eles. cuidados e proteções especiais, inclusive adequados
Artigo 51 cuidados pré e pós-natais. A criança terá direito a
1. O Secretário-Geral das Nações Unidas receberá alimentação, recreação e assistência médica adequadas.
e comunicará a todos os Estados Partes o texto das Princípio 5
reservas feitas pelos Estados no momento da ratificação À crianças incapacitadas física, mental ou
ou da adesão. socialmente serão proporcionados o tratamento, a
2. Não será permitida nenhuma reserva educação e os cuidados especiais exigidos pela sua
incompatível com o objetivo e o propósito da presente condição peculiar.
convenção. Princípio 6
3. Quaisquer reservas poderão ser retiradas a Para o desenvolvimento completo e harmonioso de
qualquer momento mediante uma notificação nesse sua personalidade, a criança precisa de amor e
sentido dirigida ao Secretário-Geral das Nações Unidas, compreensão. Criar-se-á, sempre que possível, aos
que informará a todos os Estados. Essa notificação cuidados e sob a responsabilidade dos pais e, em
entrará em vigor a partir da data de recebimento da qualquer hipótese, num ambiente de afeto e de segurança
mesma pelo Secretário-Geral. moral e material, salvo circunstâncias excepcionais, a
Artigo 52 criança da tenra idade não será apartada da mãe. À
Um Estado Parte poderá denunciar a presente sociedade e às autoridades públicas caberá a obrigação
convenção mediante notificação feita por escrito ao de propiciar cuidados especiais às crianças sem família e
Secretário-Geral das Nações Unidas. A denúncia entrará àquelas que carecem de meios adequados de
em vigor um ano após a data em que a notificação tenha subsistência. É desejável a prestação de ajuda oficial e de
sido recebida pelo Secretário-Geral. outra natureza em prol da manutenção dos filhos de
famílias numerosas.
Artigo 53
Princípio 7
Designa-se para depositário da presente
convenção o Secretário-Geral das Nações Unidas. A criança terá direito a receber educação, que será
gratuita e compulsória pelo menos no grau primário. Ser-
Artigo 54
lhe-á propiciada uma educação capaz de promover a sua
O original da presente convenção, cujos textos em cultura geral e capacitá-la a, em condições de iguais
árabe chinês, espanhol, francês, inglês e russo são oportunidades, desenvolver as suas aptidões, sua
igualmente autênticos, será depositado em poder do capacidade de emitir juízo e seu senso de
Secretário-Geral das Nações Unidas. responsabilidade moral e social, e a tornar-se um membro
Em fé do que, os plenipotenciários abaixo útil da sociedade.
assinados, devidamente autorizados por seus respectivos Os melhores interesses da criança serão a diretriz
Governos, assinaram a presente Convenção. a nortear os responsáveis pela sua educação e
orientação; esta responsabilidade cabe, em primeiro
DECLARAÇÃO DOS DIREITOS DA CRIANÇA – lugar, aos pais.
1959 PRINCÍPIO A criança terá ampla oportunidade para brincar e
divertir-se, visando os propósitos mesmos da sua
Adotada pela Assembléia das Nações Unidas de 20 de educação; a sociedade e as autoridades públicas
novembro de 1959 e ratificada pelo Brasil; através do art. empenhar-se-ão em promover o gozo deste direito.
84, inciso XXI, da Constituição, e tendo em vista o
disposto nos arts. 1º da Lei nº 91, de 28 de agosto de Princípio 8
1935, e 1º do Decreto nº 50.517, de 2 de maio de 1961. A criança figurará, em quaisquer circunstâncias,
(...) entre os primeiros a receber proteção e socorro.
Princípio 1 Princípio 9
A criança gozará todos os direitos enunciados A criança gozará de proteção contra quaisquer
nesta Declaração. Todas as crianças, absolutamente sem formas de negligência, crueldade e exploração. Não será
qualquer exceção, serão credoras destes direitos, sem jamais objeto de tráfico, sob qualquer forma.
distinção ou discriminação por motivo de raça, cor, sexo, Não será permitido à criança empregar-se antes
língua, religião, opinião política ou de outra natureza, da idade mínima conveniente; de nenhuma forma será
origem nacional ou social, riqueza, nascimento ou levada a ou ser-lhe-á permitido empenhar-se em qualquer
qualquer outra condição, quer sua ou de sua família. ocupação ou emprego que lhe prejudique a saúde ou a
Princípio 2 educação ou que interfira em seu desenvolvimento físico,
mental ou moral.
A criança gozará proteção social e ser-lhe-ão
proporcionadas oportunidades e facilidades, por lei e por Princípio 10
outros meios, a fim de lhe facultar o desenvolvimento A criança gozará de proteção contra atos que
físico, mental, moral, espiritual e social, de forma sadia e possam suscitar discriminação racial, religiosa ou de
normal e em condições de liberdade e dignidade. Na qualquer outra natureza. Criar-se-á num ambiente de
instituição das leis visando este objetivo levar-se-ão em compreensão, de tolerância, de amizade entre os povos,
conta sobretudo, os melhores interesses da criança. de paz e de fraternidade universal e em plena consciência
Princípio 3 que seu esforço e aptidão devem ser postos a serviço de
seus semelhantes.
Desde o nascimento, toda criança terá direito a um
nome e a uma nacionalidade.

www.editoradince.com - Acesse e veja se há novidades a respeito deste material – CUIDADO: cópia é crime.
DIREITOS HUMANOS - Teoria esquematizada, dicas e questões de concursos 59
QUESTÕES DE CONCURSOS B Essa convenção consagra que decisões acerca do
destino da criança considerarão o princípio do
01. (FCC - 2020 - TJ-MS - Juiz Substituto) Nos termos interesse maior da criança no que diz respeito à
expressos da Convenção Internacional sobre os guarda e a outros temas relativos à família e ao poder
Direitos da Criança, os Estados Partes buscarão familiar.
promover o estabelecimento de uma idade
C Essa convenção prevê que crianças somente poderão
A mínima antes da qual se presumirá que a criança não participar de reuniões públicas mediante autorização
tem capacidade para infringir as leis penais. do órgão estatal responsável pela proteção da criança.
B acima da qual não se imporá qualquer medida de D Essa convenção exclui qualquer tipo de intervenção
cuidados familiares alternativos sem o expresso judiciária na hipótese de ocorrer violência ou maus-
consentimento da criança. tratos contra criança que esteja sob a custódia dos
C abaixo da qual não se exigirá consentimento da criança seus pais ou de seu representante legal, restringindo-
para que receba tratamento médico, psicológico ou se as medidas de proteção adotadas pelo Estado a
funcional visando a promoção de sua saúde física e medidas sociais e educacionais apropriadas.
mental. E Essa convenção considera que a criança é um indivíduo
D mínima para que o exercício dos direitos sexuais e ainda sem capacidade de formular seus próprios
reprodutivos da criança não sejam considerados juízos ou expressar suas opiniões, razão pela qual
infração à lei penal vigente no Estado. não deve, em regra, ser ouvida em processos judiciais
E antes da qual os pais e outras pessoas responsáveis ou administrativos que a afetem.
pela criança não poderão, por ato de disposição de
vontade, antecipar a maioridade civil da criança. Gabarito: 01/A; 02/A; 03/D; 04/B

02. (FCC - 2021 - DPE-BA - Defensor (A) Público (A))


Segundo o art. 5° da Convenção Internacional sobre
QUESTÕES DE CONCURSOS DE PROVAS
os Direitos da Criança, os Estados Partes respeitarão PASSADAS
as responsabilidades, os direitos e os deveres dos SJC/SC AS AGENTE DE SEGURANÇA
pais ou, onde for o caso, dos membros da família SOCIOEDUCATIVO/2015
ampliada ou da comunidade, conforme determinem os 01. O sistema internacional de proteção dos direitos
costumes locais, dos tutores ou de outras pessoas
humanos pode apresentar diferentes âmbitos de
legalmente responsáveis, de proporcionar à criança
aplicação. Daí falar nos sistemas global e regional de
instrução e orientação adequadas e acordes com a
proteção aos direitos humanos. O sistema global é o
evolução de sua capacidade no exercício dos direitos
sistema da .....(1)..... . junto com o sistema global,
reconhecidos na presente convenção. Tal dispositivo
surgem os sistemas regionais de proteção que
consagra o princípio da
buscam internacionalizar os direitos humanos no
A autoridade parental participativa. plano regional. No plano regional o Brasil faz parte da
B matricialidade familiar. .....(2)..... . Assinale a alternativa que completa
C formação cidadã. corretamente as lacunas numeradas do texto.
D responsabilidade parental ampliada. a. ( ) (1) Organização dos Estados Americanos;
E autonomia progressiva. (2) Organização das Nações Unidas.
b. ( ) (1) Organização das Nações Unidas;
03. (VUNESP - 2019 - Prefeitura de Valinhos - SP - (2) Organização dos Estados Americanos.
Educador Social – SAS) O Fundo das Nações Unidas c. ( ) (1) Organização das Nações Unidas;
para a Infância (UNICEF) referenda: as crianças e os (2) União Europeia.
adolescentes têm todos os direitos humanos, não
d. ( ) (1) Organização dos Estados Americanos;
porque são o futuro, mas porque são seres humanos,
hoje. Dentre os princípios expressos na Declaração (2) União Europeia.
Universal dos Direitos das Crianças, o princípio 5 e. ( ) (1) União Europeia;
prevê que às crianças incapacitadas física, mental ou (2) Organização dos Estados Americanos.
socialmente serão proporcionados o tratamento, a Comentário:
educação e os cuidados especiais exigidos por sua
Não há muito o que dizer, senão que o sistema global é o
A demanda familiar. sistema criado pela ONU (Organização das Nações Unidas), que
B incapacidade temporária. possui como principais documentos:
C idade. a) Declaração Universal dos Direitos Humanos - DUDH
D condição peculiar. (1948);
E conduta. b) Pacto Internacional dos Direitos Civis e Políticos -
PIDCP (1966);
04. (CESPE - 2019 - CGE - CE - Auditor de Controle c) Pacto Internacional dos Direitos Econômicos, Sociais e
Interno - Fomento ao Controle Social) Com base na Culturais – PIDESC (1966). Já o sistema regional do qual o
Convenção sobre os Direitos da Criança — Decreto Brasil faz parte é aquele criado na OEA (Organização dos
n.º 99.710/1990 —, assinale a opção correta. Estados Americanos), tendo como principais documentos: a)
Declaração Americana dos Direitos e Deveres do Homem
A Essa convenção diferencia crianças de adolescentes: (1948);
pessoas de até doze anos de idade são consideradas
crianças, e as de idade entre doze anos e dezoito b) Convenção Americana de Direitos Humanos, também
anos são consideradas adolescentes. conhecida como Pacto de San José da Costa Rica (1969).

www.editoradince.com - Acesse e veja se há novidades a respeito deste material – CUIDADO: cópia é crime.
60 D DIREITOS HUMANOS - Teoria esquematizada, dicas e questões de concurso
02. Sobre a Corte Interamericana de Direitos Humanos, é Comentário:
correto afirmar: Com relação à questão, ela será respondida por alternativas:
a. ( ) Sediada em San José (Costa Rica) é uma Alternativa “a”: A regra diz o contrário. Não sabe prisão por
instituição judiciária autônoma cujo objetivo é a dívida, salvo no caso de inadimplemento de obrigação alimentar
aplicação e a interpretação da Convenção Americana (art. 7º, item 7). Alternativa “b”: A questão está errada, pois o
sobre Direitos Humanos. direito à liberdade de pensamento e de expressão inclui a
b. ( ) A Corte compor-se-á de cinco juízes, nacionais dos liberdade de procurar, receber e difundir informações e ideias de
Estados membros da Organização, eleitos a título qualquer natureza (art. 13, item 1). Alternativa “c”: Questão
pessoal dentre juristas da mais alta autoridade moral, correta, posto que descreve o art. 5º, item 2. Alternativa “d”: A
de reconhecida competência em matéria de direitos lei deve sim proibir propaganda a favor da guerra, bem como
humanos. apologia ao ódio nacional, racial ou religioso, para que não ha a
a limitação do direito de liberdade de expressão (art. 13, item 5).
c. ( ) Os juízes da Corte serão eleitos, em votação aberta
Alternativa “e”: A questão está incorreta pelo fato de que
e pelo voto da maioria absoluta dos Estados-Partes na
realmente o ideal é a abolição da pena de morte, mas o art. 4º,
Convenção.
item 2, ressalva que “Nos países que não houverem abolido a
d. ( ) Os juízes da Corte serão eleitos por um período de pena de morte, esta só poderá ser imposta pelos delitos mais
seis anos e não poderão ser reeleitos. graves, em cumprimento de sentença final de tribunal
e. ( ) O quorum para as deliberações da Corte é competente e em conformidade com a lei que estabeleça tal
constituído por quatro juízes. pena, promulgada antes de haver o delito sido cometido.
Comentário: Tampouco se estenderá sua aplicação a delitos aos quais não se
A Corte Interamericana de Direitos Humanos recebeu uma aplique atualmente”.
aula toda de atenção no nosso preparatório. Lendo o Estatuto da
Corte é possível extrair todas as respostas. A questão será 04. Podem submeter casos para decisão da Corte
comentada por alternativa: Alternativa “a”: É a correta. A sede Interamericana de Direitos Humanos:
realmente fica em San José, na Costa Rica (art. 3, item 1). a. ( ) Apenas os Estados-Partes da Convenção Americana
Ademais, trata-se uma instituição judiciária autônoma cujo de Direitos Humanos.
objetivo é a aplicação e a interpretação da Convenção
b. ( ) Apenas a Comissão Interamericana de Direitos
Americana sobre Direitos Humanos (art. 1). Alternativa “b”:
Humanos.
Não são cinco juízes, mas sim sete juízes (art. 4, item 1).
Alternativa “c”: O voto não é aberto, mas sim secreto (art. 9, c. ( ) Qualquer pessoa e a Comissão Interamericana de
item 1). Alternativa “d”: Ao contrário do que diz a alternativa, Direitos Humanos.
cabe uma reeleição (art. 5, item 1). Alternativa “e”: Nas aulas, d. ( ) Qualquer pessoa e os Estados-Partes da Convenção
repetimos exaustivamente que a composição não se confunde Americana de Direitos Humanos.
com o quorum. E caiu na prova! O quorum não é constituído por e. ( ) Os Estados-Partes da Convenção Americana de
quatro juízes, mas sim cinco juízes (art. 23, item 1). Direitos Humanos e a Comissão Interamericana de
Direitos Humanos.
03. Para a proteção dos direitos humanos, o instrumento Comentário:
de maior importância no sistema interamericano é a Questão muito comentada no preparatório, com diversas
Convenção Americana de Direitos Humanos assinada questões resolvidas exatamente iguais, a resposta correta era a
em 1969, também denominada de Pacto de San José letra “e”, já que somente os Estados Partes e a Comissão
da Costa Rica. No âmbito dos direitos protegidos na Interamericana de Direitos Humanos podem submeter casos à
Convenção Americana de Direitos Humanos, é correto Corte. É a literalidade do art. 61, item 1, da Convenção
afirmar: Americana de Direitos Humanos: “Somente os Estados-partes e
a. ( ) No que se refere ao direito de liberdade pessoal, a a Comissão têm direito de submeter um caso à decisão da
Convenção Americana de Direitos Humanos autoriza Corte”. Aqui não se visualiza problema algum em cobrarem a
a prisão civil por dívida, seja qual for a natureza do Convenção, já que havia referência expressa à Corte
inadimplemento. Interamericana no edital.
b. ( ) Toda pessoa tem direito à liberdade de pensamento
e de expressão. Esse direito não compreende a 05. A Declaração Universal dos Direitos Humanos
liberdade de buscar, receber e difundir informações e aprovada pela ONU em 1948 declara expressamente.
ideias de natureza política.
1. Todo ser humano tem direito à liberdade de
c. ( ) Ninguém deve ser submetido a torturas, nem a pensamento, consciência e religião nos países
penas ou tratos cruéis, desumanos ou degradantes. ocidentais e cristãos.
Toda pessoa privada da liberdade deve ser tratada
2. Afirma que todos os seres humanos nascem livres e
com o respeito devido à dignidade inerente ao ser
iguais em dignidade e direitos. São dotados de razão
humano.
e consciência e devem agir em relação uns aos outros
d. ( ) A lei não deve proibir propaganda a favor da guerra, com espírito de fraternidade.
bem como apologia ao ódio nacional, racial ou
3. Todo ser humano tem direito à liberdade de opinião e
religioso, para que não haja a limitação do direito de
expressão; este direito inclui a liberdade de, sem
liberdade de expressão.
interferência, ter opiniões e de procurar, receber e
e. ( ) Toda pessoa tem o direito de que se respeite sua transmitir informações e ideias por quaisquer meios e
vida. Esse direito deve ser protegido pela lei e, em independentemente de fronteiras.
geral, desde o momento da concepção. Ninguém pode
4. Toda pessoa vítima de perseguição tem o direito de
ser privado da vida arbitrariamente, razão pela qual os
procurar e de gozar refúgio em outros países, mesmo
Estados-Partes não deverão, em hipótese alguma,
em casos de perseguição legitimamente motivada por
admitir a pena de morte, por violar os direitos previstos
crime de direito comum.
na Convenção.

www.editoradince.com - Acesse e veja se há novidades a respeito deste material – CUIDADO: cópia é crime.
DIREITOS HUMANOS - Teoria esquematizada, dicas e questões de concursos 61
Assinale a alternativa que indica todas as afirmativas caso alguém tenha errado, sugerimos ler os motivos pelos quais
corretas. indicamos o recurso.
a. ( ) São corretas apenas as afirmativas 2 e 3.
b. ( ) São corretas apenas as afirmativas 3 e 4. 08. No plano internacional, as Diretrizes das Nações
c. ( ) São corretas apenas as afirmativas 1, 2 e 3. Unidas para a Delinquência juvenil referem-se a qual
instrumento jurídico
d. ( ) São corretas apenas as afirmativas 1, 2 e 4.
a. ( ) Regras de Riad.
e. ( ) São corretas apenas as afirmativas 2, 3 e 4.
b. ( ) Regras de Beijing.
Comentário:
c. ( ) Convenção Internacional Sobre os Direitos da
Questão que cobrou conteúdo da Declaração Universal dos
Criança de 1989.
Direitos Humanos. Como foi comentado em aula, a ideia era que
o candidato fosse fazer a prova conhecendo a Declaração de d. ( ) Declaração Universal dos Direitos da Criança de
“cabo a rabo”. Pois bem, seguem os comentários 1959.
individualizados para cada assertiva: Assertiva “1”: Trata-se de e. ( ) Declaração Universal dos Direitos Humanos de
direito previsto no art. XVIII da DUDH. Mas a questão está 1948.
incorreta por delimitar o direito aos países ocidentais e cristãos, Comentário:
já que a declaração é universal. Assertiva “2”: Correta a
De forma simplificada, sim, as regras de Riad tratam da
assertiva, pois reproduz o art. I da DUDH. Assertiva “3”:
“Prevenção da Delinquência juvenil”, sendo que a resposta
Correta a assertiva, pois reproduz o art. XIX da DUDH.
correta era a letra “a”.
Assertiva “4”: realmente toda pessoa vítima de perseguição tem
o direito de procurar e de gozar ref gio em outros países (art.
XIV, item I, da DUDH). No entanto, a assertiva está errada ao 09. No plano internacional, as regras Mínimas para a
afirmar que isso valeria mesmo em casos de perseguição Administração da Justiça, da Infância e da Juventude
legitimamente motivada por crime de direito comum, já que referem-se a qual instrumento jurídico
contraria a regra do art. XIV, item 2, da DUDH). a. ( ) Regras de Riad.
b. ( ) Regras de Beijing.
06. Assinale a alternativa que indica corretamente o c. ( ) Convenção Internacional Sobre os Direitos da
principal instrumento de proteção aos direitos dos Criança de 1989.
menores de 18 anos aprovado pela ONU, mais d. ( ) Declaração Universal dos Direitos da Criança de
abrangente e específico área da infância e adotado 1959.
por mais de 190 países.
e. ( ) Declaração Universal dos Direitos Humanos de
a. ( ) Regras de Riad. 1948.
b. ( ) Regras de Beijing.
c. ( ) Declaração Universal dos Direitos Humanos de 10 (TÉCNICO SOCIOEDUCATIVO/ ADMINISTRATIVO-
1948. DF/2015) De acordo com o que dispõe a Declaração
d. ( ) Convenção Internacional Sobre os Direitos da Universal dos Direitos Humanos, os direitos humanos
Criança de 1989 da ONU. são indivisíveis e englobam, exclusivamente, os
e. ( ) Convenção Americana de Direitos Humanos de direitos
1969. (A) civis, políticos, econômicos, sociais e culturais, não
prevendo hierarquia entre ELES.
07. A Constituição Federal de 1988, após a reforma (B) civis e políticos.
ocorrida pela Emenda Constitucional n 45 2004, (C) coletivos e individuais, estes últimos hierarquicamente
dispõe no seu artigo 5 , § 3 que: ―Os tratados e superiores.
convenções internacionais sobre direitos humanos (D) econômicos e sociais
que forem aprovados, em cada Casa do Congresso
Nacional, em dois turnos, por três quintos dos votos (E) privados e públicos, estes últimos hierarquicamente
dos respectivos membros, serão equivalentes s superiores.
emendas constitucionais.‖ Esta reforma constitucional,
no que se refere incorporação dos tratados 11. (SJC/SC AS AGENTE DE SEGURANÇA
internacionais de proteção aos direitos humanos no SOCIOEDUCATIVO/2016) Segundo a Convenção
sistema jurídico brasileiro, reconhece aos direitos Internacional sobre os Direitos da Criança,
humanos previstos nos tratados internacionais, excetuando-se os casos em que, pela lei aplicável à
expressamente o status ou força de: criança, a maioridade é antecipada, considera-se
a. ( ) Resolução. como criança todo ser humano com menos de
b. ( ) Lei constitucional. (A) 6 anos de idade.
c. ( ) Lei municipal. (B) 12 anos de idade.
d. ( ) Lei estadual. (C) 15 anos de idade.
e. ( ) Lei federal. (D) 18 anos de idade.
Comentário: (E) 21 anos de idade
Trata-se de mais uma questão amplamente comentada no
nosso curso. Inclusive por conta de questões extremamente 12. (SJC/SC AS AGENTE DE SEGURANÇA
questionáveis no concurso passado. Infelizmente, não foi SOCIOEDUCATIVO/2016) Considerando as Regras
diferente nesse novo concurso. Inicialmente, com base nas de Beijing (regras mínimas das Nações Unidas para a
alternativas dadas, não poderia ser outra a resposta senão a letra administração da justiça da infância e da juventude),
“b”, que definia o status de “Lei constitucional”. No entanto, assinale a alternativa correta.

www.editoradince.com - Acesse e veja se há novidades a respeito deste material – CUIDADO: cópia é crime.
62 D DIREITOS HUMANOS - Teoria esquematizada, dicas e questões de concurso
(A) Nos sistemas jurídicos que reconheçam o conceito de
responsabilidade penal para jovens, o começo dessa Gabarito: 01/B; 02/A; 03/C; 04/E; 05/A; 06/D; 07/B; 08/A;
responsabilidade penal não deverá se fixar em uma 09/B; 10/A; 11/D; 12/B; 13/D; 14/C
idade demasiadamente precoce, levando-se em conta,
necessariamente, as circunstâncias que acompanham
a maturidade física ou biológica, emocional, mental e REFERÊNCIAS:
intelectual. CAMPOS, José Roberto Bassul.
(B) As disposições pertinentes das regras não só se Organizações Não-Governamentais
aplicarão aos jovens infratores, mas também àqueles nas
que possam ser processados por realizar qualquer ato Áreas Ambiental, Indígena e Mineral. 1999.
concreto que não seria punível se fosse praticado por Disponível em:
adultos. <http://www.senado.gov.br/senado/conleg/artigos/espe
(C) As garantias processuais básicas, como a presunção ciais/OrganizacoesNaoGoverna mentais.pdf >
de inocência, o direito de ser informado das Acesso em 28/05/2011.
acusações, a assistência judiciária, a presença dos Declaração Universal dos Direitos Humanos.
pais ou tutores, a confrontação com testemunhas e o Disponível em:
direito a interrogá-las, a publicidade do julgamento e o <http://portal.mj.gov.br/sedh/ct/legis_intern/ddh_bib_inter
direito de apelação ante uma autoridade superior, _universal.htm> Acesso em: 26/05/2011.
serão respeitadas em todas as etapas do processo.
MIGUEL, Luis Felipe. Notas & Comentáios: cidadania
(D) Toda remissão que signifique encaminhar o jovem a e direito à informação. Brasília, 2001.
instituições da comunidade não dependerá do Disponível
consentimento dele ou de seu representante legal, em:<http://www.cebela.org.br/imagens/Materia/2001-
sendo a decisão relativa à remissão do caso 2%20191-199%20luis%20felipe.pdf> Acesso em:
submetida ao exame de uma autoridade competente. 26/05/2011.
(E) Para facilitar a adoção de uma decisão justa por parte ROSA, Rosane. Cidadania Expandida e Identidades
da autoridade competente, antes da decisão definitiva, Compartilhadas. Razón y Palabra: Primeira Revista
será efetuada uma investigação completa a respeito Digital em
do meio social, das circunstâncias de vida do jovem e
das condições em que se deu a prática da infração, em:
independentemente da gravidade ou da natureza http://www.razonypalabra.org.mx/13%20Rosa_revisa
dessa infração. do.pdf acessado em 20 abril 2011.

13. (ASSISTENTE SOCIOEDUCATIVO – FUNASE/2013) ANOTAÇÕES:


A Declaração Universal dos Direitos Humanos vem ______________________________________________
exercendo um importante papel jurídico político na ______________________________________________
difusão da projeção dos direitos humanos por todo o
______________________________________________
mundo. Os seus dispositivos contêm as seguintes
características, à EXCEÇÃO de ______________________________________________
A) inviolabilidade. ______________________________________________
B) universalidade. ______________________________________________
C) imprescritibilidade. ______________________________________________
D) renunciabilidade. ______________________________________________
E) igualdade. ______________________________________________
______________________________________________
14. (ASSISTENTE SOCIOEDUCATIVO – FUNASE/2013) ______________________________________________
35. Assinale a alternativa INCORRETA sobre a ______________________________________________
Declaração de Direitos Humanos. ______________________________________________
A) Tem como traço marcante a eleição do regime ______________________________________________
democrático, destinado à completa satisfação tanto ______________________________________________
das liberdades individuais como dos direitos sociais.
______________________________________________
B) O princípio constitucional, que prevê que ninguém será
______________________________________________
submetido à tortura nem a tratamento cruel,
desumano e degradante, reflete a cláusula ______________________________________________
internacional idêntica prevista na Declaração Universal ______________________________________________
de Direitos Humanos. ______________________________________________
C) Os tratados e as convenções internacionais de direitos ______________________________________________
humanos ingressam no direito Brasileiro por meio de ______________________________________________
medida provisória do Presidente da República.
______________________________________________
D) Está expresso no texto que todo homem tem direito a
organizar sindicatos e nele ingressar para a proteção ______________________________________________
de seus interesses. ______________________________________________
E) A Declaração Universal de Diretos Humanos também ______________________________________________
foi criada como objetivo de promover o ______________________________________________
desenvolvimento de relação amistosa entre as ______________________________________________
nações.

www.editoradince.com - Acesse e veja se há novidades a respeito deste material – CUIDADO: cópia é crime.
LEGISLAÇÃO ESPECIAL E REGIMENTO INTERNO DAS UNIDADES DO SEAS 1
o Sistema Nacional de Atendimento Socioeducativo
LEGISLAÇÃO ESPECIAL (Sinase), como um subsistema do Sistema de Garantia
(DESTACADA) e de Direitos (SGD) que, como tal, deve se comunicar e
interagir com os demais subsistemas do SGD (tais
REGIMENTO INTERNO DAS como o da Saúde, Educação, Assistência Social,
Justiça e Segurança Pública). Dentro do sistema maior
UNIDADES DO SEAS que é o SGD, o Sinase destina-se a reunir princípios,
regras e critérios a serem aplicados à execução das
Legislação destacada, dicas e questões medidas socioeducativas.
de provas anteriores. o
§ 1 Entende-se por Sinase o conjunto ordenado
Vanques de Melo de princípios, regras e critérios que envolvem a execução
de medidas socioeducativas, incluindo-se nele, por
adesão, os sistemas estaduais, distrital e municipais, bem
CONTEÚDO PROGRAMÁTICO:
como todos os planos, políticas e programas específicos
Lei Federal nº 8.069 de 13 de julho de 1990 (Estatuto da de atendimento a adolescente em conflito com a lei.
Criança e do Adolescente - ECA) e suas alterações . 19
O Sinase possuía como premissa básica a
Questões de concursos................................................ 56
necessidade de se constituir parâmetros mais objetivos
Lei Federal nº 12.594, de 18 de janeiro de 2012 e procedimentos mais justos que limitassem a
(Sistema Nacional de Atendimento Socioeducativo - discricionariedade e de reafirmar a natureza
SINASE) e suas alterações ......................................... 1 pedagógica da medida socioeducativa. A Lei nº
Questões de concursos................................................ 17 12.594/12 promoveu os ditames desse documento ao
Lei Federal Nº 9.455 de 07 de abril de 1997 (Lei da status de lei, garantindo importante avanço na
Tortura) e suas alterações ........................................ 57 promoção e na defesa dos adolescentes autores de ato
Questões de concursos................................................ 60 infracional, e estabeleceu o procedimento legal para a
execução das medidas socioeducativas, suprindo a
Lei Federal Nº 8.742, de 07/12/1993, que dispõe sobre a lacuna deixada pelo ECA.
organização da Assistência Social (SUAS) e suas o
alterações. ................................................................ 61 § 2 Entendem-se por medidas socioeducativas
o
Questões de concursos................................................ 70 as previstas no art. 112 da Lei n 8.069, de 13 de julho de
1990 (Estatuto da Criança e do Adolescente), as quais
Lei Federal Nº 8.080, de 19/09/1990, que dispõe sobre têm por objetivos:
as condições para a promoção, proteção e
recuperação da saúde, a organização e o I - a responsabilização do adolescente quanto às
funcionamento dos serviços correspondentes (SUS) consequências lesivas do ato infracional, sempre que
e suas alterações. ..................................................... 71 possível incentivando a sua reparação;
Questões de concursos................................................ 80 II - a integração social do adolescente e a garantia
de seus direitos individuais e sociais, por meio do
O Regimento Interno das Unidades da cumprimento de seu plano individual de atendimento; e
Superintendência do Sistema Estadual de
Atendimento Socioeducativo (SEAS) ........................ 81 III - a desaprovação da conduta infracional,
efetivando as disposições da sentença como parâmetro
Questões de concursos................................................ 97 máximo de privação de liberdade ou restrição de direitos,
observados os limites previstos em lei.
O rol de medidas socioeducativas está previsto no
LEI FEDERAL Nº 12.594 DE 18 DE art. 112 do ECA e não foi alterado pela Lei
12.594/2012.
JANEIRO DE 2012, QUE INSTITUI O Assim, quando um adolescente pratica um ato
SISTEMA NACIONAL DE ATENDIMENTO infracional ele poderá receber as seguintes medidas
SOCIOEDUCATIVO (SINASE). (art. 112 do ECA):
I - advertência;
ESQUEMATIZADA
II - obrigação de reparar o dano;
A PRESIDENTA DA REPÚBLICA
III - prestação de serviços à comunidade1;
Faço saber que o Congresso Nacional decreta e
IV - liberdade assistida2;
eu sanciono a seguinte Lei:
TÍTULO I
DO SISTEMA NACIONAL DE ATENDIMENTO 1
A prestação de serviços comunitários (art. 117 do ECA)
SOCIOEDUCATIVO (Sinase)
consiste na realização de tarefas gratuitas de interesse geral,
CAPÍTULO I por período não excedente a seis meses, junto a entidades
DISPOSIÇÕES GERAIS assistenciais, hospitais, escolas e outros estabelecimentos
o congêneres, bem como em programas comunitários ou
Art. 1 Esta Lei institui o Sistema Nacional de
Atendimento Socioeducativo (Sinase) e regulamenta a governamentais. As tarefas serão atribuídas conforme as
execução das medidas destinadas a adolescente que aptidões do adolescente, devendo ser cumpridas durante
pratique ato infracional. jornada máxima de oito horas semanais, aos sábados,
domingos e feriados ou em dias úteis, de modo a não
Segundo Oliveira (2002, p. 35), Sistema é um prejudicar a frequência à escola ou à jornada normal de
conjunto de partes interagentes e interdependentes trabalho.
que, conjuntamente, formam um todo unitário com 2
Liberdade assistida (art. 118 do ECA) “Baseada no instituto
determinado objetivo e efetuam determinada função. norte-americano do probation system, consiste em submeter
Tendo por base a teoria dos sistemas, foi desenvolvido o adolescente, após sua entrega aos pais ou responsável, a

www.editoradince.com - Acesse e veja se há novidades a respeito deste material – CUIDADO: cópia é crime.
2 LEGISLAÇÃO ESPECIAL E REGIMENTO INTERNO DAS UNIDADES DO SEAS
V - inserção em regime de semiliberdade; necessários ao desenvolvimento de programas de
atendimento.
VI - internação em estabelecimento educacional;
VII - qualquer uma das medidas protetivas A Lei conceitua entidade de atendimento,
previstas no art. 101, I a VI do ECA (exs: orientação, seguindo o raciocínio já inserto no artigo 90 do ECA,
matrícula obrigatória em escola, inclusão em programa como pessoas jurídicas responsáveis por instalar e
comunitário, entre outras). manter a unidade e os recursos humanos e materiais
necessários ao desenvolvimento do programa de
Ao estabelecer os objetivos das medidas
atendimento. Segundo o documento Sinase, aprovado
socioeducativas, a Lei nº 12.594/12 visou afastar as
pela Resolução n.º 119/2006 do Conanda, cabe às
infindáveis discussões doutrinárias acerca da natureza
entidades de atendimento:
sancionatória ou pedagógica dessas medidas. Parece-
nos que, a partir dessa definição expressa dos a) elaborar o Programa de Atendimento a ser
objetivos da medida, firmou-se o entendimento do executado pela respectiva Unidade;
legislador de que tais medidas possuem um caráter b) inscrever o Programa e suas posteriores
híbrido, de sanção socioeducativa, com finalidade alterações nos Conselhos de Direitos das Crianças e
pedagógica. Adolescentes (CDCA) segundo a abrangência
NÃO ESQUEÇA: territorial do atendimento (no Conselho Estadual, se a
abrangência for regional – mais de um município, ou
Somente aos adolescentes são aplicadas medidas
no Conselho Municipal);
socioeducativas. Caso uma criança pratique um ato
infracional (Ex.: criança com 11 anos promove um c) desenvolver os Programas de Atendimento
roubo), poderá ser aplicada apenas medidas aprovados pelo CDCA e
protetivas, que estão previstas no art. 101 do ECA. d) prestar contas ao órgão gestor ao qual se
o vincula.
§ 3 Entendem-se por programa de atendimento
a organização e o funcionamento, por unidade, das Cabe ressaltar que admitiu o legislador que as
condições necessárias para o cumprimento das medidas medidas socioeducativas fossem executadas por
socioeducativas. pessoas jurídicas de direito público ou privado, o que
afasta qualquer dúvida sobre a possibilidade de
A Lei conceitua o programa de atendimento como entidades não governamentais (tais como associações,
a organização e o funcionamento, por unidade, das fundações ou ONGs) serem executoras das medidas,
condições necessárias para o cumprimento das quer em meio aberto, quer em meio fechado. Essa
medidas socioeducativas. Esses programas consistirão, previsão, que já constava da interpretação do artigo
pois, nas ações voltadas para o conteúdo prático da 90, caput e §1º do ECA, foi reforçada pela Lei nº
execução dessas medidas e são divididos em quatro 12.594/12.
modalidades: prestação de serviços à comunidade, o
liberdade assistida, semiliberdade e internação. É Art. 2 O Sinase será coordenado pela União e
preciso, contudo, destacar que a Lei n.º 12.596/12 integrado pelos sistemas estaduais, distrital e
usou a palavra programa tecnicamente. Trata-se de municipais responsáveis pela implementação dos seus
termo cujo significado vem explicitado na Lei n.º respectivos programas de atendimento a adolescente ao
8.742/93 (LOAS), no artigo 243. Essa matéria será qual seja aplicada medida socioeducativa, com liberdade
mais bem detalhada nos comentários aos artigos. 9º e de organização e funcionamento, respeitados os termos
seguintes. desta Lei.
o O Sinase é um sistema nacional de atendimento
§ 4 Entende-se por unidade a base física
necessária para a organização e o funcionamento de socioeducativo e, portanto, sua coordenação cabe à
programa de atendimento. União. Esse sistema nacional é integrado pelos
sistemas estaduais, distritais e municipais, aos quais
Unidade é a base física na qual as medidas cabe a implementação dos respectivos programas de
socioeducativas serão executadas, podendo ser os atendimento para execução das medidas
centros socioeducativos ou centros de semiliberdade, socioeducativas, sendo certo que aos municípios cabe
nas medidas em meio fechado, ou as unidades físicas a execução das medidas em meio aberto e, aos
onde funcionam os CREAS ou as sedes de entidades Estados, das medidas em meio fechado.
não governamentais, no caso das medidas em meio
A Lei confere aos sistemas estaduais, municipais e
aberto. Importante observar que a Lei especificou
distrital a liberdade de organização e funcionamento.
nesse dispositivo, que todo programa de atendimento
Isso significa que compete a cada estado e município a
dependerá de uma sede física adequada para sua
indicação, no âmbito de sua estrutura administrativa,
execução, acabando com as possibilidades de
do órgão da administração direta responsável pela
improviso.
coordenação da execução e gestão dos Sistemas
o
§ 5 Entendem-se por entidade de atendimento Estaduais e Municipais de Atendimento
a pessoa jurídica de direito público ou privado que instala Socioeducativo, desde que expressamente designados
e mantém a unidade e os recursos humanos e materiais nos respectivos Planos de Atendimento
Socioeducativo (vide artigos 4º, §3º e 5º, §4º desta
Lei). Na prática, porém, verifica-se uma limitação
uma vigilância e acompanhamentos discretos, a distância, financeira para o exercício dessa autonomia, sobretudo
com o fim de impedir a reincidência e obter a para os Municípios. Isso porque a partir do ano de
ressocialização. Na prática, consiste na obrigação de o 2008, o Ministério do Desenvolvimento Social e
adolescente infrator e seus responsáveis legais Combate à Fome (MDS) passou a financiar a
comparecerem periodicamente a um posto predeterminado execução das medidas socioeducativas em meio
e, ali, entrevistarem-se com os técnicos para informar suas aberto, no âmbito dos Centros de Referência
atividades.” (Eduardo Roberto Alcântara Del-Campo e Especializados em Assistência Social (CREAS).
Thales Cezar de Oliveira).

www.editoradince.com - Acesse e veja se há novidades a respeito deste material – CUIDADO: cópia é crime.
LEGISLAÇÃO ESPECIAL E REGIMENTO INTERNO DAS UNIDADES DO SEAS 3
o
O Governo Federal, porém, estabeleceu como um § 4 À Secretaria de Direitos Humanos da
dos critérios para o cofinanciamento federal do Presidência da República (SDH/PR) competem as
Serviço de Proteção Social aos Adolescentes em funções executiva e de gestão do Sinase.
Cumprimento de Medidas Socioeducativas em Meio Vê art. 18 desta Lei.
Aberto de Liberdade Assistida e Prestação de Serviço o
Art. 4 Compete aos Estados:
à Comunidade, que a execução dessas medidas fosse
I - formular, instituir, coordenar e manter Sistema
realizada diretamente pelo CREAS.
Estadual de Atendimento Socioeducativo, respeitadas as
Assim sendo, se o Município formalizar o aceite diretrizes fixadas pela União;
do cofinanciamento federal para implementação do
II - elaborar o Plano Estadual de Atendimento
referido serviço, significa que ele aceita as regras
Socioeducativo em conformidade com o Plano Nacional;
estabelecidas pelo MDS, assumindo os compromissos
decorrentes do aceite, como uma adesão à III - criar, desenvolver e manter programas para
normatização pertinente. Por conseguinte, as aludidas a execução das medidas socioeducativas de
medidas socioeducativas deverão ser ofertadas semiliberdade e internação;
obrigatoriamente no âmbito do CREAS, sendo o mais IV - editar normas complementares para a
recomendável que a Secretaria Municipal de organização e funcionamento do seu sistema de
Assistência Social seja o órgão gestor da execução das atendimento e dos sistemas municipais;
medidas socioeducativas. V - estabelecer com os Municípios formas de
Lado outro, caso não haja cofinanciamento federal colaboração para o atendimento socioeducativo em meio
para a execução das medidas socioeducativas em meio aberto;
aberto, o Município terá liberdade para definir o órgão VI - prestar assessoria técnica e suplementação
responsável pela oferta do serviço, podendo fazê-lo, financeira aos Municípios para a oferta regular de
inclusive, por meio de convênio com entidades não programas de meio aberto;
governamentais. VII - garantir o pleno funcionamento do plantão
CAPÍTULO II interinstitucional, nos termos previstos no inciso V do art.
o
DAS COMPETÊNCIAS 88 da Lei n 8.069, de 13 de julho de 1990 (Estatuto da
o Criança e do Adolescente);
Art. 3 Compete à União:
VIII - garantir defesa técnica do adolescente a
I - formular e coordenar a execução da política
quem se atribua prática de ato infracional;
nacional de atendimento socioeducativo;
IX - cadastrar-se no Sistema Nacional de
II - elaborar o Plano Nacional de Atendimento
Informações sobre o Atendimento Socioeducativo e
Socioeducativo, em parceria com os Estados, o Distrito
fornecer regularmente os dados necessários ao
Federal e os Municípios;
povoamento e à atualização do Sistema; e
III - prestar assistência técnica e suplementação
X - cofinanciar, com os demais entes federados, a
financeira aos Estados, ao Distrito Federal e aos
execução de programas e ações destinados ao
Municípios para o desenvolvimento de seus sistemas;
atendimento inicial de adolescente apreendido para
IV - instituir e manter o Sistema Nacional de apuração de ato infracional, bem como aqueles
Informações sobre o Atendimento Socioeducativo, seu destinados a adolescente a quem foi aplicada medida
funcionamento, entidades, programas, incluindo dados socioeducativa privativa de liberdade.
relativos a financiamento e população atendida; o
§ 1 Ao Conselho Estadual dos Direitos da Criança
V - contribuir para a qualificação e ação em rede e do Adolescente competem as funções deliberativas e de
dos Sistemas de Atendimento Socioeducativo; controle do Sistema Estadual de Atendimento
VI - estabelecer diretrizes sobre a organização e Socioeducativo, nos termos previstos no inciso II do art.
funcionamento das unidades e programas de atendimento 88 da Lei nº 8.069, de 13 de julho de 1990 (Estatuto da
e as normas de referência destinadas ao cumprimento Criança e do Adolescente), bem como outras definidas na
das medidas socioeducativas de internação e legislação estadual ou distrital.
semiliberdade; ECA: Art. 88. São diretrizes da política de
VII - instituir e manter processo de avaliação dos atendimento:
Sistemas de Atendimento Socioeducativo, seus planos, II - criação de conselhos municipais, estaduais e
entidades e programas;
nacional dos direitos da criança e do adolescente,
VIII - financiar, com os demais entes federados, a órgãos deliberativos e controladores das ações em
execução de programas e serviços do Sinase; e todos os níveis, assegurada a participação popular
IX - garantir a publicidade de informações sobre paritária por meio de organizações representativas,
repasses de recursos aos gestores estaduais, distrital e segundo leis federal, estaduais e municipais;
municipais, para financiamento de programas de o
§ 2 O Plano de que trata o inciso II
atendimento socioeducativo.
o
do caput deste artigo será submetido à deliberação do
§ 1 São vedados à União o desenvolvimento e a Conselho Estadual dos Direitos da Criança e do
oferta de programas próprios de atendimento. Adolescente.
o
§ 2 Ao Conselho Nacional dos Direitos da Criança o
§ 3 Competem ao órgão a ser designado no Plano
e do Adolescente (Conanda) competem as funções de que trata o inciso II do caput deste artigo as funções
normativa, deliberativa, de avaliação e de fiscalização do executiva e de gestão do Sistema Estadual de
o
Sinase, nos termos previstos na Lei n 8.242, de 12 de Atendimento Socioeducativo.
outubro de 1991, que cria o referido Conselho. o
o
Art. 5 Compete aos Municípios:
§ 3 O Plano de que trata o inciso II
I - formular, instituir, coordenar e manter o Sistema
do caput deste artigo será submetido à deliberação do
Municipal de Atendimento Socioeducativo, respeitadas as
Conanda.
diretrizes fixadas pela União e pelo respectivo Estado;

www.editoradince.com - Acesse e veja se há novidades a respeito deste material – CUIDADO: cópia é crime.
4 LEGISLAÇÃO ESPECIAL E REGIMENTO INTERNO DAS UNIDADES DO SEAS
II - elaborar o Plano Municipal de Atendimento  Estados: quanto às medidas de semiliberdade3 e
Socioeducativo, em conformidade com o Plano Nacional e internação4 => tratam-se de medidas socioeducativas
o respectivo Plano Estadual; que implicam privação de liberdade.
III - criar e manter programas de atendimento  Municípios: quanto às medidas
para a execução das medidas socioeducativas em socioeducativas em meio aberto (prestação de
meio aberto; serviços à comunidade e liberdade assistida.
IV - editar normas complementares para a Como visto acima, a Lei 12.594/2012 delimitou,
organização e funcionamento dos programas do seu de forma expressa e peremptória, a responsabilidade
Sistema de Atendimento Socioeducativo; de cada ente público. Façamos uma síntese:
V - cadastrar-se no Sistema Nacional de  União: formular e coordenar a política nacional
Informações sobre o Atendimento Socioeducativo e de atendimento socioeducativo;
fornecer regularmente os dados necessários ao
 Estados: criar e manter programas para as
povoamento e à atualização do Sistema; e
medidas de semiliberdade e internação;
VI - cofinanciar, conjuntamente com os demais
 Municípios: criar e manter programas para as
entes federados, a execução de programas e ações
medidas socioeducativas em meio aberto.
destinados ao atendimento inicial de adolescente
apreendido para apuração de ato infracional, bem como
aqueles destinados a adolescente a quem foi aplicada
medida socioeducativa em meio aberto.
o
§ 1 Para garantir a oferta de programa de
atendimento socioeducativo de meio aberto, os
Municípios podem instituir os consórcios dos quais trata a
o
Lei n 11.107, de 6 de abril de 2005, que dispõe sobre
normas gerais de contratação de consórcios públicos e dá
outras providências, ou qualquer outro instrumento
jurídico adequado, como forma de compartilhar
responsabilidades.
o
§ 2 Ao Conselho Municipal dos Direitos da
Criança e do Adolescente competem as funções
deliberativas e de controle do Sistema Municipal de
Atendimento Socioeducativo, nos termos previstos
no inciso II do art. 88 da Lei nº 8.069, de 13 de julho de
1990 (Estatuto da Criança e do Adolescente), bem como
outras definidas na legislação municipal.
o
§ 3 O Plano de que trata o inciso II
do caput deste artigo será submetido à deliberação do
Conselho Municipal dos Direitos da Criança e do
Adolescente.
o
§ 4 Competem ao órgão a ser designado no Plano
de que trata o inciso II do caput deste artigo as funções
executiva e de gestão do Sistema Municipal de
Atendimento Socioeducativo.
o
Art. 6 Ao Distrito Federal cabem,
cumulativamente, as competências dos Estados e dos
Municípios.
O Distrito Federal, onde está sediada a capital do 3
país (Brasília) é, por definição constitucional, uma Semiliberdade (art. 120 do ECA) Pelo regime da
estrutura componente da organização político- semiliberdade, o adolescente realiza atividades externas
administrativa República Federativa do Brasil, de durante o dia, sob supervisão de equipe multidisciplinar, e
forma indissolúvel e autônoma, não podendo ser fica recolhido à noite. O regime de semiliberdade pode ser
dividido em Municípios (embora se saiba que, em seu determinado como medida inicial imposta pelo juiz ao
território, existam várias cidades-satélites). adolescente infrator, ou como forma de transição para o
meio aberto (uma espécie de “progressão”).
Em consonância com o dispositivo constitucional 4
Internação (art. 121 do ECA) Por esse regime, o adolescente
de repartição de competências previsto no §1º do fica recolhido na unidade de internação. A internação
artigo 32 da Constituição Federal, o Distrito Federal constitui medida privativa da liberdade e se sujeita aos
possui, de forma cumulativa, as competências dos princípios de brevidade, excepcionalidade e respeito à
Estados e dos Municípios, vale dizer, todas as condição peculiar de pessoa em desenvolvimento. Pode ser
competências previstas nos arts. 4º e 5º da presente lei. permitida a realização de atividades externas, a critério da
PONTO DE DESTAQUE: equipe técnica da entidade, salvo expressa determinação
De quem é a competência para criar e manter os judicial em contrário.
programas para a execução das medidas A medida não comporta prazo determinado, devendo sua
socioeducativas? manutenção ser reavaliada, mediante decisão fundamentada,
no máximo a cada seis meses.
Em nenhuma hipótese o período máximo de internação
excederá a três anos. Se o interno completar 21 anos,
deverá ser obrigatoriamente liberado, encerrando o regime
de internação.

www.editoradince.com - Acesse e veja se há novidades a respeito deste material – CUIDADO: cópia é crime.
LEGISLAÇÃO ESPECIAL E REGIMENTO INTERNO DAS UNIDADES DO SEAS 5
A Lei estabeleceu o prazo de 1 ano para que o
Poder Executivo assuma a atribuição em tais casos:
Art. 83. Os programas de atendimento
socioeducativo sob a responsabilidade do Poder
Judiciário serão, obrigatoriamente, transferidos ao
Poder Executivo no prazo máximo de 1 (um) ano a
partir da publicação desta Lei e de acordo com a
política de oferta dos programas aqui definidos. Os
Municípios que desenvolvem programas de
semiliberdade e internação, a partir da nova Lei, não
poderão mais mantê-los, devendo, no prazo de 1 ano,
fazer a transferência para os respectivos Estados-
membros:
CAPÍTULO III Art. 84. Os programas de internação e
semiliberdade sob a responsabilidade dos Municípios
DOS PLANOS DE ATENDIMENTO SOCIOEDUCATIVO
o
serão, obrigatoriamente, transferidos para o Poder
Art. 7 O Plano de que trata o inciso II do art. Executivo do respectivo Estado no prazo máximo de 1
o
3 desta Lei deverá incluir um diagnóstico da situação do (um) ano a partir da publicação desta Lei e de acordo
Sinase, as diretrizes, os objetivos, as metas, as com a política de oferta dos programas aqui definidos.
prioridades e as formas de financiamento e gestão das Desse modo, depois desse prazo, nenhum
ações de atendimento para os 10 (dez) anos seguintes, Município poderá mais criar ou manter programas de
em sintonia com os princípios elencados na Lei nº 8.069, execução de medidas de semiliberdade e internação.
de 13 de julho de 1990 (Estatuto da Criança e do
Adolescente). Seção I
o
§ 1 As normas nacionais de referência para o Disposições Gerais
o
atendimento socioeducativo devem constituir anexo ao Art. 9 Os Estados e o Distrito Federal inscreverão
o
Plano de que trata o inciso II do art. 3 desta Lei. seus programas de atendimento e alterações no Conselho
o
§ 2 Os Estados, o Distrito Federal e os Municípios Estadual ou Distrital dos Direitos da Criança e do
deverão, com base no Plano Nacional de Atendimento Adolescente, conforme o caso.
Socioeducativo, elaborar seus planos decenais Art. 10. Os Municípios inscreverão seus
correspondentes, em até 360 (trezentos e sessenta) dias programas e alterações, bem como as entidades de
a partir da aprovação do Plano Nacional. atendimento executoras, no Conselho Municipal dos
PLANO DECENAL Direitos da Criança e do Adolescente.
• diagnóstico, diretrizes, objetivos, O dispositivo esclarece que, em se tratando das
• metas, prioridades, financiamento e gestão medidas socioeducativas de meio aberto de prestação
de serviços à comunidade e liberdade assistida, cuja
• normas referenciais em anexo
execução cabe ao Município (art. 5º, III), a inscrição
• Estado e município um ano a partir da aprovação dos programas de atendimento deve ser realizada pelo
do nacional para elaborar seus planos com base no Conselho Municipal dos Direitos da Criança e do
nacional Adolescente (CMDCA).
o
Art. 8 Os Planos de Atendimento Socioeducativo Para outras considerações acerca do assunto,
deverão, obrigatoriamente, prever ações articuladas nas remetemos o leitor aos comentários ao art. 9º deste
áreas de educação, saúde, assistência social, cultura, trabalho.
capacitação para o trabalho e esporte, para os Art. 11. Além da especificação do regime, são
adolescentes atendidos, em conformidade com os requisitos obrigatórios para a inscrição de programa
princípios elencados na Lei nº 8.069, de 13 de julho de de atendimento:
1990 (Estatuto da Criança e do Adolescente).
I - a exposição das linhas gerais dos métodos e
Parágrafo único. Os Poderes Legislativos federal,
técnicas pedagógicas, com a especificação das atividades
estaduais, distrital e municipais, por meio de suas
de natureza coletiva;
comissões temáticas pertinentes, acompanharão a
execução dos Planos de Atendimento Socioeducativo dos II - a indicação da estrutura material, dos recursos
respectivos entes federados. humanos e das estratégias de segurança compatíveis
com as necessidades da respectiva unidade;
Planos de Atendimento prever obrigatoriamente:
III - regimento interno que regule o funcionamento
• Ações articuladas educação, saúde, assistência da entidade, no qual deverá constar, no mínimo:
social, cultura, capacitação para o trabalho, e esporte
a) o detalhamento das atribuições e
• Comissões temáticas legislativo acompanhará responsabilidades do dirigente, de seus prepostos, dos
respectivos Planos membros da equipe técnica e dos demais educadores;
CAPÍTULO IV b) a previsão das condições do exercício da
DOS PROGRAMAS DE ATENDIMENTO disciplina e concessão de benefícios e o respectivo
procedimento de aplicação; e
A Lei 12.594/2012 prevê ainda que a incumbência
c) a previsão da concessão de benefícios
dos programas de atendimento é do Poder Executivo,
extraordinários e enaltecimento, tendo em vista tornar
não podendo ser desempenhados pelo Poder
público o reconhecimento ao adolescente pelo esforço
Judiciário. Em alguns Estados da Federação, o Poder
realizado na consecução dos objetivos do plano
Judiciário, atualmente, mantém programas de
individual;
atendimento socioeducativo.
IV - a política de formação dos recursos humanos;

www.editoradince.com - Acesse e veja se há novidades a respeito deste material – CUIDADO: cópia é crime.
6 LEGISLAÇÃO ESPECIAL E REGIMENTO INTERNO DAS UNIDADES DO SEAS
V - a previsão das ações de acompanhamento do (Estatuto da Criança e do Adolescente), devendo citar o
adolescente após o cumprimento de medida dirigente do programa e a direção da entidade ou órgão
socioeducativa; credenciado.
VI - a indicação da equipe técnica, cuja quantidade Seção III
e formação devem estar em conformidade com as normas Dos Programas de Privação da Liberdade
de referência do sistema e dos conselhos profissionais e
Art. 15. São requisitos específicos para a inscrição
com o atendimento socioeducativo a ser realizado; e
de programas de regime de semiliberdade ou
VII - a adesão ao Sistema de Informações sobre o internação:
Atendimento Socioeducativo, bem como sua operação
I - a comprovação da existência de
efetiva.
estabelecimento educacional com instalações adequadas
Parágrafo único. O não cumprimento do previsto e em conformidade com as normas de referência;
neste artigo sujeita as entidades de atendimento, os
II - a previsão do processo e dos requisitos para a
órgãos gestores, seus dirigentes ou prepostos à aplicação
escolha do dirigente;
das medidas previstas no art. 97 da Lei nº 8.069, de 13
de julho de 1990 (Estatuto da Criança e do Adolescente). III - a apresentação das atividades de natureza
coletiva;
Art. 12. A composição da equipe técnica do
programa de atendimento deverá ser interdisciplinar, IV - a definição das estratégias para a gestão de
compreendendo, no mínimo, profissionais das áreas de conflitos, vedada a previsão de isolamento cautelar,
o
saúde, educação e assistência social, de acordo com as exceto nos casos previstos no § 2 do art. 49 desta Lei; e
normas de referência. V - a previsão de regime disciplinar nos termos do
o
§ 1 Outros profissionais podem ser acrescentados art. 72 desta Lei.
às equipes para atender necessidades específicas do Art. 16. A estrutura física da unidade deverá ser
programa. compatível com as normas de referência do Sinase.
o o
§ 2 Regimento interno deve discriminar as § 1 É vedada a edificação de unidades
atribuições de cada profissional, sendo proibida a socioeducacionais em espaços contíguos, anexos, ou de
sobreposição dessas atribuições na entidade de qualquer outra forma integrados a estabelecimentos
atendimento. penais.
o o
§ 3 O não cumprimento do previsto neste artigo § 2 A direção da unidade adotará, em caráter
sujeita as entidades de atendimento, seus dirigentes ou excepcional, medidas para proteção do interno em casos
prepostos à aplicação das medidas previstas no art. 97 de risco à sua integridade física, à sua vida, ou à de
da Lei nº 8.069, de 13 de julho de 1990 (Estatuto da outrem, comunicando, de imediato, seu defensor e o
Criança e do Adolescente). Ministério Público.
Seção II A Resolução CONANDA n.º 119/06 prevê, de
Dos Programas de Meio Aberto forma detalhada, critérios arquitetônicos a serem
Art. 13. Compete à direção do programa de seguidos para a construção, reforma e ampliação das
prestação de serviços à comunidade ou de liberdade unidades de atendimento de internação provisória,
assistida: semiliberdade e internação. A Lei faz menção a essas
normas de referência, para as quais deve ser
I - selecionar e credenciar orientadores,
necessariamente remetido o leitor.
designando-os, caso a caso, para acompanhar e avaliar o
cumprimento da medida; Para as unidades de internação, de forma bastante
resumida, estipula o Sinase que as unidades devem
II - receber o adolescente e seus pais ou
atender ao número máximo de 40 adolescentes e que
responsável e orientá-los sobre a finalidade da medida e a
devem ser constituídas de espaços residenciais
organização e funcionamento do programa;
(módulos) com capacidade para até 15 adolescentes. A
III - encaminhar o adolescente para o orientador unidade deve se organizar de forma a que os diferentes
credenciado; módulos atendam a adolescentes em distintos estágios
IV - supervisionar o desenvolvimento da medida; e de cumprimento de medida.
V - avaliar, com o orientador, a evolução do Além disso, prevê o Sinase que as unidades de
cumprimento da medida e, se necessário, propor à internação deverão assegurar espaços:
autoridade judiciária sua substituição, suspensão ou • para a realização de refeições;
extinção.
• atendimento técnico individual e em grupo;
Parágrafo único. O rol de orientadores
• para atividades coletivas e/ou espaço de estudos;
credenciados deverá ser comunicado, semestralmente, à
autoridade judiciária e ao Ministério Público. • espaço para o setor administrativo e técnico;
Art. 14. Incumbe ainda à direção do programa de • adequados para as visitas íntima e familiar;
medida de prestação de serviços à comunidade • para atendimento de saúde/ambulatório;
selecionar e credenciar entidades assistenciais, hospitais, • salas de aulas apropriadas;
escolas ou outros estabelecimentos congêneres, bem • para prática de esportes e atividades de lazer e
como os programas comunitários ou governamentais, de cultura;
acordo com o perfil do socioeducando e o ambiente no
qual a medida será cumprida. • para profissionalização.
Parágrafo único. Se o Ministério Público impugnar A previsão legal pretende distinguir o atendimento
o credenciamento, ou a autoridade judiciária considerá-lo socioeducativo do prisional, evitando que sejam
inadequado, instaurará incidente de impugnação, com a criados grandes conglomerados envolvendo presídios,
aplicação subsidiária do procedimento de apuração de penitenciárias e centros de internação ou
irregularidade em entidade de atendimento semiliberdade, o que pode ocasionar a adoção de
o
regulamentado na Lei n 8.069, de 13 de julho de 1990 metodologias similares para públicos distintos.

www.editoradince.com - Acesse e veja se há novidades a respeito deste material – CUIDADO: cópia é crime.
LEGISLAÇÃO ESPECIAL E REGIMENTO INTERNO DAS UNIDADES DO SEAS 7
o
O art. 125 da Lei nº 8.069/90 (Estatuto da Criança e § 5 O acompanhamento tem por objetivo verificar
do Adolescente) já estabelecia que é dever do Estado o cumprimento das metas dos Planos de Atendimento
zelar pela integridade física e mental dos internos, Socioeducativo.
cabendo-lhe, de ofício e imediatamente, adotar as Art. 20. O Sistema Nacional de Avaliação e
medidas adequadas de contenção e segurança. Acompanhamento da Gestão do Atendimento
Socioeducativo assegurará, na metodologia a ser
Qualificação mínima do dirigente do programa de
empregada:
semiliberdade ou de internação:
I - a realização da autoavaliação dos gestores e
Art. 17. Para o exercício da função de dirigente de
das instituições de atendimento;
programa de atendimento em regime de semiliberdade ou
de internação, além dos requisitos específicos previstos II - a avaliação institucional externa, contemplando
no respectivo programa de atendimento, é necessário: a análise global e integrada das instalações físicas,
relações institucionais, compromisso social, atividades e
I - formação de nível superior compatível com a
finalidades das instituições de atendimento e seus
natureza da função;
programas;
II - comprovada experiência no trabalho com
III - o respeito à identidade e à diversidade de
adolescentes de, no mínimo, 2 (dois) anos; e
entidades e programas;
III - reputação ilibada.
IV - a participação do corpo de funcionários das
CAPÍTULO V entidades de atendimento e dos Conselhos Tutelares da
DA AVALIAÇÃO E ACOMPANHAMENTO DA GESTÃO área de atuação da entidade avaliada; e
DO ATENDIMENTO SOCIOEDUCATIVO V - o caráter público de todos os procedimentos,
Art. 18. A União, em articulação com os Estados, o dados e resultados dos processos avaliativos.
Distrito Federal e os Municípios, realizará avaliações Art. 21. A avaliação será coordenada por uma
periódicas da implementação dos Planos de comissão permanente e realizada por comissões
Atendimento Socioeducativo em intervalos não temporárias, essas compostas, no mínimo, por 3 (três)
superiores a 3 (três) anos. especialistas com reconhecida atuação na área temática
o
§ 1 O objetivo da avaliação é verificar o e definidas na forma do regulamento.
cumprimento das metas estabelecidas e elaborar Parágrafo único. É vedado à comissão permanente
recomendações aos gestores e operadores dos designar avaliadores:
Sistemas.
o I - que sejam titulares ou servidores dos órgãos
§ 2 O processo de avaliação deverá contar com a gestores avaliados ou funcionários das entidades
participação de representantes do Poder Judiciário, do avaliadas;
Ministério Público, da Defensoria Pública e dos Conselhos
II - que tenham relação de parentesco até o
Tutelares, na forma a ser definida em regulamento. o
o 3 grau com titulares ou servidores dos órgãos gestores
§ 3 A primeira avaliação do Plano Nacional de avaliados e/ou funcionários das entidades avaliadas; e
Atendimento Socioeducativo realizar-se-á no terceiro
III - que estejam respondendo a processos
ano de vigência desta Lei, cabendo ao Poder Legislativo
criminais.
federal acompanhar o trabalho por meio de suas
comissões temáticas pertinentes. Art. 22. A avaliação da gestão terá por objetivo:
Art. 19. É instituído o Sistema Nacional de I - verificar se o planejamento orçamentário e sua
Avaliação e Acompanhamento do Atendimento execução se processam de forma compatível com as
Socioeducativo, com os seguintes objetivos: necessidades do respectivo Sistema de Atendimento
Socioeducativo;
I - contribuir para a organização da rede de
atendimento socioeducativo; II - verificar a manutenção do fluxo financeiro,
considerando as necessidades operacionais do
II - assegurar conhecimento rigoroso sobre as
atendimento socioeducativo, as normas de referência e as
ações do atendimento socioeducativo e seus resultados;
condições previstas nos instrumentos jurídicos celebrados
III - promover a melhora da qualidade da gestão e entre os órgãos gestores e as entidades de atendimento;
do atendimento socioeducativo; e
III - verificar a implementação de todos os demais
IV - disponibilizar informações sobre o atendimento compromissos assumidos por ocasião da celebração dos
socioeducativo. instrumentos jurídicos relativos ao atendimento
o
§ 1 A avaliação abrangerá, no mínimo, a gestão, socioeducativo; e
as entidades de atendimento, os programas e os IV - a articulação interinstitucional e intersetorial
resultados da execução das medidas socioeducativas. das políticas.
o
§ 2 Ao final da avaliação, será elaborado relatório Art. 23. A avaliação das entidades terá por objetivo
contendo histórico e diagnóstico da situação, as identificar o perfil e o impacto de sua atuação, por meio de
recomendações e os prazos para que essas sejam suas atividades, programas e projetos, considerando as
cumpridas, além de outros elementos a serem definidos diferentes dimensões institucionais e, entre elas,
em regulamento. obrigatoriamente, as seguintes:
o
§ 3 O relatório da avaliação deverá ser I - o plano de desenvolvimento institucional;
encaminhado aos respectivos Conselhos de Direitos,
II - a responsabilidade social, considerada
Conselhos Tutelares e ao Ministério Público.
o especialmente sua contribuição para a inclusão social e o
§ 4 Os gestores e entidades têm o dever de desenvolvimento socioeconômico do adolescente e de
colaborar com o processo de avaliação, facilitando o sua família;
acesso às suas instalações, à documentação e a todos os
III - a comunicação e o intercâmbio com a
elementos necessários ao seu efetivo cumprimento.
sociedade;

www.editoradince.com - Acesse e veja se há novidades a respeito deste material – CUIDADO: cópia é crime.
8 LEGISLAÇÃO ESPECIAL E REGIMENTO INTERNO DAS UNIDADES DO SEAS
IV - as políticas de pessoal quanto à qualificação, I - gestores, operadores e seus prepostos e
aperfeiçoamento, desenvolvimento profissional e entidades governamentais às medidas previstas no inciso
o o
condições de trabalho; I e no § 1 do art. 97 da Lei n 8.069, de 13 de julho de
V - a adequação da infraestrutura física às normas 1990 (Estatuto da Criança e do Adolescente); e
de referência; II - entidades não governamentais, seus gestores,
VI - o planejamento e a autoavaliação quanto aos operadores e prepostos às medidas previstas no inciso
o o
processos, resultados, eficiência e eficácia do projeto II e no § 1 do art. 97 da Lei n 8.069, de 13 de julho de
pedagógico e da proposta socioeducativa; 1990 (Estatuto da Criança e do Adolescente).
VII - as políticas de atendimento para os Parágrafo único. A aplicação das medidas
adolescentes e suas famílias; previstas neste artigo dar-se-á a partir da análise de
relatório circunstanciado elaborado após as avaliações,
VIII - a atenção integral à saúde dos adolescentes
sem prejuízo do que determinam os arts.
em conformidade com as diretrizes do art. 60 desta Lei; e
191 a 197, 225 a 227, 230 a 236, 243 e 245 a 247 da Lei
IX - a sustentabilidade financeira. o
n 8.069, de 13 de julho de 1990 (Estatuto da Criança e
Art. 24. A avaliação dos programas terá por do Adolescente).
objetivo verificar, no mínimo, o atendimento ao que
Responsabilização dos gestores, operadores e
determinam os arts. 94, 100, 117, 119, 120, 123 e 124 da
o entidades de atendimento:
Lei n 8.069, de 13 de julho de 1990 (Estatuto da Criança
e do Adolescente). No caso do desrespeito, mesmo que parcial, ou do
não cumprimento integral às diretrizes e
Art. 25. A avaliação dos resultados da execução
de medida socioeducativa terá por objetivo, no mínimo: determinações da Lei n. 12.594/2012, haverá
responsabilização dos envolvidos:
I - verificar a situação do adolescente após
cumprimento da medida socioeducativa, tomando por  Gestores, operadores e seus prepostos e
base suas perspectivas educacionais, sociais, entidades governamentais: Estarão sujeitos às medidas
profissionais e familiares; e previstas no inciso I e no § 1º do art. 97 do ECA
(advertência, afastamento de seus dirigentes,
II - verificar reincidência de prática de ato fechamento de unidade, interdição de programa,
infracional. suspensão das atividades, dissolução da entidade).
Art. 26. Os resultados da avaliação serão
 Entidades não governamentais, seus gestores,
utilizados para:
operadores e prepostos: Estarão sujeitos às medidas
I - planejamento de metas e eleição de prioridades previstas no inciso II e no § 1º do art. 97 do ECA
do Sistema de Atendimento Socioeducativo e seu (advertência, suspensão do repasse de verbas públicas,
financiamento; interdição de unidades, suspensão de programa,
II - reestruturação e/ou ampliação da rede de cassação do registro, suspensão das atividades,
atendimento socioeducativo, de acordo com as dissolução da entidade).
necessidades diagnosticadas; PRESTE ATENÇÃO: Todos aqueles que, mesmo
III - adequação dos objetivos e da natureza do não sendo agentes públicos, induzam ou concorram,
atendimento socioeducativo prestado pelas entidades direta ou indiretamente, para o não cumprimento da
avaliadas; Lei 12.594/2012, poderão responder por improbidade
IV - celebração de instrumentos de cooperação administrativa.
com vistas à correção de problemas diagnosticados na Art. 29. Àqueles que, mesmo não sendo agentes
avaliação; públicos, induzam ou concorram, sob qualquer forma,
V - reforço de financiamento para fortalecer a rede direta ou indireta, para o não cumprimento desta Lei,
de atendimento socioeducativo; aplicam-se, no que couber, as penalidades dispostas na
o
VI - melhorar e ampliar a capacitação dos Lei n 8.429, de 2 de junho de 1992, que dispõe sobre as
operadores do Sistema de Atendimento Socioeducativo; sanções aplicáveis aos agentes públicos nos casos de
e enriquecimento ilícito no exercício de mandato, cargo,
o
VII - os efeitos do art. 95 da Lei n 8.069, de 13 de emprego ou função na administração pública direta,
julho de 1990 (Estatuto da Criança e do Adolescente). indireta ou fundacional e dá outras providências (Lei de
Improbidade Administrativa).
Parágrafo único. As recomendações originadas da
avaliação deverão indicar prazo para seu cumprimento CAPÍTULO VII
por parte das entidades de atendimento e dos gestores DO FINANCIAMENTO E DAS PRIORIDADES
avaliados, ao fim do qual estarão sujeitos às medidas Art. 30. O Sinase será cofinanciado com recursos
previstas no art. 28 desta Lei. dos orçamentos fiscal e da seguridade social, além de
Art. 27. As informações produzidas a partir do outras fontes.
o
Sistema Nacional de Informações sobre Atendimento § 1 (VETADO).
Socioeducativo serão utilizadas para subsidiar a o
§ 2 Os entes federados que tenham instituído
avaliação, o acompanhamento, a gestão e o seus sistemas de atendimento socioeducativo terão
financiamento dos Sistemas Nacional, Distrital, Estaduais acesso aos recursos na forma de transferência adotada
e Municipais de Atendimento Socioeducativo. pelos órgãos integrantes do Sinase.
CAPÍTULO VI o
§ 3 Os entes federados beneficiados com
DA RESPONSABILIZAÇÃO DOS GESTORES, recursos dos orçamentos dos órgãos responsáveis pelas
OPERADORES E ENTIDADES DE ATENDIMENTO políticas integrantes do Sinase, ou de outras fontes, estão
Art. 28. No caso do desrespeito, mesmo que sujeitos às normas e procedimentos de monitoramento
parcial, ou do não cumprimento integral às diretrizes e estabelecidos pelas instâncias dos órgãos das políticas
determinações desta Lei, em todas as esferas, são setoriais envolvidas, sem prejuízo do disposto nos incisos
sujeitos:

www.editoradince.com - Acesse e veja se há novidades a respeito deste material – CUIDADO: cópia é crime.
LEGISLAÇÃO ESPECIAL E REGIMENTO INTERNO DAS UNIDADES DO SEAS 9
o o
IX e X do art. 4 , nos incisos V e VI do art. 5 e no art. III - o ente federado tenha assinado o Plano de Metas
o
6 desta Lei. Compromisso Todos pela Educação e elaborado o
Art. 31. Os Conselhos de Direitos, nas 3 (três) respectivo Plano de Ações Articuladas (PAR).” (NR)
esferas de governo, definirão, anualmente, o percentual TÍTULO II
de recursos dos Fundos dos Direitos da Criança e do DA EXECUÇÃO DAS MEDIDAS SOCIOEDUCATIVAS
Adolescente a serem aplicados no financiamento das
CAPÍTULO I
ações previstas nesta Lei, em especial para capacitação,
sistemas de informação e de avaliação. DISPOSIÇÕES GERAIS
Parágrafo único. Os entes federados beneficiados Princípios que regem a execução das medidas
com recursos do Fundo dos Direitos da Criança e do socioeducativas. MUITA ATENÇÃO
Adolescente para ações de atendimento socioeducativo Art. 35. A execução das medidas socioeducativas
prestarão informações sobre o desempenho dessas reger-se-á pelos seguintes princípios:
ações por meio do Sistema de Informações sobre I - legalidade, não podendo o adolescente receber
Atendimento Socioeducativo. tratamento mais gravoso do que o conferido ao adulto;
o
Art. 32. A Lei n 7.560, de 19 de dezembro de II - excepcionalidade da intervenção judicial e da
1986, passa a vigorar com as seguintes alterações: imposição de medidas, favorecendo-se meios de
o
“Art. 5 Os recursos do Funad serão destinados: autocomposição de conflitos;
....................... III - prioridade a práticas ou medidas que sejam
X - às entidades governamentais e não governamentais restaurativas e, sempre que possível, atendam às
integrantes do Sistema Nacional de Atendimento necessidades das vítimas;
Socioeducativo (Sinase). IV - proporcionalidade em relação à ofensa
..........................” (NR) cometida;
o V - brevidade da medida em resposta ao ato
“Art. 5 -A. A Secretaria Nacional de Políticas sobre
Drogas (Senad), órgão gestor do Fundo Nacional cometido, em especial o respeito ao que dispõe o art. 122
o
Antidrogas (Funad), poderá financiar projetos das da Lei n 8.069, de 13 de julho de 1990 (Estatuto da
entidades do Sinase desde que: Criança e do Adolescente);
I - o ente federado de vinculação da entidade que solicita VI - individualização, considerando-se a idade,
o recurso possua o respectivo Plano de Atendimento capacidades e circunstâncias pessoais do adolescente;
Socioeducativo aprovado; VII - mínima intervenção, restrita ao necessário
II - as entidades governamentais e não governamentais para a realização dos objetivos da medida;
integrantes do Sinase que solicitem recursos tenham VIII - não discriminação do adolescente,
participado da avaliação nacional do atendimento notadamente em razão de etnia, gênero, nacionalidade,
socioeducativo; classe social, orientação religiosa, política ou sexual, ou
III - o projeto apresentado esteja de acordo com os associação ou pertencimento a qualquer minoria
pressupostos da Política Nacional sobre Drogas e ou status; e
legislação específica.” IX - fortalecimento dos vínculos familiares e
o comunitários no processo socioeducativo.
Art. 33. A Lei n 7.998, de 11 de janeiro de 1990,
passa a vigorar acrescida do seguinte art. 19-A: CAPÍTULO II
“Art. 19-A. O Codefat poderá priorizar projetos das DOS PROCEDIMENTOS
entidades integrantes do Sistema Nacional de Art. 36. A competência para jurisdicionar a
Atendimento Socioeducativo (Sinase) desde que: execução das medidas socioeducativas segue o
o
I - o ente federado de vinculação da entidade que solicita determinado pelo art. 146 da Lei n 8.069, de 13 de julho
o recurso possua o respectivo Plano de Atendimento de 1990 (Estatuto da Criança e do Adolescente).
Socioeducativo aprovado; Art. 37. A defesa e o Ministério Público intervirão,
II - as entidades governamentais e não governamentais sob pena de nulidade, no procedimento judicial de
integrantes do Sinase que solicitem recursos tenham se execução de medida socioeducativa, asseguradas aos
o
submetido à avaliação nacional do atendimento seus membros as prerrogativas previstas na Lei n 8.069,
socioeducativo.” de 13 de julho de 1990 (Estatuto da Criança e do
o o Adolescente), podendo requerer as providências
Art. 34. O art. 2 da Lei n 5.537, de 21 de
novembro de 1968, passa a vigorar acrescido do seguinte necessárias para adequar a execução aos ditames legais
o e regulamentares.
§3 :
o Art. 38. As medidas de proteção, de advertência e
“Art. 2 ...............
...................... de reparação do dano, quando aplicadas de forma
o o isolada, serão executadas nos próprios autos do processo
§ 3 O fundo de que trata o art. 1 poderá financiar, na de conhecimento, respeitado o disposto nos arts.
forma das resoluções de seu conselho deliberativo, o
143 e 144 da Lei n 8.069, de 13 de julho de 1990
programas e projetos de educação básica relativos ao (Estatuto da Criança e do Adolescente).
Sistema Nacional de Atendimento Socioeducativo
Art. 39. Para aplicação das medidas
(Sinase) desde que:
socioeducativas de prestação de serviços à comunidade,
I - o ente federado que solicitar o recurso possua o liberdade assistida, semiliberdade ou internação, será
respectivo Plano de Atendimento Socioeducativo constituído processo de execução para cada adolescente,
aprovado; respeitado o disposto nos arts. 143 e 144 da Lei nº 8.069,
II - as entidades de atendimento vinculadas ao ente de 13 de julho de 1990 (Estatuto da Criança e do
federado que solicitar o recurso tenham se submetido à Adolescente), e com autuação das seguintes peças:
avaliação nacional do atendimento socioeducativo; e

www.editoradince.com - Acesse e veja se há novidades a respeito deste material – CUIDADO: cópia é crime.
10 LEGISLAÇÃO ESPECIAL E REGIMENTO INTERNO DAS UNIDADES DO SEAS
I - documentos de caráter pessoal do adolescente a autoridade judiciária, se necessário, designar audiência,
existentes no processo de conhecimento, especialmente no prazo máximo de 10 (dez) dias, cientificando o
os que comprovem sua idade; e defensor, o Ministério Público, a direção do programa de
II - as indicadas pela autoridade judiciária, sempre atendimento, o adolescente e seus pais ou responsável.
o
que houver necessidade e, obrigatoriamente: § 1 A audiência será instruída com o relatório da
a) cópia da representação; equipe técnica do programa de atendimento sobre a
evolução do plano de que trata o art. 52 desta Lei e com
b) cópia da certidão de antecedentes;
qualquer outro parecer técnico requerido pelas partes e
c) cópia da sentença ou acórdão; e deferido pela autoridade judiciária.
d) cópia de estudos técnicos realizados durante a
Art. 52. O cumprimento das medidas
fase de conhecimento.
socioeducativas, em regime de prestação de serviços à
Parágrafo único. Procedimento idêntico será comunidade, liberdade assistida, semiliberdade ou
observado na hipótese de medida aplicada em sede de internação, dependerá de Plano Individual de
remissão, como forma de suspensão do processo. Atendimento (PIA), instrumento de previsão, registro
Autoridade judiciária competente para o processo e gestão das atividades a serem desenvolvidas com o
de execução: adolescente.
o
Juiz da Infância e da Juventude (ou o juiz que § 2 A gravidade do ato infracional, os
exerce essa função, na forma da lei de organização antecedentes e o tempo de duração da medida não são
judiciária local). fatores que, por si, justifiquem a não substituição da
Art. 40. Autuadas as peças, a autoridade judiciária medida por outra menos grave.
o
encaminhará, imediatamente, cópia integral do expediente § 3 Considera-se mais grave a internação, em
ao órgão gestor do atendimento socioeducativo, relação a todas as demais medidas, e mais grave a
solicitando designação do programa ou da unidade de semiliberdade, em relação às medidas de meio aberto.
cumprimento da medida. Art. 43. A reavaliação da manutenção, da
Art. 41. A autoridade judiciária dará vistas da substituição ou da suspensão das medidas de meio
proposta de plano individual de que trata o art. 53 desta aberto ou de privação da liberdade e do respectivo plano
Lei ao defensor e ao Ministério Público pelo prazo individual pode ser solicitada a qualquer tempo, a pedido
sucessivo de 3 (três) dias, contados do recebimento da da direção do programa de atendimento, do defensor, do
proposta encaminhada pela direção do programa de Ministério Público, do adolescente, de seus pais ou
atendimento. responsável.
Art. 53. O PIA (Plano de Proposta Individual) Reavaliação solicitada:
será elaborado sob a responsabilidade da equipe Além da reavaliação semestral obrigatória, a
técnica do respectivo programa de atendimento, com reavaliação da manutenção, da substituição ou da
a participação efetiva do adolescente e de sua família, suspensão da medida imposta e do respectivo PIA
representada por seus pais ou responsável. pode ser solicitada ao juiz, a qualquer tempo, a pedido
o
§ 1 O defensor e o Ministério Público poderão dos seguintes legitimados:
requerer, e o Juiz da Execução poderá determinar, de  direção do programa de atendimento;
ofício, a realização de qualquer avaliação ou perícia que  defensor;
entenderem necessárias para complementação do plano
 Ministério Público;
individual.
 pais ou responsáveis;
Participação obrigatória da defesa e do MP:
A defesa e o Ministério Público intervirão, sob  o próprio adolescente.
o
pena de nulidade, no procedimento judicial de § 1 Justifica o pedido de reavaliação, entre
execução de medida socioeducativa. A defesa e o MP outros motivos:
poderão requerer as providências necessárias para I - o desempenho adequado do adolescente com
adequar a execução das medidas aos ditames legais e base no seu plano de atendimento individual, antes do
regulamentares, sendo-lhes asseguradas as prazo da reavaliação obrigatória;
prerrogativas previstas no ECA.
o II - a inadaptação do adolescente ao programa e
§ 2 A impugnação ou complementação do plano o reiterado descumprimento das atividades do plano
individual, requerida pelo defensor ou pelo Ministério individual; e
Público, deverá ser fundamentada, podendo a autoridade
III - a necessidade de modificação das
judiciária indeferi-la, se entender insuficiente a motivação.
o atividades do plano individual que importem em maior
§ 3 Admitida a impugnação, ou se entender que o restrição da liberdade do adolescente.
plano é inadequado, a autoridade judiciária designará, se o
§ 2 A autoridade judiciária poderá indeferir o
necessário, audiência da qual cientificará o defensor, o
pedido, de pronto, se entender insuficiente a motivação.
Ministério Público, a direção do programa de atendimento, o
o adolescente e seus pais ou responsável. § 3 Admitido o processamento do pedido, a
o autoridade judiciária, se necessário, designará audiência,
§ 4 A impugnação não suspenderá a execução do o
observando o princípio do § 1 do art. 42 desta Lei.
plano individual, salvo determinação judicial em contrário. o
o § 4 A substituição por medida mais gravosa
§ 5 Findo o prazo sem impugnação, considerar-
somente ocorrerá em situações excepcionais, após o
se-á o plano individual homologado.
devido processo legal, inclusive na hipótese do inciso III
Reavaliação semestral obrigatória: o
do art. 122 da Lei n 8.069, de 13 de julho de 1990
Art. 42. As medidas socioeducativas de liberdade (Estatuto da Criança e do Adolescente), e deve ser:
assistida, de semiliberdade e de internação deverão ser I - fundamentada em parecer técnico;
reavaliadas no máximo a cada 6 (seis) meses, podendo

www.editoradince.com - Acesse e veja se há novidades a respeito deste material – CUIDADO: cópia é crime.
LEGISLAÇÃO ESPECIAL E REGIMENTO INTERNO DAS UNIDADES DO SEAS 11
II - precedida de prévia audiência, e nos termos do privativa de liberdade for substituída por restritiva de
o
§ 1 do art. 42 desta Lei. direitos não haverá extinção obrigatória da medida
socioeducativa. Vale ressaltar que, o simples fato de o
PONTOS DE DESTAQUE:
maior de 18 (dezoito) anos, em cumprimento de
Na reavaliação da medida, a gravidade do ato medida socioeducativa, responder a processo-crime,
infracional praticado, os antecedentes e o tempo de poderá gerar a extinção da execução da medida
duração da medida não são fatores que, por si, sirvam socioeducativa, a critério do juiz da infância e
para o juiz recusar a substituição da medida por outra juventude, que deverá cientificar da decisão o juízo
menos grave (§ 2º do art. 42). criminal competente.
De igual modo, a oferta irregular de programas de
IV - pela condição de doença grave, que torne o
atendimento socioeducativo em meio aberto não
adolescente incapaz de submeter-se ao cumprimento da
poderá ser invocada como motivo para aplicação ou
medida; e
manutenção de medida de privação da liberdade.
V - nas demais hipóteses previstas em lei.
Art. 44. Na hipótese de substituição da medida ou o
§ 1 No caso de o maior de 18 (dezoito) anos, em
modificação das atividades do plano individual, a
cumprimento de medida socioeducativa, responder a
autoridade judiciária remeterá o inteiro teor da decisão à
processo-crime, caberá à autoridade judiciária decidir
direção do programa de atendimento, assim como as
sobre eventual extinção da execução, cientificando da
peças que entender relevantes à nova situação jurídica do
decisão o juízo criminal competente.
adolescente. o
§ 2 Em qualquer caso, o tempo de prisão
Parágrafo único. No caso de a substituição da
cautelar não convertida em pena privativa de
medida importar em vinculação do adolescente a outro
liberdade deve ser descontado do prazo de
programa de atendimento, o plano individual e o histórico
cumprimento da medida socioeducativa.
do cumprimento da medida deverão acompanhar a
transferência. Art. 47. O mandado de busca e apreensão do
adolescente terá vigência máxima de 6 (seis) meses, a
Unificação de medidas socioeducativas:
contar da data da expedição, podendo, se necessário, ser
Art. 45. Se, no transcurso da execução, sobrevier renovado, fundamentadamente.
sentença de aplicação de nova medida, a autoridade
judiciária procederá à unificação, ouvidos, previamente, o Revisão judicial de sanções disciplinares aplicadas
Ministério Público e o defensor, no prazo de 3 (três) dias ao adolescente em cumprimento de medida
sucessivos, decidindo-se em igual prazo. socioeducativa:
o O defensor, o Ministério Público, o adolescente e
§ 1 É vedado à autoridade judiciária determinar
reinício de cumprimento de medida socioeducativa, ou seus pais ou responsável poderão postular revisão
deixar de considerar os prazos máximos, e de liberação judicial de qualquer sanção disciplinar aplicada,
o podendo a autoridade judiciária suspender a execução
compulsória previstos na Lei n 8.069, de 13 de julho de
1990 (Estatuto da Criança e do Adolescente), excetuada a da sanção até decisão final do incidente.
hipótese de medida aplicada por ato infracional praticado Postulada a revisão após ouvida a autoridade
durante a execução. colegiada que aplicou a sanção e havendo provas a
o produzir em audiência, o magistrado designará a
§ 2 É vedado à autoridade judiciária aplicar nova
medida de internação, por atos infracionais praticados audiência, que será instruída com relatório da equipe
anteriormente, a adolescente que já tenha concluído técnica.
cumprimento de medida socioeducativa dessa natureza, Art. 48. O defensor, o Ministério Público, o
ou que tenha sido transferido para cumprimento de adolescente e seus pais ou responsável poderão postular
medida menos rigorosa, sendo tais atos absorvidos por revisão judicial de qualquer sanção disciplinar aplicada,
aqueles aos quais se impôs a medida socioeducativa podendo a autoridade judiciária suspender a execução da
extrema. sanção até decisão final do incidente.
o
Exemplo: Paulo, adolescente, praticou um ato § 1 Postulada a revisão após ouvida a autoridade
infracional equiparado a roubo em 2009 e outro colegiada que aplicou a sanção e havendo provas a
equiparado a tráfico de drogas em 2010. Foi julgado produzir em audiência, procederá o magistrado na forma
o
ainda em 2010 pelo roubo, tendo recebido medida do § 1 do art. 42 desta Lei.
socioeducativa de internação. Após 6 meses internado, o
§ 2 É vedada a aplicação de sanção
tal medida foi substituída por semiliberdade. Em 2011, disciplinar de isolamento a adolescente interno,
é julgado pelo tráfico. Nesse caso, Paulo não poderá EXCETO seja essa imprescindível para garantia da
cumprir medida de internação pelo tráfico por conta segurança de outros internos ou do próprio adolescente a
do § 2º do art. 45 da nova Lei. quem seja imposta a sanção, sendo necessária ainda
Art. 46. A medida socioeducativa será declarada comunicação ao defensor, ao Ministério Público e à
extinta: autoridade judiciária em até 24 (vinte e quatro) horas.
I - pela morte do adolescente; PONTO DE DESTAQUE:
II - pela realização de sua finalidade; O adolescente que esteja em regime de internação
III - pela aplicação de pena privativa de liberdade, poderá ser punido com o isolamento caso tenha
a ser cumprida em regime fechado ou semiaberto, em praticado falta grave?
execução provisória ou definitiva; Como regra, NÃO. Segundo o § 2º do art. 48 da
Ex.: a pessoa já completou 18 anos e ainda está Lei n. 12.594/2012, como regra, é vedada a aplicação
cumprindo medida socioeducativa quando é de sanção disciplinar de isolamento a adolescente
condenada à pena privativa de liberdade; se a pena interno.
recebida for apenas multa, se for pena privativa de Exceção: será possível o isolamento do
liberdade no regime aberto, ou então se a pena adolescente interno caso isso seja imprescindível para

www.editoradince.com - Acesse e veja se há novidades a respeito deste material – CUIDADO: cópia é crime.
12 LEGISLAÇÃO ESPECIAL E REGIMENTO INTERNO DAS UNIDADES DO SEAS
a garantia da segurança de outros internos ou do devidamente comprovados, de pai, mãe, filho, cônjuge,
próprio adolescente a quem seja imposta a sanção. companheiro ou irmão, com imediata comunicação ao
Nessa hipótese, será necessária a comunicação ao juízo competente.
defensor do adolescente, ao Ministério Público e à
ECA: Art. 121. A internação constitui medida
autoridade judiciária em até 24 horas.
privativa da liberdade, sujeita aos princípios de
CAPÍTULO III brevidade, excepcionalidade e respeito à condição
DOS DIREITOS INDIVIDUAIS peculiar de pessoa em desenvolvimento.
Art. 49. São direitos do adolescente submetido ao § 1º Será permitida a realização de atividades
cumprimento de medida socioeducativa, sem prejuízo de externas, a critério da equipe técnica da entidade,
outros previstos em lei: salvo expressa determinação judicial em contrário.
I - ser acompanhado por seus pais ou responsável Permissão de saída: A direção do programa de
e por seu defensor, em qualquer fase do procedimento execução de medida de privação da liberdade poderá
administrativo ou judicial; autorizar a saída, monitorada, do adolescente nos
casos de tratamento médico, doença grave ou
II - ser incluído em programa de meio aberto
falecimento, devidamente comprovados, de pai, mãe,
quando inexistir vaga para o cumprimento de medida de
filho, cônjuge, companheiro ou irmão, com imediata
privação da liberdade, exceto nos casos de ato infracional
comunicação ao juízo competente.
cometido mediante grave ameaça ou violência à pessoa,
quando o adolescente deverá ser internado em Unidade Registre-se, novamente, que se trata de saída
mais próxima de seu local de residência; autorizada pela própria direção do programa, não
sendo necessária autorização prévia do juiz, mas tão
III - ser respeitado em sua personalidade,
somente comunicação.
intimidade, liberdade de pensamento e religião e em todos
os direitos não expressamente limitados na sentença; Autorização para realização de atividades
externas:
IV - peticionar, por escrito ou verbalmente,
diretamente a qualquer autoridade ou órgão público, Além das hipóteses acima mencionadas de
devendo, obrigatoriamente, ser respondido em até 15 permissão de saída, será permitida ao adolescente que
(quinze) dias; esteja em regime de internação a realização de
atividades externas, a critério da equipe técnica da
V - ser informado, inclusive por escrito, das normas
entidade, salvo expressa determinação judicial em
de organização e funcionamento do programa de
contrário.
atendimento e também das previsões de natureza
disciplinar; Veja que não é necessária autorização judicial
prévia, mas tão somente ausência de determinação
VI - receber, sempre que solicitar, informações
judicial em contrário.
sobre a evolução de seu plano individual, participando,
obrigatoriamente, de sua elaboração e, se for o caso, Vale ressaltar, no entanto, que esta autorização
reavaliação; para saída poderá ser revista a qualquer tempo pela
autoridade judiciária.
VII - receber assistência integral à sua saúde,
conforme o disposto no art. 60 desta Lei; e Art. 51. A decisão judicial relativa à execução de
VIII - ter atendimento garantido em creche e pré- medida socioeducativa será proferida após manifestação
escola aos filhos de 0 (zero) a 5 (cinco) anos. do defensor e do Ministério Público.
o
§ 1 As garantias processuais destinadas a CAPÍTULO IV
adolescente autor de ato infracional previstas na Lei DO PLANO INDIVIDUAL DE ATENDIMENTO (PIA)
o
n 8.069, de 13 de julho de 1990 (Estatuto da Criança e Art. 52. O cumprimento das medidas
do Adolescente), aplicam-se integralmente na execução socioeducativas, em regime de prestação de serviços à
das medidas socioeducativas, inclusive no âmbito comunidade, liberdade assistida, semiliberdade ou
administrativo. internação, dependerá de Plano Individual de
o
§ 2 A oferta irregular de programas de Atendimento (PIA), instrumento de previsão, registro e
atendimento socioeducativo em meio aberto não poderá gestão das atividades a serem desenvolvidas com o
ser invocada como motivo para aplicação ou manutenção adolescente.
de medida de privação da liberdade. Parágrafo único. O PIA deverá contemplar a
PONTO DE DESTAQUE: participação dos pais ou responsáveis, os quais têm o
dever de contribuir com o processo ressocializador do
Adolescente infrator recebe, na sentença, medida
adolescente, sendo esses passíveis de responsabilização
de internação. o
administrativa, nos termos do art. 249 da Lei n 8.069, de
Ocorre que ele reside no interior do Estado, onde 13 de julho de 1990 (Estatuto da Criança e do
não existe Unidade de internação. O que acontece? Adolescente), civil e criminal.
Este adolescente deverá ser incluído em programa Art. 53. O PIA será elaborado sob a
de meio aberto (semiliberdade, p. ex.), exceto se o ato responsabilidade da equipe técnica do respectivo
infracional por ele perpetrado foi cometido mediante programa de atendimento, com a participação efetiva do
grave ameaça ou violência à pessoa, hipótese na qual o adolescente e de sua família, representada por seus pais
adolescente deverá ser internado na Unidade mais ou responsável.
próxima de seu local de residência (na capital, p. ex.).
o PIA - Plano de Proposta Individual
Art. 50. Sem prejuízo do disposto no § 1 do art.
o Vista da proposta de PIA ao defensor e ao
121 da Lei n 8.069, de 13 de julho de 1990 (Estatuto da
MP:
Criança e do Adolescente), a direção do programa de
execução de medida de privação da liberdade poderá O juiz dará vistas da proposta de PIA ao
autorizar a saída, monitorada, do adolescente nos casos defensor e ao Ministério Público pelo prazo
de tratamento médico, doença grave ou falecimento, sucessivo de 3 dias, contados do recebimento da

www.editoradince.com - Acesse e veja se há novidades a respeito deste material – CUIDADO: cópia é crime.
LEGISLAÇÃO ESPECIAL E REGIMENTO INTERNO DAS UNIDADES DO SEAS 13
proposta encaminhada pela direção do programa de Art. 59. O acesso ao plano individual será restrito
atendimento. aos servidores do respectivo programa de atendimento,
Prazo de elaboração do PIA: ao adolescente e a seus pais ou responsável, ao
Ministério Público e ao defensor, exceto expressa
No caso de semiliberdade ou internação: até 45
autorização judicial.
dias do ingresso do adolescente no programa. No
caso de prestação de serviços à comunidade e CAPÍTULO V
liberdade assistida: até 15 dias do ingresso. DA ATENÇÃO INTEGRAL À SAÚDE DE
ADOLESCENTE EM CUMPRIMENTO DE MEDIDA
Conteúdo mínimo do PIA:
SOCIOEDUCATIVA
Art. 54. Constarão do plano individual, no mínimo:
Seção I
I - os resultados da avaliação interdisciplinar;
Disposições Gerais
II - os objetivos declarados pelo adolescente;
Art. 60. A atenção integral à saúde do adolescente
III - a previsão de suas atividades de integração no Sistema de Atendimento Socioeducativo seguirá as
social e/ou capacitação profissional; seguintes diretrizes:
IV - atividades de integração e apoio à família; I - previsão, nos planos de atendimento
V - formas de participação da família para efetivo socioeducativo, em todas as esferas, da implantação de
cumprimento do plano individual; e ações de promoção da saúde, com o objetivo de integrar
VI - as medidas específicas de atenção à sua as ações socioeducativas, estimulando a autonomia, a
saúde. melhoria das relações interpessoais e o fortalecimento de
Art. 55. Para o cumprimento das medidas de redes de apoio aos adolescentes e suas famílias;
semiliberdade ou de internação, o plano individual II - inclusão de ações e serviços para a promoção,
conterá, ainda: proteção, prevenção de agravos e doenças e recuperação
I - a designação do programa de atendimento mais da saúde;
adequado para o cumprimento da medida; III - cuidados especiais em saúde mental, incluindo
II - a definição das atividades internas e externas, os relacionados ao uso de álcool e outras substâncias
individuais ou coletivas, das quais o adolescente poderá psicoativas, e atenção aos adolescentes com
participar; e deficiências;
III - a fixação das metas para o alcance de IV - disponibilização de ações de atenção à saúde
desenvolvimento de atividades externas. sexual e reprodutiva e à prevenção de doenças
sexualmente transmissíveis;
Parágrafo único. O PIA será elaborado no prazo de
até 45 (quarenta e cinco) dias da data do ingresso do V - garantia de acesso a todos os níveis de
adolescente no programa de atendimento. atenção à saúde, por meio de referência e
contrarreferência, de acordo com as normas do Sistema
Art. 56. Para o cumprimento das medidas de Único de Saúde (SUS);
prestação de serviços à comunidade e de liberdade
assistida, o PIA será elaborado no prazo de até 15 VI - capacitação das equipes de saúde e dos
(quinze) dias do ingresso do adolescente no programa de profissionais das entidades de atendimento, bem como
atendimento. daqueles que atuam nas unidades de saúde de referência
voltadas às especificidades de saúde dessa população e
Art. 57. Para a elaboração do PIA, a direção do de suas famílias;
respectivo programa de atendimento, pessoalmente ou
por meio de membro da equipe técnica, terá acesso aos VII - inclusão, nos Sistemas de Informação de
autos do procedimento de apuração do ato infracional e Saúde do SUS, bem como no Sistema de Informações
aos dos procedimentos de apuração de outros atos sobre Atendimento Socioeducativo, de dados e
infracionais atribuídos ao mesmo adolescente. indicadores de saúde da população de adolescentes em
o atendimento socioeducativo; e
§ 1 O acesso aos documentos de que trata
o caput deverá ser realizado por funcionário da entidade VIII - estruturação das unidades de internação
de atendimento, devidamente credenciado para tal conforme as normas de referência do SUS e do Sinase,
atividade, ou por membro da direção, em conformidade visando ao atendimento das necessidades de Atenção
com as normas a serem definidas pelo Poder Judiciário, Básica.
de forma a preservar o que determinam os arts. 143 e 144 Art. 61. As entidades que ofereçam programas de
o atendimento socioeducativo em meio aberto e de
da Lei n 8.069, de 13 de julho de 1990 (Estatuto da
Criança e do Adolescente). semiliberdade deverão prestar orientações aos
o
§ 2 A direção poderá requisitar, ainda: socioeducandos sobre o acesso aos serviços e às
unidades do SUS.
I - ao estabelecimento de ensino, o histórico
escolar do adolescente e as anotações sobre o seu Art. 62. As entidades que ofereçam programas de
aproveitamento; privação de liberdade deverão contar com uma equipe
mínima de profissionais de saúde cuja composição esteja
II - os dados sobre o resultado de medida em conformidade com as normas de referência do SUS.
anteriormente aplicada e cumprida em outro programa de
atendimento; e Art. 63. (VETADO).
o
III - os resultados de acompanhamento § 1 O filho de adolescente nascido nos
especializado anterior. estabelecimentos referidos no caput deste artigo não terá
tal informação lançada em seu registro de nascimento.
Art. 58. Por ocasião da reavaliação da medida, é o
obrigatória a apresentação pela direção do programa de § 2 Serão asseguradas as condições necessárias
atendimento de relatório da equipe técnica sobre a para que a adolescente submetida à execução de medida
evolução do adolescente no cumprimento do plano socioeducativa de privação de liberdade permaneça com
individual. o seu filho durante o período de amamentação.

www.editoradince.com - Acesse e veja se há novidades a respeito deste material – CUIDADO: cópia é crime.
14 LEGISLAÇÃO ESPECIAL E REGIMENTO INTERNO DAS UNIDADES DO SEAS
Seção II Art. 70. O regulamento interno estabelecerá as
Do Atendimento a Adolescente com Transtorno hipóteses de proibição da entrada de objetos na unidade
Mental e com Dependência de Álcool e de Substância de internação, vedando o acesso aos seus portadores.
Psicoativa Estrutura da unidade de internação e de
Art. 64. O adolescente em cumprimento de medida semiliberdade:
socioeducativa que apresente indícios de transtorno A estrutura física da unidade deverá ser
mental, de deficiência mental, ou associadas, deverá ser compatível com as normas de referência do Sinase. É
avaliado por equipe técnica multidisciplinar e proibido que a unidade de medida socioeducativa
multissetorial. funcione em espaços contíguos, anexos, ou de
o
§ 1 As competências, a composição e a atuação qualquer outra forma integrados a estabelecimentos
da equipe técnica de que trata o caput deverão seguir, penais (presídios, cadeias etc.).
conjuntamente, as normas de referência do SUS e do PONTO DE DESTAQUE:
Sinase, na forma do regulamento.
o
O adolescente casado ou que viva,
§ 2 A avaliação de que trata o caput subsidiará a comprovadamente, em união estável tem direito à
elaboração e execução da terapêutica a ser adotada, a visita íntima. O visitante será identificado e registrado
qual será incluída no PIA do adolescente, prevendo, se pela direção do programa de atendimento, que emitirá
necessário, ações voltadas para a família. documento de identificação, pessoal e intransferível,
o
§ 3 As informações produzidas na avaliação de específico para a realização da visita íntima.
que trata o caput são consideradas sigilosas.
o
CAPÍTULO VII
§ 4 Excepcionalmente, o juiz poderá suspender a
DOS REGIMES DISCIPLINARES
execução da medida socioeducativa, ouvidos o defensor e
o Ministério Público, com vistas a incluir o adolescente em Art. 71. Todas as entidades de atendimento
programa de atenção integral à saúde mental que melhor socioeducativo deverão, em seus respectivos regimentos,
atenda aos objetivos terapêuticos estabelecidos para o realizar a previsão de regime disciplinar que obedeça aos
seu caso específico. seguintes princípios:
§ 5
o
Suspensa a execução da medida I - tipificação explícita das infrações como leves,
socioeducativa, o juiz designará o responsável por médias e graves e determinação das correspondentes
acompanhar e informar sobre a evolução do atendimento sanções;
ao adolescente. II - exigência da instauração formal de processo
o
§ 6 A suspensão da execução da medida disciplinar para a aplicação de qualquer sanção,
socioeducativa será avaliada, no mínimo, a cada 6 (seis) garantidos a ampla defesa e o contraditório;
meses. III - obrigatoriedade de audiência do
o
§ 7 O tratamento a que se submeterá o socioeducando nos casos em que seja necessária a
o
adolescente deverá observar o previsto na Lei n 10.216, instauração de processo disciplinar;
de 6 de abril de 2001, que dispõe sobre a proteção e os IV - sanção de duração determinada;
direitos das pessoas portadoras de transtornos mentais e V - enumeração das causas ou circunstâncias que
redireciona o modelo assistencial em saúde mental. eximam, atenuem ou agravem a sanção a ser imposta ao
o
§ 8 (VETADO). socioeducando, bem como os requisitos para a extinção
Art. 65. Enquanto não cessada a jurisdição da dessa;
Infância e Juventude, a autoridade judiciária, nas VI - enumeração explícita das garantias de
hipóteses tratadas no art. 64, poderá remeter cópia dos defesa;
autos ao Ministério Público para eventual propositura de VII - garantia de solicitação e rito de apreciação
interdição e outras providências pertinentes. dos recursos cabíveis; e
Art. 66. (VETADO). VIII - apuração da falta disciplinar por comissão
CAPÍTULO VI composta por, no mínimo, 3 (três) integrantes, sendo 1
DAS VISITAS A ADOLESCENTE EM CUMPRIMENTO (um), obrigatoriamente, oriundo da equipe técnica.
DE MEDIDA DE Art. 72. O regime disciplinar é independente da
INTERNAÇÃO responsabilidade civil ou penal que advenha do ato
cometido.
Art. 67. A visita do cônjuge, companheiro, pais ou
responsáveis, parentes e amigos a adolescente a quem Art. 73. Nenhum socioeducando poderá
foi aplicada medida socioeducativa de internação desempenhar função ou tarefa de apuração disciplinar ou
observará dias e horários próprios definidos pela direção aplicação de sanção nas entidades de atendimento
do programa de atendimento. socioeducativo.
Art. 68. É assegurado ao adolescente casado ou Art. 74. Não será aplicada sanção disciplinar sem
que viva, comprovadamente, em união estável o direito à expressa e anterior previsão legal ou regulamentar e o
visita íntima. devido processo administrativo.
Parágrafo único. O visitante será identificado e Art. 75. Não será aplicada sanção disciplinar ao
registrado pela direção do programa de atendimento, que socioeducando que tenha praticado a falta:
emitirá documento de identificação, pessoal e I - por coação irresistível ou por motivo de força
intransferível, específico para a realização da visita maior;
íntima. II - em legítima defesa, própria ou de outrem.
Art. 69. É garantido aos adolescentes em CAPÍTULO VIII
cumprimento de medida socioeducativa de internação o DA CAPACITAÇÃO PARA O TRABALHO
direito de receber visita dos filhos, independentemente da
idade desses.

www.editoradince.com - Acesse e veja se há novidades a respeito deste material – CUIDADO: cópia é crime.
LEGISLAÇÃO ESPECIAL E REGIMENTO INTERNO DAS UNIDADES DO SEAS 15
o o
Art. 76. O art. 2 do Decreto-Lei n 4.048, de 22 de Art. 82. Os Conselhos dos Direitos da Criança e
janeiro de 1942, passa a vigorar acrescido do seguinte § do Adolescente, em todos os níveis federados, com os
o o
1 , renumerando-se o atual parágrafo único para § 2 : órgãos responsáveis pelo sistema de educação pública e
o as entidades de atendimento, deverão, no prazo de 1
“Art. 2 ......................
o (um) ano a partir da publicação desta Lei, garantir a
§ 1 As escolas do Senai poderão ofertar vagas aos
inserção de adolescentes em cumprimento de medida
usuários do Sistema Nacional de Atendimento
socioeducativa na rede pública de educação, em qualquer
Socioeducativo (Sinase) nas condições a serem dispostas
fase do período letivo, contemplando as diversas faixas
em instrumentos de cooperação celebrados entre os
etárias e níveis de instrução.
operadores do Senai e os gestores dos Sistemas de
Atendimento Socioeducativo locais. Art. 83. Os programas de atendimento
o socioeducativo sob a responsabilidade do Poder
§ 2 ............... ” (NR)
o o Judiciário serão, obrigatoriamente, transferidos ao Poder
Art. 77. O art. 3 do Decreto-Lei n 8.621, de 10 de Executivo no prazo máximo de 1 (um) ano a partir da
janeiro de 1946, passa a vigorar acrescido do seguinte § publicação desta Lei e de acordo com a política de oferta
o o
1 , renumerando-se o atual parágrafo único para § 2 : dos programas aqui definidos.
o
“Art. 3 ...................... Art. 84. Os programas de internação e
o
§ 1 As escolas do Senac poderão ofertar vagas aos semiliberdade sob a responsabilidade dos Municípios
usuários do Sistema Nacional de Atendimento serão, obrigatoriamente, transferidos para o Poder
Socioeducativo (Sinase) nas condições a serem dispostas Executivo do respectivo Estado no prazo máximo de 1
em instrumentos de cooperação celebrados entre os (um) ano a partir da publicação desta Lei e de acordo com
operadores do Senac e os gestores dos Sistemas de a política de oferta dos programas aqui definidos.
Atendimento Socioeducativo locais. Art. 85. A não transferência de programas de
o
§ 2 . ........... ” (NR) atendimento para os devidos entes responsáveis, no
o o
Art. 78. O art. 1 da Lei n 8.315, de 23 de prazo determinado nesta Lei, importará na interdição do
dezembro de 1991, passa a vigorar acrescido do seguinte programa e caracterizará ato de improbidade
parágrafo único: administrativa do agente responsável, vedada, ademais,
o
“Art. 1 ...................... ao Poder Judiciário e ao Poder Executivo municipal, ao
final do referido prazo, a realização de despesas para a
Parágrafo único. Os programas de formação profissional sua manutenção.
rural do Senar poderão ofertar vagas aos usuários do
Sistema Nacional de Atendimento Socioeducativo Art. 86. Os arts. 90, 97, 121, 122, 198 e 208 da Lei
o
(Sinase) nas condições a serem dispostas em n 8.069, de 13 de julho de 1990 (Estatuto da Criança e
instrumentos de cooperação celebrados entre os do Adolescente), passam a vigorar com a seguinte
operadores do Senar e os gestores dos Sistemas de redação:
Atendimento Socioeducativo locais.” (NR) “Art. 90. ..................
o o
Art. 79. O art. 3 da Lei n 8.706, de 14 de .....................
setembro de 1993, passa a vigorar acrescido do seguinte V - prestação de serviços à comunidade;
parágrafo único: VI - liberdade assistida;
o
“Art. 3 ............. VII - semiliberdade; e
Parágrafo único. Os programas de formação profissional VIII - internação.
do Senat poderão ofertar vagas aos usuários do Sistema
............................ (NR)
Nacional de Atendimento Socioeducativo (Sinase) nas
condições a serem dispostas em instrumentos de “Art. 97. (VETADO)”
cooperação celebrados entre os operadores do Senat e “Art. 121. .............
os gestores dos Sistemas de Atendimento Socioeducativo ..........................
locais.” (NR) o o
o
§ 7 A determinação judicial mencionada no § 1 poderá
Art. 80. O art. 429 do Decreto-Lei n 5.452, de ser revista a qualquer tempo pela autoridade judiciária.”
o
1 de maio de 1943, passa a vigorar acrescido do (NR)
o
seguinte § 2 :
“Art. 122. .............
“Art. 429. .................
....................
.................... o
o
§ 1 O prazo de internação na hipótese do inciso III deste
§ 2 Os estabelecimentos de que trata o caput ofertarão artigo não poderá ser superior a 3 (três) meses, devendo
vagas de aprendizes a adolescentes usuários do Sistema ser decretada judicialmente após o devido processo legal.
Nacional de Atendimento Socioeducativo (Sinase) nas
........................” (NR)
condições a serem dispostas em instrumentos de
cooperação celebrados entre os estabelecimentos e os “Art. 198. Nos procedimentos afetos à Justiça da Infância
gestores dos Sistemas de Atendimento Socioeducativo e da Juventude, inclusive os relativos à execução das
locais.” (NR) medidas socioeducativas, adotar-se-á o sistema recursal
o
da Lei n 5.869, de 11 de janeiro de 1973 (Código de
TÍTULO III
Processo Civil), com as seguintes adaptações:
DISPOSIÇÕES FINAIS E TRANSITÓRIAS
..................
Art. 81. As entidades que mantenham programas
II - em todos os recursos, salvo nos embargos de
de atendimento têm o prazo de até 6 (seis) meses após a
declaração, o prazo para o Ministério Público e para a
publicação desta Lei para encaminhar ao respectivo
defesa será sempre de 10 (dez) dias;
Conselho Estadual ou Municipal dos Direitos da Criança e
do Adolescente proposta de adequação da sua inscrição, ............................” (NR)
sob pena de interdição. “Art. 208. .......................
........................

www.editoradince.com - Acesse e veja se há novidades a respeito deste material – CUIDADO: cópia é crime.
16 LEGISLAÇÃO ESPECIAL E REGIMENTO INTERNO DAS UNIDADES DO SEAS
X - de programas de atendimento para a execução das concomitantemente com a opção de que trata o caput,
medidas socioeducativas e aplicação de medidas de respeitado o limite previsto no inciso II do art. 260.”
proteção. “Art. 260-B. A doação de que trata o inciso I do art. 260
..............................” (NR) poderá ser deduzida:
Art. 87. A Lei nº 8.069, de 13 de julho de 1990 I - do imposto devido no trimestre, para as pessoas
(Estatuto da Criança e do Adolescente), passa a vigorar jurídicas que apuram o imposto trimestralmente; e
com as seguintes alterações: II - do imposto devido mensalmente e no ajuste anual,
“Art. 260. Os contribuintes poderão efetuar doações aos para as pessoas jurídicas que apuram o imposto
Fundos dos Direitos da Criança e do Adolescente anualmente.
nacional, distrital, estaduais ou municipais, devidamente Parágrafo único. A doação deverá ser efetuada dentro do
comprovadas, sendo essas integralmente deduzidas do período a que se refere a apuração do imposto.”
imposto de renda, obedecidos os seguintes limites:
“Art. 260-C. As doações de que trata o art. 260 desta Lei
I - 1% (um por cento) do imposto sobre a renda devido podem ser efetuadas em espécie ou em bens.
apurado pelas pessoas jurídicas tributadas com base no
Parágrafo único. As doações efetuadas em espécie
lucro real; e
devem ser depositadas em conta específica, em
II - 6% (seis por cento) do imposto sobre a renda apurado instituição financeira pública, vinculadas aos respectivos
pelas pessoas físicas na Declaração de Ajuste Anual, fundos de que trata o art. 260.”
o
observado o disposto no art. 22 da Lei n 9.532, de 10 de
“Art. 260-D. Os órgãos responsáveis pela administração
dezembro de 1997.
das contas dos Fundos dos Direitos da Criança e do
............................ Adolescente nacional, estaduais, distrital e municipais
o o o
§ 5 Observado o disposto no § 4 do art. 3 da Lei devem emitir recibo em favor do doador, assinado por
o
n 9.249, de 26 de dezembro de 1995, a dedução de que pessoa competente e pelo presidente do Conselho
trata o inciso I do caput: correspondente, especificando:
I - será considerada isoladamente, não se submetendo a I - número de ordem;
limite em conjunto com outras deduções do imposto; e II - nome, Cadastro Nacional da Pessoa Jurídica (CNPJ) e
II - não poderá ser computada como despesa operacional endereço do emitente;
na apuração do lucro real.” (NR) III - nome, CNPJ ou Cadastro de Pessoas Físicas (CPF)
“Art. 260-A. A partir do exercício de 2010, ano-calendário do doador;
de 2009, a pessoa física poderá optar pela doação de que IV - data da doação e valor efetivamente recebido; e
trata o inciso II do caput do art. 260 diretamente em sua
V - ano-calendário a que se refere a doação.
Declaração de Ajuste Anual. o
o § 1 O comprovante de que trata o caput deste artigo
§ 1 A doação de que trata o caput poderá ser deduzida
pode ser emitido anualmente, desde que discrimine os
até os seguintes percentuais aplicados sobre o imposto
valores doados mês a mês.
apurado na declaração: o
§ 2 No caso de doação em bens, o comprovante deve
I - (VETADO);
conter a identificação dos bens, mediante descrição em
II - (VETADO); campo próprio ou em relação anexa ao comprovante,
III - 3% (três por cento) a partir do exercício de 2012. informando também se houve avaliação, o nome, CPF ou
o
§ 2 A dedução de que trata o caput: CNPJ e endereço dos avaliadores.”
I - está sujeita ao limite de 6% (seis por cento) do imposto “Art. 260-E. Na hipótese da doação em bens, o doador
sobre a renda apurado na declaração de que trata o inciso deverá:
II do caput do art. 260; I - comprovar a propriedade dos bens, mediante
II - não se aplica à pessoa física que: documentação hábil;
a) utilizar o desconto simplificado; II - baixar os bens doados na declaração de bens e
direitos, quando se tratar de pessoa física, e na
b) apresentar declaração em formulário; ou
escrituração, no caso de pessoa jurídica; e
c) entregar a declaração fora do prazo;
III - considerar como valor dos bens doados:
III - só se aplica às doações em espécie; e
a) para as pessoas físicas, o valor constante da última
IV - não exclui ou reduz outros benefícios ou deduções declaração do imposto de renda, desde que não exceda o
em vigor. valor de mercado;
o
§ 3 O pagamento da doação deve ser efetuado até a b) para as pessoas jurídicas, o valor contábil dos bens.
data de vencimento da primeira quota ou quota única do
Parágrafo único. O preço obtido em caso de leilão não
imposto, observadas instruções específicas da Secretaria
será considerado na determinação do valor dos bens
da Receita Federal do Brasil.
o doados, exceto se o leilão for determinado por autoridade
§ 4 O não pagamento da doação no prazo estabelecido judiciária.”
o
no § 3 implica a glosa definitiva desta parcela de
“Art. 260-F. Os documentos a que se referem os arts.
dedução, ficando a pessoa física obrigada ao
260-D e 260-E devem ser mantidos pelo contribuinte por
recolhimento da diferença de imposto devido apurado na
um prazo de 5 (cinco) anos para fins de comprovação da
Declaração de Ajuste Anual com os acréscimos legais
dedução perante a Receita Federal do Brasil.”
previstos na legislação.
o “Art. 260-G. Os órgãos responsáveis pela administração
§ 5 A pessoa física poderá deduzir do imposto apurado
das contas dos Fundos dos Direitos da Criança e do
na Declaração de Ajuste Anual as doações feitas, no
Adolescente nacional, estaduais, distrital e municipais
respectivo ano-calendário, aos fundos controlados pelos
devem:
Conselhos dos Direitos da Criança e do Adolescente
municipais, distrital, estaduais e nacional I - manter conta bancária específica destinada
exclusivamente a gerir os recursos do Fundo;

www.editoradince.com - Acesse e veja se há novidades a respeito deste material – CUIDADO: cópia é crime.
LEGISLAÇÃO ESPECIAL E REGIMENTO INTERNO DAS UNIDADES DO SEAS 17
II - manter controle das doações recebidas; e DILMA ROUSSEFF
José Eduardo Cardozo
III - informar anualmente à Secretaria da Receita Federal
Guido Mantega
do Brasil as doações recebidas mês a mês, identificando Alexandre Rocha Santos Padilha
os seguintes dados por doador: Miriam Belchior
a) nome, CNPJ ou CPF; Maria do Rosário Nunes
b) valor doado, especificando se a doação foi em espécie
ou em bens.” QUESTÕES DE CONCURSOS
“Art. 260-H. Em caso de descumprimento das obrigações
previstas no art. 260-G, a Secretaria da Receita Federal SJC/SC AS AGENTE DE SEGURANÇA
do Brasil dará conhecimento do fato ao Ministério SOCIOEDUCATIVO/2016
Público.” 01. De acordo com a Lei do SINASE (Lei no
12.594/2012), a execução das medidas
“Art. 260-I. Os Conselhos dos Direitos da Criança e do
Adolescente nacional, estaduais, distrital e municipais socioeducativas reger-se-á pelos seguintes princípios:
divulgarão amplamente à comunidade: Legalidade, não podendo o adolescente receber
tratamento mais gravoso do que o conferido ao adulto.
I - o calendário de suas reuniões; Excepcionalidade da intervenção judicial e da
II - as ações prioritárias para aplicação das políticas de imposição de medidas, favorecendo-se meios de
atendimento à criança e ao adolescente; autocomposição de conflitos, exclusivos aos atos
III - os requisitos para a apresentação de projetos a serem infracionais de natureza leve e sem grave ameaça à
beneficiados com recursos dos Fundos dos Direitos da pessoa. Prioridade a práticas ou medidas que sejam
Criança e do Adolescente nacional, estaduais, distrital ou restaurativas que atendam às necessidades das
municipais; vítimas e apenas para aqueles atos infracionais de
IV - a relação dos projetos aprovados em cada ano- natureza leve e sem grave ameaça à pessoa. Não
calendário e o valor dos recursos previstos para discriminação do adolescente, notadamente em razão
implementação das ações, por projeto; de etnia, gênero, nacionalidade, classe social,
orientação religiosa, política ou sexual, ou associação
V - o total dos recursos recebidos e a respectiva ou pertencimento a qualquer minoria ou status.
destinação, por projeto atendido, inclusive com
cadastramento na base de dados do Sistema de Assinale a alternativa que indica todas as afirmativas
Informações sobre a Infância e a Adolescência; e corretas.
VI - a avaliação dos resultados dos projetos beneficiados a. ( ) São corretas apenas as afirmativas 1 e 4.
com recursos dos Fundos dos Direitos da Criança e do b. ( ) São corretas apenas as afirmativas 2 e 4.
Adolescente nacional, estaduais, distrital e municipais.” c. ( ) São corretas apenas as afirmativas 3 e 4.
“Art. 260-J. O Ministério Público determinará, em cada d. ( ) São corretas apenas as afirmativas 1, 2 e 3.
Comarca, a forma de fiscalização da aplicação dos e. ( ) São corretas apenas as afirmativas 2, 3 e 4
incentivos fiscais referidos no art. 260 desta Lei.
Parágrafo único. O descumprimento do disposto nos arts.
02. De acordo com a Lei do Sinase (Lei no 12.594/2012),
260-G e 260-I sujeitará os infratores a responder por ação
assinale a alternativa correta no que se refere aos
judicial proposta pelo Ministério Público, que poderá atuar
de ofício, a requerimento ou representação de qualquer direitos individuais dos adolescentes submetidos ao
cidadão.” cumprimento de medidas socioeducativas:
a. ( ) As garantias processuais destinadas a adolescente
“Art. 260-K. A Secretaria de Direitos Humanos da
autor de ato infracional previstas no Estatuto da
Presidência da República (SDH/PR) encaminhará à
Secretaria da Receita Federal do Brasil, até 31 de outubro Criança e do Adolescente não se aplicam na
de cada ano, arquivo eletrônico contendo a relação execução das medidas socioeducativas.
atualizada dos Fundos dos Direitos da Criança e do b. ( ) O adolescente em cumprimento de medida
Adolescente nacional, distrital, estaduais e municipais, socioeducativa tem direito a receber, sempre que
com a indicação dos respectivos números de inscrição no solicitar, informações sobre a evolução de seu plano
CNPJ e das contas bancárias específicas mantidas em individual, participando, obrigatoriamente, de sua
instituições financeiras públicas, destinadas elaboração e, se for o caso, reavaliação.
exclusivamente a gerir os recursos dos Fundos.” c. ( ) No caso de cumprimento de medida socioeducativa
“Art. 260-L. A Secretaria da Receita Federal do Brasil de internação, o adolescente poderá ser incluído em
expedirá as instruções necessárias à aplicação do estabelecimento prisional voltado para adultos quando
disposto nos arts. 260 a 260-K.” inexistir vaga para o cumprimento da medida no
Art. 88. O parágrafo único do art. 3 da Lei
o sistema socioeducativo, devendo, neste caso,
o permanecer em cela especial e separada dos adultos.
n 12.213, de 20 de janeiro de 2010, passa a vigorar com
a seguinte redação: d. ( ) A inexistência de programas de atendimento
o
“Art. 3 ................ socioeducativo em meio aberto poderá ser invocada
como motivo para aplicação ou manutenção de
Parágrafo único. A dedução a que se refere o caput deste medida de privação da liberdade.
artigo não poderá ultrapassar 1% (um por cento) do
imposto devido.” (NR) e. ( ) O adolescente em cumprimento de medida
socioeducativa tem direito a peticionar, por escrito ou
Art. 89. (VETADO). verbalmente, diretamente a qualquer autoridade ou
Art. 90. Esta Lei entra em vigor após decorridos 90 órgão público, devendo, obrigatoriamente, ser
(noventa) dias de sua publicação oficial. respondido em até 60 dias.
Brasília, 18 de janeiro de 2012; 191o da Independência e
124o da República.

www.editoradince.com - Acesse e veja se há novidades a respeito deste material – CUIDADO: cópia é crime.
18 LEGISLAÇÃO ESPECIAL E REGIMENTO INTERNO DAS UNIDADES DO SEAS
TÉCNICO SOCIOEDUCATIVO/ ADMINISTRATIVO- 06. Conforme o disposto no artigo 13, da Lei nº
DF/2015 12.594/2012, sobre programas de meio aberto,
03. Com base no que estabelece a Lei n.º 12.594/2012 a compete à direção do programa de prestação de
respeito do Sistema Nacional de Atendimento serviços à comunidade ou de liberdade assistida:
Socioeducativo (Sinase), a execução das medidas I. encaminhar o adolescente para o orientador
socioeducativas reger-se-á pelo princípio da credenciado.
(A) proporcionalidade em relação à ofensa cometida, II. receber o adolescente e seus pais ou responsável e
podendo o adolescente receber tratamento igual ou orientá-los sobre a finalidade da medida e a
até mais gravoso que o conferido ao adulto, na organização e funcionamento do programa.
medida da sua culpabilidade. III. supervisionar o desenvolvimento da medida.
(B) individualização e adequação da medida, É correto o que está contido em
considerando-se a idade, as circunstâncias pessoais
(A) I, II e III.
do adolescente e o prolongamento da duração da
medida com o objetivo de prevenir reincidência em ato (B) I e III, apenas.
infracional. (C) II e III, apenas.
(C) legalidade, priorizando-se a intervenção judicial e a (D) I e II, apenas.
imposição de medidas de meio aberto. (E) III, apenas.
(D) intervenção ampla, alcançando-se efetivamente todos
os aspectos necessários para a ressocialização do 07. De acordo com a Lei nº 12.594/2012, sobre a
menor infrator. avaliação e o acompanhamento da gestão do
(E) prioridade em relação a práticas ou medidas que atendimento socioeducativo, analise as assertivas
sejam restaurativas e, sempre que possível, em abaixo.
relação a práticas ou medidas que atendam às I. Disponibilizar informações sobre o atendimento
necessidades das vítimas. socioeducativo é um dos objetivos do Sistema
Nacional de Avaliação e Acompanhamento do
04. Caio, técnico socioeducativo, foi acusado pela prática Atendimento Socioeducativo.
de crime de abuso de autoridade. Com base nesse II. A avaliação será coordenada por uma comissão
caso hipotético e no que dispõe a Lei n.º 4.898/1965, é permanente e realizada por comissões temporárias.
correto afirmar que Caio III. A avaliação da gestão terá por objetivo, dentre outros,
(A) estará sujeito a sanções penais, como a suspensão do a articulação interinstitucional e intersetorial das
cargo, por prazo de cinco a cento e oitenta dias, com políticas.
perda de vencimentos e vantagens. IV. É facultado à comissão permanente designar
(B) estará sujeito a sanções administrativas, como a avaliadores que sejam titulares ou servidores dos
perda do cargo e a inabilitação para o exercício de órgãos gestores avaliados. É correto o que se afirma
qualquer outra função pública, por prazo de até três em
anos. (A) I e II, apenas.
(C) terá praticado crime de abuso de autoridade caso (B) I e IV, apenas.
tenha cometido, sem competência legal, ato lesivo à
honra ou ao patrimônio de pessoa jurídica. (C) II e III, apenas.
(D) não estará sujeito a sanções administrativas caso seja (D) I, II e III, apenas.
condenado criminalmente.
(E) poderá ser acusado pela prática do crime de abuso de 08. De acordo com a Lei nº 12.594/2012, sobre a
autoridade ainda que tenha agido culposamente. avaliação e o acompanhamento da gestão do
atendimento socioeducativo, os resultados da
avaliação serão utilizados para
AGENTE DE APOIO SOCIOEDUCATIVO - FUNDAÇÃO
CASA/2014 I. execução de metas do Sistema de Atendimento
Socioeducativo.
05. Sobre os programas de atendimento de que trata a Lei
nº 12.594/2012, assinale a alternativa correta. II. reforço de financiamento para fortalecer a rede de
atendimento socioeducativo.
(A) Os Municípios inscreverão seus programas e
alterações, bem como as entidades de atendimento III. verificação de reincidência de prática de ato
executoras, no Conselho Estadual dos Direitos da infracional.
Criança e do Adolescente. É correto o que está contido em
(B) A composição da equipe técnica do programa de (A) I, apenas.
atendimento deverá compreender, no mínimo, (B) II, apenas.
profissionais da área de assistência social. (C) I e II, apenas.
(C) É requisito obrigatório para a inscrição de programa (D) II e III, apenas.
de atendimento a política de formação dos recursos
(E) I, II e III.
humanos.
(D) Os Estados e o Distrito Federal inscreverão seus
programas de atendimento e alterações no Conselho 09. Acerca do financiamento e das prioridades, leia o
Federal dos Direitos da Criança e do Adolescente. dispositivo abaixo e, em seguida, assinale a
alternativa que preenche corretamente a lacuna. Os
(E) O rol de orientadores credenciados deverá ser
Conselhos de Direitos, nas 3 (três) esferas de
comunicado, anualmente, ao Ministério Público.
governo, definirão, ______________, o percentual de
recursos dos Fundos dos Direitos da Criança e do

www.editoradince.com - Acesse e veja se há novidades a respeito deste material – CUIDADO: cópia é crime.
LEGISLAÇÃO ESPECIAL E REGIMENTO INTERNO DAS UNIDADES DO SEAS 19
Adolescente a serem aplicados no financiamento das sexo, raça, etnia ou cor, religião ou crença, deficiência,
ações previstas na Lei nº 12.594/2012, em especial condição pessoal de desenvolvimento e aprendizagem,
para capacitação, sistemas de informação e de condição econômica, ambiente social, região e local de
avaliação. moradia ou outra condição que diferencie as pessoas, as
(A) mensalmente famílias ou a comunidade em que vivem. (incluído pela
Lei nº 13.257, de 2016)
(B) semestralmente
Art. 4º É dever da família, da comunidade, da
(C) anualmente
sociedade em geral e do poder público assegurar, com
(D) a cada 90 (noventa) dias absoluta prioridade, a efetivação dos direitos referentes à
(E) em reuniões bimestrais vida, à saúde, à alimentação, à educação, ao esporte, ao
lazer, à profissionalização, à cultura, à dignidade, ao
10. Considerando a Lei nº 12.594/2012, marque V para respeito, à liberdade e à convivência familiar e
verdadeiro ou F para falso e, em seguida, assinale a comunitária.
alternativa que apresenta a sequência correta. Parágrafo único. A garantia de prioridade
( ) A atenção integral à saúde do adolescente no Sistema compreende:
de Atendimento Socioeducativo seguirá, entre outras a) primazia de receber proteção e socorro em
diretrizes, a inclusão de ações e serviços para a quaisquer circunstâncias;
promoção, proteção, prevenção de agravos e doenças b) precedência de atendimento nos serviços
e recuperação da saúde. públicos ou de relevância pública;
( ) É vedado ao adolescente que viva, comprovadamente, c) preferência na formulação e na execução das
em união estável o direito à visita íntima. políticas sociais públicas;
( ) O regime disciplinar depende da responsabilidade civil d) destinação privilegiada de recursos públicos
ou penal que advenha do ato cometido. nas áreas relacionadas com a proteção à infância e à
a) V/ V/ V juventude.
b) V/ F/ F Art. 5º Nenhuma criança ou adolescente será
c) F/ F/ V objeto de qualquer forma de negligência, discriminação,
d) F/ V/ F exploração, violência, crueldade e opressão, punido na
e) F/ F/ F forma da lei qualquer atentado, por ação ou omissão, aos
seus direitos fundamentais.
Gabarito: 01/A; 02/C; 03/E; 04/C; 05/C; 06/A; 07/D; 08/B; Art. 6º Na interpretação desta Lei levar-se-ão em
09/C; 10/B conta os fins sociais a que ela se dirige, as exigências do
bem comum, os direitos e deveres individuais e coletivos,
e a condição peculiar da criança e do adolescente como
pessoas em desenvolvimento.
LEI FEDERAL Nº 8.069 DE 13 DE JULHO Título II
Dos Direitos Fundamentais
DE 1990 (ESTATUTO DA CRIANÇA E DO
Capítulo I
ADOLESCENTE - ECA)
Do Direito à Vida e à Saúde
Dispõe sobre o Estatuto da Criança e do Art. 7º A criança e o adolescente têm direito a
Adolescente e dá outras providências. proteção à vida e à saúde, mediante a efetivação de
(Atualizado conforme Lei nº 14.154, de 26 de políticas sociais públicas que permitam o nascimento e o
maio de 2021 desenvolvimento sadio e harmonioso, em condições
O PRESIDENTE DA REPÚBLICA: dignas de existência.
o
Faço saber que o Congresso Nacional decreta e eu Art. 8 É assegurado a todas as mulheres o acesso
sanciono a seguinte Lei: aos programas e às políticas de saúde da mulher e de
TÍTULO I planejamento reprodutivo e, às gestantes, nutrição
adequada, atenção humanizada à gravidez, ao parto e ao
Das Disposições Preliminares
puerpério e atendimento pré-natal, perinatal e pós-natal
Art. 1º Esta Lei dispõe sobre a proteção integral à integral no âmbito do Sistema Único de Saúde. (Redação
criança e ao adolescente. dada pela Lei nº 13.257, de 2016)
Art. 2º Considera-se criança, para os efeitos desta o
§ 1 O atendimento pré-natal será realizado por
Lei, a pessoa até doze anos de idade incompletos, e profissionais da atenção primária. (Redação dada pela Lei
adolescente aquela entre doze e dezoito anos de idade. nº 13.257, de 2016)
Parágrafo único. Nos casos expressos em lei, o
§ 2 Os profissionais de saúde de referência da
aplica-se excepcionalmente este Estatuto às pessoas gestante garantirão sua vinculação, no último trimestre da
entre dezoito e vinte e um anos de idade. gestação, ao estabelecimento em que será realizado o
Art. 3º A criança e o adolescente gozam de todos parto, garantido o direito de opção da mulher. (Redação
os direitos fundamentais inerentes à pessoa humana, sem dada pela Lei nº 13.257, de 2016)
prejuízo da proteção integral de que trata esta Lei, o
§ 3 Os serviços de saúde onde o parto for
assegurando-se-lhes, por lei ou por outros meios, todas realizado assegurarão às mulheres e aos seus filhos
as oportunidades e facilidades, a fim de lhes facultar o recém-nascidos alta hospitalar responsável e
desenvolvimento físico, mental, moral, espiritual e social, contrarreferência na atenção primária, bem como o
em condições de liberdade e de dignidade. acesso a outros serviços e a grupos de apoio à
Parágrafo único. Os direitos enunciados nesta Lei amamentação. (Redação dada pela Lei nº 13.257, de
aplicam-se a todas as crianças e adolescentes, sem 2016)
discriminação de nascimento, situação familiar, idade,

www.editoradince.com - Acesse e veja se há novidades a respeito deste material – CUIDADO: cópia é crime.
20 LEGISLAÇÃO ESPECIAL E REGIMENTO INTERNO DAS UNIDADES DO SEAS
o
§ 4 Incumbe ao poder público proporcionar Art. 10. Os hospitais e demais estabelecimentos de
assistência psicológica à gestante e à mãe, no período atenção à saúde de gestantes, públicos e particulares,
pré e pós-natal, inclusive como forma de prevenir ou são obrigados a:
minorar as consequências do estado puerperal. (Incluído I - manter registro das atividades desenvolvidas,
pela Lei nº 12.010, de 2009) através de prontuários individuais, pelo prazo de dezoito
o o
§ 5 A assistência referida no § 4 deste artigo anos;
deverá ser prestada também a gestantes e mães que II - identificar o recém-nascido mediante o registro
manifestem interesse em entregar seus filhos para de sua impressão plantar e digital e da impressão digital
adoção, bem como a gestantes e mães que se encontrem da mãe, sem prejuízo de outras formas normatizadas pela
em situação de privação de liberdade. (Redação dada autoridade administrativa competente;
pela Lei nº 13.257, de 2016)
o
III - proceder a exames visando ao diagnóstico e
§ 6 A gestante e a parturiente têm direito a 1 (um) terapêutica de anormalidades no metabolismo do recém-
acompanhante de sua preferência durante o período do nascido, bem como prestar orientação aos pais;
pré-natal, do trabalho de parto e do pós-parto
IV - fornecer declaração de nascimento onde
imediato. (Incluído pela Lei nº 13.257, de 2016)
o
constem necessariamente as intercorrências do parto e
§ 7 A gestante deverá receber orientação sobre do desenvolvimento do neonato;
aleitamento materno, alimentação complementar saudável
V - manter alojamento conjunto, possibilitando ao
e crescimento e desenvolvimento infantil, bem como
neonato a permanência junto à mãe.
sobre formas de favorecer a criação de vínculos afetivos e
de estimular o desenvolvimento integral da VI - acompanhar a prática do processo de
criança. (Incluído pela Lei nº 13.257, de 2016) amamentação, prestando orientações quanto à técnica
o adequada, enquanto a mãe permanecer na unidade
§ 8 A gestante tem direito a acompanhamento
hospitalar, utilizando o corpo técnico já existente. (Incluído
saudável durante toda a gestação e a parto natural
pela Lei nº 13.436, de 2017) ( )
cuidadoso, estabelecendo-se a aplicação de cesariana e
outras intervenções cirúrgicas por motivos Art. 11. É assegurado acesso integral às linhas de
médicos. (Incluído pela Lei nº 13.257, de 2016) cuidado voltadas à saúde da criança e do adolescente,
o por intermédio do Sistema Único de Saúde, observado o
§ 9 A atenção primária à saúde fará a busca ativa
princípio da equidade no acesso a ações e serviços para
da gestante que não iniciar ou que abandonar as
promoção, proteção e recuperação da saúde. (Redação
consultas de pré-natal, bem como da puérpera que não
dada pela Lei nº 13.257, de 2016)
comparecer às consultas pós-parto. (Incluído pela Lei nº o
13.257, de 2016) § 1 A criança e o adolescente com deficiência
serão atendidos, sem discriminação ou segregação, em
§ 10. Incumbe ao poder público garantir, à gestante
suas necessidades gerais de saúde e específicas de
e à mulher com filho na primeira infância que se
habilitação e reabilitação. (Redação dada pela Lei nº
encontrem sob custódia em unidade de privação de
13.257, de 2016)
liberdade, ambiência que atenda às normas sanitárias e o
assistenciais do Sistema Único de Saúde para o § 2 Incumbe ao poder público fornecer
acolhimento do filho, em articulação com o sistema de gratuitamente, àqueles que necessitarem, medicamentos,
ensino competente, visando ao desenvolvimento integral órteses, próteses e outras tecnologias assistivas relativas
da criança. (Incluído pela Lei nº 13.257, de 2016) ao tratamento, habilitação ou reabilitação para crianças e
adolescentes, de acordo com as linhas de cuidado
Art. 8º-A. Fica instituída a Semana Nacional de
voltadas às suas necessidades específicas. (Redação
Prevenção da Gravidez na Adolescência, a ser realizada
dada pela Lei nº 13.257, de 2016)
anualmente na semana que incluir o dia 1º de fevereiro, o
com o objetivo de disseminar informações sobre medidas § 3 Os profissionais que atuam no cuidado diário
preventivas e educativas que contribuam para a redução ou frequente de crianças na primeira infância receberão
da incidência da gravidez na adolescência. (Incluído pela formação específica e permanente para a detecção de
Lei nº 13.798, de 2019) sinais de risco para o desenvolvimento psíquico, bem
como para o acompanhamento que se fizer
Parágrafo único. As ações destinadas a efetivar o
necessário. (Incluído pela Lei nº 13.257, de 2016)
disposto no caput deste artigo ficarão a cargo do poder
público, em conjunto com organizações da sociedade Art. 12. Os estabelecimentos de atendimento à
civil, e serão dirigidas prioritariamente ao público saúde, inclusive as unidades neonatais, de terapia
adolescente. (Incluído pela Lei nº 13.798, de 2019) intensiva e de cuidados intermediários, deverão
proporcionar condições para a permanência em tempo
Art. 9º O poder público, as instituições e os
integral de um dos pais ou responsável, nos casos de
empregadores propiciarão condições adequadas ao
internação de criança ou adolescente. (Redação dada
aleitamento materno, inclusive aos filhos de mães
pela Lei nº 13.257, de 2016)
submetidas a medida privativa de liberdade.
o Art. 13. Os casos de suspeita ou confirmação de
§ 1 Os profissionais das unidades primárias de
castigo físico, de tratamento cruel ou degradante e de
saúde desenvolverão ações sistemáticas, individuais ou
maus-tratos contra criança ou adolescente serão
coletivas, visando ao planejamento, à implementação e à
obrigatoriamente comunicados ao Conselho Tutelar da
avaliação de ações de promoção, proteção e apoio ao
respectiva localidade, sem prejuízo de outras providências
aleitamento materno e à alimentação complementar
legais. (Redação dada pela Lei nº 13.010, de 2014)
saudável, de forma contínua. (Incluído pela Lei nº 13.257, o
de 2016) § 1 As gestantes ou mães que manifestem
o interesse em entregar seus filhos para adoção serão
§ 2 Os serviços de unidades de terapia intensiva
obrigatoriamente encaminhadas, sem constrangimento, à
neonatal deverão dispor de banco de leite humano ou
Justiça da Infância e da Juventude. (Incluído pela Lei nº
unidade de coleta de leite humano. (Incluído pela Lei nº
13.257, de 2016)
13.257, de 2016)

www.editoradince.com - Acesse e veja se há novidades a respeito deste material – CUIDADO: cópia é crime.
LEGISLAÇÃO ESPECIAL E REGIMENTO INTERNO DAS UNIDADES DO SEAS 21
o
§ 2 Os serviços de saúde em suas diferentes Art. 18. É dever de todos velar pela dignidade da
portas de entrada, os serviços de assistência social em criança e do adolescente, pondo-os a salvo de qualquer
seu componente especializado, o Centro de Referência tratamento desumano, violento, aterrorizante, vexatório ou
Especializado de Assistência Social (Creas) e os demais constrangedor.
órgãos do Sistema de Garantia de Direitos da Criança e Art. 18-A. A criança e o adolescente têm o direito
do Adolescente deverão conferir máxima prioridade ao de ser educados e cuidados sem o uso de castigo físico
atendimento das crianças na faixa etária da primeira ou de tratamento cruel ou degradante, como formas de
infância com suspeita ou confirmação de violência de correção, disciplina, educação ou qualquer outro pretexto,
qualquer natureza, formulando projeto terapêutico singular pelos pais, pelos integrantes da família ampliada, pelos
que inclua intervenção em rede e, se necessário, responsáveis, pelos agentes públicos executores de
acompanhamento domiciliar. (Incluído pela Lei nº 13.257, medidas socioeducativas ou por qualquer pessoa
de 2016) encarregada de cuidar deles, tratá-los, educá-los ou
Art. 14. O Sistema Único de Saúde promoverá protegê-los. (Incluído pela Lei nº 13.010, de 2014)
programas de assistência médica e odontológica para a Parágrafo único. Para os fins desta Lei, considera-
prevenção das enfermidades que ordinariamente afetam a se: (Incluído pela Lei nº 13.010, de 2014)
população infantil, e campanhas de educação sanitária
I - castigo físico: ação de natureza disciplinar ou
para pais, educadores e alunos.
o
punitiva aplicada com o uso da força física sobre a criança
§ 1 É obrigatória a vacinação das crianças nos ou o adolescente que resulte em: (Incluído pela Lei nº
casos recomendados pelas autoridades 13.010, de 2014)
sanitárias. (Renumerado do parágrafo único pela Lei nº
a) sofrimento físico; ou (Incluído pela Lei nº 13.010,
13.257, de 2016)
o
de 2014)
§ 2 O Sistema Único de Saúde promoverá a
b) lesão; (Incluído pela Lei nº 13.010, de 2014)
atenção à saúde bucal das crianças e das gestantes, de
forma transversal, integral e intersetorial com as demais II - tratamento cruel ou degradante: conduta ou
linhas de cuidado direcionadas à mulher e à forma cruel de tratamento em relação à criança ou ao
criança. (Incluído pela Lei nº 13.257, de 2016) adolescente que: (Incluído pela Lei nº 13.010, de 2014)
o
§ 3 A atenção odontológica à criança terá função a) humilhe; ou (Incluído pela Lei nº 13.010, de
educativa protetiva e será prestada, inicialmente, antes de 2014)
o bebê nascer, por meio de aconselhamento pré-natal, e, b) ameace gravemente; ou (Incluído pela Lei nº
posteriormente, no sexto e no décimo segundo anos de 13.010, de 2014)
vida, com orientações sobre saúde bucal. (Incluído pela c) ridicularize. (Incluído pela Lei nº 13.010, de
Lei nº 13.257, de 2016) 2014)
o
§ 4 A criança com necessidade de cuidados Art. 18-B. Os pais, os integrantes da família
odontológicos especiais será atendida pelo Sistema Único ampliada, os responsáveis, os agentes públicos
de Saúde. (Incluído pela Lei nº 13.257, de 2016) executores de medidas socioeducativas ou qualquer
§ 5 º É obrigatória a aplicação a todas as crianças, pessoa encarregada de cuidar de crianças e de
nos seus primeiros dezoito meses de vida, de protocolo adolescentes, tratá-los, educá-los ou protegê-los que
ou outro instrumento construído com a finalidade de utilizarem castigo físico ou tratamento cruel ou degradante
facilitar a detecção, em consulta pediátrica de como formas de correção, disciplina, educação ou
acompanhamento da criança, de risco para o seu qualquer outro pretexto estarão sujeitos, sem prejuízo de
desenvolvimento psíquico. (Incluído pela Lei nº 13.438, de outras sanções cabíveis, às seguintes medidas, que serão
2017) ( ) aplicadas de acordo com a gravidade do caso: (Incluído
Capítulo II pela Lei nº 13.010, de 2014)
Do Direito à Liberdade, ao Respeito e à Dignidade I - encaminhamento a programa oficial ou
comunitário de proteção à família; (Incluído pela Lei nº
Art. 15. A criança e o adolescente têm direito à
13.010, de 2014)
liberdade, ao respeito e à dignidade como pessoas
humanas em processo de desenvolvimento e como II - encaminhamento a tratamento psicológico ou
sujeitos de direitos civis, humanos e sociais garantidos na psiquiátrico; (Incluído pela Lei nº 13.010, de 2014)
Constituição e nas leis. III - encaminhamento a cursos ou programas de
Art. 16. O direito à liberdade compreende os orientação; (Incluído pela Lei nº 13.010, de 2014)
seguintes aspectos: IV - obrigação de encaminhar a criança a
I - ir, vir e estar nos logradouros públicos e espaços tratamento especializado; (Incluído pela Lei nº 13.010, de
comunitários, ressalvadas as restrições legais; 2014)
II - opinião e expressão; V - advertência. (Incluído pela Lei nº 13.010, de
2014)
III - crença e culto religioso;
Parágrafo único. As medidas previstas neste artigo
IV - brincar, praticar esportes e divertir-se;
serão aplicadas pelo Conselho Tutelar, sem prejuízo de
V - participar da vida familiar e comunitária, sem outras providências legais. (Incluído pela Lei nº 13.010, de
discriminação; 2014)
VI - participar da vida política, na forma da lei; Capítulo III
VII - buscar refúgio, auxílio e orientação. Do Direito à Convivência Familiar e Comunitária
Art. 17. O direito ao respeito consiste na Seção I
inviolabilidade da integridade física, psíquica e moral da
Disposições Gerais
criança e do adolescente, abrangendo a preservação da
imagem, da identidade, da autonomia, dos valores, idéias Art. 19. É direito da criança e do adolescente ser
e crenças, dos espaços e objetos pessoais. criado e educado no seio de sua família e,
excepcionalmente, em família substituta, assegurada a

www.editoradince.com - Acesse e veja se há novidades a respeito deste material – CUIDADO: cópia é crime.
22 LEGISLAÇÃO ESPECIAL E REGIMENTO INTERNO DAS UNIDADES DO SEAS
convivência familiar e comunitária, em ambiente que entidade que desenvolva programa de acolhimento
garanta seu desenvolvimento integral. (Redação dada familiar ou institucional. (Incluído pela Lei nº 13.509, de
pela Lei nº 13.257, de 2016) 2017)
o o
§ 1 Toda criança ou adolescente que estiver § 5 Após o nascimento da criança, a vontade da
inserido em programa de acolhimento familiar ou mãe ou de ambos os genitores, se houver pai registral ou
institucional terá sua situação reavaliada, no máximo, a pai indicado, deve ser manifestada na audiência a que se
o
cada 3 (três) meses, devendo a autoridade judiciária refere o § 1 do art. 166 desta Lei, garantido o sigilo sobre
competente, com base em relatório elaborado por equipe a entrega. (Incluído pela Lei nº 13.509, de 2017)
interprofissional ou multidisciplinar, decidir de forma § 6º Na hipótese de não comparecerem à audiência
fundamentada pela possibilidade de reintegração familiar nem o genitor nem representante da família extensa para
ou pela colocação em família substituta, em quaisquer confirmar a intenção de exercer o poder familiar ou a
das modalidades previstas no art. 28 desta Lei. (Redação guarda, a autoridade judiciária suspenderá o poder
dada pela Lei nº 13.509, de 2017) familiar da mãe, e a criança será colocada sob a guarda
o
§ 2 A permanência da criança e do adolescente provisória de quem esteja habilitado a adotá-la. (Incluído
em programa de acolhimento institucional não se pela Lei nº 13.509, de 2017)
prolongará por mais de 18 (dezoito meses), salvo o
§ 7 Os detentores da guarda possuem o prazo de
comprovada necessidade que atenda ao seu superior 15 (quinze) dias para propor a ação de adoção, contado
interesse, devidamente fundamentada pela autoridade do dia seguinte à data do término do estágio de
judiciária. (Redação dada pela Lei nº 13.509, de 2017) convivência. (Incluído pela Lei nº 13.509, de 2017)
o
§ 3 A manutenção ou a reintegração de criança ou o
§ 8 Na hipótese de desistência pelos genitores -
adolescente à sua família terá preferência em relação a manifestada em audiência ou perante a equipe
qualquer outra providência, caso em que será esta interprofissional - da entrega da criança após o
incluída em serviços e programas de proteção, apoio e nascimento, a criança será mantida com os genitores, e
o
promoção, nos termos do § 1 do art. 23, dos incisos I e será determinado pela Justiça da Infância e da Juventude
IV do caput do art. 101 e dos incisos I a IV do caput do o acompanhamento familiar pelo prazo de 180 (cento e
art. 129 desta Lei. (Redação dada pela Lei nº 13.257, de oitenta) dias. (Incluído pela Lei nº 13.509, de 2017)
2016) o
o
§ 9 É garantido à mãe o direito ao sigilo sobre o
§ 4 Será garantida a convivência da criança e do nascimento, respeitado o disposto no art. 48 desta
adolescente com a mãe ou o pai privado de liberdade, por Lei. (Incluído pela Lei nº 13.509, de 2017)
meio de visitas periódicas promovidas pelo responsável ou,
§ 10. Serão cadastrados para adoção recém-
nas hipóteses de acolhimento institucional, pela entidade
nascidos e crianças acolhidas não procuradas por suas
responsável, independentemente de autorização
famílias no prazo de 30 (trinta) dias, contado a partir do
judicial. (Incluído pela Lei nº 12.962, de 2014)
o
dia do acolhimento. (Incluído pela Lei nº 13.509, de 2017)
§ 5 Será garantida a convivência integral da
Art. 19-B. A criança e o adolescente em programa
criança com a mãe adolescente que estiver em
de acolhimento institucional ou familiar poderão participar
acolhimento institucional. (Incluído pela Lei nº 13.509, de
de programa de apadrinhamento. (Incluído pela Lei nº
2017)
o
13.509, de 2017)
§ 6 A mãe adolescente será assistida por equipe o
§ 1 O apadrinhamento consiste em estabelecer e
especializada multidisciplinar. (Incluído pela Lei nº 13.509,
proporcionar à criança e ao adolescente vínculos externos
de 2017)
à instituição para fins de convivência familiar e
Art. 19-A. A gestante ou mãe que manifeste comunitária e colaboração com o seu desenvolvimento
interesse em entregar seu filho para adoção, antes ou nos aspectos social, moral, físico, cognitivo, educacional e
logo após o nascimento, será encaminhada à Justiça da financeiro. (Incluído pela Lei nº 13.509, de 2017)
Infância e da Juventude. (Incluído pela Lei nº 13.509, de
§ 2º Podem ser padrinhos ou madrinhas pessoas
2017)
o
maiores de 18 (dezoito) anos não inscritas nos cadastros
§ 1 A gestante ou mãe será ouvida pela equipe de adoção, desde que cumpram os requisitos exigidos
interprofissional da Justiça da Infância e da Juventude, pelo programa de apadrinhamento de que fazem
que apresentará relatório à autoridade judiciária, parte. (Incluído pela Lei nº 13.509, de 2017)
considerando inclusive os eventuais efeitos do estado o
§ 3 Pessoas jurídicas podem apadrinhar criança
gestacional e puerperal. (Incluído pela Lei nº 13.509, de
ou adolescente a fim de colaborar para o seu
2017)
o
desenvolvimento. (Incluído pela Lei nº 13.509, de 2017)
§ 2 De posse do relatório, a autoridade judiciária o
§ 4 O perfil da criança ou do adolescente a ser
poderá determinar o encaminhamento da gestante ou
apadrinhado será definido no âmbito de cada programa
mãe, mediante sua expressa concordância, à rede pública
de apadrinhamento, com prioridade para crianças ou
de saúde e assistência social para atendimento
adolescentes com remota possibilidade de reinserção
especializado. (Incluído pela Lei nº 13.509, de 2017)
o
familiar ou colocação em família adotiva. (Incluído pela Lei
§ 3 A busca à família extensa, conforme definida nº 13.509, de 2017)
nos termos do parágrafo único do art. 25 desta Lei, o
§ 5 Os programas ou serviços de apadrinhamento
respeitará o prazo máximo de 90 (noventa) dias,
apoiados pela Justiça da Infância e da Juventude poderão
prorrogável por igual período. (Incluído pela Lei nº 13.509,
ser executados por órgãos públicos ou por organizações
de 2017)
o
da sociedade civil. (Incluído pela Lei nº 13.509, de 2017)
§ 4 Na hipótese de não haver a indicação do o
§ 6 Se ocorrer violação das regras de
genitor e de não existir outro representante da família
apadrinhamento, os responsáveis pelo programa e pelos
extensa apto a receber a guarda, a autoridade judiciária
serviços de acolhimento deverão imediatamente notificar
competente deverá decretar a extinção do poder familiar e
a autoridade judiciária competente. (Incluído pela Lei nº
determinar a colocação da criança sob a guarda
13.509, de 2017)
provisória de quem estiver habilitado a adotá-la ou de

www.editoradince.com - Acesse e veja se há novidades a respeito deste material – CUIDADO: cópia é crime.
LEGISLAÇÃO ESPECIAL E REGIMENTO INTERNO DAS UNIDADES DO SEAS 23
Art. 20. Os filhos, havidos ou não da relação do podendo ser exercitado contra os pais ou seus herdeiros,
casamento, ou por adoção, terão os mesmos direitos e sem qualquer restrição, observado o segredo de Justiça.
qualificações, proibidas quaisquer designações Seção III
discriminatórias relativas à filiação.
Da Família Substituta
Art. 21. O pátrio poder poder familiar será exercido,
Subseção I
em igualdade de condições, pelo pai e pela mãe, na forma
do que dispuser a legislação civil, assegurado a qualquer Disposições Gerais
deles o direito de, em caso de discordância, recorrer à Art. 28. A colocação em família substituta far-se-á
autoridade judiciária competente para a solução da mediante guarda, tutela ou adoção, independentemente
divergência. (Expressão substituída pela Lei nº 12.010, de da situação jurídica da criança ou adolescente, nos
2009) termos desta Lei.
o
Art. 22. Aos pais incumbe o dever de sustento, § 1 Sempre que possível, a criança ou o
guarda e educação dos filhos menores, cabendo-lhes adolescente será previamente ouvido por equipe
ainda, no interesse destes, a obrigação de cumprir e fazer interprofissional, respeitado seu estágio de
cumprir as determinações judiciais. desenvolvimento e grau de compreensão sobre as
Parágrafo único. A mãe e o pai, ou os implicações da medida, e terá sua opinião devidamente
responsáveis, têm direitos iguais e deveres e considerada. (Redação dada pela Lei nº 12.010, de 2009)
o
responsabilidades compartilhados no cuidado e na § 2 Tratando-se de maior de 12 (doze) anos de
educação da criança, devendo ser resguardado o direito idade, será necessário seu consentimento, colhido em
de transmissão familiar de suas crenças e culturas, audiência. (Redação dada pela Lei nº 12.010, de 2009)
assegurados os direitos da criança estabelecidos nesta o
§ 3 Na apreciação do pedido levar-se-á em conta
Lei. (Incluído pela Lei nº 13.257, de 2016) o grau de parentesco e a relação de afinidade ou de
Art. 23. A falta ou a carência de recursos materiais afetividade, a fim de evitar ou minorar as consequências
não constitui motivo suficiente para a perda ou a decorrentes da medida. (Incluído pela Lei nº 12.010, de
suspensão do pátrio poder poder familiar . (Expressão 2009)
substituída pela Lei nº 12.010, de 2009) o
§ 4 Os grupos de irmãos serão colocados sob
o
§ 1 Não existindo outro motivo que por si só adoção, tutela ou guarda da mesma família substituta,
autorize a decretação da medida, a criança ou o ressalvada a comprovada existência de risco de abuso ou
adolescente será mantido em sua família de origem, a outra situação que justifique plenamente a
qual deverá obrigatoriamente ser incluída em serviços e excepcionalidade de solução diversa, procurando-se, em
programas oficiais de proteção, apoio e qualquer caso, evitar o rompimento definitivo dos vínculos
promoção. (Redação dada pela Lei nº 13.257, de 2016) fraternais. (Incluído pela Lei nº 12.010, de 2009)
o
§ 2º A condenação criminal do pai ou da mãe não § 5 A colocação da criança ou adolescente em
implicará a destituição do poder familiar, exceto na família substituta será precedida de sua preparação
hipótese de condenação por crime doloso sujeito à pena gradativa e acompanhamento posterior, realizados pela
de reclusão contra outrem igualmente titular do mesmo equipe interprofissional a serviço da Justiça da Infância e
poder familiar ou contra filho, filha ou outro da Juventude, preferencialmente com o apoio dos
descendente. (Redação dada pela Lei nº 13.715, de 2018) técnicos responsáveis pela execução da política municipal
Art. 24. A perda e a suspensão do pátrio de garantia do direito à convivência familiar. (Incluído pela
poder poder familiar serão decretadas judicialmente, em Lei nº 12.010, de 2009)
o
procedimento contraditório, nos casos previstos na § 6 Em se tratando de criança ou adolescente
legislação civil, bem como na hipótese de indígena ou proveniente de comunidade remanescente de
descumprimento injustificado dos deveres e obrigações a quilombo, é ainda obrigatório: (Incluído pela Lei nº 12.010,
que alude o art. 22. (Expressão substituída pela Lei nº de 2009)
12.010, de 2009) I - que sejam consideradas e respeitadas sua
Seção II identidade social e cultural, os seus costumes e tradições,
Da Família Natural bem como suas instituições, desde que não sejam
incompatíveis com os direitos fundamentais reconhecidos
Art. 25. Entende-se por família natural a
por esta Lei e pela Constituição Federal; (Incluído pela Lei
comunidade formada pelos pais ou qualquer deles e seus
nº 12.010, de 2009)
descendentes.
II - que a colocação familiar ocorra prioritariamente
Parágrafo único. Entende-se por família extensa ou
no seio de sua comunidade ou junto a membros da
ampliada aquela que se estende para além da unidade
mesma etnia; (Incluído pela Lei nº 12.010, de 2009)
pais e filhos ou da unidade do casal, formada por
parentes próximos com os quais a criança ou adolescente III - a intervenção e oitiva de representantes do
convive e mantém vínculos de afinidade e órgão federal responsável pela política indigenista, no
afetividade. (Incluído pela Lei nº 12.010, de 2009) caso de crianças e adolescentes indígenas, e de
antropólogos, perante a equipe interprofissional ou
Art. 26. Os filhos havidos fora do casamento
multidisciplinar que irá acompanhar o caso. (Incluído pela
poderão ser reconhecidos pelos pais, conjunta ou
Lei nº 12.010, de 2009)
separadamente, no próprio termo de nascimento, por
testamento, mediante escritura ou outro documento Art. 29. Não se deferirá colocação em família
público, qualquer que seja a origem da filiação. substituta a pessoa que revele, por qualquer modo,
incompatibilidade com a natureza da medida ou não
Parágrafo único. O reconhecimento pode preceder
ofereça ambiente familiar adequado.
o nascimento do filho ou suceder-lhe ao falecimento, se
deixar descendentes. Art. 30. A colocação em família substituta não
admitirá transferência da criança ou adolescente a
Art. 27. O reconhecimento do estado de filiação é
terceiros ou a entidades governamentais ou não-
direito personalíssimo, indisponível e imprescritível,
governamentais, sem autorização judicial.

www.editoradince.com - Acesse e veja se há novidades a respeito deste material – CUIDADO: cópia é crime.
24 LEGISLAÇÃO ESPECIAL E REGIMENTO INTERNO DAS UNIDADES DO SEAS
Art. 31. A colocação em família substituta Art. 36. A tutela será deferida, nos termos da lei
estrangeira constitui medida excepcional, somente civil, a pessoa de até 18 (dezoito) anos
admissível na modalidade de adoção. incompletos. (Redação dada pela Lei nº 12.010, de 2009)
Art. 32. Ao assumir a guarda ou a tutela, o Parágrafo único. O deferimento da tutela pressupõe
responsável prestará compromisso de bem e fielmente a prévia decretação da perda ou suspensão do pátrio
desempenhar o encargo, mediante termo nos autos. poder poder familiar e implica necessariamente o dever
Subseção II de guarda. (Expressão substituída pela Lei nº 12.010, de
2009)
Da Guarda
Art. 37. O tutor nomeado por testamento ou
Art. 33. A guarda obriga a prestação de assistência
qualquer documento autêntico, conforme previsto no
material, moral e educacional à criança ou adolescente, o
parágrafo único do art. 1.729 da Lei n 10.406, de 10 de
conferindo a seu detentor o direito de opor-se a terceiros,
janeiro de 2002 - Código Civil , deverá, no prazo de 30
inclusive aos pais. (Vide Lei nº 12.010, de 2009)
(trinta) dias após a abertura da sucessão, ingressar com
§ 1º A guarda destina-se a regularizar a posse de pedido destinado ao controle judicial do ato, observando o
fato, podendo ser deferida, liminar ou incidentalmente, procedimento previsto nos arts. 165 a 170 desta
nos procedimentos de tutela e adoção, exceto no de Lei. (Redação dada pela Lei nº 12.010, de 2009)
adoção por estrangeiros.
Parágrafo único. Na apreciação do pedido, serão
§ 2º Excepcionalmente, deferir-se-á a guarda, fora observados os requisitos previstos nos arts. 28 e 29 desta
dos casos de tutela e adoção, para atender a situações Lei, somente sendo deferida a tutela à pessoa indicada na
peculiares ou suprir a falta eventual dos pais ou disposição de última vontade, se restar comprovado que a
responsável, podendo ser deferido o direito de medida é vantajosa ao tutelando e que não existe outra
representação para a prática de atos determinados. pessoa em melhores condições de assumi-la. (Redação
§ 3º A guarda confere à criança ou adolescente a dada pela Lei nº 12.010, de 2009)
condição de dependente, para todos os fins e efeitos de Art. 38. Aplica-se à destituição da tutela o disposto
direito, inclusive previdenciários. no art. 24.
o
§ 4 Salvo expressa e fundamentada determinação Subseção IV
em contrário, da autoridade judiciária competente, ou
Da Adoção
quando a medida for aplicada em preparação para
adoção, o deferimento da guarda de criança ou Art. 39. A adoção de criança e de adolescente
adolescente a terceiros não impede o exercício do direito reger-se-á segundo o disposto nesta Lei.
o
de visitas pelos pais, assim como o dever de prestar § 1 A adoção é medida excepcional e irrevogável,
alimentos, que serão objeto de regulamentação à qual se deve recorrer apenas quando esgotados os
específica, a pedido do interessado ou do Ministério recursos de manutenção da criança ou adolescente na
Público. (Incluído pela Lei nº 12.010, de 2009) família natural ou extensa, na forma do parágrafo único do
Art. 34. O poder público estimulará, por meio de art. 25 desta Lei. (Incluído pela Lei nº 12.010, de 2009)
o
assistência jurídica, incentivos fiscais e subsídios, o § 2 É vedada a adoção por procuração. (Incluído
acolhimento, sob a forma de guarda, de criança ou pela Lei nº 12.010, de 2009)
adolescente afastado do convívio familiar. (Redação dada o
§ 3 Em caso de conflito entre direitos e interesses
pela Lei nº 12.010, de 2009) do adotando e de outras pessoas, inclusive seus pais
o
§ 1 A inclusão da criança ou adolescente em biológicos, devem prevalecer os direitos e os interesses
programas de acolhimento familiar terá preferência a seu do adotando. (Incluído pela Lei nº 13.509, de 2017)
acolhimento institucional, observado, em qualquer caso, o Art. 40. O adotando deve contar com, no máximo,
caráter temporário e excepcional da medida, nos termos dezoito anos à data do pedido, salvo se já estiver sob a
desta Lei. (Incluído pela Lei nº 12.010, de 2009) guarda ou tutela dos adotantes.
o o
§ 2 Na hipótese do § 1 deste artigo a pessoa ou Art. 41. A adoção atribui a condição de filho ao
casal cadastrado no programa de acolhimento familiar adotado, com os mesmos direitos e deveres, inclusive
poderá receber a criança ou adolescente mediante sucessórios, desligando-o de qualquer vínculo com pais e
guarda, observado o disposto nos arts. 28 a 33 desta parentes, salvo os impedimentos matrimoniais.
Lei. (Incluído pela Lei nº 12.010, de 2009)
o
§ 1º Se um dos cônjuges ou concubinos adota o
§ 3 A União apoiará a implementação de serviços filho do outro, mantêm-se os vínculos de filiação entre o
de acolhimento em família acolhedora como política adotado e o cônjuge ou concubino do adotante e os
pública, os quais deverão dispor de equipe que organize o respectivos parentes.
acolhimento temporário de crianças e de adolescentes em
§ 2º É recíproco o direito sucessório entre o
residências de famílias selecionadas, capacitadas e
adotado, seus descendentes, o adotante, seus
acompanhadas que não estejam no cadastro de
ascendentes, descendentes e colaterais até o 4º grau,
adoção. (Incluído pela Lei nº 13.257, de 2016)
o
observada a ordem de vocação hereditária.
§ 4 Poderão ser utilizados recursos federais,
Art. 42. Podem adotar os maiores de 18 (dezoito)
estaduais, distritais e municipais para a manutenção dos
anos, independentemente do estado civil. (Redação dada
serviços de acolhimento em família acolhedora,
pela Lei nº 12.010, de 2009)
facultando-se o repasse de recursos para a própria família
acolhedora. (Incluído pela Lei nº 13.257, de 2016) § 1º Não podem adotar os ascendentes e os irmãos
do adotando.
Art. 35. A guarda poderá ser revogada a qualquer o
tempo, mediante ato judicial fundamentado, ouvido o § 2 Para adoção conjunta, é indispensável que os
Ministério Público. adotantes sejam casados civilmente ou mantenham união
estável, comprovada a estabilidade da família. (Redação
Subseção III
dada pela Lei nº 12.010, de 2009)
Da Tutela
§ 3º O adotante há de ser, pelo menos, dezesseis
anos mais velho do que o adotando.

www.editoradince.com - Acesse e veja se há novidades a respeito deste material – CUIDADO: cópia é crime.
LEGISLAÇÃO ESPECIAL E REGIMENTO INTERNO DAS UNIDADES DO SEAS 25
o o
§ 4 Os divorciados, os judicialmente separados e § 4 O estágio de convivência será acompanhado
os ex-companheiros podem adotar conjuntamente, pela equipe interprofissional a serviço da Justiça da
contanto que acordem sobre a guarda e o regime de Infância e da Juventude, preferencialmente com apoio dos
visitas e desde que o estágio de convivência tenha sido técnicos responsáveis pela execução da política de
iniciado na constância do período de convivência e que garantia do direito à convivência familiar, que
seja comprovada a existência de vínculos de afinidade e apresentarão relatório minucioso acerca da conveniência
afetividade com aquele não detentor da guarda, que do deferimento da medida. (Incluído pela Lei nº 12.010,
justifiquem a excepcionalidade da concessão. (Redação de 2009)
dada pela Lei nº 12.010, de 2009) o
§ 5 O estágio de convivência será cumprido no
o o
§ 5 Nos casos do § 4 deste artigo, desde que território nacional, preferencialmente na comarca de
demonstrado efetivo benefício ao adotando, será residência da criança ou adolescente, ou, a critério do
assegurada a guarda compartilhada, conforme previsto juiz, em cidade limítrofe, respeitada, em qualquer
o
no art. 1.584 da Lei n 10.406, de 10 de janeiro de 2002 - hipótese, a competência do juízo da comarca de
Código Civil . (Redação dada pela Lei nº 12.010, de residência da criança. (Incluído pela Lei nº 13.509, de
2009) 2017)
o
§ 6 A adoção poderá ser deferida ao adotante Art. 47. O vínculo da adoção constitui-se por
que, após inequívoca manifestação de vontade, vier a sentença judicial, que será inscrita no registro civil
falecer no curso do procedimento, antes de prolatada a mediante mandado do qual não se fornecerá certidão.
sentença. (Incluído pela Lei nº 12.010, de 2009) § 1º A inscrição consignará o nome dos adotantes
Art. 43. A adoção será deferida quando apresentar como pais, bem como o nome de seus ascendentes.
reais vantagens para o adotando e fundar-se em motivos § 2º O mandado judicial, que será arquivado,
legítimos. cancelará o registro original do adotado.
Art. 44. Enquanto não der conta de sua o
§ 3 A pedido do adotante, o novo registro poderá
administração e saldar o seu alcance, não pode o tutor ou ser lavrado no Cartório do Registro Civil do Município de
o curador adotar o pupilo ou o curatelado. sua residência. (Redação dada pela Lei nº 12.010, de
Art. 45. A adoção depende do consentimento dos 2009)
pais ou do representante legal do adotando. o
§ 4 Nenhuma observação sobre a origem do ato
§ 1º. O consentimento será dispensado em relação poderá constar nas certidões do registro. (Redação dada
à criança ou adolescente cujos pais sejam desconhecidos pela Lei nº 12.010, de 2009)
ou tenham sido destituídos do pátrio poder poder o
§ 5 A sentença conferirá ao adotado o nome do
familiar . (Expressão substituída pela Lei nº 12.010, de adotante e, a pedido de qualquer deles, poderá
2009) determinar a modificação do prenome. (Redação dada
§ 2º. Em se tratando de adotando maior de doze pela Lei nº 12.010, de 2009)
anos de idade, será também necessário o seu o
§ 6 Caso a modificação de prenome seja
consentimento. requerida pelo adotante, é obrigatória a oitiva do
o o
Art. 46. A adoção será precedida de estágio de adotando, observado o disposto nos §§ 1 e 2 do art. 28
convivência com a criança ou adolescente, pelo prazo desta Lei. (Redação dada pela Lei nº 12.010, de 2009)
máximo de 90 (noventa) dias, observadas a idade da o
§ 7 A adoção produz seus efeitos a partir do
criança ou adolescente e as peculiaridades do trânsito em julgado da sentença constitutiva, exceto na
caso. (Redação dada pela Lei nº 13.509, de 2017) o
hipótese prevista no § 6 do art. 42 desta Lei, caso em
o
§ 1 O estágio de convivência poderá ser que terá força retroativa à data do óbito. (Incluído pela Lei
dispensado se o adotando já estiver sob a tutela ou nº 12.010, de 2009)
guarda legal do adotante durante tempo suficiente para o
§ 8 O processo relativo à adoção assim como
que seja possível avaliar a conveniência da constituição outros a ele relacionados serão mantidos em arquivo,
do vínculo. (Redação dada pela Lei nº 12.010, de 2009) admitindo-se seu armazenamento em microfilme ou por
o
§ 2 A simples guarda de fato não autoriza, por si outros meios, garantida a sua conservação para consulta
só, a dispensa da realização do estágio de a qualquer tempo. (Incluído pela Lei nº 12.010, de 2009)
convivência. (Redação dada pela Lei nº 12.010, de 2009) § 9º Terão prioridade de tramitação os processos
o
§ 2 -A. O prazo máximo estabelecido de adoção em que o adotando for criança ou adolescente
no caput deste artigo pode ser prorrogado por até igual com deficiência ou com doença crônica. (Incluído pela Lei
período, mediante decisão fundamentada da autoridade nº 12.955, de 2014)
judiciária. (Incluído pela Lei nº 13.509, de 2017) § 10. O prazo máximo para conclusão da ação de
o
§ 3 Em caso de adoção por pessoa ou casal adoção será de 120 (cento e vinte) dias, prorrogável uma
residente ou domiciliado fora do País, o estágio de única vez por igual período, mediante decisão
convivência será de, no mínimo, 30 (trinta) dias e, no fundamentada da autoridade judiciária. (Incluído pela Lei
máximo, 45 (quarenta e cinco) dias, prorrogável por até nº 13.509, de 2017)
igual período, uma única vez, mediante decisão Art. 48. O adotado tem direito de conhecer sua
fundamentada da autoridade judiciária. (Redação dada origem biológica, bem como de obter acesso irrestrito ao
pela Lei nº 13.509, de 2017) processo no qual a medida foi aplicada e seus eventuais
o o
§ 3 -A. Ao final do prazo previsto no § 3 deste incidentes, após completar 18 (dezoito) anos. (Redação
artigo, deverá ser apresentado laudo fundamentado pela dada pela Lei nº 12.010, de 2009)
o
equipe mencionada no § 4 deste artigo, que Parágrafo único. O acesso ao processo de adoção
recomendará ou não o deferimento da adoção à poderá ser também deferido ao adotado menor de 18
autoridade judiciária. (Incluído pela Lei nº 13.509, de (dezoito) anos, a seu pedido, assegurada orientação e
2017) assistência jurídica e psicológica. (Incluído pela Lei nº
12.010, de 2009)

www.editoradince.com - Acesse e veja se há novidades a respeito deste material – CUIDADO: cópia é crime.
26 LEGISLAÇÃO ESPECIAL E REGIMENTO INTERNO DAS UNIDADES DO SEAS
Art. 49. A morte dos adotantes não restabelece sempre que possível e recomendável, será colocado sob
o pátrio poder poder familiar dos pais naturais. (Expressão guarda de família cadastrada em programa de
substituída pela Lei nº 12.010, de 2009) acolhimento familiar. (Incluído pela Lei nº 12.010, de
Art. 50. A autoridade judiciária manterá, em cada 2009)
comarca ou foro regional, um registro de crianças e § 12. A alimentação do cadastro e a convocação
adolescentes em condições de serem adotados e outro de criteriosa dos postulantes à adoção serão fiscalizadas
pessoas interessadas na adoção. (Vide Lei nº 12.010, de pelo Ministério Público. (Incluído pela Lei nº 12.010, de
2009) 2009)
§ 1º O deferimento da inscrição dar-se-á após § 13. Somente poderá ser deferida adoção em
prévia consulta aos órgãos técnicos do juizado, ouvido o favor de candidato domiciliado no Brasil não cadastrado
Ministério Público. previamente nos termos desta Lei quando: (Incluído pela
§ 2º Não será deferida a inscrição se o interessado Lei nº 12.010, de 2009)
não satisfizer os requisitos legais, ou verificada qualquer I - se tratar de pedido de adoção
das hipóteses previstas no art. 29. unilateral; (Incluído pela Lei nº 12.010, de 2009)
o
§ 3 A inscrição de postulantes à adoção será II - for formulada por parente com o qual a criança
precedida de um período de preparação psicossocial e ou adolescente mantenha vínculos de afinidade e
jurídica, orientado pela equipe técnica da Justiça da afetividade; (Incluído pela Lei nº 12.010, de 2009)
Infância e da Juventude, preferencialmente com apoio dos III - oriundo o pedido de quem detém a tutela ou
técnicos responsáveis pela execução da política municipal guarda legal de criança maior de 3 (três) anos ou
de garantia do direito à convivência familiar. (Incluído pela adolescente, desde que o lapso de tempo de convivência
Lei nº 12.010, de 2009) comprove a fixação de laços de afinidade e afetividade, e
o
§ 4 Sempre que possível e recomendável, a não seja constatada a ocorrência de má-fé ou qualquer
o
preparação referida no § 3 deste artigo incluirá o contato das situações previstas nos arts. 237 ou 238 desta
com crianças e adolescentes em acolhimento familiar ou Lei. (Incluído pela Lei nº 12.010, de 2009)
institucional em condições de serem adotados, a ser § 14. Nas hipóteses previstas no § 13 deste artigo,
realizado sob a orientação, supervisão e avaliação da o candidato deverá comprovar, no curso do procedimento,
equipe técnica da Justiça da Infância e da Juventude, com que preenche os requisitos necessários à adoção,
apoio dos técnicos responsáveis pelo programa de conforme previsto nesta Lei. (Incluído pela Lei nº 12.010,
acolhimento e pela execução da política municipal de de 2009)
garantia do direito à convivência familiar. (Incluído pela
§ 15. Será assegurada prioridade no cadastro a
Lei nº 12.010, de 2009)
o
pessoas interessadas em adotar criança ou adolescente
§ 5 Serão criados e implementados cadastros com deficiência, com doença crônica ou com
estaduais e nacional de crianças e adolescentes em necessidades específicas de saúde, além de grupo de
condições de serem adotados e de pessoas ou casais irmãos. (Incluído pela Lei nº 13.509, de 2017)
habilitados à adoção. (Incluído pela Lei nº 12.010, de
Art. 51. Considera-se adoção internacional aquela
2009)
o
na qual o pretendente possui residência habitual em país-
§ 6 Haverá cadastros distintos para pessoas ou parte da Convenção de Haia, de 29 de maio de 1993,
casais residentes fora do País, que somente serão Relativa à Proteção das Crianças e à Cooperação em
consultados na inexistência de postulantes nacionais Matéria de Adoção Internacional, promulgada
o
habilitados nos cadastros mencionados no § 5 deste o
pelo Decreto n 3.087, de 21 junho de 1999 , e deseja
artigo. (Incluído pela Lei nº 12.010, de 2009) adotar criança em outro país-parte da
o
§ 7 As autoridades estaduais e federais em Convenção. (Redação dada pela Lei nº 13.509, de 2017)
matéria de adoção terão acesso integral aos cadastros, o
§ 1 A adoção internacional de criança ou
incumbindo-lhes a troca de informações e a cooperação adolescente brasileiro ou domiciliado no Brasil somente
mútua, para melhoria do sistema. (Incluído pela Lei nº terá lugar quando restar comprovado: (Redação dada
12.010, de 2009) pela Lei nº 12.010, de 2009)
o
§ 8 A autoridade judiciária providenciará, no prazo I - que a colocação em família adotiva é a solução
de 48 (quarenta e oito) horas, a inscrição das crianças e adequada ao caso concreto; (Redação dada pela Lei nº
adolescentes em condições de serem adotados que não 13.509, de 2017)
tiveram colocação familiar na comarca de origem, e das
II - que foram esgotadas todas as possibilidades de
pessoas ou casais que tiveram deferida sua habilitação à
colocação da criança ou adolescente em família adotiva
adoção nos cadastros estadual e nacional referidos no §
o brasileira, com a comprovação, certificada nos autos, da
5 deste artigo, sob pena de responsabilidade. (Incluído
inexistência de adotantes habilitados residentes no Brasil
pela Lei nº 12.010, de 2009)
o
com perfil compatível com a criança ou adolescente, após
§ 9 Compete à Autoridade Central Estadual zelar consulta aos cadastros mencionados nesta Lei; (Redação
pela manutenção e correta alimentação dos cadastros, dada pela Lei nº 13.509, de 2017)
com posterior comunicação à Autoridade Central Federal
III - que, em se tratando de adoção de adolescente,
Brasileira. (Incluído pela Lei nº 12.010, de 2009)
este foi consultado, por meios adequados ao seu estágio
§ 10. Consultados os cadastros e verificada a de desenvolvimento, e que se encontra preparado para a
ausência de pretendentes habilitados residentes no País medida, mediante parecer elaborado por equipe
com perfil compatível e interesse manifesto pela adoção interprofissional, observado o disposto nos §§ 1 e 2 do
o o

de criança ou adolescente inscrito nos cadastros art. 28 desta Lei. (Incluída pela Lei nº 12.010, de 2009)
existentes, será realizado o encaminhamento da criança o
§ 2 Os brasileiros residentes no exterior terão
ou adolescente à adoção internacional. (Redação dada
preferência aos estrangeiros, nos casos de adoção
pela Lei nº 13.509, de 2017)
internacional de criança ou adolescente
§ 11. Enquanto não localizada pessoa ou casal brasileiro. (Redação dada pela Lei nº 12.010, de 2009)
interessado em sua adoção, a criança ou o adolescente,

www.editoradince.com - Acesse e veja se há novidades a respeito deste material – CUIDADO: cópia é crime.
LEGISLAÇÃO ESPECIAL E REGIMENTO INTERNO DAS UNIDADES DO SEAS 27
o
§ 3 A adoção internacional pressupõe a habilitação à adoção internacional, com posterior
intervenção das Autoridades Centrais Estaduais e Federal comunicação às Autoridades Centrais Estaduais e
em matéria de adoção internacional. (Redação dada pela publicação nos órgãos oficiais de imprensa e em sítio
Lei nº 12.010, de 2009) próprio da internet. (Incluído pela Lei nº 12.010, de 2009)
o
§ 4º (Revogado pela Lei nº 12.010, de 2009) § 3 Somente será admissível o credenciamento
Art. 52. A adoção internacional observará o de organismos que: (Incluída pela Lei nº 12.010, de
procedimento previsto nos arts. 165 a 170 desta Lei, com 2009)
as seguintes adaptações: (Redação dada pela Lei nº I - sejam oriundos de países que ratificaram a
12.010, de 2009) Convenção de Haia e estejam devidamente credenciados
I - a pessoa ou casal estrangeiro, interessado em pela Autoridade Central do país onde estiverem sediados
adotar criança ou adolescente brasileiro, deverá formular e no país de acolhida do adotando para atuar em adoção
pedido de habilitação à adoção perante a Autoridade internacional no Brasil; (Incluída pela Lei nº 12.010, de
Central em matéria de adoção internacional no país de 2009)
acolhida, assim entendido aquele onde está situada sua II - satisfizerem as condições de integridade moral,
residência habitual; (Incluída pela Lei nº 12.010, de 2009) competência profissional, experiência e responsabilidade
II - se a Autoridade Central do país de acolhida exigidas pelos países respectivos e pela Autoridade
considerar que os solicitantes estão habilitados e aptos Central Federal Brasileira; (Incluída pela Lei nº 12.010, de
para adotar, emitirá um relatório que contenha 2009)
informações sobre a identidade, a capacidade jurídica e III - forem qualificados por seus padrões éticos e
adequação dos solicitantes para adotar, sua situação sua formação e experiência para atuar na área de adoção
pessoal, familiar e médica, seu meio social, os motivos internacional; (Incluída pela Lei nº 12.010, de 2009)
que os animam e sua aptidão para assumir uma adoção IV - cumprirem os requisitos exigidos pelo
internacional; (Incluída pela Lei nº 12.010, de 2009) ordenamento jurídico brasileiro e pelas normas
III - a Autoridade Central do país de acolhida estabelecidas pela Autoridade Central Federal
enviará o relatório à Autoridade Central Estadual, com Brasileira. (Incluída pela Lei nº 12.010, de 2009)
cópia para a Autoridade Central Federal o
§ 4 Os organismos credenciados deverão
Brasileira; (Incluída pela Lei nº 12.010, de 2009) ainda: (Incluído pela Lei nº 12.010, de 2009)
IV - o relatório será instruído com toda a I - perseguir unicamente fins não lucrativos, nas
documentação necessária, incluindo estudo psicossocial condições e dentro dos limites fixados pelas autoridades
elaborado por equipe interprofissional habilitada e cópia competentes do país onde estiverem sediados, do país de
autenticada da legislação pertinente, acompanhada da acolhida e pela Autoridade Central Federal
respectiva prova de ; (Incluída pela Lei nº 12.010, de Brasileira; (Incluída pela Lei nº 12.010, de 2009)
2009)
II - ser dirigidos e administrados por pessoas
V - os documentos em língua estrangeira serão qualificadas e de reconhecida idoneidade moral, com
devidamente autenticados pela autoridade consular, comprovada formação ou experiência para atuar na área
observados os tratados e convenções internacionais, e de adoção internacional, cadastradas pelo Departamento
acompanhados da respectiva tradução, por tradutor de Polícia Federal e aprovadas pela Autoridade Central
público juramentado; (Incluída pela Lei nº 12.010, de Federal Brasileira, mediante publicação de portaria do
2009) órgão federal competente; (Incluída pela Lei nº 12.010, de
VI - a Autoridade Central Estadual poderá fazer 2009)
exigências e solicitar complementação sobre o estudo III - estar submetidos à supervisão das autoridades
psicossocial do postulante estrangeiro à adoção, já competentes do país onde estiverem sediados e no país
realizado no país de acolhida; (Incluída pela Lei nº de acolhida, inclusive quanto à sua composição,
12.010, de 2009) funcionamento e situação financeira; (Incluída pela Lei nº
VII - verificada, após estudo realizado pela 12.010, de 2009)
Autoridade Central Estadual, a compatibilidade da IV - apresentar à Autoridade Central Federal
legislação estrangeira com a nacional, além do Brasileira, a cada ano, relatório geral das atividades
preenchimento por parte dos postulantes à medida dos desenvolvidas, bem como relatório de acompanhamento
requisitos objetivos e subjetivos necessários ao seu das adoções internacionais efetuadas no período, cuja
deferimento, tanto à luz do que dispõe esta Lei como da cópia será encaminhada ao Departamento de Polícia
legislação do país de acolhida, será expedido laudo de Federal; (Incluída pela Lei nº 12.010, de 2009)
habilitação à adoção internacional, que terá validade por,
V - enviar relatório pós-adotivo semestral para a
no máximo, 1 (um) ano; (Incluída pela Lei nº 12.010, de
Autoridade Central Estadual, com cópia para a Autoridade
2009)
Central Federal Brasileira, pelo período mínimo de 2
VIII - de posse do laudo de habilitação, o (dois) anos. O envio do relatório será mantido até a
interessado será autorizado a formalizar pedido de juntada de cópia autenticada do registro civil,
adoção perante o Juízo da Infância e da Juventude do estabelecendo a cidadania do país de acolhida para o
local em que se encontra a criança ou adolescente, adotado; (Incluída pela Lei nº 12.010, de 2009)
conforme indicação efetuada pela Autoridade Central
VI - tomar as medidas necessárias para garantir
Estadual. (Incluída pela Lei nº 12.010, de 2009)
o
que os adotantes encaminhem à Autoridade Central
§ 1 Se a legislação do país de acolhida assim o Federal Brasileira cópia da certidão de registro de
autorizar, admite-se que os pedidos de habilitação à nascimento estrangeira e do certificado de nacionalidade
adoção internacional sejam intermediados por organismos tão logo lhes sejam concedidos. (Incluída pela Lei nº
credenciados. (Incluída pela Lei nº 12.010, de 2009) 12.010, de 2009)
o
§ 2 Incumbe à Autoridade Central Federal o
§ 5 A não apresentação dos relatórios referidos no
Brasileira o credenciamento de organismos nacionais e o
§ 4 deste artigo pelo organismo credenciado poderá
estrangeiros encarregados de intermediar pedidos de

www.editoradince.com - Acesse e veja se há novidades a respeito deste material – CUIDADO: cópia é crime.
28 LEGISLAÇÃO ESPECIAL E REGIMENTO INTERNO DAS UNIDADES DO SEAS
acarretar a suspensão de seu credenciamento. (Incluído processo de adoção tenha sido processado em
pela Lei nº 12.010, de 2009) conformidade com a legislação vigente no país de
o
§ 6 O credenciamento de organismo nacional ou residência e atendido o disposto na Alínea “c” do Artigo 17
estrangeiro encarregado de intermediar pedidos de da referida Convenção, será automaticamente
adoção internacional terá validade de 2 (dois) recepcionada com o reingresso no Brasil. (Incluído pela
anos. (Incluído pela Lei nº 12.010, de 2009) Lei nº 12.010, de 2009)
o o
§ 7 A renovação do credenciamento poderá ser § 1 Caso não tenha sido atendido o disposto na
concedida mediante requerimento protocolado na Alínea “c” do Artigo 17 da Convenção de Haia, deverá a
Autoridade Central Federal Brasileira nos 60 (sessenta) sentença ser homologada pelo Superior Tribunal de
dias anteriores ao término do respectivo prazo de Justiça. (Incluído pela Lei nº 12.010, de 2009)
o
validade. (Incluído pela Lei nº 12.010, de 2009) § 2 O pretendente brasileiro residente no exterior
o em país não ratificante da Convenção de Haia, uma vez
§ 8 Antes de transitada em julgado a decisão que
concedeu a adoção internacional, não será permitida a reingressado no Brasil, deverá requerer a homologação
saída do adotando do território nacional. (Incluído pela Lei da sentença estrangeira pelo Superior Tribunal de
nº 12.010, de 2009) Justiça. (Incluído pela Lei nº 12.010, de 2009)
o Art. 52-C. Nas adoções internacionais, quando o
§ 9 Transitada em julgado a decisão, a autoridade
judiciária determinará a expedição de alvará com Brasil for o país de acolhida, a decisão da autoridade
autorização de viagem, bem como para obtenção de competente do país de origem da criança ou do
passaporte, constando, obrigatoriamente, as adolescente será conhecida pela Autoridade Central
características da criança ou adolescente adotado, como Estadual que tiver processado o pedido de habilitação dos
idade, cor, sexo, eventuais sinais ou traços peculiares, pais adotivos, que comunicará o fato à Autoridade Central
assim como foto recente e a aposição da impressão Federal e determinará as providências necessárias à
digital do seu polegar direito, instruindo o documento com expedição do Certificado de Naturalização
cópia autenticada da decisão e certidão de trânsito em Provisório. (Incluído pela Lei nº 12.010, de 2009)
o
julgado. (Incluído pela Lei nº 12.010, de 2009) § 1 A Autoridade Central Estadual, ouvido o
§ 10. A Autoridade Central Federal Brasileira Ministério Público, somente deixará de reconhecer os
poderá, a qualquer momento, solicitar informações sobre efeitos daquela decisão se restar demonstrado que a
a situação das crianças e adolescentes adotados (Incluído adoção é manifestamente contrária à ordem pública ou
pela Lei nº 12.010, de 2009) não atende ao interesse superior da criança ou do
adolescente. (Incluído pela Lei nº 12.010, de 2009)
§ 11. A cobrança de valores por parte dos o
organismos credenciados, que sejam considerados § 2 Na hipótese de não reconhecimento da
o
abusivos pela Autoridade Central Federal Brasileira e que adoção, prevista no § 1 deste artigo, o Ministério Público
não estejam devidamente comprovados, é causa de seu deverá imediatamente requerer o que for de direito para
descredenciamento. (Incluído pela Lei nº 12.010, de resguardar os interesses da criança ou do adolescente,
2009) comunicando-se as providências à Autoridade Central
Estadual, que fará a comunicação à Autoridade Central
§ 12. Uma mesma pessoa ou seu cônjuge não
Federal Brasileira e à Autoridade Central do país de
podem ser representados por mais de uma entidade
origem. (Incluído pela Lei nº 12.010, de 2009)
credenciada para atuar na cooperação em adoção
internacional. (Incluído pela Lei nº 12.010, de 2009) Art. 52-D. Nas adoções internacionais, quando o
Brasil for o país de acolhida e a adoção não tenha sido
§ 13. A habilitação de postulante estrangeiro ou
deferida no país de origem porque a sua legislação a
domiciliado fora do Brasil terá validade máxima de 1 (um)
delega ao país de acolhida, ou, ainda, na hipótese de,
ano, podendo ser renovada. (Incluído pela Lei nº 12.010,
mesmo com decisão, a criança ou o adolescente ser
de 2009)
oriundo de país que não tenha aderido à Convenção
§ 14. É vedado o contato direto de representantes referida, o processo de adoção seguirá as regras da
de organismos de adoção, nacionais ou estrangeiros, com adoção nacional. (Incluído pela Lei nº 12.010, de 2009)
dirigentes de programas de acolhimento institucional ou
Capítulo IV
familiar, assim como com crianças e adolescentes em
condições de serem adotados, sem a devida autorização Do Direito à Educação, à Cultura, ao Esporte e ao
judicial. (Incluído pela Lei nº 12.010, de 2009) Lazer
§ 15. A Autoridade Central Federal Brasileira Art. 53. A criança e o adolescente têm direito à
poderá limitar ou suspender a concessão de novos educação, visando ao pleno desenvolvimento de sua
credenciamentos sempre que julgar necessário, mediante pessoa, preparo para o exercício da cidadania e
ato administrativo fundamentado. (Incluído pela Lei nº qualificação para o trabalho, assegurando-se-lhes:
12.010, de 2009) I - igualdade de condições para o acesso e
Art. 52-A. É vedado, sob pena de responsabilidade permanência na escola;
e descredenciamento, o repasse de recursos II - direito de ser respeitado por seus educadores;
provenientes de organismos estrangeiros encarregados III - direito de contestar critérios avaliativos,
de intermediar pedidos de adoção internacional a podendo recorrer às instâncias escolares superiores;
organismos nacionais ou a pessoas físicas. (Incluído pela
IV - direito de organização e participação em
Lei nº 12.010, de 2009)
entidades estudantis;
Parágrafo único. Eventuais repasses somente
V - acesso à escola pública e gratuita, próxima de
poderão ser efetuados via Fundo dos Direitos da Criança
sua residência, garantindo-se vagas no mesmo
e do Adolescente e estarão sujeitos às deliberações do
estabelecimento a irmãos que frequentem a mesma etapa
respectivo Conselho de Direitos da Criança e do
ou ciclo de ensino da educação básica. (Redação dada
Adolescente (Incluído pela Lei nº 12.010, de 2009)
pela Lei nº 13.845, de 2019)
Art. 52-B. A adoção por brasileiro residente no
exterior em país ratificante da Convenção de Haia, cujo

www.editoradince.com - Acesse e veja se há novidades a respeito deste material – CUIDADO: cópia é crime.
LEGISLAÇÃO ESPECIAL E REGIMENTO INTERNO DAS UNIDADES DO SEAS 29
Parágrafo único. É direito dos pais ou responsáveis Art. 60. É proibido qualquer trabalho a menores de
ter ciência do processo pedagógico, bem como participar quatorze anos de idade, salvo na condição de
da definição das propostas educacionais. aprendiz. (Vide Constituição Federal)
Art. 53-A. É dever da instituição de ensino, clubes e Art. 61. A proteção ao trabalho dos adolescentes é
agremiações recreativas e de estabelecimentos regulada por legislação especial, sem prejuízo do disposto
congêneres assegurar medidas de conscientização, nesta Lei.
prevenção e enfrentamento ao uso ou dependência de Art. 62. Considera-se aprendizagem a formação
drogas ilícitas. (Incluído pela Lei nº 13.840, de 2019) técnico-profissional ministrada segundo as diretrizes e
Art. 54. É dever do Estado assegurar à criança e ao bases da legislação de educação em vigor.
adolescente: Art. 63. A formação técnico-profissional obedecerá
I - ensino fundamental, obrigatório e gratuito, aos seguintes princípios:
inclusive para os que a ele não tiveram acesso na idade I - garantia de acesso e freqüência obrigatória ao
própria; ensino regular;
II - progressiva extensão da obrigatoriedade e II - atividade compatível com o desenvolvimento do
gratuidade ao ensino médio; adolescente;
III - atendimento educacional especializado aos III - horário especial para o exercício das
portadores de deficiência, preferencialmente na rede atividades.
regular de ensino;
Art. 64. Ao adolescente até quatorze anos de idade
IV – atendimento em creche e pré-escola às é assegurada bolsa de aprendizagem.
crianças de zero a cinco anos de idade; (Redação dada
Art. 65. Ao adolescente aprendiz, maior de
pela Lei nº 13.306, de 2016)
quatorze anos, são assegurados os direitos trabalhistas e
V - acesso aos níveis mais elevados do ensino, da previdenciários.
pesquisa e da criação artística, segundo a capacidade de
Art. 66. Ao adolescente portador de deficiência é
cada um;
assegurado trabalho protegido.
VI - oferta de ensino noturno regular, adequado às
Art. 67. Ao adolescente empregado, aprendiz, em
condições do adolescente trabalhador;
regime familiar de trabalho, aluno de escola técnica,
VII - atendimento no ensino fundamental, através assistido em entidade governamental ou não-
de programas suplementares de material didático-escolar, governamental, é vedado trabalho:
transporte, alimentação e assistência à saúde.
I - noturno, realizado entre as vinte e duas horas de
§ 1º O acesso ao ensino obrigatório e gratuito é um dia e as cinco horas do dia seguinte;
direito público subjetivo.
II - perigoso, insalubre ou penoso;
§ 2º O não oferecimento do ensino obrigatório pelo
III - realizado em locais prejudiciais à sua formação
poder público ou sua oferta irregular importa
e ao seu desenvolvimento físico, psíquico, moral e social;
responsabilidade da autoridade competente.
IV - realizado em horários e locais que não
§ 3º Compete ao poder público recensear os
permitam a freqüência à escola.
educandos no ensino fundamental, fazer-lhes a chamada
e zelar, junto aos pais ou responsável, pela freqüência à Art. 68. O programa social que tenha por base o
escola. trabalho educativo, sob responsabilidade de entidade
governamental ou não-governamental sem fins lucrativos,
Art. 55. Os pais ou responsável têm a obrigação de
deverá assegurar ao adolescente que dele participe
matricular seus filhos ou pupilos na rede regular de
condições de capacitação para o exercício de atividade
ensino.
regular remunerada.
Art. 56. Os dirigentes de estabelecimentos de
§ 1º Entende-se por trabalho educativo a atividade
ensino fundamental comunicarão ao Conselho Tutelar os
laboral em que as exigências pedagógicas relativas ao
casos de:
desenvolvimento pessoal e social do educando
I - maus-tratos envolvendo seus alunos; prevalecem sobre o aspecto produtivo.
II - reiteração de faltas injustificadas e de evasão § 2º A remuneração que o adolescente recebe pelo
escolar, esgotados os recursos escolares; trabalho efetuado ou a participação na venda dos
III - elevados níveis de repetência. produtos de seu trabalho não desfigura o caráter
Art. 57. O poder público estimulará pesquisas, educativo.
experiências e novas propostas relativas a calendário, Art. 69. O adolescente tem direito à
seriação, currículo, metodologia, didática e avaliação, com profissionalização e à proteção no trabalho, observados
vistas à inserção de crianças e adolescentes excluídos do os seguintes aspectos, entre outros:
ensino fundamental obrigatório. I - respeito à condição peculiar de pessoa em
Art. 58. No processo educacional respeitar-se-ão desenvolvimento;
os valores culturais, artísticos e históricos próprios do II - capacitação profissional adequada ao mercado
contexto social da criança e do adolescente, garantindo- de trabalho.
se a estes a liberdade da criação e o acesso às fontes de
Título III
cultura.
Da Prevenção
Art. 59. Os municípios, com apoio dos estados e da
União, estimularão e facilitarão a destinação de recursos Capítulo I
e espaços para programações culturais, esportivas e de Disposições Gerais
lazer voltadas para a infância e a juventude. Art. 70. É dever de todos prevenir a ocorrência de
Capítulo V ameaça ou violação dos direitos da criança e do
Do Direito à Profissionalização e à Proteção no adolescente.
Trabalho

www.editoradince.com - Acesse e veja se há novidades a respeito deste material – CUIDADO: cópia é crime.
30 LEGISLAÇÃO ESPECIAL E REGIMENTO INTERNO DAS UNIDADES DO SEAS
Art. 70-A. A União, os Estados, o Distrito Federal e Art. 71. A criança e o adolescente têm direito a
os Municípios deverão atuar de forma articulada na informação, cultura, lazer, esportes, diversões,
elaboração de políticas públicas e na execução de ações espetáculos e produtos e serviços que respeitem sua
destinadas a coibir o uso de castigo físico ou de condição peculiar de pessoa em desenvolvimento.
tratamento cruel ou degradante e difundir formas não Art. 72. As obrigações previstas nesta Lei não
violentas de educação de crianças e de adolescentes, excluem da prevenção especial outras decorrentes dos
tendo como principais ações: (Incluído pela Lei nº 13.010, princípios por ela adotados.
de 2014)
Art. 73. A inobservância das normas de prevenção
I - a promoção de campanhas educativas importará em responsabilidade da pessoa física ou
permanentes para a divulgação do direito da criança e do jurídica, nos termos desta Lei.
adolescente de serem educados e cuidados sem o uso de
Capítulo II
castigo físico ou de tratamento cruel ou degradante e dos
instrumentos de proteção aos direitos humanos; (Incluído Da Prevenção Especial
pela Lei nº 13.010, de 2014) Seção I
II - a integração com os órgãos do Poder Judiciário, Da informação, Cultura, Lazer, Esportes, Diversões e
do Ministério Público e da Defensoria Pública, com o Espetáculos
Conselho Tutelar, com os Conselhos de Direitos da Art. 74. O poder público, através do órgão
Criança e do Adolescente e com as entidades não competente, regulará as diversões e espetáculos
governamentais que atuam na promoção, proteção e públicos, informando sobre a natureza deles, as faixas
defesa dos direitos da criança e do adolescente; (Incluído etárias a que não se recomendem, locais e horários em
pela Lei nº 13.010, de 2014) que sua apresentação se mostre inadequada.
III - a formação continuada e a capacitação dos Parágrafo único. Os responsáveis pelas diversões
profissionais de saúde, educação e assistência social e e espetáculos públicos deverão afixar, em lugar visível e
dos demais agentes que atuam na promoção, proteção e de fácil acesso, à entrada do local de exibição, informação
defesa dos direitos da criança e do adolescente para o destacada sobre a natureza do espetáculo e a faixa etária
desenvolvimento das competências necessárias à especificada no certificado de classificação.
prevenção, à identificação de evidências, ao diagnóstico e Art. 75. Toda criança ou adolescente terá acesso
ao enfrentamento de todas as formas de violência contra às diversões e espetáculos públicos classificados como
a criança e o adolescente; (Incluído pela Lei nº 13.010, de adequados à sua faixa etária.
2014)
Parágrafo único. As crianças menores de dez anos
IV - o apoio e o incentivo às práticas de resolução somente poderão ingressar e permanecer nos locais de
pacífica de conflitos que envolvam violência contra a apresentação ou exibição quando acompanhadas dos
criança e o adolescente; (Incluído pela Lei nº 13.010, de pais ou responsável.
2014)
Art. 76. As emissoras de rádio e televisão somente
V - a inclusão, nas políticas públicas, de ações que exibirão, no horário recomendado para o público infanto
visem a garantir os direitos da criança e do adolescente, juvenil, programas com finalidades educativas, artísticas,
desde a atenção pré-natal, e de atividades junto aos pais culturais e informativas.
e responsáveis com o objetivo de promover a informação,
a reflexão, o debate e a orientação sobre alternativas ao Parágrafo único. Nenhum espetáculo será
uso de castigo físico ou de tratamento cruel ou apresentado ou anunciado sem aviso de sua
degradante no processo educativo; (Incluído pela Lei nº classificação, antes de sua transmissão, apresentação ou
13.010, de 2014) exibição.
VI - a promoção de espaços intersetoriais locais Art. 77. Os proprietários, diretores, gerentes e
para a articulação de ações e a elaboração de planos de funcionários de empresas que explorem a venda ou
atuação conjunta focados nas famílias em situação de aluguel de fitas de programação em vídeo cuidarão para
violência, com participação de profissionais de saúde, de que não haja venda ou locação em desacordo com a
assistência social e de educação e de órgãos de classificação atribuída pelo órgão competente.
promoção, proteção e defesa dos direitos da criança e do Parágrafo único. As fitas a que alude este artigo
adolescente. (Incluído pela Lei nº 13.010, de 2014) deverão exibir, no invólucro, informação sobre a natureza
Parágrafo único. As famílias com crianças e da obra e a faixa etária a que se destinam.
adolescentes com deficiência terão prioridade de Art. 78. As revistas e publicações contendo material
atendimento nas ações e políticas públicas de prevenção impróprio ou inadequado a crianças e adolescentes
e proteção. (Incluído pela Lei nº 13.010, de 2014) deverão ser comercializadas em embalagem lacrada, com
Art. 70-B. As entidades, públicas e privadas, que a advertência de seu conteúdo.
atuem nas áreas a que se refere o art. 71, dentre outras, Parágrafo único. As editoras cuidarão para que as
devem contar, em seus quadros, com pessoas capas que contenham mensagens pornográficas ou
capacitadas a reconhecer e comunicar ao Conselho obscenas sejam protegidas com embalagem opaca.
Tutelar suspeitas ou casos de maus-tratos praticados Art. 79. As revistas e publicações destinadas ao
contra crianças e adolescentes. (Incluído pela Lei nº público infanto-juvenil não poderão conter ilustrações,
13.046, de 2014) fotografias, legendas, crônicas ou anúncios de bebidas
Parágrafo único. São igualmente responsáveis pela alcoólicas, tabaco, armas e munições, e deverão respeitar
comunicação de que trata este artigo, as pessoas os valores éticos e sociais da pessoa e da família.
encarregadas, por razão de cargo, função, ofício, Art. 80. Os responsáveis por estabelecimentos que
ministério, profissão ou ocupação, do cuidado, assistência explorem comercialmente bilhar, sinuca ou congênere ou
ou guarda de crianças e adolescentes, punível, na forma por casas de jogos, assim entendidas as que realizem
deste Estatuto, o injustificado retardamento ou omissão, apostas, ainda que eventualmente, cuidarão para que não
culposos ou dolosos. (Incluído pela Lei nº 13.046, de seja permitida a entrada e a permanência de crianças e
2014)

www.editoradince.com - Acesse e veja se há novidades a respeito deste material – CUIDADO: cópia é crime.
LEGISLAÇÃO ESPECIAL E REGIMENTO INTERNO DAS UNIDADES DO SEAS 31
adolescentes no local, afixando aviso para orientação do governamentais, da União, dos estados, do Distrito
público. Federal e dos municípios.
Seção II Art. 87. São linhas de ação da política de
Dos Produtos e Serviços atendimento:
Art. 81. É proibida a venda à criança ou ao I - políticas sociais básicas;
adolescente de: II - serviços, programas, projetos e benefícios de
I - armas, munições e explosivos; assistência social de garantia de proteção social e de
prevenção e redução de violações de direitos, seus
II - bebidas alcoólicas;
agravamentos ou reincidências; (Redação dada pela Lei
III - produtos cujos componentes possam causar nº 13.257, de 2016)
dependência física ou psíquica ainda que por utilização
III - serviços especiais de prevenção e atendimento
indevida;
médico e psicossocial às vítimas de negligência, maus-
IV - fogos de estampido e de artifício, exceto tratos, exploração, abuso, crueldade e opressão;
aqueles que pelo seu reduzido potencial sejam incapazes
IV - serviço de identificação e localização de pais,
de provocar qualquer dano físico em caso de utilização
responsável, crianças e adolescentes desaparecidos;
indevida;
V - proteção jurídico-social por entidades de defesa
V - revistas e publicações a que alude o art. 78;
dos direitos da criança e do adolescente.
VI - bilhetes lotéricos e equivalentes.
VI - políticas e programas destinados a prevenir ou
Art. 82. É proibida a hospedagem de criança ou abreviar o período de afastamento do convívio familiar e a
adolescente em hotel, motel, pensão ou estabelecimento garantir o efetivo exercício do direito à convivência familiar
congênere, salvo se autorizado ou acompanhado pelos de crianças e adolescentes; (Incluído pela Lei nº 12.010,
pais ou responsável. de 2009)
Seção III VII - campanhas de estímulo ao acolhimento sob
Da Autorização para Viajar forma de guarda de crianças e adolescentes afastados do
Art. 83. Nenhuma criança ou adolescente menor de convívio familiar e à adoção, especificamente inter-racial,
16 (dezesseis) anos poderá viajar para fora da comarca de crianças maiores ou de adolescentes, com
onde reside desacompanhado dos pais ou dos necessidades específicas de saúde ou com deficiências e
responsáveis sem expressa autorização de grupos de irmãos. (Incluído pela Lei nº 12.010, de
judicial. (Redação dada pela Lei nº 13.812, de 2019) 2009)
§ 1º A autorização não será exigida quando: Art. 88. São diretrizes da política de atendimento:
a) tratar-se de comarca contígua à da residência da I - municipalização do atendimento;
criança ou do adolescente menor de 16 (dezesseis) anos, II - criação de conselhos municipais, estaduais e
se na mesma unidade da Federação, ou incluída na nacional dos direitos da criança e do adolescente, órgãos
mesma região metropolitana; (Redação dada pela Lei nº deliberativos e controladores das ações em todos os
13.812, de 2019) níveis, assegurada a participação popular paritária por
b) a criança ou o adolescente menor de 16 meio de organizações representativas, segundo leis
(dezesseis) anos estiver acompanhado: (Redação dada federal, estaduais e municipais;
pela Lei nº 13.812, de 2019) III - criação e manutenção de programas
1) de ascendente ou colateral maior, até o terceiro específicos, observada a descentralização político-
grau, comprovado documentalmente o parentesco; administrativa;
2) de pessoa maior, expressamente autorizada IV - manutenção de fundos nacional, estaduais e
pelo pai, mãe ou responsável. municipais vinculados aos respectivos conselhos dos
§ 2º A autoridade judiciária poderá, a pedido dos direitos da criança e do adolescente;
pais ou responsável, conceder autorização válida por dois V - integração operacional de órgãos do Judiciário,
anos. Ministério Público, Defensoria, Segurança Pública e
Art. 84. Quando se tratar de viagem ao exterior, a Assistência Social, preferencialmente em um mesmo
autorização é dispensável, se a criança ou adolescente: local, para efeito de agilização do atendimento inicial a
adolescente a quem se atribua autoria de ato infracional;
I - estiver acompanhado de ambos os pais ou
responsável; VI - integração operacional de órgãos do Judiciário,
Ministério Público, Defensoria, Conselho Tutelar e
II - viajar na companhia de um dos pais, autorizado encarregados da execução das políticas sociais básicas e
expressamente pelo outro através de documento com de assistência social, para efeito de agilização do
firma reconhecida. atendimento de crianças e de adolescentes inseridos em
Art. 85. Sem prévia e expressa autorização judicial, programas de acolhimento familiar ou institucional, com
nenhuma criança ou adolescente nascido em território vista na sua rápida reintegração à família de origem ou,
nacional poderá sair do País em companhia de se tal solução se mostrar comprovadamente inviável, sua
estrangeiro residente ou domiciliado no exterior. colocação em família substituta, em quaisquer das
Parte Especial modalidades previstas no art. 28 desta Lei; (Redação
Título I dada pela Lei nº 12.010, de 2009)
Da Política de Atendimento VII - mobilização da opinião pública para a
indispensável participação dos diversos segmentos da
Capítulo I
sociedade. (Incluído pela Lei nº 12.010, de 2009)
Disposições Gerais
VIII - especialização e formação continuada dos
Art. 86. A política de atendimento dos direitos da profissionais que trabalham nas diferentes áreas da
criança e do adolescente far-se-á através de um conjunto atenção à primeira infância, incluindo os conhecimentos
articulado de ações governamentais e não-

www.editoradince.com - Acesse e veja se há novidades a respeito deste material – CUIDADO: cópia é crime.
32 LEGISLAÇÃO ESPECIAL E REGIMENTO INTERNO DAS UNIDADES DO SEAS
sobre direitos da criança e sobre desenvolvimento Ministério Público e pela Justiça da Infância e da
infantil; (Incluído pela Lei nº 13.257, de 2016) Juventude; (Incluído pela Lei nº 12.010, de 2009)
IX - formação profissional com abrangência dos III - em se tratando de programas de acolhimento
diversos direitos da criança e do adolescente que institucional ou familiar, serão considerados os índices de
favoreça a intersetorialidade no atendimento da criança e sucesso na reintegração familiar ou de adaptação à
do adolescente e seu desenvolvimento integral; (Incluído família substituta, conforme o caso. (Incluído pela Lei nº
pela Lei nº 13.257, de 2016) 12.010, de 2009)
X - realização e divulgação de pesquisas sobre Art. 91. As entidades não-governamentais somente
desenvolvimento infantil e sobre prevenção da poderão funcionar depois de registradas no Conselho
violência. (Incluído pela Lei nº 13.257, de 2016) Municipal dos Direitos da Criança e do Adolescente, o
Art. 89. A função de membro do conselho nacional qual comunicará o registro ao Conselho Tutelar e à
e dos conselhos estaduais e municipais dos direitos da autoridade judiciária da respectiva localidade.
o
criança e do adolescente é considerada de interesse § 1 Será negado o registro à entidade
público relevante e não será remunerada. que: (Incluído pela Lei nº 12.010, de 2009)
Capítulo II a) não ofereça instalações físicas em condições
Das Entidades de Atendimento adequadas de habitabilidade, higiene, salubridade e
segurança;
Seção I
b) não apresente plano de trabalho compatível com
Disposições Gerais
os princípios desta Lei;
Art. 90. As entidades de atendimento são
c) esteja irregularmente constituída;
responsáveis pela manutenção das próprias unidades,
assim como pelo planejamento e execução de programas d) tenha em seus quadros pessoas inidôneas.
de proteção e sócio-educativos destinados a crianças e e) não se adequar ou deixar de cumprir as
adolescentes, em regime de: resoluções e deliberações relativas à modalidade de
I - orientação e apoio sócio-familiar; atendimento prestado expedidas pelos Conselhos de
Direitos da Criança e do Adolescente, em todos os
II - apoio sócio-educativo em meio aberto;
níveis. (Incluída pela Lei nº 12.010, de 2009)
III - colocação familiar; o
§ 2 O registro terá validade máxima de 4 (quatro)
IV - acolhimento institucional; (Redação dada pela anos, cabendo ao Conselho Municipal dos Direitos da
Lei nº 12.010, de 2009) Criança e do Adolescente, periodicamente, reavaliar o
V - prestação de serviços à comunidade; (Redação cabimento de sua renovação, observado o disposto no §
o
dada pela Lei nº 12.594, de 2012) 1 deste artigo. (Incluído pela Lei nº 12.010, de 2009)
VI - liberdade assistida; (Redação dada pela Lei nº Art. 92. As entidades que desenvolvam programas
12.594, de 2012) de acolhimento familiar ou institucional deverão adotar os
VII - semiliberdade; e (Redação dada pela Lei nº seguintes princípios: (Redação dada pela Lei nº 12.010,
12.594, de 2012) de 2009)
VIII - internação. (Incluído pela Lei nº 12.594, de I - preservação dos vínculos familiares e promoção
2012) da reintegração familiar; (Redação dada pela Lei nº
o
§ 1 As entidades governamentais e não 12.010, de 2009)
governamentais deverão proceder à inscrição de seus II - integração em família substituta, quando
programas, especificando os regimes de atendimento, na esgotados os recursos de manutenção na família natural
forma definida neste artigo, no Conselho Municipal dos ou extensa; (Redação dada pela Lei nº 12.010, de 2009)
Direitos da Criança e do Adolescente, o qual manterá III - atendimento personalizado e em pequenos
registro das inscrições e de suas alterações, do que fará grupos;
comunicação ao Conselho Tutelar e à autoridade IV - desenvolvimento de atividades em regime de
judiciária. (Incluído pela Lei nº 12.010, de 2009) co-educação;
o
§ 2 Os recursos destinados à implementação e V - não desmembramento de grupos de irmãos;
manutenção dos programas relacionados neste artigo
VI - evitar, sempre que possível, a transferência
serão previstos nas dotações orçamentárias dos órgãos
para outras entidades de crianças e adolescentes
públicos encarregados das áreas de Educação, Saúde e
abrigados;
Assistência Social, dentre outros, observando-se o
princípio da prioridade absoluta à criança e ao VII - participação na vida da comunidade local;
adolescente preconizado pelo caput do art. 227 da VIII - preparação gradativa para o desligamento;
Constituição Federal e pelo caput e parágrafo único do IX - participação de pessoas da comunidade no
o
art. 4 desta Lei. (Incluído pela Lei nº 12.010, de 2009) processo educativo.
o o
§ 3 Os programas em execução serão reavaliados § 1 O dirigente de entidade que desenvolve
pelo Conselho Municipal dos Direitos da Criança e do programa de acolhimento institucional é equiparado ao
Adolescente, no máximo, a cada 2 (dois) anos, guardião, para todos os efeitos de direito. (Incluído pela
constituindo-se critérios para renovação da autorização de Lei nº 12.010, de 2009)
funcionamento: (Incluído pela Lei nº 12.010, de 2009) o
§ 2 Os dirigentes de entidades que desenvolvem
I - o efetivo respeito às regras e princípios desta programas de acolhimento familiar ou institucional
Lei, bem como às resoluções relativas à modalidade de remeterão à autoridade judiciária, no máximo a cada 6
atendimento prestado expedidas pelos Conselhos de (seis) meses, relatório circunstanciado acerca da situação
Direitos da Criança e do Adolescente, em todos os de cada criança ou adolescente acolhido e sua família,
níveis; (Incluído pela Lei nº 12.010, de 2009) o
para fins da reavaliação prevista no § 1 do art. 19 desta
II - a qualidade e eficiência do trabalho Lei. (Incluído pela Lei nº 12.010, de 2009)
desenvolvido, atestadas pelo Conselho Tutelar, pelo

www.editoradince.com - Acesse e veja se há novidades a respeito deste material – CUIDADO: cópia é crime.
LEGISLAÇÃO ESPECIAL E REGIMENTO INTERNO DAS UNIDADES DO SEAS 33
o
§ 3 Os entes federados, por intermédio dos VI - comunicar à autoridade judiciária,
Poderes Executivo e Judiciário, promoverão periodicamente, os casos em que se mostre inviável ou
conjuntamente a permanente qualificação dos impossível o reatamento dos vínculos familiares;
profissionais que atuam direta ou indiretamente em VII - oferecer instalações físicas em condições
programas de acolhimento institucional e destinados à adequadas de habitabilidade, higiene, salubridade e
colocação familiar de crianças e adolescentes, incluindo segurança e os objetos necessários à higiene pessoal;
membros do Poder Judiciário, Ministério Público e
VIII - oferecer vestuário e alimentação suficientes e
Conselho Tutelar. (Incluído pela Lei nº 12.010, de 2009)
o
adequados à faixa etária dos adolescentes atendidos;
§ 4 Salvo determinação em contrário da
IX - oferecer cuidados médicos, psicológicos,
autoridade judiciária competente, as entidades que
odontológicos e farmacêuticos;
desenvolvem programas de acolhimento familiar ou
institucional, se necessário com o auxílio do Conselho X - propiciar escolarização e profissionalização;
Tutelar e dos órgãos de assistência social, estimularão o XI - propiciar atividades culturais, esportivas e de
contato da criança ou adolescente com seus pais e lazer;
parentes, em cumprimento ao disposto nos incisos I e VIII XII - propiciar assistência religiosa àqueles que
do caput deste artigo. (Incluído pela Lei nº 12.010, de desejarem, de acordo com suas crenças;
2009) XIII - proceder a estudo social e pessoal de cada
o
§ 5 As entidades que desenvolvem programas de caso;
acolhimento familiar ou institucional somente poderão XIV - reavaliar periodicamente cada caso, com
receber recursos públicos se comprovado o atendimento intervalo máximo de seis meses, dando ciência dos
dos princípios, exigências e finalidades desta resultados à autoridade competente;
Lei. (Incluído pela Lei nº 12.010, de 2009)
o XV - informar, periodicamente, o adolescente
§ 6 O descumprimento das disposições desta Lei internado sobre sua situação processual;
pelo dirigente de entidade que desenvolva programas de
acolhimento familiar ou institucional é causa de sua XVI - comunicar às autoridades competentes todos
destituição, sem prejuízo da apuração de sua os casos de adolescentes portadores de moléstias
responsabilidade administrativa, civil e criminal. (Incluído infecto-contagiosas;
pela Lei nº 12.010, de 2009) XVII - fornecer comprovante de depósito dos
o
§ 7 Quando se tratar de criança de 0 (zero) a 3 pertences dos adolescentes;
(três) anos em acolhimento institucional, dar-se-á especial XVIII - manter programas destinados ao apoio e
atenção à atuação de educadores de referência estáveis acompanhamento de egressos;
e qualitativamente significativos, às rotinas específicas e XIX - providenciar os documentos necessários ao
ao atendimento das necessidades básicas, incluindo as exercício da cidadania àqueles que não os tiverem;
de afeto como prioritárias. (Incluído pela Lei nº 13.257, de XX - manter arquivo de anotações onde constem
2016) data e circunstâncias do atendimento, nome do
Art. 93. As entidades que mantenham programa de adolescente, seus pais ou responsável, parentes,
acolhimento institucional poderão, em caráter excepcional endereços, sexo, idade, acompanhamento da sua
e de urgência, acolher crianças e adolescentes sem formação, relação de seus pertences e demais dados que
prévia determinação da autoridade competente, fazendo possibilitem sua identificação e a individualização do
comunicação do fato em até 24 (vinte e quatro) horas ao atendimento.
Juiz da Infância e da Juventude, sob pena de o
§ 1 Aplicam-se, no que couber, as obrigações
responsabilidade. (Redação dada pela Lei nº 12.010, de constantes deste artigo às entidades que mantêm
2009) programas de acolhimento institucional e
Parágrafo único. Recebida a comunicação, a familiar. (Redação dada pela Lei nº 12.010, de 2009)
autoridade judiciária, ouvido o Ministério Público e se § 2º No cumprimento das obrigações a que alude
necessário com o apoio do Conselho Tutelar local, tomará este artigo as entidades utilizarão preferencialmente os
as medidas necessárias para promover a imediata recursos da comunidade.
reintegração familiar da criança ou do adolescente ou, se
Art. 94-A. As entidades, públicas ou privadas, que
por qualquer razão não for isso possível ou
abriguem ou recepcionem crianças e adolescentes, ainda
recomendável, para seu encaminhamento a programa de
que em caráter temporário, devem ter, em seus quadros,
acolhimento familiar, institucional ou a família substituta,
o profissionais capacitados a reconhecer e reportar ao
observado o disposto no § 2 do art. 101 desta
Conselho Tutelar suspeitas ou ocorrências de maus-
Lei. (Incluído pela Lei nº 12.010, de 2009)
tratos. (Incluído pela Lei nº 13.046, de 2014)
Art. 94. As entidades que desenvolvem programas
Seção II
de internação têm as seguintes obrigações, entre outras:
Da Fiscalização das Entidades
I - observar os direitos e garantias de que são
titulares os adolescentes; Art. 95. As entidades governamentais e não-
governamentais referidas no art. 90 serão fiscalizadas
II - não restringir nenhum direito que não tenha sido
pelo Judiciário, pelo Ministério Público e pelos Conselhos
objeto de restrição na decisão de internação;
Tutelares.
III - oferecer atendimento personalizado, em
Art. 96. Os planos de aplicação e as prestações de
pequenas unidades e grupos reduzidos;
contas serão apresentados ao estado ou ao município,
IV - preservar a identidade e oferecer ambiente de conforme a origem das dotações orçamentárias.
respeito e dignidade ao adolescente;
Art. 97. São medidas aplicáveis às entidades de
V - diligenciar no sentido do restabelecimento e da atendimento que descumprirem obrigação constante do
preservação dos vínculos familiares; art. 94, sem prejuízo da responsabilidade civil e criminal
de seus dirigentes ou prepostos:

www.editoradince.com - Acesse e veja se há novidades a respeito deste material – CUIDADO: cópia é crime.
34 LEGISLAÇÃO ESPECIAL E REGIMENTO INTERNO DAS UNIDADES DO SEAS
I - às entidades governamentais: crianças e a adolescentes por esta Lei e pela Constituição
a) advertência; Federal, salvo nos casos por esta expressamente
ressalvados, é de responsabilidade primária e solidária
b) afastamento provisório de seus dirigentes;
das 3 (três) esferas de governo, sem prejuízo da
c) afastamento definitivo de seus dirigentes; municipalização do atendimento e da possibilidade da
d) fechamento de unidade ou interdição de execução de programas por entidades não
programa. governamentais; (Incluído pela Lei nº 12.010, de 2009)
II - às entidades não-governamentais: IV - interesse superior da criança e do adolescente:
a) advertência; a intervenção deve atender prioritariamente aos
b) suspensão total ou parcial do repasse de verbas interesses e direitos da criança e do adolescente, sem
públicas; prejuízo da consideração que for devida a outros
interesses legítimos no âmbito da pluralidade dos
c) interdição de unidades ou suspensão de interesses presentes no caso concreto; (Incluído pela Lei
programa; nº 12.010, de 2009)
d) cassação do registro. V - privacidade: a promoção dos direitos e proteção
o
§ 1 Em caso de reiteradas infrações cometidas da criança e do adolescente deve ser efetuada no
por entidades de atendimento, que coloquem em risco os respeito pela intimidade, direito à imagem e reserva da
direitos assegurados nesta Lei, deverá ser o fato sua vida privada; (Incluído pela Lei nº 12.010, de 2009)
comunicado ao Ministério Público ou representado VI - intervenção precoce: a intervenção das
perante autoridade judiciária competente para as autoridades competentes deve ser efetuada logo que a
providências cabíveis, inclusive suspensão das atividades situação de perigo seja conhecida; (Incluído pela Lei nº
ou dissolução da entidade. (Redação dada pela Lei nº 12.010, de 2009)
12.010, de 2009)
o VII - intervenção mínima: a intervenção deve ser
§ 2 As pessoas jurídicas de direito público e as exercida exclusivamente pelas autoridades e instituições
organizações não governamentais responderão pelos cuja ação seja indispensável à efetiva promoção dos
danos que seus agentes causarem às crianças e aos direitos e à proteção da criança e do
adolescentes, caracterizado o descumprimento dos adolescente; (Incluído pela Lei nº 12.010, de 2009)
princípios norteadores das atividades de proteção
específica. (Redação dada pela Lei nº 12.010, de 2009) VIII - proporcionalidade e atualidade: a intervenção
deve ser a necessária e adequada à situação de perigo
Título II em que a criança ou o adolescente se encontram no
Das Medidas de Proteção momento em que a decisão é tomada; (Incluído pela Lei
Capítulo I nº 12.010, de 2009)
Disposições Gerais IX - responsabilidade parental: a intervenção deve
Art. 98. As medidas de proteção à criança e ao ser efetuada de modo que os pais assumam os seus
adolescente são aplicáveis sempre que os direitos deveres para com a criança e o adolescente; (Incluído
reconhecidos nesta Lei forem ameaçados ou violados: pela Lei nº 12.010, de 2009)
I - por ação ou omissão da sociedade ou do X - prevalência da família: na promoção de direitos
Estado; e na proteção da criança e do adolescente deve ser dada
prevalência às medidas que os mantenham ou reintegrem
II - por falta, omissão ou abuso dos pais ou
na sua família natural ou extensa ou, se isso não for
responsável;
possível, que promovam a sua integração em família
III - em razão de sua conduta. adotiva; (Redação dada pela Lei nº 13.509, de 2017)
Capítulo II XI - obrigatoriedade da informação: a criança e o
Das Medidas Específicas de Proteção adolescente, respeitado seu estágio de desenvolvimento
Art. 99. As medidas previstas neste Capítulo e capacidade de compreensão, seus pais ou responsável
poderão ser aplicadas isolada ou cumulativamente, bem devem ser informados dos seus direitos, dos motivos que
como substituídas a qualquer tempo. determinaram a intervenção e da forma como esta se
processa; (Incluído pela Lei nº 12.010, de 2009)
Art. 100. Na aplicação das medidas levar-se-ão em
conta as necessidades pedagógicas, preferindo-se XII - oitiva obrigatória e participação: a criança e o
aquelas que visem ao fortalecimento dos vínculos adolescente, em separado ou na companhia dos pais, de
familiares e comunitários. responsável ou de pessoa por si indicada, bem como os
seus pais ou responsável, têm direito a ser ouvidos e a
Parágrafo único. São também princípios que regem
participar nos atos e na definição da medida de promoção
a aplicação das medidas: (Incluído pela Lei nº 12.010, de
dos direitos e de proteção, sendo sua opinião
2009)
devidamente considerada pela autoridade judiciária
I - condição da criança e do adolescente como o
competente, observado o disposto nos §§ 1 e 2 do art.
o

sujeitos de direitos: crianças e adolescentes são os 28 desta Lei. (Incluído pela Lei nº 12.010, de 2009)
titulares dos direitos previstos nesta e em outras Leis,
Art. 101. Verificada qualquer das hipóteses
bem como na Constituição Federal; (Incluído pela Lei nº
previstas no art. 98, a autoridade competente poderá
12.010, de 2009)
determinar, dentre outras, as seguintes medidas:
II - proteção integral e prioritária: a interpretação e
I - encaminhamento aos pais ou responsável,
aplicação de toda e qualquer norma contida nesta Lei
mediante termo de responsabilidade;
deve ser voltada à proteção integral e prioritária dos
direitos de que crianças e adolescentes são II - orientação, apoio e acompanhamento
titulares; (Incluído pela Lei nº 12.010, de 2009) temporários;
III - responsabilidade primária e solidária do poder III - matrícula e freqüência obrigatórias em
público: a plena efetivação dos direitos assegurados a estabelecimento oficial de ensino fundamental;

www.editoradince.com - Acesse e veja se há novidades a respeito deste material – CUIDADO: cópia é crime.
LEGISLAÇÃO ESPECIAL E REGIMENTO INTERNO DAS UNIDADES DO SEAS 35
o
IV - inclusão em serviços e programas oficiais ou § 6 Constarão do plano individual, dentre
comunitários de proteção, apoio e promoção da família, outros: (Incluído pela Lei nº 12.010, de 2009)
da criança e do adolescente; (Redação dada pela Lei nº I - os resultados da avaliação
13.257, de 2016) interdisciplinar; (Incluído pela Lei nº 12.010, de 2009)
V - requisição de tratamento médico, psicológico ou II - os compromissos assumidos pelos pais ou
psiquiátrico, em regime hospitalar ou ambulatorial; responsável; e (Incluído pela Lei nº 12.010, de 2009)
VI - inclusão em programa oficial ou comunitário de III - a previsão das atividades a serem
auxílio, orientação e tratamento a alcoólatras e desenvolvidas com a criança ou com o adolescente
toxicômanos; acolhido e seus pais ou responsável, com vista na
VII - acolhimento institucional; (Redação dada pela reintegração familiar ou, caso seja esta vedada por
Lei nº 12.010, de 2009) expressa e fundamentada determinação judicial, as
VIII - inclusão em programa de acolhimento providências a serem tomadas para sua colocação em
familiar; (Redação dada pela Lei nº 12.010, de 2009) família substituta, sob direta supervisão da autoridade
judiciária. (Incluído pela Lei nº 12.010, de 2009)
IX - colocação em família substituta. (Incluído pela o
Lei nº 12.010, de 2009) § 7 O acolhimento familiar ou institucional
o ocorrerá no local mais próximo à residência dos pais ou
§ 1 O acolhimento institucional e o acolhimento
do responsável e, como parte do processo de
familiar são medidas provisórias e excepcionais,
reintegração familiar, sempre que identificada a
utilizáveis como forma de transição para reintegração
necessidade, a família de origem será incluída em
familiar ou, não sendo esta possível, para colocação em
programas oficiais de orientação, de apoio e de promoção
família substituta, não implicando privação de
social, sendo facilitado e estimulado o contato com a
liberdade. (Incluído pela Lei nº 12.010, de 2009)
o criança ou com o adolescente acolhido. (Incluído pela Lei
§ 2 Sem prejuízo da tomada de medidas nº 12.010, de 2009)
emergenciais para proteção de vítimas de violência ou o
§ 8 Verificada a possibilidade de reintegração
abuso sexual e das providências a que alude o art. 130
familiar, o responsável pelo programa de acolhimento
desta Lei, o afastamento da criança ou adolescente do
familiar ou institucional fará imediata comunicação à
convívio familiar é de competência exclusiva da
autoridade judiciária, que dará vista ao Ministério Público,
autoridade judiciária e importará na deflagração, a pedido
pelo prazo de 5 (cinco) dias, decidindo em igual
do Ministério Público ou de quem tenha legítimo interesse,
prazo. (Incluído pela Lei nº 12.010, de 2009)
de procedimento judicial contencioso, no qual se garanta o
aos pais ou ao responsável legal o exercício do § 9 Em sendo constatada a impossibilidade de
contraditório e da ampla defesa. (Incluído pela Lei nº reintegração da criança ou do adolescente à família de
12.010, de 2009) origem, após seu encaminhamento a programas oficiais
o ou comunitários de orientação, apoio e promoção social,
§ 3 Crianças e adolescentes somente poderão ser
será enviado relatório fundamentado ao Ministério
encaminhados às instituições que executam programas
Público, no qual conste a descrição pormenorizada das
de acolhimento institucional, governamentais ou não, por
providências tomadas e a expressa recomendação,
meio de uma Guia de Acolhimento, expedida pela
subscrita pelos técnicos da entidade ou responsáveis pela
autoridade judiciária, na qual obrigatoriamente constará,
execução da política municipal de garantia do direito à
dentre outros: (Incluído pela Lei nº 12.010, de 2009)
convivência familiar, para a destituição do poder familiar,
I - sua identificação e a qualificação completa de ou destituição de tutela ou guarda. (Incluído pela Lei nº
seus pais ou de seu responsável, se conhecidos; (Incluído 12.010, de 2009)
pela Lei nº 12.010, de 2009)
§ 10. Recebido o relatório, o Ministério Público terá
II - o endereço de residência dos pais ou do o prazo de 15 (quinze) dias para o ingresso com a ação
responsável, com pontos de referência; (Incluído pela Lei de destituição do poder familiar, salvo se entender
nº 12.010, de 2009) necessária a realização de estudos complementares ou
III - os nomes de parentes ou de terceiros de outras providências indispensáveis ao ajuizamento da
interessados em tê-los sob sua guarda; (Incluído pela Lei demanda. (Redação dada pela Lei nº 13.509, de 2017)
nº 12.010, de 2009) § 11. A autoridade judiciária manterá, em cada
IV - os motivos da retirada ou da não reintegração comarca ou foro regional, um cadastro contendo
ao convívio familiar. (Incluído pela Lei nº 12.010, de informações atualizadas sobre as crianças e adolescentes
2009) em regime de acolhimento familiar e institucional sob sua
o
§ 4 Imediatamente após o acolhimento da criança responsabilidade, com informações pormenorizadas sobre
ou do adolescente, a entidade responsável pelo programa a situação jurídica de cada um, bem como as
de acolhimento institucional ou familiar elaborará um providências tomadas para sua reintegração familiar ou
plano individual de atendimento, visando à reintegração colocação em família substituta, em qualquer das
familiar, ressalvada a existência de ordem escrita e modalidades previstas no art. 28 desta Lei. (Incluído pela
fundamentada em contrário de autoridade judiciária Lei nº 12.010, de 2009)
competente, caso em que também deverá contemplar sua § 12. Terão acesso ao cadastro o Ministério
colocação em família substituta, observadas as regras e Público, o Conselho Tutelar, o órgão gestor da
princípios desta Lei. (Incluído pela Lei nº 12.010, de Assistência Social e os Conselhos Municipais dos Direitos
2009) da Criança e do Adolescente e da Assistência Social, aos
o quais incumbe deliberar sobre a implementação de
§ 5 O plano individual será elaborado sob a
responsabilidade da equipe técnica do respectivo políticas públicas que permitam reduzir o número de
programa de atendimento e levará em consideração a crianças e adolescentes afastados do convívio familiar e
opinião da criança ou do adolescente e a oitiva dos pais abreviar o período de permanência em programa de
ou do responsável. (Incluído pela Lei nº 12.010, de 2009) acolhimento. (Incluído pela Lei nº 12.010, de 2009)

www.editoradince.com - Acesse e veja se há novidades a respeito deste material – CUIDADO: cópia é crime.
36 LEGISLAÇÃO ESPECIAL E REGIMENTO INTERNO DAS UNIDADES DO SEAS
Art. 102. As medidas de proteção de que trata este Art. 109. O adolescente civilmente identificado não
Capítulo serão acompanhadas da regularização do será submetido a identificação compulsória pelos órgãos
registro civil. (Vide Lei nº 12.010, de 2009) policiais, de proteção e judiciais, salvo para efeito de
§ 1º Verificada a inexistência de registro anterior, o confrontação, havendo dúvida fundada.
assento de nascimento da criança ou adolescente será Capítulo III
feito à vista dos elementos disponíveis, mediante Das Garantias Processuais
requisição da autoridade judiciária.
Art. 110. Nenhum adolescente será privado de sua
§ 2º Os registros e certidões necessários à liberdade sem o devido processo legal.
regularização de que trata este artigo são isentos de
Art. 111. São asseguradas ao adolescente, entre
multas, custas e emolumentos, gozando de absoluta
outras, as seguintes garantias:
prioridade.
o I - pleno e formal conhecimento da atribuição de
§ 3 Caso ainda não definida a paternidade, será
ato infracional, mediante citação ou meio equivalente;
deflagrado procedimento específico destinado à sua
o
averiguação, conforme previsto pela Lei n 8.560, de 29 II - igualdade na relação processual, podendo
de dezembro de 1992. (Incluído pela Lei nº 12.010, de confrontar-se com vítimas e testemunhas e produzir todas
2009) as provas necessárias à sua defesa;
o o
§ 4 Nas hipóteses previstas no § 3 deste artigo, III - defesa técnica por advogado;
é dispensável o ajuizamento de ação de investigação de IV - assistência judiciária gratuita e integral aos
paternidade pelo Ministério Público se, após o não necessitados, na forma da lei;
comparecimento ou a recusa do suposto pai em assumir a V - direito de ser ouvido pessoalmente pela
paternidade a ele atribuída, a criança for encaminhada autoridade competente;
para adoção. (Incluído pela Lei nº 12.010, de 2009) VI - direito de solicitar a presença de seus pais ou
o
§ 5 Os registros e certidões necessários à responsável em qualquer fase do procedimento.
inclusão, a qualquer tempo, do nome do pai no assento Capítulo IV
de nascimento são isentos de multas, custas e
emolumentos, gozando de absoluta prioridade. (Incluído Das Medidas Sócio-Educativas
dada pela Lei nº 13.257, de 2016) Seção I
o Disposições Gerais
§ 6 São gratuitas, a qualquer tempo, a averbação
requerida do reconhecimento de paternidade no assento Art. 112. Verificada a prática de ato infracional, a
de nascimento e a certidão correspondente. (Incluído autoridade competente poderá aplicar ao adolescente as
dada pela Lei nº 13.257, de 2016) seguintes medidas:
Título III I - advertência;
Da Prática de Ato Infracional II - obrigação de reparar o dano;
Capítulo I III - prestação de serviços à comunidade;
Disposições Gerais IV - liberdade assistida;
Art. 103. Considera-se ato infracional a conduta V - inserção em regime de semi-liberdade;
descrita como crime ou contravenção penal. VI - internação em estabelecimento educacional;
Art. 104. São penalmente inimputáveis os menores VII - qualquer uma das previstas no art. 101, I a VI.
de dezoito anos, sujeitos às medidas previstas nesta Lei.
§ 1º A medida aplicada ao adolescente levará em
Parágrafo único. Para os efeitos desta Lei, deve ser conta a sua capacidade de cumpri-la, as circunstâncias e
considerada a idade do adolescente à data do fato. a gravidade da infração.
Art. 105. Ao ato infracional praticado por criança § 2º Em hipótese alguma e sob pretexto algum,
corresponderão as medidas previstas no art. 101. será admitida a prestação de trabalho forçado.
Capítulo II § 3º Os adolescentes portadores de doença ou
Dos Direitos Individuais deficiência mental receberão tratamento individual e
Art. 106. Nenhum adolescente será privado de sua especializado, em local adequado às suas condições.
liberdade senão em flagrante de ato infracional ou por Art. 113. Aplica-se a este Capítulo o disposto nos
ordem escrita e fundamentada da autoridade judiciária arts. 99 e 100.
competente. Art. 114. A imposição das medidas previstas nos
Parágrafo único. O adolescente tem direito à incisos II a VI do art. 112 pressupõe a existência de
identificação dos responsáveis pela sua apreensão, provas suficientes da autoria e da materialidade da
devendo ser informado acerca de seus direitos. infração, ressalvada a hipótese de remissão, nos termos
Art. 107. A apreensão de qualquer adolescente e o do art. 127.
local onde se encontra recolhido serão incontinenti Parágrafo único. A advertência poderá ser aplicada
comunicados à autoridade judiciária competente e à sempre que houver prova da materialidade e indícios
família do apreendido ou à pessoa por ele indicada. suficientes da autoria.
Parágrafo único. Examinar-se-á, desde logo e sob Seção II
pena de responsabilidade, a possibilidade de liberação Da Advertência
imediata.
Art. 115. A advertência consistirá em admoestação
Art. 108. A internação, antes da sentença, pode ser verbal, que será reduzida a termo e assinada.
determinada pelo prazo máximo de quarenta e cinco dias.
Seção III
Parágrafo único. A decisão deverá ser
Da Obrigação de Reparar o Dano
fundamentada e basear-se em indícios suficientes de
autoria e materialidade, demonstrada a necessidade Art. 116. Em se tratando de ato infracional com
imperiosa da medida. reflexos patrimoniais, a autoridade poderá determinar, se

www.editoradince.com - Acesse e veja se há novidades a respeito deste material – CUIDADO: cópia é crime.
LEGISLAÇÃO ESPECIAL E REGIMENTO INTERNO DAS UNIDADES DO SEAS 37
for o caso, que o adolescente restitua a coisa, promova o § 1º Será permitida a realização de atividades
ressarcimento do dano, ou, por outra forma, compense o externas, a critério da equipe técnica da entidade, salvo
prejuízo da vítima. expressa determinação judicial em contrário.
Parágrafo único. Havendo manifesta § 2º A medida não comporta prazo determinado,
impossibilidade, a medida poderá ser substituída por outra devendo sua manutenção ser reavaliada, mediante
adequada. decisão fundamentada, no máximo a cada seis meses.
Seção IV § 3º Em nenhuma hipótese o período máximo de
Da Prestação de Serviços à Comunidade internação excederá a três anos.
Art. 117. A prestação de serviços comunitários § 4º Atingido o limite estabelecido no parágrafo
consiste na realização de tarefas gratuitas de interesse anterior, o adolescente deverá ser liberado, colocado em
geral, por período não excedente a seis meses, junto a regime de semi-liberdade ou de liberdade assistida.
entidades assistenciais, hospitais, escolas e outros § 5º A liberação será compulsória aos vinte e um
estabelecimentos congêneres, bem como em programas anos de idade.
comunitários ou governamentais. § 6º Em qualquer hipótese a desinternação será
Parágrafo único. As tarefas serão atribuídas precedida de autorização judicial, ouvido o Ministério
conforme as aptidões do adolescente, devendo ser Público.
cumpridas durante jornada máxima de oito horas o
§ 7 A determinação judicial mencionada no §
semanais, aos sábados, domingos e feriados ou em dias o
1 poderá ser revista a qualquer tempo pela autoridade
úteis, de modo a não prejudicar a freqüência à escola ou judiciária. (Incluído pela Lei nº 12.594, de 2012)
à jornada normal de trabalho.
Art. 122. A medida de internação só poderá ser
Seção V aplicada quando:
Da Liberdade Assistida I - tratar-se de ato infracional cometido mediante
Art. 118. A liberdade assistida será adotada sempre grave ameaça ou violência a pessoa;
que se afigurar a medida mais adequada para o fim de II - por reiteração no cometimento de outras
acompanhar, auxiliar e orientar o adolescente. infrações graves;
§ 1º A autoridade designará pessoa capacitada III - por descumprimento reiterado e injustificável da
para acompanhar o caso, a qual poderá ser recomendada medida anteriormente imposta.
por entidade ou programa de atendimento. o
§ 1 O prazo de internação na hipótese do inciso III
§ 2º A liberdade assistida será fixada pelo prazo deste artigo não poderá ser superior a 3 (três) meses,
mínimo de seis meses, podendo a qualquer tempo ser devendo ser decretada judicialmente após o devido
prorrogada, revogada ou substituída por outra medida, processo legal. (Redação dada pela Lei nº 12.594, de
ouvido o orientador, o Ministério Público e o defensor. 2012)
Art. 119. Incumbe ao orientador, com o apoio e a § 2º. Em nenhuma hipótese será aplicada a
supervisão da autoridade competente, a realização dos internação, havendo outra medida adequada.
seguintes encargos, entre outros:
Art. 123. A internação deverá ser cumprida em
I - promover socialmente o adolescente e sua entidade exclusiva para adolescentes, em local distinto
família, fornecendo-lhes orientação e inserindo-os, se daquele destinado ao abrigo, obedecida rigorosa
necessário, em programa oficial ou comunitário de auxílio separação por critérios de idade, compleição física e
e assistência social; gravidade da infração.
II - supervisionar a freqüência e o aproveitamento Parágrafo único. Durante o período de internação,
escolar do adolescente, promovendo, inclusive, sua inclusive provisória, serão obrigatórias atividades
matrícula; pedagógicas.
III - diligenciar no sentido da profissionalização do Art. 124. São direitos do adolescente privado de
adolescente e de sua inserção no mercado de trabalho; liberdade, entre outros, os seguintes:
IV - apresentar relatório do caso. I - entrevistar-se pessoalmente com o
Seção VI representante do Ministério Público;
Do Regime de Semi-liberdade II - peticionar diretamente a qualquer autoridade;
Art. 120. O regime de semi-liberdade pode ser III - avistar-se reservadamente com seu defensor;
determinado desde o início, ou como forma de transição IV - ser informado de sua situação processual,
para o meio aberto, possibilitada a realização de sempre que solicitada;
atividades externas, independentemente de autorização
V - ser tratado com respeito e dignidade;
judicial.
VI - permanecer internado na mesma localidade ou
§ 1º São obrigatórias a escolarização e a
naquela mais próxima ao domicílio de seus pais ou
profissionalização, devendo, sempre que possível, ser
responsável;
utilizados os recursos existentes na comunidade.
VII - receber visitas, ao menos, semanalmente;
§ 2º A medida não comporta prazo determinado
aplicando-se, no que couber, as disposições relativas à VIII - corresponder-se com seus familiares e
internação. amigos;
Seção VII IX - ter acesso aos objetos necessários à higiene e
asseio pessoal;
Da Internação
X - habitar alojamento em condições adequadas de
Art. 121. A internação constitui medida privativa da
higiene e salubridade;
liberdade, sujeita aos princípios de brevidade,
excepcionalidade e respeito à condição peculiar de XI - receber escolarização e profissionalização;
pessoa em desenvolvimento.

www.editoradince.com - Acesse e veja se há novidades a respeito deste material – CUIDADO: cópia é crime.
38 LEGISLAÇÃO ESPECIAL E REGIMENTO INTERNO DAS UNIDADES DO SEAS
XII - realizar atividades culturais, esportivas e de X - suspensão ou destituição do pátrio poder poder
lazer: familiar . (Expressão substituída pela Lei nº 12.010, de
XIII - ter acesso aos meios de comunicação social; 2009)
XIV - receber assistência religiosa, segundo a sua Parágrafo único. Na aplicação das medidas
crença, e desde que assim o deseje; previstas nos incisos IX e X deste artigo, observar-se-á o
disposto nos arts. 23 e 24.
XV - manter a posse de seus objetos pessoais e
dispor de local seguro para guardá-los, recebendo Art. 130. Verificada a hipótese de maus-tratos,
comprovante daqueles porventura depositados em poder opressão ou abuso sexual impostos pelos pais ou
da entidade; responsável, a autoridade judiciária poderá determinar,
como medida cautelar, o afastamento do agressor da
XVI - receber, quando de sua desinternação, os
moradia comum.
documentos pessoais indispensáveis à vida em
sociedade. Parágrafo único. Da medida cautelar constará,
ainda, a fixação provisória dos alimentos de que
§ 1º Em nenhum caso haverá incomunicabilidade.
necessitem a criança ou o adolescente dependentes do
§ 2º A autoridade judiciária poderá suspender agressor. (Incluído pela Lei nº 12.415, de 2011)
temporariamente a visita, inclusive de pais ou
Título V
responsável, se existirem motivos sérios e fundados de
sua prejudicialidade aos interesses do adolescente. Do Conselho Tutelar
Art. 125. É dever do Estado zelar pela integridade Capítulo I
física e mental dos internos, cabendo-lhe adotar as Disposições Gerais
medidas adequadas de contenção e segurança. Art. 131. O Conselho Tutelar é órgão permanente e
Capítulo V autônomo, não jurisdicional, encarregado pela sociedade
Da Remissão de zelar pelo cumprimento dos direitos da criança e do
adolescente, definidos nesta Lei.
Art. 126. Antes de iniciado o procedimento judicial
para apuração de ato infracional, o representante do Art. 132. Em cada Município e em cada Região
Ministério Público poderá conceder a remissão, como Administrativa do Distrito Federal haverá, no mínimo, 1
forma de exclusão do processo, atendendo às (um) Conselho Tutelar como órgão integrante da
circunstâncias e conseqüências do fato, ao contexto administração pública local, composto de 5 (cinco)
social, bem como à personalidade do adolescente e sua membros, escolhidos pela população local para mandato
maior ou menor participação no ato infracional. de 4 (quatro) anos, permitida recondução por novos
processos de escolha. (Redação dada pela Lei nº 13.824,
Parágrafo único. Iniciado o procedimento, a
de 2019)
concessão da remissão pela autoridade judiciária
importará na suspensão ou extinção do processo. Art. 133. Para a candidatura a membro do
Conselho Tutelar, serão exigidos os seguintes requisitos:
Art. 127. A remissão não implica necessariamente
o reconhecimento ou comprovação da responsabilidade, I - reconhecida idoneidade moral;
nem prevalece para efeito de antecedentes, podendo II - idade superior a vinte e um anos;
incluir eventualmente a aplicação de qualquer das III - residir no município.
medidas previstas em lei, exceto a colocação em regime
Art. 134. Lei municipal ou distrital disporá sobre o
de semi-liberdade e a internação.
local, dia e horário de funcionamento do Conselho Tutelar,
Art. 128. A medida aplicada por força da remissão inclusive quanto à remuneração dos respectivos
poderá ser revista judicialmente, a qualquer tempo, membros, aos quais é assegurado o direito a: (Redação
mediante pedido expresso do adolescente ou de seu dada pela Lei nº 12.696, de 2012)
representante legal, ou do Ministério Público.
I - cobertura previdenciária; (Incluído pela Lei nº
Título IV 12.696, de 2012)
Das Medidas Pertinentes aos Pais ou Responsável II - gozo de férias anuais remuneradas, acrescidas
Art. 129. São medidas aplicáveis aos pais ou de 1/3 (um terço) do valor da remuneração
responsável: mensal; (Incluído pela Lei nº 12.696, de 2012)
I - encaminhamento a serviços e programas oficiais III - licença-maternidade; (Incluído pela Lei nº
ou comunitários de proteção, apoio e promoção da 12.696, de 2012)
família; (Redação dada dada pela Lei nº 13.257, de 2016) IV - licença-paternidade; (Incluído pela Lei nº
II - inclusão em programa oficial ou comunitário de 12.696, de 2012)
auxílio, orientação e tratamento a alcoólatras e V - gratificação natalina. (Incluído pela Lei nº
toxicômanos; 12.696, de 2012)
III - encaminhamento a tratamento psicológico ou Parágrafo único. Constará da lei orçamentária
psiquiátrico; municipal e da do Distrito Federal previsão dos recursos
IV - encaminhamento a cursos ou programas de necessários ao funcionamento do Conselho Tutelar e à
orientação; remuneração e formação continuada dos conselheiros
V - obrigação de matricular o filho ou pupilo e tutelares. (Redação dada pela Lei nº 12.696, de 2012)
acompanhar sua freqüência e aproveitamento escolar; Art. 135. O exercício efetivo da função de
VI - obrigação de encaminhar a criança ou conselheiro constituirá serviço público relevante e
adolescente a tratamento especializado; estabelecerá presunção de idoneidade moral. (Redação
dada pela Lei nº 12.696, de 2012)
VII - advertência;
Capítulo II
VIII - perda da guarda;
Das Atribuições do Conselho
IX - destituição da tutela;
Art. 136. São atribuições do Conselho Tutelar:

www.editoradince.com - Acesse e veja se há novidades a respeito deste material – CUIDADO: cópia é crime.
LEGISLAÇÃO ESPECIAL E REGIMENTO INTERNO DAS UNIDADES DO SEAS 39
I - atender as crianças e adolescentes nas território nacional a cada 4 (quatro) anos, no primeiro
hipóteses previstas nos arts. 98 e 105, aplicando as domingo do mês de outubro do ano subsequente ao da
medidas previstas no art. 101, I a VII; eleição presidencial. (Incluído pela Lei nº 12.696, de
II - atender e aconselhar os pais ou responsável, 2012)
o
aplicando as medidas previstas no art. 129, I a VII; § 2 A posse dos conselheiros tutelares ocorrerá
III - promover a execução de suas decisões, no dia 10 de janeiro do ano subsequente ao processo de
podendo para tanto: escolha. (Incluído pela Lei nº 12.696, de 2012)
o
a) requisitar serviços públicos nas áreas de saúde, § 3 No processo de escolha dos membros do
educação, serviço social, previdência, trabalho e Conselho Tutelar, é vedado ao candidato doar, oferecer,
segurança; prometer ou entregar ao eleitor bem ou vantagem pessoal
de qualquer natureza, inclusive brindes de pequeno
b) representar junto à autoridade judiciária nos
valor. (Incluído pela Lei nº 12.696, de 2012)
casos de descumprimento injustificado de suas
deliberações. Capítulo V
IV - encaminhar ao Ministério Público notícia de Dos Impedimentos
fato que constitua infração administrativa ou penal contra Art. 140. São impedidos de servir no mesmo
os direitos da criança ou adolescente; Conselho marido e mulher, ascendentes e descendentes,
V - encaminhar à autoridade judiciária os casos de sogro e genro ou nora, irmãos, cunhados, durante o
sua competência; cunhadio, tio e sobrinho, padrasto ou madrasta e enteado.
VI - providenciar a medida estabelecida pela Parágrafo único. Estende-se o impedimento do
autoridade judiciária, dentre as previstas no art. 101, de I conselheiro, na forma deste artigo, em relação à
a VI, para o adolescente autor de ato infracional; autoridade judiciária e ao representante do Ministério
Público com atuação na Justiça da Infância e da
VII - expedir notificações;
Juventude, em exercício na comarca, foro regional ou
VIII - requisitar certidões de nascimento e de óbito distrital.
de criança ou adolescente quando necessário;
Título VI
IX - assessorar o Poder Executivo local na
Do Acesso à Justiça
elaboração da proposta orçamentária para planos e
programas de atendimento dos direitos da criança e do Capítulo I
adolescente; Disposições Gerais
X - representar, em nome da pessoa e da família, Art. 141. É garantido o acesso de toda criança ou
contra a violação dos direitos previstos no art. 220, § 3º, adolescente à Defensoria Pública, ao Ministério Público e
inciso II, da Constituição Federal ; ao Poder Judiciário, por qualquer de seus órgãos.
XI - representar ao Ministério Público para efeito § 1º. A assistência judiciária gratuita será prestada
das ações de perda ou suspensão do poder familiar, após aos que dela necessitarem, através de defensor público
esgotadas as possibilidades de manutenção da criança ou ou advogado nomeado.
do adolescente junto à família natural. (Redação dada § 2º As ações judiciais da competência da Justiça
pela Lei nº 12.010, de 2009) da Infância e da Juventude são isentas de custas e
XII - promover e incentivar, na comunidade e nos emolumentos, ressalvada a hipótese de litigância de má-
grupos profissionais, ações de divulgação e treinamento fé.
para o reconhecimento de sintomas de maus-tratos em Art. 142. Os menores de dezesseis anos serão
crianças e adolescentes. (Incluído pela Lei nº 13.046, de representados e os maiores de dezesseis e menores de
2014) vinte e um anos assistidos por seus pais, tutores ou
Parágrafo único. Se, no exercício de suas curadores, na forma da legislação civil ou processual.
atribuições, o Conselho Tutelar entender necessário o Parágrafo único. A autoridade judiciária dará
afastamento do convívio familiar, comunicará incontinenti curador especial à criança ou adolescente, sempre que os
o fato ao Ministério Público, prestando-lhe informações interesses destes colidirem com os de seus pais ou
sobre os motivos de tal entendimento e as providências responsável, ou quando carecer de representação ou
tomadas para a orientação, o apoio e a promoção social assistência legal ainda que eventual.
da família. (Incluído pela Lei nº 12.010, de 2009)
Art. 143. E vedada a divulgação de atos judiciais,
Art. 137. As decisões do Conselho Tutelar somente policiais e administrativos que digam respeito a crianças e
poderão ser revistas pela autoridade judiciária a pedido de adolescentes a que se atribua autoria de ato infracional.
quem tenha legítimo interesse.
Parágrafo único. Qualquer notícia a respeito do fato
Capítulo III não poderá identificar a criança ou adolescente, vedando-
Da Competência se fotografia, referência a nome, apelido, filiação,
Art. 138. Aplica-se ao Conselho Tutelar a regra de parentesco, residência e, inclusive, iniciais do nome e
competência constante do art. 147. sobrenome. (Redação dada pela Lei nº 10.764, de
12.11.2003)
Capítulo IV
Art. 144. A expedição de cópia ou certidão de atos
Da Escolha dos Conselheiros
a que se refere o artigo anterior somente será deferida
Art. 139. O processo para a escolha dos membros pela autoridade judiciária competente, se demonstrado o
do Conselho Tutelar será estabelecido em lei municipal e interesse e justificada a finalidade.
realizado sob a responsabilidade do Conselho Municipal
Capítulo II
dos Direitos da Criança e do Adolescente, e a fiscalização
do Ministério Público. (Redação dada pela Lei nº 8.242, Da Justiça da Infância e da Juventude
de 12.10.1991) Seção I
o
§ 1 O processo de escolha dos membros do Disposições Gerais
Conselho Tutelar ocorrerá em data unificada em todo o

www.editoradince.com - Acesse e veja se há novidades a respeito deste material – CUIDADO: cópia é crime.
40 LEGISLAÇÃO ESPECIAL E REGIMENTO INTERNO DAS UNIDADES DO SEAS
Art. 145. Os estados e o Distrito Federal poderão e) conceder a emancipação, nos termos da lei civil,
criar varas especializadas e exclusivas da infância e da quando faltarem os pais;
juventude, cabendo ao Poder Judiciário estabelecer sua f) designar curador especial em casos de
proporcionalidade por número de habitantes, dotá-las de apresentação de queixa ou representação, ou de outros
infra-estrutura e dispor sobre o atendimento, inclusive em procedimentos judiciais ou extrajudiciais em que haja
plantões. interesses de criança ou adolescente;
Seção II g) conhecer de ações de alimentos;
Do Juiz h) determinar o cancelamento, a retificação e o
Art. 146. A autoridade a que se refere esta Lei é o suprimento dos registros de nascimento e óbito.
Juiz da Infância e da Juventude, ou o juiz que exerce essa Art. 149. Compete à autoridade judiciária
função, na forma da lei de organização judiciária local. disciplinar, através de portaria, ou autorizar, mediante
Art. 147. A competência será determinada: alvará:
I - pelo domicílio dos pais ou responsável; I - a entrada e permanência de criança ou
II - pelo lugar onde se encontre a criança ou adolescente, desacompanhado dos pais ou responsável,
adolescente, à falta dos pais ou responsável. em:
§ 1º. Nos casos de ato infracional, será competente a) estádio, ginásio e campo desportivo;
a autoridade do lugar da ação ou omissão, observadas as b) bailes ou promoções dançantes;
regras de conexão, continência e prevenção. c) boate ou congêneres;
§ 2º A execução das medidas poderá ser delegada d) casa que explore comercialmente diversões
à autoridade competente da residência dos pais ou eletrônicas;
responsável, ou do local onde sediar-se a entidade que
e) estúdios cinematográficos, de teatro, rádio e
abrigar a criança ou adolescente.
televisão.
§ 3º Em caso de infração cometida através de
II - a participação de criança e adolescente em:
transmissão simultânea de rádio ou televisão, que atinja
mais de uma comarca, será competente, para aplicação a) espetáculos públicos e seus ensaios;
da penalidade, a autoridade judiciária do local da sede b) certames de beleza.
estadual da emissora ou rede, tendo a sentença eficácia § 1º Para os fins do disposto neste artigo, a
para todas as transmissoras ou retransmissoras do autoridade judiciária levará em conta, dentre outros
respectivo estado. fatores:
Art. 148. A Justiça da Infância e da Juventude é a) os princípios desta Lei;
competente para: b) as peculiaridades locais;
I - conhecer de representações promovidas pelo c) a existência de instalações adequadas;
Ministério Público, para apuração de ato infracional
atribuído a adolescente, aplicando as medidas cabíveis; d) o tipo de freqüência habitual ao local;
II - conceder a remissão, como forma de e) a adequação do ambiente a eventual
suspensão ou extinção do processo; participação ou freqüência de crianças e adolescentes;
III - conhecer de pedidos de adoção e seus f) a natureza do espetáculo.
incidentes; § 2º As medidas adotadas na conformidade deste
IV - conhecer de ações civis fundadas em artigo deverão ser fundamentadas, caso a caso, vedadas
interesses individuais, difusos ou coletivos afetos à as determinações de caráter geral.
criança e ao adolescente, observado o disposto no art. Seção III
209; Dos Serviços Auxiliares
V - conhecer de ações decorrentes de Art. 150. Cabe ao Poder Judiciário, na elaboração
irregularidades em entidades de atendimento, aplicando de sua proposta orçamentária, prever recursos para
as medidas cabíveis; manutenção de equipe interprofissional, destinada a
VI - aplicar penalidades administrativas nos casos assessorar a Justiça da Infância e da Juventude.
de infrações contra norma de proteção à criança ou Art. 151. Compete à equipe interprofissional dentre
adolescente; outras atribuições que lhe forem reservadas pela
VII - conhecer de casos encaminhados pelo legislação local, fornecer subsídios por escrito, mediante
Conselho Tutelar, aplicando as medidas cabíveis. laudos, ou verbalmente, na audiência, e bem assim
Parágrafo único. Quando se tratar de criança ou desenvolver trabalhos de aconselhamento, orientação,
adolescente nas hipóteses do art. 98, é também encaminhamento, prevenção e outros, tudo sob a
competente a Justiça da Infância e da Juventude para o imediata subordinação à autoridade judiciária, assegurada
fim de: a livre manifestação do ponto de vista técnico.
a) conhecer de pedidos de guarda e tutela; Parágrafo único. Na ausência ou insuficiência de
servidores públicos integrantes do Poder Judiciário
b) conhecer de ações de destituição do pátrio responsáveis pela realização dos estudos psicossociais
poder poder familiar , perda ou modificação da tutela ou ou de quaisquer outras espécies de avaliações técnicas
guarda; (Expressão substituída pela Lei nº 12.010, de exigidas por esta Lei ou por determinação judicial, a
2009) autoridade judiciária poderá proceder à nomeação de
c) suprir a capacidade ou o consentimento para o perito, nos termos do art. 156 da Lei no 13.105, de 16 de
casamento; março de 2015 (Código de Processo Civil) . (Incluído pela
d) conhecer de pedidos baseados em discordância Lei nº 13.509, de 2017)
paterna ou materna, em relação ao exercício do pátrio Capítulo III
poder poder familiar ; (Expressão substituída pela Lei nº Dos Procedimentos
12.010, de 2009)

www.editoradince.com - Acesse e veja se há novidades a respeito deste material – CUIDADO: cópia é crime.
LEGISLAÇÃO ESPECIAL E REGIMENTO INTERNO DAS UNIDADES DO SEAS 41
o
Seção I disposto no § 6 do art. 28 desta Lei. (Incluído pela Lei nº
Disposições Gerais 13.509, de 2017)
Art. 152. Aos procedimentos regulados nesta Lei Art. 158. O requerido será citado para, no prazo de
aplicam-se subsidiariamente as normas gerais previstas dez dias, oferecer resposta escrita, indicando as provas a
na legislação processual pertinente. serem produzidas e oferecendo desde logo o rol de
testemunhas e documentos.
§ 1º É assegurada, sob pena de responsabilidade, o
prioridade absoluta na tramitação dos processos e § 1 A citação será pessoal, salvo se esgotados
procedimentos previstos nesta Lei, assim como na todos os meios para sua realização. (Incluído pela Lei nº
execução dos atos e diligências judiciais a eles 12.962, de 2014)
o
referentes. (Incluído pela Lei nº 12.010, de 2009) § 2 O requerido privado de liberdade deverá ser
§ 2º Os prazos estabelecidos nesta Lei e aplicáveis citado pessoalmente. (Incluído pela Lei nº 12.962, de
aos seus procedimentos são contados em dias corridos, 2014)
o
excluído o dia do começo e incluído o dia do vencimento, § 3 Quando, por 2 (duas) vezes, o oficial de
vedado o prazo em dobro para a Fazenda Pública e o justiça houver procurado o citando em seu domicílio ou
Ministério Público. (Incluído pela Lei nº 13.509, de 2017) residência sem o encontrar, deverá, havendo suspeita de
Art. 153. Se a medida judicial a ser adotada não ocultação, informar qualquer pessoa da família ou, em
corresponder a procedimento previsto nesta ou em outra sua falta, qualquer vizinho do dia útil em que voltará a fim
lei, a autoridade judiciária poderá investigar os fatos e de efetuar a citação, na hora que designar, nos termos
o
ordenar de ofício as providências necessárias, ouvido o do art. 252 e seguintes da Lei n 13.105, de 16 de março
Ministério Público. de 2015 (Código de Processo Civil) . (Incluído pela Lei nº
13.509, de 2017)
Parágrafo único. O disposto neste artigo não se o
aplica para o fim de afastamento da criança ou do § 4 Na hipótese de os genitores encontrarem-se
adolescente de sua família de origem e em outros em local incerto ou não sabido, serão citados por edital no
procedimentos necessariamente contenciosos. (Incluído prazo de 10 (dez) dias, em publicação única, dispensado
pela Lei nº 12.010, de 2009) o envio de ofícios para a localização. (Incluído pela Lei nº
13.509, de 2017)
Art. 154. Aplica-se às multas o disposto no art. 214.
Art. 159. Se o requerido não tiver possibilidade de
Seção II
constituir advogado, sem prejuízo do próprio sustento e
Da Perda e da Suspensão do Pátrio Poder Poder de sua família, poderá requerer, em cartório, que lhe seja
Familiar nomeado dativo, ao qual incumbirá a apresentação de
(Expressão substituída pela Lei nº 12.010, de 2009) resposta, contando-se o prazo a partir da intimação do
Art. 155. O procedimento para a perda ou a despacho de nomeação.
suspensão do pátrio poder poder familiar terá início por Parágrafo único. Na hipótese de requerido privado
provocação do Ministério Público ou de quem tenha de liberdade, o oficial de justiça deverá perguntar, no
legítimo interesse. (Expressão substituída pela Lei nº momento da citação pessoal, se deseja que lhe seja
12.010, de 2009) nomeado defensor. (Incluído pela Lei nº 12.962, de 2014)
Art. 156. A petição inicial indicará: Art. 160. Sendo necessário, a autoridade judiciária
I - a autoridade judiciária a que for dirigida; requisitará de qualquer repartição ou órgão público a
II - o nome, o estado civil, a profissão e a apresentação de documento que interesse à causa, de
residência do requerente e do requerido, dispensada a ofício ou a requerimento das partes ou do Ministério
qualificação em se tratando de pedido formulado por Público.
representante do Ministério Público; Art. 161. Se não for contestado o pedido e tiver
III - a exposição sumária do fato e o pedido; sido concluído o estudo social ou a perícia realizada por
IV - as provas que serão produzidas, oferecendo, equipe interprofissional ou multidisciplinar, a autoridade
desde logo, o rol de testemunhas e documentos. judiciária dará vista dos autos ao Ministério Público, por 5
(cinco) dias, salvo quando este for o requerente, e
Art. 157. Havendo motivo grave, poderá a decidirá em igual prazo. (Redação dada pela Lei nº
autoridade judiciária, ouvido o Ministério Público, decretar 13.509, de 2017)
a suspensão do pátrio poder poder familiar , liminar ou
incidentalmente, até o julgamento definitivo da causa, § 1º A autoridade judiciária, de ofício ou a
ficando a criança ou adolescente confiado a pessoa requerimento das partes ou do Ministério Público,
idônea, mediante termo de responsabilidade. (Expressão determinará a oitiva de testemunhas que comprovem a
substituída pela Lei nº 12.010, de 2009) presença de uma das causas de suspensão ou
o destituição do poder familiar previstas nos arts. 1.637 e
§ 1 Recebida a petição inicial, a autoridade o
1.638 da Lei n 10.406, de 10 de janeiro de 2002 (Código
judiciária determinará, concomitantemente ao despacho Civil) , ou no art. 24 desta Lei. (Redação dada pela Lei nº
de citação e independentemente de requerimento do 13.509, de 2017)
interessado, a realização de estudo social ou perícia por o
equipe interprofissional ou multidisciplinar para comprovar § 2 (Revogado) . (Redação dada pela Lei nº
a presença de uma das causas de suspensão ou 13.509, de 2017)
o
destituição do poder familiar, ressalvado o disposto no § § 3 Se o pedido importar em modificação de
o guarda, será obrigatória, desde que possível e razoável, a
10 do art. 101 desta Lei, e observada a Lei n 13.431, de
4 de abril de 2017 . (Incluído pela Lei nº 13.509, de 2017) oitiva da criança ou adolescente, respeitado seu estágio
o
§ 2 Em sendo os pais oriundos de comunidades de desenvolvimento e grau de compreensão sobre as
indígenas, é ainda obrigatória a intervenção, junto à implicações da medida. (Incluído pela Lei nº 12.010, de
equipe interprofissional ou multidisciplinar referida no § 2009)
o
1 deste artigo, de representantes do órgão federal § 4º É obrigatória a oitiva dos pais sempre que eles
responsável pela política indigenista, observado o forem identificados e estiverem em local conhecido,
ressalvados os casos de não comparecimento perante a

www.editoradince.com - Acesse e veja se há novidades a respeito deste material – CUIDADO: cópia é crime.
42 LEGISLAÇÃO ESPECIAL E REGIMENTO INTERNO DAS UNIDADES DO SEAS
Justiça quando devidamente citados. (Redação dada pela Art. 166. Se os pais forem falecidos, tiverem sido
Lei nº 13.509, de 2017) destituídos ou suspensos do poder familiar, ou houverem
o aderido expressamente ao pedido de colocação em
§ 5 Se o pai ou a mãe estiverem privados de
liberdade, a autoridade judicial requisitará sua família substituta, este poderá ser formulado diretamente
apresentação para a oitiva. (Incluído pela Lei nº 12.962, em cartório, em petição assinada pelos próprios
de 2014) requerentes, dispensada a assistência de
advogado. (Redação dada pela Lei nº 12.010, de 2009)
Art. 162. Apresentada a resposta, a autoridade o
judiciária dará vista dos autos ao Ministério Público, por § 1 Na hipótese de concordância dos pais, o
cinco dias, salvo quando este for o requerente, juiz: (Redação dada pela Lei nº 13.509, de 2017)
designando, desde logo, audiência de instrução e I - na presença do Ministério Público, ouvirá as
julgamento. partes, devidamente assistidas por advogado ou por
§ 1º (Revogado) . (Redação dada pela Lei nº defensor público, para verificar sua concordância com a
13.509, de 2017) adoção, no prazo máximo de 10 (dez) dias, contado da
o data do protocolo da petição ou da entrega da criança em
§ 2 Na audiência, presentes as partes e o
juízo, tomando por termo as declarações; e (Incluído pela
Ministério Público, serão ouvidas as testemunhas,
Lei nº 13.509, de 2017)
colhendo-se oralmente o parecer técnico, salvo quando
apresentado por escrito, manifestando-se sucessivamente II - declarará a extinção do poder familiar. (Incluído
o requerente, o requerido e o Ministério Público, pelo pela Lei nº 13.509, de 2017)
o
tempo de 20 (vinte) minutos cada um, prorrogável por § 2 O consentimento dos titulares do poder
mais 10 (dez) minutos. (Redação dada pela Lei nº 13.509, familiar será precedido de orientações e esclarecimentos
de 2017) prestados pela equipe interprofissional da Justiça da
o Infância e da Juventude, em especial, no caso de adoção,
§ 3 A decisão será proferida na audiência,
podendo a autoridade judiciária, excepcionalmente, sobre a irrevogabilidade da medida. (Incluído pela Lei nº
designar data para sua leitura no prazo máximo de 5 12.010, de 2009)
o
(cinco) dias. (Incluído pela Lei nº 13.509, de 2017) § 3 São garantidos a livre manifestação de
o vontade dos detentores do poder familiar e o direito ao
§ 4 Quando o procedimento de destituição de
poder familiar for iniciado pelo Ministério Público, não sigilo das informações. (Redação dada pela Lei nº 13.509,
haverá necessidade de nomeação de curador especial em de 2017)
o
favor da criança ou adolescente. (Incluído pela Lei nº § 4 O consentimento prestado por escrito não terá
13.509, de 2017) validade se não for ratificado na audiência a que se refere
o
Art. 163. O prazo máximo para conclusão do o § 1 deste artigo. (Redação dada pela Lei nº 13.509, de
procedimento será de 120 (cento e vinte) dias, e caberá 2017)
o
ao juiz, no caso de notória inviabilidade de manutenção § 5 O consentimento é retratável até a data da
o
do poder familiar, dirigir esforços para preparar a criança realização da audiência especificada no § 1 deste artigo,
ou o adolescente com vistas à colocação em família e os pais podem exercer o arrependimento no prazo de
substituta. (Redação dada pela Lei nº 13.509, de 2017) 10 (dez) dias, contado da data de prolação da sentença
Parágrafo único. A sentença que decretar a perda de extinção do poder familiar. (Redação dada pela Lei nº
ou a suspensão do poder familiar será averbada à 13.509, de 2017)
o
margem do registro de nascimento da criança ou do § 6 O consentimento somente terá valor se for
adolescente. (Incluído pela Lei nº 12.010, de 2009) dado após o nascimento da criança. (Incluído pela Lei nº
Seção III 12.010, de 2009)
o
Da Destituição da Tutela § 7 A família natural e a família substituta
receberão a devida orientação por intermédio de equipe
Art. 164. Na destituição da tutela, observar-se-á o
técnica interprofissional a serviço da Justiça da Infância e
procedimento para a remoção de tutor previsto na lei
da Juventude, preferencialmente com apoio dos técnicos
processual civil e, no que couber, o disposto na seção
responsáveis pela execução da política municipal de
anterior.
garantia do direito à convivência familiar. (Redação dada
Seção IV pela Lei nº 13.509, de 2017)
Da Colocação em Família Substituta Art. 167. A autoridade judiciária, de ofício ou a
Art. 165. São requisitos para a concessão de requerimento das partes ou do Ministério Público,
pedidos de colocação em família substituta: determinará a realização de estudo social ou, se possível,
I - qualificação completa do requerente e de seu perícia por equipe interprofissional, decidindo sobre a
eventual cônjuge, ou companheiro, com expressa concessão de guarda provisória, bem como, no caso de
anuência deste; adoção, sobre o estágio de convivência.
II - indicação de eventual parentesco do requerente Parágrafo único. Deferida a concessão da guarda
e de seu cônjuge, ou companheiro, com a criança ou provisória ou do estágio de convivência, a criança ou o
adolescente, especificando se tem ou não parente vivo; adolescente será entregue ao interessado, mediante
III - qualificação completa da criança ou termo de responsabilidade. (Incluído pela Lei nº 12.010,
adolescente e de seus pais, se conhecidos; de 2009)
IV - indicação do cartório onde foi inscrito Art. 168. Apresentado o relatório social ou o laudo
nascimento, anexando, se possível, uma cópia da pericial, e ouvida, sempre que possível, a criança ou o
respectiva certidão; adolescente, dar-se-á vista dos autos ao Ministério
Público, pelo prazo de cinco dias, decidindo a autoridade
V - declaração sobre a existência de bens, direitos judiciária em igual prazo.
ou rendimentos relativos à criança ou ao adolescente.
Art. 169. Nas hipóteses em que a destituição da
Parágrafo único. Em se tratando de adoção, tutela, a perda ou a suspensão do pátrio poder poder
observar-se-ão também os requisitos específicos. familiar constituir pressuposto lógico da medida principal

www.editoradince.com - Acesse e veja se há novidades a respeito deste material – CUIDADO: cópia é crime.
LEGISLAÇÃO ESPECIAL E REGIMENTO INTERNO DAS UNIDADES DO SEAS 43
de colocação em família substituta, será observado o policial. À falta de repartição policial especializada, o
procedimento contraditório previsto nas Seções II e III adolescente aguardará a apresentação em dependência
deste Capítulo. (Expressão substituída pela Lei nº 12.010, separada da destinada a maiores, não podendo, em
de 2009) qualquer hipótese, exceder o prazo referido no parágrafo
Parágrafo único. A perda ou a modificação da anterior.
guarda poderá ser decretada nos mesmos autos do Art. 176. Sendo o adolescente liberado, a
procedimento, observado o disposto no art. 35. autoridade policial encaminhará imediatamente ao
Art. 170. Concedida a guarda ou a tutela, observar- representante do Ministério Público cópia do auto de
se-á o disposto no art. 32, e, quanto à adoção, o contido apreensão ou boletim de ocorrência.
no art. 47. Art. 177. Se, afastada a hipótese de flagrante,
Parágrafo único. A colocação de criança ou houver indícios de participação de adolescente na prática
adolescente sob a guarda de pessoa inscrita em de ato infracional, a autoridade policial encaminhará ao
programa de acolhimento familiar será comunicada pela representante do Ministério Público relatório das
autoridade judiciária à entidade por este responsável no investigações e demais documentos.
prazo máximo de 5 (cinco) dias. (Incluído pela Lei nº Art. 178. O adolescente a quem se atribua autoria
12.010, de 2009) de ato infracional não poderá ser conduzido ou
Seção V transportado em compartimento fechado de veículo
policial, em condições atentatórias à sua dignidade, ou
Da Apuração de Ato Infracional Atribuído a
Adolescente que impliquem risco à sua integridade física ou mental,
sob pena de responsabilidade.
Art. 171. O adolescente apreendido por força de
Art. 179. Apresentado o adolescente, o
ordem judicial será, desde logo, encaminhado à
representante do Ministério Público, no mesmo dia e à
autoridade judiciária.
vista do auto de apreensão, boletim de ocorrência ou
Art. 172. O adolescente apreendido em flagrante de relatório policial, devidamente autuados pelo cartório
ato infracional será, desde logo, encaminhado à judicial e com informação sobre os antecedentes do
autoridade policial competente. adolescente, procederá imediata e informalmente à sua
Parágrafo único. Havendo repartição policial oitiva e, em sendo possível, de seus pais ou responsável,
especializada para atendimento de adolescente e em se vítima e testemunhas.
tratando de ato infracional praticado em co-autoria com Parágrafo único. Em caso de não apresentação, o
maior, prevalecerá a atribuição da repartição representante do Ministério Público notificará os pais ou
especializada, que, após as providências necessárias e responsável para apresentação do adolescente, podendo
conforme o caso, encaminhará o adulto à repartição requisitar o concurso das polícias civil e militar.
policial própria.
Art. 180. Adotadas as providências a que alude o
Art. 173. Em caso de flagrante de ato infracional artigo anterior, o representante do Ministério Público
cometido mediante violência ou grave ameaça a pessoa, poderá:
a autoridade policial, sem prejuízo do disposto nos arts.
I - promover o arquivamento dos autos;
106, parágrafo único, e 107, deverá:
II - conceder a remissão;
I - lavrar auto de apreensão, ouvidos as
testemunhas e o adolescente; III - representar à autoridade judiciária para
aplicação de medida sócio-educativa.
II - apreender o produto e os instrumentos da
infração; Art. 181. Promovido o arquivamento dos autos ou
concedida a remissão pelo representante do Ministério
III - requisitar os exames ou perícias necessários à
Público, mediante termo fundamentado, que conterá o
comprovação da materialidade e autoria da infração.
resumo dos fatos, os autos serão conclusos à autoridade
Parágrafo único. Nas demais hipóteses de judiciária para homologação.
flagrante, a lavratura do auto poderá ser substituída por
§ 1º Homologado o arquivamento ou a remissão, a
boletim de ocorrência circunstanciada.
autoridade judiciária determinará, conforme o caso, o
Art. 174. Comparecendo qualquer dos pais ou cumprimento da medida.
responsável, o adolescente será prontamente liberado
§ 2º Discordando, a autoridade judiciária fará
pela autoridade policial, sob termo de compromisso e
remessa dos autos ao Procurador-Geral de Justiça,
responsabilidade de sua apresentação ao representante
mediante despacho fundamentado, e este oferecerá
do Ministério Público, no mesmo dia ou, sendo
representação, designará outro membro do Ministério
impossível, no primeiro dia útil imediato, exceto quando,
Público para apresentá-la, ou ratificará o arquivamento ou
pela gravidade do ato infracional e sua repercussão
a remissão, que só então estará a autoridade judiciária
social, deva o adolescente permanecer sob internação
obrigada a homologar.
para garantia de sua segurança pessoal ou manutenção
da ordem pública. Art. 182. Se, por qualquer razão, o representante
do Ministério Público não promover o arquivamento ou
Art. 175. Em caso de não liberação, a autoridade
conceder a remissão, oferecerá representação à
policial encaminhará, desde logo, o adolescente ao
autoridade judiciária, propondo a instauração de
representante do Ministério Público, juntamente com
procedimento para aplicação da medida sócio-educativa
cópia do auto de apreensão ou boletim de ocorrência.
que se afigurar a mais adequada.
§ 1º Sendo impossível a apresentação imediata, a
§ 1º A representação será oferecida por petição,
autoridade policial encaminhará o adolescente à entidade
que conterá o breve resumo dos fatos e a classificação do
de atendimento, que fará a apresentação ao
ato infracional e, quando necessário, o rol de
representante do Ministério Público no prazo de vinte e
testemunhas, podendo ser deduzida oralmente, em
quatro horas.
sessão diária instalada pela autoridade judiciária.
§ 2º Nas localidades onde não houver entidade de
atendimento, a apresentação far-se-á pela autoridade

www.editoradince.com - Acesse e veja se há novidades a respeito deste material – CUIDADO: cópia é crime.
44 LEGISLAÇÃO ESPECIAL E REGIMENTO INTERNO DAS UNIDADES DO SEAS
§ 2º A representação independe de prova pré- Art. 188. A remissão, como forma de extinção ou
constituída da autoria e materialidade. suspensão do processo, poderá ser aplicada em qualquer
Art. 183. O prazo máximo e improrrogável para a fase do procedimento, antes da sentença.
conclusão do procedimento, estando o adolescente Art. 189. A autoridade judiciária não aplicará
internado provisoriamente, será de quarenta e cinco dias. qualquer medida, desde que reconheça na sentença:
Art. 184. Oferecida a representação, a autoridade I - estar provada a inexistência do fato;
judiciária designará audiência de apresentação do II - não haver prova da existência do fato;
adolescente, decidindo, desde logo, sobre a decretação
III - não constituir o fato ato infracional;
ou manutenção da internação, observado o disposto no
art. 108 e parágrafo. IV - não existir prova de ter o adolescente
concorrido para o ato infracional.
§ 1º O adolescente e seus pais ou responsável
serão cientificados do teor da representação, e notificados Parágrafo único. Na hipótese deste artigo, estando
a comparecer à audiência, acompanhados de advogado. o adolescente internado, será imediatamente colocado em
liberdade.
§ 2º Se os pais ou responsável não forem
localizados, a autoridade judiciária dará curador especial Art. 190. A intimação da sentença que aplicar
ao adolescente. medida de internação ou regime de semi-liberdade será
feita:
§ 3º Não sendo localizado o adolescente, a
autoridade judiciária expedirá mandado de busca e I - ao adolescente e ao seu defensor;
apreensão, determinando o sobrestamento do feito, até a II - quando não for encontrado o adolescente, a
efetiva apresentação. seus pais ou responsável, sem prejuízo do defensor.
§ 4º Estando o adolescente internado, será § 1º Sendo outra a medida aplicada, a intimação
requisitada a sua apresentação, sem prejuízo da far-se-á unicamente na pessoa do defensor.
notificação dos pais ou responsável. § 2º Recaindo a intimação na pessoa do
Art. 185. A internação, decretada ou mantida pela adolescente, deverá este manifestar se deseja ou não
autoridade judiciária, não poderá ser cumprida em recorrer da sentença.
estabelecimento prisional. Seção V-A
§ 1º Inexistindo na comarca entidade com as (Incluído pela Lei nº 13.441, de 2017)
características definidas no art. 123, o adolescente deverá Da Infiltração de Agentes de Polícia para a
ser imediatamente transferido para a localidade mais Investigação de Crimes contra a Dignidade Sexual de
próxima. Criança e de Adolescente”
§ 2º Sendo impossível a pronta transferência, o Art. 190-A. A infiltração de agentes de polícia na
adolescente aguardará sua remoção em repartição internet com o fim de investigar os crimes previstos
policial, desde que em seção isolada dos adultos e com nos arts. 240 , 241 , 241-A , 241-B , 241-C e 241-D desta
instalações apropriadas, não podendo ultrapassar o prazo Lei e nos arts. 154-A , 217-A , 218 , 218-A e 218-B do
máximo de cinco dias, sob pena de responsabilidade. Decreto-Lei nº 2.848, de 7 de dezembro de 1940 (Código
Art. 186. Comparecendo o adolescente, seus pais Penal) , obedecerá às seguintes regras: (Incluído pela Lei
ou responsável, a autoridade judiciária procederá à oitiva nº 13.441, de 2017)
dos mesmos, podendo solicitar opinião de profissional I – será precedida de autorização judicial
qualificado. devidamente circunstanciada e fundamentada, que
§ 1º Se a autoridade judiciária entender adequada estabelecerá os limites da infiltração para obtenção de
a remissão, ouvirá o representante do Ministério Público, prova, ouvido o Ministério Público; (Incluído pela Lei nº
proferindo decisão. 13.441, de 2017)
§ 2º Sendo o fato grave, passível de aplicação de II – dar-se-á mediante requerimento do Ministério
medida de internação ou colocação em regime de semi- Público ou representação de delegado de polícia e
liberdade, a autoridade judiciária, verificando que o conterá a demonstração de sua necessidade, o alcance
adolescente não possui advogado constituído, nomeará das tarefas dos policiais, os nomes ou apelidos das
defensor, designando, desde logo, audiência em pessoas investigadas e, quando possível, os dados de
continuação, podendo determinar a realização de conexão ou cadastrais que permitam a identificação
diligências e estudo do caso. dessas pessoas; (Incluído pela Lei nº 13.441, de 2017)
§ 3º O advogado constituído ou o defensor III – não poderá exceder o prazo de 90 (noventa)
nomeado, no prazo de três dias contado da audiência de dias, sem prejuízo de eventuais renovações, desde que o
apresentação, oferecerá defesa prévia e rol de total não exceda a 720 (setecentos e vinte) dias e seja
testemunhas. demonstrada sua efetiva necessidade, a critério da
§ 4º Na audiência em continuação, ouvidas as autoridade judicial. (Incluído pela Lei nº 13.441, de 2017)
testemunhas arroladas na representação e na defesa § 1 º A autoridade judicial e o Ministério Público
prévia, cumpridas as diligências e juntado o relatório da poderão requisitar relatórios parciais da operação de
equipe interprofissional, será dada a palavra ao infiltração antes do término do prazo de que trata o inciso
representante do Ministério Público e ao defensor, II do § 1 º deste artigo. (Incluído pela Lei nº 13.441, de
sucessivamente, pelo tempo de vinte minutos para cada 2017)
um, prorrogável por mais dez, a critério da autoridade § 2 º Para efeitos do disposto no inciso I do §
judiciária, que em seguida proferirá decisão. 1 º deste artigo, consideram-se: (Incluído pela Lei nº
Art. 187. Se o adolescente, devidamente notificado, 13.441, de 2017)
não comparecer, injustificadamente à audiência de I – dados de conexão: informações referentes a
apresentação, a autoridade judiciária designará nova hora, data, início, término, duração, endereço de
data, determinando sua condução coercitiva. Protocolo de Internet (IP) utilizado e terminal de origem da
conexão; (Incluído pela Lei nº 13.441, de 2017)

www.editoradince.com - Acesse e veja se há novidades a respeito deste material – CUIDADO: cópia é crime.
LEGISLAÇÃO ESPECIAL E REGIMENTO INTERNO DAS UNIDADES DO SEAS 45
II – dados cadastrais: informações referentes a Art. 193. Apresentada ou não a resposta, e sendo
nome e endereço de assinante ou de usuário registrado necessário, a autoridade judiciária designará audiência de
ou autenticado para a conexão a quem endereço de IP, instrução e julgamento, intimando as partes.
identificação de usuário ou código de acesso tenha sido § 1º Salvo manifestação em audiência, as partes e
atribuído no momento da conexão. o Ministério Público terão cinco dias para oferecer
§ 3 º A infiltração de agentes de polícia na internet alegações finais, decidindo a autoridade judiciária em
não será admitida se a prova puder ser obtida por outros igual prazo.
meios. (Incluído pela Lei nº 13.441, de 2017) § 2º Em se tratando de afastamento provisório ou
Art. 190-B. As informações da operação de definitivo de dirigente de entidade governamental, a
infiltração serão encaminhadas diretamente ao juiz autoridade judiciária oficiará à autoridade administrativa
responsável pela autorização da medida, que zelará por imediatamente superior ao afastado, marcando prazo
seu sigilo. (Incluído pela Lei nº 13.441, de 2017) para a substituição.
Parágrafo único. Antes da conclusão da operação, § 3º Antes de aplicar qualquer das medidas, a
o acesso aos autos será reservado ao juiz, ao Ministério autoridade judiciária poderá fixar prazo para a remoção
Público e ao delegado de polícia responsável pela das irregularidades verificadas. Satisfeitas as exigências,
operação, com o objetivo de garantir o sigilo das o processo será extinto, sem julgamento de mérito.
investigações. (Incluído pela Lei nº 13.441, de 2017) § 4º A multa e a advertência serão impostas ao
Art. 190-C. Não comete crime o policial que oculta dirigente da entidade ou programa de atendimento.
a sua identidade para, por meio da internet, colher Seção VII
indícios de autoria e materialidade dos crimes previstos
Da Apuração de Infração Administrativa às Normas de
nos arts. 240 , 241 , 241-A , 241-B , 241-C e 241-D desta
Proteção à Criança e ao Adolescente
Lei e nos arts. 154-A , 217-A , 218 , 218-A e 218-B do
Decreto-Lei nº 2.848, de 7 de dezembro de 1940 (Código Art. 194. O procedimento para imposição de
Penal) . (Incluído pela Lei nº 13.441, de 2017) penalidade administrativa por infração às normas de
proteção à criança e ao adolescente terá início por
Parágrafo único. O agente policial infiltrado que
representação do Ministério Público, ou do Conselho
deixar de observar a estrita finalidade da investigação
Tutelar, ou auto de infração elaborado por servidor efetivo
responderá pelos excessos praticados. (Incluído pela Lei
ou voluntário credenciado, e assinado por duas
nº 13.441, de 2017)
testemunhas, se possível.
Art. 190-D. Os órgãos de registro e cadastro
§ 1º No procedimento iniciado com o auto de
público poderão incluir nos bancos de dados próprios,
infração, poderão ser usadas fórmulas impressas,
mediante procedimento sigiloso e requisição da
especificando-se a natureza e as circunstâncias da
autoridade judicial, as informações necessárias à
infração.
efetividade da identidade fictícia criada. (Incluído pela Lei
nº 13.441, de 2017) § 2º Sempre que possível, à verificação da infração
seguir-se-á a lavratura do auto, certificando-se, em caso
Parágrafo único. O procedimento sigiloso de que
contrário, dos motivos do retardamento.
trata esta Seção será numerado e tombado em livro
específico. (Incluído pela Lei nº 13.441, de 2017) Art. 195. O requerido terá prazo de dez dias para
apresentação de defesa, contado da data da intimação,
Art. 190-E. Concluída a investigação, todos os atos
que será feita:
eletrônicos praticados durante a operação deverão ser
registrados, gravados, armazenados e encaminhados ao I - pelo autuante, no próprio auto, quando este for
juiz e ao Ministério Público, juntamente com relatório lavrado na presença do requerido;
circunstanciado. (Incluído pela Lei nº 13.441, de 2017) II - por oficial de justiça ou funcionário legalmente
Parágrafo único. Os atos eletrônicos registrados habilitado, que entregará cópia do auto ou da
citados no caput deste artigo serão reunidos em autos representação ao requerido, ou a seu representante legal,
apartados e apensados ao processo criminal juntamente lavrando certidão;
com o inquérito policial, assegurando-se a preservação da III - por via postal, com aviso de recebimento, se
identidade do agente policial infiltrado e a intimidade das não for encontrado o requerido ou seu representante
crianças e dos adolescentes envolvidos. (Incluído pela Lei legal;
nº 13.441, de 2017) IV - por edital, com prazo de trinta dias, se incerto
Seção VI ou não sabido o paradeiro do requerido ou de seu
Da Apuração de Irregularidades em Entidade de representante legal.
Atendimento Art. 196. Não sendo apresentada a defesa no prazo
Art. 191. O procedimento de apuração de legal, a autoridade judiciária dará vista dos autos do
irregularidades em entidade governamental e não- Ministério Público, por cinco dias, decidindo em igual
governamental terá início mediante portaria da autoridade prazo.
judiciária ou representação do Ministério Público ou do Art. 197. Apresentada a defesa, a autoridade
Conselho Tutelar, onde conste, necessariamente, resumo judiciária procederá na conformidade do artigo anterior,
dos fatos. ou, sendo necessário, designará audiência de instrução e
Parágrafo único. Havendo motivo grave, poderá a julgamento. (Incluído pela Lei nº 12.010, de 2009)
autoridade judiciária, ouvido o Ministério Público, decretar Parágrafo único. Colhida a prova oral, manifestar-
liminarmente o afastamento provisório do dirigente da se-ão sucessivamente o Ministério Público e o procurador
entidade, mediante decisão fundamentada. do requerido, pelo tempo de vinte minutos para cada um,
Art. 192. O dirigente da entidade será citado para, prorrogável por mais dez, a critério da autoridade
no prazo de dez dias, oferecer resposta escrita, podendo judiciária, que em seguida proferirá sentença.
juntar documentos e indicar as provas a produzir. Seção VIII
(Incluída pela Lei nº 12.010, de 2009)
Da Habilitação de Pretendentes à Adoção

www.editoradince.com - Acesse e veja se há novidades a respeito deste material – CUIDADO: cópia é crime.
46 LEGISLAÇÃO ESPECIAL E REGIMENTO INTERNO DAS UNIDADES DO SEAS
Art. 197-A. Os postulantes à adoção, domiciliados à convivência familiar. (Redação dada pela Lei nº 13.509,
no Brasil, apresentarão petição inicial na qual de 2017)
conste: (Incluído pela Lei nº 12.010, de 2009) o
§ 3 É recomendável que as crianças e os
I - qualificação completa; (Incluído pela Lei nº adolescentes acolhidos institucionalmente ou por família
12.010, de 2009) acolhedora sejam preparados por equipe interprofissional
II - dados familiares; (Incluído pela Lei nº 12.010, antes da inclusão em família adotiva. (Incluído pela Lei nº
de 2009) 13.509, de 2017)
III - cópias autenticadas de certidão de nascimento Art. 197-D. Certificada nos autos a conclusão da
ou casamento, ou declaração relativa ao período de união participação no programa referido no art. 197-C desta Lei,
estável; (Incluído pela Lei nº 12.010, de 2009) a autoridade judiciária, no prazo de 48 (quarenta e oito)
horas, decidirá acerca das diligências requeridas pelo
IV - cópias da cédula de identidade e inscrição no
Ministério Público e determinará a juntada do estudo
Cadastro de Pessoas Físicas; (Incluído pela Lei nº
psicossocial, designando, conforme o caso, audiência de
12.010, de 2009)
instrução e julgamento. (Incluído pela Lei nº 12.010, de
V - comprovante de renda e domicílio; (Incluído 2009)
pela Lei nº 12.010, de 2009)
Parágrafo único. Caso não sejam requeridas
VI - atestados de sanidade física e mental (Incluído diligências, ou sendo essas indeferidas, a autoridade
pela Lei nº 12.010, de 2009) judiciária determinará a juntada do estudo psicossocial,
VII - certidão de antecedentes criminais; (Incluído abrindo a seguir vista dos autos ao Ministério Público, por
pela Lei nº 12.010, de 2009) 5 (cinco) dias, decidindo em igual prazo. (Incluído pela Lei
VIII - certidão negativa de distribuição nº 12.010, de 2009)
cível. (Incluído pela Lei nº 12.010, de 2009) Art. 197-E. Deferida a habilitação, o postulante será
Art. 197-B. A autoridade judiciária, no prazo de 48 inscrito nos cadastros referidos no art. 50 desta Lei, sendo
(quarenta e oito) horas, dará vista dos autos ao Ministério a sua convocação para a adoção feita de acordo com
Público, que no prazo de 5 (cinco) dias poderá: (Incluído ordem cronológica de habilitação e conforme a
pela Lei nº 12.010, de 2009) disponibilidade de crianças ou adolescentes
I - apresentar quesitos a serem respondidos pela adotáveis. (Incluído pela Lei nº 12.010, de 2009)
o
equipe interprofissional encarregada de elaborar o estudo § 1 A ordem cronológica das habilitações somente
técnico a que se refere o art. 197-C desta Lei; (Incluído poderá deixar de ser observada pela autoridade judiciária
pela Lei nº 12.010, de 2009) nas hipóteses previstas no § 13 do art. 50 desta Lei,
II - requerer a designação de audiência para oitiva quando comprovado ser essa a melhor solução no
dos postulantes em juízo e testemunhas; (Incluído pela interesse do adotando. (Incluído pela Lei nº 12.010, de
Lei nº 12.010, de 2009) 2009)
o
III - requerer a juntada de documentos § 2 A habilitação à adoção deverá ser renovada
complementares e a realização de outras diligências que no mínimo trienalmente mediante avaliação por equipe
entender necessárias. (Incluído pela Lei nº 12.010, de interprofissional. (Redação dada pela Lei nº 13.509, de
2009) 2017)
o
Art. 197-C. Intervirá no feito, obrigatoriamente, § 3 Quando o adotante candidatar-se a uma nova
equipe interprofissional a serviço da Justiça da Infância e adoção, será dispensável a renovação da habilitação,
da Juventude, que deverá elaborar estudo psicossocial, bastando a avaliação por equipe
que conterá subsídios que permitam aferir a capacidade e interprofissional. (Incluído pela Lei nº 13.509, de 2017)
o
o preparo dos postulantes para o exercício de uma § 4 Após 3 (três) recusas injustificadas, pelo
paternidade ou maternidade responsável, à luz dos habilitado, à adoção de crianças ou adolescentes
requisitos e princípios desta Lei. (Incluído pela Lei nº indicados dentro do perfil escolhido, haverá reavaliação
12.010, de 2009) da habilitação concedida. (Incluído pela Lei nº 13.509, de
o
§ 1 É obrigatória a participação dos postulantes 2017)
o
em programa oferecido pela Justiça da Infância e da § 5 A desistência do pretendente em relação à
Juventude, preferencialmente com apoio dos técnicos guarda para fins de adoção ou a devolução da criança ou
responsáveis pela execução da política municipal de do adolescente depois do trânsito em julgado da sentença
garantia do direito à convivência familiar e dos grupos de de adoção importará na sua exclusão dos cadastros de
apoio à adoção devidamente habilitados perante a Justiça adoção e na vedação de renovação da habilitação, salvo
da Infância e da Juventude, que inclua preparação decisão judicial fundamentada, sem prejuízo das demais
psicológica, orientação e estímulo à adoção inter-racial, sanções previstas na legislação vigente. (Incluído pela Lei
de crianças ou de adolescentes com deficiência, com nº 13.509, de 2017)
doenças crônicas ou com necessidades específicas de Art. 197-F. O prazo máximo para conclusão da
saúde, e de grupos de irmãos. (Redação dada pela Lei nº habilitação à adoção será de 120 (cento e vinte) dias,
13.509, de 2017) prorrogável por igual período, mediante decisão
o fundamentada da autoridade judiciária. (Incluído pela Lei
§ 2 Sempre que possível e recomendável, a etapa
o nº 13.509, de 2017)
obrigatória da preparação referida no § 1 deste artigo
incluirá o contato com crianças e adolescentes em regime Capítulo IV
de acolhimento familiar ou institucional, a ser realizado Dos Recursos
sob orientação, supervisão e avaliação da equipe técnica
Art. 198. Nos procedimentos afetos à Justiça da
da Justiça da Infância e da Juventude e dos grupos de
Infância e da Juventude, inclusive os relativos à execução
apoio à adoção, com apoio dos técnicos responsáveis
das medidas socioeducativas, adotar-se-á o sistema
pelo programa de acolhimento familiar e institucional e o
recursal da Lei n 5.869, de 11 de janeiro de 1973
pela execução da política municipal de garantia do direito
(Código de Processo Civil) , com as seguintes
adaptações: (Redação dada pela Lei nº 12.594, de 2012)

www.editoradince.com - Acesse e veja se há novidades a respeito deste material – CUIDADO: cópia é crime.
LEGISLAÇÃO ESPECIAL E REGIMENTO INTERNO DAS UNIDADES DO SEAS 47
I - os recursos serão interpostos II - promover e acompanhar os procedimentos
independentemente de preparo; relativos às infrações atribuídas a adolescentes;
II - em todos os recursos, salvo nos embargos de III - promover e acompanhar as ações de alimentos
declaração, o prazo para o Ministério Público e para a e os procedimentos de suspensão e destituição do pátrio
defesa será sempre de 10 (dez) dias; (Redação dada pela poder poder familiar , nomeação e remoção de tutores,
Lei nº 12.594, de 2012) curadores e guardiães, bem como oficiar em todos os
III - os recursos terão preferência de julgamento e demais procedimentos da competência da Justiça da
dispensarão revisor; Infância e da Juventude; (Expressão substituída pela Lei
nº 12.010, de 2009)
IV - (Revogado pela Lei nº 12.010, de 2009)
IV - promover, de ofício ou por solicitação dos
V - (Revogado pela Lei nº 12.010, de 2009)
interessados, a especialização e a inscrição de hipoteca
VI - (Revogado pela Lei nº 12.010, de 2009) legal e a prestação de contas dos tutores, curadores e
VII - antes de determinar a remessa dos autos à quaisquer administradores de bens de crianças e
superior instância, no caso de apelação, ou do adolescentes nas hipóteses do art. 98;
instrumento, no caso de agravo, a autoridade judiciária V - promover o inquérito civil e a ação civil pública
proferirá despacho fundamentado, mantendo ou para a proteção dos interesses individuais, difusos ou
reformando a decisão, no prazo de cinco dias; coletivos relativos à infância e à adolescência, inclusive os
VIII - mantida a decisão apelada ou agravada, o definidos no art. 220, § 3º inciso II, da Constituição
escrivão remeterá os autos ou o instrumento à superior Federal ;
instância dentro de vinte e quatro horas, VI - instaurar procedimentos administrativos e, para
independentemente de novo pedido do recorrente; se a instruí-los:
reformar, a remessa dos autos dependerá de pedido
a) expedir notificações para colher depoimentos ou
expresso da parte interessada ou do Ministério Público,
esclarecimentos e, em caso de não comparecimento
no prazo de cinco dias, contados da intimação.
injustificado, requisitar condução coercitiva, inclusive pela
Art. 199. Contra as decisões proferidas com base polícia civil ou militar;
no art. 149 caberá recurso de apelação.
b) requisitar informações, exames, perícias e
Art. 199-A. A sentença que deferir a adoção produz documentos de autoridades municipais, estaduais e
efeito desde logo, embora sujeita a apelação, que será federais, da administração direta ou indireta, bem como
recebida exclusivamente no efeito devolutivo, salvo se se promover inspeções e diligências investigatórias;
tratar de adoção internacional ou se houver perigo de
c) requisitar informações e documentos a
dano irreparável ou de difícil reparação ao
particulares e instituições privadas;
adotando. (Incluído pela Lei nº 12.010, de 2009)
VII - instaurar sindicâncias, requisitar diligências
Art. 199-B. A sentença que destituir ambos ou
investigatórias e determinar a instauração de inquérito
qualquer dos genitores do poder familiar fica sujeita a
policial, para apuração de ilícitos ou infrações às normas
apelação, que deverá ser recebida apenas no efeito
de proteção à infância e à juventude;
devolutivo. (Incluído pela Lei nº 12.010, de 2009)
VIII - zelar pelo efetivo respeito aos direitos e
Art. 199-C. Os recursos nos procedimentos de
garantias legais assegurados às crianças e adolescentes,
adoção e de destituição de poder familiar, em face da
promovendo as medidas judiciais e extrajudiciais cabíveis;
relevância das questões, serão processados com
prioridade absoluta, devendo ser imediatamente IX - impetrar mandado de segurança, de injunção e
distribuídos, ficando vedado que aguardem, em qualquer habeas corpus, em qualquer juízo, instância ou tribunal,
situação, oportuna distribuição, e serão colocados em na defesa dos interesses sociais e individuais
mesa para julgamento sem revisão e com parecer urgente indisponíveis afetos à criança e ao adolescente;
do Ministério Público. (Incluído pela Lei nº 12.010, de X - representar ao juízo visando à aplicação de
2009) penalidade por infrações cometidas contra as normas de
Art. 199-D. O relator deverá colocar o processo em proteção à infância e à juventude, sem prejuízo da
mesa para julgamento no prazo máximo de 60 (sessenta) promoção da responsabilidade civil e penal do infrator,
dias, contado da sua conclusão. (Incluído pela Lei nº quando cabível;
12.010, de 2009) XI - inspecionar as entidades públicas e
Parágrafo único. O Ministério Público será intimado particulares de atendimento e os programas de que trata
da data do julgamento e poderá na sessão, se entender esta Lei, adotando de pronto as medidas administrativas
necessário, apresentar oralmente seu parecer. (Incluído ou judiciais necessárias à remoção de irregularidades
pela Lei nº 12.010, de 2009) porventura verificadas;
Art. 199-E. O Ministério Público poderá requerer a XII - requisitar força policial, bem como a
instauração de procedimento para apuração de colaboração dos serviços médicos, hospitalares,
responsabilidades se constatar o descumprimento das educacionais e de assistência social, públicos ou
providências e do prazo previstos nos artigos privados, para o desempenho de suas atribuições.
anteriores. (Incluído pela Lei nº 12.010, de 2009) § 1º A legitimação do Ministério Público para as
Capítulo V ações cíveis previstas neste artigo não impede a de
terceiros, nas mesmas hipóteses, segundo dispuserem a
Do Ministério Público
Constituição e esta Lei.
Art. 200. As funções do Ministério Público previstas
§ 2º As atribuições constantes deste artigo não
nesta Lei serão exercidas nos termos da respectiva lei
excluem outras, desde que compatíveis com a finalidade
orgânica.
do Ministério Público.
Art. 201. Compete ao Ministério Público:
§ 3º O representante do Ministério Público, no
I - conceder a remissão como forma de exclusão do exercício de suas funções, terá livre acesso a todo local
processo; onde se encontre criança ou adolescente.

www.editoradince.com - Acesse e veja se há novidades a respeito deste material – CUIDADO: cópia é crime.
48 LEGISLAÇÃO ESPECIAL E REGIMENTO INTERNO DAS UNIDADES DO SEAS
§ 4º O representante do Ministério Público será II - de atendimento educacional especializado aos
responsável pelo uso indevido das informações e portadores de deficiência;
documentos que requisitar, nas hipóteses legais de sigilo. III – de atendimento em creche e pré-escola às
§ 5º Para o exercício da atribuição de que trata o crianças de zero a cinco anos de idade; (Redação dada
inciso VIII deste artigo, poderá o representante do pela Lei nº 13.306, de 2016)
Ministério Público: IV - de ensino noturno regular, adequado às
a) reduzir a termo as declarações do reclamante, condições do educando;
instaurando o competente procedimento, sob sua V - de programas suplementares de oferta de
presidência; material didático-escolar, transporte e assistência à saúde
b) entender-se diretamente com a pessoa ou do educando do ensino fundamental;
autoridade reclamada, em dia, local e horário previamente VI - de serviço de assistência social visando à
notificados ou acertados; proteção à família, à maternidade, à infância e à
c) efetuar recomendações visando à melhoria dos adolescência, bem como ao amparo às crianças e
serviços públicos e de relevância pública afetos à criança adolescentes que dele necessitem;
e ao adolescente, fixando prazo razoável para sua VII - de acesso às ações e serviços de saúde;
perfeita adequação.
VIII - de escolarização e profissionalização dos
Art. 202. Nos processos e procedimentos em que adolescentes privados de liberdade.
não for parte, atuará obrigatoriamente o Ministério Público
IX - de ações, serviços e programas de orientação,
na defesa dos direitos e interesses de que cuida esta Lei,
apoio e promoção social de famílias e destinados ao
hipótese em que terá vista dos autos depois das partes,
pleno exercício do direito à convivência familiar por
podendo juntar documentos e requerer diligências,
crianças e adolescentes. (Incluído pela Lei nº 12.010, de
usando os recursos cabíveis.
2009)
Art. 203. A intimação do Ministério Público, em
X - de programas de atendimento para a execução
qualquer caso, será feita pessoalmente.
das medidas socioeducativas e aplicação de medidas de
Art. 204. A falta de intervenção do Ministério proteção. (Incluído pela Lei nº 12.594, de 2012)
Público acarreta a nulidade do feito, que será declarada
XI - de políticas e programas integrados de
de ofício pelo juiz ou a requerimento de qualquer
atendimento à criança e ao adolescente vítima ou
interessado.
testemunha de violência. (Incluído pela Lei nº 13.431, de
Art. 205. As manifestações processuais do representante 2017) ( )
do Ministério Público deverão ser fundamentadas. o
§ 1 As hipóteses previstas neste artigo não
Capítulo VI excluem da proteção judicial outros interesses individuais,
Do Advogado difusos ou coletivos, próprios da infância e da
Art. 206. A criança ou o adolescente, seus pais ou adolescência, protegidos pela Constituição e pela
responsável, e qualquer pessoa que tenha legítimo Lei. (Renumerado do Parágrafo único pela Lei nº 11.259,
interesse na solução da lide poderão intervir nos de 2005)
o
procedimentos de que trata esta Lei, através de § 2 A investigação do desaparecimento de
advogado, o qual será intimado para todos os atos, crianças ou adolescentes será realizada imediatamente
pessoalmente ou por publicação oficial, respeitado o após notificação aos órgãos competentes, que deverão
segredo de justiça. comunicar o fato aos portos, aeroportos, Polícia
Parágrafo único. Será prestada assistência Rodoviária e companhias de transporte interestaduais e
judiciária integral e gratuita àqueles que dela internacionais, fornecendo-lhes todos os dados
necessitarem. necessários à identificação do desaparecido. (Incluído
pela Lei nº 11.259, de 2005)
Art. 207. Nenhum adolescente a quem se atribua a
prática de ato infracional, ainda que ausente ou foragido, Art. 209. As ações previstas neste Capítulo serão
será processado sem defensor. propostas no foro do local onde ocorreu ou deva ocorrer a
ação ou omissão, cujo juízo terá competência absoluta
§ 1º Se o adolescente não tiver defensor, ser-lhe-á
para processar a causa, ressalvadas a competência da
nomeado pelo juiz, ressalvado o direito de, a todo tempo,
Justiça Federal e a competência originária dos tribunais
constituir outro de sua preferência.
superiores.
§ 2º A ausência do defensor não determinará o
Art. 210. Para as ações cíveis fundadas em
adiamento de nenhum ato do processo, devendo o juiz
interesses coletivos ou difusos, consideram-se
nomear substituto, ainda que provisoriamente, ou para o
legitimados concorrentemente:
só efeito do ato.
I - o Ministério Público;
§ 3º Será dispensada a outorga de mandato,
quando se tratar de defensor nomeado ou, sido II - a União, os estados, os municípios, o Distrito
constituído, tiver sido indicado por ocasião de ato formal Federal e os territórios;
com a presença da autoridade judiciária. III - as associações legalmente constituídas há pelo
Capítulo VII menos um ano e que incluam entre seus fins institucionais
a defesa dos interesses e direitos protegidos por esta Lei,
Da Proteção Judicial dos Interesses Individuais,
dispensada a autorização da assembléia, se houver
Difusos e Coletivos
prévia autorização estatutária.
Art. 208. Regem-se pelas disposições desta Lei as
§ 1º Admitir-se-á litisconsórcio facultativo entre os
ações de responsabilidade por ofensa aos direitos
Ministérios Públicos da União e dos estados na defesa
assegurados à criança e ao adolescente, referentes ao
dos interesses e direitos de que cuida esta Lei.
não oferecimento ou oferta irregular:
§ 2º Em caso de desistência ou abandono da ação
I - do ensino obrigatório;
por associação legitimada, o Ministério Público ou outro
legitimado poderá assumir a titularidade ativa.

www.editoradince.com - Acesse e veja se há novidades a respeito deste material – CUIDADO: cópia é crime.
LEGISLAÇÃO ESPECIAL E REGIMENTO INTERNO DAS UNIDADES DO SEAS 49
Art. 211. Os órgãos públicos legitimados poderão Art. 219. Nas ações de que trata este Capítulo, não
tomar dos interessados compromisso de ajustamento de haverá adiantamento de custas, emolumentos, honorários
sua conduta às exigências legais, o qual terá eficácia de periciais e quaisquer outras despesas.
título executivo extrajudicial. Art. 220. Qualquer pessoa poderá e o servidor
Art. 212. Para defesa dos direitos e interesses público deverá provocar a iniciativa do Ministério Público,
protegidos por esta Lei, são admissíveis todas as prestando-lhe informações sobre fatos que constituam
espécies de ações pertinentes. objeto de ação civil, e indicando-lhe os elementos de
§ 1º Aplicam-se às ações previstas neste Capítulo convicção.
as normas do Código de Processo Civil. Art. 221. Se, no exercício de suas funções, os
§ 2º Contra atos ilegais ou abusivos de autoridade juízos e tribunais tiverem conhecimento de fatos que
pública ou agente de pessoa jurídica no exercício de possam ensejar a propositura de ação civil, remeterão
atribuições do poder público, que lesem direito líquido e peças ao Ministério Público para as providências cabíveis.
certo previsto nesta Lei, caberá ação mandamental, que Art. 222. Para instruir a petição inicial, o
se regerá pelas normas da lei do mandado de segurança. interessado poderá requerer às autoridades competentes
Art. 213. Na ação que tenha por objeto o as certidões e informações que julgar necessárias, que
cumprimento de obrigação de fazer ou não fazer, o juiz serão fornecidas no prazo de quinze dias.
concederá a tutela específica da obrigação ou Art. 223. O Ministério Público poderá instaurar, sob
determinará providências que assegurem o resultado sua presidência, inquérito civil, ou requisitar, de qualquer
prático equivalente ao do adimplemento. pessoa, organismo público ou particular, certidões,
§ 1º Sendo relevante o fundamento da demanda e informações, exames ou perícias, no prazo que assinalar,
havendo justificado receio de ineficácia do provimento o qual não poderá ser inferior a dez dias úteis.
final, é lícito ao juiz conceder a tutela liminarmente ou § 1º Se o órgão do Ministério Público, esgotadas
após justificação prévia, citando o réu. todas as diligências, se convencer da inexistência de
§ 2º O juiz poderá, na hipótese do parágrafo fundamento para a propositura da ação cível, promoverá
anterior ou na sentença, impor multa diária ao réu, o arquivamento dos autos do inquérito civil ou das peças
independentemente de pedido do autor, se for suficiente informativas, fazendo-o fundamentadamente.
ou compatível com a obrigação, fixando prazo razoável § 2º Os autos do inquérito civil ou as peças de
para o cumprimento do preceito. informação arquivados serão remetidos, sob pena de se
§ 3º A multa só será exigível do réu após o trânsito incorrer em falta grave, no prazo de três dias, ao
em julgado da sentença favorável ao autor, mas será Conselho Superior do Ministério Público.
devida desde o dia em que se houver configurado o § 3º Até que seja homologada ou rejeitada a
descumprimento. promoção de arquivamento, em sessão do Conselho
Art. 214. Os valores das multas reverterão ao fundo Superior do Ministério público, poderão as associações
gerido pelo Conselho dos Direitos da Criança e do legitimadas apresentar razões escritas ou documentos,
Adolescente do respectivo município. que serão juntados aos autos do inquérito ou anexados às
peças de informação.
§ 1º As multas não recolhidas até trinta dias após o
trânsito em julgado da decisão serão exigidas através de § 4º A promoção de arquivamento será submetida
execução promovida pelo Ministério Público, nos mesmos a exame e deliberação do Conselho Superior do
autos, facultada igual iniciativa aos demais legitimados. Ministério Público, conforme dispuser o seu regimento.
§ 2º Enquanto o fundo não for regulamentado, o § 5º Deixando o Conselho Superior de homologar a
dinheiro ficará depositado em estabelecimento oficial de promoção de arquivamento, designará, desde logo, outro
crédito, em conta com correção monetária. órgão do Ministério Público para o ajuizamento da ação.
Art. 215. O juiz poderá conferir efeito suspensivo Art. 224. Aplicam-se subsidiariamente, no que
aos recursos, para evitar dano irreparável à parte. couber, as disposições da Lei n.º 7.347, de 24 de julho de
1985 .
Art. 216. Transitada em julgado a sentença que
impuser condenação ao poder público, o juiz determinará Título VII
a remessa de peças à autoridade competente, para Dos Crimes e Das Infrações Administrativas
apuração da responsabilidade civil e administrativa do Capítulo I
agente a que se atribua a ação ou omissão.
Dos Crimes
Art. 217. Decorridos sessenta dias do trânsito em
Seção I
julgado da sentença condenatória sem que a associação
autora lhe promova a execução, deverá fazê-lo o Disposições Gerais
Ministério Público, facultada igual iniciativa aos demais Art. 225. Este Capítulo dispõe sobre crimes
legitimados. praticados contra a criança e o adolescente, por ação ou
Art. 218. O juiz condenará a associação autora a omissão, sem prejuízo do disposto na legislação penal.
pagar ao réu os honorários advocatícios arbitrados na Art. 226. Aplicam-se aos crimes definidos nesta Lei
conformidade do § 4º do art. 20 da Lei n.º 5.869, de 11 de as normas da Parte Geral do Código Penal e, quanto ao
janeiro de 1973 (Código de Processo Civil) , quando processo, as pertinentes ao Código de Processo Penal.
reconhecer que a pretensão é manifestamente infundada. Art. 227. Os crimes definidos nesta Lei são de ação
Parágrafo único. Em caso de litigância de má-fé, a pública incondicionada.
associação autora e os diretores responsáveis pela Art. 227-A Os efeitos da condenação prevista
propositura da ação serão solidariamente condenados ao no inciso I do caput do art. 92 do Decreto-Lei nº 2.848, de
décuplo das custas, sem prejuízo de responsabilidade por 7 de dezembro de 1940 (Código Penal), para os crimes
perdas e danos. previstos nesta Lei, praticados por servidores públicos
com abuso de autoridade, são condicionados à ocorrência
de reincidência. (Incluído pela Lei nº 13.869. de 2019)

www.editoradince.com - Acesse e veja se há novidades a respeito deste material – CUIDADO: cópia é crime.
50 LEGISLAÇÃO ESPECIAL E REGIMENTO INTERNO DAS UNIDADES DO SEAS
Parágrafo único. A perda do cargo, do mandato ou Parágrafo único. Incide nas mesmas penas quem
da função, nesse caso, independerá da pena aplicada na oferece ou efetiva a paga ou recompensa.
reincidência. (Incluído pela Lei nº 13.869. de 2019) Art. 239. Promover ou auxiliar a efetivação de ato
Seção II destinado ao envio de criança ou adolescente para o
Dos Crimes em Espécie exterior com inobservância das formalidades legais ou
com o fito de obter lucro:
Art. 228. Deixar o encarregado de serviço ou o
dirigente de estabelecimento de atenção à saúde de Pena - reclusão de quatro a seis anos, e multa.
gestante de manter registro das atividades desenvolvidas, Parágrafo único. Se há emprego de violência,
na forma e prazo referidos no art. 10 desta Lei, bem como grave ameaça ou fraude: (Incluído pela Lei nº 10.764, de
de fornecer à parturiente ou a seu responsável, por 12.11.2003)
ocasião da alta médica, declaração de nascimento, onde Pena - reclusão, de 6 (seis) a 8 (oito) anos, além da
constem as intercorrências do parto e do desenvolvimento pena correspondente à violência.
do neonato:
Art. 240. Produzir, reproduzir, dirigir, fotografar,
Pena - detenção de seis meses a dois anos. filmar ou registrar, por qualquer meio, cena de sexo
Parágrafo único. Se o crime é culposo: explícito ou pornográfica, envolvendo criança ou
Pena - detenção de dois a seis meses, ou multa. adolescente: (Redação dada pela Lei nº 11.829, de 2008)
Art. 229. Deixar o médico, enfermeiro ou dirigente Pena – reclusão, de 4 (quatro) a 8 (oito) anos, e
de estabelecimento de atenção à saúde de gestante de multa. (Redação dada pela Lei nº 11.829, de 2008)
o
identificar corretamente o neonato e a parturiente, por § 1 Incorre nas mesmas penas quem agencia,
ocasião do parto, bem como deixar de proceder aos facilita, recruta, coage, ou de qualquer modo intermedeia
exames referidos no art. 10 desta Lei: a participação de criança ou adolescente nas cenas
Pena - detenção de seis meses a dois anos. referidas no caput deste artigo, ou ainda quem com esses
contracena. (Redação dada pela Lei nº 11.829, de 2008)
Parágrafo único. Se o crime é culposo: o
§ 2 Aumenta-se a pena de 1/3 (um terço) se o
Pena - detenção de dois a seis meses, ou multa.
agente comete o crime: (Redação dada pela Lei nº
Art. 230. Privar a criança ou o adolescente de sua 11.829, de 2008)
liberdade, procedendo à sua apreensão sem estar em
I – no exercício de cargo ou função pública ou a
flagrante de ato infracional ou inexistindo ordem escrita da
pretexto de exercê-la; (Redação dada pela Lei nº 11.829,
autoridade judiciária competente:
de 2008)
Pena - detenção de seis meses a dois anos.
II – prevalecendo-se de relações domésticas, de
Parágrafo único. Incide na mesma pena aquele que coabitação ou de hospitalidade; ou (Redação dada pela
procede à apreensão sem observância das formalidades Lei nº 11.829, de 2008)
legais.
III – prevalecendo-se de relações de parentesco
Art. 231. Deixar a autoridade policial responsável consangüíneo ou afim até o terceiro grau, ou por adoção,
pela apreensão de criança ou adolescente de fazer de tutor, curador, preceptor, empregador da vítima ou de
imediata comunicação à autoridade judiciária competente quem, a qualquer outro título, tenha autoridade sobre ela,
e à família do apreendido ou à pessoa por ele indicada: ou com seu consentimento. (Incluído pela Lei nº 11.829,
Pena - detenção de seis meses a dois anos. de 2008)
Art. 232. Submeter criança ou adolescente sob sua Art. 241. Vender ou expor à venda fotografia, vídeo
autoridade, guarda ou vigilância a vexame ou a ou outro registro que contenha cena de sexo explícito ou
constrangimento: pornográfica envolvendo criança ou
Pena - detenção de seis meses a dois anos. adolescente: (Redação dada pela Lei nº 11.829, de 2008)
Art. 233. (Revogado pela Lei nº 9.455, de 7.4.1997: Pena – reclusão, de 4 (quatro) a 8 (oito) anos, e
Art. 234. Deixar a autoridade competente, sem multa. (Redação dada pela Lei nº 11.829, de 2008)
justa causa, de ordenar a imediata liberação de criança ou Art. 241-A. Oferecer, trocar, disponibilizar,
adolescente, tão logo tenha conhecimento da ilegalidade transmitir, distribuir, publicar ou divulgar por qualquer
da apreensão: meio, inclusive por meio de sistema de informática ou
Pena - detenção de seis meses a dois anos. telemático, fotografia, vídeo ou outro registro que
contenha cena de sexo explícito ou pornográfica
Art. 235. Descumprir, injustificadamente, prazo envolvendo criança ou adolescente: (Incluído pela Lei nº
fixado nesta Lei em benefício de adolescente privado de 11.829, de 2008)
liberdade:
Pena – reclusão, de 3 (três) a 6 (seis) anos, e
Pena - detenção de seis meses a dois anos. multa. (Incluído pela Lei nº 11.829, de 2008)
Art. 236. Impedir ou embaraçar a ação de o
§ 1 Nas mesmas penas incorre quem: (Incluído
autoridade judiciária, membro do Conselho Tutelar ou pela Lei nº 11.829, de 2008)
representante do Ministério Público no exercício de
função prevista nesta Lei: I – assegura os meios ou serviços para o
armazenamento das fotografias, cenas ou imagens de
Pena - detenção de seis meses a dois anos. que trata o caput deste artigo; (Incluído pela Lei nº
Art. 237. Subtrair criança ou adolescente ao poder 11.829, de 2008)
de quem o tem sob sua guarda em virtude de lei ou ordem II – assegura, por qualquer meio, o acesso por rede
judicial, com o fim de colocação em lar substituto: de computadores às fotografias, cenas ou imagens de
Pena - reclusão de dois a seis anos, e multa. que trata o caput deste artigo. (Incluído pela Lei nº
Art. 238. Prometer ou efetivar a entrega de filho ou 11.829, de 2008)
o
pupilo a terceiro, mediante paga ou recompensa: § 2 As condutas tipificadas nos incisos I e II do §
o
Pena - reclusão de um a quatro anos, e multa. 1 deste artigo são puníveis quando o responsável legal

www.editoradince.com - Acesse e veja se há novidades a respeito deste material – CUIDADO: cópia é crime.
LEGISLAÇÃO ESPECIAL E REGIMENTO INTERNO DAS UNIDADES DO SEAS 51
pela prestação do serviço, oficialmente notificado, deixa Art. 241-E. Para efeito dos crimes previstos nesta
de desabilitar o acesso ao conteúdo ilícito de que trata Lei, a expressão “cena de sexo explícito ou pornográfica”
o caput deste artigo. (Incluído pela Lei nº 11.829, de compreende qualquer situação que envolva criança ou
2008) adolescente em atividades sexuais explícitas, reais ou
Art. 241-B. Adquirir, possuir ou armazenar, por simuladas, ou exibição dos órgãos genitais de uma
qualquer meio, fotografia, vídeo ou outra forma de registro criança ou adolescente para fins primordialmente
que contenha cena de sexo explícito ou pornográfica sexuais. (Incluído pela Lei nº 11.829, de 2008)
envolvendo criança ou adolescente: (Incluído pela Lei nº Art. 242. Vender, fornecer ainda que gratuitamente
11.829, de 2008) ou entregar, de qualquer forma, a criança ou adolescente
Pena – reclusão, de 1 (um) a 4 (quatro) anos, e arma, munição ou explosivo:
multa. (Incluído pela Lei nº 11.829, de 2008) Pena - reclusão, de 3 (três) a 6 (seis)
o anos. (Redação dada pela Lei nº 10.764, de 12.11.2003)
§ 1 A pena é diminuída de 1 (um) a 2/3 (dois
terços) se de pequena quantidade o material a que se Art. 243. Vender, fornecer, servir, ministrar ou
refere o caput deste artigo. (Incluído pela Lei nº 11.829, entregar, ainda que gratuitamente, de qualquer forma, a
de 2008) criança ou a adolescente, bebida alcoólica ou, sem justa
o causa, outros produtos cujos componentes possam
§ 2 Não há crime se a posse ou o armazenamento
tem a finalidade de comunicar às autoridades causar dependência física ou psíquica: (Redação dada
competentes a ocorrência das condutas descritas nos pela Lei nº 13.106, de 2015)
arts. 240, 241, 241-A e 241-C desta Lei, quando a Pena - detenção de 2 (dois) a 4 (quatro) anos, e
comunicação for feita por: (Incluído pela Lei nº 11.829, de multa, se o fato não constitui crime mais grave. (Redação
2008) dada pela Lei nº 13.106, de 2015)
I – agente público no exercício de suas Art. 244. Vender, fornecer ainda que gratuitamente
funções; (Incluído pela Lei nº 11.829, de 2008) ou entregar, de qualquer forma, a criança ou adolescente
II – membro de entidade, legalmente constituída, fogos de estampido ou de artifício, exceto aqueles que,
que inclua, entre suas finalidades institucionais, o pelo seu reduzido potencial, sejam incapazes de provocar
recebimento, o processamento e o encaminhamento de qualquer dano físico em caso de utilização indevida:
notícia dos crimes referidos neste parágrafo; (Incluído Pena - detenção de seis meses a dois anos, e
pela Lei nº 11.829, de 2008) multa.
III – representante legal e funcionários Art. 244-A. Submeter criança ou adolescente, como
o
responsáveis de provedor de acesso ou serviço prestado tais definidos no caput do art. 2 desta Lei, à prostituição
por meio de rede de computadores, até o recebimento do ou à exploração sexual: (Incluído pela Lei nº 9.975, de
material relativo à notícia feita à autoridade policial, ao 23.6.2000)
Ministério Público ou ao Poder Judiciário. (Incluído pela Pena – reclusão de quatro a dez anos e multa,
Lei nº 11.829, de 2008) além da perda de bens e valores utilizados na prática
o o
§ 3 As pessoas referidas no § 2 deste artigo criminosa em favor do Fundo dos Direitos da Criança e do
deverão manter sob sigilo o material ilícito Adolescente da unidade da Federação (Estado ou Distrito
referido. (Incluído pela Lei nº 11.829, de 2008) Federal) em que foi cometido o crime, ressalvado o direito
Art. 241-C. Simular a participação de criança ou de terceiro de boa-fé. (Redação dada pela Lei nº 13.440,
adolescente em cena de sexo explícito ou pornográfica de 2017)
o
por meio de adulteração, montagem ou modificação de § 1 Incorrem nas mesmas penas o proprietário, o
fotografia, vídeo ou qualquer outra forma de gerente ou o responsável pelo local em que se verifique a
representação visual: (Incluído pela Lei nº 11.829, de submissão de criança ou adolescente às práticas
2008) referidas no caput deste artigo. (Incluído pela Lei nº 9.975,
Pena – reclusão, de 1 (um) a 3 (três) anos, e de 23.6.2000)
o
multa. (Incluído pela Lei nº 11.829, de 2008) § 2 Constitui efeito obrigatório da condenação a
Parágrafo único. Incorre nas mesmas penas quem cassação da licença de localização e de funcionamento
vende, expõe à venda, disponibiliza, distribui, publica ou do estabelecimento. (Incluído pela Lei nº 9.975, de
divulga por qualquer meio, adquire, possui ou armazena o 23.6.2000)
material produzido na forma do caput deste Art. 244-B. Corromper ou facilitar a corrupção de
artigo. (Incluído pela Lei nº 11.829, de 2008) menor de 18 (dezoito) anos, com ele praticando infração
Art. 241-D. Aliciar, assediar, instigar ou penal ou induzindo-o a praticá-la: (Incluído pela Lei nº
constranger, por qualquer meio de comunicação, criança, 12.015, de 2009)
com o fim de com ela praticar ato libidinoso: (Incluído pela Pena - reclusão, de 1 (um) a 4 (quatro)
Lei nº 11.829, de 2008) anos. (Incluído pela Lei nº 12.015, de 2009)
o
Pena – reclusão, de 1 (um) a 3 (três) anos, e § 1 Incorre nas penas previstas no caput deste
multa. (Incluído pela Lei nº 11.829, de 2008) artigo quem pratica as condutas ali tipificadas utilizando-
Parágrafo único. Nas mesmas penas incorre se de quaisquer meios eletrônicos, inclusive salas de
quem: (Incluído pela Lei nº 11.829, de 2008) bate-papo da internet. (Incluído pela Lei nº 12.015, de
2009)
I – facilita ou induz o acesso à criança de material o
contendo cena de sexo explícito ou pornográfica com o § 2 As penas previstas no caput deste artigo são
fim de com ela praticar ato libidinoso; (Incluído pela Lei nº aumentadas de um terço no caso de a infração cometida
o
11.829, de 2008) ou induzida estar incluída no rol do art. 1 da Lei
o
n 8.072, de 25 de julho de 1990 . (Incluído pela Lei nº
II – pratica as condutas descritas no caput deste
12.015, de 2009)
artigo com o fim de induzir criança a se exibir de forma
pornográfica ou sexualmente explícita. (Incluído pela Lei Capítulo II
nº 11.829, de 2008) Das Infrações Administrativas

www.editoradince.com - Acesse e veja se há novidades a respeito deste material – CUIDADO: cópia é crime.
52 LEGISLAÇÃO ESPECIAL E REGIMENTO INTERNO DAS UNIDADES DO SEAS
Art. 245. Deixar o médico, professor ou destacada sobre a natureza da diversão ou espetáculo e
responsável por estabelecimento de atenção à saúde e de a faixa etária especificada no certificado de classificação:
ensino fundamental, pré-escola ou creche, de comunicar Pena - multa de três a vinte salários de referência,
à autoridade competente os casos de que tenha aplicando-se o dobro em caso de reincidência.
conhecimento, envolvendo suspeita ou confirmação de
Art. 253. Anunciar peças teatrais, filmes ou
maus-tratos contra criança ou adolescente:
quaisquer representações ou espetáculos, sem indicar os
Pena - multa de três a vinte salários de referência, limites de idade a que não se recomendem:
aplicando-se o dobro em caso de reincidência.
Pena - multa de três a vinte salários de referência,
Art. 246. Impedir o responsável ou funcionário de duplicada em caso de reincidência, aplicável,
entidade de atendimento o exercício dos direitos separadamente, à casa de espetáculo e aos órgãos de
constantes nos incisos II, III, VII, VIII e XI do art. 124 desta divulgação ou publicidade.
Lei:
Art. 254. Transmitir, através de rádio ou televisão,
Pena - multa de três a vinte salários de referência, espetáculo em horário diverso do autorizado ou sem aviso
aplicando-se o dobro em caso de reincidência. de sua classificação: (Expressão declarada
Art. 247. Divulgar, total ou parcialmente, sem inconstitucional pela ADI 2.404).
autorização devida, por qualquer meio de comunicação, Pena - multa de vinte a cem salários de referência;
nome, ato ou documento de procedimento policial, duplicada em caso de reincidência a autoridade judiciária
administrativo ou judicial relativo a criança ou adolescente poderá determinar a suspensão da programação da
a que se atribua ato infracional: emissora por até dois dias.
Pena - multa de três a vinte salários de referência, Art. 255. Exibir filme, trailer, peça, amostra ou
aplicando-se o dobro em caso de reincidência. congênere classificado pelo órgão competente como
§ 1º Incorre na mesma pena quem exibe, total ou inadequado às crianças ou adolescentes admitidos ao
parcialmente, fotografia de criança ou adolescente espetáculo:
envolvido em ato infracional, ou qualquer ilustração que Pena - multa de vinte a cem salários de referência;
lhe diga respeito ou se refira a atos que lhe sejam na reincidência, a autoridade poderá determinar a
atribuídos, de forma a permitir sua identificação, direta ou suspensão do espetáculo ou o fechamento do
indiretamente. estabelecimento por até quinze dias.
§ 2º Se o fato for praticado por órgão de imprensa Art. 256. Vender ou locar a criança ou adolescente
ou emissora de rádio ou televisão, além da pena prevista fita de programação em vídeo, em desacordo com a
neste artigo, a autoridade judiciária poderá determinar a classificação atribuída pelo órgão competente:
apreensão da publicação ou a suspensão da
Pena - multa de três a vinte salários de referência;
programação da emissora até por dois dias, bem como da
em caso de reincidência, a autoridade judiciária poderá
publicação do periódico até por dois números. (Expressão
determinar o fechamento do estabelecimento por até
declarada inconstitucional pela ADIN 869).
quinze dias.
Art. 248. (Revogado pela Lei nº 13.431, de 2017) ( )
Art. 257. Descumprir obrigação constante dos arts.
Art. 249. Descumprir, dolosa ou culposamente, os 78 e 79 desta Lei:
deveres inerentes ao pátrio poder poder familiar ou
Pena - multa de três a vinte salários de referência,
decorrente de tutela ou guarda, bem assim determinação
duplicando-se a pena em caso de reincidência, sem
da autoridade judiciária ou Conselho Tutelar: (Expressão
prejuízo de apreensão da revista ou publicação.
substituída pela Lei nº 12.010, de 2009)
Art. 258. Deixar o responsável pelo
Pena - multa de três a vinte salários de referência,
estabelecimento ou o empresário de observar o que
aplicando-se o dobro em caso de reincidência.
dispõe esta Lei sobre o acesso de criança ou adolescente
Art. 250. Hospedar criança ou adolescente aos locais de diversão, ou sobre sua participação no
desacompanhado dos pais ou responsável, ou sem espetáculo:
autorização escrita desses ou da autoridade judiciária, em
Pena - multa de três a vinte salários de referência;
hotel, pensão, motel ou congênere: (Redação dada pela
em caso de reincidência, a autoridade judiciária poderá
Lei nº 12.038, de 2009).
determinar o fechamento do estabelecimento por até
Pena – multa. (Redação dada pela Lei nº 12.038, quinze dias.
de 2009).
Art. 258-A. Deixar a autoridade competente de
§ 1 º Em caso de reincidência, sem prejuízo da providenciar a instalação e operacionalização dos
pena de multa, a autoridade judiciária poderá determinar o cadastros previstos no art. 50 e no § 11 do art. 101 desta
fechamento do estabelecimento por até 15 (quinze) Lei: (Incluído pela Lei nº 12.010, de 2009)
dias. (Incluído pela Lei nº 12.038, de 2009).
Pena - multa de R$ 1.000,00 (mil reais) a R$
§ 2 º Se comprovada a reincidência em período 3.000,00 (três mil reais). (Incluído pela Lei nº 12.010, de
inferior a 30 (trinta) dias, o estabelecimento será 2009)
definitivamente fechado e terá sua licença
Parágrafo único. Incorre nas mesmas penas a
cassada. (Incluído pela Lei nº 12.038, de 2009).
autoridade que deixa de efetuar o cadastramento de
Art. 251. Transportar criança ou adolescente, por crianças e de adolescentes em condições de serem
qualquer meio, com inobservância do disposto nos arts. adotadas, de pessoas ou casais habilitados à adoção e de
83, 84 e 85 desta Lei: crianças e adolescentes em regime de acolhimento
Pena - multa de três a vinte salários de referência, institucional ou familiar. (Incluído pela Lei nº 12.010, de
aplicando-se o dobro em caso de reincidência. 2009)
Art. 252. Deixar o responsável por diversão ou Art. 258-B. Deixar o médico, enfermeiro ou
espetáculo público de afixar, em lugar visível e de fácil dirigente de estabelecimento de atenção à saúde de
acesso, à entrada do local de exibição, informação gestante de efetuar imediato encaminhamento à
autoridade judiciária de caso de que tenha conhecimento

www.editoradince.com - Acesse e veja se há novidades a respeito deste material – CUIDADO: cópia é crime.
LEGISLAÇÃO ESPECIAL E REGIMENTO INTERNO DAS UNIDADES DO SEAS 53
de mãe ou gestante interessada em entregar seu filho § 3º O Departamento da Receita Federal, do
para adoção: (Incluído pela Lei nº 12.010, de 2009) Ministério da Economia, Fazenda e Planejamento,
Pena - multa de R$ 1.000,00 (mil reais) a R$ regulamentará a comprovação das doações feitas aos
3.000,00 (três mil reais). (Incluído pela Lei nº 12.010, de fundos, nos termos deste artigo. (Incluído pela Lei nº
2009) 8.242, de 12.10.1991)
Parágrafo único. Incorre na mesma pena o § 4º O Ministério Público determinará em cada
funcionário de programa oficial ou comunitário destinado à comarca a forma de fiscalização da aplicação, pelo Fundo
garantia do direito à convivência familiar que deixa de Municipal dos Direitos da Criança e do Adolescente, dos
efetuar a comunicação referida no caput deste incentivos fiscais referidos neste artigo. (Incluído pela Lei
artigo. (Incluído pela Lei nº 12.010, de 2009) nº 8.242, de 12.10.1991)
o o o
Art. 258-C. Descumprir a proibição estabelecida no § 5 Observado o disposto no § 4 do art. 3 da
o
inciso II do art. 81: (Redação dada pela Lei nº 13.106, de Lei n 9.249, de 26 de dezembro de 1995 , a dedução de
2015) que trata o inciso I do caput : (Redação dada pela Lei nº
12.594, de 2012)
Pena - multa de R$ 3.000,00 (três mil reais) a R$
10.000,00 (dez mil reais); (Redação dada pela Lei nº I - será considerada isoladamente, não se
13.106, de 2015) submetendo a limite em conjunto com outras deduções do
imposto; e (Incluído pela Lei nº 12.594, de 2012)
Medida Administrativa - interdição do
estabelecimento comercial até o recolhimento da multa II - não poderá ser computada como despesa
aplicada. (Redação dada pela Lei nº 13.106, de 2015) operacional na apuração do lucro real. (Incluído pela Lei
nº 12.594, de 2012)
Disposições Finais e Transitórias
Art. 260-A. A partir do exercício de 2010, ano-
Art. 259. A União, no prazo de noventa dias
calendário de 2009, a pessoa física poderá optar pela
contados da publicação deste Estatuto, elaborará projeto
doação de que trata o inciso II do caput do art. 260
de lei dispondo sobre a criação ou adaptação de seus
diretamente em sua Declaração de Ajuste Anual. (Incluído
órgãos às diretrizes da política de atendimento fixadas no
pela Lei nº 12.594, de 2012)
art. 88 e ao que estabelece o Título V do Livro II. o
§ 1 A doação de que trata o caput poderá ser
Parágrafo único. Compete aos estados e
deduzida até os seguintes percentuais aplicados sobre o
municípios promoverem a adaptação de seus órgãos e
imposto apurado na declaração: (Incluído pela Lei nº
programas às diretrizes e princípios estabelecidos nesta
12.594, de 2012)
Lei.
I - (VETADO); (Incluído pela Lei nº 12.594, de
Art. 260. Os contribuintes poderão efetuar doações
2012)
aos Fundos dos Direitos da Criança e do Adolescente
nacional, distrital, estaduais ou municipais, devidamente II - (VETADO); (Incluído pela Lei nº 12.594, de
comprovadas, sendo essas integralmente deduzidas do 2012)
imposto de renda, obedecidos os seguintes III - 3% (três por cento) a partir do exercício de
limites: (Redação dada pela Lei nº 12.594, de 2012) 2012. (Incluído pela Lei nº 12.594, de 2012)
o
I - 1% (um por cento) do imposto sobre a renda § 2 A dedução de que trata o caput : (Incluído
devido apurado pelas pessoas jurídicas tributadas com pela Lei nº 12.594, de 2012)
base no lucro real; e (Redação dada pela Lei nº 12.594, I - está sujeita ao limite de 6% (seis por cento) do
de 2012) imposto sobre a renda apurado na declaração de que
II - 6% (seis por cento) do imposto sobre a renda trata o inciso II do caput do art. 260; (Incluído pela Lei nº
apurado pelas pessoas físicas na Declaração de Ajuste 12.594, de 2012)
o
Anual, observado o disposto no art. 22 da Lei n 9.532, de II - não se aplica à pessoa física que: (Incluído pela
10 de dezembro de 1997 . (Redação dada pela Lei nº Lei nº 12.594, de 2012)
12.594, de 2012)
a) utilizar o desconto simplificado; (Incluído pela Lei
§ 1º - (Revogado pela Lei nº 9.532, de nº 12.594, de 2012)
1997) (Produção de efeito)
o
b) apresentar declaração em formulário;
§ 1 -A. Na definição das prioridades a serem ou (Incluído pela Lei nº 12.594, de 2012)
atendidas com os recursos captados pelos fundos
c) entregar a declaração fora do prazo; (Incluído
nacional, estaduais e municipais dos direitos da criança e
pela Lei nº 12.594, de 2012)
do adolescente, serão consideradas as disposições do
Plano Nacional de Promoção, Proteção e Defesa do III - só se aplica às doações em espécie; e (Incluído
Direito de Crianças e Adolescentes à Convivência pela Lei nº 12.594, de 2012) (
Familiar e Comunitária e as do Plano Nacional pela IV - não exclui ou reduz outros benefícios ou
Primeira Infância. (Redação dada dada pela Lei nº deduções em vigor. (Incluído pela Lei nº 12.594, de 2012)
13.257, de 2016) o
§ 3 O pagamento da doação deve ser efetuado
o
§ 2 Os conselhos nacional, estaduais e municipais até a data de vencimento da primeira quota ou quota
dos direitos da criança e do adolescente fixarão critérios única do imposto, observadas instruções específicas da
de utilização, por meio de planos de aplicação, das Secretaria da Receita Federal do Brasil. (Incluído pela Lei
dotações subsidiadas e demais receitas, aplicando nº 12.594, de 2012)
necessariamente percentual para incentivo ao o
§ 4 O não pagamento da doação no prazo
o
acolhimento, sob a forma de guarda, de crianças e estabelecido no § 3 implica a glosa definitiva desta
adolescentes e para programas de atenção integral à parcela de dedução, ficando a pessoa física obrigada ao
primeira infância em áreas de maior carência recolhimento da diferença de imposto devido apurado na
socioeconômica e em situações de calamidade. (Redação Declaração de Ajuste Anual com os acréscimos legais
dada dada pela Lei nº 13.257, de 2016) previstos na legislação. (Incluído pela Lei nº 12.594, de
2012)

www.editoradince.com - Acesse e veja se há novidades a respeito deste material – CUIDADO: cópia é crime.
54 LEGISLAÇÃO ESPECIAL E REGIMENTO INTERNO DAS UNIDADES DO SEAS
o
§ 5 A pessoa física poderá deduzir do imposto III - considerar como valor dos bens
apurado na Declaração de Ajuste Anual as doações doados: (Incluído pela Lei nº 12.594, de 2012)
feitas, no respectivo ano-calendário, aos fundos a) para as pessoas físicas, o valor constante da
controlados pelos Conselhos dos Direitos da Criança e do última declaração do imposto de renda, desde que não
Adolescente municipais, distrital, estaduais e nacional exceda o valor de mercado; (Incluído pela Lei nº 12.594,
concomitantemente com a opção de que trata o caput , de 2012)
respeitado o limite previsto no inciso II do art.
b) para as pessoas jurídicas, o valor contábil dos
260. (Incluído pela Lei nº 12.594, de 2012)
bens. (Incluído pela Lei nº 12.594, de 2012)
Art. 260-B. A doação de que trata o inciso I do art.
Parágrafo único. O preço obtido em caso de leilão
260 poderá ser deduzida: (Incluído pela Lei nº 12.594, de
não será considerado na determinação do valor dos bens
2012)
doados, exceto se o leilão for determinado por autoridade
I - do imposto devido no trimestre, para as pessoas judiciária. (Incluído pela Lei nº 12.594, de 2012)
jurídicas que apuram o imposto trimestralmente;
Art. 260-F. Os documentos a que se referem os
e (Incluído pela Lei nº 12.594, de 2012)
arts. 260-D e 260-E devem ser mantidos pelo contribuinte
II - do imposto devido mensalmente e no ajuste por um prazo de 5 (cinco) anos para fins de comprovação
anual, para as pessoas jurídicas que apuram o imposto da dedução perante a Receita Federal do Brasil. (Incluído
anualmente. (Incluído pela Lei nº 12.594, de 2012) pela Lei nº 12.594, de 2012)
Parágrafo único. A doação deverá ser efetuada Art. 260-G. Os órgãos responsáveis pela
dentro do período a que se refere a apuração do administração das contas dos Fundos dos Direitos da
imposto. (Incluído pela Lei nº 12.594, de 2012) Criança e do Adolescente nacional, estaduais, distrital e
Art. 260-C. As doações de que trata o art. 260 municipais devem: (Incluído pela Lei nº 12.594, de 2012)
desta Lei podem ser efetuadas em espécie ou em I - manter conta bancária específica destinada
bens. (Incluído pela Lei nº 12.594, de 2012) exclusivamente a gerir os recursos do Fundo; (Incluído
Parágrafo único. As doações efetuadas em espécie pela Lei nº 12.594, de 2012)
devem ser depositadas em conta específica, em II - manter controle das doações recebidas;
instituição financeira pública, vinculadas aos respectivos e (Incluído pela Lei nº 12.594, de 2012)
fundos de que trata o art. 260. (Incluído pela Lei nº
III - informar anualmente à Secretaria da Receita
12.594, de 2012)
Federal do Brasil as doações recebidas mês a mês,
Art. 260-D. Os órgãos responsáveis pela identificando os seguintes dados por doador: (Incluído
administração das contas dos Fundos dos Direitos da pela Lei nº 12.594, de 2012)
Criança e do Adolescente nacional, estaduais, distrital e
a) nome, CNPJ ou CPF; (Incluído pela Lei nº
municipais devem emitir recibo em favor do doador,
12.594, de 2012)
assinado por pessoa competente e pelo presidente do
Conselho correspondente, especificando: (Incluído pela b) valor doado, especificando se a doação foi em
Lei nº 12.594, de 2012) espécie ou em bens. (Incluído pela Lei nº 12.594, de
2012)
I - número de ordem; (Incluído pela Lei nº 12.594,
de 2012) Art. 260-H. Em caso de descumprimento das
obrigações previstas no art. 260-G, a Secretaria da
II - nome, Cadastro Nacional da Pessoa Jurídica
Receita Federal do Brasil dará conhecimento do fato ao
(CNPJ) e endereço do emitente; (Incluído pela Lei nº
Ministério Público. (Incluído pela Lei nº 12.594, de 2012)
12.594, de 2012)
Art. 260-I. Os Conselhos dos Direitos da Criança e
III - nome, CNPJ ou Cadastro de Pessoas Físicas
do Adolescente nacional, estaduais, distrital e municipais
(CPF) do doador; (Incluído pela Lei nº 12.594, de 2012)
divulgarão amplamente à comunidade: (Incluído pela Lei
IV - data da doação e valor efetivamente recebido; nº 12.594, de 2012)
e (Incluído pela Lei nº 12.594, de 2012)
I - o calendário de suas reuniões; (Incluído pela Lei
V - ano-calendário a que se refere a nº 12.594, de 2012)
doação. (Incluído pela Lei nº 12.594, de 2012)
o
II - as ações prioritárias para aplicação das políticas
§ 1 O comprovante de que trata o caput deste de atendimento à criança e ao adolescente; (Incluído pela
artigo pode ser emitido anualmente, desde que discrimine Lei nº 12.594, de 2012)
os valores doados mês a mês. (Incluído pela Lei nº
III - os requisitos para a apresentação de projetos a
12.594, de 2012)
o
serem beneficiados com recursos dos Fundos dos
§ 2 No caso de doação em bens, o comprovante Direitos da Criança e do Adolescente nacional, estaduais,
deve conter a identificação dos bens, mediante descrição distrital ou municipais; (Incluído pela Lei nº 12.594, de
em campo próprio ou em relação anexa ao comprovante, 2012)
informando também se houve avaliação, o nome, CPF ou
IV - a relação dos projetos aprovados em cada ano-
CNPJ e endereço dos avaliadores. (Incluído pela Lei nº
calendário e o valor dos recursos previstos para
12.594, de 2012)
implementação das ações, por projeto; (Incluído pela Lei
Art. 260-E. Na hipótese da doação em bens, o nº 12.594, de 2012)
doador deverá: (Incluído pela Lei nº 12.594, de 2012)
V - o total dos recursos recebidos e a respectiva
I - comprovar a propriedade dos bens, mediante destinação, por projeto atendido, inclusive com
documentação hábil; (Incluído pela Lei nº 12.594, de cadastramento na base de dados do Sistema de
2012) Informações sobre a Infância e a Adolescência;
II - baixar os bens doados na declaração de bens e e (Incluído pela Lei nº 12.594, de 2012)
direitos, quando se tratar de pessoa física, e na VI - a avaliação dos resultados dos projetos
escrituração, no caso de pessoa jurídica; e (Incluído pela beneficiados com recursos dos Fundos dos Direitos da
Lei nº 12.594, de 2012) Criança e do Adolescente nacional, estaduais, distrital e
municipais. (Incluído pela Lei nº 12.594, de 2012)

www.editoradince.com - Acesse e veja se há novidades a respeito deste material – CUIDADO: cópia é crime.
LEGISLAÇÃO ESPECIAL E REGIMENTO INTERNO DAS UNIDADES DO SEAS 55
Art. 260-J. O Ministério Público determinará, em Pena - reclusão de três a nove anos.»
cada Comarca, a forma de fiscalização da aplicação dos Art. 264. O art. 102 da Lei n.º 6.015, de 31 de
incentivos fiscais referidos no art. 260 desta Lei. (Incluído dezembro de 1973 , fica acrescido do seguinte item:
pela Lei nº 12.594, de 2012)
"Art. 102 .........................
Parágrafo único. O descumprimento do disposto
6º) a perda e a suspensão do pátrio poder. "
nos arts. 260-G e 260-I sujeitará os infratores a responder
por ação judicial proposta pelo Ministério Público, que Art. 265. A Imprensa Nacional e demais gráficas da
poderá atuar de ofício, a requerimento ou representação União, da administração direta ou indireta, inclusive
de qualquer cidadão. (Incluído pela Lei nº 12.594, de fundações instituídas e mantidas pelo poder público
2012) federal promoverão edição popular do texto integral deste
Estatuto, que será posto à disposição das escolas e das
Art. 260-K. A Secretaria de Direitos Humanos da
entidades de atendimento e de defesa dos direitos da
Presidência da República (SDH/PR) encaminhará à
criança e do adolescente.
Secretaria da Receita Federal do Brasil, até 31 de outubro
de cada ano, arquivo eletrônico contendo a relação Art. 265-A. O poder público fará periodicamente
atualizada dos Fundos dos Direitos da Criança e do ampla divulgação dos direitos da criança e do adolescente
Adolescente nacional, distrital, estaduais e municipais, nos meios de comunicação social. (Incluído pela Lei nº
com a indicação dos respectivos números de inscrição no 13.257, de 2016)
CNPJ e das contas bancárias específicas mantidas em Parágrafo único. A divulgação a que se refere
instituições financeiras públicas, destinadas o caput será veiculada em linguagem clara,
exclusivamente a gerir os recursos dos Fundos. (Incluído compreensível e adequada a crianças e adolescentes,
pela Lei nº 12.594, de 2012) especialmente às crianças com idade inferior a 6 (seis)
Art. 260-L. A Secretaria da Receita Federal do anos. (Incluído pela Lei nº 13.257, de 2016)
Brasil expedirá as instruções necessárias à aplicação do Art. 266. Esta Lei entra em vigor noventa dias após
disposto nos arts. 260 a 260-K. (Incluído pela Lei nº sua publicação.
12.594, de 2012) Parágrafo único. Durante o período de vacância
Art. 261. A falta dos conselhos municipais dos deverão ser promovidas atividades e campanhas de
direitos da criança e do adolescente, os registros, divulgação e esclarecimentos acerca do disposto nesta
inscrições e alterações a que se referem os arts. 90, Lei.
parágrafo único, e 91 desta Lei serão efetuados perante a Art. 267. Revogam-se as Leis n.º 4.513, de 1964,
autoridade judiciária da comarca a que pertencer a e 6.697, de 10 de outubro de 1979 (Código de Menores),
entidade. e as demais disposições em contrário.
Parágrafo único. A União fica autorizada a repassar Brasília, 13 de julho de 1990; 169º da Independência e
aos estados e municípios, e os estados aos municípios, 102º da República.
os recursos referentes aos programas e atividades FERNANDO COLLOR
previstos nesta Lei, tão logo estejam criados os conselhos Bernardo Cabral
dos direitos da criança e do adolescente nos seus Carlos Chiarelli
respectivos níveis. Antônio Magri
Margarida Procópio
Art. 262. Enquanto não instalados os Conselhos
Tutelares, as atribuições a eles conferidas serão
exercidas pela autoridade judiciária. QUESTÕES DE CONCURSOS
Art. 263. O Decreto-Lei n.º 2.848, de 7 de 01. (OMNI - 2021 - Prefeitura de Salesópolis - SP -
dezembro de 1940 (Código Penal), passa a vigorar com Assistente Social - Área Psicologia dos Recursos
as seguintes alterações: Humanos) Em relação ao exposto no Estatuto da
1) Art. 121 ........... Criança e do Adolescente, assinale a alternativa
§ 4º No homicídio culposo, a pena é aumentada de um correta:
terço, se o crime resulta de inobservância de regra técnica A A permanência da criança e do adolescente em
de profissão, arte ou ofício, ou se o agente deixa de programa de acolhimento institucional não se
prestar imediato socorro à vítima, não procura diminuir as prolongará por mais de 24 (vinte e quatro meses),
conseqüências do seu ato, ou foge para evitar prisão em salvo comprovada necessidade que atenda ao seu
flagrante. Sendo doloso o homicídio, a pena é aumentada superior interesse, devidamente fundamentada pela
de um terço, se o crime é praticado contra pessoa menor autoridade judiciária.
de catorze anos. B Toda criança ou adolescente que estiver inserido em
2) Art. 129 .......................... programa de acolhimento familiar ou institucional terá
§ 7º Aumenta-se a pena de um terço, se ocorrer qualquer sua situação reavaliada, no máximo, a cada 6 (seis)
das hipóteses do art. 121, § 4º. meses, devendo a autoridade judiciária competente,
§ 8º Aplica-se à lesão culposa o disposto no § 5º do art. com base em relatório elaborado por equipe
121. interprofissional ou multidisciplinar, decidir de forma
fundamentada pela possibilidade de reintegração
3) Art. 136..................... familiar ou pela colocação em família substituta.
§ 3º Aumenta-se a pena de um terço, se o crime é C Os dirigentes de entidades que desenvolvem
praticado contra pessoa menor de catorze anos. programas de acolhimento familiar ou institucional
4) Art. 213 .................. remeterão à autoridade judiciária, no máximo a cada 6
Parágrafo único. Se a ofendida é menor de catorze anos: (seis) meses, relatório circunstanciado acerca da
Pena - reclusão de quatro a dez anos. situação de cada criança ou adolescente acolhido e
sua família.
5) Art. 214....................
D A prestação de serviços comunitários consiste na
Parágrafo único. Se o ofendido é menor de catorze anos:
realização de tarefas gratuitas de interesse geral, por

www.editoradince.com - Acesse e veja se há novidades a respeito deste material – CUIDADO: cópia é crime.
56 LEGISLAÇÃO ESPECIAL E REGIMENTO INTERNO DAS UNIDADES DO SEAS
período não excedente a doze meses, junto a I. A remissão não implica necessariamente o
entidades assistenciais, hospitais, escolas e outros reconhecimento ou comprovação da responsabilidade,
estabelecimentos congêneres, bem como em nem prevalece para efeito de antecedentes, podendo
programas comunitários ou governamentais. incluir eventualmente a aplicação de qualquer das
medidas previstas em lei, exceto a colocação em
regime de semiliberdade e a internação.
02. (CETREDE - 2021 - Prefeitura de Frecheirinha - CE -
Assistente Social) De acordo com o art. 126 do II. Verificada a hipótese de maus-tratos, opressão ou
Estatuto da Criança e do Adolescente, Lei nº 8069/90: abuso sexual impostos pelos pais ou responsável, a
“Antes de iniciado o procedimento judicial para autoridade judiciária poderá determinar, como medida
apuração de ato infracional, o representante do cautelar, o afastamento do agressor da moradia
Ministério Público poderá conceder, como forma de comum.
exclusão do processo, atendendo às circunstâncias e III. O Conselho Tutelar é órgão permanente e autônomo,
consequências do fato, ao contexto social, bem como não jurisdicional, encarregado pela sociedade de zela
à personalidade do adolescente e sua maior ou menor pelo cumprimento dos direitos da criança e do
participação no ato infracional”. A sentença se refere adolescente. IV. São impedidos de servir no mesmo
ao(à) Conselho marido e mulher, ascendentes e
A perdão. descendentes, sogro e genro ou nora, irmãos,
cunhados, durante o cunhadio, tio e sobrinho,
B remissão.
padrasto ou madrasta e enteado.
C advertência em caráter reservado.
A sequência correta é:
D advertência em caráter ostensivo.
A Apenas as assertivas I e IV estão corretas.
E repreensão verbal.
B As assertivas I, II e IV estão corretas.
C As alternativas I, II, III e IV estão corretas.
03. (VUNESP - 2020 - Prefeitura de Cananéia - SP -
D As alternativas I, II, III e IV estão incorretas.
Coordenador da Casa da Criança e do Adolescente)
De acordo com art. 157 do ECA, havendo motivo
grave, poderá a autoridade judiciária, ouvido o 06. (AOCP - 2020 - Prefeitura de Novo Hamburgo - RS -
Ministério Público, decretar a suspensão do poder Advogado do CREAS/SUAS) Assinale a alternativa
familiar. De acordo com o parágrafo primeiro, recebida correta acerca do Estatuto da Criança e do
a petição inicial, a autoridade judiciária determinará a Adolescente (ECA).
realização de A Toda criança ou adolescente que estiver inserido em
A consentimento dos titulares do poder familiar, precedido programa de acolhimento familiar ou institucional terá
de orientações e esclarecimentos prestados pela sua situação reavaliada, no máximo, a cada 06 (seis)
equipe interprofissional da Justiça da Infância e da meses, devendo a autoridade judiciária competente,
Juventude. com base em relatório elaborado por equipe
B prevenção ao rompimento dos vínculos familiares, interprofissional ou multidisciplinar, decidir de forma
qualificação do atendimento dos Serviços de fundamentada pela possibilidade de reintegração
Acolhimento e investimento para o retorno ao convívio familiar ou pela colocação em família substituta.
com a família de origem. B A permanência da criança e do adolescente em
C avaliação exclusiva dos profissionais do serviço de programa de acolhimento institucional não se
acolhimento, com levantamento de atos contrários à prolongará por mais de 18 (dezoito meses), salvo
moral e aos bons costumes, mediante comprovação comprovada necessidade que atenda ao seu superior
destes atos. interesse, devidamente fundamentada pela autoridade
judiciária.
D rigorosa avaliação dos profissionais do Poder Judiciário,
visto que a destituição do poder familiar ocorre C Será garantida a convivência da criança e do
exclusivamente por vulnerabilidades econômicas. adolescente com a mãe ou o pai privado de liberdade,
por meio de visitas periódicas promovidas pelo
E estudo social ou perícia por equipe interprofissional ou
responsável ou, nas hipóteses de acolhimento
multidisciplinar para comprovar a presença de uma
institucional, pela entidade responsável, desde que
das causas de suspensão ou destituição do poder
haja autorização judicial.
familiar.
D A busca à família extensa respeitará o prazo máximo de
180 (cento e oitenta) dias, prorrogável por igual
04. (IBGP - 2021 - Prefeitura de Itabira - MG - Professor período.
de Séries Finais do Ensino Fundamental - Educação
E Os detentores da guarda possuem o prazo de 30 (trinta)
Física) De acordo com o Estatuto da Criança e do
dias para propor a ação de adoção, contado do dia
Adolescente (Lei nº 8.069/1990), a idade que se deve
seguinte à data do término do estágio de convivência.
considerar um indivíduo como criança e adolescente
respectivamente DEVE ser:
A Doze anos incompletos e entre doze e dezoito anos. Gabarito: 01/C; 02/B; 03/E; 04/A; 05/C; 06/B
B Onze anos incompletos e entre onze e dezoito anos.
C Treze anos incompletos e entre treze e dezoito anos.
D Quinze anos incompletos e entre quinze e dezoito anos. LEI Nº 9.455/1997 (CRIMES DE
TORTURA)
05. (IESES - 2021 - Prefeitura de Palhoça - SC -
Fonoaudiólogo) Leia o apresentado abaixo e Define os crimes de tortura e dá outras
identifique a sequência correta de acordo com o providências.
Estatuto da Criança e do Adolescente: O PRESIDENTE DA REPÚBLICA,

www.editoradince.com - Acesse e veja se há novidades a respeito deste material – CUIDADO: cópia é crime.
LEGISLAÇÃO ESPECIAL E REGIMENTO INTERNO DAS UNIDADES DO SEAS 57
Faço saber que o Congresso Nacional decreta e eu violência ou grave ameaça, causando-lhe sofrimento
sanciono a seguinte Lei: físico ou mental, com o fim de obter informação,
Art. 1º Constitui crime de tortura: declaração ou confissão da vítima ou de terceira
pessoa.
Comentário:
b) Segundo crime: tortura-crime. Constranger
Definição:
alguém com emprego de violência ou grave ameaça,
O artigo 1º da Convenção contra a Tortura e causando-lhe sofrimento físico ou mental, para
outros Tratamentos ou Penas Cruéis, Desumanos ou provocar ação ou omissão de natureza criminosa.
Degradantes, adotada pela Organização das Nações
c) Terceiro crime: tortura-racismo. Constranger
Unidas em 28 de setembro de 1984, define tortura
alguém com emprego de violência ou grave ameaça,
como “qualquer ato pelo qual dores ou sofrimentos
causando-lhe sofrimento físico ou mental, em razão de
agudos, físicos ou mentais, são infligidos
discriminação racial ou religiosa.
intencionalmente a uma pessoa a fim de obter, dela
ou de uma terceira pessoa, informações ou Objeto (bem) jurídico protegido: é a integridade
confissões; de castiga-la por ato que ela ou uma corporal e a saúde física e psicológica das pessoas. No
terceira pessoa tenha cometido ou seja suspeita de caso de o crime ser praticado por agente público,
ter cometido; de intimidar ou coagir esta pessoa ou tutela-se também, secundariamente, a Administração
outras pessoas; ou por qualquer motivo baseado em Pública, traída em seus objetivos de legalidade,
discriminação de qualquer natureza; quando tais impessoalidade, moralidade e eficiência.
dores ou sofrimentos são infligidos por um Sujeito ativo: pode ser qualquer pessoa (crime
funcionário público ou outra pessoa no exercício de comum).
funções públicas, ou por sua instigação, ou com o DICA DE CONCURSO: O crime de tortura é
seu conhecimento ou aquiescência”. crime comum, podendo ser praticado por
Fundamento Constitucional: Reza o artigo 5º, qualquer pessoa, não sendo próprio de agente
III da Carta: “Ninguém será submetido a tortura nem público, circunstância esta que, acaso
a tratamento desumano ou degradante”. demonstrada, determinará a incidência de
E no inciso XLIII do mesmo artigo: “A lei aumento da pena. (Agente/PC/ES/
considerará crimes inafiançáveis e insuscetíveis de CESPE/2009).
graça ou anistia a prática da tortura, o tráfico ilícito Sujeito passivo: a pessoa que sofre a tortura.
de entorpecentes e drogas afins, o terrorismo e os
Núcleo do tipo: o verbo “constranger” (forçar,
delitos como crimes hediondos, por eles
coagir, compelir).
respondendo os mandantes, os executores e os que,
podendo evitá-los, se omitirem”. Objeto material: é a pessoa que sofre a tortura,
ou seja, a pessoa sobre a qual recai a ação do agente.
A Lei 8.072/90 estabeleceu que a tortura,
Elemento subjetivo: dolo específico, não
equiparada a CRIME HEDIONDO, seria
existindo a forma culposa. Há elemento subjetivo do
insuscetível de anistia, graça, indulto, fiança e
tipo específico: “obter informação, declaração ou
liberdade provisória, além de determinar o regime
confissão da vítima ou de terceira pessoa”; “provocar
fechado para o cumprimento das penas e a fração de
ação ou omissão de natureza criminosa” e “por motivo
2/3 para a concessão de livramento condicional.
de discriminação racial ou religiosa”.
ATENÇÃO: Súmula nº 698 do STF: "Não se
estende aos demais crimes hediondos a ATENÇÃO! Necessário intenso sofrimento físico
admissibilidade de progressão no regime de ou mental à vítima.
execução da pena aplicada ao crime de tortura". Consumação: com o constrangimento,
independentemente do resultado.
Semelhança entre o delito de tortura e o
crime de constrangimento ilegal, previsto no art. Espécies de tortura (Luís Flávio Gomes):
146 do Código Penal: em ambos o agente visa a Tortura prova - prevista na alínea ‘a’ do inc. I
obter da vítima um determinado comportamento do art. 1º, da Lei nº 9.455/97,
positivo ou negativo. que é aquela aplicada para obtenção de confissão
I - constranger alguém com emprego de violência ou outra prova, cuja ilicitude é, desde logo,
ou grave ameaça, causando-lhe sofrimento físico ou incontestável;
mental: Tortura meio - prevista na alínea ‘b’ (inc. I do
a) com o fim de obter informação, declaração ou art. 1º), que se distingue como uma coação para que
confissão da vítima ou de terceira pessoa; outrem pratique crime;
b) para provocar ação ou omissão de natureza Tortura discriminatória ou racial- prevista na
criminosa; alínea ‘c’ (inc. I do art. 1º). Esta seria, para o autor
c) em razão de discriminação racial ou religiosa; referido, grave defeito da lei, visto que exige uma
especial motivação do agente, inviabilizando a
Comentário:
persecutio criminis fora das hipóteses expressamente
Referido inciso possui três alíneas, as quais especificadas, não tendo incidência, pois, sobre, por
funcionam como elemento subjetivo do tipo. São elas: exemplo, discriminações sexuais ou por vingança; e
a) com o fim de obter informação, declaração ou
Tortura pena (art. 1.º, II) - caracterizada pela
confissão da vítima ou de terceira pessoa; b) para
aplicação de tortura a alguém sob sua „guarda, poder
provocar ação ou omissão de natureza criminosa; c)
ou autoridade‟.
em razão de discriminação racial ou religiosa. Dessa
forma, no art. 1º, I, da Lei n.9.455/97, estão previstos DICA DE CONCURSO: Considere a seguinte
três crimes de tortura: situação hipotética. Carlos, após a prática de atos
a) Primeiro crime: tortura-persecutória ou eficientes para causar intenso sofrimento físico e
tortura-prova. Constranger alguém com emprego de mental em José, visando à obtenção de informações

www.editoradince.com - Acesse e veja se há novidades a respeito deste material – CUIDADO: cópia é crime.
58 LEGISLAÇÃO ESPECIAL E REGIMENTO INTERNO DAS UNIDADES DO SEAS
sigilosas, matou-o para que sua conduta não fosse agente ativo. Na realidade deve-se fazer uma
descoberta. Nesse caso, Carlos responderá pelo separação entre aquele que pode agir (comunicar o
crime de tortura simples em concurso material, com fato criminoso as autoridades) para evitar o resultado,
o delito de homicídio. (Escrivão/ daquele que se omite em apurar os fatos. Para o
Tocantins/CESPE/2008). primeiro, deve incidir a regra geral do art. 13, § 2º do
II - submeter alguém, sob sua guarda, poder ou Código Penal. Ou seja, entendo que deve responder
autoridade, com emprego de violência ou grave ameaça, como partícipe. Àquele que tem o dever jurídico de
a intenso sofrimento físico ou mental, como forma de apurar os fatos, e não o faz, incide o § 2º, como
aplicar castigo pessoal ou medida de caráter preventivo. tipificação autônoma.
Pena - reclusão, de dois a oito anos. Objeto jurídico protegido: a dignidade da pessoa
humana.
Comentário:
Sujeito ativo: aquele que tem o dever jurídico de
Pelo delito tipificado no inciso II, responde apenas apurar a prática de tortura, como o policial, delegado
quem possui autoridade, guarda ou vigilância da de polícia, agente penitenciário, legista que
vítima. É, portanto, um crime próprio, distinto do intencionalmente, em seu laudo omite a prática da
inciso I. Assim, o crime pode ser cometido contra filho tortura.
menor ou incapaz, tutelado, preso, interno em
estabelecimento de ensino ou hospitalar, etc. É Sujeito passivo: a pessoa que sobre o castigo
importante registrar que por falta de previsão legal não pessoal ou medida de caráter preventivo.
pode ser enquadrado como tortura (inciso II) o crime Núcleo do tipo: o verbo “constranger”.
de esposo contra a esposa e pai contra filho maior (por Elemento subjetivo: dolo específico.
não haver subordinação). O mesmo ocorrendo em
ATENÇÃO!! Respondem pelo crime de tortura
relação ao crime de maus-tratos. Porém, caso ocorra
tanto as pessoas que a praticam quanto as que,
tal hipótese, deverá ser enquadrado no Inciso I da
podendo evitá-la, se omitem.
referida lei (constranger alguém...), desde que
satisfeitos os requisitos das alíneas A,B e C. DICA DE CONCURSO: No momento de seu
interrogatório policial, João, acusado por tráfico de
Objeto jurídico protegido: integridade corporal
entorpecentes, foi submetido pelos policiais
ou saúde mental de pessoa sujeita a guarda, poder ou
responsáveis pelo procedimento a choques elétricos
autoridade de outrem.
e asfixia parcial, visando à obtenção de informações
Sujeito ativo: Crime próprio: cometido por quem sobre o endereço utilizado pelo suposto traficante
possui autoridade, guarda ou poder sobre a vítima. Ex: como depósito da droga. João, após as agressões,
pai, tutor, curador, diretor ou funcionário de hospital, comunicou o fato à autoridade policial de plantão, a
colégio, etc. qual, apesar de não ter participado da prática
Sujeito passivo: a pessoa que sofre a tortura. delituosa, não adotou nenhuma providência no
Núcleo do tipo: o verbo “submeter”. sentido de apurar a notícia de tortura. Nessa
situação, a autoridade policial responderá por sua
Consumação: no momento em que a pessoa é
omissão, conforme previsão expressa na Lei de
submetida à guarda, ao poder ou à autoridade.
Tortura. (Escrivão/Tocantins/ CESPE/2008).
§ 1º Na mesma pena incorre quem submete
pessoa presa ou sujeita a medida de segurança a § 3º Se resulta lesão corporal de natureza grave ou
sofrimento físico ou mental, por intermédio da prática de gravíssima, a pena é de reclusão de quatro a dez anos;
ato não previsto em lei ou não resultante de medida legal. se resulta morte, a reclusão é de oito a dezesseis anos.
Comentário: Comentário:
“Mesmo o homem desfigurado pela prática do Tortura qualificada
crime e afastado do convívio com a sociedade, A figura qualificada da tortura está prevista no
mediante recolhimento ao cárcere, merece ter sua artigo 1°, em seu § 3°, o qual prevê outra pena em
integridade física e sua dignidade preservadas. A pena abstrato, qual seja, a pena de 4 a 10 anos ou de 08 a 16
imposta limita-se à privação da liberdade, não anos, a depender do resultado.
podendo ser acompanhada de outras medidas aflitivas,  Se o agente assumir o risco de matar e
nem de humilhações”. (CAPEZ, Fernando. Curso de empregar a tortura = art. 121, parág. 2º, III, CP.
direito penal. São Paulo: Saraiva, vol. 4, p. 666) § 4º Aumenta-se a pena de um sexto até um terço:
Direitos do preso I - se o crime é cometido por agente público;
Art. 38 - O preso conserva todos os direitos não II – se o crime é cometido contra criança, gestante,
atingidos pela perda da liberdade, impondo-se a todas portador de deficiência, adolescente ou maior de 60
as autoridades o respeito à sua integridade física e (sessenta) anos; (Redação dada pela Lei nº 10.741, de 2003)
moral.
III - se o crime é cometido mediante sequestro.
Art. 40 - Impõe-se a todas as autoridades o
respeito à integridade física e moral dos condenados e Comentário:
dos presos provisórios. (LEP). O artigo 1°, em seu parágrafo 4° prevê as causas
§ 2º Aquele que se omite em face dessas de aumento de pena.
condutas, quando tinha o dever de evitá-las ou apurá-las, Verifica-se a existência de seis causas de aumento
incorre na pena de detenção de um a quatro anos. de pena.
Comentário: Crime é cometido por agente público = a prática
Responsabilidade criminal pela omissão do crime por agente público é causa de aumento de
pena, e a condenação acarretará a perda do cargo,
O ponto tratado aqui é a incidência daqueles que função ou emprego público. A expressão agente
contribuem para o crime de forma omissiva. O público deve ser interpretada no sentido de funcionário
parágrafo pune de forma mais branda o omisso que o público, cujo conceito é fornecido pelo art. 327 do

www.editoradince.com - Acesse e veja se há novidades a respeito deste material – CUIDADO: cópia é crime.
LEGISLAÇÃO ESPECIAL E REGIMENTO INTERNO DAS UNIDADES DO SEAS 59
Código Penal. Trata-se, como se vê, de delito Trata-se de efeito da condenação, cuja aplicação
funcional. independe do montante da pena fixada na sentença. A
Crime é cometido contra criança - Por criança, simples condenação transitada em julgado acarreta os
devendo entender-se como tal, a pessoa que ainda não efeitos mencionados acima.
atingiu doze anos de idade (art. 20 da Lei n° 8.069/90). DICA DE CONCURSO: Um agente de polícia civil
Nesse aspecto, cumpre recordar que nem toda criança, foi condenado a 6 anos de reclusão pela prática de
à semelhança do que ocorre com o delito de tortura contra preso que estava sob sua autoridade.
constrangimento ilegal, pode ser vítima do crime de Nessa situação, o policial condenado deve perder seu
tortura, previsto no inc. I do art. 10 da Lei n° 9.455/97. cargo público e, durante 12 anos, se-lhe-á vedado
Isso porque o bem jurídico protegido é a liberdade exercer cargos, funções ou empregos públicos.
física e psíquica da pessoa, notadamente a sua (CESPE/DPF/ PERITO/ 2004)
liberdade de autodeterminação, de modo que se a
§ 6º O crime de tortura é inafiançável e insuscetível
capacidade de entendimento for totalmente nula,
de graça ou anistia.
exclui-se a possibilidade de a criança ou doente mental
figurar como sujeito passivo do crime. Entretanto, Comentário:
desde que ostentem o mínimo de capacidade para O dispositivo em apreço simplesmente repete o
entender e sentir o temor produzido pelo mal inc. XLIII do art. 5° da Magna Carta. Note-se que a lei
ameaçado, criança e louco podem ser sujeitos passivos não veda a concessão do indulto. Aliás, eventual
do delito em estudo. No tocante à tortura do inc. II do vedação estaria eivada de flagrante
art. 10 e seu § 10 da Lei n° 9.455/97, o delito se inconstitucionalidade, porque a Magna Carta, no inc.
caracteriza ainda que a criança ou doente mental não XII de seu art. 84, não cria restrições à concessão do
ostentem o mínimo de capacidade de entendimento, indulto. Não se pode equiparar a graça ao indulto,
porque o tipo não faz menção a nenhum porque é proibida a analogia in malam partem.
comportamento do ofendido. Proibição de graça ou anistia
Crime é cometido contra gestante. Nesse caso, Graça, anistia e indulto são causas extintivas da
exclui-se a agravante genérica do art. 61, inc. lI, alínea punibilidade, sendo que a graça é concedida
h, do Código Penal, uma vez que essa agravante já individualmente e precisa ser pedida, enquanto o
funciona como causa de aumento de pena, não indulto é concedido coletivamente, independentemente
podendo, evidentemente, ser duplamente valorada, sob de pedido. A concessão cabe ao Presidente da
pena de consagração do bis in idem. Cumpre destacar, República e atinge somente a execução da pena,
ainda, que o agente deve ter ciência da gravidez. remanescendo os demais efeitos. Já a anistia,
Finalmente, para a incidência da majorante, urge que o concedida pelo Congresso Nacional com a aprovação
agente não queira e nem assuma o risco de produzir o do Presidente, encerra a noção de perdão geral,
aborto. De fato, o dolo em relação ao aborto dá ensejo atingindo, portanto, não só a execução, como todos os
ao delito do art. 125 do Código Penal, em sua forma efeitos do crime.
consumada ou tentada, em concurso com o crime de
Como não há menção ao indulto, há divergência
tortura simples (Lei n° 9.455/97), sem a incidência da
quanto à possibilidade de sua aplicação ao crime de
causa de aumento de pena.
tortura.
Crime é cometido contra deficiente. A
Crime inafiançável
fragilidade da vítima justifica o aumento da pena. A
lei não distingue entre o deficiente físico e o mental, Prevê ainda §6º ser a tortura crime inafiançável,
de modo que em ambas as hipóteses incide a seguindo o mandamento constitucional. Tal previsão,
majorante. Entretanto, tratando-se de doente mental, à no entanto, é de pouca utilidade, já que impede a
exceção da tortura prevista no inc. II do art. 10 e seu § concessão de fiança, mas não a de liberdade
10 da Lei n° 9.455/97, mister o mínimo de provisória. É que, pela sistemática processual penal,
discernimento, sob pena de descaracterização do delito será concedida liberdade provisória sempre que não
de tortura previsto no inc. I do art. 10 da Lei n° estiverem presentes os motivos ensejadores da prisão
9.455/97. preventiva. Assim, se estiverem ausentes os motivos
da prisão preventiva, independentemente do fato de
Crime é cometido contra adolescente. Mais uma
não ser afiançável, o acusado deverá responder o
vez a imaturidade da vítima, à semelhança do que
processo em liberdade, e sem precisar prestar fiança.
ocorre com a criança, justifica o aumento da pena.
Adolescente é a pessoa que tem entre doze a dezoito § 7º O condenado por crime previsto nesta Lei,
anos (art. 2° da Lei n° 8.069/90). salvo a hipótese do § 2º, iniciará o cumprimento da pena
em regime fechado.
Crime é cometido mediante sequestro. Nesse
caso, exclui-se a incidência do delito previsto no art. Comentário:
148 do Código Penal, que já funciona como causa de Qualquer que seja a quantidade da pena, mesmo
aumento de pena do crime de tortura. A absorção do quando o réu for primário e tiver bons antecedentes, o
delito de sequestro é fundamentada no princípio da regime inicial é o fechado. Todavia, não há vedação da
subsidiariedade implícita. progressão para os regimes semi-aberto e aberto.
Nesse aspecto, a lei é mais benigna que a dos crimes
ATENÇÃO!! A prática de tortura mediante
hediondos (Lei n° 8.072/90), que, além de impor o
sequestro é uma causa de aumento de pena, e não
regime fechado, ainda proíbe a progressão para os
uma qualificadora.
regimes menos rigorosos. Por consequência, a vedação
§ 5º A condenação acarretará a perda do cargo, da progressão de regimes, prevista na Lei n ° 8.072/92,
função ou emprego público e a interdição para seu não é mais aplicada ao delito de tortura. Cumpre ainda
exercício pelo dobro do prazo da pena aplicada. destacar que na hipótese do § 2° do art. 1° não há
Comentário: sequer a obrigatoriedade de a pena iniciar-se no
Efeitos da condenação regime fechado.

www.editoradince.com - Acesse e veja se há novidades a respeito deste material – CUIDADO: cópia é crime.
60 LEGISLAÇÃO ESPECIAL E REGIMENTO INTERNO DAS UNIDADES DO SEAS
 STF, Súmula 698 = “Não se estende aos cometendo infração, praticar intencionalmente algum
demais crimes hediondos a admissibilidade de ato para causar sofrimento mental a essa pessoa,
progressão no regime de execução da pena aplicada ao essa conduta poderá ser caracterizada como tortura.
crime de tortura.”
DICA DE CONCURSO: Permitida nos casos de 03. (CESPE / CEBRASPE - 2020 - MPE-CE - Analista
prática do crime de tortura, a progressão de regime Ministerial - Direito) A respeito da Lei de Crimes de
não alcança os crimes hediondos, o tráfico ilícito de Tortura (Lei n.º 9.455/1997), julgue o próximo item.
entorpecentes e o terrorismo, segundo orientação do A Lei de Crimes de Tortura, ao prever sua incidência
STF. (CESPE/DEFENSOR PÚBLICO DE mesmo sobre crimes que tenham sido cometidos fora
ALAGOAS – 2003) do território nacional, estabelece hipótese de
Art. 2º O disposto nesta Lei aplica-se ainda quando extraterritorialidade incondicionada.
o crime não tenha sido cometido em território nacional,
sendo a vítima brasileira ou encontrando-se o agente em 04. (CESPE - 2018 - PC-MA - Investigador de Polícia) Se,
local sob jurisdição brasileira. com o objetivo de obter confissão, determinado agente
Comentário: de polícia, por meio de grave ameaça, constranger
Extraterritorialidade da lei pessoa presa, causando-lhe sofrimento psicológico,
A lei de tortura prevê em seu artigo 2º a sua A e a vítima for adolescente, o crime será qualificado.
aplicação, mesmo fora do território nacional, quando a B estará configurada uma causa de aumento de pena.
vítima for brasileira, consagrando, assim, para a C a critério do juiz, a condenação poderá acarretar a
espécie, o princípio da defesa ou da proteção. perda do cargo.
 Regra geral: territorialidade: aplica-se a lei D provado o fato, a pena será de detenção.
brasileira aos crimes cometidos no território brasileiro
E quem presenciar o crime e se omitir, incorrerá na
 Se a vítima for brasileira: extraterritorialidade mesma pena do agente.
incondicionada.
 Se o agente encontrar-se em território brasileiro:
princípio da jurisdição universal, justiça cosmopolita, 05. (VUNESP - 2013 - SEJUS-ES - Agente Penitenciário)
ou da jurisdição mundial. Analise as assertivas a seguir:
Art. 3º Esta Lei entra em vigor na data de sua I. constranger alguém, mediante violência ou grave
publicação. ameaça, a não fazer o que a lei permite, ou a fazer o
que ela não manda;
Art. 4º Revoga-se o art. 233 da Lei nº 8.069, de 13 de
julho de 1990 - Estatuto da Criança e do Adolescente. II. submeter alguém, sob sua guarda, poder ou
autoridade, com emprego de violência ou grave
Comentário: ameaça, a intenso sofrimento físico ou mental, como
Direito intertemporal forma de aplicar castigo pessoal ou medida de caráter
O art. 4° revogou expressamente o art. 233 da Lei preventivo;
n° 8.069/90. III. constranger alguém com emprego de grave ameaça,
Assim, a tortura cometida contra criança ou causando-lhe sofrimento mental em razão de
adolescente passou a ser disciplinada pela Lei n° discriminação religiosa.
9.455/97, que, nessa hipótese, prevê o aumento da À luz da Lei n.º 9.455/97, constitui crime de tortura o que
pena de um sexto até um terço (art. 1º, § 4°, lI). se afirma em
A nova lei é mais severa, em relação a tortura a) I e III, apenas.
simples e A tortura qualificada pela lesão corporal b) II, apenas.
grave. Portanto, nesse aspecto, não pode ser aplicada
aos casos anteriores à sua vigência. Todavia, a tortura c) I e II, apenas.
qualificada pela morte ou lesão gravíssima apresenta d) I, II e III.
pena mais branda que a cominada nos § § 2° e 3° do e) II e III, apenas.
art. 233 da Lei n° 8.069/90, impondo-se, destarte, a
retroatividade da lei benigna.
06. (VUNESP - 2013 - SEJUS-ES - Agente Penitenciário)
Brasília, 7 de abril de 1997; 176º da Independência Quanto ao crime de tortura previsto na Lei n°
e 109º da República. 9.455/97, pode-se afirmar que:
a) incorre na pena prevista para o crime de tortura quem
QUESTÕES DE CONCURSOS submete pessoa presa ou sujeita a medida de
segurança a sofrimento físico ou mental, por
01. (CESPE / CEBRASPE - 2021 - DEPEN - Cargo 8 - intermédio da prática de ato não previsto em lei ou
Agente Federal de Execução Penal) Com base na não resultante de medida legal.
legislação especial, julgue o próximo item.
b) o cumprimento da pena deve ocorrer integralmente em
O crime de tortura é inafiançável, devendo o condenado regime fechado.
por esse crime iniciar o cumprimento da pena em
regime fechado. c) admite fiança nas hipóteses legais.
d) não incorre na prática do crime de tortura aquele que
se omite em face de sua prática, ainda que tenha o
02. (CESPE / CEBRASPE - 2020 - PRF - Policial dever de evitá-la.
Rodoviário Federal - Curso de Formação) No que se
refere ao uso diferenciado da força, julgue o item a e) a condenação acarretará a perda do cargo, função ou
seguir. emprego público e a interdição para seu exercício pelo
prazo da pena aplicada.
Se um policial rodoviário federal, com o objetivo de obter
confissão de uma pessoa que tenha sido flagrada

www.editoradince.com - Acesse e veja se há novidades a respeito deste material – CUIDADO: cópia é crime.
LEGISLAÇÃO ESPECIAL E REGIMENTO INTERNO DAS UNIDADES DO SEAS 61
o
Gabarito: 01/C; 02/C; 03/C; 04/B; 05/E; 06/A § 1 São de atendimento aquelas entidades que,
de forma continuada, permanente e planejada, prestam
serviços, executam programas ou projetos e concedem
benefícios de prestação social básica ou especial,
LEI FEDERAL Nº 8.742, DE 07/12/1993, dirigidos às famílias e indivíduos em situações de
vulnerabilidade ou risco social e pessoal, nos termos
QUE DISPÕE SOBRE A ORGANIZAÇÃO desta Lei, e respeitadas as deliberações do Conselho
DA ASSISTÊNCIA SOCIAL (SUAS) E Nacional de Assistência Social (CNAS), de que tratam os
SUAS ALTERAÇÕES. incisos I e II do art. 18. (Incluído pela Lei nº 12.435, de
2011)
O PRESIDENTE DA REPÚBLICA, faço saber que o
§ 2 São de assessoramento aquelas que, de
o Congresso Nacional decreta e eu sanciono a seguinte
forma continuada, permanente e planejada, prestam
lei:
serviços e executam programas ou projetos voltados
LEI ORGÂNICA DA ASSISTÊNCIA SOCIAL prioritariamente para o fortalecimento dos movimentos
CAPÍTULO I sociais e das organizações de usuários, formação e
Das Definições e dos Objetivos capacitação de lideranças, dirigidos ao público da política
de assistência social, nos termos desta Lei, e respeitadas
Art. 1º A assistência social, direito do cidadão e
as deliberações do CNAS, de que tratam os incisos I e II
dever do Estado, é Política de Seguridade Social não
contributiva, que provê os mínimos sociais, realizada do art. 18. (Incluído pela Lei nº 12.435, de 2011)
o
através de um conjunto integrado de ações de iniciativa § 3 São de defesa e garantia de direitos aquelas
pública e da sociedade, para garantir o atendimento às que, de forma continuada, permanente e planejada,
necessidades básicas. prestam serviços e executam programas e projetos
o voltados prioritariamente para a defesa e efetivação dos
Art. 2 A assistência social tem por objetivos:
direitos socioassistenciais, construção de novos direitos,
(Redação dada pela Lei nº 12.435, de 2011)
promoção da cidadania, enfrentamento das
I - a proteção social, que visa à garantia da vida, à desigualdades sociais, articulação com órgãos públicos
redução de danos e à prevenção da incidência de riscos, de defesa de direitos, dirigidos ao público da política de
especialmente: (Redação dada pela Lei nº 12.435, de assistência social, nos termos desta Lei, e respeitadas as
2011) deliberações do CNAS, de que tratam os incisos I e II do
a) a proteção à família, à maternidade, à infância, art. 18. (Incluído pela Lei nº 12.435, de 2011)
à adolescência e à velhice; (Incluído pela Lei nº 12.435, CAPÍTULO II
de 2011)
Dos Princípios e das Diretrizes
b) o amparo às crianças e aos adolescentes
SEÇÃO I
carentes; (Incluído pela Lei nº 12.435, de 2011)
Dos Princípios
c) a promoção da integração ao mercado de
trabalho; (Incluído pela Lei nº 12.435, de 2011) Art. 4º A assistência social rege-se pelos
seguintes princípios:
d) a habilitação e reabilitação das pessoas com
deficiência e a promoção de sua integração à vida I - supremacia do atendimento às necessidades
comunitária; e (Incluído pela Lei nº 12.435, de 2011) sociais sobre as exigências de rentabilidade econômica;
e) a garantia de 1 (um) salário-mínimo de benefício II - universalização dos direitos sociais, a fim de
mensal à pessoa com deficiência e ao idoso que tornar o destinatário da ação assistencial alcançável pelas
comprovem não possuir meios de prover a própria demais políticas públicas;
manutenção ou de tê-la provida por sua família; (Incluído III - respeito à dignidade do cidadão, à sua
pela Lei nº 12.435, de 2011) autonomia e ao seu direito a benefícios e serviços de
II - a vigilância socioassistencial, que visa a qualidade, bem como à convivência familiar e comunitária,
analisar territorialmente a capacidade protetiva das vedando-se qualquer comprovação vexatória de
famílias e nela a ocorrência de vulnerabilidades, de necessidade;
ameaças, de vitimizações e danos; (Redação dada pela IV - igualdade de direitos no acesso ao
Lei nº 12.435, de 2011) atendimento, sem discriminação de qualquer natureza,
III - a defesa de direitos, que visa a garantir o pleno garantindo-se equivalência às populações urbanas e
acesso aos direitos no conjunto das provisões rurais;
socioassistenciais. (Redação dada pela Lei nº 12.435, de V - divulgação ampla dos benefícios, serviços,
2011) programas e projetos assistenciais, bem como dos
Parágrafo único. Para o enfrentamento da recursos oferecidos pelo Poder Público e dos critérios
pobreza, a assistência social realiza-se de forma para sua concessão.
integrada às políticas setoriais, garantindo mínimos SEÇÃO II
sociais e provimento de condições para atender Das Diretrizes
contingências sociais e promovendo a universalização
Art. 5º A organização da assistência social tem
dos direitos sociais. (Redação dada pela Lei nº 12.435,
como base as seguintes diretrizes:
de 2011)
o I - descentralização político-administrativa para os
Art. 3 Consideram-se entidades e organizações
Estados, o Distrito Federal e os Municípios, e comando
de assistência social aquelas sem fins lucrativos que,
único das ações em cada esfera de governo;
isolada ou cumulativamente, prestam atendimento e
assessoramento aos beneficiários abrangidos por esta II - participação da população, por meio de
Lei, bem como as que atuam na defesa e garantia de organizações representativas, na formulação das políticas
direitos. (Redação dada pela Lei nº 12.435, de 2011) e no controle das ações em todos os níveis;

www.editoradince.com - Acesse e veja se há novidades a respeito deste material – CUIDADO: cópia é crime.
62 LEGISLAÇÃO ESPECIAL E REGIMENTO INTERNO DAS UNIDADES DO SEAS
III - primazia da responsabilidade do Estado na defesa de direito, o fortalecimento das potencialidades e
condução da política de assistência social em cada esfera aquisições e a proteção de famílias e indivíduos para o
de governo. enfrentamento das situações de violação de direitos.
CAPÍTULO III (Incluído pela Lei nº 12.435, de 2011)
Da Organização e da Gestão Parágrafo único. A vigilância socioassistencial é
o um dos instrumentos das proteções da assistência social
Art. 6 A gestão das ações na área de assistência
que identifica e previne as situações de risco e
social fica organizada sob a forma de sistema
vulnerabilidade social e seus agravos no
descentralizado e participativo, denominado Sistema
território. (Incluído pela Lei nº 12.435, de 2011)
Único de Assistência Social (Suas), com os seguintes o
objetivos: (Redação dada pela Lei nº 12.435, de 2011) Art. 6 -B. As proteções sociais básica e especial
serão ofertadas pela rede socioassistencial, de forma
I - consolidar a gestão compartilhada, o
integrada, diretamente pelos entes públicos e/ou pelas
cofinanciamento e a cooperação técnica entre os entes
entidades e organizações de assistência social vinculadas
federativos que, de modo articulado, operam a proteção
ao Suas, respeitadas as especificidades de cada
social não contributiva; (Incluído pela Lei nº 12.435, de
ação. (Incluído pela Lei nº 12.435, de 2011)
2011) o
§ 1 A vinculação ao Suas é o reconhecimento
II - integrar a rede pública e privada de serviços,
pelo Ministério do Desenvolvimento Social e Combate à
programas, projetos e benefícios de assistência social, na
o Fome de que a entidade de assistência social integra a
forma do art. 6 -C; (Incluído pela Lei nº 12.435, de 2011)
rede socioassistencial. (Incluído pela Lei nº 12.435, de
III - estabelecer as responsabilidades dos entes 2011)
federativos na organização, regulação, manutenção e o o
§ 2 Para o reconhecimento referido no § 1 , a
expansão das ações de assistência social;
entidade deverá cumprir os seguintes requisitos: (Incluído
IV - definir os níveis de gestão, respeitadas as pela Lei nº 12.435, de 2011)
diversidades regionais e municipais; (Incluído pela Lei nº
I - constituir-se em conformidade com o disposto
12.435, de 2011) o
no art. 3 ; (Incluído pela Lei nº 12.435, de 2011)
V - implementar a gestão do trabalho e a educação
II - inscrever-se em Conselho Municipal ou do
permanente na assistência social; (Incluído pela Lei nº o
Distrito Federal, na forma do art. 9 ; (Incluído pela Lei nº
12.435, de 2011)
12.435, de 2011)
VI - estabelecer a gestão integrada de serviços e
III - integrar o sistema de cadastro de entidades de
benefícios; e (Incluído pela Lei nº 12.435, de 2011)
que trata o inciso XI do art. 19. (Incluído pela Lei nº
VII - afiançar a vigilância socioassistencial e a 12.435, de 2011)
garantia de direitos. (Incluído pela Lei nº 12.435, de 2011) o
o § 3 As entidades e organizações de assistência
§ 1 As ações ofertadas no âmbito do Suas têm social vinculadas ao Suas celebrarão convênios,
por objetivo a proteção à família, à maternidade, à contratos, acordos ou ajustes com o poder público para a
infância, à adolescência e à velhice e, como base de execução, garantido financiamento integral, pelo Estado,
organização, o território. (Incluído pela Lei nº 12.435, de de serviços, programas, projetos e ações de assistência
2011) social, nos limites da capacidade instalada, aos
o
§ 2 O Suas é integrado pelos entes federativos, beneficiários abrangidos por esta Lei, observando-se as
pelos respectivos conselhos de assistência social e pelas disponibilidades orçamentárias. (Incluído pela Lei nº
entidades e organizações de assistência social 12.435, de 2011)
abrangidas por esta Lei. (Incluído pela Lei nº 12.435, de o
§ 4 O cumprimento do disposto no § 3 será
o
2011) informado ao Ministério do Desenvolvimento Social e
o
§ 3 A instância coordenadora da Política Nacional Combate à Fome pelo órgão gestor local da assistência
de Assistência Social é o Ministério do Desenvolvimento social. (Incluído pela Lei nº 12.435, de 2011)
Social e Combate à Fome. (Incluído pela Lei nº 12.435, o
Art. 6 -C. As proteções sociais, básica e especial,
de 2011) serão ofertadas precipuamente no Centro de Referência
§ 4º Cabe à instância coordenadora da Política de Assistência Social (Cras) e no Centro de Referência
Nacional de Assistência Social normatizar e padronizar o Especializado de Assistência Social (Creas),
emprego e a divulgação da identidade visual do Suas. respectivamente, e pelas entidades sem fins lucrativos de
o
(Incluído pela Lei nº 13.714, de 2018) assistência social de que trata o art. 3 desta
§ 5º A identidade visual do Suas deverá prevalecer Lei. (Incluído pela Lei nº 12.435, de 2011)
o
na identificação de unidades públicas estatais, entidades § 1 O Cras é a unidade pública municipal, de base
e organizações de assistência social, serviços, territorial, localizada em áreas com maiores índices de
programas, projetos e benefícios vinculados ao vulnerabilidade e risco social, destinada à articulação dos
Suas. (Incluído pela Lei nº 13.714, de 2018) serviços socioassistenciais no seu território de
o abrangência e à prestação de serviços, programas e
Art. 6 -A. A assistência social organiza-se pelos
seguintes tipos de proteção: (Incluído pela Lei nº 12.435, projetos socioassistenciais de proteção social básica às
de 2011) famílias. (Incluído pela Lei nº 12.435, de 2011)
o
I - proteção social básica: conjunto de serviços, § 2 O Creas é a unidade pública de abrangência e
programas, projetos e benefícios da assistência social que gestão municipal, estadual ou regional, destinada à
visa a prevenir situações de vulnerabilidade e risco social prestação de serviços a indivíduos e famílias que se
por meio do desenvolvimento de potencialidades e encontram em situação de risco pessoal ou social, por
aquisições e do fortalecimento de vínculos familiares e violação de direitos ou contingência, que demandam
comunitários; (Incluído pela Lei nº 12.435, de 2011) intervenções especializadas da proteção social especial.
II - proteção social especial: conjunto de serviços, (Incluído pela Lei nº 12.435, de 2011)
o
programas e projetos que tem por objetivo contribuir para § 3 Os Cras e os Creas são unidades públicas
a reconstrução de vínculos familiares e comunitários, a estatais instituídas no âmbito do Suas, que possuem

www.editoradince.com - Acesse e veja se há novidades a respeito deste material – CUIDADO: cópia é crime.
LEGISLAÇÃO ESPECIAL E REGIMENTO INTERNO DAS UNIDADES DO SEAS 63
interface com as demais políticas públicas e articulam, II - cofinanciar, por meio de transferência
coordenam e ofertam os serviços, programas, projetos e automática, o aprimoramento da gestão, os serviços, os
benefícios da assistência social. (Incluído pela Lei nº programas e os projetos de assistência social em âmbito
12.435, de 2011) nacional; (Redação dada pela Lei nº 12.435, de 2011)
o
Art. 6 -D. As instalações dos Cras e dos Creas III - atender, em conjunto com os Estados, o
devem ser compatíveis com os serviços neles ofertados, Distrito Federal e os Municípios, às ações assistenciais de
com espaços para trabalhos em grupo e ambientes caráter de emergência.
específicos para recepção e atendimento reservado das IV - realizar o monitoramento e a avaliação da
famílias e indivíduos, assegurada a acessibilidade às política de assistência social e assessorar Estados,
pessoas idosas e com deficiência. (Incluído pela Lei nº Distrito Federal e Municípios para seu desenvolvimento.
12.435, de 2011) (Incluído pela Lei nº 12.435, de 2011)
o
Art. 6 -E. Os recursos do cofinanciamento do Art. 12-A. A União apoiará financeiramente o
Suas, destinados à execução das ações continuadas de aprimoramento à gestão descentralizada dos serviços,
assistência social, poderão ser aplicados no pagamento programas, projetos e benefícios de assistência social,
dos profissionais que integrarem as equipes de referência, por meio do Índice de Gestão Descentralizada (IGD) do
responsáveis pela organização e oferta daquelas ações, Sistema Único de Assistência Social (Suas), para a
conforme percentual apresentado pelo Ministério do utilização no âmbito dos Estados, dos Municípios e do
Desenvolvimento Social e Combate à Fome e aprovado Distrito Federal, destinado, sem prejuízo de outras ações
pelo CNAS. (Incluído pela Lei nº 12.435, de 2011) a serem definidas em regulamento, a: (Incluído pela Lei
Parágrafo único. A formação das equipes de nº 12.435, de 2011)
referência deverá considerar o número de famílias e I - medir os resultados da gestão descentralizada
indivíduos referenciados, os tipos e modalidades de do Suas, com base na atuação do gestor estadual,
atendimento e as aquisições que devem ser garantidas municipal e do Distrito Federal na implementação,
aos usuários, conforme deliberações do CNAS. (Incluído execução e monitoramento dos serviços, programas,
pela Lei nº 12.435, de 2011) projetos e benefícios de assistência social, bem como na
Art. 7º As ações de assistência social, no âmbito articulação intersetorial; (Incluído pela Lei nº 12.435, de
das entidades e organizações de assistência social, 2011)
observarão as normas expedidas pelo Conselho Nacional II - incentivar a obtenção de resultados qualitativos
de Assistência Social (CNAS), de que trata o art. 17 desta na gestão estadual, municipal e do Distrito Federal do
lei. Suas; e (Incluído pela Lei nº 12.435, de 2011)
Art. 8º A União, os Estados, o Distrito Federal e os III - calcular o montante de recursos a serem
Municípios, observados os princípios e diretrizes repassados aos entes federados a título de apoio
estabelecidos nesta lei, fixarão suas respectivas Políticas financeiro à gestão do Suas. (Incluído pela Lei nº 12.435,
de Assistência Social. de 2011)
Art. 9º O funcionamento das entidades e o
§ 1 Os resultados alcançados pelo ente federado
organizações de assistência social depende de prévia na gestão do Suas, aferidos na forma de regulamento,
inscrição no respectivo Conselho Municipal de Assistência serão considerados como prestação de contas dos
Social, ou no Conselho de Assistência Social do Distrito recursos a serem transferidos a título de apoio financeiro.
Federal, conforme o caso. (Incluído pela Lei nº 12.435, de 2011)
§ 1º A regulamentação desta lei definirá os o
§ 2 As transferências para apoio à gestão
critérios de inscrição e funcionamento das entidades com descentralizada do Suas adotarão a sistemática do Índice
atuação em mais de um município no mesmo Estado, ou de Gestão Descentralizada do Programa Bolsa Família,
em mais de um Estado ou Distrito Federal. o o
previsto no art. 8 da Lei n 10.836, de 9 de janeiro de
§ 2º Cabe ao Conselho Municipal de Assistência 2004, e serão efetivadas por meio de procedimento
Social e ao Conselho de Assistência Social do Distrito integrado àquele índice. (Incluído pela Lei nº 12.435, de
Federal a fiscalização das entidades referidas no caput na 2011)
forma prevista em lei ou regulamento. o
§ 3 (VETADO). (Incluído pela Lei nº 12.435, de
§ 3º (Revogado pela Lei nº 12.101, de 2009) 2011)
o
§ 4º As entidades e organizações de assistência § 4 Para fins de fortalecimento dos Conselhos de
social podem, para defesa de seus direitos referentes à Assistência Social dos Estados, Municípios e Distrito
inscrição e ao funcionamento, recorrer aos Conselhos Federal, percentual dos recursos transferidos deverá ser
Nacional, Estaduais, Municipais e do Distrito Federal. gasto com atividades de apoio técnico e operacional
Art. 10. A União, os Estados, os Municípios e o àqueles colegiados, na forma fixada pelo Ministério do
Distrito Federal podem celebrar convênios com entidades Desenvolvimento Social e Combate à Fome, sendo
e organizações de assistência social, em conformidade vedada a utilização dos recursos para pagamento de
com os Planos aprovados pelos respectivos Conselhos. pessoal efetivo e de gratificações de qualquer natureza a
servidor público estadual, municipal ou do Distrito Federal.
Art. 11. As ações das três esferas de governo na
(Incluído pela Lei nº 12.435, de 2011)
área de assistência social realizam-se de forma
articulada, cabendo a coordenação e as normas gerais à Art. 13. Compete aos Estados:
esfera federal e a coordenação e execução dos I - destinar recursos financeiros aos Municípios, a
programas, em suas respectivas esferas, aos Estados, ao título de participação no custeio do pagamento dos
Distrito Federal e aos Municípios. benefícios eventuais de que trata o art. 22, mediante
Art. 12. Compete à União: critérios estabelecidos pelos Conselhos Estaduais de
Assistência Social; (Redação dada pela Lei nº 12.435,
I - responder pela concessão e manutenção dos
de 2011)
benefícios de prestação continuada definidos no art. 203
da Constituição Federal; II - cofinanciar, por meio de transferência
automática, o aprimoramento da gestão, os serviços, os

www.editoradince.com - Acesse e veja se há novidades a respeito deste material – CUIDADO: cópia é crime.
64 LEGISLAÇÃO ESPECIAL E REGIMENTO INTERNO DAS UNIDADES DO SEAS
programas e os projetos de assistência social em âmbito III - o Conselho de Assistência Social do Distrito
regional ou local; (Redação dada pela Lei nº 12.435, de Federal;
2011) IV - os Conselhos Municipais de Assistência
III - atender, em conjunto com os Municípios, às Social.
ações assistenciais de caráter de emergência; Parágrafo único. Os Conselhos de Assistência
IV - estimular e apoiar técnica e financeiramente Social estão vinculados ao órgão gestor de assistência
as associações e consórcios municipais na prestação de social, que deve prover a infraestrutura necessária ao seu
serviços de assistência social; funcionamento, garantindo recursos materiais, humanos e
V - prestar os serviços assistenciais cujos custos financeiros, inclusive com despesas referentes a
ou ausência de demanda municipal justifiquem uma rede passagens e diárias de conselheiros representantes do
regional de serviços, desconcentrada, no âmbito do governo ou da sociedade civil, quando estiverem no
respectivo Estado. exercício de suas atribuições. (Incluído pela Lei nº
12.435, de 2011)
VI - realizar o monitoramento e a avaliação da
política de assistência social e assessorar os Municípios Art. 17. Fica instituído o Conselho Nacional de
para seu desenvolvimento. (Incluído pela Lei nº 12.435, Assistência Social (CNAS), órgão superior de deliberação
de 2011) colegiada, vinculado à estrutura do órgão da
Administração Pública Federal responsável pela
Art. 14. Compete ao Distrito Federal:
coordenação da Política Nacional de Assistência Social,
I - destinar recursos financeiros para custeio do cujos membros, nomeados pelo Presidente da República,
pagamento dos benefícios eventuais de que trata o art. têm mandato de 2 (dois) anos, permitida uma única
22, mediante critérios estabelecidos pelos Conselhos de recondução por igual período.
Assistência Social do Distrito Federal; (Redação dada
§ 1º O Conselho Nacional de Assistência Social
pela Lei nº 12.435, de 2011)
(CNAS) é composto por 18 (dezoito) membros e
II - efetuar o pagamento dos auxílios natalidade e respectivos suplentes, cujos nomes são indicados ao
funeral; órgão da Administração Pública Federal responsável pela
III - executar os projetos de enfrentamento da coordenação da Política Nacional de Assistência Social,
pobreza, incluindo a parceria com organizações da de acordo com os critérios seguintes:
sociedade civil; I - 9 (nove) representantes governamentais,
IV - atender às ações assistenciais de caráter de incluindo 1 (um) representante dos Estados e 1 (um) dos
emergência; Municípios;
V - prestar os serviços assistenciais de que trata o II - 9 (nove) representantes da sociedade civil,
art. 23 desta lei. dentre representantes dos usuários ou de organizações
VI - cofinanciar o aprimoramento da gestão, os de usuários, das entidades e organizações de assistência
serviços, os programas e os projetos de assistência social social e dos trabalhadores do setor, escolhidos em foro
em âmbito local; (Incluído pela Lei nº 12.435, de 2011) próprio sob fiscalização do Ministério Público Federal.
VII - realizar o monitoramento e a avaliação da § 2º O Conselho Nacional de Assistência Social
política de assistência social em seu âmbito. (Incluído (CNAS) é presidido por um de seus integrantes, eleito
pela Lei nº 12.435, de 2011) dentre seus membros, para mandato de 1 (um) ano,
Art. 15. Compete aos Municípios: permitida uma única recondução por igual período.
I - destinar recursos financeiros para custeio do § 3º O Conselho Nacional de Assistência Social
pagamento dos benefícios eventuais de que trata o art. (CNAS) contará com uma Secretaria Executiva, a qual
22, mediante critérios estabelecidos pelos Conselhos terá sua estrutura disciplinada em ato do Poder Executivo.
o
Municipais de Assistência Social; (Redação dada pela § 4 Os Conselhos de que tratam os incisos II, III e
Lei nº 12.435, de 2011) IV do art. 16, com competência para acompanhar a
II - efetuar o pagamento dos auxílios natalidade e execução da política de assistência social, apreciar e
funeral; aprovar a proposta orçamentária, em consonância com as
diretrizes das conferências nacionais, estaduais, distrital e
III - executar os projetos de enfrentamento da municipais, de acordo com seu âmbito de atuação,
pobreza, incluindo a parceria com organizações da deverão ser instituídos, respectivamente, pelos Estados,
sociedade civil; pelo Distrito Federal e pelos Municípios, mediante lei
IV - atender às ações assistenciais de caráter de específica. (Redação dada pela Lei nº 12.435, de 2011)
emergência; Art. 18. Compete ao Conselho Nacional de
V - prestar os serviços assistenciais de que trata o Assistência Social:
art. 23 desta lei. I - aprovar a Política Nacional de Assistência
VI - cofinanciar o aprimoramento da gestão, os Social;
serviços, os programas e os projetos de assistência social II - normatizar as ações e regular a prestação de
em âmbito local; (Incluído pela Lei nº 12.435, de 2011) serviços de natureza pública e privada no campo da
VII - realizar o monitoramento e a avaliação da assistência social;
política de assistência social em seu âmbito. (Incluído III - acompanhar e fiscalizar o processo de
pela Lei nº 12.435, de 2011) certificação das entidades e organizações de assistência
Art. 16. As instâncias deliberativas do Suas, de social no Ministério do Desenvolvimento Social e
caráter permanente e composição paritária entre governo Combate à Fome; (Redação dada pela Lei nº 12.101, de
e sociedade civil, são: (Redação dada pela Lei nº 12.435, 2009)
de 2011) IV - apreciar relatório anual que conterá a relação
I - o Conselho Nacional de Assistência Social; de entidades e organizações de assistência social
II - os Conselhos Estaduais de Assistência Social; certificadas como beneficentes e encaminhá-lo para

www.editoradince.com - Acesse e veja se há novidades a respeito deste material – CUIDADO: cópia é crime.
LEGISLAÇÃO ESPECIAL E REGIMENTO INTERNO DAS UNIDADES DO SEAS 65
conhecimento dos Conselhos de Assistência Social dos trimestrais e anuais de atividades e de realização
Estados, Municípios e do Distrito Federal; (Redação financeira dos recursos;
dada pela Lei nº 12.101, de 2009) VIII - prestar assessoramento técnico aos
V - zelar pela efetivação do sistema Estados, ao Distrito Federal, aos Municípios e às
descentralizado e participativo de assistência social; entidades e organizações de assistência social;
VI - a partir da realização da II Conferência IX - formular política para a qualificação
Nacional de Assistência Social em 1997, convocar sistemática e continuada de recursos humanos no campo
ordinariamente a cada quatro anos a Conferência da assistência social;
Nacional de Assistência Social, que terá a atribuição de X - desenvolver estudos e pesquisas para
avaliar a situação da assistência social e propor diretrizes fundamentar as análises de necessidades e formulação
para o aperfeiçoamento do sistema; (Redação dada pela de proposições para a área;
Lei nº 9.720, de 26.4.1991)
XI - coordenar e manter atualizado o sistema de
VII - (Vetado.) cadastro de entidades e organizações de assistência
VIII - apreciar e aprovar a proposta orçamentária social, em articulação com os Estados, os Municípios e o
da Assistência Social a ser encaminhada pelo órgão da Distrito Federal;
Administração Pública Federal responsável pela XII - articular-se com os órgãos responsáveis
coordenação da Política Nacional de Assistência Social; pelas políticas de saúde e previdência social, bem como
IX - aprovar critérios de transferência de recursos com os demais responsáveis pelas políticas sócio-
para os Estados, Municípios e Distrito Federal, econômicas setoriais, visando à elevação do patamar
considerando, para tanto, indicadores que informem sua mínimo de atendimento às necessidades básicas;
regionalização mais eqüitativa, tais como: população, XIII - expedir os atos normativos necessários à
renda per capita, mortalidade infantil e concentração de gestão do Fundo Nacional de Assistência Social (FNAS),
renda, além de disciplinar os procedimentos de repasse de acordo com as diretrizes estabelecidas pelo Conselho
de recursos para as entidades e organizações de Nacional de Assistência Social (CNAS);
assistência social, sem prejuízo das disposições da Lei de
XIV - elaborar e submeter ao Conselho Nacional
Diretrizes Orçamentárias;
de Assistência Social (CNAS) os programas anuais e
X - acompanhar e avaliar a gestão dos recursos, plurianuais de aplicação dos recursos do Fundo Nacional
bem como os ganhos sociais e o desempenho dos de Assistência Social (FNAS).
programas e projetos aprovados;
Parágrafo único. A atenção integral à saúde,
XI - estabelecer diretrizes, apreciar e aprovar os inclusive a dispensação de medicamentos e produtos de
programas anuais e plurianuais do Fundo Nacional de interesse para a saúde, às famílias e indivíduos em
Assistência Social (FNAS); situações de vulnerabilidade ou risco social e pessoal, nos
XII - indicar o representante do Conselho Nacional termos desta Lei, dar-se-á independentemente da
de Assistência Social (CNAS) junto ao Conselho Nacional apresentação de documentos que comprovem domicílio
da Seguridade Social; ou inscrição no cadastro no Sistema Único de Saúde
XIII - elaborar e aprovar seu regimento interno; (SUS), em consonância com a diretriz de articulação das
ações de assistência social e de saúde a que se refere o
XIV - divulgar, no Diário Oficial da União, todas as
inciso XII deste artigo. (Incluído pela Lei nº 13.714, de
suas decisões, bem como as contas do Fundo Nacional
2018)
de Assistência Social (FNAS) e os respectivos pareceres
emitidos. CAPÍTULO IV
Parágrafo único. (Revogado pela Lei nº 12.101, de Dos Benefícios, dos Serviços, dos Programas e dos
2009) Projetos de Assistência Social
Art. 19. Compete ao órgão da Administração SEÇÃO I
Pública Federal responsável pela coordenação da Política Do Benefício de Prestação Continuada
Nacional de Assistência Social: Art. 20. O benefício de prestação continuada é a
I - coordenar e articular as ações no campo da garantia de um salário-mínimo mensal à pessoa com
assistência social; deficiência e ao idoso com 65 (sessenta e cinco) anos ou
II - propor ao Conselho Nacional de Assistência mais que comprovem não possuir meios de prover a
Social (CNAS) a Política Nacional de Assistência Social, própria manutenção nem de tê-la provida por sua família.
suas normas gerais, bem como os critérios de prioridade (Redação dada pela Lei nº 12.435, de 2011) (Vide Lei nº
e de elegibilidade, além de padrões de qualidade na 13.985, de 2020)
o
prestação de benefícios, serviços, programas e projetos; § 1 Para os efeitos do disposto no caput, a família
III - prover recursos para o pagamento dos é composta pelo requerente, o cônjuge ou companheiro,
benefícios de prestação continuada definidos nesta lei; os pais e, na ausência de um deles, a madrasta ou o
padrasto, os irmãos solteiros, os filhos e enteados
IV - elaborar e encaminhar a proposta
solteiros e os menores tutelados, desde que vivam sob o
orçamentária da assistência social, em conjunto com as
mesmo teto. (Redação dada pela Lei nº 12.435, de
demais da Seguridade Social;
2011)
V - propor os critérios de transferência dos o
§ 2 Para efeito de concessão do benefício de
recursos de que trata esta lei;
prestação continuada, considera-se pessoa com
VI - proceder à transferência dos recursos deficiência aquela que tem impedimento de longo prazo
destinados à assistência social, na forma prevista nesta de natureza física, mental, intelectual ou sensorial, o qual,
lei; em interação com uma ou mais barreiras, pode obstruir
VII - encaminhar à apreciação do Conselho sua participação plena e efetiva na sociedade em
Nacional de Assistência Social (CNAS) relatórios igualdade de condições com as demais

www.editoradince.com - Acesse e veja se há novidades a respeito deste material – CUIDADO: cópia é crime.
66 LEGISLAÇÃO ESPECIAL E REGIMENTO INTERNO DAS UNIDADES DO SEAS
pessoas. (Redação dada pela Lei nº 13.146, de prestação continuada a outro idoso ou pessoa com
2015) (Vigência) deficiência da mesma família, no cálculo da renda a que
§ 3º Observados os demais critérios de se refere o § 3º deste artigo. (Incluído pela Lei nº 13.982,
elegibilidade definidos nesta Lei, terão direito ao benefício de 2020)
financeiro de que trata o caput deste artigo a pessoa com § 15. O benefício de prestação continuada será
deficiência ou a pessoa idosa com renda familiar devido a mais de um membro da mesma família enquanto
mensal per capita igual ou inferior a 1/4 (um quarto) do atendidos os requisitos exigidos nesta Lei. (Incluído pela
salário-mínimo. (Redação dada pela Lei nº 14.176, de Lei nº 13.982, de 2020)
2021) Art. 20-A. (Revogado pela Lei nº 14.176, de 2021)
I – (revogado); (Redação dada pela Lei nº 14.176, Art. 21. O benefício de prestação continuada deve
de 2021) ser revisto a cada 2 (dois) anos para avaliação da
II - (VETADO). (Incluído pela Lei nº 13.982, de continuidade das condições que lhe deram origem. (Vide
2020) Lei nº 9.720, de 30.11.1998)
o
§ 4 O benefício de que trata este artigo não pode § 1º O pagamento do benefício cessa no
ser acumulado pelo beneficiário com qualquer outro no momento em que forem superadas as condições referidas
âmbito da seguridade social ou de outro regime, salvo os no caput, ou em caso de morte do beneficiário.
da assistência médica e da pensão especial de natureza § 2º O benefício será cancelado quando se
indenizatória. (Redação dada pela Lei nº 12.435, de constatar irregularidade na sua concessão ou utilização.
2011) o
o
§ 3 O desenvolvimento das capacidades
§ 5 A condição de acolhimento em instituições de cognitivas, motoras ou educacionais e a realização de
longa permanência não prejudica o direito do idoso ou da atividades não remuneradas de habilitação e reabilitação,
pessoa com deficiência ao benefício de prestação entre outras, não constituem motivo de suspensão ou
continuada. (Redação dada pela Lei nº 12.435, de 2011) cessação do benefício da pessoa com deficiência.
§ 6º A concessão do benefício ficará sujeita à (Incluído pela Lei nº 12.435, de 2011)
avaliação da deficiência e do grau de impedimento de que § 4º A cessação do benefício de prestação
o
trata o § 2 , composta por avaliação médica e avaliação continuada concedido à pessoa com deficiência não
social realizadas por médicos peritos e por assistentes impede nova concessão do benefício, desde que
sociais do Instituto Nacional de Seguro Social - atendidos os requisitos definidos em regulamento.
INSS. (Redação dada pela Lei nº 12.470, de 2011) (Redação dada pela Lei nº 12.470, de 2011)
o
§ 7 Na hipótese de não existirem serviços no § 5º O beneficiário em gozo de benefício de
município de residência do beneficiário, fica assegurado, prestação continuada concedido judicial ou
na forma prevista em regulamento, o seu administrativamente poderá ser convocado para avaliação
encaminhamento ao município mais próximo que contar das condições que ensejaram sua concessão ou
com tal estrutura. (Incluído pela Lei nº 9.720, de manutenção, sendo-lhe exigida a presença dos requisitos
30.11.1998) previstos nesta Lei e no regulamento. (Incluído pela Lei
o
§ 8 A renda familiar mensal a que se refere o § nº 14.176, de 2021)
o
3 deverá ser declarada pelo requerente ou seu Art. 21-A. O benefício de prestação continuada
representante legal, sujeitando-se aos demais será suspenso pelo órgão concedente quando a pessoa
procedimentos previstos no regulamento para o com deficiência exercer atividade remunerada, inclusive
deferimento do pedido. (Incluído pela Lei nº 9.720, de na condição de microempreendedor individual. (Incluído
30.11.1998) pela Lei nº 12.470, de 2011)
o
§ 9 Os rendimentos decorrentes de estágio o
§ 1 Extinta a relação trabalhista ou a atividade
supervisionado e de aprendizagem não serão empreendedora de que trata o caput deste artigo e,
computados para os fins de cálculo da renda familiar per quando for o caso, encerrado o prazo de pagamento do
o
capita a que se refere o § 3 deste artigo. (Redação dada seguro-desemprego e não tendo o beneficiário adquirido
pela Lei nº 13.146, de 2015) (Vigência) direito a qualquer benefício previdenciário, poderá ser
§ 10. Considera-se impedimento de longo prazo, requerida a continuidade do pagamento do benefício
o
para os fins do § 2 deste artigo, aquele que produza suspenso, sem necessidade de realização de perícia
efeitos pelo prazo mínimo de 2 (dois) anos. (Incluído pela médica ou reavaliação da deficiência e do grau de
Lei nº 12.470, de 2011) incapacidade para esse fim, respeitado o período de
§ 11. Para concessão do benefício de que trata revisão previsto no caput do art. 21. (Incluído pela Lei nº
o caput deste artigo, poderão ser utilizados outros 12.470, de 2011)
o
elementos probatórios da condição de miserabilidade do § 2 A contratação de pessoa com deficiência
grupo familiar e da situação de vulnerabilidade, conforme como aprendiz não acarreta a suspensão do benefício de
regulamento. (Incluído pela Lei nº 13.146, de 2015) prestação continuada, limitado a 2 (dois) anos o
(Vigência) recebimento concomitante da remuneração e do
§ 12. São requisitos para a concessão, a benefício. (Incluído pela Lei nº 12.470, de 2011)
manutenção e a revisão do benefício as inscrições no SEÇÃO II
Cadastro de Pessoas Físicas (CPF) e no Cadastro Único Dos Benefícios Eventuais
para Programas Sociais do Governo Federal - Cadastro
Art. 22. Entendem-se por benefícios eventuais as
Único, conforme previsto em regulamento. (Incluído pela
provisões suplementares e provisórias que integram
Lei nº 13.846, de 2019)
organicamente as garantias do Suas e são prestadas aos
§ 14. O benefício de prestação continuada ou o cidadãos e às famílias em virtude de nascimento, morte,
benefício previdenciário no valor de até 1 (um) salário- situações de vulnerabilidade temporária e de calamidade
mínimo concedido a idoso acima de 65 (sessenta e cinco) pública. (Redação dada pela Lei nº 12.435, de 2011)
anos de idade ou pessoa com deficiência não será
computado, para fins de concessão do benefício de

www.editoradince.com - Acesse e veja se há novidades a respeito deste material – CUIDADO: cópia é crime.
LEGISLAÇÃO ESPECIAL E REGIMENTO INTERNO DAS UNIDADES DO SEAS 67
o
§ 1 A concessão e o valor dos benefícios de que Parágrafo único. Regulamento definirá as diretrizes
trata este artigo serão definidos pelos Estados, Distrito e os procedimentos do Paif. (Incluído pela Lei nº 12.435,
Federal e Municípios e previstos nas respectivas leis de 2011)
orçamentárias anuais, com base em critérios e prazos Art. 24-B. Fica instituído o Serviço de Proteção e
definidos pelos respectivos Conselhos de Assistência Atendimento Especializado a Famílias e Indivíduos
Social. (Redação dada pela Lei nº 12.435, de 2011) (Paefi), que integra a proteção social especial e consiste
o
§ 2 O CNAS, ouvidas as respectivas no apoio, orientação e acompanhamento a famílias e
representações de Estados e Municípios dele indivíduos em situação de ameaça ou violação de direitos,
participantes, poderá propor, na medida das articulando os serviços socioassistenciais com as diversas
disponibilidades orçamentárias das 3 (três) esferas de políticas públicas e com órgãos do sistema de garantia de
governo, a instituição de benefícios subsidiários no valor direitos. (Incluído pela Lei nº 12.435, de 2011)
de até 25% (vinte e cinco por cento) do salário-mínimo Parágrafo único. Regulamento definirá as diretrizes
para cada criança de até 6 (seis) anos de e os procedimentos do Paefi. (Incluído pela Lei nº 12.435,
idade. (Redação dada pela Lei nº 12.435, de 2011) de 2011)
o
§ 3 Os benefícios eventuais subsidiários não Art. 24-C. Fica instituído o Programa de
poderão ser cumulados com aqueles instituídos pelas Leis Erradicação do Trabalho Infantil (Peti), de caráter
o o
n 10.954, de 29 de setembro de 2004, e n 10.458, de 14 intersetorial, integrante da Política Nacional de
de maio de 2002. (Redação dada pela Lei nº 12.435, de Assistência Social, que, no âmbito do Suas, compreende
2011) transferências de renda, trabalho social com famílias e
SEÇÃO III oferta de serviços socioeducativos para crianças e
Dos Serviços adolescentes que se encontrem em situação de trabalho.
(Incluído pela Lei nº 12.435, de 2011)
Art. 23. Entendem-se por serviços o
socioassistenciais as atividades continuadas que visem à § 1 O Peti tem abrangência nacional e será
melhoria de vida da população e cujas ações, voltadas desenvolvido de forma articulada pelos entes federados,
para as necessidades básicas, observem os objetivos, com a participação da sociedade civil, e tem como
princípios e diretrizes estabelecidos nesta Lei. (Redação objetivo contribuir para a retirada de crianças e
dada pela Lei nº 12.435, de 2011) adolescentes com idade inferior a 16 (dezesseis) anos em
o situação de trabalho, ressalvada a condição de aprendiz,
§ 1 O regulamento instituirá os serviços
a partir de 14 (quatorze) anos. (Incluído pela Lei nº
socioassistenciais. (Incluído pela Lei nº 12.435, de 2011)
o 12.435, de 2011)
§ 2 Na organização dos serviços da assistência o
§ 2 As crianças e os adolescentes em situação de
social serão criados programas de amparo, entre outros:
trabalho deverão ser identificados e ter os seus dados
(Incluído pela Lei nº 12.435, de 2011)
inseridos no Cadastro Único para Programas Sociais do
I - às crianças e adolescentes em situação de risco Governo Federal (CadÚnico), com a devida identificação
pessoal e social, em cumprimento ao disposto no art. 227 das situações de trabalho infantil. (Incluído pela Lei nº
o
da Constituição Federal e na Lei n 8.069, de 13 de julho 12.435, de 2011)
de 1990 (Estatuto da Criança e do Adolescente);
SEÇÃO V
(Incluído pela Lei nº 12.435, de 2011)
Dos Projetos de Enfrentamento da Pobreza
II - às pessoas que vivem em situação de
rua. (Incluído pela Lei nº 12.435, de 2011) Art. 25. Os projetos de enfrentamento da pobreza
compreendem a instituição de investimento econômico-
SEÇÃO IV
social nos grupos populares, buscando subsidiar,
Dos Programas de Assistência Social financeira e tecnicamente, iniciativas que lhes garantam
Art. 24. Os programas de assistência social meios, capacidade produtiva e de gestão para melhoria
compreendem ações integradas e complementares com das condições gerais de subsistência, elevação do padrão
objetivos, tempo e área de abrangência definidos para da qualidade de vida, a preservação do meio-ambiente e
qualificar, incentivar e melhorar os benefícios e os sua organização social.
serviços assistenciais. Art. 26. O incentivo a projetos de enfrentamento
§ 1º Os programas de que trata este artigo serão da pobreza assentar-se-á em mecanismos de articulação
definidos pelos respectivos Conselhos de Assistência e de participação de diferentes áreas governamentais e
Social, obedecidos os objetivos e princípios que regem em sistema de cooperação entre organismos
esta lei, com prioridade para a inserção profissional e governamentais, não governamentais e da sociedade
social. civil.
o
§ 2 Os programas voltados para o idoso e a Seção VI
integração da pessoa com deficiência serão devidamente (Incluído pela Lei nº 14.176, de 2021)
articulados com o benefício de prestação continuada Do Auxílio-Inclusão
estabelecido no art. 20 desta Lei. (Redação dada pela Lei Art. 26-A. Terá direito à concessão do auxílio-
nº 12.435, de 2011) inclusão de que trata o art. 94 da Lei nº 13.146, de 6 de
Art. 24-A. Fica instituído o Serviço de Proteção e julho de 2015 (Estatuto da Pessoa com Deficiência), a
Atendimento Integral à Família (Paif), que integra a pessoa com deficiência moderada ou grave que,
proteção social básica e consiste na oferta de ações e cumulativamente: (Incluído pela Lei nº 14.176, de 2021)
serviços socioassistenciais de prestação continuada, nos I – receba o benefício de prestação continuada, de
Cras, por meio do trabalho social com famílias em que trata o art. 20 desta Lei, e passe a exercer atividade:
situação de vulnerabilidade social, com o objetivo de
prevenir o rompimento dos vínculos familiares e a a) que tenha remuneração limitada a 2 (dois)
violência no âmbito de suas relações, garantindo o direito salários-mínimos; e
à convivência familiar e comunitária. (Incluído pela Lei nº b) que enquadre o beneficiário como segurado
12.435, de 2011) obrigatório do Regime Geral de Previdência Social ou

www.editoradince.com - Acesse e veja se há novidades a respeito deste material – CUIDADO: cópia é crime.
68 LEGISLAÇÃO ESPECIAL E REGIMENTO INTERNO DAS UNIDADES DO SEAS
como filiado a regime próprio de previdência social da I – deixar de atender aos critérios de manutenção
União, dos Estados, do Distrito Federal ou dos do benefício de prestação continuada; ou
Municípios; II – deixar de atender aos critérios de concessão
II – tenha inscrição atualizada no CadÚnico no do auxílio-inclusão.
momento do requerimento do auxílio-inclusão; Parágrafo único. Ato do Poder Executivo federal
III – tenha inscrição regular no CPF; e disporá sobre o procedimento de verificação dos critérios
IV – atenda aos critérios de manutenção do de manutenção e de revisão do auxílio-inclusão.
benefício de prestação continuada, incluídos os critérios Art. 26-E. O auxílio-inclusão não está sujeito a
relativos à renda familiar mensal per capita exigida para o desconto de qualquer contribuição e não gera direito a
acesso ao benefício, observado o disposto no § 4º deste pagamento de abono anual. (Incluído pela Lei nº 14.176,
artigo. de 2021) (Vigência)
§ 1º O auxílio-inclusão poderá ainda ser Art. 26-F. Compete ao Ministério da Cidadania a
concedido, nos termos do inciso I do caput deste artigo, gestão do auxílio-inclusão, e ao INSS a sua
mediante requerimento e sem retroatividade no operacionalização e pagamento. (Incluído pela Lei nº
pagamento, ao beneficiário: 14.176, de 2021) (
I – que tenha recebido o benefício de prestação Art. 26-G. As despesas decorrentes do pagamento
continuada nos 5 (cinco) anos imediatamente anteriores do auxílio-inclusão correrão à conta do orçamento do
ao exercício da atividade remunerada; e Ministério da Cidadania. (Incluído pela Lei nº 14.176, de
II – que tenha tido o benefício suspenso nos 2021)
termos do art. 21-A desta Lei. § 1º O Poder Executivo federal compatibilizará o
§ 2º O valor do auxílio-inclusão percebido por um quantitativo de benefícios financeiros do auxílio-inclusão
membro da família não será considerado no cálculo da de que trata o art. 26-A desta Lei com as dotações
renda familiar mensal per capita de que trata o inciso IV orçamentárias existentes.
do caput deste artigo, para fins de concessão e de § 2º O regulamento indicará o órgão do Poder
manutenção de outro auxílio-inclusão no âmbito do Executivo responsável por avaliar os impactos da
mesmo grupo familiar. concessão do auxílio-inclusão na participação no mercado
§ 3º O valor do auxílio-inclusão e o da de trabalho, na redução de desigualdades e no exercício
remuneração do beneficiário do auxílio-inclusão de que dos direitos e liberdades fundamentais das pessoas com
trata a alínea “a” do inciso I do caput deste artigo deficiência, nos termos do § 16 do art. 37 da Constituição
percebidos por um membro da família não serão Federal.
considerados no cálculo da renda familiar mensal per Art. 26-H. No prazo de 10 (dez) anos, contado da
capita de que tratam os §§ 3º e 11-A do art. 20 desta Lei data de publicação desta Seção, será promovida a
para fins de manutenção de benefício de prestação revisão do auxílio-inclusão, observado o disposto no § 2º
continuada concedido anteriormente a outra pessoa do do art. 26-G desta Lei, com vistas a seu aprimoramento e
mesmo grupo familiar. ampliação. (Incluído pela Lei nº 14.176, de 2021) (
§ 4º Para fins de cálculo da renda familiar per CAPÍTULO V
capita de que trata o inciso IV do caput deste artigo, serão Do Financiamento da Assistência Social
desconsideradas:
Art. 27. Fica o Fundo Nacional de Ação
I – as remunerações obtidas pelo requerente em Comunitária (Funac), instituído pelo Decreto nº 91.970, de
decorrência de exercício de atividade laboral, desde que o 22 de novembro de 1985, ratificado pelo Decreto
total recebido no mês seja igual ou inferior a 2 (dois) Legislativo nº 66, de 18 de dezembro de 1990,
salários-mínimos; e transformado no Fundo Nacional de Assistência Social
II – as rendas oriundas dos rendimentos (FNAS).
decorrentes de estágio supervisionado e de Art. 28. O financiamento dos benefícios, serviços,
aprendizagem. programas e projetos estabelecidos nesta lei far-se-á com
Art. 26-B. O auxílio-inclusão será devido a partir da os recursos da União, dos Estados, do Distrito Federal e
data do requerimento, e o seu valor corresponderá a 50% dos Municípios, das demais contribuições sociais
(cinquenta por cento) do valor do benefício de prestação previstas no art. 195 da Constituição Federal, além
continuada em vigor. (Incluído pela Lei nº 14.176, de daqueles que compõem o Fundo Nacional de Assistência
2021) Social (FNAS).
o
Parágrafo único. Ao requerer o auxílio-inclusão, o § 1 Cabe ao órgão da Administração Pública
beneficiário autorizará a suspensão do benefício de responsável pela coordenação da Política de Assistência
prestação continuada, nos termos do art. 21-A desta Lei. Social nas 3 (três) esferas de governo gerir o Fundo de
Art. 26-C. O pagamento do auxílio-inclusão não Assistência Social, sob orientação e controle dos
será acumulado com o pagamento de: (Incluído pela Lei respectivos Conselhos de Assistência Social. (Redação
nº 14.176, de 2021) dada pela Lei nº 12.435, de 2011)
I – benefício de prestação continuada de que trata § 2º O Poder Executivo disporá, no prazo de 180
o art. 20 desta Lei; (cento e oitenta) dias a contar da data de publicação
desta lei, sobre o regulamento e funcionamento do Fundo
II – prestações a título de aposentadoria, de
Nacional de Assistência Social (FNAS).
pensões ou de benefícios por incapacidade pagos por o
qualquer regime de previdência social; ou § 3 O financiamento da assistência social no Suas
deve ser efetuado mediante cofinanciamento dos 3 (três)
III – seguro-desemprego.
entes federados, devendo os recursos alocados nos
Art. 26-D. O pagamento do auxílio-inclusão fundos de assistência social ser voltados à
cessará na hipótese de o beneficiário: (Incluído pela Lei nº operacionalização, prestação, aprimoramento e
14.176, de 2021) viabilização dos serviços, programas, projetos e

www.editoradince.com - Acesse e veja se há novidades a respeito deste material – CUIDADO: cópia é crime.
LEGISLAÇÃO ESPECIAL E REGIMENTO INTERNO DAS UNIDADES DO SEAS 69
benefícios desta política. (Incluído pela Lei nº 12.435, de CAPÍTULO VI
2011) Das Disposições Gerais e Transitórias
Art. 28-A (Revogado pela Lei nº 13.813, de 2019) Art. 31. Cabe ao Ministério Público zelar pelo
Art. 29. Os recursos de responsabilidade da efetivo respeito aos direitos estabelecidos nesta lei.
União destinados à assistência social serão Art. 32. O Poder Executivo terá o prazo de 60
automaticamente repassados ao Fundo Nacional de (sessenta) dias, a partir da publicação desta lei,
Assistência Social (FNAS), à medida que se forem obedecidas as normas por ela instituídas, para elaborar e
realizando as receitas. encaminhar projeto de lei dispondo sobre a extinção e
Parágrafo único. Os recursos de responsabilidade reordenamento dos órgãos de assistência social do
da União destinados ao financiamento dos benefícios de Ministério do Bem-Estar Social.
prestação continuada, previstos no art. 20, poderão ser § 1º O projeto de que trata este artigo definirá
repassados pelo Ministério da Previdência e Assistência formas de transferências de benefícios, serviços,
Social diretamente ao INSS, órgão responsável pela sua programas, projetos, pessoal, bens móveis e imóveis para
execução e manutenção. (Incluído pela Lei nº 9.720, de a esfera municipal.
30.11.1998)
§ 2º O Ministro de Estado do Bem-Estar Social
Art. 30. É condição para os repasses, aos indicará Comissão encarregada de elaborar o projeto de
Municípios, aos Estados e ao Distrito Federal, dos lei de que trata este artigo, que contará com a
recursos de que trata esta lei, a efetiva instituição e participação das organizações dos usuários, de
funcionamento de: trabalhadores do setor e de entidades e organizações de
I - Conselho de Assistência Social, de composição assistência social.
paritária entre governo e sociedade civil; Art. 33. Decorrido o prazo de 120 (cento e vinte)
II - Fundo de Assistência Social, com orientação e dias da promulgação desta lei, fica extinto o Conselho
controle dos respectivos Conselhos de Assistência Social; Nacional de Serviço Social (CNSS), revogando-se, em
III - Plano de Assistência Social. conseqüência, os Decretos-Lei nºs 525, de 1º de julho de
1938, e 657, de 22 de julho de 1943.
Parágrafo único. É, ainda, condição para
transferência de recursos do FNAS aos Estados, ao § 1º O Poder Executivo tomará as providências
Distrito Federal e aos Municípios a comprovação necessárias para a instalação do Conselho Nacional de
orçamentária dos recursos próprios destinados à Assistência Social (CNAS) e a transferência das
Assistência Social, alocados em seus respectivos Fundos atividades que passarão à sua competência dentro do
de Assistência Social, a partir do exercício de 1999. prazo estabelecido no caput, de forma a assegurar não
(Incluído pela Lei nº 9.720, de 30.11.1998) haja solução de continuidade.
Art. 30-A. O cofinanciamento dos serviços, § 2º O acervo do órgão de que trata o caput será
programas, projetos e benefícios eventuais, no que transferido, no prazo de 60 (sessenta) dias, para o
couber, e o aprimoramento da gestão da política de Conselho Nacional de Assistência Social (CNAS), que
assistência social no Suas se efetuam por meio de promoverá, mediante critérios e prazos a serem fixados, a
transferências automáticas entre os fundos de assistência revisão dos processos de registro e certificado de
social e mediante alocação de recursos próprios nesses entidade de fins filantrópicos das entidades e organização
fundos nas 3 (três) esferas de governo. (Incluído pela Lei de assistência social, observado o disposto no art. 3º
nº 12.435, de 2011) desta lei.
Parágrafo único. As transferências automáticas de Art. 34. A União continuará exercendo papel
recursos entre os fundos de assistência social efetuadas à supletivo nas ações de assistência social, por ela
conta do orçamento da seguridade social, conforme o art. atualmente executadas diretamente no âmbito dos
204 da Constituição Federal, caracterizam-se como Estados, dos Municípios e do Distrito Federal, visando à
despesa pública com a seguridade social, na forma do art. implementação do disposto nesta lei, por prazo máximo
o de 12 (doze) meses, contados a partir da data da
24 da Lei Complementar n 101, de 4 de maio de 2000.
(Incluído pela Lei nº 12.435, de 2011) publicação desta lei.
Art. 30-B. Caberá ao ente federado responsável Art. 35. Cabe ao órgão da Administração Pública
pela utilização dos recursos do respectivo Fundo de Federal responsável pela coordenação da Política
Assistência Social o controle e o acompanhamento dos Nacional de Assistência Social operar os benefícios de
serviços, programas, projetos e benefícios, por meio dos prestação continuada de que trata esta lei, podendo, para
respectivos órgãos de controle, independentemente de tanto, contar com o concurso de outros órgãos do
ações do órgão repassador dos recursos. (Incluído pela Governo Federal, na forma a ser estabelecida em
Lei nº 12.435, de 2011) regulamento.
Art. 30-C. A utilização dos recursos federais Parágrafo único. O regulamento de que trata o
descentralizados para os fundos de assistência social dos caput definirá as formas de comprovação do direito ao
Estados, dos Municípios e do Distrito Federal será benefício, as condições de sua suspensão, os
declarada pelos entes recebedores ao ente transferidor, procedimentos em casos de curatela e tutela e o órgão de
anualmente, mediante relatório de gestão submetido à credenciamento, de pagamento e de fiscalização, dentre
apreciação do respectivo Conselho de Assistência Social, outros aspectos.
que comprove a execução das ações na forma de Art. 36. As entidades e organizações de
regulamento. (Incluído pela Lei nº 12.435, de 2011) assistência social que incorrerem em irregularidades na
Parágrafo único. Os entes transferidores poderão aplicação dos recursos que lhes foram repassados pelos
requisitar informações referentes à aplicação dos recursos poderes públicos terão a sua vinculação ao Suas
oriundos do seu fundo de assistência social, para fins de cancelada, sem prejuízo de responsabilidade civil e penal.
análise e acompanhamento de sua boa e regular (Redação dada pela Lei nº 12.435, de 2011)
utilização. (Incluído pela Lei nº 12.435, de 2011) Art. 37. O benefício de prestação continuada será
devido após o cumprimento, pelo requerente, de todos os

www.editoradince.com - Acesse e veja se há novidades a respeito deste material – CUIDADO: cópia é crime.
70 LEGISLAÇÃO ESPECIAL E REGIMENTO INTERNO DAS UNIDADES DO SEAS
requisitos legais e regulamentares exigidos para a sua
concessão, inclusive apresentação da documentação QUESTÕES DE CONCURSOS
necessária, devendo o seu pagamento ser efetuado em
até quarenta e cinco dias após cumpridas as exigências 01. (FUNDATEC - 2021 - Prefeitura de Candelária - RS -
de que trata este artigo. (Redação dada pela Lei nº Assistente Social) Assinale a alternativa correta
9.720, de 30.11.1998) (Vide Lei nº 9.720, de 30.11.1998) acerca das informações extraídas da Lei nº
Parágrafo único. No caso de o primeiro 8.742/1993 (LOAS).
pagamento ser feito após o prazo previsto no caput, A A assistência social, direito do cidadão e dever do
aplicar-se-á na sua atualização o mesmo critério adotado Estado, é Política de Seguridade Social contributiva,
pelo INSS na atualização do primeiro pagamento de que provê as necessidades básicas, realizada através
benefício previdenciário em atraso. (Incluído pela Lei nº de um conjunto integrado de ações de iniciativa
9.720, de 30.11.1998) pública e da sociedade, para garantir o atendimento
Art. 38. (Revogado pela Lei nº 12.435, de 2011) às necessidades básicas.
Art. 39. O Conselho Nacional de Assistência B A assistência social tem por objetivo a descentralização
Social (CNAS), por decisão da maioria absoluta de seus político-administrativa para os Estados, o Distrito
membros, respeitados o orçamento da seguridade social Federal e os Municípios, e comando único das ações
e a disponibilidade do Fundo Nacional de Assistência em cada esfera de governo.
Social (FNAS), poderá propor ao Poder Executivo a C A vigilância socioassistencial visa integrar a rede
alteração dos limites de renda mensal per capita definidos pública e privada de serviços, programas, projetos e
no § 3º do art. 20 e caput do art. 22. benefícios de assistência social.
Art. 40. Com a implantação dos benefícios D A assistência social tem por objetivo a proteção à
previstos nos arts. 20 e 22 desta lei, extinguem-se a renda família, à maternidade, à infância, à adolescência e à
mensal vitalícia, o auxílio-natalidade e o auxílio-funeral velhice.
existentes no âmbito da Previdência Social, conforme o E Cabe à instância coordenadora do Conselho Regional
disposto na Lei nº 8.213, de 24 de julho de 1991. de Serviço Social normatizar e padronizar o emprego
§ 1º A transferência dos benefíciários do sistema e a divulgação da identidade visual da Norma
previdenciário para a assistência social deve ser Operacional Básica/SUAS/2005.
estabelecida de forma que o atendimento à população
não sofra solução de continuidade. (Redação dada pela 02. (AEVSF/FACAPE - 2021 - Prefeitura de Petrolina -
Lei nº 9.711, de 20.11.1998 PE - Psicólogo) A assistência social, conforme a
§ 2º É assegurado ao maior de setenta anos e ao LOAS/1993, tem por objetivos a Proteção Social, que
inválido o direito de requerer a renda mensal vitalícia junto visa à garantia da vida, redução de danos e à
ao INSS até 31 de dezembro de 1995, desde que atenda, prevenção da incidência de riscos, EXCETO:
alternativamente, aos requisitos estabelecidos nos incisos A A garantia de 1 (um) salário-mínimo de benefício
I, II ou III do § 1º do art. 139 da Lei nº 8.213, de 24 de mensal à pessoa com deficiência e ao idoso que
julho de 1991. (Redação dada pela Lei nº 9.711, de comprovem não possuir meios de prover a própria
20.11.1998 manutenção ou de tê-la provida por sua família.
Art. 40-A. Os benefícios monetários decorrentes B A proteção à família, à maternidade, à infância, à
do disposto nos arts. 22, 24-C e 25 desta Lei serão pagos adolescência e à velhice.
preferencialmente à mulher responsável pela unidade
familiar, quando cabível. (Incluído pela Lei nº 13.014, de C O amparo às crianças e aos adolescentes carentes.
2014) D A supremacia do atendimento às necessidades sociais
Art. 40-B. Enquanto não estiver regulamentado o sobre as exigências de rentabilidade econômica.
instrumento de avaliação de que tratam os §§ 1º e 2º do E A habilitação e reabilitação das pessoas com
art. 2º da Lei nº 13.146, de 6 de julho de 2015 (Estatuto deficiência e a promoção de sua integração à vida
da Pessoa com Deficiência), a concessão do benefício de comunitária.
prestação continuada à pessoa com deficiência ficará
sujeita à avaliação do grau da deficiência e do 03. (CEV-URCA - 2021 - Prefeitura de Crato - CE -
impedimento de que trata o § 2º do art. 20 desta Lei, Assistente Social) Consoante a Lei Nº 8.742/1993 e
composta por avaliação médica e avaliação social seus dispositivos em vigência, são objetivos da
realizadas, respectivamente, pela Perícia Médica Federal Assistência Social, com exceção da:
e pelo serviço social do INSS, com a utilização de
A Proteção social, por meio da redução de danos, da
instrumentos desenvolvidos especificamente para esse
prevenção da ocorrência de riscos e da garantia da
fim. (Incluído pela Lei nº 14.176, de 2021)
vida.
Art. 40-C. Os eventuais débitos do beneficiário
B Garantia de um salário-mínimo e meio como forma de
decorrentes de recebimento irregular do benefício de
benefício social àqueles que se enquadrem nos
prestação continuada ou do auxílio-inclusão poderão ser
requisitos legais.
consignados no valor mensal desses benefícios, nos
termos do regulamento. (Incluído pela Lei nº 14.176, de C Defesa de direitos, para o fim de alcançar o pleno
2021) acesso aos direitos no conjunto das provisões socio-
assistenciais.
Art. 41. Esta lei entra em vigor na data da sua
publicação. D Promoção da integração ao mercado de trabalho.
Art. 42. Revogam-se as disposições em contrário. E Vigilância socioassistencial.
Brasília, 7 de dezembro de 1993, 172º da Independência
e 105º da República. 04. (VUNESP - 2021 - Prefeitura de Ferraz de
ITAMAR FRANCO Vasconcelos - SP - Assistente Social) A vigilância
Jutahy Magalhães Júnior socioassistencial está prevista na Lei Orgânica da

www.editoradince.com - Acesse e veja se há novidades a respeito deste material – CUIDADO: cópia é crime.
LEGISLAÇÃO ESPECIAL E REGIMENTO INTERNO DAS UNIDADES DO SEAS 71
Assistência Social (LOAS), que, no seu art. 6° – A – destinam a garantir às pessoas e à coletividade
Parágrafo único, tem como propósitos identificar e condições de bem-estar físico, mental e social.
prevenir situações de vulnerabilidade e risco, TÍTULO II
caracterizando-se como uma importante ferramenta DO SISTEMA ÚNICO DE SAÚDE
de gestão estratégica, estimulando a prática
DISPOSIÇÃO PRELIMINAR
A do registro, planejamento, monitoramento e avaliação
Art. 4º O conjunto de ações e serviços de saúde,
da política.
prestados por órgãos e instituições públicas federais,
B de ações propostas para disseminar a cultura do estaduais e municipais, da Administração direta e indireta
ajustamento. e das fundações mantidas pelo Poder Público, constitui o
C de atitudes voltadas para a distribuição de materiais Sistema Único de Saúde (SUS).
para a população. § 1º Estão incluídas no disposto neste artigo as
D de caráter punitivo e arbitrário em busca da ordem. instituições públicas federais, estaduais e municipais de
E do pensamento positivista e individualista. controle de qualidade, pesquisa e produção de insumos,
medicamentos, inclusive de sangue e hemoderivados, e
de equipamentos para saúde.
Gabarito: 091/D; 02/D; 03/B; 04/A
§ 2º A iniciativa privada poderá participar do
Sistema Único de Saúde (SUS), em caráter
complementar.
CAPÍTULO I
LEI FEDERAL Nº 8.080, DE 19/09/1990, Dos Objetivos e Atribuições
QUE DISPÕE SOBRE AS CONDIÇÕES Art. 5º São objetivos do Sistema Único de Saúde
PARA A PROMOÇÃO, PROTEÇÃO E SUS:
RECUPERAÇÃO DA SAÚDE, A I - a identificação e divulgação dos fatores
ORGANIZAÇÃO E O FUNCIONAMENTO condicionantes e determinantes da saúde;
II - a formulação de política de saúde destinada a
DOS SERVIÇOS CORRESPONDENTES promover, nos campos econômico e social, a
(SUS) E SUAS ALTERAÇÕES. observância do disposto no § 1º do art. 2º desta lei;
Dispõe sobre as condições para a promoção, III - a assistência às pessoas por intermédio de
proteção e recuperação da saúde, a ações de promoção, proteção e recuperação da saúde,
organização e o funcionamento dos serviços com a realização integrada das ações assistenciais e das
correspondentes e dá outras providências. atividades preventivas.
(Atualizada pela Lei nº 14.141, de 2021) Art. 6º Estão incluídas ainda no campo de atuação
O PRESIDENTE DA REPÚBLICA, do Sistema Único de Saúde (SUS):
I - a execução de ações:
Faço saber que o Congresso Nacional decreta e eu
sanciono a seguinte lei: a) de vigilância sanitária;
DISPOSIÇÃO PRELIMINAR b) de vigilância epidemiológica;
Art. 1º Esta lei regula, em todo o território c) de saúde do trabalhador; e
nacional, as ações e serviços de saúde, executados d) de assistência terapêutica integral, inclusive
isolada ou conjuntamente, em caráter permanente ou farmacêutica;
eventual, por pessoas naturais ou jurídicas de direito II - a participação na formulação da política e na
Público ou privado. execução de ações de saneamento básico;
TÍTULO I III - a ordenação da formação de recursos
DAS DISPOSIÇÕES GERAIS humanos na área de saúde;
Art. 2º A saúde é um direito fundamental do ser IV - a vigilância nutricional e a orientação
humano, devendo o Estado prover as condições alimentar;
indispensáveis ao seu pleno exercício.
V - a colaboração na proteção do meio ambiente,
§ 1º O dever do Estado de garantir a saúde nele compreendido o do trabalho;
consiste na formulação e execução de políticas
VI - a formulação da política de medicamentos,
econômicas e sociais que visem à redução de riscos de
equipamentos, imunobiológicos e outros insumos de
doenças e de outros agravos e no estabelecimento de
interesse para a saúde e a participação na sua produção;
condições que assegurem acesso universal e igualitário
às ações e aos serviços para a sua promoção, proteção e VII - o controle e a fiscalização de serviços,
recuperação. produtos e substâncias de interesse para a saúde;
§ 2º O dever do Estado não exclui o das pessoas, VIII - a fiscalização e a inspeção de alimentos,
da família, das empresas e da sociedade. água e bebidas para consumo humano;
Art. 3º A saúde tem como fatores determinantes e IX - a participação no controle e na fiscalização da
condicionantes, entre outros, a alimentação, a moradia, o produção, transporte, guarda e utilização de substâncias
saneamento básico, o meio ambiente, o trabalho, a e produtos psicoativos, tóxicos e radioativos;
renda, a educação, o transporte, o lazer e o acesso aos X - o incremento, em sua área de atuação, do
bens e serviços essenciais; os níveis de saúde da desenvolvimento científico e tecnológico;
população expressam a organização social e econômica XI - a formulação e execução da política de
do País. sangue e seus derivados.
Parágrafo único. Dizem respeito também à saúde § 1º Entende-se por vigilância sanitária um
as ações que, por força do disposto no artigo anterior, se conjunto de ações capaz de eliminar, diminuir ou prevenir

www.editoradince.com - Acesse e veja se há novidades a respeito deste material – CUIDADO: cópia é crime.
72 LEGISLAÇÃO ESPECIAL E REGIMENTO INTERNO DAS UNIDADES DO SEAS
riscos à saúde e de intervir nos problemas sanitários art. 198 da Constituição Federal, obedecendo ainda aos
decorrentes do meio ambiente, da produção e circulação seguintes princípios:
de bens e da prestação de serviços de interesse da I - universalidade de acesso aos serviços de saúde
saúde, abrangendo: em todos os níveis de assistência;
I - o controle de bens de consumo que, direta ou II - integralidade de assistência, entendida como
indiretamente, se relacionem com a saúde, conjunto articulado e contínuo das ações e serviços
compreendidas todas as etapas e processos, da preventivos e curativos, individuais e coletivos, exigidos
produção ao consumo; e para cada caso em todos os níveis de complexidade do
II - o controle da prestação de serviços que se sistema;
relacionam direta ou indiretamente com a saúde. III - preservação da autonomia das pessoas na
§ 2º Entende-se por vigilância epidemiológica um defesa de sua integridade física e moral;
conjunto de ações que proporcionam o conhecimento, a IV - igualdade da assistência à saúde, sem
detecção ou prevenção de qualquer mudança nos fatores preconceitos ou privilégios de qualquer espécie;
determinantes e condicionantes de saúde individual ou
V - direito à informação, às pessoas assistidas,
coletiva, com a finalidade de recomendar e adotar as
sobre sua saúde;
medidas de prevenção e controle das doenças ou
agravos. VI - divulgação de informações quanto ao
potencial dos serviços de saúde e a sua utilização pelo
§ 3º Entende-se por saúde do trabalhador, para
usuário;
fins desta lei, um conjunto de atividades que se destina,
através das ações de vigilância epidemiológica e VII - utilização da epidemiologia para o
vigilância sanitária, à promoção e proteção da saúde dos estabelecimento de prioridades, a alocação de recursos e
trabalhadores, assim como visa à recuperação e a orientação programática;
reabilitação da saúde dos trabalhadores submetidos aos VIII - participação da comunidade;
riscos e agravos advindos das condições de trabalho, IX - descentralização político-administrativa, com
abrangendo: direção única em cada esfera de governo:
I - assistência ao trabalhador vítima de acidentes a) ênfase na descentralização dos serviços para
de trabalho ou portador de doença profissional e do os municípios;
trabalho; b) regionalização e hierarquização da rede de
II - participação, no âmbito de competência do serviços de saúde;
Sistema Único de Saúde (SUS), em estudos, pesquisas, X - integração em nível executivo das ações de
avaliação e controle dos riscos e agravos potenciais à saúde, meio ambiente e saneamento básico;
saúde existentes no processo de trabalho;
XI - conjugação dos recursos financeiros,
III - participação, no âmbito de competência do tecnológicos, materiais e humanos da União, dos
Sistema Único de Saúde (SUS), da normatização, Estados, do Distrito Federal e dos Municípios na
fiscalização e controle das condições de produção, prestação de serviços de assistência à saúde da
extração, armazenamento, transporte, distribuição e população;
manuseio de substâncias, de produtos, de máquinas e de
equipamentos que apresentam riscos à saúde do XII - capacidade de resolução dos serviços em
trabalhador; todos os níveis de assistência; e
IV - avaliação do impacto que as tecnologias XIII - organização dos serviços públicos de modo a
provocam à saúde; evitar duplicidade de meios para fins idênticos.
V - informação ao trabalhador e à sua respectiva XIV – organização de atendimento público
entidade sindical e às empresas sobre os riscos de específico e especializado para mulheres e vítimas de
acidentes de trabalho, doença profissional e do trabalho, violência doméstica em geral, que garanta, entre outros,
bem como os resultados de fiscalizações, avaliações atendimento, acompanhamento psicológico e cirurgias
ambientais e exames de saúde, de admissão, periódicos plásticas reparadoras, em conformidade com a Lei
e de demissão, respeitados os preceitos da ética nº 12.845, de 1º de agosto de 2013. (Redação dada pela Lei
profissional; nº 13.427, de 2017)
VI - participação na normatização, fiscalização e CAPÍTULO III
controle dos serviços de saúde do trabalhador nas Da Organização, da Direção e da Gestão
instituições e empresas públicas e privadas; Art. 8º As ações e serviços de saúde, executados
VII - revisão periódica da listagem oficial de pelo Sistema Único de Saúde (SUS), seja diretamente ou
doenças originadas no processo de trabalho, tendo na mediante participação complementar da iniciativa
sua elaboração a colaboração das entidades sindicais; e privada, serão organizados de forma regionalizada e
hierarquizada em níveis de complexidade crescente.
VIII - a garantia ao sindicato dos trabalhadores de
requerer ao órgão competente a interdição de máquina, Art. 9º A direção do Sistema Único de Saúde
de setor de serviço ou de todo ambiente de trabalho, (SUS) é única, de acordo com o inciso I do art. 198 da
quando houver exposição a risco iminente para a vida ou Constituição Federal, sendo exercida em cada esfera de
saúde dos trabalhadores. governo pelos seguintes órgãos:
CAPÍTULO II I - no âmbito da União, pelo Ministério da Saúde;
Dos Princípios e Diretrizes II - no âmbito dos Estados e do Distrito Federal,
Art. 7º As ações e serviços públicos de saúde e os pela respectiva Secretaria de Saúde ou órgão
serviços privados contratados ou conveniados que equivalente; e
integram o Sistema Único de Saúde (SUS), são III - no âmbito dos Municípios, pela respectiva
desenvolvidos de acordo com as diretrizes previstas no Secretaria de Saúde ou órgão equivalente.

www.editoradince.com - Acesse e veja se há novidades a respeito deste material – CUIDADO: cópia é crime.
LEGISLAÇÃO ESPECIAL E REGIMENTO INTERNO DAS UNIDADES DO SEAS 73
Art. 10. Os municípios poderão constituir Art. 14-B. O Conselho Nacional de Secretários de
consórcios para desenvolver em conjunto as ações e os Saúde (Conass) e o Conselho Nacional de Secretarias
serviços de saúde que lhes correspondam. Municipais de Saúde (Conasems) são reconhecidos
§ 1º Aplica-se aos consórcios administrativos como entidades representativas dos entes estaduais e
intermunicipais o princípio da direção única, e os municipais para tratar de matérias referentes à saúde e
respectivos atos constitutivos disporão sobre sua declarados de utilidade pública e de relevante função
observância. social, na forma do regulamento. (Incluído pela Lei nº
12.466, de 2011).
§ 2º No nível municipal, o Sistema Único de Saúde
(SUS), poderá organizar-se em distritos de forma a § 1° O Conass e o Conasems receberão recursos
integrar e articular recursos, técnicas e práticas voltadas do orçamento geral da União por meio do Fundo
para a cobertura total das ações de saúde. Nacional de Saúde, para auxiliar no custeio de suas
despesas institucionais, podendo ainda celebrar
Art. 11. (Vetado).
convênios com a União. (Incluído pela Lei nº 12.466, de
Art. 12. Serão criadas comissões intersetoriais de 2011).
âmbito nacional, subordinadas ao Conselho Nacional de
§ 2° Os Conselhos de Secretarias Municipais de
Saúde, integradas pelos Ministérios e órgãos
Saúde (Cosems) são reconhecidos como entidades que
competentes e por entidades representativas da
representam os entes municipais, no âmbito estadual,
sociedade civil.
para tratar de matérias referentes à saúde, desde que
Parágrafo único. As comissões intersetoriais terão vinculados institucionalmente ao Conasems, na forma
a finalidade de articular políticas e programas de que dispuserem seus estatutos. (Incluído pela Lei nº
interesse para a saúde, cuja execução envolva áreas não 12.466, de 2011).
compreendidas no âmbito do Sistema Único de Saúde
CAPÍTULO IV
(SUS).
Da Competência e das Atribuições
Art. 13. A articulação das políticas e programas, a
Seção I
cargo das comissões intersetoriais, abrangerá, em
Das Atribuições Comuns
especial, as seguintes atividades:
Art. 15. A União, os Estados, o Distrito Federal e
I - alimentação e nutrição;
os Municípios exercerão, em seu âmbito administrativo,
II - saneamento e meio ambiente; as seguintes atribuições:
III - vigilância sanitária e farmacoepidemiologia; I - definição das instâncias e mecanismos de
IV - recursos humanos; controle, avaliação e de fiscalização das ações e serviços
V - ciência e tecnologia; e de saúde;
VI - saúde do trabalhador. II - administração dos recursos orçamentários e
Art. 14. Deverão ser criadas Comissões financeiros destinados, em cada ano, à saúde;
Permanentes de integração entre os serviços de saúde e III - acompanhamento, avaliação e divulgação do
as instituições de ensino profissional e superior. nível de saúde da população e das condições ambientais;
Parágrafo único. Cada uma dessas comissões terá IV - organização e coordenação do sistema de
por finalidade propor prioridades, métodos e estratégias informação de saúde;
para a formação e educação continuada dos recursos V - elaboração de normas técnicas e
humanos do Sistema Único de Saúde (SUS), na esfera estabelecimento de padrões de qualidade e parâmetros
correspondente, assim como em relação à pesquisa e à de custos que caracterizam a assistência à saúde;
cooperação técnica entre essas instituições. VI - elaboração de normas técnicas e
Art. 14-A. As Comissões Intergestores Bipartite e estabelecimento de padrões de qualidade para promoção
Tripartite são reconhecidas como foros de negociação e da saúde do trabalhador;
pactuação entre gestores, quanto aos aspectos VII - participação de formulação da política e da
operacionais do Sistema Único de Saúde (SUS). (Incluído execução das ações de saneamento básico e
pela Lei nº 12.466, de 2011).
colaboração na proteção e recuperação do meio
Parágrafo único. A atuação das Comissões ambiente;
Intergestores Bipartite e Tripartite terá por objetivo: VIII - elaboração e atualização periódica do plano
(Incluído pela Lei nº 12.466, de 2011).
de saúde;
I - decidir sobre os aspectos operacionais,
IX - participação na formulação e na execução da
financeiros e administrativos da gestão compartilhada do
política de formação e desenvolvimento de recursos
SUS, em conformidade com a definição da política
humanos para a saúde;
consubstanciada em planos de saúde, aprovados pelos
conselhos de saúde; (Incluído pela Lei nº 12.466, de X - elaboração da proposta orçamentária do
2011). Sistema Único de Saúde (SUS), de conformidade com o
plano de saúde;
II - definir diretrizes, de âmbito nacional, regional e
intermunicipal, a respeito da organização das redes de XI - elaboração de normas para regular as
ações e serviços de saúde, principalmente no tocante à atividades de serviços privados de saúde, tendo em vista
sua governança institucional e à integração das ações e a sua relevância pública;
serviços dos entes federados; (Incluído pela Lei nº XII - realização de operações externas de natureza
12.466, de 2011). financeira de interesse da saúde, autorizadas pelo
III - fixar diretrizes sobre as regiões de saúde, Senado Federal;
distrito sanitário, integração de territórios, referência e XIII - para atendimento de necessidades coletivas,
contrarreferência e demais aspectos vinculados à urgentes e transitórias, decorrentes de situações de
integração das ações e serviços de saúde entre os entes perigo iminente, de calamidade pública ou de irrupção de
federados. (Incluído pela Lei nº 12.466, de 2011). epidemias, a autoridade competente da esfera
administrativa correspondente poderá requisitar bens e

www.editoradince.com - Acesse e veja se há novidades a respeito deste material – CUIDADO: cópia é crime.
74 LEGISLAÇÃO ESPECIAL E REGIMENTO INTERNO DAS UNIDADES DO SEAS
serviços, tanto de pessoas naturais como de jurídicas, X - formular, avaliar, elaborar normas e participar
sendo-lhes assegurada justa indenização; na execução da política nacional e produção de insumos
XIV - implementar o Sistema Nacional de Sangue, e equipamentos para a saúde, em articulação com os
Componentes e Derivados; demais órgãos governamentais;
XV - propor a celebração de convênios, acordos e XI - identificar os serviços estaduais e municipais
protocolos internacionais relativos à saúde, saneamento de referência nacional para o estabelecimento de
e meio ambiente; padrões técnicos de assistência à saúde;
XVI - elaborar normas técnico-científicas de XII - controlar e fiscalizar procedimentos, produtos
promoção, proteção e recuperação da saúde; e substâncias de interesse para a saúde;
XVII - promover articulação com os órgãos de XIII - prestar cooperação técnica e financeira aos
fiscalização do exercício profissional e outras entidades Estados, ao Distrito Federal e aos Municípios para o
representativas da sociedade civil para a definição e aperfeiçoamento da sua atuação institucional;
controle dos padrões éticos para pesquisa, ações e XIV - elaborar normas para regular as relações
serviços de saúde; entre o Sistema Único de Saúde (SUS) e os serviços
XVIII - promover a articulação da política e dos privados contratados de assistência à saúde;
planos de saúde; XV - promover a descentralização para as
XIX - realizar pesquisas e estudos na área de Unidades Federadas e para os Municípios, dos serviços
saúde; e ações de saúde, respectivamente, de abrangência
estadual e municipal;
XX - definir as instâncias e mecanismos de
controle e fiscalização inerentes ao poder de polícia XVI - normatizar e coordenar nacionalmente o
sanitária; Sistema Nacional de Sangue, Componentes e Derivados;
XXI - fomentar, coordenar e executar programas e XVII - acompanhar, controlar e avaliar as ações e
projetos estratégicos e de atendimento emergencial. os serviços de saúde, respeitadas as competências
estaduais e municipais;
Seção II
Da Competência XVIII - elaborar o Planejamento Estratégico
Nacional no âmbito do SUS, em cooperação técnica com
Art. 16. A direção nacional do Sistema Único da
os Estados, Municípios e Distrito Federal;
Saúde (SUS) compete:
XIX - estabelecer o Sistema Nacional de Auditoria
I - formular, avaliar e apoiar políticas de
e coordenar a avaliação técnica e financeira do SUS em
alimentação e nutrição;
todo o Território Nacional em cooperação técnica com os
II - participar na formulação e na implementação Estados, Municípios e Distrito Federal. (Vide Decreto nº
das políticas: 1.651, de 1995)
a) de controle das agressões ao meio ambiente; § 1º A União poderá executar ações de vigilância
b) de saneamento básico; e epidemiológica e sanitária em circunstâncias especiais,
c) relativas às condições e aos ambientes de como na ocorrência de agravos inusitados à saúde, que
trabalho; possam escapar do controle da direção estadual do
III - definir e coordenar os sistemas: Sistema Único de Saúde (SUS) ou que representem risco
de disseminação nacional. (Renumerado do parágrafo
a) de redes integradas de assistência de alta único pela Lei nº 14.141, de 2021)
complexidade;
§ 2º Em situações epidemiológicas que
b) de rede de laboratórios de saúde pública; caracterizem emergência em saúde pública, poderá ser
c) de vigilância epidemiológica; e adotado procedimento simplificado para a remessa de
d) vigilância sanitária; patrimônio genético ao exterior, na forma do
IV - participar da definição de normas e regulamento. (Incluído pela Lei nº 14.141, de 2021)
mecanismos de controle, com órgão afins, de agravo § 3º Os benefícios resultantes da exploração
sobre o meio ambiente ou dele decorrentes, que tenham econômica de produto acabado ou material reprodutivo
repercussão na saúde humana; oriundo de acesso ao patrimônio genético de que trata o
V - participar da definição de normas, critérios e § 2º deste artigo serão repartidos nos termos da Lei nº
padrões para o controle das condições e dos ambientes 13.123, de 20 de maio de 2015. (Incluído pela Lei nº 14.141,
de trabalho e coordenar a política de saúde do de 2021)
trabalhador; Art. 17. À direção estadual do Sistema Único de
VI - coordenar e participar na execução das ações Saúde (SUS) compete:
de vigilância epidemiológica; I - promover a descentralização para os Municípios
VII - estabelecer normas e executar a vigilância dos serviços e das ações de saúde;
sanitária de portos, aeroportos e fronteiras, podendo a II - acompanhar, controlar e avaliar as redes
execução ser complementada pelos Estados, Distrito hierarquizadas do Sistema Único de Saúde (SUS);
Federal e Municípios; III - prestar apoio técnico e financeiro aos
VIII - estabelecer critérios, parâmetros e métodos Municípios e executar supletivamente ações e serviços
para o controle da qualidade sanitária de produtos, de saúde;
substâncias e serviços de consumo e uso humano; IV - coordenar e, em caráter complementar,
IX - promover articulação com os órgãos executar ações e serviços:
educacionais e de fiscalização do exercício profissional, a) de vigilância epidemiológica;
bem como com entidades representativas de formação b) de vigilância sanitária;
de recursos humanos na área de saúde;
c) de alimentação e nutrição; e
d) de saúde do trabalhador;

www.editoradince.com - Acesse e veja se há novidades a respeito deste material – CUIDADO: cópia é crime.
LEGISLAÇÃO ESPECIAL E REGIMENTO INTERNO DAS UNIDADES DO SEAS 75
V - participar, junto com os órgãos afins, do XI - controlar e fiscalizar os procedimentos dos
controle dos agravos do meio ambiente que tenham serviços privados de saúde;
repercussão na saúde humana; XII - normatizar complementarmente as ações e
VI - participar da formulação da política e da serviços públicos de saúde no seu âmbito de atuação.
execução de ações de saneamento básico; Art. 19. Ao Distrito Federal competem as
VII - participar das ações de controle e avaliação atribuições reservadas aos Estados e aos Municípios.
das condições e dos ambientes de trabalho; CAPÍTULO V
VIII - em caráter suplementar, formular, executar, Do Subsistema de Atenção à Saúde Indígena
acompanhar e avaliar a política de insumos e (Incluído pela Lei nº 9.836, de 1999)
equipamentos para a saúde; Art. 19-A. As ações e serviços de saúde voltados
IX - identificar estabelecimentos hospitalares de para o atendimento das populações indígenas, em todo o
referência e gerir sistemas públicos de alta território nacional, coletiva ou individualmente,
complexidade, de referência estadual e regional; obedecerão ao disposto nesta Lei. (Incluído pela Lei nº
X - coordenar a rede estadual de laboratórios de 9.836, de 1999)
saúde pública e hemocentros, e gerir as unidades que Art. 19-B. É instituído um Subsistema de Atenção
permaneçam em sua organização administrativa; à Saúde Indígena, componente do Sistema Único de
o
XI - estabelecer normas, em caráter suplementar, Saúde – SUS, criado e definido por esta Lei, e pela Lei n
para o controle e avaliação das ações e serviços de 8.142, de 28 de dezembro de 1990, com o qual
saúde; funcionará em perfeita integração. (Incluído pela Lei nº
9.836, de 1999)
XII - formular normas e estabelecer padrões, em
caráter suplementar, de procedimentos de controle de Art. 19-C. Caberá à União, com seus recursos
qualidade para produtos e substâncias de consumo próprios, financiar o Subsistema de Atenção à Saúde
humano; Indígena. (Incluído pela Lei nº 9.836, de 1999)
XIII - colaborar com a União na execução da Art. 19-D. O SUS promoverá a articulação do
vigilância sanitária de portos, aeroportos e fronteiras; Subsistema instituído por esta Lei com os órgãos
responsáveis pela Política Indígena do País. (Incluído
XIV - o acompanhamento, a avaliação e pela Lei nº 9.836, de 1999)
divulgação dos indicadores de morbidade e mortalidade
no âmbito da unidade federada. Art. 19-E. Os Estados, Municípios, outras
instituições governamentais e não-governamentais
Art. 18. À direção municipal do Sistema de Saúde
poderão atuar complementarmente no custeio e
(SUS) compete: execução das ações. (Incluído pela Lei nº 9.836, de
I - planejar, organizar, controlar e avaliar as ações 1999)
e os serviços de saúde e gerir e executar os serviços § 1º A União instituirá mecanismo de
públicos de saúde; financiamento específico para os Estados, o Distrito
II - participar do planejamento, programação e Federal e os Municípios, sempre que houver necessidade
organização da rede regionalizada e hierarquizada do de atenção secundária e terciária fora dos territórios
Sistema Único de Saúde (SUS), em articulação com sua indígenas. (Incluído pela Lei nº 14.021, de 2020)
direção estadual; § 2º Em situações emergenciais e de calamidade
III - participar da execução, controle e avaliação pública: (Incluído pela Lei nº 14.021, de 2020)
das ações referentes às condições e aos ambientes de
I - a União deverá assegurar aporte adicional de
trabalho; recursos não previstos nos planos de saúde dos Distritos
IV - executar serviços: Sanitários Especiais Indígenas (Dseis) ao Subsistema de
a) de vigilância epidemiológica; Atenção à Saúde Indígena; (Incluído pela Lei nº 14.021, de
b) vigilância sanitária; 2020)
c) de alimentação e nutrição; II - deverá ser garantida a inclusão dos povos
indígenas nos planos emergenciais para atendimento dos
d) de saneamento básico; e
pacientes graves das Secretarias Municipais e Estaduais
e) de saúde do trabalhador; de Saúde, explicitados os fluxos e as referências para o
V - dar execução, no âmbito municipal, à política atendimento em tempo oportuno. (Incluído pela Lei nº
de insumos e equipamentos para a saúde; 14.021, de 2020)
VI - colaborar na fiscalização das agressões ao Art. 19-F. Dever-se-á obrigatoriamente levar em
meio ambiente que tenham repercussão sobre a saúde consideração a realidade local e as especificidades da
humana e atuar, junto aos órgãos municipais, estaduais e cultura dos povos indígenas e o modelo a ser adotado
federais competentes, para controlá-las; para a atenção à saúde indígena, que se deve pautar por
VII - formar consórcios administrativos uma abordagem diferenciada e global, contemplando os
intermunicipais; aspectos de assistência à saúde, saneamento básico,
VIII - gerir laboratórios públicos de saúde e nutrição, habitação, meio ambiente, demarcação de
hemocentros; terras, educação sanitária e integração institucional.
(Incluído pela Lei nº 9.836, de 1999)
IX - colaborar com a União e os Estados na
Art. 19-G. O Subsistema de Atenção à Saúde
execução da vigilância sanitária de portos, aeroportos e
Indígena deverá ser, como o SUS, descentralizado,
fronteiras;
hierarquizado e regionalizado. (Incluído pela Lei nº 9.836,
X - observado o disposto no art. 26 desta Lei, de 1999)
celebrar contratos e convênios com entidades o
§ 1 O Subsistema de que trata o caput deste
prestadoras de serviços privados de saúde, bem como
artigo terá como base os Distritos Sanitários Especiais
controlar e avaliar sua execução;
Indígenas. (Incluído pela Lei nº 9.836, de 1999)

www.editoradince.com - Acesse e veja se há novidades a respeito deste material – CUIDADO: cópia é crime.
76 LEGISLAÇÃO ESPECIAL E REGIMENTO INTERNO DAS UNIDADES DO SEAS
o
§ 1º-A. A rede do SUS deverá obrigatoriamente § 2 As ações destinadas a viabilizar o pleno
fazer o registro e a notificação da declaração de raça ou exercício dos direitos de que trata este artigo constarão
cor, garantindo a identificação de todos os indígenas do regulamento da lei, a ser elaborado pelo órgão
atendidos nos sistemas públicos de saúde. § 1º-B. A competente do Poder Executivo. (Incluído pela Lei nº
União deverá integrar os sistemas de informação da rede 11.108, de 2005)
do SUS com os dados do Subsistema de Atenção à Art. 19-L. (VETADO) (Incluído pela Lei nº 11.108, de
Saúde Indígena. (Incluído pela Lei nº 14.021, de 2020) 2005)
§ 1º-B. A União deverá integrar os sistemas de CAPÍTULO VIII
informação da rede do SUS com os dados do (Incluído pela Lei nº 12.401, de 2011)
Subsistema de Atenção à Saúde Indígena. (Incluído pela
DA ASSISTÊNCIA TERAPÊUTICA E DA
Lei nº 14.021, de 2020)
o INCORPORAÇÃO DE
§ 2 O SUS servirá de retaguarda e referência ao
TECNOLOGIA EM SAÚDE”
Subsistema de Atenção à Saúde Indígena, devendo, para
isso, ocorrer adaptações na estrutura e organização do Art. 19-M. A assistência terapêutica integral a que
SUS nas regiões onde residem as populações indígenas, se refere a alínea d do inciso I do art. 6o consiste
para propiciar essa integração e o atendimento em: (Incluído pela Lei nº 12.401, de 2011)
necessário em todos os níveis, sem discriminações. I - dispensação de medicamentos e produtos de
(Incluído pela Lei nº 9.836, de 1999) interesse para a saúde, cuja prescrição esteja em
o conformidade com as diretrizes terapêuticas definidas em
§ 3 As populações indígenas devem ter acesso
garantido ao SUS, em âmbito local, regional e de centros protocolo clínico para a doença ou o agravo à saúde a
especializados, de acordo com suas necessidades, ser tratado ou, na falta do protocolo, em conformidade
compreendendo a atenção primária, secundária e com o disposto no art. 19-P; (Incluído pela Lei nº 12.401,
terciária à saúde. (Incluído pela Lei nº 9.836, de 1999) de 2011)
Art. 19-H. As populações indígenas terão direito a II - oferta de procedimentos terapêuticos, em
participar dos organismos colegiados de formulação, regime domiciliar, ambulatorial e hospitalar, constantes
acompanhamento e avaliação das políticas de saúde, tais de tabelas elaboradas pelo gestor federal do Sistema
como o Conselho Nacional de Saúde e os Conselhos Único de Saúde - SUS, realizados no território nacional
Estaduais e Municipais de Saúde, quando for o por serviço próprio, conveniado ou contratado.
caso. (Incluído pela Lei nº 9.836, de 1999) Art. 19-N. Para os efeitos do disposto no art. 19-
CAPÍTULO VI M, são adotadas as seguintes definições:
DO SUBSISTEMA DE ATENDIMENTO E INTERNAÇÃO I - produtos de interesse para a saúde: órteses,
DOMICILIAR próteses, bolsas coletoras e equipamentos médicos;
(Incluído pela Lei nº 10.424, de 2002) II - protocolo clínico e diretriz terapêutica:
Art. 19-I. São estabelecidos, no âmbito do Sistema documento que estabelece critérios para o diagnóstico da
Único de Saúde, o atendimento domiciliar e a internação doença ou do agravo à saúde; o tratamento preconizado,
domiciliar. (Incluído pela Lei nº 10.424, de 2002) com os medicamentos e demais produtos apropriados,
o quando couber; as posologias recomendadas; os
§ 1 Na modalidade de assistência de atendimento
e internação domiciliares incluem-se, principalmente, os mecanismos de controle clínico; e o acompanhamento e
procedimentos médicos, de enfermagem, a verificação dos resultados terapêuticos, a serem
fisioterapêuticos, psicológicos e de assistência social, seguidos pelos gestores do SUS. (Incluído pela Lei nº
entre outros necessários ao cuidado integral dos 12.401, de 2011)
pacientes em seu domicílio. (Incluído pela Lei nº 10.424, Art. 19-O. Os protocolos clínicos e as diretrizes
de 2002) terapêuticas deverão estabelecer os medicamentos ou
o produtos necessários nas diferentes fases evolutivas da
§ 2 O atendimento e a internação domiciliares
serão realizados por equipes multidisciplinares que doença ou do agravo à saúde de que tratam, bem como
atuarão nos níveis da medicina preventiva, terapêutica e aqueles indicados em casos de perda de eficácia e de
reabilitadora. (Incluído pela Lei nº 10.424, de 2002) surgimento de intolerância ou reação adversa relevante,
o
§ 3 O atendimento e a internação domiciliares só provocadas pelo medicamento, produto ou procedimento
poderão ser realizados por indicação médica, com de primeira escolha. (Incluído pela Lei nº 12.401, de
expressa concordância do paciente e de sua família. 2011)
(Incluído pela Lei nº 10.424, de 2002) Parágrafo único. Em qualquer caso, os
CAPÍTULO VII medicamentos ou produtos de que trata o caput deste
DO SUBSISTEMA DE ACOMPANHAMENTO DURANTE artigo serão aqueles avaliados quanto à sua eficácia,
O segurança, efetividade e custo-efetividade para as
TRABALHO DE PARTO, PARTO E PÓS-PARTO diferentes fases evolutivas da doença ou do agravo à
IMEDIATO saúde de que trata o protocolo. (Incluído pela Lei nº
(Incluído pela Lei nº 11.108, de 2005) 12.401, de 2011)
Art. 19-J. Os serviços de saúde do Sistema Único Art. 19-P. Na falta de protocolo clínico ou de
de Saúde - SUS, da rede própria ou conveniada, ficam diretriz terapêutica, a dispensação será
obrigados a permitir a presença, junto à parturiente, de 1 realizada: (Incluído pela Lei nº 12.401, de 2011)
(um) acompanhante durante todo o período de trabalho I - com base nas relações de medicamentos
de parto, parto e pós-parto imediato. (Incluído pela Lei nº instituídas pelo gestor federal do SUS, observadas as
11.108, de 2005) competências estabelecidas nesta Lei, e a
o
§ 1 O acompanhante de que trata o caput deste responsabilidade pelo fornecimento será pactuada na
artigo será indicado pela parturiente. (Incluído pela Lei nº Comissão Intergestores Tripartite; (Incluído pela Lei nº
11.108, de 2005) 12.401, de 2011)

www.editoradince.com - Acesse e veja se há novidades a respeito deste material – CUIDADO: cópia é crime.
LEGISLAÇÃO ESPECIAL E REGIMENTO INTERNO DAS UNIDADES DO SEAS 77
II - no âmbito de cada Estado e do Distrito Federal, § 2o (VETADO). (Incluído pela Lei nº 12.401, de
de forma suplementar, com base nas relações de 2011)
medicamentos instituídas pelos gestores estaduais do Art. 19-S. (VETADO). (Incluído pela Lei nº 12.401,
SUS, e a responsabilidade pelo fornecimento será de 2011)
pactuada na Comissão Intergestores Bipartite; (Incluído
Art. 19-T. São vedados, em todas as esferas de
pela Lei nº 12.401, de 2011)
gestão do SUS: (Incluído pela Lei nº 12.401, de 2011)
III - no âmbito de cada Município, de forma
I - o pagamento, o ressarcimento ou o reembolso
suplementar, com base nas relações de medicamentos
de medicamento, produto e procedimento clínico ou
instituídas pelos gestores municipais do SUS, e a
cirúrgico experimental, ou de uso não autorizado pela
responsabilidade pelo fornecimento será pactuada no
Agência Nacional de Vigilância Sanitária -
Conselho Municipal de Saúde. (Incluído pela Lei nº
ANVISA; (Incluído pela Lei nº 12.401, de 2011)
12.401, de 2011)
II - a dispensação, o pagamento, o ressarcimento
Art. 19-Q. A incorporação, a exclusão ou a
ou o reembolso de medicamento e produto, nacional ou
alteração pelo SUS de novos medicamentos, produtos e
importado, sem registro na Anvisa.”
procedimentos, bem como a constituição ou a alteração
de protocolo clínico ou de diretriz terapêutica, são Art. 19-U. A responsabilidade financeira pelo
atribuições do Ministério da Saúde, assessorado pela fornecimento de medicamentos, produtos de interesse
Comissão Nacional de Incorporação de Tecnologias no para a saúde ou procedimentos de que trata este
SUS. (Incluído pela Lei nº 12.401, de 2011) Capítulo será pactuada na Comissão Intergestores
Tripartite. (Incluído pela Lei nº 12.401, de 2011)
§ 1o A Comissão Nacional de Incorporação de
Tecnologias no SUS, cuja composição e regimento são TÍTULO III
definidos em regulamento, contará com a participação de DOS SERVIÇOS PRIVADOS DE ASSISTÊNCIA À
1 (um) representante indicado pelo Conselho Nacional de SAÙDE
Saúde e de 1 (um) representante, especialista na área, CAPÍTULO I
indicado pelo Conselho Federal de Medicina. (Incluído Do Funcionamento
pela Lei nº 12.401, de 2011) Art. 20. Os serviços privados de assistência à
§ 2o O relatório da Comissão Nacional de saúde caracterizam-se pela atuação, por iniciativa
Incorporação de Tecnologias no SUS levará em própria, de profissionais liberais, legalmente habilitados, e
consideração, necessariamente: (Incluído pela Lei nº de pessoas jurídicas de direito privado na promoção,
12.401, de 2011) proteção e recuperação da saúde.
I - as evidências científicas sobre a eficácia, a Art. 21. A assistência à saúde é livre à iniciativa
acurácia, a efetividade e a segurança do medicamento, privada.
produto ou procedimento objeto do processo, acatadas Art. 22. Na prestação de serviços privados de
pelo órgão competente para o registro ou a autorização assistência à saúde, serão observados os princípios
de uso; (Incluído pela Lei nº 12.401, de 2011) éticos e as normas expedidas pelo órgão de direção do
II - a avaliação econômica comparativa dos Sistema Único de Saúde (SUS) quanto às condições
benefícios e dos custos em relação às tecnologias já para seu funcionamento.
incorporadas, inclusive no que se refere aos Art. 23. É permitida a participação direta ou
atendimentos domiciliar, ambulatorial ou hospitalar, indireta, inclusive controle, de empresas ou de capital
quando cabível. (Incluído pela Lei nº 12.401, de 2011) estrangeiro na assistência à saúde nos seguintes
Art. 19-R. A incorporação, a exclusão e a casos: (Redação dada pela Lei nº 13.097, de 2015)
alteração a que se refere o art. 19-Q serão efetuadas I - doações de organismos internacionais
mediante a instauração de processo administrativo, a ser vinculados à Organização das Nações Unidas, de
concluído em prazo não superior a 180 (cento e oitenta) entidades de cooperação técnica e de financiamento e
dias, contado da data em que foi protocolado o pedido, empréstimos; (Incluído pela Lei nº 13.097, de 2015)
admitida a sua prorrogação por 90 (noventa) dias II - pessoas jurídicas destinadas a instalar,
corridos, quando as circunstâncias exigirem. (Incluído operacionalizar ou explorar: (Incluído pela Lei nº 13.097, de
pela Lei nº 12.401, de 2011) 2015)
§ 1o O processo de que trata o caput deste artigo a) hospital geral, inclusive filantrópico, hospital
observará, no que couber, o disposto na Lei no 9.784, de especializado, policlínica, clínica geral e clínica
29 de janeiro de 1999, e as seguintes determinações especializada; e (Incluído pela Lei nº 13.097, de 2015)
especiais: (Incluído pela Lei nº 12.401, de 2011)
b) ações e pesquisas de planejamento familiar;
I - apresentação pelo interessado dos documentos (Incluído pela Lei nº 13.097, de 2015)
e, se cabível, das amostras de produtos, na forma do
regulamento, com informações necessárias para o III - serviços de saúde mantidos, sem finalidade
atendimento do disposto no § 2o do art. 19-Q; (Incluído lucrativa, por empresas, para atendimento de seus
pela Lei nº 12.401, de 2011) empregados e dependentes, sem qualquer ônus para a
seguridade social; e (Incluído pela Lei nº 13.097, de 2015)
II - (VETADO); (Incluído pela Lei nº 12.401, de
2011) IV - demais casos previstos em legislação
específica. (Incluído pela Lei nº 13.097, de 2015)
III - realização de consulta pública que inclua a
divulgação do parecer emitido pela Comissão Nacional CAPÍTULO II
de Incorporação de Tecnologias no SUS; (Incluído pela Da Participação Complementar
Lei nº 12.401, de 2011) Art. 24. Quando as suas disponibilidades forem
IV - realização de audiência pública, antes da insuficientes para garantir a cobertura assistencial à
tomada de decisão, se a relevância da matéria justificar o população de uma determinada área, o Sistema Único de
evento. (Incluído pela Lei nº 12.401, de 2011) Saúde (SUS) poderá recorrer aos serviços ofertados pela
iniciativa privada.

www.editoradince.com - Acesse e veja se há novidades a respeito deste material – CUIDADO: cópia é crime.
78 LEGISLAÇÃO ESPECIAL E REGIMENTO INTERNO DAS UNIDADES DO SEAS
Parágrafo único. A participação complementar dos TÍTULO V
serviços privados será formalizada mediante contrato ou DO FINANCIAMENTO
convênio, observadas, a respeito, as normas de direito CAPÍTULO I
público. Dos Recursos
Art. 25. Na hipótese do artigo anterior, as Art. 31. O orçamento da seguridade social
entidades filantrópicas e as sem fins lucrativos terão destinará ao Sistema Único de Saúde (SUS) de acordo
preferência para participar do Sistema Único de Saúde com a receita estimada, os recursos necessários à
(SUS). realização de suas finalidades, previstos em proposta
Art. 26. Os critérios e valores para a remuneração elaborada pela sua direção nacional, com a participação
de serviços e os parâmetros de cobertura assistencial dos órgãos da Previdência Social e da Assistência Social,
serão estabelecidos pela direção nacional do Sistema tendo em vista as metas e prioridades estabelecidas na
Único de Saúde (SUS), aprovados no Conselho Nacional Lei de Diretrizes Orçamentárias.
de Saúde. Art. 32. São considerados de outras fontes os
§ 1° Na fixação dos critérios, valores, formas de recursos provenientes de:
reajuste e de pagamento da remuneração aludida neste I - (Vetado)
artigo, a direção nacional do Sistema Único de Saúde
II - Serviços que possam ser prestados sem
(SUS) deverá fundamentar seu ato em demonstrativo
prejuízo da assistência à saúde;
econômico-financeiro que garanta a efetiva qualidade de
execução dos serviços contratados. III - ajuda, contribuições, doações e donativos;
§ 2° Os serviços contratados submeter-se-ão às IV - alienações patrimoniais e rendimentos de
normas técnicas e administrativas e aos princípios e capital;
diretrizes do Sistema Único de Saúde (SUS), mantido o V - taxas, multas, emolumentos e preços públicos
equilíbrio econômico e financeiro do contrato. arrecadados no âmbito do Sistema Único de Saúde
§ 3° (Vetado). (SUS); e
§ 4° Aos proprietários, administradores e VI - rendas eventuais, inclusive comerciais e
dirigentes de entidades ou serviços contratados é vedado industriais.
exercer cargo de chefia ou função de confiança no § 1° Ao Sistema Único de Saúde (SUS) caberá
Sistema Único de Saúde (SUS). metade da receita de que trata o inciso I deste artigo,
TÍTULO IV apurada mensalmente, a qual será destinada à
DOS RECURSOS HUMANOS recuperação de viciados.
Art. 27. A política de recursos humanos na área § 2° As receitas geradas no âmbito do Sistema
da saúde será formalizada e executada, articuladamente, Único de Saúde (SUS) serão creditadas diretamente em
pelas diferentes esferas de governo, em cumprimento contas especiais, movimentadas pela sua direção, na
dos seguintes objetivos: esfera de poder onde forem arrecadadas.
I - organização de um sistema de formação de § 3º As ações de saneamento que venham a ser
recursos humanos em todos os níveis de ensino, executadas supletivamente pelo Sistema Único de Saúde
inclusive de pós-graduação, além da elaboração de (SUS), serão financiadas por recursos tarifários
programas de permanente aperfeiçoamento de pessoal; específicos e outros da União, Estados, Distrito Federal,
Municípios e, em particular, do Sistema Financeiro da
II - (Vetado)
Habitação (SFH).
III - (Vetado)
§ 4º (Vetado).
IV - valorização da dedicação exclusiva aos
§ 5º As atividades de pesquisa e desenvolvimento
serviços do Sistema Único de Saúde (SUS).
científico e tecnológico em saúde serão co-financiadas
Parágrafo único. Os serviços públicos que pelo Sistema Único de Saúde (SUS), pelas universidades
integram o Sistema Único de Saúde (SUS) constituem e pelo orçamento fiscal, além de recursos de instituições
campo de prática para ensino e pesquisa, mediante de fomento e financiamento ou de origem externa e
normas específicas, elaboradas conjuntamente com o receita própria das instituições executoras.
sistema educacional.
§ 6º (Vetado).
Art. 28. Os cargos e funções de chefia, direção e
CAPÍTULO II
assessoramento, no âmbito do Sistema Único de Saúde
Da Gestão Financeira
(SUS), só poderão ser exercidas em regime de tempo
integral. Art. 33. Os recursos financeiros do Sistema Único
de Saúde (SUS) serão depositados em conta especial,
§ 1° Os servidores que legalmente acumulam dois
em cada esfera de sua atuação, e movimentados sob
cargos ou empregos poderão exercer suas atividades em
fiscalização dos respectivos Conselhos de Saúde.
mais de um estabelecimento do Sistema Único de Saúde
(SUS). § 1º Na esfera federal, os recursos financeiros,
originários do Orçamento da Seguridade Social, de
§ 2° O disposto no parágrafo anterior aplica-se
outros Orçamentos da União, além de outras fontes,
também aos servidores em regime de tempo integral,
serão administrados pelo Ministério da Saúde, através do
com exceção dos ocupantes de cargos ou função de
Fundo Nacional de Saúde.
chefia, direção ou assessoramento.
§ 2º (Vetado).
Art. 29. (Vetado).
§ 3º (Vetado).
Art. 30. As especializações na forma de
treinamento em serviço sob supervisão serão § 4º O Ministério da Saúde acompanhará, através
regulamentadas por Comissão Nacional, instituída de de seu sistema de auditoria, a conformidade à
acordo com o art. 12 desta Lei, garantida a participação programação aprovada da aplicação dos recursos
das entidades profissionais correspondentes. repassados a Estados e Municípios. Constatada a
malversação, desvio ou não aplicação dos recursos,

www.editoradince.com - Acesse e veja se há novidades a respeito deste material – CUIDADO: cópia é crime.
LEGISLAÇÃO ESPECIAL E REGIMENTO INTERNO DAS UNIDADES DO SEAS 79
caberá ao Ministério da Saúde aplicar as medidas Art. 37. O Conselho Nacional de Saúde
previstas em lei. estabelecerá as diretrizes a serem observadas na
Art. 34. As autoridades responsáveis pela elaboração dos planos de saúde, em função das
distribuição da receita efetivamente arrecadada características epidemiológicas e da organização dos
transferirão automaticamente ao Fundo Nacional de serviços em cada jurisdição administrativa.
Saúde (FNS), observado o critério do parágrafo único Art. 38. Não será permitida a destinação de
deste artigo, os recursos financeiros correspondentes às subvenções e auxílios a instituições prestadoras de
dotações consignadas no Orçamento da Seguridade serviços de saúde com finalidade lucrativa.
Social, a projetos e atividades a serem executados no DAS DISPOSIÇÕES FINAIS E TRANSITÓRIAS
âmbito do Sistema Único de Saúde (SUS).
Art. 39. (Vetado).
Parágrafo único. Na distribuição dos recursos
§ 1º (Vetado).
financeiros da Seguridade Social será observada a
mesma proporção da despesa prevista de cada área, no § 2º (Vetado).
Orçamento da Seguridade Social. § 3º (Vetado).
Art. 35. Para o estabelecimento de valores a § 4º (Vetado).
serem transferidos a Estados, Distrito Federal e § 5º A cessão de uso dos imóveis de propriedade
Municípios, será utilizada a combinação dos seguintes do Inamps para órgãos integrantes do Sistema Único de
critérios, segundo análise técnica de programas e Saúde (SUS) será feita de modo a preservá-los como
projetos: patrimônio da Seguridade Social.
I - perfil demográfico da região; § 6º Os imóveis de que trata o parágrafo anterior
II - perfil epidemiológico da população a ser serão inventariados com todos os seus acessórios,
coberta; equipamentos e outros
III - características quantitativas e qualitativas da § 7º (Vetado).
rede de saúde na área; § 8º O acesso aos serviços de informática e bases
IV - desempenho técnico, econômico e financeiro de dados, mantidos pelo Ministério da Saúde e pelo
no período anterior; Ministério do Trabalho e da Previdência Social, será
V - níveis de participação do setor saúde nos assegurado às Secretarias Estaduais e Municipais de
orçamentos estaduais e municipais; Saúde ou órgãos congêneres, como suporte ao processo
de gestão, de forma a permitir a gerencia informatizada
VI - previsão do plano quinquenal de investimentos
das contas e a disseminação de estatísticas sanitárias e
da rede;
epidemiológicas médico-hospitalares.
VII - ressarcimento do atendimento a serviços
Art. 40. (Vetado)
prestados para outras esferas de governo.
Art. 41. As ações desenvolvidas pela Fundação
§ 1º (Revogado pela Lei Complementar nº 141, de
das Pioneiras Sociais e pelo Instituto Nacional do Câncer,
2012)
supervisionadas pela direção nacional do Sistema Único
§ 2º Nos casos de Estados e Municípios sujeitos a de Saúde (SUS), permanecerão como referencial de
notório processo de migração, os critérios demográficos prestação de serviços, formação de recursos humanos e
mencionados nesta lei serão ponderados por outros para transferência de tecnologia.
indicadores de crescimento populacional, em especial o
Art. 42. (Vetado).
número de eleitores registrados.
Art. 43. A gratuidade das ações e serviços de
§ 3º (Vetado).
saúde fica preservada nos serviços públicos contratados,
§ 4º (Vetado). ressalvando-se as cláusulas dos contratos ou convênios
§ 5º (Vetado). estabelecidos com as entidades privadas.
§ 6º O disposto no parágrafo anterior não Art. 44. (Vetado).
prejudica a atuação dos órgãos de controle interno e Art. 45. Os serviços de saúde dos hospitais
externo e nem a aplicação de penalidades previstas em universitários e de ensino integram-se ao Sistema Único
lei, em caso de irregularidades verificadas na gestão dos de Saúde (SUS), mediante convênio, preservada a sua
recursos transferidos. autonomia administrativa, em relação ao patrimônio, aos
CAPÍTULO III recursos humanos e financeiros, ensino, pesquisa e
Do Planejamento e do Orçamento extensão nos limites conferidos pelas instituições a que
Art. 36. O processo de planejamento e orçamento estejam vinculados.
do Sistema Único de Saúde (SUS) será ascendente, do § 1º Os serviços de saúde de sistemas estaduais e
nível local até o federal, ouvidos seus órgãos municipais de previdência social deverão integrar-se à
deliberativos, compatibilizando-se as necessidades da direção correspondente do Sistema Único de Saúde
política de saúde com a disponibilidade de recursos em (SUS), conforme seu âmbito de atuação, bem como
planos de saúde dos Municípios, dos Estados, do Distrito quaisquer outros órgãos e serviços de saúde.
Federal e da União. § 2º Em tempo de paz e havendo interesse
§ 1º Os planos de saúde serão a base das recíproco, os serviços de saúde das Forças Armadas
atividades e programações de cada nível de direção do poderão integrar-se ao Sistema Único de Saúde (SUS),
Sistema Único de Saúde (SUS), e seu financiamento conforme se dispuser em convênio que, para esse fim,
será previsto na respectiva proposta orçamentária. for firmado.
§ 2º É vedada a transferência de recursos para o Art. 46. O Sistema Único de Saúde (SUS),
financiamento de ações não previstas nos planos de estabelecerá mecanismos de incentivos à participação do
saúde, exceto em situações emergenciais ou de setor privado no investimento em ciência e tecnologia e
calamidade pública, na área de saúde. estimulará a transferência de tecnologia das
universidades e institutos de pesquisa aos serviços de

www.editoradince.com - Acesse e veja se há novidades a respeito deste material – CUIDADO: cópia é crime.
80 LEGISLAÇÃO ESPECIAL E REGIMENTO INTERNO DAS UNIDADES DO SEAS
saúde nos Estados, Distrito Federal e Municípios, e às e aos serviços para a sua promoção, proteção e
empresas nacionais. recuperação.
Art. 47. O Ministério da Saúde, em articulação
com os níveis estaduais e municipais do Sistema Único 02. (GS Assessoria e Concursos - 2021 - Prefeitura de
de Saúde (SUS), organizará, no prazo de dois anos, um Entre Rios - SC - Agente Comunitário de Saúde)
sistema nacional de informações em saúde, integrado em Responda a questão de acordo com a Lei nº
todo o território nacional, abrangendo questões 8080/1990
epidemiológicas e de prestação de serviços. De acordo com o Art. 5º São objetivos do SUS:
Art. 48. (Vetado). I - a identificação e divulgação dos fatores condicionantes
Art. 49. (Vetado). e determinantes da saúde;
Art. 50. Os convênios entre a União, os Estados e II - a formulação de política de saúde destinada a
promover, nos campos econômico e social, a
os Municípios, celebrados para implantação dos
observância do disposto no § 1º do art. 2º desta lei;
Sistemas Unificados e Descentralizados de Saúde,
ficarão rescindidos à proporção que seu objeto for sendo III - a assistência às pessoas por intermédio de ações de
absorvido pelo Sistema Único de Saúde (SUS). promoção, proteção e recuperação da saúde, com a
realização integrada das ações assistenciais e das
Art. 51. (Vetado).
atividades preventivas. Marque a alternativa correta.
Art. 52. Sem prejuízo de outras sanções cabíveis, A Apenas o item I está correto.
constitui crime de emprego irregular de verbas ou rendas B Apenas o item II está correto.
públicas (Código Penal, art. 315) a utilização de recursos
C Apenas o item III está correto.
financeiros do Sistema Único de Saúde (SUS) em
finalidades diversas das previstas nesta lei. D Todos os itens estão corretos.
Art. 53. (Vetado).
03. (OMNI - 2021 - Prefeitura de Lençóis Paulista - SP -
Art. 53-A. Na qualidade de ações e serviços de Agente Comunitário de Saúde) Conforme explicitado
saúde, as atividades de apoio à assistência à saúde são no Art. 3° da Lei Org nica da Saúde: Os níveis de
aquelas desenvolvidas pelos laboratórios de genética saúde expressam a organi ação social e econ mica
humana, produção e fornecimento de medicamentos e do País, tendo a saúde como determinantes e
produtos para saúde, laboratórios de analises clínicas, condicionantes, entre outros, a alimentação, a
anatomia patológica e de diagnóstico por imagem e são moradia, o saneamento básico, o meio ambiente, o
livres à participação direta ou indireta de empresas ou de trabalho, a renda, a educação, a atividade física, o
capitais estrangeiros. (Incluído pela Lei nº 13.097, de 2015) transporte, o la er e o acesso aos bens e serviços
Art. 54. Esta lei entra em vigor na data de sua essenciais. Di em respeito tamb m saúde as ações
publicação. que, por força do disposto nesse artigo, se destinam a
Art. 55. São revogadas a Lei nº. 2.312, de 3 de garantir s pessoas e coletividade condições de:
setembro de 1954, a Lei nº. 6.229, de 17 de julho de A Bem-estar físico, mental e econ mico.
1975, e demais disposições em contrário. B Bem-estar físico, mental e social.
Brasília, 19 de setembro de 1990; 169º da Independência e 102º C Bem-estar físico, mental e cultural.
da República. D Bem-estar físico, ambiental e social.
FERNANDO COLLOR
Alceni Guerra
04. (VUNESP - 2021 - Prefeitura de Marília - SP -
Este texto não substitui o publicado no D.O.U. de 20.9.1990
Supervisor de Saúde) O processo de transferência de
responsabilidades de gestão de saúde para os
QUESTÕES DE CONCURSOS municípios, atendendo às determinações
01. (AMAUC - 2021 - Prefeitura de Lindóia do Sul - SC - constitucionais e legais, é o que caracteriza um
Agente Comunitário de Saúde) De acordo com a lei importante princípio do SUS denominado
orgânica de saúde 8080/90, o dever do Estado de A Equidade.
garantir a saúde consiste: B Universalidade.
A Na organização social e econômica do País, tendo a C Participação Social.
saúde como determinantes e condicionantes, entre D Descentralização.
outros, a alimentação, a moradia, o saneamento E Integralidade.
básico, o meio ambiente, o trabalho, a renda, a
educação, a atividade física, o transporte, o lazer e o 05. (VUNESP - 2021 - Prefeitura de Marília - SP -
acesso aos bens e serviços essenciais. Supervisor de Saúde) Assinale a alternativa correta a
B De ações e serviços de saúde, prestados por órgãos e respeito dos Conselhos de Saúde.
instituições públicas federais, estaduais e municipais, A Atuam conjuntamente com os órgãos responsáveis pela
da Administração direta e indireta e das fundações gestão e execução de serviços, compartilhando a
mantidas pelo Poder Público. prestação dos serviços de saúde.
C Na identificação e divulgação dos fatores B Representam a sociedade, e sua composição conta
condicionantes e determinantes da saúde. com 50% de representantes de usuários do SUS.
D Na formulação da política de medicamentos, C Têm caráter permanente, e sua extinção só pode ser
equipamentos, imunobiológicos e outros insumos de realizada por meio de decreto do Executivo.
interesse para a saúde e a participação na sua D São consultivos, e suas decisões são encaminhadas ao
produção. poder público como recomendação.
E Na formulação e execução de políticas econômicas e E São subordinados ao Poder Executivo, tendo os
sociais que visem à redução de riscos de doenças e Secretários de Saúde como presidentes.
de outros agravos e no estabelecimento de condições
que assegurem acesso universal e igualitário às ações

www.editoradince.com - Acesse e veja se há novidades a respeito deste material – CUIDADO: cópia é crime.
LEGISLAÇÃO ESPECIAL E REGIMENTO INTERNO DAS UNIDADES DO SEAS 81
06. (VUNESP - 2021 - Prefeitura de Marília - SP - como prazo máximo para a permanência do adolescente
Supervisor de Saúde) Todo cidadão, sem qualquer na unidade de recepção 24 (vinte quatro) horas.
tipo de discriminação, incluindo os estrangeiros e § 2º A medida socioeducativa de internação
refugiados, tem acesso ao Sistema Único de Saúde. provisória é aplicada ao adolescente, antes da sentença,
Trata de um princípio do SUS denominado a teor do artigo 108 do ECA, e não deve ultrapassar o
A Equidade. prazo máximo de 45 dias.
B Universalidade. § 3º A medida socioeducativa de semiliberdade
C Integralidade. pode ser aplicada como primeira medida, ou como forma
D Participação Social. de progressão do regime para aqueles adolescentes já
E Solidariedade. privados de liberdade, conforme artigo 120 do ECA.
Nela, o educando fica sob a custódia do estado, sendo
Gabarito: 01/E; 02/D; 03/B; 04/D; 05/B; 06/B possível a realização de atividades externas,
independente de autorização judicial.
§ 4º A medida socioeducativa de internação
sanção pode ser aplicada por descumprimento reiterado
O REGIMENTO INTERNO DAS e injustificável da medida anteriormente imposta,
UNIDADES DA SUPERINTENDÊNCIA DO conforme artigo 122, inciso III, parágrafo 1º, do ECA.
§ 5º A medida socioeducativa de internação é
SISTEMA ESTADUAL DE ATENDIMENTO aplicada ao adolescente autor de ato infracional e deve
SOCIOEDUCATIVO (SEAS) 5 observar o previsto no artigo 122 do ECA, somente
Apresentação podendo ser aplicada nos casos e hipóteses previstas
em lei.
A Secretaria do Trabalho e Desenvolvimento
Social do Ceará (STDS) Coordena as políticas do Art. 2º. As unidades de medidas socioeducativas
trabalho, de assistência social e de segurança alimentar estão sob a administração direta do governo do estado
e nutricional no estado. No âmbito da política de do Ceará.
assistência social, a STDS desenvolve e apoia Art. 3º. Os procedimentos internos de
programas de proteção social, entre eles, o programa de funcionamento, de atendimento e de segurança serão
apoio às reformas sociais do Ceará (proares II), cujas administrados pelo corpo diretivo das unidades de
ações se destinam a crianças, adolescentes, jovens e medidas socioeducativas, sob a responsabilidade
suas famílias que se encontram em situação de destes, respeitando as diretrizes do presente estatuto.
vulnerabilidade social. Seção I - Dos Direitos dos Adolescentes
Este regimento dispõe sobre as unidades de Art. 4º. Serão garantidos aos adolescentes os
medidas socioeducativas do estado do Ceará, seguintes direitos, dentre outros:
responsáveis pela recepção, internação provisória, I. Ter Respeitada a sua individualidade e estar
semiliberdade, internação sanção e internação por livre de preconceito e julgamento moral.
sentença, previstas no Estatuto da Criança e do
Adolescente – eca, e nas recomendações preconizadas II. Estar salvo de qualquer tratamento desumano,
no sistema nacional de atendimento socioeducativo – vexatório ou que atente contra a dignidade da pessoa
SINASE. em desenvolvimento.
O presente documento é composto por títulos, III. Participar de atividades escolares,
capítulos, seções e subseções, que trata sobre os pedagógicas, profissionalizantes, culturais, esportivas e
parâmetros de normatização e funcionamento das de lazer, devendo ser garantida a carga horária
unidades de atendimento socioeducativo do estado do educacional que dispõe a lei de Diretrizes e Bases da
Ceará. Educação Nacional.
A elaboração deste regimento contou com a IV. Receber assistência médica e odontológica,
coordenação do plano estratégico estadual - PEE, priorizando os serviços públicos e comunitários.
gestores, técnicos, instrutores educacionais das V. Ter o direito à ampla defesa e ao contraditório
unidades de medidas socioeducativas e representantes quando lhe for atribuída conduta faltosa, antes de lhe ser
do sistema de garantia de direitos. aplicada a medida disciplinar.
Título I - Das Medidas Socioeducativas VI. Ter garantida a convivência familiar e
Capítulo I - Disposições Gerais comunitária, respeitando os critérios previamente
definidos neste regimento.
Art. 1º. Este regimento dispõe sobre as unidades
de medidas socioeducativas do estado do Ceará, VII. Receber atendimento técnico, no mínimo,
responsáveis pela recepção, internação provisória, quinzenalmente.
semiliberdade, internação sanção e internação por VIII. Ter garantido os documentos indispensáveis
sentença, previstas no Estatuto da Criança e do à vida em sociedade.
Adolescente – eca, e nas recomendações preconizadas IX. Ter acesso aos meios de comunicação social,
no sistema nacional de atendimento socioeducativo – a partir de critérios predefinidos pela equipe
SINASE. socioeducativa.
§ 1º A recepção caracteriza-se pelo acolhimento X. Receber visitas, obedecendo a critérios
de natureza transitória ao adolescente acusado da estabelecidos pela direção e pela equipe técnica da
prática de ato infracional, proveniente da delegacia da unidade de medida socioeducativa, salvo em caso de
criança e do adolescente e das comarcas do interior do suspensão temporária desse direito pelo juízo
estado, que deverá ser apresentado ao judiciário, tendo responsável pela execução da medida socioeducativa.
XI. Ter acesso a condições adequadas de higiene
5
e asseio pessoal.
DOECE, 18/06/2015 (Caderno 2)

www.editoradince.com - Acesse e veja se há novidades a respeito deste material – CUIDADO: cópia é crime.
82 LEGISLAÇÃO ESPECIAL E REGIMENTO INTERNO DAS UNIDADES DO SEAS
XII. Habitar em alojamentos em condições de IV. Tomar a medicação nos horários
higiene e salubridade. estabelecidos, em caso de prescrição e orientação
XIII. Receber, quando do seu desligamento, os médica.
documentos pessoais. V. Cuidar da higiene pessoal, do asseio de seu
XIV. Ter acesso, quando necessário, a atividades alojamento e de seus objetos pessoais, bem como dos
psicoterapêuticas. espaços de convivência.
XV. Ser ouvido pela direção e equipe técnica em VI. Colaborar com a limpeza e conservação da
suas queixas, problemas, dúvidas e reivindicações. unidade de medida socioeducativa, assim como de
todos os seus bens.
XVI. Corresponder-se com familiares.
VII. Portar-se sempre de forma respeitosa dentro
XVII. Receber assistência religiosa, segundo sua
e fora da unidade de medida socioeducativa.
crença, desde que assim o deseje, e que não coloque
em risco sua segurança física e mental. VIII. Acessar os espaços restritos da unidade de
medida socioeducativa somente com autorização ou
XVIII. Avistar-se e entrevistar-se com o
acompanhado de funcionário da unidade,
representante do ministério público e defensoria pública.
preferencialmente do instrutor educacional.
XIX. Peticionar diretamente a qualquer
IX. Respeitar as normas da instituição no que se
autoridade.
refere às saídas e atividades externas realizadas
XX. Ser informado, sempre que solicitar, sobre durante o cumprimento da medida socioeducativa.
sua situação processual.
X. Submeter-se à revista nas seguintes situações:
XXI. Ter acesso a atividades e serviços fora dos saída e retorno da unidade; após o recebimento da visita
limites da instituição, nas condições estipuladas pela de familiares; após o término das atividades de sala de
direção, salvo expressa determinação judicial em aula, oficinas, e quando se fizer necessário.
contrário.
XI. Submeter-se à revista em seu alojamento e
XXII. Solicitar medida de convivência protetora, em seus pertences, conforme rotina estabelecida pela
assegurando-se espaço físico apropriado, quando unidade de medida socioeducativa, e quando esta
estiver em situação de risco à sua integridade física ou entender necessário.
psicológica ou à vida;
XII. Cumprir as medidas sancionatórias que lhe
XXIII. Receber orientação das regras de forem impostas, quando autor de transgressão
funcionamento da unidade e das normas deste disciplinar ou de novo ato infracional.
regimento interno, especialmente quanto ao regulamento
XIII. Dirigir-se aos profissionais, colegas e
disciplinar.
visitantes de forma educada.
Art. 5º. Para a garantia dos direitos elencados no
XIV. Respeitar a integridade física e moral de
artigo anterior, as unidades de medidas socioeducativas
seus colegas e profissionais.
deverão:
Título II - Das Fases do Atendimento Socioeducativo
I. Manter contato permanente com os órgãos que
compõem o sistema de garantia de direitos, com a Art. 7º. O atendimento socioeducativo dependerá
comunidade local e com a sociedade em geral, a partir do plano individual de atendimento (pia), instrumento de
de parcerias previamente articuladas pela STDS. previsão, registro e gestão das atividades a serem
desenvolvidas com o (a) adolescente.
II. Envolver a família no processo do cumprimento
da respectiva medida, favorecendo o fortalecimento dos Art. 8º. As fases do atendimento socioeducativo,
vínculos sociais. conforme a resolução 119/2006 do comanda, são:
Parágrafo único. O conceito de família deste I - Fase inicial de atendimento: período de
regimento se referencia no plano nacional de promoção acolhimento, de reconhecimento e de elaboração por
e defesa do direito de crianças e adolescentes à parte do adolescente do processo de convivência, de
convivência familiar e comunitária, no qual se entende orientação sobre as normas e regimento da unidade, de
por família um grupo de pessoas unidas por laços de realização do diagnóstico polidimensional e elaboração
consanguinidade, de aliança e/ou de afinidade, do seu plano individual de atendimento (pia), no prazo
constituídos por representações, práticas e relações que de 45 dias, conforme dispõe, o artigo 55, parágrafo
implicam em obrigações mútuas e exercem a função de único da lei nº 12.594/2012.
proteção e socialização do adolescente. II - Fase intermediária: período de
Seção II - Dos Deveres dos Adolescentes compartilhamento em que o adolescente apresenta
avanços relacionados nas metas consensuadas no
Art. 6º. São deveres do adolescente, entre
outros: plano individual de atendimento e de desenvolvimento
da proposta pedagógica.
I. Cumprir o previsto neste regimento e na rotina
III - Fase conclusiva: período em que o
institucional, além dos demais procedimentos da
adolescente apresenta clareza e conscientização das
unidade de medida socioeducativa.
metas conquistadas em seu processo socioeducativo e
II. Frequentar assiduamente e participar das em que há a preparação para o desligamento do
atividades escolares e de outras atividades propostas adolescente e sua reinserção sociofamiliar.
pela unidade de medida socioeducativa, salvo nas
Seção I – Da Fase Inicial de Atendimento
situações em que se justifique sua ausência, devendo
esta ser autorizada pela equipe socioeducativa, com Art. 9º. Os profissionais que acolhem o
posterior ciência ao diretor e equipe técnica. adolescente na unidade de medida socioeducativa
devem ter postura de respeito, para que seja iniciada a
III. Cumprir com todas as suas obrigações de
formação de vínculos positivos com o socioeducando.
aluno na escola e cursos profissionalizantes que estiver
inserido. Art. 10. A chegada do adolescente deve ser
registrada e, quando necessário, realizada a emissão

www.editoradince.com - Acesse e veja se há novidades a respeito deste material – CUIDADO: cópia é crime.
LEGISLAÇÃO ESPECIAL E REGIMENTO INTERNO DAS UNIDADES DO SEAS 83
dos devidos documentos de recebimento deste às adolescente e sua família, a partir da análise dos
autoridades judiciárias competentes. compromissos assumidos, da consciência crítica do
Art. 11. Os pertences do adolescente devem ser adolescente e das metas alcançadas por meio de plano
conferidos e registrados sob sua presença e guardados, individual de atendimento.
devendo ser providenciados a refeição, materiais de Art. 26. Haverá a elaboração de relatório
higiene pessoal, vestuário e roupas de cama e banho, conclusivo do adolescente, observando a avaliação
ficando sob a responsabilidade dos instrutores destacada na evolução do seu PIA e do cumprimento da
educacionais a orientação quanto à higiene pessoal e medida socioeducativa.
revistas pessoais do adolescente. Art. 27. Cabe à equipe técnica observar as
Art. 12. O adolescente, nesta fase, deverá condições externas para a reinserção do adolescente na
conhecer as normas e rotinas da unidade, sobremaneira comunidade, preparando:
no que concerne ao regimento disciplinar. I. O acolhimento familiar;
Art. 13. O adolescente deverá ser encaminhado II. Os encaminhamentos para cursos na
para dormitório específico, em observância aos critérios comunidade;
de faixa etária, compleição física e análise preliminar
III. O encaminhamento para a escola em que o
dos riscos de conflito com outros adolescentes
adolescente será inserido;
presentes na Unidade.
IV. Os equipamentos da assistência social, para o
Art. 14. O adolescente deverá receber
acompanhamento do adolescente e sua família;
atendimento multiprofissional e ser entrevistado,
observando-se o amparo emocional que se fizer V. Os equipamentos de saúde, caso necessário.
necessário nesta fase de atendimento. Título III – Do Acesso à Unidade de Medida
Art. 15. O adolescente deverá ser informado Socioeducativa
sobre todas as atividades oferecidas na Unidade de Art. 28. O acesso à unidade de medida
medida socioeducativa e inserido às rotinas, socioeducativa obedecerá aos seguintes critérios:
despertando seus interesses e orientando suas opções I. A entrada de pessoas, devidamente
de participação nas atividades. identificadas, nas dependências da unidade, será
Art. 16. A família deverá ser atendida desde o precedida de autorização do diretor e registro em
início da recepção do adolescente na Unidade de formulário próprio.
medida socioeducativa. II. Os representantes do poder judiciário, do
Art. 17. Realizar-se-á o diagnóstico ministério público, do poder legislativo, da defensoria
polidimensional do adolescente (estudo de caso), a pública, dos conselhos tutelares, dos conselhos de
partir da identificação das necessidades nos aspectos direitos da criança e do adolescente e do comitê
Jurídico, Psicológico, Social, Pedagógico e de Saúde. estadual de prevenção e combate à tortura terão acesso
Art. 18. Dar-se-á na fase inicial a elaboração do irrestrito às dependências da Unidade de medida
diagnóstico polidimensional e da construção do plano socioeducativa.
individual de atendimento – PIA, no prazo de 45 dias da III. O acesso do advogado ao adolescente dar-se-
acolhida do adolescente na Unidade. á nos termos do Estatuto da Ordem dos Advogados do
Art. 19. O diagnóstico polidimensional subsidiará Brasil, bem como da legislação civil e processual
o plano individual de atendimento e, a partir da pertinente à matéria, mediante a apresentação da
execução do PIA, deverá ser realizada a avaliação dos identidade funcional de advogado e procuração assinada
avanços ocorridos no período de cumprimento da pela família.
medida socioeducativa. § 1º O acesso será permitido após identificação
Art. 20. O adolescente e sua família devem § 2º O acesso ao adolescente ocorrerá em dias
participar ativamente do seu diagnóstico úteis, no horário das 8 às 17 horas, e deverá ficar
polidimensional. restrito à área administrativa da Unidade.
Art. 21. O plano individual de atendimento deve Art. 29. É vedado nas dependências da unidade
ser enviado ao juízo da infância e da juventude para de medida socioeducativa:
homologação e acompanhamento da resposta da I. A entrada de visitantes portando armas ou
medida socioeducativa, nos termos da lei. qualquer outro objeto que possa colocar em risco a
Seção II – Da Fase Intermediária segurança, salvo as autoridades policiais, quando a
Art. 22. Acompanhamento do PIA, mensalmente, situação exigir e devidamente autorizadas pela direção
pela equipe técnica junto ao adolescente para verificar da Unidade.
sua evolução e o cumprimento da medida II. A entrada de profissionais portando objetos
socioeducativa, possibilitando, quando necessário, a pessoais ou qualquer outro que possa colocar em risco a
criação de novas estratégias. segurança, salvo os que serão utilizados nas atividades
Art. 23. Avaliação do adolescente sobre sua socioeducativas planejadas para o dia.
participação nas atividades e rotinas da Unidade de III. Fotografar as dependências da unidade ou os
internação, no mínimo, mensalmente. adolescentes, salvo com autorização prévia da direção
Art. 24. O acesso ao PIA será restrito à direção, à da unidade ou da coordenação das medidas
equipe técnica, ao socioeducador de referência para o socioeducativas.
adolescente, ao adolescente e aos seus pais ou Título IV – Da Rotina Institucional
responsável, ao ministério público e ao defensor, exceto Capítulo I - Disposições Gerais
expressa autorização judicial em contrário. Art. 30. Para regular a convivência, definem-se
Seção III – Da Fase Conclusiva como critérios normativos das unidades de medidas
Art. 25. Nessa fase, dar-se-á o atendimento de socioeducativas:
avaliação conclusiva da equipe técnica, referente ao I. Rotina Institucional.

www.editoradince.com - Acesse e veja se há novidades a respeito deste material – CUIDADO: cópia é crime.
84 LEGISLAÇÃO ESPECIAL E REGIMENTO INTERNO DAS UNIDADES DO SEAS
II. Regulamento Disciplinar. Art. 35. É proibido ao profissional envolvido no
Art. 31. As Unidades de medidas socioeducativas fato em apuração participar da apuração do ocorrido e
deverão estabelecer a rotina institucional devendo aplicar medida disciplinar, podendo tomar parte apenas
contemplar, dentre outros, os seguintes itens: como informante para fins de prova, se assim
demandado.
I. Horário de acordar e de dormir.
Capítulo II – Da Visita Familiar, Social e Intima
II. Horário das refeições.
Art. 36. O instituto da visita no âmbito do
III. Dias e horários das atividades escolares,
cumprimento de medida socioeducativa de internação
esportivas, culturais, de lazer, de assistência religiosa,
destina-se a manter e fortalecer vínculos familiares e
oficinas, cursos, grupos temáticos.
comunitários do adolescente, obedecendo às seguintes
IV. Dias e horários dos atendimentos. orientações:
V. Dias e horários da visita dos familiares à I - A visita ao adolescente poderá ser realizada
unidade. pelos parentes em linha reta, colaterais, por afinidade,
VI. Descrição dos objetos de uso pessoal (kit responsáveis legais e amigos, considerandose a família
pessoal) e a periodicidade da sua entrega ou troca. natural, socioafetiva, extensa e a rede social de apoio do
VII. Lista e quantidade de materiais/objetos adolescente, que deverá ser identificada na fase inicial
autorizados a permanecer no dormitório. de elaboração do PIA.
VIII. Lista e quantidade de materiais/objetos § 1º Deverá ser garantido o direito à visita ao
autorizados a serem trazidos pelos visitantes/ familiares. adolescente no mínimo uma vez por semana, em dia e
IX. Quantidade e duração das ligações horário definidos pela unidade de medida
telefônicas, devendo ser, no mínimo, uma ligação por socioeducativa, devendo ser observada a capacidade
semana. máxima de 03 pessoas por visita e seguindo os critérios
de cadastramento, conforme art. 39, § 1º.
X. Definição da programação da TV e do Rádio.
§ 2º Após prévia avaliação da equipe técnica, os
XI. Quantidade, horário e duração do banho. adolescentes só receberão visitas de pessoas por eles
Parágrafo Único. Cada Unidade de medida autorizados e por seus familiares.
socioeducativa deverá, no prazo máximo de trinta dias Art. 37. A visita íntima, exclusiva para internação,
da implantação deste regimento, encaminhar à é garantida aos adolescentes maiores de 14 anos,
coordenadoria de proteção social especial a sua rotina independentemente de gênero e orientação sexual,
institucional para validação, mantendo-a atualizada desde que comprovada a convivência afetiva anterior,
quanto às alterações que venham a ocorrer. com autorização escrita dos pais ou responsáveis do
Art. 32. O regulamento disciplinar estabelece as socioeducando e do companheiro.
transgressões e as medidas disciplinares aplicáveis. Parágrafo Único. A visita íntima será
§ 1º A medida disciplinar é uma sanção aplicada condicionada a participação dos envolvidos em
ao adolescente que cometeu algum ato definido como atendimentos individuais e/ou em grupos referentes à:
transgressão às normas da unidade de medida orientação sexual e reprodutiva, métodos contraceptivos,
socioeducativa, devendo ser aplicada pela comissão doenças sexualmente transmissíveis e aids e outros
disciplinar. temas pertinentes, que devem ser realizados no âmbito
§ 2º Nenhum adolescente receberá medida do sistema socioeducativo.
disciplinar sem que lhe seja garantido à apuração da Art. 38. A convivência afetiva anterior poderá ser
transgressão disciplinar, o direito ao contraditório e à comprovada a partir da declaração dos conviventes,
ampla defesa. confirmada pelos pais, responsáveis ou familiares no
§ 3º Somente serão passíveis de medida momento da elaboração do PIA.
disciplinar as transgressões previstas neste Art. 39. As visitas terão tempo mínimo de 1 (uma)
regulamento. hora.
§ 4º A advertência deverá ser priorizada sempre § 1º Para o cadastramento e entrada na unidade,
que cabível. exige-se certidão de nascimento para os menores de 12
§ 5º As medidas disciplinares têm caráter anos e documento oficial com foto para os maiores de
preponderantemente educativo e respeitarão os direitos 12 anos.
humanos. § 2º A entrada de visitantes menores de 18 anos
§ 6º As medidas disciplinares respeitarão a somente ocorrerá se acompanhados dos pais ou
individualização da conduta do adolescente, sendo responsáveis legais, ou a quem estes designarem,
vedada a aplicação de medida coletiva aos mediante autorização presencial reduzida a termo, ou
adolescentes. com firma reconhecida em cartório ou alvará judicial.
Art. 33. São proibidas a incomunicabilidade e a § 3º Ao familiar que residir em outras comarcas é
suspensão de visita familiar ao adolescente em permitido o cadastramento e autorização no dia da
cumprimento de medida disciplinar. visita.
Art. 34. O cumprimento de medida disciplinar não § 4º As visitas do adolescente serão submetidas
deverá prejudicar a escolarização, a profissionalização e ao detector de metais, bem como a outros recursos
as medidas especiais de atenção à saúde. tecnológicos relativos aos procedimentos de revista
Parágrafo Único. Deverá ser propiciado ao pessoal.
adolescente com medida disciplinar de restrição ao seu § 5º O visitante que se recusar ao procedimento
dormitório, nos dias em que não houver atividades de revista pessoal não poderá visitar o adolescente.
escolares ou profissionalizantes, permanência de 30 § 6º Serão vistoriados todos os objetos trazidos
minutos em atividade ao ar livre. pelo visitante, destinados a ele ou aos adolescentes, e

www.editoradince.com - Acesse e veja se há novidades a respeito deste material – CUIDADO: cópia é crime.
LEGISLAÇÃO ESPECIAL E REGIMENTO INTERNO DAS UNIDADES DO SEAS 85
os objetos não permitidos serão listados e guardados em caso seja esta a única forma de promover o acesso à
local próprio e devolvidos ao final da visitação. convivência familiar e comunitária para o adolescente.
§ 7º Em caso de ato ilícito cometido pelo visitante Art. 43. As saídas e atividades externas não
no interior da unidade, a direção deverá acionar a polícia deverão prejudicar a frequência e o desempenho escolar
militar para as providências cabíveis, bem como solicitar dos adolescentes em qualquer atividade de caráter
ao poder judiciário o impedimento temporário de visitas pedagógico ou de qualificação profissional.
futuras. Título V – Da Medida de Convivência Protetora
§ 8º A Unidade de medida socioeducativa deverá Artigo 44. O adolescente poderá, em caráter
impedir a entrada de visitante, se houver contra esta excepcional, ser incluído em medida de convivência
decisão judicial de suspensão de visita. protetora, em local apropriado, sem prejuízo das
Capítulo III – Das Saídas e Atividades Externas à atividades obrigatórias, quando existir situação de risco
Unidade à sua integridade física e psicológica ou à vida, que
Art. 40. Será considerada saída da unidade de impeça a permanência com os demais adolescentes,
medida socioeducativa sempre que o adolescente tiver recebendo, o mais breve possível, atenção especial da
um destino com objetivo predefinido, acompanhado ou equipe psicossocial.
não por um profissional da unidade. § 1º. A inclusão poderá ser realizada a
Parágrafo Único. As atividades externas à requerimento do adolescente, que expressará os
unidade de medida socioeducativa são consideradas motivos que tornam necessária a medida, ou por
saídas e poderão ser desenvolvidas individual ou determinação formal do diretor da unidade, mediante
coletivamente, desde que programadas e orientadas fundadas informações nos termos do “CAPUT”.
com um objetivo predefinido, podendo ocorrer com ou § 2º. O diretor, ouvida a equipe técnica, fixará o
sem o acompanhamento de profissionais da instituição. prazo de convivência protetora, que não poderá
Art. 41. São modalidades de saídas: ultrapassar 30 (trinta) dias, e providenciará, de imediato,
as medidas necessárias para a proteção do adolescente,
I. Atividades culturais, esportivas, de lazer e de
cabendo a equipe técnica a elaboração de um plano de
assistência religiosa.
reinclusão do adolescente no convívio da unidade de
II. Atividades de escolarização, atendimento.
profissionalização, trabalho.
§ 3º. Caso não seja possível a transferência ou
III. Atividades que promovam o exercício da não exista solução mais adequada para a proteção do
cidadania. adolescente, o diretor poderá, condicionado ao parecer
IV. Atividades que promovam a convivência da equipe técnica, prorrogar o prazo de permanência,
familiar e comunitária. enquanto persistir o risco.
V. Visitas à família e outros eventos § 4º. O diretor ou equipe técnica deverá
circunstanciais de natureza familiar, tais como comunicar, imediatamente, os pais ou responsáveis
nascimento, óbito, doença grave e paternidade. legais quando da inclusão do adolescente em medida
VI. Atendimento na rede de saúde. protetora, seu período de duração e eventuais
VII. Aleitamento materno e coleta de leite, no que prorrogações.
se refere às adolescentes do sexo feminino. § 5º. O diretor deverá comunicar, em 24 (vinte e
VIII. Determinações judiciais. quatro) horas, por escrito, ao juízo competente, inclusive
para fins de comunicação ao ministério público e ao
IX. Convocações extrajudiciais. defensor do adolescente, a decretação de convivência
§ 1º As saídas previstas nos incisos de I a IV são protetora, seu período de duração e eventuais
consideradas atividades externas e ocorrerão desde que prorrogações.
avaliadas pela equipe técnica da Unidade, seguindo § 6º. Em casos excepcionais, feriados ou finais de
critérios judiciais e técnicos, registradas devidamente no semana, as comunicações previstas nos § 4ºe § 5º
plano individual de atendimento (PIA). serão realizadas, impreterivelmente, no primeiro dia útil.
§ 2º A necessidade de acompanhamento do § 7º. No mesmo prazo do parágrafo 5º, deverá o
instrutor educacional será avaliada pela equipe da diretor da unidade de atendimento enviar cópia da
Unidade. comunicação à célula de atenção às medidas
Art. 42. As visitas à família, previstas no inciso V, socioeducativas, da Coordenadoria de Proteção Social
do artigo 34, ocorrerão desde que resguarde a finalidade Especial.
de fortalecer a convivência familiar e comunitária e a Título VI - Do Regulamento Disciplinar para o
partir de avaliação da equipe técnica da unidade, sendo Adolescente
obedecidas às seguintes diretrizes:
Capítulo I - Do Processo de Apuração das
I. A equipe técnica deverá realizar estudo de Transgressões Disciplinares dos Adolescentes
caso.
Art. 45. É dever do profissional que, por qualquer
II. Nas visitas à família, o adolescente sairá meio, presenciar ou tiver conhecimento de falta
acompanhado pelo responsável, após assinatura destes disciplinar, de qualquer natureza, elaborar registro de
no termo de entrega e responsabilidade. III. As datas de ocorrência, que conterá:
saídas e de retornos dos adolescentes, cujas famílias
residam em municípios distantes da unidade de medida I - O nome e a identificação do adolescente;
socioeducativa, serão definidas levando em II - Local e hora da ocorrência;
consideração, além de critérios técnicos, a distância e a III - O ato que lhe é atribuído;
forma de locomoção entre os municípios. IV - A descrição sucinta dos fatos;
Parágrafo Único. Os dias programados para a V - O rol, de no máximo, 3 (três) testemunhas e o
visita à família poderão ser revistos, individualmente, (s) nome (s) da (s) eventual (is) vítima (s);

www.editoradince.com - Acesse e veja se há novidades a respeito deste material – CUIDADO: cópia é crime.
86 LEGISLAÇÃO ESPECIAL E REGIMENTO INTERNO DAS UNIDADES DO SEAS
§ 1º. O registro de ocorrência será enviado, Art. 52. No processo de apuração das
imediatamente, à comissão disciplinar da unidade. transgressões disciplinares, especialmente no que
Art. 46. A comissão disciplinar designará data, a concerne às transgressões de natureza leve, a comissão
mais breve possível, para ouvir o adolescente, as disciplinar priorizará a utilização de uma abordagem
testemunhas e as vítimas (s) eventualmente indicadas restaurativa e de autocomposição de conflitos para
no registro de ocorrência. responsabilização do adolescente e, se possível,
reparação de danos, em conformidade com o disposto
Parágrafo Único - As oitivas serão reduzidas a
no art. 36, da lei 12.594/2012, por meio da suspensão
termo e, após a leitura, serão assinadas pelos
do processo ordinário e encaminhamento do caso aos
respectivos depoentes.
facilitadores de práticas restaurativas da unidade.
Art. 47. Encerradas as oitivas e não sendo
§ 1º - A viabilidade da utilização da abordagem
necessária a produção de outras provas, a comissão
restaurativa será verificada mediante a voluntariedade
disciplinar, assegurada a ampla defesa e o contraditório,
da participação dos envolvidos, bem como da existência
proferirá decisão e, se for o caso, aplicará a sanção, no
de condições seguras para a promoção do encontro;
prazo máximo de 03 (três) dias da ocorrência do fato.
§ 2º - O início da abordagem restaurativa deverá
§ 1º. A decisão deverá ser fundamentada e
suspender o procedimento ordinário de apuração das
descreverá, em relação a cada adolescente,
transgressões disciplinares, que só será retomado caso
separadamente, a falta disciplinar que lhe é atribuída, as
o processo não resulte em acordo viável ou na
provas colhidas, as razões da decisão e, se for o caso, a
possibilidade de acordo, este não for cumprido
sanção a ser aplicada.
satisfatoriamente;
§ 2º. A comissão disciplinar levará em conta, na
§ 3º - Em caso de o processo resultar em acordo
aplicação da medida disciplinar, as circunstâncias
de responsabilização cumprido, arquivar-se-á o
atenuantes e agravantes previstas nos artigos 58 e 59
procedimento de apuração das transgressões
deste regimento.
disciplinares.
§ 3º. Inobservado o prazo do CAPUT, extingue a
Art. 53. O processo restaurativo será
pretensão da aplicação da medida disciplinar.
desenvolvido por um ou dois facilitadores de práticas
§ 4º. É facultado ao adolescente no procedimento restaurativas, profissionais habilitados, capacitados em
disciplinar requisitar defesa técnica. metodologias de resolução de conflitos apropriadas.
§ 5º. O defensor público, o ministério público, o Art. 54. O procedimento restaurativo será
adolescente e seus pais ou responsável poderão composto por três fases:
postular revisão judicial de qualquer sanção disciplinar
I - Preparação: em que cada pessoa envolvida no
aplicada, podendo a autoridade judiciária suspender a
processo deverá ser escutada individualmente pelos
execução da sanção até decisão final do incidente, nos
facilitadores, sendo instruída acerca do funcionamento
termos do art. 48 da lei 12.594/2012.
do processo restaurativo, bem como questionada sobre
Art. 48. Quando da ocorrência de alguma o ato de indisciplina;
situação excepcional que envolva risco para segurança
II - Encontro: em que os envolvidos serão
do adolescente, a direção da unidade poderá determinar
reunidos, com a mediação do facilitador que, com a
medida cautelar em caráter provisório e protetivo, sem
metodologia restaurativa adequada, deverá facilitar o
prejuízo para as atividades socioeducativas.
processo de diálogo e a construção de um acordo de
Art. 49. Da decisão da comissão disciplinar responsabilização;
caberá recurso à comissão de apuração vinculada à
III - Monitoramento: que consiste em novo
coordenadoria de proteção social especial, no prazo de
encontro em que o facilitador deverá verificar o
5 (cinco) dias úteis, a contar da decisão, devendo o
cumprimento ou não do acordo.
mesmo ser deliberado, no máximo, em igual prazo,
podendo ser prorrogada em casos excepcionais e § 1º - Verificado o cumprimento do acordo, o
devidamente justificados. facilitador deverá encaminhar informe à comissão
disciplinar, solicitando o arquivamento do processo
Parágrafo Único. Caso a comissão de apuração
disciplinar.
manifeste-se contrária à decisão da comissão disciplinar
da unidade, o ocorrido não deverá constar no relatório § 2º - Verificado o descumprimento, o facilitador
do adolescente, cabendo à comissão de apuração irá verificar a possibilidade de estabelecimento de novo
informar ao juízo sobre a revisão da decisão da medida prazo para que o acordo seja cumprido ou, sendo este
aplicada no prazo de 5 (cinco) dias úteis. inviável, encaminhará à comissão disciplinar o informe
do descumprimento, solicitando a reabertura do
Art. 50. Logo após a decisão da comissão
procedimento ordinário de apuração das transgressões
disciplinar da Unidade o diretor determinará as seguintes
disciplinares.
providências:
Art. 55. Ressalvado o acordo de
I – Ciência ao adolescente;
responsabilização resultado do processo restaurativo,
II – Ciência aos pais ou responsável legal do todas as informações resultantes dos procedimentos de
adolescente; preparação e de encontro serão sigilosas, ficando seu
III – Registro da medida disciplinar aplicada no conhecimento restrito às pessoas diretamente
prontuário do adolescente; envolvidas em cada etapa do processo, não podendo
Art. 51. O Socioeducando que concorrer para o ser usadas como prova no processo ordinário de
cometimento da falta disciplinar incidirá nas mesmas apuração de transgressões disciplinares, sob pena de
sanções cominadas ao autor, na medida de sua responsabilização, conforme previsão neste regimento.
participação e culpabilidade. Art. 56. O processo restaurativo não deverá
Seção I - Da Abordagem Restaurativa no Processo exceder, em hipótese nenhuma, o prazo de 30 (trinta)
de Apuração das Transgressões Disciplinares dias, desde sua abertura até o informe final à comissão
disciplinar.

www.editoradince.com - Acesse e veja se há novidades a respeito deste material – CUIDADO: cópia é crime.
LEGISLAÇÃO ESPECIAL E REGIMENTO INTERNO DAS UNIDADES DO SEAS 87
Seção II - Da Comissão Disciplinar VI. Recusar-se a se deslocar de uma atividade a
Art. 57. A comissão disciplinar é uma instância outra para atender ao previsto no agendamento das
formal colegiada por meio da qual se apura, de forma atividades da unidade;
individualizada, a ocorrência de falta disciplinar de VII. Obstruir a visão do alojamento;
natureza leve, média e grave praticada por VIII. Desobedecer às normas de circulação e
socioeducando, aplicando-se a sanção disciplinar trânsito interno;
cabível, sendo assegurado o direito à ampla defesa e ao
IX. Deixar de trocar as roupas de cama e toalhas
contraditório, nos termos do artigo 5º, inciso lV,
ou não devolvê-las, no prazo estabelecido pelo
da Constituição Federal
cronograma da unidade;
I. A comissão disciplinar deverá ser composta
X. Manusear equipamentos e materiais sem
pelo diretor da unidade, por dois representantes da
autorização;
equipe técnica e dois representantes dos
socioeducadores. XI. Trocar ou doar dentro da unidade, objeto lícito
que lhe pertença;
II. Em se tratando de transgressões de natureza
leve, a comissão disciplinar poderá, excepcionalmente, XII. Atrasar-se, sem justa causa, no retorno à
exercer suas funções com apenas 3 (três) de seus unidade, no caso de saída autorizada;
membros, sendo obrigatória a participação do diretor da XIII. Furtar objetos que não ofereçam risco a
unidade e de um representante da equipe técnica. integridade física de outrem;
III. A resposta disciplinar no caso de XIV. Induzir ou instigar pessoa a praticar falta
transgressões leves, ainda que obedeça ao prazo disciplinar de natureza leve.
máximo de 3 (três) dias, deverá observar, de modo Seção II - Das Transgressões Médias
preponderante, o princípio da celeridade e da eficácia da Art. 61. Constituem-se transgressões médias:
medida disciplinar.
I. Adentrar em dormitório alheio e causar tumulto;
§ 1º. A coordenadoria de proteção social
especial, anuirá representantes que comporão a II. Impedir ou perturbar a realização de atividades
comissão disciplinar; socioeducativas dentro ou fora da unidade, bem como o
repouso;
§ 2º. Ficam impossibilitados de compor a
comissão disciplinar o (s) servidor (es) que esteja (m) III. Vender, dentro da unidade, objeto lícito que
envolvido (s) no fato da apuração, podendo participar lhe pertença, sem autorização da direção e equipe
apenas como parte informante para fins de prova, se técnica da unidade;
assim demandado, devendo o substituto ser designado IV. Trocar de dormitório sem autorização;
pela direção. V. Danificar roupas ou objetos de outrem,
Seção III - Da Comissão Permanente de fornecidos pela unidade ou familiares;
Monitoramento e Fiscalização do uso do Isolamento VI. Atrasar-se, sem justa causa, no retorno à
Cautelar e Aplicação de Sanções unidade, no caso de saída autorizada;
Art. 58. É uma instância formal e colegiada VII. Dificultar ou recusar-se a submeter-se à
incumbida de fiscalizar o uso do isolamento cautelar e revista pessoal, de seu dormitório, bens ou pertences;
aplicação de sanções nas Unidades Socioeducativas e VIII. Fazer uso de medicação prescrita para outro
emitir relatórios. A comissão é composta por servidores adolescente;
e colaboradores indicados pala Coordenadoria de
IX. Sair para qualquer atividade externa e
Proteção Social Especial – CPSE e publicizada através
desviar-se de seu percurso ou separar-se sem
de portaria do secretário
autorização, quando acompanhado ou não de um
Capítulo II - Das Transgressões Disciplinares dos funcionário da unidade;
Adolescentes
X. Injuriar, difamar, caluniar, agredir verbalmente
MUITA ATENÇÃO NOS ARTS. 59 A 66. ou proferir ameaças a adolescentes, funcionários ou
Art. 59. As transgressões classificam-se em: visitantes;
I. Leves; XI. Tentar fugir ou facilitar fuga sem êxito de
II. Médias; outrem da unidade sem ameaça ou violência;
III. Graves. XII. Praticar ato obsceno com a exposição das
partes íntimas quando estiver em áreas coletivas, junto a
Seção I - Das Transgressões Leves
outros adolescentes ou qualquer membro da equipe
Art. 60. Constituem-se transgressões leves: técnica e socioeducadores na unidade;
I. Possuir, portar ou utilizar valores não XIII. Danificar bens e materiais fornecidos pelo
concedidos ou não autorizados pela unidade; estado;
II. Desperdiçar materiais fornecidos pelo estado; XIV. Tentar provocar incêndio;
III. Entregar correspondência, bilhete ou similar, XV. Praticar lesão corporal leve;
sem o conhecimento e autorização dos profissionais da
XVI. Induzir ou instigar pessoa a praticar falta
unidade;
disciplinar de natureza média;
IV. Descumprir, injustificadamente, os horários
XVII. Cometimento reiterado de infrações de
estabelecidos para o funcionamento interno da unidade;
natureza leve.
V. Recusar-se, sem justificativa cabível e
Seção III - Das Transgressões Graves
autorização, a participar ou se ausentar de atividades de
escolarização e profissionalização já iniciadas; Art. 62. Constituem-se transgressões graves:
I. Incitar ou participar de movimento para
subverter a ordem ou a disciplina;

www.editoradince.com - Acesse e veja se há novidades a respeito deste material – CUIDADO: cópia é crime.
88 LEGISLAÇÃO ESPECIAL E REGIMENTO INTERNO DAS UNIDADES DO SEAS
II. Criar e divulgar notícia que perturbem a ordem V. Privação de produtos autorizados a entrar em
ou a disciplina na unidade; dias de visita.
III. Furtar objetos utilizados nas atividades ou § 1º Caso necessária a aplicação conjunta de
atendimentos de qualquer natureza que possa oferecer mais de um inciso, deve ser obedecido o limite de três
risco para si ou para outrem; incisos e ser respeitada a particularidade da
IV. Retornar à unidade com sintomas de uso de transgressão.
drogas ou álcool; § 2º As medidas previstas neste artigo podem ser
V. Receber, fabricar, portar, ter, consumir, cumuladas ou substituídas por outras de natureza
fornecer ou concorrer para que haja na unidade bebida pedagógica e/ou educativas, devendo ser avaliadas pela
alcoólica ou substâncias que possam causar reações direção e pela equipe técnica da unidade.
adversas às normas de conduta, dependência física ou Art. 65. São medidas disciplinares aplicadas ao
psíquica; adolescente que comete transgressões médias:
VI. Portar, usar, possuir ou fornecer aparelho I. Advertência escrita, assinada pelo adolescente
telefônico celular ou outros meios de comunicação não e/ou duas testemunhas, e arquivada ao seu prontuário.
autorizados; II. Suspensão da prática recreativa e de lazer
VII. Fabricar, guardar, portar ou fornecer objeto pelo prazo de até três dias. III. Suspensão da prática
destinado à fuga; esportiva pelo prazo de até três dias.
VIII. Fabricar, guardar, portar ou fornecer objetos IV. Privação de produtos autorizados a entrar em
cortantes ou perfurantes que possam ser utilizados para dias de visita.
intimidar ou ferir pessoas; IX. Induzir ou instigar pessoa V. Retratação verbal à pessoa ofendida.
a praticar falta disciplinar de natureza grave;
§ 1º Caso necessária a aplicação conjunta de
X. Provocar autolesão para imputar mais de um inciso, deve ser obedecido o limite de três
responsabilidade a outra pessoa, com o intuito de induzir incisos e respeitada a particularidade da transgressão.
as autoridades a erro;
§ 2º As medidas previstas neste artigo podem ser
XI. Praticar ato infracional não previsto no cumuladas ou substituídas por outras de natureza
regimento, sem prejuízo do processo judicial; pedagógica e/ou educativas, devendo ser avaliadas pela
XII. Estabelecer relação de exploração física ou direção e pela equipe técnica da unidade.
de trabalho com outro adolescente, mediante violência Art. 66. São medidas disciplinares aplicadas a
ou grave ameaça; quem comete transgressões graves:
XIII. Evadir-se durante atividades externas e I. Suspensão da prática recreativa e de lazer pelo
saídas temporárias; prazo de até cinco dias.
XIV. Roubar ou extorquir qualquer objeto; II. Suspensão da prática esportiva pelo prazo de
XV. Receber objetos ilícitos; até cinco dias.
XVI. Cometimento reiterado de infrações de III. Privação de produtos autorizados a entrar em
natureza média; dias de visita.
XVII. Cometer homicídios; IV. Restrição do adolescente ao dormitório por no
XVIII. Provocar incêndio; máximo cinco dias.
XIX. Praticar lesão corporal média e leve; § 1º No âmbito da aplicação de medida
disciplinar, são absolutamente proibidas a
XX. Facilitar fuga de outrem da unidade,
incomunicabilidade e a suspensão de visita, assim como
utilizando-se de ameaça ou violência;
qualquer sanção que importe prejuízo às atividades
XXI. Estabelecer relação sexual com outro obrigatórias, consistentes na escolarização,
adolescente, mediante violência ou grave ameaça. profissionalização e nas medidas de atenção à saúde.
Art. 63. No caso de tumultos, rebeliões, incêndios § 2º Caso necessária a aplicação conjunta de
e homicídios, a polícia militar deverá ser acionada para mais de um inciso, deve ser obedecido o limite de três
as providências cabíveis, devendo ser realizada a incisos e respeitada a particularidade da transgressão.
imediata ciência à coordenação das medidas
§ 3º As medidas previstas neste artigo podem ser
socioeducativas.
cumuladas ou substituídas por outras de natureza
Parágrafo Único - Adotadas as providências pedagógica e/ou educativas, devendo ser avaliadas pela
legais e administrativas, a unidade de medida direção e pela equipe técnica da unidade.
socioeducativa manterá cópia dos documentos para
Art. 67. As medidas disciplinares serão aplicadas
arquivamento no prontuário do adolescente.
sem prejuízo das demais implicações e providências de
Capítulo III - Das Medidas Disciplinares Aplicadas cunho sancionatórios cabíveis ao caso.
aos Adolescentes
Art. 68. Quando do cometimento de mais de uma
Seção I - Das Medidas Aplicáveis transgressão disciplinar no mesmo evento, a
Art. 64. São medidas disciplinares aplicáveis ao transgressão mais grave absorve a menos grave.
adolescente que comete transgressões leves: Art. 69. A medida disciplinar poderá ser revista
I. Advertência verbal. pela comissão disciplinar no decorrer do cumprimento,
II. Advertência escrita, assinada pelo adolescente havendo motivo justificável, desde que não seja mais
e/ou duas testemunhas, e arquivada ao seu prontuário. gravosa para o adolescente.
III. Suspensão da prática recreativa e de lazer Art. 70. Não será aplicada sanção disciplinar
pelo prazo de até dois dias. ao adolescente que tenha praticado a falta:
IV. Suspensão da prática esportiva pelo prazo de I – Em estado de necessidade;
até três dias. II – Em legítima defesa própria ou de outrem;

www.editoradince.com - Acesse e veja se há novidades a respeito deste material – CUIDADO: cópia é crime.
LEGISLAÇÃO ESPECIAL E REGIMENTO INTERNO DAS UNIDADES DO SEAS 89
III – Por coação irresistível; II. Ter o adolescente cometido transgressão
IV – Por motivo de força maior. disciplinar por motivo fútil ou torpe ou para facilitar ou
assegurar a execução, ocultação, impunidade, ou
§ 1º Considera-se em estado de necessidade
vantagem em outra falta disciplinar.
quem pratica o fato para salvar de perigo atual, que não
provocou por sua vontade, nem podia de outro modo III. Cometer a transgressão disciplinar à traição,
evitar, direito próprio ou alheio, cujo sacrifício, nas de emboscada, com dissimulação ou abuso de
circunstâncias, não era razoável exigir-se. confiança.
§ 2º Entende-se em legítima defesa quem, IV. Cometer a transgressão disciplinar com
usando moderadamente dos meios necessários, repele emprego de fogo, tortura ou outro meio insidioso ou
injusta agressão, atual ou iminente, a direito seu ou de cruel, ou de que podia resultar perigo comum.
outrem. V. Sob o efeito de substância psicoativa.
§ 3º A coação irresistível pode ser física ou moral. VI. Mediante promoção, organização ou
A física se caracteriza quando o esforço físico/muscular cooperação no cometimento de falta disciplinar ou ainda
do autor é insuficiente para livrá-lo da ação do coator. A quando dirige, comanda ou por qualquer forma lidera a
coação moral se apresenta sob forma de ameaça feita atividade dos demais participantes.
pelo coator ao autor, que é compelido a praticar ação a VII. Quando coagir ou induzir outros adolescentes
delituosa, sob pena de suportar um prejuízo maior. à execução de falta. VIII. Quando instiga ou determina o
§ 4º Força maior é um acontecimento relacionado cometimento de falta à pessoa não punível em virtude
a fatos externos, independentes da vontade humana, da condição ou qualidade pessoal.
que impedem o cumprimento das obrigações. Esses IX. Quando executa a falta disciplinar, ou nela
fatos externos podem ser: ordem de autoridades (fato do participa, mediante pagamento ou promessa de
príncipe), fenômenos naturais (Raios, Terremotos, recompensa.
Inundações, etc.) E ocorrências políticas (Guerras,
X. Ter cometido a transgressão disciplinar com o
Revoluções, etc.)
envolvimento de duas ou mais pessoas.
Seção II - Das Circunstâncias Atenuantes
XI. Ter liderado conflitos, motins, tumultos e
Art. 71. São circunstâncias atenuantes à rebeliões dentro da unidade socioeducativa.
aplicação de qualquer medida disciplinar ao
XII. Ter cometido a transgressão contra
adolescente:
adolescente impossibilitado de se defender.
I. Primariedade em falta disciplinar.
XIII. Ter cometido a transgressão com
II. Bom comportamento na unidade. premeditação.
III. Assiduidade e bom comportamento nas Capítulo IV - Do Desvio de Percurso, da
atividades pedagógicas
Evasão e da Fuga
IV. Bom desempenho nas metas do plano
Art. 73. Após as 24 horas da evasão do
individualizado de atendimento (pia).
adolescente, a Unidade de medida socioeducativa
V. Ter o adolescente desistido de prosseguir na comunicará ao juizado da infância e da juventude.
execução da transgressão disciplinar.
§ 1º O prazo de 24 horas inicia-se a partir do
VI. No caso de o adolescente, por sua própria horário que a Unidade de medida socioeducativa
iniciativa e com eficiência, logo após cometer a estabeleceu para o retorno da saída ou atividade
transgressão disciplinar, evitar-lhe consequências mais externa. § 2º O adolescente que retornar da evasão em
graves ou minorar as consequências da falta. até 24 horas será recebido pela Unidade de medida
VII. Ter cometido a falta disciplinar sob coação a socioeducativa, sendo necessário o envio de relatório
que podia resistir, ou em cumprimento de ordem, ou sob circunstanciado ao poder judiciário.
a influência de violenta emoção provocada por ato § 3º Após o prazo de 24 horas, caso o
injusto de terceira pessoa. adolescente se apresente na Unidade de medida
VIII. Ter o adolescente confessado socioeducativa, deverá ser imediatamente encaminhado
espontaneamente, perante a autoridade administrativa, a ao juizado da infância e da juventude para que as
autoria da falta disciplinar. autoridades competentes procedam Na forma da lei.
IX. Ter cometido a transgressão disciplinar sob Art. 74. Toda evasão deverá ser comunicada à
influência de tumulto, se não o provocou. família do adolescente e ao juizado da infância e da
X. Ter o adolescente confessado juventude para as providências cabíveis.
espontaneamente, perante a equipe da unidade, a Art. 75. Toda evasão deverá ser comunicada à
autoria da infração disciplinar, até então ignorada ou coordenação de medidas socioeducativas, sendo
atribuída a outro. necessário o envio do relatório circunstanciado à
XI. Evitar ou minorar, logo após a transgressão, referida coordenação em até 24 horas.
suas consequências ou se propuser a reparar o dano. Art. 76. Em caso de evasão, os pertences e a
Parágrafo Único. A medida disciplinar poderá documentação do adolescente serão entregues a ele
ainda ser atenuada em razão de circunstância relevante, próprio, a seus familiares ou responsáveis, em caso de
anterior ou posterior à falta disciplinar, embora não determinação judicial, ou após a extinção da medida
expressamente regulamentada. socioeducativa, mediante assinatura de recibo.
Seção III - Das Circunstâncias Agravantes Título VII - Dos Direitos, Deveres e da Disciplina do
Orientador de Célula, da Equipe Administrativa, da
Art. 72. São circunstâncias agravantes para a
Equipe Técnica, da Equipe de apoio Técnico e da
aplicação de qualquer medida disciplinar ao
Equipe Operacional de Apoio.
adolescente:
Capítulo I – Dos Direitos
I. Reincidência em falta disciplinar.
MUITA ATENÇÃO NO ART. 77

www.editoradince.com - Acesse e veja se há novidades a respeito deste material – CUIDADO: cópia é crime.
90 LEGISLAÇÃO ESPECIAL E REGIMENTO INTERNO DAS UNIDADES DO SEAS
Art. 77. São garantidos aos funcionários públicos, XVII – Respeitar os horários de comparecimento
prestadores de serviço e colaboradores, além daqueles ao trabalho e intervalos estipulados para a refeição;
descritos no manual do socioeducador: XVIII – Prestar informações à direção sobre o
I – Ter conhecimento e consultar, quando comportamento e desempenho dos adolescentes nas
necessário, o regimento interno; atividades em que tiver participação ou sob sua
II – Ser orientado, quando necessário, a buscar condução;
atendimento especializado ao apresentar XIX – Demonstrar respeito às diversidades
comportamento que afete o desempenho de suas étnicas, culturais, de gênero, credo e orientação sexual
funções; dos adolescentes, colegas de trabalhos e outros;
III – Ser ouvido, perante qualquer situação de XX – Zelar pela segurança dos adolescentes,
conflito que envolva funcionários e/ou adolescentes; evitando situações que ponham em risco sua integridade
IV – Ter espaço adequado para atendimento, física, moral e psicológica;
guarda de pertences, higiene pessoal e realização de XXI – Participar de reuniões de rotina, encontros
refeições; de aperfeiçoamento e formação profissional,
V – Participar de reuniões de rotina, formação planejamento das ações, avaliação das atividades e
continuada, planejamento das ações, avaliação das integração da equipe de trabalho, sempre que
atividades e integração da equipe de trabalho. convocado;
Capítulo II – Dos Deveres XXII – Apresentar atestados médicos em casos
de faltas ao trabalho, no primeiro dia útil subsequente.
MUITA ATENÇÃO NO ART. 78
§ 1º- O funcionário fora de serviço não poderá ter
Art. 78. São deveres dos funcionários públicos e acesso à unidade, sem o consentimento do chefe
prestadores de serviços das unidades de internação imediato.
socioeducativa para adolescentes:
§ 2º - Não será permitida saída de funcionário
I - Todos os previstos no estatuto do servidor antes do término do serviço ou plantão, sem a devida
público, no caso dos funcionários públicos; autorização da direção da unidade socioeducativa,
II – Todos os previstos no manual dos devendo esta ser registrada no livro de ocorrências.
socioeducadores; Capítulo III – Da Disciplina
III – Cumprir a proposta de atendimento do Art. 79. A disciplina consiste na observância e
projeto político pedagógico das unidades de internação; obediência às determinações deste regimento no
IV – Cumprir e fazer cumprir este regimento; exercício funcional de cada profissional.
V – Obedecer às determinações previstas, Art. 80. Os procedimentos disciplinares devem
na Constituição Federal, no Estatuto da Criança e do contribuir para a segurança e a construção de um
Adolescente, no sistema nacional de atendimento ambiente tranquilo e produtivo, imbuindo um sentimento
socioeducativo, neste regimento e nas normas de justiça e de respeito pelos direitos fundamentais à
disciplinares e de rotina das unidades; dignidade de toda pessoa humana.
VI – Registrar a frequência ou outro instrumento Art. 81. Não serão aplicadas medidas
que comprove a jornada de trabalho; disciplinares que comprometam a integridade física,
VII – Agir com postura ética, como requer a psíquica e moral do profissional.
especificidade do trabalho, assim como nas questões Art. 82. O funcionário que não cumprir as
privativas do adolescente; determinações contidas neste regimento será submetido
VIII – Não trocar, nem vender objetos de qualquer ao procedimento administrativo próprio, sem prejuízo
natureza com adolescentes e familiares dos das penalidades legais cabíveis.
socioeducandos; Art. 83. Em caso específico de violação da
IX – respeitar os direitos da pessoa humana; integridade física de adolescentes em cumprimento de
X – Não infligir sofrimentos físicos ou psíquicos, medida socioeducativa de internação e semiliberdade de
como meio de intimidação, castigo pessoal, medida responsabilidade do estado do Ceará, em decorrência
preventiva ou qualquer outro fim; de quaisquer atos de violência física ou tortura nas
dependências das unidades de atendimento
XI – Usar trajes adequados, considerando a
socioeducativo, em que havendo indícios de autoria e
especificidade do trabalho;
materialidade do ilícito, o servidor público sujeita-se à
XII – Submeter-se a revista realizada pelas sindicância ou procedimento administrativo disciplinar,
equipes de controle de entrada e saída de pessoas e sem prejuízo da responsabilidade civil e criminal,
materiais da unidade; decorrente de irregularidades que possam configurar
XIII – Manter sigilo sobre procedimentos de prática de atos ilícitos quando no exercício do cargo,
segurança, sobre história de vida e situação judicial dos emprego ou função, conforme preceitua o estatuto dos
adolescentes; funcionários público civis do estado do Ceará, lei nº
XIV – Comunicar ao seu superior imediato e à 9.826/1974.
direção da unidade, qualquer irregularidade ou situação § 1º São penalidades para as infrações
que possam ameaçar a segurança da unidade; cometidas pelo servidor no caso específico deste
XV – Prestar esclarecimento, em sindicâncias ou CAPUT:
processos, sobre fatos de quer tiver ciência; A) Repreensão;
XVI – Comparecer e cumprir a jornada de B) Suspensão;
trabalho ordinário e, quando convocado, extraordinário, C) Multa;
executando as atividades que lhe competem,
D) Destituição de cargo comissionado;
assegurando-se os direitos previstos em lei;
E) Demissão;

www.editoradince.com - Acesse e veja se há novidades a respeito deste material – CUIDADO: cópia é crime.
LEGISLAÇÃO ESPECIAL E REGIMENTO INTERNO DAS UNIDADES DO SEAS 91
F) Cassação de aposentadoria, sendo-lhe VIII – Utilizar-se na apuração de transgressões
concedido o direito de ampla defesa; disciplinares contra adolescente de informações a que
§ 2º Caso o autor da violação da integridade teve acesso por meio do processo restaurativo;
física contra adolescentes em cumprimento de medida IX - Divulgar informações sobre a história de vida
socioeducativa de internação e semiliberdade seja dos adolescentes em cumprimento de medidas, bem
colaborador ou terceirizado em regime celetista, após como a situação processual;
apurado ou ato ilícito por meio de procedimento X - Divulgar procedimentos e rotinas das
administrativo e, sendo constatado qualquer prática de unidades;
violência, o procedimento será encaminhado
XI - Divulgar imagens e filmagens das
imediatamente a entidade ou empresa empregadora a
dependências internas das unidades e dos
fim de adotar as medidas cabíveis, assim como poderá
adolescentes, sem autorização expressa da direção da
ser enviado à autoridade especializada em infrações
unidade;
penais cometidas contra crianças e adolescentes.
XII - Reincidência, em falta de natureza média.
Capítulo IV – Das Sanções e Faltas
Parágrafo Único. Em caso de faltas que
Art. 84. Na aplicação das sanções disciplinares,
configurem crimes previstos no Código Penal e demais
levar-se-á em conta o comportamento apresentado, a
dispositivos pertinentes, a direção da unidade deverá
natureza e as circunstâncias do fato, bem como as suas
comunicar à autoridade especializada em infrações
consequências.
penais.
Parágrafo Único. Em caso de desrespeito às Título VIII- Da Segurança dos Adolescentes Internos
normas previstas nesse regimento, o funcionário deverá
e Profissionais das Unidades
ser encaminhado à comissão disciplinar da Secretaria do
Trabalho e Desenvolvimento Social (STDS), para as Art. 88. Deve ser garantida a segurança dos
providências cabíveis. adolescentes internos e dos profissionais das unidades,
sendo balizador fundamental da ação dos profissionais e
Art. 85. São consideradas faltas leves:
da polícia na garantia de tal segurança a preservação da
I - Faltar ao serviço, sem justificativa legal; integridade física e psicológica dos adolescentes
II – Chegar frequentemente atrasado; internos.
III – Ficar fora do setor em que está lotado, sem o Art. 89. A polícia deverá ser acionada em caráter
devido conhecimento da direção da unidade; excepcional e como última medida, estritamente nas
IV - Usar trajes inadequados no exercício de suas seguintes hipóteses:
atividades; I - Quando da ocorrência de tumulto generalizado
V - Não cumprir com as funções para as quais no interior da unidade que envolva a maioria dos
fora contratado; adolescentes e/ou alojamentos e os adolescentes
internos encontrem-se fora dos seus dormitórios, sem
VI - Uso constante do telefone no horário de
condições de contenção por parte dos socioeducadores
trabalho, interferindo no desenvolvimento das funções.
e da equipe técnica;
Art. 86. São faltas médias:
II - Quando da ameaça de invasão da unidade,
I – Omitir-se, na resolução dos problemas que ponha em risco a vida de adolescente interno ou
envolvendo adolescentes; profissional;
II - Agressões verbais aos colegas e III - Quando da ameaça à integridade física de
adolescentes; familiares de adolescentes ou terceiros que se
III - Tentar infamar a imagem do local de trabalho encontrem na unidade.
ou de outros setores da instituição; Art. 90. A polícia não deve ser acionada em
IV - Usar do cargo que ocupa para se favorecer, situações cotidianas de vistoria ou para qualquer medida
diante dos adolescentes e seus familiares; de segurança preventiva junto aos internos, devendo
V – Reincidência em falta leve, anteriormente estas serem realizadas pelos socioeducadores.
cometida. Art. 91. Quando acionada a polícia, a direção da
Art. 87. São faltas graves: unidade deve, imediatamente, comunicar à coordenação
I – Portar, fornecer ou facilitar a entrada de das medidas socioeducativas, que por sua vez
armas, serras, bebidas alcoólicas, tóxicos para comunicará ao juiz da vara de execução da infância e da
adolescentes ou para uso próprio; juventude, e ao ministério público.
II - Facilitar fugas e incentivos a motins; Art. 92. Quando for necessário o
encaminhamento de adolescente para iniciar novo
III– Sabotar ou dificultar o bom andamento do procedimento na delegacia da criança e do adolescente
serviço; em razão do cometimento de ato infracional, este deve
IV - Danificar material da instituição ou de ser acompanhado preferencialmente de algum membro
adolescentes, intencionalmente; da equipe técnica.
V – Utilizar-se de qualquer tipo de bebida Artigo 93. Cessado o tumulto generalizado no
alcoólica ou substância psicoativa, antes e durante o interior da unidade a partir da atuação da polícia, os
serviço; adolescentes envolvidos devem, imediatamente, ser
VI - Fazer transações com adolescentes ou seus encaminhados para exame de corpo de delito e oitiva
familiares, como empréstimos, trocas, compras, vendas, junto ao ministério público.
etc.; Título IX – Das Atribuições da Equipe da Unidade de
VII - Coagir o adolescente, com intenção de Medida Socioeducativa
abusos e/ou assédio sexual; Capítulo I - Das Atribuições do Orientador de Célula
e da Equipe Administrativa

www.editoradince.com - Acesse e veja se há novidades a respeito deste material – CUIDADO: cópia é crime.
92 LEGISLAÇÃO ESPECIAL E REGIMENTO INTERNO DAS UNIDADES DO SEAS
Art. 94. A direção da unidade de medida legislação vigente, relativamente a exigências sanitárias
socioeducativa é exercida por um profissional nomeado e padrões de qualidade nutricional.
pela Secretaria do Trabalho e Desenvolvimento Social – XVIII. Participar da comissão disciplinar, dando
STDS, denominado orientador de célula, observando-se encaminhamento às decisões tomadas coletivamente.
o perfil descrito no art. 17, inciso III, da lei 12.594/2012.
XIX. Definir horário e escalas de trabalho dos
Art. 95. A direção responde diretamente pela profissionais que atuam na unidade.
administração da unidade de medida socioeducativa e a
XX. Manter articulação com órgãos
ela estão subordinadas todas as demais instâncias da
governamentais e nãogovernamentais para
unidade.
estabelecimento de parcerias, acordos, fluxos e
Art. 96. A equipe administrativa da unidade de procedimentos, atendendo às orientações e diretrizes da
medida socioeducativa é composta por: STDS.
I. Agente administrativo; XXI. Solicitar desligamento e/ou substituição de
II. Auxiliar administrativo; profissional da unidade, observando as instruções
III. Coordenador de disciplina. emanadas pela célula de medidas socioeducativas da
STDS.
Parágrafo Único. Entende-se como auxiliar
administrativo o profissional que exerce a função de XXII. Assegurar o cumprimento das orientações
gerente da unidade. técnicas de vigilância sanitária e epidemiológica.
Seção I – Das Atribuições do Orientador de Célula XXIII. Viabilizar salas adequadas para a
educação formal, atendimentos técnicos e qualificação
Art. 97. Compete ao orientador da célula:
profissional.
I. Administrar e supervisionar os serviços técnicos
XXIV. Assegurar a realização do processo de
e administrativos executados na unidade.
avaliação institucional da unidade, quando solicitado.
II. Planejar, coordenar, controlar e avaliar a
XXV. Zelar pelo sigilo de informações pessoais
execução dos programas e atividades administrativas e
de adolescentes, das famílias e de todo o quadro de
técnicas realizadas na unidade.
pessoal da unidade.
III. Viabilizar o cumprimento das determinações
XXVI. Promover relacionamento cooperativo de
judiciais relativas aos adolescentes assistidos na
trabalho com os profissionais que atuam na unidade.
unidade.
XXVII. Encaminhar para a célula de medidas
IV. Coordenar o acompanhamento dos prazos
socioeducativas informações e relatórios sobre situações
legais relativos aos adolescentes, juntamente com o
de natureza grave ocorridas na unidade e sobre as
advogado.
atividades desenvolvidas.
V. Zelar pelo cumprimento das obrigações das
XXVIII. Cumprir e fazer cumprir o disposto neste
entidades que atendem adolescentes, previstas
regimento.
no estatuto da criança e do adolescente.
XXIX. Cumprir e fazer cumprir a legislação em
VI. Responsabilizar-se pelo patrimônio público,
vigor.
zelando pela manutenção e conservação das
instalações físicas e bens materiais da unidade. XXX. Realizar outras atividades inerentes à
função.
VII. Coordenar a elaboração e acompanhar a
implementação do projeto político-pedagógico Seção II – Das Atribuições da Equipe Administrativa
institucional da unidade. Art. 98. Compete ao agente administrativo:
VIII. Prestar contas dos materiais e equipamentos I. Digitar todos os documentos para o orientador
recebidos, zelando pelo bom uso destes. de célula e demais profissionais, relativos ao trabalho da
IX. Incentivar e facilitar a qualificação permanente unidade.
dos profissionais que atuam na unidade. II. Confeccionar o prontuário dos adolescentes.
X. Coordenar a revisão coletiva do regimento III. Organizar os arquivos de documentos da
interno, em consonância com a legislação em vigor. direção da unidade.
XI. Participar da análise e definição de projetos a IV. Organizar os endereços e telefones de
serem inseridos no projeto político-pedagógico da órgãos, entidades e todo tipo de equipamento social que
unidade. compõe a rede de atendimento ao adolescente.
XII. Garantir o fluxo de informações na unidade e V. Atender as ligações telefônicas para a direção
desta com os demais órgãos da administração estadual; ou administração da unidade, prestando informação,
XIII. Encaminhar à Secretaria do Trabalho e quando necessário.
Desenvolvimento Social – STDS, as propostas de VI. Conferir diariamente a presença dos
modificações no ambiente institucional, quando funcionários que registraram ponto, apontando possíveis
necessárias e/ou solicitadas. irregularidades.
XIV. Orientar e acompanhar o ingresso de novos VII. Manter atualizado o registro do patrimônio,
funcionários da unidade. composto pelos bens móveis da unidade.
XV. Acompanhar, juntamente com o pedagogo, VIII. Manter atualizada a relação de funcionários
as ações de escolarização formal na unidade. da unidade, contendo nome, cargo, endereço, telefone,
XVI. Assegurar o cumprimento dos dias letivos, celular e e-mail.
horas-aula e horasatividade estabelecidos pela IX. Manter a escala de trabalho dos funcionários
secretaria de educação do estado do Ceará. atualizada e fixada em local visível.
XVII. Supervisionar o preparo da alimentação, X. Efetuar registro de controle de frequência e
quanto ao cumprimento das normas estabelecidas na enviá-los ao a Célula de Gestão de Pessoas da STDS.

www.editoradince.com - Acesse e veja se há novidades a respeito deste material – CUIDADO: cópia é crime.
LEGISLAÇÃO ESPECIAL E REGIMENTO INTERNO DAS UNIDADES DO SEAS 93
XI. Efetuar o controle de possíveis horas-extras III. Prestar informações aos demais profissionais
realizadas e as devidas compensações. da equipe técnica sobre o adolescente, para compor os
XII. Efetuar o controle de atrasos e absenteísmos. relatórios e estudos de caso.
XII. Realizar os devidos registros, controles e IV. Acompanhar as demandas dos adolescentes
encaminhamentos de licenças médicas, acidentes de encaminhadas aos setores específicos.
trabalho, luto, casamento, nascimento de filho, etc. V. Tomar conhecimento dos relatos diários
XIV. Elaborar quadro de programação anual de contidos no livro de ocorrências.
férias, juntamente com o orientador de célula. VI. Comunicar de imediato à direção as
XV. Manter atualizado e dinamizado o quadro ocorrências relevantes que possam colocar em risco a
mural de informes, esclarecimentos e orientações aos segurança da unidade, dos adolescentes e dos
funcionários. funcionários.
XVI. Digitar documentos, declarações, VII. Coordenar o trabalho das equipes de
certificados e relatórios. instrutores educacionais.
XVII. Realizar outros serviços inerentes à função. VIII. Coordenar o desenvolvimento das atividades
pedagógicas, orientando os instrutores educacionais,
Art. 99. Compete ao auxiliar administrativo:
para que os adolescentes mantenham a ordem,
I. Realizar as compras emergenciais, utilizando a disciplina, respeito e cooperação durante as atividades.
verba de suprimento de fundo.
IX. Elaborar, com o diretor, as escalas de
II. Providenciar a solicitação mensal de materiais plantões e férias dos instrutores educacionais.
de consumo, tais como: gêneros alimentícios, materiais
X. Apurar as transgressões disciplinares com a
de expediente, pedagógicos e medicamentos.
comissão disciplinar.
III. Receber e conferir as mercadorias,
XI. Participar da elaboração do PIA, quando
organizando-as no almoxarifado.
solicitado.
IV. Administrar a liberação de mercadorias do
XII. Realizar outras atividades específicas à
almoxarifado.
função.
V. Manter registros e controles de consumo de
Capítulo II – Das Atribuições da Equipe Técnica
gêneros alimentícios, produtos de higiene, limpeza,
material de expediente, etc. Art. 101. A equipe técnica da unidade de medida
socioeducativa é composta por:
VI. Realizar levantamento das necessidades
mensais de suprimento de vestuário, roupa de cama e I. Assistente social;
banho, utensílio de copa e cozinha, materiais II. Psicólogo
pedagógicos, esportivos, recreativos, materiais para III. Pedagogo
oficinas e outros.
IV. Advogado
VII. Controlar o uso e funcionamento de materiais
V. Médico.
permanentes, providenciando a baixa por inservibilidade,
quando necessário. Art. 102. Compete ao assistente social:
VIII. Providenciar a prestação de contas dos I. Participar da recepção e acolhida dos
recursos utilizados para material de consumo. adolescentes, buscando formas de integrá-los à rotina
da unidade.
IX. Controlar os gastos com energia elétrica,
água/esgoto e telefonia; II. Elaborar estudos de caso e relatórios técnicos.
X. Providenciar para que sejam atendidas as III. Realizar atendimentos individuais e de grupo
necessidades referentes à coleta de lixo; com os adolescentes e familiares.
XI. Controlar a validade dos extintores de IV. Atender às famílias dos adolescentes,
incêndio, providenciando a reposição, sempre que favorecendo a sua corresponsabilidade no processo
necessário. socioeducativo.
XII. Providenciar a realização da manutenção das V. Providenciar a documentação civil dos
áreas externas da unidade, incluindo os serviços de adolescentes.
limpeza e jardinagem. VI. Realizar visitas domiciliares às famílias dos
XIII. Providenciar a manutenção e limpeza da adolescentes, caso necessário.
caixa de água, gerador e iluminação da unidade. VII. Acompanhar o adolescente em audiência,
XIV. Providenciar e controlar o uso de botijões de quando solicitado.
gás. VIII. Participar da elaboração do PIA com o
XV. Acompanhar o desenvolvimento de cardápio adolescente.
e a preparação da alimentação, obedecendo a I. Manter contato com órgãos governamentais e
procedimentos operacionais básicos. não governamentais para obter informações sobre o
XVI. Realizar outros serviços inerentes à função. adolescente.
Art. 100. Compete ao coordenador de disciplina: X. Manter registro de dados e informações para
levantamentos estatísticos.
I. Recepcionar o adolescente recém-chegado,
efetuando o seu registro, assim como de seus XI. Verificar a correspondência dos adolescentes
pertences. e acompanhar os contatos telefônicos realizados por
eles.
II. Providenciar o atendimento às necessidades
de higiene, asseio, repouso e alimentação do XII. Coordenar e orientar a visitação dos
adolescente. familiares aos adolescentes.

www.editoradince.com - Acesse e veja se há novidades a respeito deste material – CUIDADO: cópia é crime.
94 LEGISLAÇÃO ESPECIAL E REGIMENTO INTERNO DAS UNIDADES DO SEAS
XIII. Preparar os adolescentes para o III. Realizar a avaliação educacional e o
desligamento, fortalecendo suas relações levantamento do histórico escolar do adolescente para
sociofamiliares. compor os relatórios técnicos e o estudo de caso.
XIV. Supervisionar estagiários do setor de serviço IV. Efetuar o registro de documentação de
social. alunos: matrícula e todos os registros sobre processo
XV. Realizar visitas institucionais. escolar, utilizando os devidos formulários.
XVI. Avaliar e acompanhar a aplicação de V. Participar da recepção e acolhida dos
medidas disciplinares. adolescentes, buscando formas de integrá-los à rotina
da unidade.
XVII. Realizar outras atividades específicas à
profissão. VI. Avaliar e acompanhar a aplicação de medidas
disciplinares.
Seção II – Das Atribuições do Psicólogo
VII. Participar da elaboração do PIA
Art. 103. Compete ao psicólogo:
VIII. Identificar o adolescente com transtornos de
I. Participar da recepção e acolhida dos
aprendizagem e necessidades especiais para traçar um
adolescentes, buscando formas de integrá-los à rotina
plano de intervenção individualizado.
da unidade.
IX. Orientar as famílias do adolescente para
II. Planejar, coordenar e executar as atividades
garantir a continuidade das atividades escolares após o
da área de psicologia.
desligamento.
III. Elaborar estudos de caso e relatórios técnicos.
X. Acompanhar e coordenar a execução das
IV. Realizar diagnóstico e avaliações atividades de qualificação profissional.
psicológicas, procedendo às indicações terapêuticas
XI. Coordenar a equipe de professores,
adequadas a cada caso.
instrutores de ofício e supervisionar estagiários do setor
V. Realizar atendimento psicológico individual e pedagógico.
de grupo com os adolescentes e seus familiares.
XII. Analisar e verificar os avanços dos
VI. Acompanhar o adolescente em audiência, adolescentes na escolarização formal e informal.
quando solicitado.
XIII. Acompanhar o planejamento e execução dos
VII. Manter contato com órgãos governamentais e planos de aula de professores e instrutores de oficina.
não governamentais para obter informações sobre o
XIV. Acompanhar as ações de voluntariado e
adolescente.
espiritualidade.
VIII. Realizar intervenções terapêuticas com os
XV. Elaborar prestação de conta mensal dos
adolescentes, visando facilitar a dinâmica relacional com
recursos obtidos com as vendas dos materiais
ele e com o outro.
produzidos pelos adolescentes, em oficinas
IX. Participar da elaboração do PIA. profissionalizantes.
X. Atender às famílias, orientando-as e realizando XVI. Visitar escolas em que os adolescentes se
intervenções que lhes forneçam subsídios para o encontram matriculados.
desempenho qualitativo das suas funções parentais.
XVIII. Analisar os documentos formais da
XI. Realizar visita domiciliar à família do escolarização, planos de aula e plano de trabalho
adolescente, quando necessário. docente.
XII. Planejar e desenvolver projetos com vistas a XVIII. Conduzir processo de classificação e
orientar os profissionais da unidade no trato com os reclassificação dos adolescentes, para adequação da
adolescentes e famílias. matrícula escolar e defasagem idadesérie e também
XIII. Buscar e articular recursos da rede sus, para mediante os exames nacionais.
o acompanhamento da saúde mental dos adolescentes. XIX. Em caso de transferência, repassar
XIV. Preparar os adolescentes para o documentos e informações escolares, materiais
desligamento, fortalecendo suas relações escolares e produções do adolescente transferido à
sociofamiliares. unidade receptora.
XV. Manter registro de dados e informações para XX. Providenciar matrícula e contato com a
levantamento estatístico. escola que irá receber o adolescente.
XVI. Supervisionar estagiários do setor de XXI. Organizar os procedimentos de substituição
psicologia. e recepção de professores.
XVII. Realizar outras atividades específicas à XXII. Organizar o plano e o calendário escolar,
profissão. tendo como base as orientações da SEDUC.
Seção III – Das Atribuições do Pedagogo XXIII. Organizar e divulgar os materiais
Art. 104. Compete ao pedagogo: pedagógicos para uso dos professores e/ou instrutores
I. Planejar, coordenar e desenvolver as ações de ofício.
pedagógicas da unidade, incluindo as atividades XXIV. Providenciar matrículas, transferências,
escolares, oficinas formativas, ocupacionais e obtenção de históricos escolares, aproveitamento de
profissionalizantes, atividades recreativas, culturais e estudos e certificação dos adolescentes.
esportivas. XXV. Providenciar a avaliação diagnóstica do
II. Realizar a programação das atividades nível escolar do adolescente.
pedagógicas, a formação das turmas e o XXVI. Elaborar o plano de ação pedagógica com
acompanhamento das atividades. os professores e instrutores e acompanhar a execução
das atividades.

www.editoradince.com - Acesse e veja se há novidades a respeito deste material – CUIDADO: cópia é crime.
LEGISLAÇÃO ESPECIAL E REGIMENTO INTERNO DAS UNIDADES DO SEAS 95
XXVII. Promover estudos e avaliações sobre as VII. Orientar a família do adolescente quanto a
experiências pedagógicas e o processo de ensino e atitudes, procedimentos e posturas para a promoção da
aprendizagem. saúde do adolescente e dos membros da família.
XXVIII. Realizar outras atividades específicas à VIII. Realizar ações educativas de promoção à
profissão. saúde e prevenção de doenças para adolescentes.
Seção IV – Das Atribuições do Advogado IX. Elaborar planos de intervenção em saúde
Art. 105. Compete ao advogado: para o desenvolvimento da ação socioeducativa
personalizada para o adolescente.
I. Participar da recepção e acolhida dos
adolescentes, buscando formas de integrá-los à rotina X. Orientar a equipe quanto a procedimentos e
da unidade. ações terapêuticas, preventivas e promotoras da saúde.
II. Participar da elaboração do PIA. XI. Supervisionar estagiários de medicina.
III. Esclarecer a situação processual do XII. Realizar outras atividades específicas à
adolescente, familiares, direção e equipe técnica. profissão.
IV. Manter-se atualizado sobre o processo. Capítulo III – Das Atribuições da Equipe de Apoio
Técnico
V. Realizar pesquisas e levantamentos referentes
aos autos judiciais e ao histórico infracional dos Art. 107. A equipe de apoio técnica da unidade de
adolescentes. medida socioeducativa é composta por:
VI. Participar de audiências; I. Auxiliar de enfermagem
VII. Orientar o adolescente e sua família quanto à II. Instrutor educacional.
postura na audiência, bem como informá-los sobre o Seção I – Das Atribuições do Auxiliar de
agendamento destas. Enfermagem
VIII. Avaliar e acompanhar a aplicação de Art. 108. Compete ao auxiliar de enfermagem:
medidas disciplinares. I. Desempenhar serviços auxiliares de
IX. Pleitear pela progressão de medida, quando o enfermagem, prestando apoio às ações do médico.
estudo de caso assim orientar. II. Programar e organizar as consultas dos
X. Promover palestras informativas aos adolescentes com o médico da unidade.
adolescentes, familiares e funcionários, quando III. Agendar e acompanhar os adolescentes nas
necessário. consultas médicas na unidade.
XI. Organizar documentos para a transferência do IV. Agendar e acompanhar os adolescentes nas
adolescente a outras unidades. consultas e exames externos.
XII. Participar dos espaços coletivos de V. Manter atualizadas e organizadas as fichas de
discussão. atendimento de saúde dos adolescentes.
XIII. Manter contato com outras comarcas para VI. Ministrar medicamentos e tratamento aos
obter maiores informações sobre o processo jurídico dos adolescentes, atendendo às orientações médicas.
adolescentes.
VII. Realizar atendimentos de primeiros socorros,
XIV. Atualizar periodicamente a lista de dados quando necessário.
sociojurídicos dos adolescentes.
VIII. Manter a organização da enfermaria e dos
XV. Elaborar ofícios em resposta à trajetória materiais utilizados.
jurídicoprocessual do adolescente;
IX. Realizar ações educativas sobre cuidados de
XVI. Informar aos técnicos, com antecedência, a higiene pessoal, alimentação e cuidados específicos
necessidade de elaboração do relatório de avaliação para promoção da saúde, esclarecendo sobre os
biopsicossocial e educacional; recursos disponíveis no município, para
XVII. Organizar a relação diária de saídas encaminhamento dos adolescentes, quando necessário.
externas (audiências) de adolescentes, juntamente com X. Manter organizado, os estoques de medicação
os demais técnicos da unidade. e de outros insumos utilizados nos tratamentos de
XVII. Supervisionar estagiários do setor de direito. saúde.
XIX. Realizar outras atividades específicas à XI. Orientar a equipe sobre as condutas prévias
profissão. ou posteriores a consultas e exames.
Seção V – Das Atribuições do Médico XII. Realizar outras atividades específicas à
Art. 106. Compete ao médico: profissão.
I. Planejar, executar e avaliar as ações MUITA ATENÇÃO NO ART. 109
relacionadas à saúde integral do adolescente. Seção II – Das Atribuições do Socioeducador
II. Realizar a avaliação clínica das condições de Art. 109. Compete ao socioeducador:
saúde do adolescente. I. Recepcionar o adolescente recém-chegado,
III. Emitir diagnósticos e indicar os procedimentos efetuando o seu registro, assim como de seus
terapêuticos adequados ao caso. pertences.
IV. Tratar as intercorrências de nível ambulatorial. II. Providenciar o atendimento às necessidades
V. Articular e formalizar o fluxo de atendimento à de higiene, asseio, repouso e alimentação do
saúde integral do adolescente na rede de serviços adolescente;
ofertados pelo município. III. Zelar pela segurança e bem-estar do
VI. Encaminhar o adolescente para exames e adolescente, observando-o e acompanhando-o em todos
tratamentos especializados ofertados pela rede de os locais de atividades diurnas e noturnas.
saúde do SUS.

www.editoradince.com - Acesse e veja se há novidades a respeito deste material – CUIDADO: cópia é crime.
96 LEGISLAÇÃO ESPECIAL E REGIMENTO INTERNO DAS UNIDADES DO SEAS
IV. Acompanhá-lo nas atividades da rotina diária, XXVI. Realizar outras atividades específicas à
orientando-o quanto às normas de conduta, cuidados profissão.
pessoais e relacionamento com outros internos e Capítulo IV – Atribuições da Equipe Operacional de
funcionários. Apoio
V. Relatar no diário de comunicação interna o Art. 110. A equipe operacional de apoio é
desenvolvimento da rotina diária, bem como tomar composta por:
conhecimento dos relatos anteriores.
I. Motorista;
VI. Monitorar e auxiliar atividades recreativas,
II. Auxiliar de manutenção;
esportivas, culturais, artesanais e artísticas, seguindo as
orientações do setor pedagógico. III. Porteiro/vigia;
VII. Auxiliar no desenvolvimento das atividades IV. Auxiliares de serviços gerais de limpeza, copa
pedagógicas e acompanhar os adolescentes nos e lavanderia.
atendimentos técnicos. Seção I – Das Atribuições do Motorista
VIII. Guardar e organizar os pertences dos Art. 111. Compete ao motorista:
adolescentes. I. Transportar adolescentes em viagens,
IX. Participar das atividades interagindo com os audiências, consultas médicas, transferências de
adolescentes. unidade e outros que se fizerem necessários.
X. Prestar informações aos demais profissionais II. Definir rotas e percursos de modo a garantir a
da equipe técnica sobre o andamento do adolescente, economia de combustível e otimização do uso do
para compor os relatórios e estudos de caso. veículo.
XI. Acompanhar o adolescente em seu III. Conduzir os técnicos da unidade a diversos
deslocamento na comunidade, não descuidando da locais para atender às necessidades técnicas e
vigilância e segurança. administrativas.
XII. Inspecionar as instalações físicas da unidade, IV. Respeitar a legislação, normas e
recolhendo objetos que possam comprometer a recomendações de direção defensiva.
segurança. V. Preencher diariamente o mapa de atividades
XIII. Efetuar rondas periódicas para verificar diárias e as requisições de abastecimento do veículo.
portas, janelas e portões, assegurando-se de que estão VI. Verificar diariamente as condições de uso do
devidamente fechados, atentando para eventuais veículo.
anormalidades. VII. Solicitar à administração reparos nos
XIV. Identificar as demandas dos adolescentes, veículos, sempre que necessário.
encaminhando-as aos setores específicos. VIII. Manter o veículo limpo e em condições
XV. Manter-se atento às condições de saúde do adequadas de higiene e funcionamento.
adolescente, sugerindo que sejam providenciados IX. Auxiliar no carregamento e descarregamento
atendimentos e encaminhamentos aos serviços de materiais transportados no veículo.
médicos, sempre que necessário.
X. Efetuar a prestação de contas das despesas
XVI. Atender às determinações e orientações de manutenção do veículo.
médicas, ministrando os medicamentos prescritos,
quando necessário. XI. Manter sigilo acerca das situações
vivenciadas na unidade.
XVII. Realizar revistas pessoais nos adolescentes
nos momentos de recepção, final das atividades e XII. Realizar outras atividades específicas à
sempre que se fizer necessário, impedindo que profissão.
mantenham a posse de objetos e substâncias não Seção II – Das Atribuições do Auxiliar de
autorizadas. Manutenção
XVIII. Acompanhar o processo de entrada das Art. 112. Compete ao auxiliar de manutenção:
visitas dos adolescentes, registrando-as em livro, I. Efetuar a conservação das edificações,
fazendo revistas e verificação de alimentos, bebidas ou executando serviços de alvenaria, carpintaria, pintura,
outros itens trazidos por elas. eletricidade e encanamento.
XVIX. Comunicar, de imediato, ao coordenador II. Realizar pequenos reparos em máquinas,
de disciplina as ocorrências relevantes que possam equipamentos e móveis.
colocar em risco a segurança da unidade, dos III. Inspecionar as instalações elétricas e
adolescentes e dos funcionários. hidráulicas da unidade.
XX. Fornecer material de higiene para os IV. Zelar pela manutenção das tubulações,
adolescentes, controlando e orientando seu uso. válvulas, registros, filtros, instrumentos e acessórios,
XXI. Providenciar o fornecimento de vestuários, limpando, lubrificando e substituindo as partes
roupa de cama e banho, orientando seu uso. danificadas.
XXII. Recepcionar e identificar os visitantes, V. Operar os dispositivos dos reservatórios de
encaminhando-os aos diferentes setores. água.
XXIII. Guardar e devolver os pertences dos VI. Zelar pela conservação e guarda de
visitantes dos adolescentes. ferramentas e equipamentos utilizados na unidade;
XXIV. Seguir procedimento e normas de VII. Observar, cumprir e utilizar normas e
segurança da unidade. procedimentos de segurança.
XXV. Participar da elaboração do PIA, quando VIII. Realizar outras atividades específicas à
solicitado. profissão.
Seção III – Das Atribuições do Porteiro/Vigia

www.editoradince.com - Acesse e veja se há novidades a respeito deste material – CUIDADO: cópia é crime.
LEGISLAÇÃO ESPECIAL E REGIMENTO INTERNO DAS UNIDADES DO SEAS 97
Art. 113. Compete ao porteiro/vigia: Subseção III – Das Atribuições do Auxiliar de
I. Certificar-se da observância das Serviços de Lavanderia
recomendações quanto à prevenção de incêndios, Art. 116. Compete ao auxiliar de serviços de
mantendo-se preparado para adotar procedimento de lavanderia:
combate ao fogo, caso necessário. I. Lavar e higienizar as roupas pessoais e de
II. Efetuar rondas periódicas de inspeção da parte cama e banho dos adolescentes.
externa da unidade, examinando portas, janelas e II. Passar as roupas dos adolescentes,
portões, para eventuais anormalidades. organizando-as e separando-as para ser distribuídas.
III. Fiscalizar a entrada e saída de pessoas na III. Realizar pequenos reparos de costura nas
unidade ou setor de pessoas, veículos, bens e materiais roupas dos adolescentes.
não autorizados pela direção.
IV. Realizar outras atividades específicas à
IV. Observar a movimentação de pessoas nas profissão.
imediações do seu posto de trabalho, comunicando à
Título IX – Disposições Transitórias
direção qualquer irregularidade ou atitude suspeita
observada. Art. 117. Este regimento deverá ser executado de
acordo com os seguintes procedimentos:
V. Atender e prestar informações ao público.
I. A Secretaria do Trabalho e Desenvolvimento
VI. Responsabilizar-se pelo controle de abrir e
Social – STDS, deverá realizar formação profissional
fechar o portão.
com as equipes das unidades de medidas
VII. Manter o registro de todas as ocorrências socioeducativas sobre este regimento.
verificadas durante seu turno de trabalho.
II. A direção da unidade de medidas
VIII. Registrar e controlar a entrada e saída de socioeducativas deverá realizar uma assembleia com os
público externo na unidade. adolescentes, para apresentação deste regimento e de
IX. Realizar outras atividades específicas à quaisquer alterações, sempre que houver.
profissão. III. Uma cópia deste regimento deverá
Seção IV – Das Atribuições dos Auxiliares de permanecer em local de fácil acesso e visibilidade tanto
Serviços de Limpeza, Copa e Lavanderia para os adolescentes e familiares quanto para a equipe
Subseção I – Das Atribuições do Auxiliar de Serviços da unidade de medidas socioeducativas.
de Copa IV. A capitulação das sanções leves, médias,
Art. 114. Compete ao auxiliar de serviços de graves e as sanções aplicáveis deverão ser afixadas em
copa: local de fácil acesso, na área administrativa e nos
demais setores da unidade.
I. Preparar o café da manhã, almoço, lanches,
jantar e ceia para adolescentes e funcionários da V. A equipe da unidade de medida socioeducativa
unidade. deverá apresentar este regimento a todos os
adolescentes, no ato da admissão.
II. Servir refeições, organizando o refeitório ou
preparando os pratos para os adolescentes. Art. 118. A Secretaria do Trabalho e
Desenvolvimento Social, por meio da Coordenadoria de
III. Limpar todos os utensílios, louças e
Proteção Especial deverá realizar cursos de formação
equipamentos utilizados para as refeições.
para facilitadores restaurativos no prazo de um ano e ou
IV. Organizar e manter limpos e em ordem os dois anos para adaptações estruturais, a contar da data
armários, geladeira, freezer e almoxarifado da cozinha. da publicação deste regimento.
V. Manter o controle dos gastos com os gêneros Art. 119. Este regimento passa a vigorar em 45
alimentícios, levantando as necessidades de reposição dias contados da data de sua publicação.
para informar ao auxiliar administrativo.
Art. 120. Ficam revogadas todas as disposições
VI. Realizar outras atividades específicas à em contrário.
profissão.
Subseção II – Das Atribuições do Auxiliar de
Serviços de Limpeza QUESTÕES DE CONCURSOS
Art. 115. Compete ao auxiliar de serviços de 01. (UECE-CEV - 2017 - SEAS - CE - Socioeducador)
limpeza: Assinale a opção que NÃO corresponde a uma das
I. Limpar diariamente o prédio da unidade, competências do Coordenador de Disciplina da
incluindo as áreas internas e externas. Unidade Socioeducativa.
II. Realizar faxinas gerais. A Participar da elaboração do PIA (Plano Individual de
Atendimento), quando solicitado.
III. Manter em ordem e higienizado o
almoxarifado de produtos de limpeza, realizando B Comunicar de imediato à direção as ocorrências
levantamento de necessidades para o auxiliar relevantes que possam colocar em risco a segurança
administrativo da unidade. da Unidade, dos adolescentes e dos funcionários.
IV. Manter e limpar as áreas externas da unidade, C Atender às famílias dos adolescentes, favorecendo a
incluindo pátios, canteiros e jardins. sua corresponsabilidade no processo socioeducativo.
V. Efetuar o recolhimento do lixo, providenciando DApurar as transgressões disciplinares com a comissão
para que ele seja colocado no local adequado de coleta. disciplinar.
VI. Efetuar o transporte e descarga de materiais
diversos. 02. (UECE-CEV - 2017 - SEAS - CE - Socioeducador)
VII. Realizar outras atividades específicas à No que diz respeito às disposições pertinentes aos
profissão. direitos, deveres e disciplina do servidor público que
trabalha nas Unidades Socioeducativas, relacione

www.editoradince.com - Acesse e veja se há novidades a respeito deste material – CUIDADO: cópia é crime.
98 LEGISLAÇÃO ESPECIAL E REGIMENTO INTERNO DAS UNIDADES DO SEAS
corretamente as colunas a seguir, numerando a ______________________________________________
Coluna II de acordo com a Coluna I. ______________________________________________
Coluna I ______________________________________________
1. Direito ______________________________________________
2. Dever ______________________________________________
3. Penalidade ______________________________________________
4. Falta leve ______________________________________________
5. Falta média ______________________________________________
6. Falta grave ______________________________________________
Coluna II ______________________________________________
( ) Cumprir a proposta de atendimento do projeto ______________________________________________
político-pedagógico das unidades de internação.
______________________________________________
( ) Portar, fornecer ou facilitar a entrada de armas,
______________________________________________
serras, bebidas alcoólicas, tóxicos para adolescentes
ou para uso próprio. ______________________________________________
( ) Ser orientado, quando necessário, a buscar ______________________________________________
atendimento especializado ao apresentar ______________________________________________
comportamento que afete o desempenho de suas ______________________________________________
funções. ______________________________________________
( ) Ser repreendido conforme preceitua o Estatuto dos ______________________________________________
Funcionários Públicos Civis do Estado do Ceará.
______________________________________________
A sequência correta, de cima para baixo, é
______________________________________________
A 2, 5, 3, 1.
______________________________________________
B 2, 6, 1, 3.
______________________________________________
C 1, 5, 2, 6.
______________________________________________
D 3, 1, 2, 4.
______________________________________________
______________________________________________
03. (UECE-CEV - 2017 - SEAS - CE - Socioeducador)
Dentre as medidas disciplinares apresentadas a ______________________________________________
seguir, assinale a que é aplicada ao adolescente que ______________________________________________
comete transgressão média. ______________________________________________
A Advertência verbal. ______________________________________________
B Restrição do adolescente ao dormitório por no máximo ______________________________________________
cinco dias. ______________________________________________
C Retratação verbal à pessoa ofendida. ______________________________________________
D Suspensão da prática esportiva pelo prazo de até ______________________________________________
cinco dias.
______________________________________________
______________________________________________
04. (UECE-CEV - 2017 - SEAS - CE - Socioeducador)
______________________________________________
Para regular a convivência nas Unidades de Medidas
Socioeducativas, definem-se como critérios ______________________________________________
normativos a Rotina Institucional e o Regulamento ______________________________________________
Disciplinar. No que concerne à Rotina Institucional, ______________________________________________
assinale a opção que NÃO é contemplada por esse
______________________________________________
critério normativo.
______________________________________________
A Ter horários de dormir e de acordar.
______________________________________________
B Ter horários pré-estabelecidos para as refeições.
______________________________________________
C Ter os dias e horários de visitas escolhidos pelos
próprios familiares visitantes. ______________________________________________
D Ter a quantidade, os horários e a duração do banho ______________________________________________
pré-estabelecidos. ______________________________________________
______________________________________________
Gabarito: 01/C; 02/B; 03/C; 04/C ______________________________________________
______________________________________________
______________________________________________
ANOTAÇÕES: ______________________________________________
______________________________________________ ______________________________________________
______________________________________________ ______________________________________________
______________________________________________ ______________________________________________
______________________________________________ ______________________________________________
______________________________________________

www.editoradince.com - Acesse e veja se há novidades a respeito deste material – CUIDADO: cópia é crime.

Você também pode gostar